You are on page 1of 444

Law 321_Corporation LAW_ Case Digest

In Partial Fulfilment
Of the Requirements for the Subject
Law 321 (Corporation Law)

Corporation Law
Case Digest

Submitted to:

Atty. Maria Lulu G. Reyes

Submitted by:

ARUMIN, Lesley Jane B.


BAGUIDUDOL, Valentin Jr. G.
BAGUILAT, Lauriz G.
BUENO, Marc Crisante C.
CAMSOL, Haryeth M.
LUBANTE, Jessica B.
ORALLO, Joanna Marie C.
ORAS, Phylian Corazon W.
SANTOS, Hyacinth B.
SECTEL, Florence O.
TUGUIC, Joshua B.

Date Submitted:

March 8, 2014

CORPORATION CODE OF THE PHILIPPINES


(Batas Pambansa Blg. 68)

1|Page
Law 321_Corporation LAW_ Case Digest

I. FORMATION AND ORGANIZATION OF CORPORATIONS

A. General Principles

1. History of Business Organizations


2. Constitutional Basis, Art. XII, Sec. 16
3. Definitions of Corporation (Sec. 2)
4. Attributes of Corporation
 Petron v. NCBA, 516 S 168 ___________________________________20
 APT v. CA, 300 S 582 ________________________________________21
 Mambulao Lumber v. PNB, 22 S 359
 Hanil v. CA, 362 S 1
 Bache and Co. v. Ruiz , 637 S 823
 Sulo ng Bayan v. Araneta, 72 S 347

B. Classification of Corporations

1. Private v. Public Corporation


 Boy Scout of the Phil. v. COA, June 7, 2011 ____________________26
 Liban v. Gordon, July 15, 2009
 Baluyot v. Holganza, 325 S 526
 Vet. Fel. Of the Phil. v. Reyes, 483 S 526
 MIA v. CA, 495 S 591

2. Stock (Sec. 3) v. non-Stock (Sec. 3 and 87)


3. Open v. Close Corporation (Sec. 96 et. seq.)
4. Domestic v. foreign Corporation (Sec. 123 et. seq.)
5. Special Charter Corporation
6. Educational (Sec. 106 et. seq.)
7. Religious sole and aggregate (Sec. 109 et. seq.)

C. Stages in the Formation/Organization of a Corporation

1. Promotion
 March II Marketing v. Joson, December 12, 2011 _______________32
 Cagayan Fishing v. Sandiko, 65 P 223
 Caram v. CA, 151 S 372
 Pioneer Insurance v. CA, 175 S 668
 Rizal Light v. Municipality of Morong, 25 S 258

2. Incorporation
3. Organization

D. Articles of Incorporation (Sec. 14 and 15): Contents


 Lanuza v. CA, 454 S 54 ______________________________________37

1. Corporate Name (Sec. 18)


 Alonso v. Cebu, 417 S 115 ___________________________________38
 Industrial Refractories v. CA, 390 S 252
 Ang mga Kaanib sa Iglesia ng Diyos v. Iglesia, December 12, 2001
 Universal Mills v. Universal Textile Mills, 78 S 62
 Lyceum of the Phil. v. CA, 219 S 610
 Indiana Aerospace University v. CHED, April 4, 2001
 Philips Export BV v. CA, 206 S 457

2. Primary Purpose (Sec. 14)


 Gala v. Ellice, 418 S 431 _____________________________________45
 Heirs of Pael v. CA, December 7, 2001
 Uy Siulong v. Director, 40 P 541

2|Page
Law 321_Corporation LAW_ Case Digest

 Asuncion v. De Yriarte, 28 P 67

3. Secondary Purpose/s (Sec. 14)

4. Principal Office/Domicile (Sec. 14)


 Davao Light and Power Co. v. CA, August 20, 2001 _____________49
 Clavecilla Radio Sytem v. Antillon, 19 S 379
 Sy v. Tyson Enterprise, 119 S 367
 Young Auto Supply v. CA, 223 S 670

5. Term (Sec. 11 in rel. to Sec. 37, 81 and 120)


 Alhambra Cigar and Cigarette Mfg. v. SEC, 24 S 269 _____________53

6. Incorporators (Secs. 10 and 5)

7. Incorporating Directors (Sec. 14)

8. Capital Stock
a) Authorized (Sec. 12)
b) Subscribed (Sec. 13)
c) Paid-up (Sec 13)

 MISCI-NACUSIP Local Chapter v. NWPC, 269 S 173 ______________54

d) Outstanding (Sec. 143)


e) Minimum Requirements for Incorporation

9. Classification of Shares (Sec. 6)


a) Common v. Preferred
b) Par value v. No par value shares
c) Voting v. Non-voting
d) Founder’s Shares (Sec. 7)
e) Redeemable Preferred (Sec. 8)
f) Treasury (Sec. 9)

10. Subscribers (Sec. 14)


11. Treasurer-in-trust (Sec. 15)
12. Special Provisions
a) “No Transfer” Clause

13. Amendment and/or rejection of Articles of Incorporation (Secs. 16


and 17)
 Republic Planters Bank v. CA, 216 S 738 _______________________55

E. Commencement of Corporate Existence (Sec. 19)/Theory of Concession

F. Doctrine of Corporate Entity

G. Doctrine of Piercing the Veil of Corporate Fiction: Instances

1. Public Convenience Cases;


2. Fraud Cases;
3. Alter Ego/Instrumentality Cases.
 PNB v. Hydro Resources, March 13, 2013 ______________________57
 Ramirez v. Mar Fishing, Inc., June 13, 2012
 Sarona v. NLRC, January 18, 2012
 Gold Line Tours v. heirs of Lacsa, June 18, 2012
 Hacianda Luisita v. Presidential Agrarian Council, January 22, 2011
 Pantranco Employees Assoc., et al. v. NLRC, March 17, 2009
 Cagayan Valley Drug Corp v. CIR, 545 S 10
 Heirs of Pajarillo v. CA, 537 S 96
 Petron v. NLRC, 505 S 596

3|Page
Law 321_Corporation LAW_ Case Digest

 China Banking v. Dyne-Sem, 494 S 493


 Marubeni v. Lirag, August 10, 2001
 Francisco v. Mejia, August 14, 2001
 PNB v. Andrada Electric, 382 S 244
 AZCOR Mfg. v. NLRC, 303 S 26
 Claparols v. CIR, 65 S 613
 CIR v. Norton and Harrison, August 31, 1964
 Concept Builders v. NLRC, 257 S 149
 Complex Electronics Employees Assoc. v. NLRC, 310 S 403
 Cordon v. Balicanta, October 4, 2002
 Delpher Trades v. IAC, January 2, 1988
 Del Rosario v. NLRC, July 24, 1990
 First International Bank v. CA, 252 S 259
 Francisco Motors v. CA, 309 S 73
 Laguio v. NLRC, 262 S 709
 Lim v. CA, 323 S 102
 Matuguina Integrated Wood Products v. CA, 263 S 490
 Manila Hotel Corp. v. NLRC, October 13, 2000
 Norton and Harrison v. Collector, 11 S 74
 San Juan Structural v. CA, 296 S 634
 Tan Boon Bee v. Jarencio, 163 S 205
 Telephone Eng’g and Service Co. v. WCC, 104 S 354
 Umali v. CA, September 13, 1990
 Vlason Enterprises v. CA, 310 S 26
 Villa Rey transit v. Ferrer, October 29, 1968

H. De Facto Corporation (Sec. 20)


 Hall v. Piccio, 86 P 603 ______________________________________90

I. Corporation by Estoppel (Sec. 21)


 International Express v. CA, 343 S 74 __________________________91
 Lim Tiong v. PFGI, Inc., 317 S 728
 Albert v. University Publishing, 13 S 84

J. Non-user of Charters v. Continuous Inoperation (Sec. 22)


 Loyola Grand Villas v. CA, 276 S 681 __________________________94

II. BOARD OF DIRECTORS (SEC. 22 ET. SEQ.)

A. Nature of Office

B. Requirements

1. Qualifications/Qualifying shares (Sec. 24)


 Villafuerte v. Moreno, October 2, 2009 ________________________95
 Baguio v. CA, 26S 366
 Detective and Protective Bureau v. Cloribel, 26 S 255
 Grace Christian HS. CA, 281 S 133
 Lee v. CA, 205 S 752

2. Disqualifications (Sec. 27)


 Brias v. Hord, 24 P 286 _____________________________________100

3. Residence
4. Nationality
C. Election (Sec. 24)
1. Quorum
2. Voting
 Aurbach v. Sanitary Wares, 180 S 131 ________________________101
 Bataan Shipyard v. PCGG, 150 S 181

4|Page
Law 321_Corporation LAW_ Case Digest

D. Report on Election (Sec. 26)


 Premium Marble v. CA, 264 S 11 _____________________________103

E. Term of Office/Holdover
 Seneres v. COMELEC and Robles, April 16, 2009 _______________104

F. How removed (Sec. 28)


 Lambert v. Fox, 26 P 588 ___________________________________105

G. How Vacancy filled (Sec. 29)


 Valle Verde Country Club v. Africa, September 4, 209 __________106

H. How Compensated (Sec. 30)


 Singson, et al. v. COA, August 9, 2010 ________________________107
 Western institute v. Salas, 278 S 216
 Central Coop Exchange v. Tibe, 33 S 593
 Lingayen Gulf v. Baltazar, 93 P 404

I. Authority of the Board of Directors (Sec. 24)


 La Buga’al v. Ramos, 421 S 148 _________________________111
 Shipside v. CA, 352 S 334
 ABS-CBN v. CA, 301 S 573
 Asset Privatization Trust v. CA, 300 S 582
 BA Savings Bank v. Sia, 336 S 484
 Montelibano v. Bacolod Murcia, 5 S 36
 Powers v. Marshall, May 9, 1988
 Premium Marble v. CA, 264 S 11
 Ramirez v. Orientalist, 38 P 634

J. Delegation of Authority to Corporate Officers

1. Corporate Officers/meaning of “Office” vis-à-vis Employment


 Real v. Sangu Phil., January 19, 2011 _________________________120
 Matling v. Coros, October 13, 2010
 Manila Metal v. PNB, 511 S 444
 Ongkiko v. NLRC, 270 S 613
 Lao v. CA, 325 S 694
 De Tevera v. Phil. Tuberculosis Society, 112 S 243

2. Corporate Officers (Sec. 25); Qualifications and Disqualifications;


Authority and Liabilities
 Matling v. Coros, October 13, 2010 __________________________126
 Okol v. Slimmers World, December 11, 2011
 Gomez v. PNOC DMC, November 27, 2009
 E.B. Villarosa and Partners, Co. v. Benito, 312 S 65
 SSPC v. Bardaje, 522 S 155
 Cagayan Valley Drug Corp v. CIR, 545 S 10
 Pabon v. NLRC, 296 S 8
 Vlason Enterprise v. CA, 310 S 26
 Prime White Cement v. IAC, 220 S 103
 Louis Vuitton SA v. Villanueva, 216 S 121
3. Executive Committee (Sec. 35)

4. “Doctrine of Apparent Authority’


 Banate v. Philippine Countryside, July 13, 2010 ________________136
 Sargasso v. PPA, July 5, 2010
 Associated Bank v. Sps. Ponstroller, 3 September 2009
 Acuna v. Batac Producers, 20 S 326
 Board of Liquidators v. Kalaw, 20 S 987

5|Page
Law 321_Corporation LAW_ Case Digest

 Francisco v. GSIS, 7 S 577


 Rural Bank v. Ocfemia, 325 S 99

K. Three-Fold Duties of Directors and Officers: Diligence, Loyalty and Obedience

1. Duties (Sec. 31): Business Judgment Rule

2. Personal Liability of Directors and other Corporate Officers


 Ever Electrical v. Samahang Manggagawa, 13 June 2012 ________143
 Harpoon v. Francisco, 2 March 2011
 Ty v. NBI, 15 December 2010
 Queensland-Tokyo Commodities v. George, 8 September 2010
 Wensha Spa Center v. Yung, 16 August 2010
 Cebu Mactan v. Masahiro, 17 July 2009
 David v. National Federation of Labor Unions, 21 April 2009
 Soriano v. People, BSP and PDIC, 30 June 2009
 Cebu Country Club v. Elizagaque, 542 SCRA 65
 Caltex Inc. v. NLRC, 536 SCRA 175
 Atrium Management v. CA, 353 SCRA 23
 ARB Construction v. CA 332 SCRA 426
 Lim v. CA, 232 SCRA 102
 Francisco v. Mejia, 14 August 2001
 DBP v. CA, 16 August 2001
 AHS Philippines v. CA, 257 SCRA 319
 Complex Electronics v. NLRC, 310 SCRA 403
 Crisologo-Jose v. CA, 15 September 1989
 FCY Construction v. CA, 324 SCRA 270
 Llamado v. CA, 270 SCRA 423
 MAM Realty Development v. NLRC, 244 SCRA 797
 Naguiat v. NLRC, 269 SCRA 564
 Progress Homes v. NLRC, 269 SCRA 274
 REAHS Corporation v. NLRC, 271 SCRA 247
 Santos v. NLRC, 254 SCRA 673
 Sia v. People, 121 SCRA 655
 Tramat Mercantile v. CA, 238 SCRA 14

3. Self-Dealing Director/Officer
 Cojuangco v. Republic, 12 April 2011 _________________________170
 Mead v. McCullough, 21 P 95
 Prime White Cement v. IAC, 220 SCRA 103

4. Contracts between Corporations with Interlocking Directors


 Palting v. San Jose Petroleum, 18 SCRA 924 ____________________173
 DBP v. CA, 363 SCRA 307

5. Disloyalty
 Gokongwei Jr. V. SEC, 89 SCRA 336 __________________________175
 Strong v. Repide, 41 P 947

6. Watered Stocks
 Lirag Textile Mills v. SSS, 31 August 1987 ______________________177
 Nava v. Peers Marketing, 25 November 1976

7. Derivative Suit: Remedies to Enforce Personal Liability


 Ang v. Ang, 19 June 2013 __________________________________179
 Legaspi Towers 300 v. Muer, et. al, 18 July 2012
 Lisam Enterprises v. BDO, 23 April 2012
 STRADEC v. Radstock & PNCC, 4 December 2009

6|Page
Law 321_Corporation LAW_ Case Digest

 Yu v. Yukayguan, 18 June 2009


 Gochan v. Young, 12 March 2001
 Western Institute v. Salas, 278 SCRA 216
 First International Bank v. CA, 252 SCRA 259
 Commart Philippines v. SEC, 198 SCRA 73
 Chase v. Buencamino, 136 SCRA 367
 San Miguel Corporation v. Kahn, 11 August 1989
 Everett v. Asia Banking, 49 P 512
 Gamboa v. Victoriano, 90 SCRA 40
 Reyes v. Tan, 3 SCRA 198
 Pascual v. Orozco, 19 P 84

III. POWERS OF CORPORATION (SECS. 36, ET. SEQ.)

A. In General (Sec. 36 in rel. to Arts. 44-46 of the Civil Code of the Philippines)

1. Theory of Special Capacities v. Theory of General Capacities


 Acebedo Optical v. CA, 31 March 2000 _______________________193

2. Express, Implied and Incidental Powers, Distinguished


 Pilipinas v. SEC, 356 SCRA 193 _______________________________194
 Luneta Motors v. Santos, 5 SCRA 809
 Teresa Electric v. PSC, 21 SCRA 199
 Powers v. Marshall, 9 May 1988

3. Power to Have/Use Corporate Name and Seal


 Laureano Investment v. CA, 272 SCRA 253 ____________________198

4. Power to sue and be sued


 Tam v. Hon. Makasiar, 29 January 2001 _______________________199
 Bitong v. CA, 292 SCRA 503
 Special Services Corporation v. Centro La Paz, 28 April 1983
 R Transport Corporation v. CA, 241 SCRA 76

5. Power to acquire, dispose, encumber property


Art. XII, Section 2-3, 1987 Constitution
 Director of Lands v. CA, 14 March 1988 _______________________203

6. Power to Make Donations


 Pirovano v. Dela Rama Steamship Co., 96 P 335 ________________204

7. Other Powers

B. To Increase or Decrease Capital Stock (Sec. 38)


 Madrigal V. Zamora, 51 S 355 ______________________________________205
 Philtrust v. Rivera, 44 P 469

C. To Incur, Create, Or Increase Bonded Indebtedness (Sec. 38)

D. To Deny Pre-Emptive Rights (Sec. 39)


 Datu Benito v. SEC, 123 S 722 and __________________________________207
 Dee v. SEC, 199 S 238 as clarified through
 SEC Letter Opinion, 10 March 2000
 PCGG v. SEC, 30 June 1988
 Republic v. Sandiganbayan, 4 December 2000

E. To Sell Or Otherwise Dispose Of All or Substantially All Of Corporate Assets


(Sec. 40)
In relation to Bulk Sales Law

7|Page
Law 321_Corporation LAW_ Case Digest

 PNB v. Andrada Electric, 381 S 244 __________________________________211


 Islamic Directorate v. CA, 272 S 454
 Edward J. Nell Co. v. Pacific Farms, 15 S 415
 Esguerra v. CA, 3 February 1997
 Lopez Realty v. Fontecha, 247 S 183

F. To Invest Corporate Funds In Another Corporation or Business (Sec. 42)


 Gokongwei v. SEC, 89 S 336 _______________________________________216
 Dela Rama v. Ma-ao Sugar, 7 S 247

G. To Acquire Own Shares (Sec. 41)


 Boman Environmental v. CA, 22 November 1988 _____________________218
 Steinberg v. Velasco, 52 P 953

H. To Declare Dividends

1. Kinds: Cash, Stock, Property, Scrip


 Conjuangco v. Republic, 24 April 2009 ________________________220

2. Declaration, Payment and Record Dates


 Cojuangco, et al v. Sandiganbayan, 24 April 2009 _______supra (220)

3. Limitation on Retention of Surplus Profits


 Steinberg v. Velasco, 52 P 953 _______________________________222
 Nielson v. Lepanto, 26 S 540
 CIR v. Manning, 66 S 14
 Madrigal v. Zamora, 151 S 355
 Republic Planters v. Agana, 269 S 1
 Bitong v. CA, 292 S 503
 CIR v. CA, 301 S 152

I. To Enter into a Management Contract (Sec. 44)


 Aurbach v. Sanitary Wares, 180 S 131 (joint venture) __________________229
 PNB v. Producers’ Warehouse, 42 P 608
 Nielson and Co. v. Lepanto Mining, 26 S 541
 Tuason v. Bolanos, 28 May 1954

J. Ultra Vires Acts (Sec. 45)


 Heirs of Pael v. CA, 371 S 587 ______________________________________233
 Pilipinas Loan v. SEC, 356 S 193
 Crisologo v. Ca, 117 S 594
 Carlos v. Mindoro Sugar, 57 P 343
 Pirovano v. Dela Rama Steamship Co., 96 P 335
 Republic v. Acoje Mining, 7 S 361
 Republic v. Security Credit and Acceptance Corp., 19 S 58

IV. BY-LAWS (Sec. 46, et. seq)

A. Function
 Nakpil v. IBC, 370 S 653 ___________________________________________240
 PMI Colleges v. NLRC, 277 S 462
 Loyola Grand Villas v. CA, 276 S 681
 Citibank NA v. Chua, 220 S 75

B. Kinds

C. When to adopt and file (Sec. 46)


 Loyola Grand Villas v. Ca, 276 S 681 _________________________________244

8|Page
Law 321_Corporation LAW_ Case Digest

D. Contents (Sec. 47)

1. SEC policy on date of annual stockholders’ meeting

2. Authority to elect additional by-laws officers


 Fleischer v. Botica Nolasco, 47 P 583 __________________________245
 Gokongwei Jr. v. SEC, 89 S 336
 Government v. El Hogar Filipino, 50 P 399

E. Amendment and/or rejection of By Laws


 Salafranca v. PhilAmLife, 300 S 469 _________________________________248

V. MEETINGS OF STOCKHOLDERS AND THE BOARD OF DIRECTORS

A. Kinds (Sec. 49)


 Pena v CA, 193 S 717 ________________________________________249

B. When and where held (Secs. 50, 51 and 52 in rel. to Sec 93)

C. Notice required (Secs. 50 and 53)


 Board of Liquidators v. Tan, 105 P 426 _______________________________250

D. Quorum required (Secs. 25 and 52)


 Javellana v. Tayo, 29 December 1962 _______________________________251

E. Who presides (Sec. 54)

F. Who could attend and vote (Secs. 25 and 58)


 Sales v. SEC, 13 January 1989 ______________________________________252
 Ponce v. Encarnacion, 94 P 81
 Lopez v. Ericta, 45 S 539

VI. VOTING

A. Who May Exercise


 Gamboa v. Teves, 28 June 2011 ____________________________________255
 COCOFED, et al. vs. Republic, 11 February 2010 ______________________257
 Republic v. COCOFED, 372 S 462
 Lee v. CA, 205 S 752
 Republic v. Sandiganbayan, 402 S 84

B. Pledgors, mortgagors, executors, receivers and administrators (Sec. 55)

C. Joint owners of stack, ITF shares, and/or shares (Sec. 56)


D. Non-voting shares (Sec. 6)
E. Treasury Shares (Sec. 57 in rel. to Sec. 9)
F. Proxies (Sec. 58)

G. Voting trust agreement (Sec. 59)


 Cordon v. Balicanta, 4 October 2002 ________________________________261
 NIDC v. Aquino, 163 S 153
 Lambert v. Fox, 26 P 588

VII. CAPITAL STRUCTURE STOCKS AND STOCKHOLDERS

A. Capital Stock, Meaning

1. Distinguished from Capital

2. Authorized (Sec. 12)

9|Page
Law 321_Corporation LAW_ Case Digest

3. Subscribed (Sec. 13)

4. Paid-up (Sec. 13)

5. Outstanding (Sec. 143)

6. Pre-requisites to Incorporation (Sec. 13)

7. As Legal/Stated Capital: Trust Fund Doctrine


 PLDT v. NTC, 539 S 365 _____________________________________264
 NTC v CA, 370 P 538 (1999)

8. As Nationality Basis: Control Test vs. “Grandfather” Rule

9. Voting Control Test v. Beneficial Control Test


R.A. 7042, Foreign Investment Act, as amended
SEC Letter-Opinion dated 28 November 2007
 Gamboa v. Teves, 28 June 2011 and 9 October 2012 ___________266
 Express Investment v. BayanTel, 5 December 2012 _____________268
 Redmont Consolidated v. McArthur Mining, SEC En Banc Case No. 09-09-177,
25 March 2010 _____________________________________270
 Agan v. PIATCO, 21 January 2004 ____________________________271

B. Classification of Shares (Sec. 6)

1. Par Value v. No Par Value Shares


2. Voting v. Non-voting
 Gamboa v. Teves, 28 June 2011 _______________________supra (266)
 Castillo v. Balinghasay, 18 October 2004 ______________________272
 Sales v. SEC, 169 S 109

3. Common v. Preferred
4. Kinds of Preferred Shares
5. Founders’ Shares (Sec. 7)

6. Redeemable Preferred (Sec. 8)


 Republic Planters’ Bank v. Agana, 269 S 1 _____________________274

7. Treasury (Sec. 9)
 CIR v. Manning, 66 S 14 _____________________________________275
 San Miguel Corporation v. Sandiganbayan, 14 September 2000

C. Trust Fund Doctrine


 National Telecommunications Commission v. SEC, 311 S 509 ________277
 Ong v. Tiu, 401 S 1

D. What is an “issue”

E. What is a “subscription” (Sec. 60)


 Ong v. Tiu, 401 S 1 ____________________________________________279
 Bayla v. Silang Traffic, 73 P 557
 Salmon, Dexter and Co. v. Unson, 47 P 649
 Sunset View Condominium v. Campos, 104 S 295
 Velasco v. Poizat, 37 P 802

F. Acquisition and Ownership of Shares in a Corporation; Extent of Proprietary


Right/Doctrine of Limited Liability

 Cojuangco v. Republic, 12 April 2011 ____________________________284


 Espiritu v. Petron, 24 November 2009

10 | P a g e
Law 321_Corporation LAW_ Case Digest

 Crisostomo v. SEC, 179 S 146


 Garcia v. Lim, 59 P 562
 Magsaysay-Labrador v. CA, 180 S 266
 Nicolas v. CA, 27 March 1998
 Ramos v. CA, 179 S 719
 Saw v. CA, 195 S 740

G. Pre-incorporation Subscriptions (Sec. 61)

H. Consideration for Stocks (Sec. 62)


 Apodaca v. NLRC, 172 S 442 ____________________________________292
 Fua Cun v. Summers, 44 P 705
 National Exchange Co. v. Dexter, 51 P 601
 Nielson and Co. v. Lepanto Mining, 26 S 541
 Trillana v. Quezon College, 93 P 383

I. Unpaid Subscriptions

1. Interest on unpaid subscriptions (Sec. 66)


2. Right of unpaid shares (Sec. 72)
3. Collection of unpaid subscription

a) Call: When necessary (Sec. 67)


 Garcia v. Suarez, 67 P 441 _________________________297
 PNB v. Bitulok Sawmill, 23 S 1366
 Velasco v. Poizat, 37 P 802

b) Court action (Sec. 70)


 Lumanlan v. Cura, 59 P 746 ________________________300
 Edward Keller v. COB Group Marketing, 16 January 1986

4. How shares become delinquent (Sec. 67)

5. Effect of Delinquency (Sec. 71 in rel. to Sec. 43)


 Valley Golf & Country Club, Inc. v. Caram, 16 April 2009 ______302
 Calatagan Golf Club, Inc. v. Clemente, Jr., 16 April 2009

6. Delinquency Sale (Sec. 68)


7. Grounds to Question Delinquency Sale (Sec. 69)

J. Issuance of Certificates of Stock (Sec. 64)


 Fua Cun v. Summers, 44 P 705 as compared with __________________304
 Baltazar v. Lingayen Gulf, 14 S 522
 Tan v. SEC, 206 S 740
 Embassy Farms v. CA, 188 S 492

K. Right to Transfer of Shares/Validity of Restrictions on Right (Sec. 98 in rel. to S15)


 Makati Sports Club v. Cheng, 16 June 2010 _______________________308
 Fleischer v. Botica Nolasco, 47 P 583
 Padgett and Babcock v. Templeton, 59 P 232
 Rural Bank of Salinas v. CA, 210 S 510
 Thompson v. CA, 298 S 280
 Yuchengco v. Velayo, 115 S 307
 Lim Tay v. CA, 293 S 634

L. Transfer of Shares of Stock and Registration (Sec. 63)


 Musni Puno v. Puno Enterprises, 11 September 2009 _______________315
 Cojuangco, et al v. Sandiganbayan, 24 April 2009
 Republic v. Sandiganbayan, 402 S 84
 Rural Bank of LIpa v. CA, 366 S 188
 BLTB v. Bitanga, 10 August 2001

11 | P a g e
Law 321_Corporation LAW_ Case Digest

 Abejo v. Dela Cruz, 149 S 643


 Batong Buhay Gold Mines v. CA, 147 S 4
 Chemphil Export v. CA, 251 S 257
 Chua Guan v. Samahang Magsasaka , 62 P 472
 CIR v. Anglo-California Bank, 106 P 903
 Delos Santos v. Republic, 96 P 577
 De Erquiaga v. CA, 27 September 1989
 Garcia v. Jomouad, 26 January 2000
 Lopez v. CA, 114 S 671
 Monserrat v. Ceron, 58 P 469
 Puyat v. De Guzman, 113 S 31
 Razon v. IAC, 207 S 234
 Rivera v. Florendo, 144 S643
 Santamaria v. Hongkong and Shanghai Bank, 89 P781
 Torres v. CA, 278 S 793
 Won v. Wack-wack Golf and Country Club, 104 P 466

M. Lost or Destroy Certificates (Sec. 73)


 PHILEX Mining v. Reyes, 118 S602 _______________________________336

VIII. CORPORATE BOOKS AND RECORDS

A. Books to be kept (Sec. 74)

1. Stock and transfer book


 Bitong v. CA, 292 S 503 __________________________________337

2. Stock transfer agent (Sec. 74)


3. Minutes book
4. Record of Business transactions

B. Inspection of corporate books and records (Sec. 74)


 Sy, et al. v. 30 March 2009 ______________________________________338
 Africa v. PCGG, 205 S39
 RP V. Sandiganbayan, 199 S 39
 Gokongwei v. SEC, 89 S 336
 Gonzales v. PNB, 122 S 489
 Pardo v. Hercules Lumber, 47 P 964
 Philpotts v. Philippine Manufacturing Co, 40 P 471
 Republic v. Sandiganbayan, 199 S 39

C. Right to financial statements (Sec. 75)

IX. MERGER AND CONSOLIDATION

A. Corporate Combinations, Purposes and Methods


B. “Constituent” corporation vs. “consolidated” corporation (Sec. 76)
C. Corporate approvals required (Sec. 77)
D. Plan of merger or consolidation (Sec. 76)
E. Articles of merger or consolidation (Sec. 78)

F. Effects of merger or consolidation (Sec. 80)


 BPI v. BPI Employees Union, 18 August 2010 ______________________346
 PNB v. Andrada Electric, 381 S 244
 Babst v. CA, 135 S 37
 Associated Bank v. CA, 290 S 639
 Alger Electric v. CA, 135 S 37
 CIR v. Norton and Harrison, 11 S 714
 CIR v. Rufino, 27 February 1987
 CIR v. Bio Hong, 8 April 1991

12 | P a g e
Law 321_Corporation LAW_ Case Digest

X. APPRAISAL RIGHT

A. Instances of appraisal right (Sec. 81)


B. Requirements of exercise of appraisal right (Sec. 82 and 86)
C. Effect of demand (Sec. 83)
D. Who bears costs of appraisal (Sec. 85)
E. Notation on stock certificates of dissenting stockholders (Sec. 86)

XI. NON-STOCK CORPORATIONS

A. Purposes (Sec. 88)


 Chinese YMCA v. Ching, S 460 __________________________________354
 CIR v. Club Filipino, 5 S 321

B. Distribution of Income (Sec. 87)


C. Scope to right to vote (Sec. 89)

D. Voting (Sec. 89)


 Litonjua v. CA, 286 S136 _______________________________________356
 PPSTA v. Apostol, 55 S 743

E. Transferability of interest or membership (Sec. 90)


F. Governing Board (Sec. 92)
1. Number
2. Term

G. Election of Officers (Sec. 92)


H. Place of meetings (Sec. 93)
I. Distribution of assets in case of dissolution (Sec. 94)

XII. CLOSE CORPORATIONS

A. Requirements for formation (Sec. 96)


 Dulay Enterprises v. CA, 225 S 658 _______________________________358
 San Juan Structural Steel v. CA, 296 S 63
 Naguiat v. NLRC, 269 S 54

B. Restrictions on formation of close corporations (Sec. 96)

C. Distinctions from regular corporations


1. Management (Sec. 97)
2. Meetings (Sec. 101)
3. Voting (Sec. 97)
4. Quorum (Sec. 97)
5. Board authority (Sec. 97)
6. Pre-emptive rights (Sec. 102)
7. Buy back of shares
8. Resolutions of deadlocks (Sec. 104)

D. Provisional director (Sec. 104)


E. Appraisal right in regular corporations vs. withdrawal right of a stockholder of a
close corporation (Sec. 105)

XIII. SPECIAL CORPORATIONS

A. Educational corporations
1. Distinguished from ordinary stocks / Non-Stock Corporations
2. Art. IV, Sec. 28 (3) in rel. to Art. XIV, Sec. 4 (2)(3)(4), 1987 Constitution
B. Religious corporations

1. Art. IV, Sec. 28 (3) and Art. 29 (2), 1987 Constitution


 RP v. IAC, 15 January 1988 _______________________________361

13 | P a g e
Law 321_Corporation LAW_ Case Digest

 Director of Lands v. CA, 14 March 1988

2. Corporation sole

3. Corporation aggregate
 IEMELIF, Inc., et al. v. Bishop Lazaro, et al., 6 July 2010 _______363
 IEMELIF, Inc., et al. v. Juane, 18 September 2009

XIV. DISSOLUTION OF CORPORATIONS

A. Methods

1. Voluntary

a. Where creditors are not affected (Sec. 118)


 Vesagas v. CA, 5 December 2001 ___________________365

b. Where creditors are affected (Sec. 119)


 Avon Dale Garments v. NLRC, 246 S 733 ____________366
 Daguhoy Enterprises v. Ponce, 96 P 15

2. Involuntary (Sec. 121 in rel. to Sec. 6, par. N, PD 902-A)


 PNB v. CFI of Pasig, 209 S 294 ____________________________368

3. Shortening of corporate term (Sec. 120 in rel. to Secs. 16 and 37)

B. Liquidation (Sec. 122)

1. Methods
 Metropolitan v. Centro Development, 13 June 2012 _________369
 Metropolitan Bank Inc. v. Riverside Mills, 8 September 2010
 Yam v. CA, 303 S 1
 Alhambra Cigar and Cigarette Mfg. v. 24 S 269
 Chungka Bio v. IAC, 26 July 1988
 Republic v. Marsman Dev., 27 April 1972
 Tan Tiong Bio v. CIR, 4S 986

2. Duration
 Reynolds, Phil. V. ca, 169 s 220 ___________________________376
 Mambulao v. PNB, 22 S 359

3. Powers of corporation at liquidation


 Aguirre v. FQB7, 9 January 2013 __________________________378
 Catmon Sales v. Liquidator, 15 January 2010
 Knecht v. United Cigarette, 384 S 45
 Chua v. NLRC, 190 S 558
 Clemente v. CA, 242 S 717
 Gelano v. CA, 103 S 90
 Reburian v. CA, 301 S 344
 Republic Planters Bank v. CA, 216 S 738

XV. FOREIGN CORPORATIONS

A. Definition and Rights


 Avon v. Court of Appeals, August 29, 1997 ________________________386
 San Jose Petroleum v. Court of Appeals, 18 SCRA 591

B. Requirements for the Establishment of a Branch/License to do Business in the


Philippines

1. Documentary

14 | P a g e
Law 321_Corporation LAW_ Case Digest

 Georg Grotjahn vs. Isnani, 235 SCRA 216 __________________388

2. Deposit

3. Appointment of Resident Agent


 New York Marine Managers vs. CA, 249 S 417 ______________389

C. Applicable laws

D. Amendment of License
 Aetna Casualty vs. Pacific Star, 29 December 1977 __________________390
 Bulakhidas vs. Navarro, 7 April 1986
 Shmid and Oberly vs RJL, 18 October 1988

E. “Doing business” with or without license: Suits By or Against Foreign Corporation


 AM No. 11-3-6-SC New Rule on Service Summons on Foreign Juridical
Entities
 Steelcase, Inc. vs. Desing International, 18 April 2012 _______________393
 PDIC vs. Citibank, 11 April 2012
 Cargill, Inc. vs. Intra Strata, 15 March 2010
 Sehwani vs. In and Out Burger, 536 S 225
 MR Holdings vs. Bajar, 380 S 617
 Commissioner of Customs vs. KMK Gani, 182 S 591
 Communications an Materials Designs vs. CA, 260 S 144
 Columbia Pictures vs. Court of Appeals, 261 S 144
 Eriks PTE Ltd. Vs. CA, 276 S 567
 Far East International vs. Nnkai Kogyo, 6 S 725
 Facilities Management vs. Dela Osa, 89 S 131
 HB Zachray and Co. vs. Court of Appeals, 232 S 29
 Hutchison Ports vs. SBMA, 31 August 2000
 La Chemise Lacoste vs. Fernandez, 129 S 373
 Marubeni Nederlands vs. Tensuan, 28 September 1990
 Phil. Columbia vs. Lantin, 39 S 376
 Philip Morris vs. Fortune Tobacco, 493 S 333
 Puma vs. IAC, 158 S 233
 SBMA vs. Universal International, 14 September 2000
 Tibe vs. Reyes, 39 S 304
 Universal Rubber vs. Court of Appeals, 130 S 104
 Van Zuiden vs. GTVL Industries, 523 S 233
SECURITIES AND EXCHANGE COMMISSION LAW
(P.D. No. 902-A, as Amended by R.A. No. 8799 or Securities Regulation Code)

I. STRUCTURE OF THE SECURITIES AND EXCHANGE COMMISSION

II. ENTITIES UNDER ABSOLUTE JURISDICTION, SUPERVISION AND CONTROL OF THE SEC

A. Corporations, partnerships or associations which are grantees of primary


franchises
B. Investment Houses
C. Financing Companies

III. POWERS AND FUNCTIONS OF THE SEC


 SEC vs. PFEC, 495 S 579 _______________________________________________415
 Arranza vs. B.F. Homes, Inc., 19 June 2000
 Quasha vs. SEC, 83 S 557
 Traders’ Royal Bank vs. Court of Appeals, 26 September 1989
 VICMAR Development vs. Court of Appeals, 185 S 634

IV. ORIGINAL AND EXCLUSIVE JURISDICTION OF THE REGIONAL TRIAL COURTS


 Orendain vs. BF Homes, 506 S 634 _____________________________________420

15 | P a g e
Law 321_Corporation LAW_ Case Digest

 Pascual vs. Court of Appeals, 339 S 117

A. Devices of schemes amounting to fraud of misrepresentation


 Fabia vs. Court of Appeals, 388 S 574 ____________________________422
 A & A continental vs. SEC, 225 S 314
 Alleje vs. Court of Appeals, 240 S 495
 Banez vs. Dimensional Construction , 140 S 249
 Sesbreno vs. Court of Appeals, 240 S 606

B. Controversies arising out of intra-corporate of partnership relations


 Aguirre vs. FQB7, 9 January 2013 _______________________________427
 Go, Lim, et al. vs. Distinction Properties, 25 April 2012
 Strategic Alliance vs. Star Infrastructure, 17 November 2010
 GD Express vs. Court of Appeals, 8 May 2009
 Iglesia vs. Juane 18 September 2009
 GD Express Worldwide N.V. vs. Court of Appeals, 8 May 2009
 Intestate Estate of Ty vs. Court of Appeals, 356 S 661
 Fabia vs. Court of Appeals, 363 S 427
 Vesagas vs. Court of Appeals, 371 S 508
 Abejo vs. Dela Cruz, 149 S 654
 Aguinaldo vs. SEC, 163 S 262
 Pereyra vs. IAC, 181 S 244
 Mainland Construction vs. Molvilla, 250 S 290
 SEC vs. Court of Appeals, 201 S 124
 Sunsetview Condominum vs. Campos, 104 S 295
 Western Institute of Technology vs. Salas, 21 August 1997

C. Controversies in the Election or appointment of corporate officers


 Real v. Sangu Phil., 19 January 2011 _____________________________442
 March II marketing v. Joson, 12 December 2011
 Matling v. Coros, 13 October 2010
 Garcia v. Eastern Telecom, 1 April 2009
 De Rossi v. NLRC, 314 S 245
 Espino v. NRC, 240 S 52
 Estrada v. NLRC, 262 S 709
 Islamic Directorate v. CA 272 S 454
 Ongkiko v. NLRC, 270 S 613
 Paguio v. NLRC, 253 S 166
 Pearson and George v. NLRC, 253 S 136
 Apodaca v. NLRC, 172 S 442
 PSBA v. Leano, 127 S 778
 Tabang v. NLRC, 266 S 462
 Union Motors v. NLRC, 314 S 531

D. Petitions for declaration in the state of suspension of payments


R.A. 10142 – The Financial Rehabilitation and Insolvency Act of 2010
A.M. No. 12-12-11-SC 2013 Financial Rehabilitation Rules of Procedure
 Express Investment v. Bayantel, 5 December 2012 _________________457
 Advent Capital v. Alcantara, 25 January 2012
 Siochi Fichery v. BPI, 19 October 2011
 Panililio v. RTC, 2 February 2011
 Castillo v. Uniwide Warehouse, 30 April 2010
 Pacific Wide v. Puerto Asul, 25 November 2009
 PNB and ECPIB v. CA, 20 January 2009
 Pryce Corp. v. CA, 543 S 657
 Uniwide v. Jandecs Corp. 541 S 158
 BPI v. SEC, 541 S 294
 Ching v. Land Bank of the Philippines, 201 S 191
 PCIB v. CA, 18 April 1989
 Radiola-Toshiba v. IAC, 18 July 1991

16 | P a g e
Law 321_Corporation LAW_ Case Digest

 RCBC v. IAC, 213 S 223


 Rubberworld v. NLRC, 305 S 722
 Union Bank v. CA, 19 May 1998

SECURITIES REGULATIONS CODE


(Republic Act No. 8799)

I. OVERVIEW OF THE FINANCIAL MARKETS

A. Capital Markets
1. Equities Capital (e.g. stock market)
2. Debt Capital (e.g. money market or bond market)

B. Non-Capital Markets
1. Commodities Market
2. Foreign Exchange Market
3. Options Market

II. REGISTRATION OF SECURITIES

A. Securities Defined (Sec. 3.1)


B. Elements of an Investment Contract
 Securities and Exchange Commission v. W.J. howey Co., 328 U.S. 293 (1946)
 Securities and Exchange Commission v. prosperity.Com, Inc., 25 January 2012
 Power homes Unlimited v. SEC and Manero, 2 February 2008
 SFC v. Performance, 495 S 579
 Suzuki v. De Guzman, 496 S 651
 Baviera v. Paglinawan, 515 S 170

17 | P a g e
Law 321_Corporation LAW_ Case Digest

C. What securities are required to be registered (Sec. 8)


 Timeshare Realty v. Lao, 544 S 254
 Makati Stock Exchange v. SEC, 14 S 620
 La Orden v. Stiver and Philtrust, 93 P 341
 Philippine Stock Exchenge v. SEC, 281 S 232

D. Exempt Securities (Sec. 9)

E. Exempt Transactions 9Sec 10)


 Timeshare Realty Co. v. Lao, 522 S 254
 Nestle Phils. V. CA, 203 S 504

F. Public Companies
 Philippine Veterans Bank v. Callangan, 3 August 2011

G. Registration Statement (Sec. 12)


1. Contents (Sec. 12.1-2.3)
2. Attachments
3. Signature (Sec. 12.4)

H. Grounds for Rejection of Registration Statement (Sec. 13)

I. False Registration (Sec. 56)


J. Limitations on Actions for False Registration Statement (Sec. 62)

III. TRADING IN SECURITIES

A. Margin Requirements (Sec. 48) and Restrictions on Borrowings (Sec. 49)

 Carolina Industries v. CMS Stock Brokerage, 97 S 734

B. Brokers/Dealers: Chinese Wall; Self-Regulatory Organizations; Stock


Exchange
C. Regulation of Options Trading (Sec. 25)
1. Option
2. Put
3. Call
4. Straddle or Spread

D. Manipulations of Security Prices; Devices and Practices (Sec. 24)


1. Boiler Room Operations
2. Wash Sales
3. Daisy Chain
4. Painting the Tape
5. Marketing the Close
6. Hype and Dump
7. Short Sale
8. Matched Order
9. Stop-Loss Order

E. Fraudulent Transactions (Sec. 26)

 Phil. Asso. Of Stock Transfer and Agencies v. CA


 SEC v. CA, 246 S 738
 Onapal v. CA, 218 S 281

F. Insider’s Duty to Disclose When Trading (Sec. 27)


1. Insider Defined
2. Material or Significant Facts
 Strong v. Repide, 41 P 947

18 | P a g e
Law 321_Corporation LAW_ Case Digest

3. Disclosure Regulations for Publicly-Listed Shares


 Union Bank of the Phil. v. SEC, June 2001

IV. PROTECTION OF SHAREHOLDER’S INTEREST

A. Tender Offers (Sec. 19)


 CEMCO v. National Life, 7 August 2007
B. Transactions of Directors, Officers and Principal Stockholders (Sec 23)

V. LIABILITIES

A. Administrative Sanctions (Sec. 54)


B. Civil Liabilities (Secs. 58-61)
1. Amount and Kinds of Damages (Sec. 63.1)
2. Solidary Liability (Secs. 63.2 and 63.3)
3. Limitation of Actions (Sec. 62)
C. Criminal Liabilities (Sec. 73)
D. Settlements/Nolo Contendere or Consent Decree (Sec. 55)

CORPORATION CODE
(Batas Pambansa Blg. 68)

Formation and Organization of Corporations


 Attributes of Corporation

PETRON CORPORATION
vs.
NATIONAL COLLEGE OF BUSINESS AND ARTS
G.R. No. 155683. February 16, 2007

FACTS:

The V. Mapa properties owned by Felipe and Enrique Monserrat, Jr., were mortgaged to DBP as
part of the security for the loan of P5.2 million by MYTC and Monserrat Co. MYTC mortgaged four
parcels of land located in Manila. One-half of Felipe’s undivided interest in the V. Mapa properties was
levied upon in execution of a money judgment rendered by the RTC in the Manila case. DBP challenged
the levy through a third-party claim asserting that the V. Mapa properties were mortgaged to it and
were, for that reason, exempt from levy or attachment. The RTC quashed it. MYTC and the Monserrats
got DBP to accept a dacion en pago arrangement whereby MYTC conveyed to the bank the four
mortgaged Quiapo properties as full settlement of their loan obligation. But despite this agreement, DBP
did not release the V. Mapa properties from the mortgage. Felipe, acting for himself and as Enrique’s
attorney-in-fact, sold the V. Mapa properties to respondent NCBA. The Monserrats failed to comply with
this undertaking. This instigated the civil action filed by NCBA.
During the pendency of the case, ½ of Enrique’s undivided interest in the V. Mapa properties was
levied on in execution of a judgment of the Makati case holding him liable to Petron on a 1972

19 | P a g e
Law 321_Corporation LAW_ Case Digest

promissory note. Petron, the highest bidder, acquired both Felipe’s and Enrique’s undivided interests in
the property. Petron intervened in the NCBA case.

ISSUE:

Whether or not Petron should be held liable for exemplary damages and attorney’s fees.

RULING:

NO.

Article 2208(5) contemplates a situation where one refuses unjustifiably and in evident bad faith
to satisfy another’s plainly valid, just and demandable claim, compelling the latter needlessly to seek
redress from the courts. In such a case, the law allows recovery of money the plaintiff had to spend for a
lawyer’s assistance in suing the defendant – expenses the plaintiff would not have incurred if not for the
defendant’s refusal to comply with the most basic rules of fair dealing. It does not mean, however, that
the losing party should be made to pay attorney’s fees merely because the court finds his legal position
to be erroneous and upholds that of the other party, for that would be an intolerable transgression of
the policy that no one should be penalized for exercising the right to have contending claims settled by a
court of law. In fact, even a clearly untenable defense does not justify an award of attorney’s fees unless
it amounts to gross and evident bad faith.
No gross and evident bad faith could be imputed to Petron merely for intervening in NCBA’s suit
against DBP and the Monserrats in order to assert what it believed and had good reason to believe. The
rule in this jurisdiction is that the plaintiff must show that he is entitled to moral, temperate or
compensatory damages before the court may even consider the question of whether exemplary damages
should be awarded. No exemplary damages may be awarded without the plaintiff’s right to moral,
temperate, liquidated or compensatory damages having first been established.
ASSET PRIVATIZATION TRUST
vs.
COURT OF APPEALS, JESUS S. CABARRUS, SR., JESUS S. CABARRUS, JR., JAIME T. CABARRUS,
JOSE MIGUEL CABARRUS, ALEJANDRO S. PASTOR, JR., ANTONIO U. MIRANDA, and MIGUEL M.
ANTONIO, as Minority Stock-Holders of Marinduque Mining and Industrial Corporation
G.R. No. 121171. December 29, 1998

FACTS:

MMIC, PNB and DBP executed a Mortgage Trust Agreement whereby MMIC, as mortgagor, agreed
to constitute a mortgage in favor or PNB and DBP as mortgagees, over all MMIC's assets. Article IV of
the Mortgage Trust Agreement provides for Events of Default, which expressly includes the event that
the MORTGAGOR shall fail to pay any amount secured by this Mortgage Trust Agreement when due. In
various requests for advances/remittances of loans if huge amounts, Deeds of Undertaking, Promissory
Notes, Loan Documents, Deeds of Real Estate Mortgages, MMIC invariably committed to pay either on
demand or under certain terms the loans and accommodations secured from or guaranteed by both
DBP and PNB. Because of the tremendous loans obtained, a financial restructuring plan (FRP) designed
to reduce MMIC's interest expense through debt conversion to equity was drafted SGV, however, it was
never adopted.
The various loans and advances made by DBP and PNB to MMIC had become overdue and since
any restructuring program relative to the loans was no longer feasible, and in compliance with the
directive of Presidential Decree No. 385, DBP and PNB as mortgagees of MMIC assets, decided to
exercise their right to extrajudicially foreclose the mortgages in accordance with the Mortgage Trust
Agreement. The assets were eventually transferred to APT. SHs of MMIC thereafter filed a derivative suit
against DBP and PNB praying that the foreclosure be annulled, that the FRP be followed and damages.
In arbitration proceedings, MMIC obtained a favorable decision. Court of Appeals denied due course
and dismissed the petition for certiorari.

ISSUE:

Whether or not the MMIC is entitled to moral damages.

RULING:

NO.

20 | P a g e
Law 321_Corporation LAW_ Case Digest

As a rule, a corporation exercises its powers, including the power to enter into contracts, through
its board of directors. While a corporation may appoint agents to enter into a contract in its behalf, the
agent should not exceed his authority. In the case at bar, there was no showing that the representatives
of PNB and DBP in MMIC even had the requisite authority to enter into a debt-for-equity swap. And if
they had such authority, there was no showing that the banks, through their board of directors, had
ratified the FRP.
Further, how the MMIC could be entitled to a big amount of moral damages when its credit
reputation was not exactly something to be considered sound and wholesome. Under Article 2217 of the
Civil Code, moral damages include besmirched reputation which a corporation may possibly suffer. A
corporation whose overdue and unpaid debts to the Government alone reached a tremendous amount of
P22 Billion Pesos cannot certainly have a solid business reputation to brag about.

MAMBULAO LUMBER COMPANY


vs.
PHILIPPINE NATIONAL BANK and ANACLETO HERALDO Deputy Provincial Sheriff of Camarines
Norte
G.R. No.L-22973.January 30, 1968

FACTS:

Plaintiff applied for an industrial loan of P155, 000.00 with the PNB and the former offered real
estate, machinery, logging and transportation equipment as collaterals. The application was approved
for a loan of P100, 000.00 only. To secure the payment of the loan, the plaintiff mortgaged to defendant
PNB a parcel of land, together with the buildings and improvements existing thereon, situated in the
province of Camarines Norte, and covered by TCT No. 381 of the land records of said province, as well as
various sawmill equipment, rolling unit and other fixed assets of the plaintiff, all situated in its
compound in the aforementioned municipality.
PNB released from the approved loan the sum of P27, 500.00, for which the plaintiff signed a
promissory note wherein it promised to pay to the PNB. PNB made another release of P15, 500.00 as
part of the approved loan granted to the plaintiff and so on the said date, the latter executed another
promissory note. Plaintiff failed to pay the amortization on the amounts released to and received by it.
Repeated demands were made upon the plaintiff to pay its obligation but it failed or otherwise refused to
do so. Upon inspection and verification made by employees of the PNB, it was found that the plaintiff
had already stopped operation.
PNB initiated steps to have the properties extrajudicially foreclosed. The Plaintiff opposed. The
foreclosure sale of the parcel of land, together with the buildings and improvements thereon, was held
and the said property was sold to the PNB for the sum of P56, 908.00, subject to the right of the plaintiff
to redeem the same within a period of one year. PNB sold the properties to Mariano Bundok. The
Security guard of the properties refused to let PNB’s successor in interest to retrieve properties inside
the premises of the property bought by them.
RTC sentenced the Mambulao Lumber Company to pay to the defendant PNB. Mambulao
therefore appealed.

ISSUE:

Whether or not a corporation can be awarded moral damages.

RULING:

NO.

An artificial person like herein appellant corporation cannot experience physical sufferings,
mental anguish, fright, serious anxiety, wounded feelings, moral shock or social humiliation which are
basis or moral damages.
A Corporation may have a good reputation if besmirched, may also be a ground for the award of
moral damages. The same cannot be considered under the facts of this case, however, not only because

21 | P a g e
Law 321_Corporation LAW_ Case Digest

it is admitted that herein appellant had already ceased in its business operation at the time of the
foreclosure sale of the chattels, but also for the reason that whatever adverse effects of the foreclosure
sale of the chattels could have upon its reputation or business standing would undoubtedly be the same
whether the sale was conducted at Camarines Norte, or in Manila which is the place agreed upon by the
parties in the mortgage contract.
But for the wrongful acts of herein appellee bank and the deputy sheriff of Camarines Norte in
proceeding with the sale in utter disregard of the agreement to have the chattels sold in Manila as
provided for in the mortgage contract, to which their attentions were timely called by herein appellant,
and in disposing of the chattels in gross for the miserable amount of P4, 200.00, herein appellant
should be awarded exemplary damages in the sum of P10, 000.00. The circumstances of the case also
warrant the award of P3, 000.00 as attorney's fees for herein appellant.

HANIL DEVELOPMENT CO., LTD.


vs.
COURT OF APPEALS AND M.R. ESCOBAR EXPLOSIVE ENGINEERS, INC.
G.R. No. 113176.July 30, 2001

FACTS:

MPWH awarded petitioner Hanil the contract to construct the 200-kilometer Iligan-Cagayan de
Oro-Butuan Highway Project. Hanil sub-let the rock-blasting work portion of the contract to private
respondent Escobar. For the duration of the contract, it worked on the segments of the construction
undertaking designated in the agreement as A-2, B-2, B-3, B-4, and C-1. It was fully paid for the areas
A-2 and B-4. It claimed, however, that Hanil still partially owes it one million three hundred forty one
thousand seven hundred twenty-seven and 40/100 (P1, 341, 727.40) pesos for blastings done in the B-
2, B-3 and C-1 areas. The claim was predicated on the theory that the rocks it caused to explode in the
contested areas were solid in nature, and therefore the volume should be computed using the cross-
section approach.
Escobar filed an action for recovery of a sum of money with damages against Hanil in the CFI.
CFI ordered Hanil to pay P1, 341, 727.40 for the value of rocks blasted by Escobar; 10% of the amount
due for attorney's fees; and the costs of suit. CFI garnished the bank accounts of Hanil and levied its
equipment. CFI also granted Escobar's Ex-parte Motion to Deposit Cash praying that the Finance
Manager of the NAPOCOR be directed to withdraw Hanil's funds from the NAPOCOR and deposit the
same with the Clerk of Court. Hanil challenged the Orders before the CA, who voided said orders.

ISSUE:

Whether or not Hanil should be awarded a much higher grant of nominal damages and attorney’s
fees and whether they are entitled to moral and exemplary damages.

RULING:

NO.

As to the temperate damages in form of nominal damages, Hanil is not entitled for it failed to
prove that it deserves a grant of a higher amount. Thus, P20, 000.00 is just. Hanil failed to prove the
actual value of pecuniary injury which it sustained as a consequence of Escobar's institution of an
unfounded civil suit. The testimony of one of its witnesses presented in the CFI, to the effect that "the
filing of the complaint affected Hanil's reputation and that it affected the management and engineers
working in the site," is not enough proof. The institution of the suit, unfounded though it may be, does
not always lead to pecuniary loss as to warrant an award of actual or temperate damages. The link
between the cause (the suit) and the effect (the loss) must be established by the required proof.
Its demand for payment of moral damages must also fail. The rule is that moral damages cannot
be granted in favor of a corporation. Being an artificial person and having existence only in legal
contemplation, a corporation has no feelings, no emotions, and no senses. It cannot, therefore,
experience physical suffering, mental anguish, fright, serious anxiety, wounded feelings or moral shock
or social humiliation, which can be suffered only by one having a nervous system. Hanil's prayer for
exemplary damages must likewise be denied. It must be remembered that this kind of damages cannot
be recovered as a matter of right. Its allowance rests in the sound discretion of the court, and only upon
a showing of its legal foundation. Under the Civil Code, the claimant must first establish that he is
entitled to moral, temperate, compensatory or liquidated damages before it may be imposed in his favor.
Hanil failed to do so, hence, it cannot claim exemplary damages.

22 | P a g e
Law 321_Corporation LAW_ Case Digest

BACHE & CO. (PHIL.), INC. and FREDERICK E. SEGGERMAN


vs.
HON. JUDGE VIVENCIO M. RUIZ, MISAEL P. VERA, in his capacity as Commissioner of Internal
Revenue, et al.
G.R. No.L-32409. February 27, 1971

FACTS:

Misael P. Vera, CIR wrote a letter to respondent Judge Vivencio M. Ruiz requesting the issuance
of a search warrant against petitioners for violation of Section 46(a) of the National Internal Revenue
Code, in relation to all other pertinent provisions thereof, particularly Sections 53, 72, 73, 208 and 209,
and authorizing Revenue Examiner Rodolfo de Leon, one of herein respondents, to make and file the
application for search warrant which was attached to the letter. De Leon and his witness, respondent
Arturo Logronio, went to the Court of First Instance of Rizal. They brought with them the following
papers: respondent Vera’s aforesaid letter-request; an application for search warrant already filled up
but still unsigned by respondent De Leon; an affidavit of respondent Logronio subscribed before
respondent De Leon; a deposition in printed form of respondent Logronio already accomplished and
signed by him but not yet subscribed; and a search warrant already accomplished but still unsigned by
respondent Judge. Judge was hearing a certain case so he instructed his Deputy Clerk of Court to take
the depositions of respondents De Leon and Logronio. After the session had adjourned, respondent
Judge was informed that the depositions had already been taken. Respondent Judge signed respondent
de Leon’s application for search warrant and respondent Logronio’s deposition, Search Warrant No. 2-M-
70 was then sign by respondent Judge and accordingly issued.
BIR agents served the search warrant petitioners at the offices of petitioner corporation.
Petitioners’ lawyers protested the search on the ground that no formal complaint or transcript of
testimony was attached to the warrant. The agents nevertheless proceeded with their search which
yielded six boxes of documents.

ISSUES:

Whether or not a corporation is entitled to protection against unreasonable search and seizures.

RULING:

YES.

Although, for the reasons above stated, the Supreme Court is of the opinion that an officer of a
corporation which is charged with a violation of a statute of the state of its creation, or of an act of
Congress passed in the exercise of its constitutional powers, cannot refuse to produce the books and
papers of such corporation, the Court does not wish to be understood as holding that a corporation is
not entitled to immunity against unreasonable searches and seizures. A corporation is, after all, but an
association of individuals under an assumed name and with a distinct legal entity. In organizing itself as
a collective body it waives no constitutional immunities appropriate to such body. Its property cannot be
taken without compensation. It can only be proceeded against by due process of law, and is protected
against unlawful discrimination.

SULO NG BAYAN INC.


vs.
GREGORIO ARANETA, INC., PARADISE FARMS, INC., NATIONAL WATERWORKS & SEWERAGE
AUTHORITY, HACIENDA CARETAS, INC, and REGISTER OF DEEDS OF BULACAN
G.R. No.L-31061. August 17, 1976

23 | P a g e
Law 321_Corporation LAW_ Case Digest

FACTS:

Sulo ng Bayan, Inc. filed an accion de revindicacion with the CFI against defendants-appellees to
recover the ownership and possession of a large tract of land. The complaint specifically alleged that
plaintiff is a corporation organized and existing under the laws of the Philippines, with its principal
office and place of business at San Jose del Monte, Bulacan; that its membership is composed of
natural persons residing at San Jose del Monte, Bulacan; that the members of the plaintiff corporation,
through themselves and their predecessors-in-interest, had pioneered in the clearing of the fore-
mentioned tract of land, cultivated the same since the Spanish regime and continuously possessed the
said property openly and public under concept of ownership adverse against the whole world.

ISSUE:

Whether or not plaintiff corporation may institute an action in behalf of its individual members
for the recovery of certain parcels of land allegedly owned by said members; for the nullification of the
transfer certificates of title issued in favor of defendants appellees covering the aforesaid parcels of land;
for a declaration of "plaintiff's members as absolute owners of the property" and the issuance of the
corresponding certificate of title; and for damages.

RULING:

NO.

It is a doctrine well-established and obtains both at law and in equity that a corporation is a
distinct legal entity to be considered as separate and apart from the individual stockholders or members
who compose it, and is not affected by the personal rights, obligations and transactions of its
stockholders or members. The property of the corporation is its property and not that of the
stockholders, as owners, although they have equities in it. Properties registered in the name of the
corporation are owned by it as an entity separate and distinct from its members. Conversely, a
corporation ordinarily has no interest in the individual property of its stockholders unless transferred to
the corporation, "even in the case of a one-man corporation. The mere fact that one is president of a
corporation does not render the property which he owns or possesses the property of the corporation,
since the president, as individual, and the corporation are separate similarities. Similarly, stockholders
in a corporation engaged in buying and dealing in real estate whose certificates of stock entitled the
holder thereof to an allotment in the distribution of the land of the corporation upon surrender of their
stock certificates were considered not to have such legal or equitable title or interest in the land, as
would support a suit for title, especially against parties other than the corporation.
It must be noted, however, that the juridical personality of the corporation, as separate and
distinct from the persons composing it, is but a legal fiction introduced for the purpose of convenience
and to subserve the ends of justice. This separate personality of the corporation may be disregarded, or
the veil of corporate fiction pierced, in cases where it is used as a cloak or cover for fraud or illegality, or
to work -an injustice, or where necessary to achieve equity.
Clearly, no right of action exists in favor of plaintiff corporation, for as shown heretofore it does
not have any interest in the subject matter of the case which is material and, direct so as to entitle it to
file the suit as a real party in interest.

 Private v. Public Corporation

BOY SCOUTS OF THE PHILIPPINES


vs.
COMMISSION ON AUDIT
G.R. No. 177131.June 7, 2011

FACTS:

The COA maintains that the functions of the BSP that include, among others, the teaching to the
youth of patriotism, courage, self-reliance, and kindred virtues, are undeniably sovereign functions
enshrined under the Constitution and discussed by the Court in Boy Scouts of the Philippines v.
National Labor Relations Commission. The COA contends that any attempt to classify the BSP as a
private corporation would be incomprehensible since no less than the law which created it had
designated it as a public corporation and its statutory mandate embraces performance of sovereign

24 | P a g e
Law 321_Corporation LAW_ Case Digest

functions. The COA claims that the only reason why the BSP employees fell within the scope of the Civil
Service Commission even before the 1987 Constitution was the fact that it was a government-owned or
controlled corporation; that as an attached agency of the Department of Education, Culture and Sports
(DECS), the BSP is an agency of the government; and that the BSP is a chartered institution under
Section 1(12) of the Revised Administrative Code of 1987, embraced under the term government
instrumentality. The COA concludes that being a government agency, the funds and property owned or
held by the BSP are subject to the audit authority of the COA pursuant to Section 2(1), Article IX (D) of
the 1987 Constitution.
BSP claims that it has a unique characteristic which "neither classifies it as a purely public nor a
purely private corporation"; that it is not a quasi-public corporation; and that it may belong to a
different class altogether.

ISSUE:

Whether or not the BSP is public corporation.

RULING:

YES.

BSP is a public corporation and its funds are subject to the COA’s audit jurisdiction. It is a public
corporation or a government agency or instrumentality with juridical personality, which does not fall
within the constitutional prohibition in Article XII, Section 16, notwithstanding the amendments to its
charter. Not all corporations, which are not government owned or controlled, are ipso facto to be
considered private corporations as there exist another distinct class of corporations or chartered
institutions which are otherwise known as "public corporations." These corporations are treated by law
as agencies or instrumentalities of the government which are not subject to the tests of ownership or
control and economic viability but to different criteria relating to their public purposes/interests or
constitutional policies and objectives and their administrative relationship to the government or any of
its Departments or Offices.
Note that the Administrative Code of 1987 designates the BSP as one of the attached agencies of
the Department of Education, Culture and Sports ("DECS"). An "agency of the Government" is defined as
referring to any of the various units of the Government including a department, bureau, office, and
instrumentality, government-owned or -controlled corporation, or local government or distinct unit
therein. BSP still remains an instrumentality of the national government. It is a public corporation
created by law for a public purpose, attached to the DECS pursuant to its Charter and the
Administrative Code of 1987. It is not a private corporation which is required to be owned or controlled
by the government and be economically viable to justify its existence under a special law.

DANTE V. LIBAN, REYNALDO M. BERNARDO, and SALVADOR M. VIARI


vs.
RICHARD J. GORDON
G.R. No. 175352.July 15, 2009

FACTS:

Petitioners filed with this Court a Petition to Declare Richard J. Gordon as Having Forfeited His
Seat in the Senate. Petitioners are officers of the Board of Directors of the Quezon City Red Cross
Chapter while respondent is Chairman of the Philippine National Red Cross (PNRC) Board of Governors.
During respondent’s incumbency as a member of the Senate of the Philippines, he was elected
Chairman of the PNRC during the February 23, 2006 meeting of the PNRC Board of Governors.
Petitioners allege that by accepting the chairmanship of the PNRC Board of Governors, respondent has
ceased to be a member of the Senate as provided in Section 13, Article VI of the Constitution, which
reads: “No Senator or Member of the House of Representatives may hold any other office or employment
in the Government, or any subdivision, agency, or instrumentality thereof, including government-owned
or controlled corporations or their subsidiaries, during his term without forfeiting his seat. Neither shall
he be appointed to any office which may have been created or the emoluments thereof increased during
the term for which he was elected.”

ISSUE:

25 | P a g e
Law 321_Corporation LAW_ Case Digest

Whether or not the office of the PNRC Chairman is a government office or an office in a
government-owned or controlled corporation for purposes of the prohibition in Section 13, Article VI of
the Constitution.

RULING:

NO.

PNRC is a Private Organization Performing Public Functions. The Republic of the Philippines,
adhering to the Geneva Conventions, established the PNRC as a voluntary organization for the purpose
contemplated in the Geneva Convention of 27 July 1929. The PNRC must not appear to be an
instrument or agency that implements government policy; otherwise, it cannot merit the trust of all and
cannot effectively carry out its mission as a National Red Cross Society. It is imperative that the PNRC
must be autonomous, neutral, and independent in relation to the State. To ensure and maintain its
autonomy, neutrality, and independence, the PNRC cannot be owned or controlled by the government.
Indeed, the Philippine government does not own the PNRC. The PNRC does not have government assets
and does not receive any appropriation from the Philippine Congress. The PNRC is financed primarily by
contributions from private individuals and private entities obtained through solicitation campaigns
organized by its Board of Governors. The government does not control the PNRC. Under the PNRC
Charter, as amended, only six of the thirty members of the PNRC Board of Governors are appointed by
the President of the Philippines.
The PNRC is not government-owned but privately owned. The vast majority of the thousands of
PNRC members are private individuals, including students. Under the PNRC Charter, those who
contribute to the annual fund campaign of the PNRC are entitled to membership in the PNRC for one
year. Thus, the PNRC is a privately owned, privately funded, and privately run charitable organization.
Hence, the office of the PNRC Chairman is not a government office or an office in a government-
owned or controlled corporation for purposes of the prohibition in Section 13, Article VI of the 1987
Constitution. However, since the PNRC Charter is void insofar as it creates the PNRC as a private
corporation, the PNRC should incorporate under the Corporation Code and register with the Securities
and Exchange Commission if it wants to be a private corporation.

DANTE V. LIBAN, REYNALDO M. BERNARDO and SALVADOR M. VIARI


vs.
RICHARD J. GORDON
G. R. No. 175352.January 18, 2011

ISSUE:

Whether or not considering the PNRC’s structure is sui generis; it is a class of its own, while it is
performing humanitarian functions as an auxiliary to government, it is a neutral entity separate and
independent of government control, yet it does not qualify as strictly private in character.

RULING:

YES.

A National Society partakes of a sui generis character. It is a protected component of the Red
Cross movement under Articles 24 and 26 of the First Geneva Convention, especially in times of armed
conflict. These provisions require that the staff of a National Society shall be respected and protected in
all circumstances. Such protection is not ordinarily afforded by an international treaty to ordinary
private entities or even non-governmental organizations (NGOs). This sui generis character is also
emphasized by the Fourth Geneva Convention which holds that an Occupying Power cannot require any
change in the personnel or structure of a National Society. National societies are therefore organizations
that are directly regulated by international humanitarian law, in contrast to other ordinary private
entities, including NGOs.
The PNRC, as a National Society of the International Red Cross and Red Crescent Movement, can
neither "be classified as an instrumentality of the State, so as not to lose its character of neutrality" as
well as its independence, nor strictly as a private corporation since it is regulated by international
humanitarian law and is treated as an auxiliary of the State. The PNRC is one of the National Red Cross
and Red Crescent Societies, which, together with the International Committee of the Red Cross (ICRC)
and the IFRC and RCS, make up the International Red Cross and Red Crescent Movement (the
Movement). They constitute a worldwide humanitarian movement.

26 | P a g e
Law 321_Corporation LAW_ Case Digest

Based on the above, the sui generis status of the PNRC is now sufficiently established. Although
it is neither a subdivision, agency, or instrumentality of the government, nor a government-owned or
-controlled corporation or a subsidiary thereof, as succinctly explained in the Decision of July 15, 2009,
so much so that respondent, under the Decision, was correctly allowed to hold his position as Chairman
thereof concurrently while he served as a Senator, such a conclusion does not ipso facto imply that the
PNRC is a "private corporation" within the contemplation of the provision of the Constitution, that must
be organized under the Corporation Code. As correctly mentioned by Justice Roberto A. Abad, the sui
generis character of PNRC requires us to approach controversies involving the PNRC on a case-to-case
basis.
In sum, the PNRC enjoys a special status as an important ally and auxiliary of the government in
the humanitarian field in accordance with its commitments under international law. This Court cannot
all of a sudden refuse to recognize its existence, especially since the issue of the constitutionality of the
PNRC Charter was never raised by the parties.
Thus, Respondent Richard J. Gordon’s Motion for Clarification and/or for Reconsideration and
movant-intervenor PNRC’s Motion for Partial Reconsideration of the Decision in G.R. No. 175352 dated
July 15, 2009 are GRANTED. The constitutionality of R.A. No. 95, as amended, the charter of the
Philippine National Red Cross, was not raised by the parties as an issue and should not have been
passed upon by this Court. The structure of the PNRC is sui generis¸ being neither strictly private nor
public in nature. R.A. No. 95 remains valid and constitutional in its entirety.

FRANCISCA S. BALUYOT
vs.
PAUL E. HOLGANZA and the OFFICE OF THE OMBUDSMAN (VISAYAS) represented by its Deputy
Ombudsman for the Visayas ARTURO C. MOJICA, Director VIRGINIA PALANCA-SANTIAGO, and
Graft Investigation Officer I ANNA MARIE P. MILITANTE
G.R. No. 136374.February 9, 2000

FACTS:

During a spot audit in 1977, the auditors from the Philippine National Red Cross (PNRC)
headquarters discovered a case shortage in the funds of its Bohol chapter. The chapter administrator,
petitioner Baluyot, was held accountable and thereafter, respondent Holganza as member of the board
Bohol chapter, filed a complaint with the Ofc. of the Ombudsman for malversation. Upon
recommendation of respondent Militante, an administratiave docket of dishonesty was also opened
against Baluyot. Baluyot raised the defense that the Ombudsman had no jurisdiction as he had
authority only over government owned or controlled corporations which the PNRC was not. She gives as
evidence of its private character 1) it does not receive budgetary support from the government and all
money given to it by the latter and its instrumentalities become private funds of the organization. 2)
funds for the payment of personnel’s salaries and other emoluments come from yearly fund campaigns,
private contributions and rentals from its properties. 3) it is not audited by COA. PNRC, petitioner
claims falls under the International Federation of Red Cross, Swiss-based organization.

ISSUE:

Whether or not PNRC is a government owned or controlled corporation.

RULING:

YES.

PNRC is a government owned and controlled corporation, with an original charter under RA No.
95, as amended. The test to determine whether a corporation is government owned or controlled or
private in nature is simple. Is it created by its own charter for the exercise of a public function, or by
incorporation under the general corporation law? Those with special charters are government
corporations subject to its provisions, and its employees are under the jurisdiction of the Civil Service
Commission, and are compulsory members of the GSIS.
The PNRC was not “impliedly converted to a private corporation” simply because its charter was
amended to vest in it the authority to secure loans, be exempted from payment of all duties, taxes, fees
and other charges of all kinds on all importations and purchases for its exclusive use, on donations for
its disaster relief work and other services and in its benefits and fund raising drives.” Clearly then,
public respondent has jurisdiction over the matter.

27 | P a g e
Law 321_Corporation LAW_ Case Digest

THE VETERANS FEDERATION OF THE PHILIPPINES represented by Esmeraldo R. Acorda


vs.
Hon. ANGELO T. REYES in his capacity as Secretary of National Defense; and Hon. EDGARDO E.
BATENGA in his capacity as Undersecretary for Civil Relations and Administration of the
Department of National Defense
G. R. No. 155027.February 28, 2006

FACTS:

Petitioner claims that it is not a public nor a governmental entity but a private organization, and
advances this claim to prove that the issuance of DND Department Circular No. 04 is an invalid exercise
of respondent Secretary’s control and supervision. Petitioner claims that its funds are not public funds
because no budgetary appropriations or government funds have been released to the VFP directly or
indirectly from the DBM, and because VFP funds come from membership dues and lease rentals earned
from administering government lands reserved for the VFP.

ISSUE:

Whether or not the VFPA is a private corporation.

RULING:

NO.

The functions of petitioner corporation enshrined in Section 4 of Rep. Act No. 2640 should most
certainly fall within the category of sovereign functions. The protection of the interests of war veterans is
not only meant to promote social justice, but is also intended to reward patriotism. All of the functions
in Section 4 concern the well-being of war veterans, our countrymen who risked their lives and lost their
limbs in fighting for and defending our nation. It would be injustice of catastrophic proportions to say
that it is beyond sovereignty’s power to reward the people who defended her.
Like the holding of the National Centennial Celebrations, the functions of the VFP are executive
functions, designed to implement not just the provisions of Rep. Act No. 2640, but also, and more
importantly, the Constitutional mandate for the State to provide immediate and adequate care, benefits
and other forms of assistance to war veterans and veterans of military campaigns, their surviving
spouses and orphans.
The fact that no budgetary appropriations have been released to the VFP does not prove that it is
a private corporation. The DBM indeed did not see it fit to propose budgetary appropriations to the VFP,
having itself believed that the VFP is a private corporation. If the DBM, however, is mistaken as to its
conclusion regarding the nature of VFP’s incorporation, its previous assertions will not prevent future
budgetary appropriations to the VFP. The erroneous application of the law by public officers does not
bar a subsequent correct application of the law.
Since petitioner VFP is a public corporation. As such, it can be placed under the control and
supervision of the Secretary of National Defense, who consequently has the power to conduct an
extensive management audit of Petitioner Corporation.

28 | P a g e
Law 321_Corporation LAW_ Case Digest

MANILA INTERNATIONAL AIRPORT AUTHORITY


vs.
COURT OF APPEALS, CITY OF PARAÑAQUE, CITY MAYOR OF PARAÑAQUE, SANGGUNIANG
PANGLUNGSOD NG PARAÑAQUE, CITY ASSESSOR OF PARAÑAQUE, and CITY TREASURER OF
PARAÑAQUE
G.R. No. 155650. July 20, 2006

FACTS:

MIAA operates the Ninoy Aquino International Airport (NAIA) Complex in Parañaque City under
E.O. No. 903, otherwise known as the Revised Charter of the Manila International Airport Authority
("MIAA Charter"). Executive Order No. 903 was issued on 21 July 1983 by then President Ferdinand E.
Marcos. Subsequently, E.O. Nos. 909 and 298 amended the MIAA Charter.
As operator of the international airport, MIAA administers the land, improvements and equipment
within the NAIA Complex. The MIAA Charter transferred to MIAA approximately 600 hectares of land,
including the runways and buildings and provided that no portion of the land transferred to MIAA shall
be disposed of through sale or any other mode unless specifically approved by the President of the
Philippines.
In 2001, the Parañaque city government issued notices of levy and warrants of levy on MIAA’s
Airport Lands and Buildings for its failure to pay real estate taxes plus penalties amounting to Php624.5
million for the taxable years 1992 to 2001. The city government then put the subject properties up for
sale at a public auction. Thus, MIAA brought a case for prohibition to the Court of Appeals against the
City of Parañaque.

ISSUES:

Whether or not MIAA is a public corporation owned by the State.

RULING:

YES.

MIAA is a government instrumentality vested with corporate powers to perform efficiently its
governmental functions. MIAA is like any other government instrumentality; the only difference is that
MIAA is vested with corporate powers.
When the law vests in a government instrumentality corporate powers, the instrumentality does
not become a corporation. Unless the government instrumentality is organized as a stock or non-stock
corporation, it remains a government instrumentality exercising not only governmental but also
corporate powers. Thus, MIAA exercises the governmental powers of eminent domain, police authority
and the levying of fees and charges. At the same time, MIAA exercises "all the powers of a corporation
under the Corporation Law, insofar as these powers are not inconsistent with the provisions of this
Executive Order."
Many government instrumentalities are vested with corporate powers but they do not become
stock or non-stock corporations, which is a necessary condition before an agency or instrumentality is
deemed a government-owned or controlled corporation. Examples are the Mactan International Airport
Authority, the Philippine Ports Authority, the University of the Philippines and Bangko Sentral ng
Pilipinas. All these government instrumentalities exercise corporate powers but they are not organized
as stock or non-stock corporations as required by Section 2(13) of the Introductory Provisions of the
Administrative Code. These government instrumentalities are sometimes loosely called government
corporate entities. However, they are not government-owned or controlled corporations in the strict
sense as understood under the Administrative Code, which is the law defining the legal relationship and
status of government entities.

 Promotion

MARC II MARKETING, INC. and LUCILA V. JOSON

29 | P a g e
Law 321_Corporation LAW_ Case Digest

vs.
ALFREDO M. JOSON
G.R. No. 171993.December 12, 2011

FACTS:

Marc II Marketing, Inc. and Lucila Joson is assailing the decision of the CA for reversing and
settling aside the Resolution of the National Labor Relations Commission. Marc II Marketing, Inc. is a
corporation duly organized and existing under and by virtue of the laws of the Philippines. It is primarily
engaged in buying, marketing, selling and distributing in retail or wholesale for export or import
household appliances and products and other items. Petitioner Lucila is the President and majority
stockholder of the corporation. Before Marc II Marketing, Inc. was officially incorporated, Alfredo has
already been engaged by Lucila, in her capacity as President, to work as General Manager of the
corporation and it was formalized through the execution of a Management Contract dated in 1994 under
Marc Marketing, Inc., as Marc II Marketing, Inc. was yet to be incorporated. For occupying the said
position, respondent was among the corporation’s corporate officers by the express provision of Section
1, Article IV of its by-laws.
Alfredo was appointed as one of its officers with the designation or title of General Manager to
function as a managing director with other duties and responsibilities that the Board may provide and
authorized. However, in 1997, Marc II Marketing Inc. decided to stop and cease its operation as
evidenced by an Affidavit of Non-Operation due to poor sales collection aggravated by the inefficient
management of its affairs. Alfredo was informed of the cessation of its business operations and the
termination of his services as General Manager. He filed action for reinstatement and money claim
against petitioners.

ISSUE:

Whether or not Marc II Marketing Inc.’s Board of Directors could create a position for corporate
officers through an enabling clause found in its corporate by-laws.

RULING:

YES.

The Court held that in the context of PD 902-A, corporate officers are those officers of a
corporation who are given that character either by the Corporation Code or by the corporation’s by-laws.
Section 25 of the Corporation Code specifically enumerated who are these corporate officers, namely:
president, secretary, treasurer and such other officers as may be provided for in the by-laws. A careful
examination of Marc II Marketing Inc.’s by-laws, particularly paragraph 1, Section 1of Article IV
explicitly revealed that its corporate officers are composed only of chairman, president, one/more vice
president, treasurer and secretary. The position of general manager was not among those enumerated.
Meanwhile, paragraph 2, Section 1 of Article IV of the corporation’s by-laws empowered its Board of
Directors to appoint such officers as it may determine necessary or proper, making this an enabling
provision for approving a resolution to make the position of general manager a corporate officer. All of
these acts were done without first amending its by-laws so as to include the General Manager in its
roster of corporate officers. Though the Board of Directors may create appointive positions other than
the positions of corporate officers, the persons occupying such positions cannot be viewed as corporate
officers under Section 25 of the Corporation Code. The said provision of the Corporation Code
safeguards the constitutionally enshrined right of every employee to security of tenure and prevents the
creation of a corporate officer position by a simple inclusion in the corporate by-laws of an enabling
clause empowering the Board of Directors.
CAGAYAN FISHING DEVELOPMENT CO., INC.
vs.
TEODORO SANDIKO
G.R. No. L-43350. December 23, 1937

FACTS:

Manuel Tabora is the registered owner of four parcels of land. To guarantee the payment of a
loan in the sum of P8,000, Manuel Tabora executed in favor of the Philippine National Bank a first
mortgage on the four parcels of land above-mentioned. A second mortgage in favor of the same bank was
executed by Tabora over the same lands to guarantee the payment of another loan amounting to P7,000.
A third mortgage on the same lands was executed in favor of Severina Buzon to whom Tabora was

30 | P a g e
Law 321_Corporation LAW_ Case Digest

indebted in the sum of P2,9000. These mortgages were registered and annotations thereof appear at the
back of transfer certificate of title No. 217.
The board of directors of plaintiff adopted a resolution authorizing its president, Jose Ventura, to
sell the four parcels of lands in question to Teodoro Sandiko for P42,000. The defendant having failed to
pay the sum stated in the promissory note, plaintiff, brought this action in the Court of First Instance of
Manila praying that judgment be rendered against the defendant for the sum of P25,300, with interest
at legal rate from the date of the filing of the complaint, and the costs of the suits.

ISSUE:

Whether or not the transfers were valid.

RULING:

NO.

The contract here was entered into not between Manuel Tabora and a non-existent corporation
but between the Manuel Tabora as owner of the four parcels of lands on the one hand and the same
Manuel Tabora, his wife and others, as mere promoters of a corporations on the other hand. For reasons
that are self-evident, these promoters could not have acted as agent for a projected corporation since
that which no legal existence could have no agent. A corporation, until organized, has no life and
therefore no faculties. It is, as it were, a child in ventre sa mere. This is not saying that under no
circumstances may the acts of promoters of a corporation be ratified by the corporation if and when
subsequently organized. There are, of course, but under the peculiar facts and circumstances of the
present case we decline to extend the doctrine of ratification which would result in the commission of
injustice or fraud to the candid and unwary.
If the plaintiff corporation could not and did not acquire the four parcels of land here involved, it
follows that it did not possess any resultant right to dispose of them by sale to the defendant, Teodoro
Sandiko.
Some of the members of this court are also of the opinion that the transfer from Manuel Tabora to
the Cagayan Fishing Development Company, Inc., which transfer is evidenced by Exhibit A, was subject
to a condition precedent (condicion suspensiva), namely, the payment of the mortgage debt of said
Tabora to the Philippine National Bank, and that this condition not having been complied with by the
Cagayan Fishing Development Company, Inc., the transfer was ineffective. However, having arrived at
the conclusion that the transfer by Manuel Tabora to the Cagayan Fishing Development Company, Inc.
was null because at the time it was affected the corporation was non-existent, we deem it unnecessary
to discuss this point.

FERMIN Z. CARAM, JR. and ROSA O. DE CARAM


vs.
THE HONORABLE COURT OF APPEALS and ALBERTO V. ARELLANO
G.R. No. L-48627. June 30, 1987

FACTS:

The petitioners claim that this order has no support in fact and law because they had no contract
whatsoever with the private respondent regarding the above-mentioned services. Their position is that as
mere subsequent investors in the corporation that was later created, they should not be held solidarily
liable with the Filipinas Orient Airways, a separate juridical entity, and with Barretto and Garcia, their
co-defendants in the lower court, ** who were the ones who requested the said services from the private
respondent.

ISSUE:

Whether or not the petitioners should be held liable.

31 | P a g e
Law 321_Corporation LAW_ Case Digest

RULING:

NO.

The petitioners were not involved in the initial stages of the organization of the airline. They were
merely among the financiers whose interest was to be invited and who were in fact persuaded, on the
strength of the project study, to invest in the proposed airline.
There was no showing that the Airline was a fictitious corporation and did not have a separate
juridical personality to justify making the petitioners, as principal stockholders thereof, responsible for
its obligations. As a bona fide corporation, the Airline should alone be liable for its corporate acts as
duly authorized by its officers and directors. Granting that the petitioners benefited from the services
rendered, such is no justification to hold them personally liable therefor. Otherwise, all the other
stockholders of the corporation, including those who came in late, and regardless of the amount of their
shareholdings, would be equally and personally liable also with the petitioner for the claims of the
private respondent.
Petitioners cannot be held personally liable for the compensation claimed by the private
respondent for the services performed by him in the organization of the corporation. To repeat, the
petitioners did not contract such services. It was only the results of such services that Barretto and
Garcia presented to them and which persuaded them to invest in the proposed airline. The most that
can be said is that they benefited from such services, but that surely is no justification to hold them
personally liable therefor.
A promoter could not have acted as agent for a corporation that had no legal existence. A
corporation, until organized, has no life therefore no faculties. The corporation had no juridical
personality to enter into a contract.

PIONEER INSURANCE & SURETY CORPORATION


vs.
THE HON. COURT OF APPEALS, BORDER MACHINERY & HEAVY EQUIPMENT, INC.,
(BORMAHECO), CONSTANCIO M. MAGLANA and JACOB S. LIM
G.R. No. 84197. July 28, 1989

FACTS:

In 1965, Jacob S. Lim, owner-operator of Southern Airlines (SAL), a single proprietorship entered
into a sales contract with regarding Japan Domestic Airlines (JDA) regarding 2 DC-#A type aircrafts, 1
set of necessary spare parts where a Total of $ 190,000 in instalments are to be paid. Pioneer
Insurance and Surety Corp. as surety executed its surety bond in favor of JDA on behalf of its principal
Lim. Border Machinery and Heavy Equipment Co, Inc. of Francisco and Modesto Cervantes and
Constancio Maglana contributed funds for the transaction based on the misrepresentation of Lim that
they will form a new corporation to expand his business.
Lim as owner operator for SAL executed in favor of Pioneer a deed of chattel mortgage as security.
A restructuring of obligation to change the maturity was done twice without the knowledge of other
defendants made the surety of JDA prescribed so not entitled to reimbursement. Upon default on the
2/8 payments, Pioneer paid for him and filed a petition for the foreclosure of chattel mortgage as
security. CA affirmed Trial of Merits: Only Lim is liable to pay

ISSUE:

Whether or not there is failure of respondents to incorporate leading to a de facto partnership.

32 | P a g e
Law 321_Corporation LAW_ Case Digest

RULING:

NO.

Partnership inter se does not necessarily exist, for ordinarily CANNOT be made to assume the
relation of partners as between themselves, when their purpose is that no partnership shall exists. It
should be implied only when necessary to do justice between the parties (i.e. only pretend to make
others liable). Lim never intended to form a corporation.

RIZAL LIGHT & ICE CO., INC.


vs.
THE MUNICIPALITY OF MORONG, RIZAL and THE PUBLIC SERVICE COMMISSION
G.R. No. L-20993.September 28, 1968

FACTS:

The bulk of petitioner's arguments assailing the personality of Morong Electric dwells on the
proposition that since a franchise is a contract, 23 at least two competent parties are necessary to the
execution thereof, and parties are not competent except when they are in being. Hence, it is contended
that until a corporation has come into being, in this jurisdiction, by the issuance of a certificate of
incorporation by the Securities and Exchange Commission (SEC) it cannot enter into any contract as a
corporation.
The certificate of incorporation of the Morong Electric was issued by the SEC on October 17,
1962, so only from that date, not before, did it acquire juridical personality and legal existence.
Petitioner concludes that the franchise granted to Morong Electric on May 6, 1962 when it was not yet
in esse is null and void and cannot be the subject of the Commission's consideration. On the other
hand, Morong Electric argues, and to which argument the Commission agrees, that it was a de facto
corporation at the time the franchise was granted and, as such, it was not incapacitated to enter into
any contract or to apply for and accept a franchise. Not having been incapacitated, Morong Electric
maintains that the franchise granted to it is valid and the approval or disapproval thereof can be
properly determined by the Commission.

ISSUE:

Whether the lack or corporate existence on the part of Morong rendered the franchise valid.

RULING:

YES.

33 | P a g e
Law 321_Corporation LAW_ Case Digest

The incorporation of (Morong) and its acceptance of the franchise as shown by this action in
prosecuting the application filed with the Commission for approval of said franchise, not only perfected a
contract between the municipality and Morong but also cured the deficiency pointed out by the petition.
The fact that Morong did not have a corporate existence on the day the franchise was granted does not
render the franchise invalid, as Morong later obtained its certificate of incorporation and accepted the
franchise.
The two decisions of the Public Service Commission, appealed from, should be, as they are hereby
affirmed, with costs in the two cases against petitioner Rizal Light & Ice Co., Inc.

 Articles of Incorporation

JESUS V. LANUZA, MAGADYA REYES, BAYANI REYES and ARIEL REYES


vs.
COURT OF APPEALS, SECURITIES AND EXCHANGE COMMISSION, DOLORES ONRUBIA, ELENITA
NOLASCO, JUAN O. NOLASCO III, ESTATE OF FAUSTINA M. ONRUBIA, PHILIPPINE MERCHANT
MARINE SCHOOL, INC.
G.R. No. 131394.March 28, 2005

FACTS:

Philippine Merchant Marine School, Inc. (PMMSI) had seven hundred founders’ shares and
seventy-six common shares as its initial capital stock subscription reflected in the articles of
incorporation. However, private respondents and their predecessors who were in control of PMMSI
registered the company’s stock and transfer book for the first time in 1978, recording thirty-three (33)
common shares as the only issued and outstanding shares of PMMSI.
Sometime in 1979, a special stockholders’ meeting was called and held on the basis of what was
considered as a quorum of twenty-seven common shares, representing more than two-thirds of the
common shares issued and outstanding. In 1982, the heirs of one of the original incorporators, Juan
Acayan, filed a petition with the SEC for the registration of their property rights over one hundred (120)
founders’ shares and twelve (12) common shares owned by their father. The SEC held that the heirs
were entitled to the claimed shares and called for a special stockholders’ meeting to elect a new set of
officers. As a result, the shares of Acayan were recorded in the stock and transfer book.
A special stockholders’ meeting was held to elect a new set of directors. Private respondents
thereafter filed a petition with the SEC questioning the validity of the 06 May 1992 stockholders’
meeting, alleging that the quorum for the said meeting should not be based on the 165 issued and
outstanding shares as per the stock and transfer book, but on the initial subscribed capital stock of
seven hundred seventy-six (776) shares, as reflected in the 1952 Articles of Incorporation.

ISSUE:

Whether or not the basis of quorum for a stockholders’ meeting is the outstanding capital stock
as indicated in the articles of incorporation.

RULING:

YES.

The stock and transfer book of PMMSI cannot be used as the sole basis for determining the
quorum as it does not reflect the totality of shares which have been subscribed, more so when the

34 | P a g e
Law 321_Corporation LAW_ Case Digest

articles of incorporation show a significantly larger amount of shares issued and outstanding as
compared to that listed in the stock and transfer book. A stock and transfer book is one which records
the names and addresses of all stockholders arranged alphabetically, the instalments paid and unpaid
on all stock for which subscription has been made, and the date of payment thereof; a statement of
every alienation, sale or transfer of stock made, the date thereof and by and to whom made; and such
other entries as may be prescribed by law.
To base the computation of quorum solely on the deficient stock and transfer book, and
completely disregarding the issued and outstanding shares as indicated in the articles of incorporation
would work injustice to the owners and/or successors in interest of the said shares.
It is to be explained, that if at the onset of incorporation a corporation has 771 shares
subscribed, the Stock and Transfer Book should likewise reflect 771 shares. Any sale, disposition or
even reacquisition of the company of its own shares, in which it becomes treasury shares, would not
affect the total number of shares in the Stock and Transfer Book. All that will change are the entries as
to the owners of the shares but not as to the amount of shares already subscribed.
 Corporate Name

FRANCISCO M. ALONSO, substituted by his heirs


vs.
CEBU COUNTRY CLUB, INC.
G.R. No. 130876.January 31, 2002

FACTS:

Petitioner died pendente lite and substituted by his legal heirs, a lawyer by profession, the only
son and sole heir of the late Tomas N. Alonso and Asuncion Medalle, who died on June 16, 1962 and
August 18, 1963, respectively. Cebu Country Club, Inc. is a non-stock, non-profit corporation duly
organized and existing under Philippine Laws the purpose of which is to cater to the recreation and
leisure of its members.
Sometime in 1992, petitioner discovered documents and records of Friar Lands Sale Certificate
Register/Instalment Record Certificate No. 734, Sales Certificate No. 734 and Assignment of Sales
Certificate showing that his father acquired Lot No. 727 of the Banilad Friar Lands Estate from the
Government of the Philippine Islands in or about the year 1911 in accordance with the Friar Lands Act
(Act No. 1120). The documents show that one Leoncio Alburo, the original vendee of Lot No. 727,
assigned his sales certificate to petitioner’s father on December 18, 1911, who completed the required
instalment payments thereon under Act No. 1120 and was consequently issued Patent No. 14353 on
March 24, 1926. On March 27, 1926, the Director of Lands, acting for and in behalf of the government,
executed a final deed of sale in favor of petitioner’s father Tomas N. Alonso. It appears, however, that the
deed was not registered with the Register of Deeds because of lack of technical requirements, among
them the approval of the deed of sale by the Secretary of Agriculture and Natural Resources, as required
by law.

ISSUE:

Whether or not the Court of Appeals erred in sustaining respondent’s claim of ownership over Lot
No. 727.

RULING:

YES.

Under Act No. 1120, which governs the administration and disposition of friar lands, the
purchase by an actual and bona fide settler or occupant of any portion of friar land shall be "agreed
upon between the purchaser and the Director of Lands, subject to the approval of the Secretary of
Agriculture and Natural Resources (mutatis mutandis)."
The instruments do not bear the signature of the Director of Lands and the Secretary of the
Interior. They also do not bear the approval of the Secretary of Agriculture and Natural Resources. The
approval by the Secretary of Agriculture and Commerce of the sale of friar lands is indispensable for its
validity, hence, the absence of such approval made the sale null and void ab initio. Necessarily, there
can be no valid titles issued on the basis of such sale or assignment. Consequently, petitioner
Francisco’s father did not have any registered title to the land in question. Having none, he could not
transmit anything to his sole heir, petitioner Francisco Alonso or the latter’s heirs.

35 | P a g e
Law 321_Corporation LAW_ Case Digest

Hence, the Court ruled that neither Tomas N. Alonso nor his son Francisco M. Alonso or the
latter’s heirs are the lawful owners of Lot No. 727 in dispute. Neither has the respondent Cebu Country
Club, Inc. been able to establish a clear title over the contested estate. The reconstitution of a title is
simply the re-issuance of a lost duplicate certificate of title in its original form and condition. It does not
determine or resolve the ownership of the land covered by the lost or destroyed title. A reconstituted
title, like the original certificate of title, by itself does not vest ownership of the land or estate covered
thereby.
INDUSTRIAL REFRACTORIES CORPORATION OF THE PHILIPPINES,
vs.
COURT OF APPEALS, SECURITIES AND EXCHANGE COMMISSION and REFRACTORIES
CORPORATION OF THE PHILIPPINES
G.R. No. 122174.October 3, 2002

FACTS:

Refractories Corporation of the Philippines (RCP) is a corporation duly organized for the purpose
of engaging in the business of manufacturing, producing, selling, exporting and otherwise dealing in any
and all refractory bricks, its by-products and derivatives. On June 22, 1977, it registered its corporate
and business name with the Bureau of Domestic Trade. IRCP was originally under the name "Synclaire
Manufacturing Corporation". It amended its Articles of Incorporation on August 23, 1985 to change its
corporate name to "Industrial Refractories Corp. of the Philippines". It is engaged in the business of
manufacturing all kinds of ceramics and other products, except paints and zincs. Both companies are
the only local suppliers of monolithic gunning mix.
Discovering that petitioner was using such corporate name, respondent RCP filed on April 14,
1988 with the Securities and Exchange Commission (SEC) a petition to compel petitioner to change its
corporate name on the ground that its corporate name is confusingly similar with that of petitioner’s
such that the public may be confused or deceived into believing that they are one and the same
corporation.

ISSUE:

Whether or not the corporate names are confusingly similar.

RULING:

YES.

"Refractories are structural materials used at high temperatures to industrial furnaces. They are
supplied mainly in the form of brick of standard sizes and of special shapes. Refractories also include
refractory cements, bonding mortars, plastic firebrick, castables, ramming mixtures, and other bulk
materials such as dead-burned grain magneside, chrome or ground ganister and special clay."
As regards the first requisite, it has been held that the right to the exclusive use of a corporate
name with freedom from infringement by similarity is determined by priority of adoption. In this case,
respondent RCP was incorporated on October 13, 1976 and since then has been using the corporate
name "Refractories Corp. of the Philippines". Meanwhile, petitioner was incorporated on August 23,
1979 originally under the name "Synclaire Manufacturing Corporation". It only started using the name
"Industrial Refractories Corp. of the Philippines" when it amended its Articles of Incorporation on August
23, 1985, or nine (9) years after respondent RCP started using its name. Thus, being the prior
registrant, respondent RCP has acquired the right to use the word "Refractories" as part of its corporate
name.
Anent the second requisite, in determining the existence of confusing similarity in corporate
names, the test is whether the similarity is such as to mislead a person using ordinary care and
discrimination and the Court must look to the record as well as the names themselves. Petitioner’s
corporate name is "Industrial Refractories Corp. of the Phils.", while respondent’s is "Refractories Corp.
of the Phils." Obviously, both names contain the identical words "Refractories", "Corporation" and
"Philippines". The only word that distinguishes petitioner from respondent RCP is the word "Industrial"
which merely identifies a corporation’s general field of activities or operations. We need not linger on
these two corporate names to conclude that they are patently similar that even with reasonable care and
observation, confusion might arise. It must be noted that both cater to the same clientele, i.e.¸ the steel
industry. In fact, the SEC found that there were instances when different steel companies were actually
confused between the two, especially since they also have similar product packaging.

ANG MGA KAANIB SA IGLESIA NG DIOS KAY KRISTO HESUS, HSK SA BANSANG PILIPINAS INC.

36 | P a g e
Law 321_Corporation LAW_ Case Digest

vs.
IGLESIA NG DIOS KAY CRISTO JESUS, HALIGI AT SUHAY NG KATOTOHANAN
GR 137592, 12 DECEMBER 2001

FACTS:

The Iglesia ng Dios Kay Cristo Jesus, Haligi at Suhay ng Katotohanan (IDCJ-HSK; Church of God
in Christ Jesus, the Pillar and Ground of Truth), is a non-stock religious society or corporation
registered in 1936. Sometime in 1976, one Eliseo Soriano and several other members of said corporation
disassociated themselves from the latter and succeeded in registering on 30 March 1977 a new non-
stock religious society or corporation, named Iglesia ng Dios Kay Kristo Hesus, Haligi at Saligan ng
Katotohanan (IDKJ-HSK). On 16 July 1979, IDCJ-HSK filed with the SEC a petition to compel IDKJ-
HSK to change its corporate name.
The SEC rendered judgment in favor of IDCJ-HSK, ordering IDKJ-HSK to change its corporate
name to another name that is not similar or identical to any name already used by a corporation,
partnership or association registered with the Commission.

ISSUE:

Whether the corporate names of AK[IDKH-HSK]BP and IDCH-HSK are confusingly similar.

RULING:

YES.

The additional words "Ang Mga Kaanib " and "Sa Bansang Pilipinas, Inc." in AK[IDKH-HSK]BP's
name are merely descriptive of and also referring to the members, or kaanib, of IDCH-HSK who are
likewise residing in the Philippines. These words can hardly serve as an effective differentiating medium
necessary to avoid confusion or difficulty in distinguishing AK[IDKH-HSK]BP from IDCH-HSK. This is
especially so, since both AK[IDKH-HSK]BP and IDCH-HSK are using the same acronym — H.S.K.; not to
mention the fact that both are espousing religious beliefs and operating in the same place.
Parenthetically, it is well to mention that the acronym H.S.K. used by AK[IDKH-HSK]BP stands for
"Haligi at Saligan ng Katotohanan." Then, too, the records reveal that in holding out their corporate
name to the public, AK[IDKH-HSK]BP highlights the dominant words "IGLESIA NG DIOS KAY KRISTO
HESUS, HALIGI AT SALIGAN NG KATOTOHANAN," which is strikingly similar to IDCH-HSK's corporate
name, thus making it even more evident that the additional words "Ang Mga Kaanib" and "Sa Bansang
Pilipinas, Inc.", are merely descriptive of and pertaining to the members of IDCH-HSK. Significantly, the
only difference between the corporate names of AK[IDKH-HSK]BP and IDCH-HSK are the words
SALIGAN and SUHAY. These words are synonymous — both mean ground, foundation or support.
Hence, the Court ruled that the corporate names Universal are indisputably so similar that even under
the test of "reasonable care and observation" confusion may arise.
The wholesale appropriation by AK[IDKH-HSK]BP of IDCH-HSK's corporate name cannot find
justification under the generic word rule. A contrary ruling would encourage other corporations to adopt
verbatim and register an existing and protected corporate name, to the detriment of the public. The fact
that there are other non-stock religious societies or corporations using the names Church of the Living
God, Inc., Church of God Jesus Christ the Son of God the Head, Church of God in Christ & By the Holy
Spirit, and other similar names, is of no consequence. It does not authorize the use by AK[IDKH-HSK]BP
of the essential and distinguishing feature of IDCH-HSK's registered and protected corporate name.

UNIVERSAL MILLS CORPORATION


vs.
UNIVERSAL TEXTILE MILLS, INC.
G.R. No. L-28351, July 28, 1977

FACTS:

Universal Textile Mills, Inc. was organized on December 29, 1953, as a textile manufacturing
firm for which it was issued a certificate of registration on January 8, 1954. The Universal Mills
Corporation, on the other hand, was registered with the Commission on October 27, 1954, under its
original name, Universal Hosiery Mills Corporation, having as its primary purposes the "manufacture
and production of hosieries and wearing apparel of all kinds." On May 24, 1963, it filed an amendment

37 | P a g e
Law 321_Corporation LAW_ Case Digest

to its articles of incorporation changing its name to Universal Mills Corporation, its present name, for
which it was issued the certificate of approval on June 10, 1963.
The immediate cause of this complaint was the occurrence of a fire which gutted petitioner’s
spinning mills in Pasig, Rizal. Universal Textile Mills, Inc. alleged that as a result of this fire and
because of the similarity of petitioner's name to that of the former, the news items appearing in the
various metropolitan newspapers carrying reports on the fire created uncertainty and confusion among
its bankers, friends, stockholders and customers prompting respondent to make announcements,
clarifying the real Identity of the corporation whose property was burned.
The Commission then issued an order enjoining Universal Mills Corporation from using its
present corporate name because it is confusingly and deceptively similar with Universal Textile Mills,
Inc.

ISSUE:

Whether or not the order of the SEC is proper.

RULING:

YES.

The corporate names in question are not Identical, but they are indisputably so similar that even
under the test of "reasonable care and observation as the public generally are capable of using and may
be expected to exercise" invoked by appellant. The Supreme Court ruled that confusion will usually
arise, considering that under the second amendment of its articles of incorporation on August 14, 1964,
appellant included among its primary purposes the "manufacturing, dyeing, finishing and selling of
fabrics of all kinds" in which respondent had been engaged for more than a decade ahead of petitioner.
And since respondent is not claiming damages in this proceeding, it is, of course, immaterial
whether or not appellant has acted in good faith, but the SC cannot perceive why of all names, petitioner
had to choose a name already being used by another firm engaged in practically the same business for
more than a decade enjoying well-earned patronage and goodwill, when there are so many other
appropriate names it could possibly adopt without arousing any suspicion as to its motive and, more
importantly, any degree of confusion in the mind of the public which could mislead even its own
customers, existing or prospective.

LYCEUM OF THE PHILIPPINES, INC.


vs.
COURT OF APPEALS, ET AL.
G.R. No. 101897 March 5, 1993

FACTS:

Lyceum of the Philippines is an educational institution duly registered with the Securities and
Exchange Commission. Petitioner instituted proceedings before the SEC to compel the private
respondents, which are also educational institutions, to delete the word "Lyceum" from their corporate
names and permanently to enjoin them from using "Lyceum" as part of their respective names. The SEC
hearing officer rendered a decision sustaining petitioner's claim to an exclusive right to use the word
"Lyceum." The hearing officer relied upon the SEC ruling in the Lyceum of Baguio, Inc. case and held
that the word "Lyceum" was capable of appropriation and that petitioner had acquired an enforceable
exclusive right to the use of that word.
On appeal, however, by private respondents the SEC En Banc did not consider the word
"Lyceum" to have become so identified with petitioner as to render use thereof by other institutions as
productive of confusion about the identity of the schools concerned in the mind of the general public.
Unlike its hearing officer, the SEC En Banc held that the attaching of geographical names to the word

38 | P a g e
Law 321_Corporation LAW_ Case Digest

"Lyceum" served sufficiently to distinguish the schools from one another, especially in view of the fact
that the campuses of petitioner and those of the private respondents were physically quite remote from
each other.

ISSUE:

Whether or not the word Lyceum has not acquired a secondary meaning.

RULING:

NO.

The Articles of Incorporation of a corporation must, among other things, set out the name of the
corporation. The policy underlying the prohibition in Section 18 against the registration of a corporate
name which is "identical or deceptively or confusingly similar" to that of any existing corporation or
which is "patently deceptive" or "patently confusing" or "contrary to existing laws," is the avoidance of
fraud upon the public which would have occasion to deal with the entity concerned, the evasion of legal
obligations and duties, and the reduction of difficulties of administration and supervision over
corporations.
"Under the doctrine of secondary meaning, a word or phrase originally incapable of exclusive
appropriation with reference to an article in the market, because geographical or otherwise descriptive
might nevertheless have been used so long and so exclusively by one producer with reference to this
article that, in that trade and to that group of the purchasing public, the word or phrase has come to
mean that the article was his produce. The appellant failed to satisfy the requisites. No evidence was
ever presented in the hearing before the Commission which sufficiently proved that the word 'Lyceum'
has indeed acquired secondary meaning in favor of the appellant.

INDIANA AEROSPACE UNIVERSITY


vs.
COMMISSION ON HIGHER EDUCATION
G.R. NO. 139371, 04 APRIL 2001

FACTS:

In 1996, Dr. Reynaldo B. Vera, Chairman, Technical Panel for Engineering, Architecture, and
Maritime Education (TPRAM) of [CHED], received a letter dated October 18, 1998 (Annex 'C') from
Douglas R. Macias, Chairman, Board of Aeronautical Engineering, Professional Regulatory Commission
(PRC) and Chairman, Technical Committee for Aeronautical Engineering (TPRAME) inquiring whether
petitioner had already acquired University status in view of the latter's advertisement in the Manila
Bulletin.
In a letter dated October 24, 1996, Dr. Vera formally referred the aforesaid letter to Chairman
Alcala with a request that the concerned Regional Office of [CHED] be directed to conduct appropriate
investigation on the alleged misrepresentation by petitioner. Thereafter, CHED referred the matter to its
Regional Director in Cebu City, requesting said office to conduct an investigation and submit its report.
The Report stated that there was a violation [committed by] his institution when it used the term
university unless the school had complied [with] the basic requirement of being a university as
prescribed in CHED Memorandum Order No. 48, s. 1996.'

ISSUE:

Whether or not the Petitioner can use the word “University”.

RULING:

39 | P a g e
Law 321_Corporation LAW_ Case Digest

YES.

The Court found that there was no grave abuse of discretion in the RTC's denial of the Motion to
Dismiss, as contained in the August 14, 1998 Order. The CA erred in ruling otherwise. The trial court
stated in its Decision that petitioner was an educational institution, originally registered with the
Securities and Exchange Commission as the "Indiana School of Aeronautics, Inc." That name was
subsequently changed to "Indiana Aerospace University" after the Department of Education, Culture and
Sports had interposed no objection to such change.
Respondent issued a formal Cease and Desist Order directing petitioner to stop using the word
"university" in its corporate name. The former also published an announcement in the March 21, 1998
issue of Freeman, a local newspaper in Cebu City, that there was no institution of learning by that
name. The counsel of respondent was quoted as saying in the March 28, 1998 issue of the newspaper
Today that petitioner had been ordered closed by the respondent for illegal advertisement, fraud and
misrepresentation of itself as a university. Such acts, according to the RTC undermined the public's
confidence in petitioner as an educational institution. This was a clear statement of a sufficient cause of
action.
When a motion to dismiss is grounded on the failure to state a cause of action, a ruling thereon
should be based only on the facts alleged in the complaint. The court must pass upon this issue based
solely on such allegations, assuming them to be true. For it to do otherwise would be a procedural error
and a denial of plaintiff's right to due process.

PHILIPS EXPORT B.V., PHILIPS ELECTRICAL LAMPS,INC. and PHILIPS INDUSTRIAL


DEVELOPMENT, INC.
vs.
COURT OF APPEALS, SECURITIES & EXCHANGE COMMISSION and STANDARD PHILIPS
CORPORATION
G.R. No. 96161. February 21, 1992

FACTS:

Petitioner is a foreign corporation organized under the laws of Netherlands although not engaged
in any business here in the Philippines, is the registered owner of the trademarks Philips and Philips
Shield Emblem. Petitioners Philips Electrical Lamps, Inc, and Philips Industrial Development Inc are the
authorized users of the trademark of petitioner Philips BV. All petitioner corporations belong to the
Philips Group of Companies.
Private respondent Standard Philips Corporation was issued a certificate of registration by the
respondent Securities and Exchange Commission. Petitioner filed a letter complaint with the SEC
asking for the cancellation of the word “Philips” from private respondent’s corporate name in view of its
prior registration with the Bureau of Patent alleging that private respondent’s use of the word Philips
amounts to an infringement and clear violation of petitioner’s exclusive right to use the same
considering that both parties are engaged in the same business.

ISSUE:

Whether or not Standard Philips’ use of the word PHILIPS amounts to an infringement and clear
violation of Petitioner's exclusive right to use the same considering that both parties engage in the same
business.

RULING:

YES.

The requisite no less exists in this case. In determining the existence of confusing similarity in
corporate names, the test is whether the similarity is such as to mislead a person using ordinary care
and discrimination. In so doing, the Court must look to the record as well as the names themselves.

40 | P a g e
Law 321_Corporation LAW_ Case Digest

While the corporate names of Petitioners and Private Respondent are not identical, a reading of
Petitioner's corporate names, to wit: PHILIPS EXPORT B.V., PHILIPS ELECTRICAL LAMPS, INC. and
PHILIPS INDUSTRIAL DEVELOPMENT, INC., inevitably leads one to conclude that "PHILIPS" is, indeed,
the dominant word in that all the companies affiliated or associated with the principal corporation,
PEBV, are known in the Philippines and abroad as the PHILIPS Group of Companies.
It is settled that proof of actual confusion need not be shown. It suffices that confusion is
probably or likely to occur. Under the Guidelines in the Approval of Corporate and Partnership Names
formulated by the SEC, the proposed name "should not be similar to one already used by another
corporation or partnership. If the proposed name contains a word already used as part of the firm name
or style of a registered company, the proposed name must contain two other words different from the
company already registered". Private Respondents' name, however, contains only a single word, that is,
"STANDARD", different from that of Petitioners inasmuch as the inclusion of the term "Corporation" or
"Corp." merely serves the purpose of distinguishing the corporation from partnerships and other
business organizations.
It is obvious that private respondent’s choice of Philips as part of its corporate name tends to
show said respondent’s intention to ride on the popularity and established goodwill of the said
petitioner’s business throughout the world. The subsequent appropriator of the name or one-confusingly
similar thereto usually seeks an unfair advantage, a free ride on another’s goodwill. Besides, there is
showing that private respondent not only manufactured and sold ballasts for fluorescent lamps with
their corporate name printed thereon but also advertised the same as Standard Philips.
 Primary Purpose

ALICIA E. GALA, GUIA G. DOMINGO and RITA G. BENSON


vs.
ELLICE AGRO-INDUSTRIAL CORPORATION, MARGO MANAGEMENT AND DEVELOPMENT
CORPORATION, RAUL E. GALA, VITALIANO N. AGUIRRE II, ADNAN V. ALONTO, ELIAS N.
CRESENCIO, MOISES S. MANIEGO, RODOLFO B. REYNO, RENATO S. GONZALES, VICENTE C.
NOLAN, NESTOR N. BATICULON
G.R. No. 156819. December 11, 2003

FACTS:

On March 28, 1979, the Ellice Agro-Industrial Corporation was formed and organized. The total
subscribed capital stock of the corporation was P3.5 Million with 35,000 shares. Additional shares were
acquired and subscribed from said corporation. Subsequently, on September 16, 1982, the Margo
Management and Development Corporation (Margo) was incorporated. The total subscribed capital
stock of Margo was 20,000 shares at P200, 000.00. Several transfers of shares of Ellice to Margo were
made by the stockholders and some payments of subscription were made by transferring parcels of land
by the Gala Spouses.
In essence, petitioners want this Court to disregard the separate juridical personalities of Ellice
and Margo for the purpose of treating all property purportedly owned by said corporations as property
solely owned by the Gala spouses. The petitioners’ contention in support of this theory is that the
purposes for which Ellice and Margo were organized should be declared as illegal and contrary to public
policy. They claim that the respondents never pursued exemption from land reform coverage in good
faith and instead merely used the corporations as tools to circumvent land reform laws and to avoid
estate taxes. Specifically, they point out that respondents have not shown that the transfers of the land
in favor of Ellice were executed in compliance with the requirements of Section 13 of R.A. 3844.
Furthermore, they alleged that respondent corporations were run without any of the conventional
corporate formalities.

ISSUE:

Whether or not the purpose of the creation of the two corporations is illegal and against public
policy.

RULING:

NO.

Impugning the legality of the purposes for which Ellice and Margo were organized, amount to
collateral attacks which are prohibited in this jurisdiction. The best proof of the purpose of a corporation
is its articles of incorporation and by-laws. The articles of incorporation must state the primary and
secondary purposes of the corporation, while the by-laws outline the administrative organization of the

41 | P a g e
Law 321_Corporation LAW_ Case Digest

corporation, which, in turn, is supposed to insure or facilitate the accomplishment of said purpose. A
perusal of the Articles of Incorporation of Ellice and Margo shows no sign of the allegedly illegal
purposes that petitioners are complaining of. If a corporation’s purpose, as stated in the Articles of
Incorporation, is lawful, then the SEC has no authority to inquire whether the corporation has purposes
other than those stated, and mandamus will lie to compel it to issue the certificate of incorporation.
With regard to their claim that Ellice and Margo were meant to be used as mere tools for the
avoidance of estate taxes, suffice it say that the legal right of a taxpayer to reduce the amount of what
otherwise could be his taxes or altogether avoid them, by means which the law permits, cannot be
doubted.
Thus, even if Ellice and Margo were organized for the purpose of exempting the properties of the
Gala spouses from the coverage of land reform legislation and avoiding estate taxes, the court cannot
disregard their separate juridical personalities.
HEIRS OF ANTONIO PAEL and ANDREA ALCANTARA and CRISANTO PAEL
vs.
COURT OF APPEALS, JORGE H. CHIN and RENATO B. MALLARI
G.R. No. 133547.December 7, 2001

FACTS:

PFINA acquired the properties from the Heirs of Pael by virtue of a deed of assignment dated
January 25, 1983. It filed a motion to intervene before the Court of Appeals; however, before it filed its
motion for intervention, or for a long period of fifteen (15) years, PFINA and the Heirs of Pael were totally
silent about the alleged deed of assignment. No steps were taken by either of them to register the deed
or secure transfer certificate of title evidencing the change of ownership during this long period of time.
At the time PFINA acquired the disputed properties in 1983, its corporate name was PFINA
Mining and Exploration, Inc., a mining company which had no valid grounds to engage in the highly
speculative business of urban real estate development.

ISSUE:

Whether or not the 1983 transfer produces legal effect.

RULING:

NO.

As correctly ruled by the courts, the alleged transfer in 1983 was not only dubious and
fabricated; it could produce no legal effect as the Paels were no longer owners of the land they allegedly
assigned.
The Court highlighted the citation in the comment of Intervenor U.P., specifically citing the
decision in Roberto A. Pael et al. v. Court of Appeals, et al., supra, wherein the title of the Paels was
declared to be of dubious origin and a fabrication. Hence, since respondents derive their titles from a
defective title, their titles should also be null and void.
The motion for intervention of the University of the Philippines is GRANTED. The case is
REMANDED to the Court of Appeals for reception of evidence on the conflicting claims over the property
covered by TCT Nos. 52928 and 52929 between the intervernor University of the Philippines, on the one
hand, and respondents Jorge H. Chin and Renato B. Mallari, on the other hand. The motions for
reconsideration filed by petitioners are DENIED for lack of merit. This denial is FINAL and no further
pleadings from petitioners will be entertained.

42 | P a g e
Law 321_Corporation LAW_ Case Digest

UY SIULIONG, MARIANO LIMJAP, GACU UNG JIENG, EDILBERTO CALIXTO and UY CHO YEE
vs.
THE DIRECTOR OF COMMERCE AND INDUSTRY
G.R. No.L-15429. December 1, 1919

FACTS:

Prior to the presentation of the petition the petitioners had been associated together as partners,
which partnership was known as "mercantil regular colectiva, under the style and firm name of
"Siuliong y Cia. That the petitioners herein, who had theretofore been members of said partnership of
"Siuliong y Cia.," desired to dissolve said partnership and to form a corporation composed of the same
persons as incorporators, to be known as "Siulong y Compañia, Incorporada.
While the articles of incorporation of "Siuliong y Cia., Inc." states that its purpose is to acquire
and continue the business, with some of its objects or purposes, of Siuliong & Co., it will be found upon
an examination of the purposes enumerated in the proposed articles of incorporation of "Siuliong y Cia.,
Inc.," that some of the purposes of the original partnership of "Siuliong y Cia." have been omitted.

ISSUE:

Whether or not a corporation can engage in other purposes other than that stated in the purpose
clause of its articles of incorporation.

RULING:

YES.

A corporation may be organized under the laws of the Philippine Islands for mercantile purposes,
and to engage in such incidental business as may be necessary and advisable to give effect to, and aid
in, the successful operation and conduct of the principal business. All of the power and authority
included in the articles of incorporation of "Siuliong y Cia., Inc.," enumerated above in paragraph 4 of
the Articles of Incorporation are only incidental to the principal purpose of said proposed incorporation,
to wit: "mercantile business." The purchase and sale, importation and exportation of the products of the
country, as well as of foreign countries, might make it necessary to purchase and discount promissory
notes, bills of exchange, bonds, negotiable instruments, stock, and interest in other mercantile and
industrial associations. It might also become important and advisable for the successful operation of
the corporation to act as agent for insurance companies as well as to buy, sell and equip boats and to
buy and sell other establishments, and industrial and mercantile businesses. The proposed articles of
incorporation do not authorize the petitioners to engage in a business with more than one purpose, the
Court do not mean to be understood as having decided that corporations under the laws of the
Philippine Islands may not engage in a business with more than one purpose. Such an interpretation
might work a great injustice to corporations organized under the Philippine laws. Such an
interpretation would give foreign corporations, which are permitted to be registered under the laws here
and which may be organized for more than one purpose, a great advantage over domestic corporations.
It was not the intention of the legislature to give foreign corporations such an advantage over domestic
corporations.

NORBERTO ASUNCION, ET AL.


vs.
MANUEL DE YRIARTE
G.R. No. 9321.September 24, 1914

43 | P a g e
Law 321_Corporation LAW_ Case Digest

FACTS:

This is an action to obtain a writ of mandamus to compel the Chief of the Division of Achieves of
the Executive Bureau to file a certain articles of incorporation. The Chief of the Division of Archives
refused to file the articles of incorporation upon the ground that the object of the corporation, as stated
in the articles, was not lawful and that, in pursuance of section 6 of Act No. 1459, they were not
registerable.
The proposed incorporators began an action in the CFI of Manila to compel the Chief of the
Division of Archives to receive and register said articles of incorporation and to do any and all acts
necessary for the complete incorporation of the persons named in the articles.

ISSUE:

Whether or not the purposes of the corporation as stated in the articles of incorporation are
lawful within the meaning of the Corporation Law.

RULING:

YES.

When on the face of the articles of incorporation presented for registration it is shown that it is
organized for a purpose contrary to law or public policy, the same may be denied outright registration.
The object of the proposed corporation, as appears from the articles offered for registration, is to
make of the barrio of Pulo or San Miguel a corporation which will become the owner of and have the
right to control and administer any property belonging to the municipality of Pasig found within the
limits of that barrio. This clearly cannot be permitted. Otherwise municipalities as now established by
law could be deprived of the property which they now own and administer. Each barrio of the
municipality would become under the scheme proposed, a separate corporation, would take over the
ownership, administration, and control of that portion of the municipal territory within its limits. This
would disrupt, in a sense, the municipalities of the Islands by dividing them into a series of smaller
municipalities entirely independent of the original municipality. The object of the proposed corporation
is clearly repugnant to the provisions of the Municipal Code and the governments of municipalities as
they have been organized thereunder.

 Principal Office/Domicile

DAVAO LIGHT & POWER CO., INC.


vs.
THE HON. COURT OF APPEALS, HON. RODOLFO M. BELLAFLOR, Presiding Judge of Branch 11,
RTC-Cebu and FRANCISCO TESORERO
G.R. No. 111685.August 20, 2001

FACTS:

44 | P a g e
Law 321_Corporation LAW_ Case Digest

In 1992 Davao Light & Power Co., Inc. filed a complaint for damages against private respondent
Francisco Tesorero before the Regional Trial Court of Cebu for damages in the amount of P11,
000,000.00. In turn, the latter filed a motion to dismiss claiming among others that the venue was
improperly laid since the principal place of business of the plaintiff is Davao City as indicated in the
lease executed by petitioner, and the same determines the venue of the action, instead of Banilad City
which the company indicated in its complaint. The trial court granted the said motion. Petitioner’s
motion for reconsideration was denied, as well as its appeal to the Court of Appeals. Hence, this petition.

ISSUE:

Whether or not the company’s principal place is in Davao City.

RULING:

YES.

Davao City is the Principal place of business which determines venue. A corporation has no
residence in the same sense in which this term is applied to a natural person. But for practical
purposes, a corporation is in a metaphysical sense a resident of the place where its principal office is
located as stated in the articles of incorporation. The Corporation Code precisely requires each
corporation to specify in its articles of incorporation the "place where the principal office of the
corporation is to be located which must be within the Philippines". The purpose of this requirement is to
fix the residence of a corporation in a definite place, instead of allowing it to be ambulatory.
The same considerations apply to the instant case. It cannot be disputed that petitioner's
principal office is in Cebu City, per its amended articles of incorporation and by-laws. However, Tesorero
is not a party to any of the contracts presented before the court. Those documents were between the
petitioner and NAPOCOR and therefore estoppel may not lie against the private respondent. He is a
stranger to those documents even if he says that by being a member of the public for whose benefit the
electric generating contracts were entered into. There is no estoppel because there is no showing that he
relied on the representations made by the petitioner.

CLAVECILLIA RADIO SYSTEM


vs.
HON. AGUSTIN ANTILLON, as City Judge of the Municipal Court of Cagayan de Oro Cityand NEW
CAGAYAN GROCERY
G.R. No.L-22238.February 18, 1967

FACTS:

On June 22, 1963, the New Cagayan Grocery filed a complaint against the Clavecilla Radio
System alleging, in effect, that on March 12, 1963, the following message, addressed to the former, was
filed at the latter's Bacolod Branch Office for transmittal thru its branch office at Cagayan de Oro:
“NECAGRO CAGAYAN DE ORO (CLAVECILLA): REURTEL WASHED NOT AVAILABLE REFINED
TWENTY FIFTY IF AGREEABLE SHALL SHIP LATER REPLY POHANG.” The Cagayan de Oro branch
office having received the said message omitted, in delivering the same to the New Cagayan Grocery, the
word "NOT" between the words "WASHED" and "AVAILABLE," thus changing entirely the contents and
purport of the same and causing the said addressee to suffer damages. After service of summons, the
Clavecilla Radio System filed a motion to dismiss the complaint on the grounds that it states no cause of

45 | P a g e
Law 321_Corporation LAW_ Case Digest

action and that the venue is improperly laid. The New Cagayan Grocery interposed an opposition to
which the Clavecilla Radio System filed its rejoinder. Thereafter, the City Judge, on September 18, 1963,
denied the motion to dismiss for lack of merit and set the case for hearing.
Hence, the Clavecilla Radio System filed a petition for prohibition with preliminary injunction
with the Court of First Instance praying that the City Judge, Honorable Agustin Antillon, be enjoined
from further proceeding with the case on the ground of improper venue. The respondents filed a motion
to dismiss the petition but this was opposed by the petitioner. Later, the motion was submitted for
resolution on the pleadings.

ISSUE:

Whether or not the place is the proper venue to sue Clavecilla Radio System?

RULING:

NO.

In this case, the suit for damages filed with the city court is based upon tort and not upon a
written contract. Section 1 of Rule 4 of the New Rules of Court, governing venue of actions in inferior
courts, provides in its paragraph (b)(3) that when "the action is not upon a written contract, then in the
municipality where the defendant or any of the defendants resides or may be served with summons."
Settled is the principle in corporation law that the residence of a corporation is the place where its
principal office is established. Since it is not disputed that the Clavecilla Radio System has its principal
office in Manila, it follows that the suit against it may properly be filed in the City of Manila.
The appellee maintain, however, that with the filing of the action in Cagayan de Oro City, venue
was properly laid on the principle that the appellant may also be served with summons in that city
where it maintains a branch office. The term "may be served with summons" does not apply when the
defendant resides in the Philippines for, in such case, he may be sued only in the municipality of his
residence, regardless of the place where he may be found and served with summons. As any other
corporation, the Clavecilla Radio System maintains a residence which is Manila in this case, and a
person can have only one residence at a time (See Alcantara vs. Secretary of the Interior, 61 Phil. 459;
Evangelists vs. Santos, 86 Phil. 387). The fact that it maintains branch offices in some parts of the
country does not mean that it can be sued in any of these places. To allow an action to be instituted in
any place where a corporate entity has its branch offices would create confusion and work untold
inconvenience to the corporation.

JOHN SY and UNIVERSAL PARTS SUPPLY CORPORATION


vs.
TYSON ENTERPRISES, INC., JUDGE GREGORIO G. PINEDA of the Court of First Instance of Rizal,
Pasig Branch XXI and COURT OF APPEALS
G.R. No.L-56763. December 15, 1982

FACTS:

On August 29, 1979, Tyson Enterprises, Inc. filed against John Sy and Universal Parts Supply
Corporation, residents of Bacolod, a complaint for the collection of money in Pasig, Rizal. However,
there is no allegation in the complaint as to the office or place of business of plaintiff Tyson Enterprises,
Inc., which is located in Manila. What is alleged is the postal address or residence of Dominador Ti, the
president and general manager of plaintiff firm, which is in San Juan, Rizal.
Defendant Sy and Universal Parts Supply Corporation filed a motion to dismiss on the ground of
improper venue. The plaintiff opposed the motion to dismiss which the trial court denied. On appeal,
the Appellate Court dismissed the petition. It ruled that the parties did not intend Manila as the
exclusive venue of the actions arising under their transactions and that since the action was filed in
Pasig, which is near Manila, no useful purpose would be served by dismissing the same and ordering
that it be filed in Manila.

ISSUE:

Whether or not venue was improperly laid in this case.

RULING:

YES.

46 | P a g e
Law 321_Corporation LAW_ Case Digest

The place of business of plaintiff Tyson Enterprises, Inc., which for purposes of venue is
considered as its residence is in Manila and not in Rizal. The residence of its president is not the
residence of the corporation because a corporation has a personality separate and distinct from that of
its officers and stockholders. Consequently, the collection suit should have been filed in Manila, the
residence of plaintiff corporation and the place designated in its sales invoice, or it could have been filed
also in Bacolod City, the residence of defendant Sy.
The decision of the Court of Appeals and the order of respondent judge denying the motion to
dismiss are reversed and set aside. The writ of prohibition is granted. Civil Case No. 34302 should be
considered dismissed without prejudice to refiling it in the Court of First Instance of Manila or Bacolod
City at the election of plaintiff which should be allowed to withdraw the documentary evidence
submitted in that case. All the proceedings in said case, including the decision, were also set aside.
The decision of the Court of Appeals and the order of respondent judge denying the motion to
dismiss are reversed and set aside.

YOUNG AUTO SUPPLY CO. AND NEMESIO GARCIA


vs.
THE HONORABLE COURT OF APPEALS (THIRTEENTH DIVISION) AND GEORGE CHIONG ROXAS
G.R. No. 104175. June 25, 1993

FACTS:

On October 28, 1987, Young Auto Supply Co. Inc. (YASCO) represented by Nemesio Garcia, its
president, Nelson Garcia and Vicente Sy, sold all of their shares of stock in Consolidated Marketing &
Development Corporation (CMDC) to Roxas. The purchase price was P8,000,000.00 payable as follows:
a down payment of P4,000,000.00 and the balance of P4,000,000.00 in four postdated checks of
P1,000,000.00 each. The first check of P4, 000,000.00, representing the down payment, was honored
by the drawee bank but the four other checks representing the balance of P4, 000,000.00 were
dishonored.
On June 10, 1988, petitioners filed a complaint against Roxas in the Regional Trial Court, Branch
11, Cebu City, praying that Roxas be ordered to pay petitioners the sum of P3, 400,000.00 or that full
control of the three markets be turned over to YASCO and Garcia. The complaint also prayed for the
forfeiture of the partial payment of P4, 600,000.00 and the payment of attorney's fees and costs.

ISSUE:

Whether the proper venue is in Pasay City.

RULING:

NO.

The Court of Appeals erred in holding that the venue was improperly laid in Cebu City. Young
Auto Supply Co., Inc. ("YASCO") is a domestic corporation duly organized and existing under Philippine
laws with principal place of business at M.J. Cuenco Avenue, Cebu City. It also has a branch office at
1708 Dominga Street, Pasay City, Metro Manila. The Article of Incorporation of YASCO states that the
place where the principal office of the corporation is to be established or located is at Cebu City,
Philippines.

47 | P a g e
Law 321_Corporation LAW_ Case Digest

A corporation has no residence in the same sense in which this term is applied to a natural
person. But for practical purposes, a corporation is in a metaphysical sense a resident of the place
where its principal office is located as stated in the articles of incorporation. The Corporation Code
precisely requires each corporation to specify in its articles of incorporation the "place where the
principal office of the corporation is to be located which must be within the Philippines" The purpose of
this requirement is to fix the residence of a corporation in a definite place, instead of allowing it to be
ambulatory. With the finding that the residence of YASCO for purposes of venue is in Cebu City, where
its principal place of business is located, it becomes unnecessary to decide whether Garcia is also a
resident of Cebu City and whether Roxas was in estoppel from questioning the choice of Cebu City as
the venue. Hence, it should be in Cebu City.

 Term

ALHAMBRA CIGAR & CIGARETTE MANUFACTURING COMPANY, INC.


vs.
SECURITIES & EXCHANGE COMMISSION
G.R. No. L-23606.July 29, 1968

FACTS:

Incorporated under Philippine laws on January 15, 1912, petitioner Alhambra Cigars Mfg. Co
(ACCMI) was to exist for fifty (50) years from incorporation. Its term of existence expired on January 15,
1962. On that date, it ceased transacting business and entered into a state of liquidation.
Thereafter, a new corporation — Alhambra Industries, Inc. was formed to carry on the business of
Alhambra. On May 1, 1962, Alhambra's stockholders, by resolution, named Angel S. Gamboa trustee to
take charge of its liquidation. On June 20, 1963,within Alhambra's three-year statutory period for
liquidation — Republic Act 3531 was enacted into law amending Section 18 of the Corporation Law and
enabling domestic private corporations to extend their corporate life beyond the period fixed by the
articles of incorporation for a term not to exceed fifty years in any one instance.
On July 15, 1963 Alhambra's board of directors resolved to amend paragraph "Fourth" of its
articles of incorporation to extend its corporate life for an additional fifty years, or a total of 100 years
from its incorporation. Its stockholders, representing more than two-thirds of its subscribed capital
stock, voted to approve the foregoing resolution. SEC, however, returned said amended articles of
incorporation with the ruling that RA 3531 which took effect only on June 20, 1963, cannot be availed
of by the said corporation, for the reason that its term of existence had already expired when the said
law took effect; in short, said law has no retroactive effect."

ISSUE:

Whether or not a corporation may extend its life by amendment of its articles of incorporation
effected during the three-year statutory period for liquidation when its original term of existence had
already expired.

RULING:

NO.

Provided by Section 77 of the Corporation Law, the continuance of a "dissolved" corporation as a


body corporate for three years has for its purpose the final closure of its affairs, and no other; the
corporation is specifically enjoined from continuing the business for which it was established. The
liquidation of the corporation's affairs set forth in Section 77 became necessary precisely because its life
had ended. For this reason alone, the corporate existence and juridical personality of that corporation to

48 | P a g e
Law 321_Corporation LAW_ Case Digest

do business may no longer be extended. The provisions of RA 3531 merely empower a corporation to act
in liquidation, and not to extend its corporate existence.

 Paid-up Capital Stock

MSCI-NACUSIP Local Chapter


vs.
NATIONAL WAGES AND PRODUCTIVITY COMMISSION and MONOMER SUGAR CENTRAL, INC.
G.R. No. 125198. March 3, 1997

FACTS:

On January 11, 1990, Asturias Sugar Central, Inc. (ASCI), executed a Memorandum of
Agreement with Monomer Trading Industries, Inc. (MTII), whereby MTII shall acquire the assets of ASCI
by way of a Deed of Assignment provided that an entirely new organization in place of MTII shall be
organized, which new corporation shall be the assignee of the assets of ASCI.
By virtue of this Agreement, a new corporation was organized and incorporated on February 15,
1990 under the corporate name Monomer Sugar Central, Inc. or MSCI, the private respondent herein.
On January 16, 1991, MSCI applied for exemption from the coverage of Wage Order No. RO VI-01 issued
by the Board on the ground that it is a distressed employer.
On January 16, 1991, MSCI applied for exemption from the coverage of Wage Order No. RO VI-01
issued by the Board on the ground that it is a distressed employer. In support thereto, MSCI submitted
its audited financial statements and income tax returns duly stamped "received" by the Bureau of
Internal Revenue (BIR) and the Securities and Exchange Commission (SEC) for the period beginning
February 15, 1990 and ending August 31, 1990, including the quarterly financial statements and
income tax returns for the two quarters ending November 30, 1990 and February 28, 1991.

ISSUE:

Whether or not the correct paid-up capital of MSCI for the pertinent period covered by the
application for exemption is P5 million, not P64,688,528.00.

RULING:

YES.

It is P5 million. The Supreme Court held that in the case under consideration, there is no
dispute, and the Board even mentioned in its August 17, 1993 Decision, that MSCI was organized and
incorporated on February 15, 1990 with an authorized capital stock of P60 million, P20 million of which
was subscribed. Of the P20 million subscribed capital stock, P5 million was paid-up. This fact is only
too glaring for the Board to have been misled into believing that MSCI'S paid-up capital stock was P64
million plus and not P5 million.
Power to increase or decrease capital stock; incur, create or increase bonded indebtedness. No
corporation shall increase or decrease its capital stock or incur, create or increase any bonded
indebtedness unless approved by a majority vote of the board of directors and, at a stockholders'
meeting duly called for the purpose, two-thirds (2/3) of the outstanding capital stock shall favor the
increase or diminution of the capital stock, or the incurring, creating or increasing of any bonded
indebtedness.
The above requirements, which are condition precedents before the capital stock of a corporation
may be increased, were unquestionably not observed in this case. Henceforth, the paid-up capital stock

49 | P a g e
Law 321_Corporation LAW_ Case Digest

of MSCI for the period covered by the application for exemption still stood at P5 million. The losses,
therefore, amounting to P3,400,738.00 for the period February 15, 1990 to August 31, 1990 impaired
MSCI's paid-up capital of P5 million by as much as 68%. Likewise, the losses incurred by MSCI for the
interim period from September 1, 1990 to November 30, 1990, as found by the Commission, per MSCI's
quarterly income statements, amounting to P13,554,337.33 impaired the company's paid-up capital of
P5 million by a whopping 271.08%, more than enough to qualify MSCI as a distressed employer.
 Amendment and/or Rejection of Articles of Incorporation

REPUBLIC PLANTERS BANK


vs.
COURT OF APPEALS and FERMIN CANLAS
G.R. No. 93073. December 21, 1992

FACTS:

On September 18, 1961, private respondent Corporation secured a loan from petitioner in the
amount of P120,000.00. As part of the proceeds of the loan, preferred shares of stocks were issued to
private respondent Corporation, through its officers then, private respondent Adalia F. Robes and one
Carlos F. Robes. In other words, instead of giving the legal tender totaling to the full amount of the loan,
which is P120,000.00, petitioner lent such amount partially in the form of money and partially in the
form of stock certificates. Said stock certificates were in the name of private respondent Adalia F. Robes
and Carlos F. Robes, who subsequently, however, endorsed his shares in favor of Adalia F. Robes. Said
certificates of stock bear the following terms and conditions:
The Preferred Stock shall have the following rights, preferences, qualifications and limitations, to
wit: Of the right to receive a quarterly dividend of One Per Centum (1%), cumulative and participating.
That such preferred shares may be redeemed, by the system of drawing lots, at any time after two (2)
years from the date of issue at the option of the Corporation
On January 31, 1979, private respondents proceeded against petitioner and filed a Complaint
anchored on private respondents' alleged rights to collect dividends under the preferred shares in
question and to have petitioner redeem the same under the terms and conditions of the stock
certificates.

ISSUES:

Whether or not there is a difference between a preferred share from a redeemable share.
Whether or not petitioner can be compelled by defendant to redeem the preferred shares issued to
the private respondent.

RULING:

YES.

A preferred share of stock is one which entitles the holder thereof to certain preferences over the
holders of common stock. The preferences are designed to induce persons to subscribe for shares of a
corporation. Preferred shares take a multiplicity of forms. The most common forms may be classified
into two: (1) preferred shares as to assets; and (2) preferred shares as to dividends. The former is a share
which gives the holder thereof preference in the distribution of the assets of the corporation in case of
liquidation; the latter is a share the holder of which is entitled to receive dividends on said share to the
extent agreed upon before any dividends at all are paid to the holders of common stock. There is no
guaranty, however, that the share will receive any dividends.
Under the old Corporation Law in force at the time the contract between the petitioner and the
private respondents was entered into, it was provided that "no corporation shall make or declare any
dividend except from the surplus profits arising from its business, or distribute its capital stock or
property other than actual profits among its members or stockholders until after the payment of its
debts and the termination of its existence by limitation or lawful dissolution." Similarly, the present
Corporation Code provides that the board of directors of a stock corporation may declare dividends only
out of unrestricted retained earnings. The Code, in Section 43, adopting the change made in accounting
terminology, substituted the phrase "unrestricted retained earnings," which may be a more precise term,
in place of "surplus profits arising from its business" in the former law.
Thus, the declaration of dividends is dependent upon the availability of surplus profit or
unrestricted retained earnings, as the case may be.
Preferences granted to preferred stockholders, moreover, do not give them a lien upon the
property of the corporation nor make them creditors of the corporation, the right of the former being

50 | P a g e
Law 321_Corporation LAW_ Case Digest

always subordinate to the latter. Dividends are thus payable only when there are profits earned by the
corporation and as a general rule, even if there are existing profits, the board of directors has the
discretion to determine whether or not dividends are to be declared. Shareholders, both common and
preferred, are considered risk takers who invest capital in the business and who can look only to what is
left after corporate debts and liabilities are fully paid.
Redeemable shares, on the other hand, are shares usually preferred, which by their terms are
redeemable at a fixed date, or at the option of either issuing corporation, or the stockholder, or both at a
certain redemption price. Redemption by the corporation of its stock is, in a sense, a repurchase of it for
cancellation. The present Code allows redemption of shares even if there are no unrestricted retained
earnings on the books of the corporation. This is a new provision which in effect qualifies the general
rule that the corporation cannot purchase its own shares except out of current retained earnings.
However, while redeemable shares may be redeemed regardless of the existence of unrestricted retained
earnings, this is subject to the condition that the corporation has, after such redemption, assets in its
books to cover debts and liabilities inclusive of capital stock. Redemption, therefore, may not be made
where the corporation is insolvent or if such redemption will cause insolvency or inability of the
corporation to meet its debts as they mature.

YES.

While it is true that the very wordings of the terms and conditions in said stock certificates clearly
allows redemption, the option to do was clearly vested in the petitioner bank. The redemption therefore
is clearly the type known as "optional". Thus, except as otherwise provided in the stock certificate, the
redemption rests entirely with the corporation and the stockholder is without right to either compel or
refuse the redemption of its stock. Furthermore, the terms and conditions set forth therein use the
word "may". It is a settled doctrine in statutory construction that the word "may" denotes discretion, and
cannot be construed as having a mandatory effect.
CA decision is reversed and set aside. Judgement is hereby rendered declaring private
respondent Fermin Canlas jointly and severally liable on all the nine promissory notes with the following
sums and at 16% interest per annum from the dates indicated.

 Alter Ego/Instrumentality Cases

PHILIPPINE NATIONAL BANK


vs.
HYDRO RESOURCES CONTRACTORS CORPORATION
G.R. No. 167561
ASSET PRIVATIZATION TRUST
vs.
HYDRO RESOURCES CONTRACTORS CORPORATION
G.R. No. 167603
DEVELOPMENT BANK OF THE PHILIPPINES

51 | P a g e
Law 321_Corporation LAW_ Case Digest

vs.
HYDRO RESOURCES CONTRACTORS CORPORATION
G.R. No. 167530. March 13, 2013

FACTS:

A contract was entered into between Hydro and NIA for the project of the latter. The contract price
is to be payable partly in Philippine peso and US dollars. Once the project was being executed, there was
depreciation in value of Peso resulting to price differential. In order to resolve the issue, the
administrator of NIA, Mr Tek, and Hydro made a joint computation of the amount corresponding to the
foreign currency differential. The computation showed that NIA owed Hydro for the differential. When a
demand was made by Hydro against NIA, NIA refused to pay contending that Mr Tek has no authority to
participate into a joint computation of the foreign currency differential and that Mr Tek has no authority
to bind NIA.

ISSUE:

Whether or not the corporate entity of PNB and DBP must be pierced.

RULING:

NO.

A corporation is an artificial entity created by operation of law. It possesses the right of


succession and such powers, attributes, and properties expressly authorized by law or incident to its
existence. It has a personality separate and distinct from that of its stockholders and from that of other
corporations to which it may be connected. As a consequence of its status as a distinct legal entity and
as a result of a conscious policy decision to promote capital formation, a corporation incurs its own
liabilities and is legally responsible for payment of its obligations.40 In other words, by virtue of the
separate juridical personality of a corporation, the corporate debt or credit is not the debt or credit of the
stockholder. This protection from liability for shareholders is the principle of limited liability.
Equally well-settled is the principle that the corporate mask may be removed or the corporate veil
pierced when the corporation is just an alter ego of a person or of another corporation. For reasons of
public policy and in the interest of justice, the corporate veil will justifiably be impaled only when it
becomes a shield for fraud, illegality or inequity committed against third persons.
However, the rule is that a court should be careful in assessing the milieu where the doctrine of
the corporate veil may be applied. Otherwise an injustice, although unintended, may result from its
erroneous application. Thus, cutting through the corporate cover requires an approach characterized by
due care and caution.
Hence, any application of the doctrine of piercing the corporate veil should be done with caution.
A court should be mindful of the milieu where it is to be applied. It must be certain that the corporate
fiction was misused to such an extent that injustice, fraud, or crime was committed against another, in
disregard of its rights. The wrongdoing must be clearly and convincingly established; it cannot be
presumed.

VIVIAN T. RAMIREZ, ALBERTO B. DIGNO, DANILO M. CASQUITE, JUMADIYA A. KADIL, FAUJIA


SALIH, ANTONIO FABIAN, ROMEL DANAG, et.al.
vs.
MAR FISHING CO., INC., MIRAMAR FISHING CO., INC., ROBERT BUEHS AND JEROME SPITZ
G.R. No. 168208. June 13, 2012

FACTS:

Mar Fishing Co., Inc. (Mar Fishing), engaged in the business of fishing and canning of tuna, sold
its principal assets to co-respondent Miramar through public bidding. Proceeds of the sale were paid to
the Trade and Investment Corp. to cover Mar Fishing’s outstanding obligation in the amount of ₱
897,560,041.00.
In view of that transfer, Mar Fishing issued a Memorandum informing all its workers that the
company would cease to operate by the end of the month. It notified the DOLE of the closure of its
business operations. Then, Mar Fishing’s labor union, Mar Fishing Workers Union – NFL –
and Miramar entered into a Memorandum of Agreement for the acquiring company, Miramar, to absorb
Mar Fishing’s regular rank and file employees whose performance was satisfactory, without loss of
seniority rights and privileges previously enjoyed. Unfortunately, petitioners, who worked as rank and

52 | P a g e
Law 321_Corporation LAW_ Case Digest

file employees, were not hired or given separation pay by Miramar, so they filed Complaints for illegal
dismissal with money claims before the Arbitration Branch of the NLRC.

ISSUE:

Whether or not Mar Fishing and Miramar are solidarily liable to the employees.

RULING:

NO.

Mar Fishing, and not Miramar, is required to compensate petitioners. Indeed, the back wages and
retirement pay earned from the former employer cannot be filed against the new owners or operators of
an enterprise.
Miramar and Mar Fishing are separate and distinct entities, based on the marked differences in
their stock ownership. Also, the fact that Mar Fishing’s officers remained as such in Miramar does not
by itself warrant a conclusion that the two companies are one and the same. The mere showing that the
corporations had a common director sitting in all the boards without more does not authorize
disregarding their separate juridical personalities.
Neither can the veil of corporate fiction between the two companies be pierced by the rest of
petitioners’ submissions, namely, the alleged take-over by Miramar of Mar Fishing’s operations and the
evident similarity of their businesses. Since piercing the veil of corporate fiction is frowned upon, those
who seek to pierce the veil must clearly establish that the separate and distinct personalities of the
corporations are set up to justify a wrong, protect a fraud, or perpetrate a deception. This,
unfortunately, petitioners have failed to do.

TIMOTEO H. SARONA
vs.
NATIONAL LABOR RELATIONS COMMISSION, ROYALE SECURITY AGENCY (FORMERLY SCEPTRE
SECURITY AGENCY) and CESAR S. TAN
G.R. No. 185280. January 18, 2012

FACTS:

Petitioner, a security guard in Sceptre since April 1976, was asked by Sceptre’s operations
manager to submit a resignation letter as a requirement for an application in Royale and to fill up an
employment application form for the said company. He was then assigned at Highlight Metal Craft Inc.
from July 29 to August 8, 2003 and was later transferred to Wide Wide World Express Inc.
On September 2003, he was informed that his assignment at WWWE Inc. was withdrawn because
Royale has been allegedly replaced by another security agency which he later discovered to be untrue.
Nevertheless, he was once again assigned at Highlight Metal sometime in September 2003and when he
reported at Royale’s office on October 1, 2003, he was informed that he would no longer be given any
assignment as instructed by Sceptre’s general manager.
He thus filed a complaint for illegal dismissal.

ISSUE:

Whether or not Royale’s corporate fiction should be pierced for the purpose of compelling it to
recognize the petitioner’s length of service with Sceptre and for holding it liable for the benefits that have
accrued to him arising from his employment with Sceptre.

53 | P a g e
Law 321_Corporation LAW_ Case Digest

RULING:

YES.

The doctrine of piercing the corporate veil is applicable on alter ego cases, where a corporation is
merely a farce since it is a mere alter ego or business conduit of a person, or where the corporation is so
organized and controlled and its affairs are so conducted as to make it merely an instrumentality,
agency, conduit or adjunct of another corporation.
The respondents’ scheme reeks of bad faith and fraud and compassionate justice dictates that
Royale and Sceptre be merged as a single entity, compelling Royale to credit and recognize the
petitioner’s length of service with Sceptre. The respondents cannot use the legal fiction of a separate
corporate personality for ends subversive of the policy and purpose behind its creation or which could
not have been intended by law to which it owed its being.
Also, Sceptre and Royale have the same principal place of business. As early as October 14, 1994,
Aida and Wilfredo became the owners of the property used by Sceptre as its principal place of business
by virtue of a Deed of Absolute Sale they executed with Roso. Royale, shortly after its incorporation,
started to hold office in the same property. These, the respondents failed to dispute. Royale also claimed
a right to the cash bond which the petitioner posted when he was still with Sceptre. If Sceptre and
Royale are indeed separate entities, Sceptre should have released the petitioner’s cash bond when he
resigned and Royale would have required the petitioner to post a new cash bond in its favor.
The way on how petitioner was made to resign from Sceptre then later on made an employee of
Royale, reflects the use of the legal fiction of the separate corporate personality and is an implication of
continued employment. Royale is a continuation or successor or Sceptre since the employees of Sceptre
and of Royale are the same and said companies have the same principal place of business.

GOLD LINE TOURS, INC.


vs.
HEIRS OF MARIA CONCEPCION LACSA
G.R. No. 159108. June 18, 2012

FACTS:

Concepcion boarded a Gold Line passenger bus owned and operated by Travel & Tours Advisers,
Inc. Before reaching their destination, the Gold Line bus collided with a passenger jeepney and as a
result, a metal part of the jeepney was detached and struck Concepcion in the chest, causing her
instant death. Then, Concepcion’s heirs, represented by Teodoro Lacsa, instituted in the RTC a suit
against Travel & Tours Advisers Inc. to recover damages arising from breach of contract of carriage.

ISSUE:

Whether or not the proposition of the third party claimant by the petitioner where Travel & Tours
Advises, Inc. has an existence separate and/or distinct from Gold Line Tours, Inc.

RULING:

NO.

The two corporations are liable to the death of Ma. Concepcion Lacsa. The Court was not
persuaded by the proposition of the third party claimant that a corporation has an existence separate
and/or distinct from its members insofar as this case at bar is concerned, for the reason that whenever
necessary for the interest of the public or for the protection of enforcement of their rights, the notion of
legal entity should not and is not to be used to defeat public convenience, justify wrong, protect fraud or
defend crime.
Where the main purpose in forming the corporation was to evade one’s subsidiary liability for
damages in a criminal case, the corporation may not be heard to say that it has a personality separate
and distinct from its members, because to allow it to do so would be to sanction the use of fiction of
corporate entity as a shield to further an end subversive of justice.
This is what the third party claimant wants to do including the defendant in this case, to use the
separate and distinct personality of the two corporations as a shield to further an end subversive of
justice by avoiding the execution of a final judgment of the court.

54 | P a g e
Law 321_Corporation LAW_ Case Digest

The RTC thus rightly ruled that petitioner might not be shielded from liability under the final
judgment through the use of the doctrine of separate corporate identity. Truly, this fiction of law could
not be employed to defeat the ends of justice.

HACIENDA LUISITA, INCORPORATED


LUISITA INDUSTRIAL PARK CORPORATION and RIZAL COMMERCIAL BANKING CORPORATION
vs.
PRESIDENTIAL AGRARIAN REFORM COUNCIL; SECRETARY NASSER PANGANDAMAN OF THE
DEPARTMENT OF AGRARIAN REFORM; ALYANSA NG MGA MANGGAGAWANG BUKID NG
HACIENDA LUISITA, RENE GALANG, NOEL MALLARI, and JULIO SUNIGA1 and his SUPERVISORY
GROUP OF THE HACIENDA LUISITA, INC. and WINDSOR ANDAYA
G.R. No. 171101. November 22, 2011

FACTS:

On March 17, 1988, during the administration of President Corazon Cojuangco Aquino, the Office
of the Solicitor General moved to withdraw the government’s case against Tadeco, et al. The CA
dismissed the case, subject to the PARC’s approval of Tadeco’s proposed stock distribution plan (SDP) in
favor of its farmworkers. [Under EO 229 and later RA 6657, Tadeco had the option of availing stock
distribution as an alternative modality to actual land transfer to the farmworkers.] On August 23, 1988,
Tadeco organized a spin-off corporation, herein petitioner HLI, as vehicle to facilitate stock acquisition
by the farmworkers. For this purpose, Tadeco conveyed to HLI the agricultural land portion (4,915.75
hectares) and other farm-related properties of Hacienda Luisita in exchange for HLI shares of stock.
On May 9, 1989, some 93% of the then farmworker-beneficiaries (FWBs) complement of Hacienda
Luisita signified in a referendum their acceptance of the proposed HLI’s Stock Distribution Option Plan
(SODP). On May 11, 1989, the SDOA was formally entered into by Tadeco, HLI, and the 5,848 qualified
FWBs. This attested to by then DAR Secretary Philip Juico. The SDOA embodied the basis and
mechanics of HLI’s SDP, which was eventually approved by the PARC after a follow-up referendum
conducted by the DAR on October 14, 1989, in which 5,117 FWBs, out of 5,315 who participated, opted
to receive shares in HLI.
On August 15, 1995, HLI applied for the conversion of 500 hectares of land of the hacienda from
agricultural to industrial use, pursuant to Sec. 65 of RA 6657. The DAR approved the application on
August 14, 1996, subject to payment of three percent (3%) of the gross selling price to the FWBs and to
HLI’s continued compliance with its undertakings under the SDP, among other conditions.

ISSUE:

Whether or not the revocation of the HLI’s SDP valid.

RULING:

YES.

The PARC did NOT gravely abuse its discretion in revoking the subject SDP and placing the
hacienda under CARP’s compulsory acquisition and distribution scheme. The revocation of the
approval of the SDP is valid: (1) the mechanics and timelines of HLI’s stock distribution violate DAO 10

55 | P a g e
Law 321_Corporation LAW_ Case Digest

because the minimum individual allocation of each original FWB of 18,804.32 shares was diluted as a
result of the use of “man days” and the hiring of additional farmworkers; (2) the 30-year timeframe for
HLI-to-FWBs stock transfer is contrary to what Sec. 11 of DAO 10 prescribes.
It is clear as day that the original 6,296 FWBs, who were qualified beneficiaries at the time of the
approval of the SDP, suffered from watering down of shares. As determined earlier, each original FWB is
entitled to 18,804.32 HLI shares. The original FWBs got less than the guaranteed 18,804.32 HLI shares
per beneficiary, because the acquisition and distribution of the HLI shares were based on “man days” or
“number of days worked” by the FWB in a year’s time.

PANTRANCO EMPLOYEES ASSOCIATION (PEA-PTGWO) and PANTRANCO RETRENCHED


EMPLOYEES ASSOCIATION (PANREA)
vs.
NATIONAL LABOR RELATIONS COMMISSION (NLRC), PANTRANCO NORTH EXPRESS, INC. (PNEI),
PHILIPPINE NATIONAL BANK (PNB), PHILIPPINE NATIONAL BANK-MANAGEMENT AND
DEVELOPMENT CORPORATION (PNB-MADECOR), and MEGA PRIME REALTY AND HOLDINGS
CORPORATION (MEGA PRIME)
G.R. No. 170689. March 17, 2009

FACTS:

The Gonzales family owned two corporations, namely, the PNEI and Macris Realty Corporation.
PNEI provided transportation services to the public, and had its bus terminal at the corner of Quezon
and Roosevelt Avenues in Quezon City. The terminal stood on four valuable pieces of real estate
registered under the name of Macris. The Gonzales family later incurred huge financial losses despite
attempts of rehabilitation and loan infusion. In March 1975, their creditors took over the management
of PNEI and Macris. By 1978, full ownership was transferred to one of their creditors, the National
Investment Development Corporation (NIDC), a subsidiary of the PNB.
In 1985, NIDC sold PNEI to North Express Transport, Inc. (NETI), a company owned by Gregorio
Araneta III. In 1992, PNEI applied with the Securities and Exchange Commission (SEC) for suspension
of payments.

ISSUE:

Whether PNEI employees can attach the properties (specifically the Pantranco properties) of PNB,
PNB-Madecor and Mega Prime to satisfy their unpaid labor claims against PNEI.

RULING:

NO.

First, the subject property is not owned by the judgment debtor, that is, PNEI. Nowhere in the
records was it shown that PNEI owned the Pantranco properties. Settled is the rule that the power of the
court in executing judgments extends only to properties unquestionably belonging to the judgment
debtor alone. To be sure, one man’s goods shall not be sold for another man’s debts. A sheriff is not
authorized to attach or levy on property not belonging to the judgment debtor, and even incurs liability if
he wrongfully levies upon the property of a third person.
Second, PNB, PNB-Madecor and Mega Prime are corporations with personalities separate and
distinct from that of PNEI. PNB is sought to be held liable because it acquired PNEI through NIDC at the
time when PNEI was suffering financial reverses. PNB-Madecor is being made to answer for petitioners’
labor claims as the owner of the subject Pantranco properties and as a subsidiary of PNB. Mega Prime is
also included for having acquired PNB’s shares over PNB-Madecor.
The general rule is that a corporation has a personality separate and distinct from those of its
stockholders and other corporations to which it may be connected. This is a fiction created by law for
convenience and to prevent injustice. Obviously, PNB, PNB-Madecor, Mega Prime, and PNEI are
corporations with their own personalities. Neither can we merge the personality of PNEI with PNB simply
because the latter acquired the former. Settled is the rule that where one corporation sells or otherwise
transfers all its assets to another corporation for value, the latter is not, by that fact alone, liable for the
debts and liabilities of the transferor.
Lastly, while we recognize that there are peculiar circumstances or valid grounds that may exist
to warrant the piercing of the corporate veil, none applies in the present case whether between PNB and
PNEI; or PNB and PNB-Madecor.

56 | P a g e
Law 321_Corporation LAW_ Case Digest

CAGAYAN VALLEY DRUG CORPORATION


vs.
COMMISSIONER OF INTERNAL REVENUE
G.R. No. 151413. February 13, 2008

FACTS:

Petitioner granted 20% sales discounts to qualified senior citizens on purchases of medicine
pursuant to RA 7432 and its IRR. Petitioner filed with the BIR a claim for tax refund/tax credit of the
full amount of the 20% sales discount it granted to senior citizens for the year 1995, in accordance with
RA 7432.
The BIR’s inaction on petitioner’s claim for refund/tax credit compelled petitioner to file a petition
for review before the CTA, until the case reached the CA. The CA dismissed the petition because the
person who signed the verification and certification of absence of forum shopping, a certain Jacinto J.
Concepcion, President of petitioner, failed to adduce proof that he was duly authorized by the board of
directors to do so.

ISSUE:

Whether or not it is valid when the petitioner’s president signs the subject verification and
certification sans the approval of its Board of Directors.

RULING:

NO.

A corporation has a separate and distinct personality from its directors and officers and can only
exercise its corporate powers through the board of directors. Thus, it is clear that an individual
corporate officer cannot solely exercise any corporate power pertaining to the corporation without
authority from the board of directors.
Only individuals vested with authority by a valid board resolution may sign the certificate of non-
forum shopping on behalf of a corporation. The action can be dismissed if the certification was
submitted unaccompanied by proof of the signatory’s authority. Hence, the power to sue and be sued in
any court or quasi-judicial tribunal is necessarily lodged with the said board.
There is a complete listing of authorized signatories to the verification and certification required
by the rules, the determination of the sufficiency of the authority was done on a case to case basis. The
rationale applied in the foregoing cases is to justify the authority of corporate officers or representatives
of the corporation to sign the verification or certificate against forum shopping, being in a position to
verify the truthfulness and correctness of the allegations in the petition.

THE HEIRS OF THE LATE PANFILO


vs.
PAJARILLO VS. CA, NLRC, et al.
G.R. No. 155056-57. October 19, 2007

57 | P a g e
Law 321_Corporation LAW_ Case Digest

FACTS:

Private respondents were employed as drivers, conductors and conductresses by Panfilo. In sum,
each of the private respondents earned an average daily commission of about P150.00 a day. They were
not given emergency cost of living allowance, 13th month pay, legal holiday pay and service incentive
leave pay.The following were deducted from the private respondents’ daily commissions. Thereafter,
private respondents and several co-employees formed a union called “SAMAHAN NG MGA
MANGGAGAWA NG PANFILO V. PAJARILLO”. The Department of Labor and Employment issued a
Certificate of Registration in favor of the respondent union. Upon learning of the formation of respondent
union, Panfilo and his children ordered some of the private respondents to sign a document affirming
their trust and confidence in Panfilo and denying any irregularities on his part. Other private
respondents were directed to sign a blank document which turned out to be a resignation letter. Private
respondents refused to sign the said documents; hence, they were barred from working or were
dismissed without hearing and notice. Panfilo and his children and relatives also formed a company
union where they acted as its directors and officers.
On 25 August 1987, respondent union and several employees filed a Complaint for unfair labor
practice and illegal deduction before the Labor Arbiter with “Panfilo V. Pajarillo Liner” as party-
respondent. After hearing and submission by both parties of their respective position papers and
memoranda, Labor Arbiter Manuel P. Asuncion rendered a Decision dated 28 December 1992,
dismissing the consolidated complaints for lack of merit. Respondent union appealed to the NLRC. On
18 June 1996, the NLRC reversed the decision of Arbiter Asuncion and ordered the reinstatement and
payment of backwages, ECOLA, 13th month pay, legal holiday pay and service incentive leave pay to,
private respondents.

ISSUE:

Whether the Honorable Court of Appeals seriously erred in piercing the veil of corporate entity of
Pvp Pajarillo Liner Inc.

RULING:

NO.

Hence, when the notion of separate juridical personality is used to defeat public convenience,
justify wrong, protect fraud or defend crime, or is used as a device to defeat labor laws, this separate
personality of the corporation may be disregarded or the veil of the corporate fiction pierced. This is true
likewise when the corporation is merely an adjunct, a business conduit or an alter ego of another
corporation. The corporate mask may be lifted and the corporate veil may be pierced when a corporation
is but the alter ego of a person or another corporation. It is clear from the foregoing that P.V. Pajarillo
Liner Inc. was a mere continuation and successor of the sole proprietorship of Panfilo. It is also quite
obvious that Panfilo transformed his sole proprietorship into a family corporation in a surreptitious
attempt to evade the charges of respondent union. Given these considerations, Panfilo and P.V. Pajarillo
Liner Inc. should be treated as one and the same person for purposes of liability.

PETRON CORPORATION AND PETER C. MALIGRO


vs.
NATIONAL LABOR RELATIONS COMMISSION AND CHITO S. MANTOS
G.R. No. 154532. October 27, 2006

FACTS

On May 15, 1990, Petron, through its Cebu District Office, hired the herein private respondent
Chito S. Mantos, an Industrial Engineer, as a managerial, professional and technical employee with
initial designation as a Bulk Plant Engineering Trainee. He attained regular employment status on
November 15, 1990 and was later on designated as a Bulk Plant Relief Supervisor, remaining as such

58 | P a g e
Law 321_Corporation LAW_ Case Digest

for the next five years while being assigned to the different plants and offices of Petron within the
Visayas area.
It was while assigned at Petron’s Cebu District Office with petitioner Peter Maligro as his
immediate superior, when Mantos, thru a Notice of Disciplinary Action was suspended for 30 days from
November 1 to 30, 1996 for violating company rules and regulations regarding AWOL, not having
reported for work during the period August 5 to 27, 1996. Subsequently, his services was terminated
effective December 1, 1996, by reason of his continued absences from August 28, 1996 onwards, as well
as for Insubordination/Discourtesy for making false accusations against his superior. Meanwhile,
contending that he has been constructively dismissed as of August 5, 1996, Mantos filed with the NLRC-
RAB, Cebu City, a complaint for illegal dismissal.

ISSUES:

Whether or not Maligro is solidarily liable with Petron.

RULING:

NO.

The NLRC erred in holding petitioner Peter Maligro jointly and severally liable with petitioner
Petron for the money claims of the private respondent.
Settled is the rule in this jurisdiction that a corporation is invested by law with a legal personality
separate and distinct from those acting for and in its behalf and, in general, from the people comprising
it. Thus, obligations incurred by corporate officers acting as corporate agents are not theirs but the
direct accountabilities of the corporation they represent. True, solidary liabilities may at times be
incurred by corporate officers, but only when exceptional circumstances so warrant.
In the present case, the apparent basis for the NLRC in holding petitioner Maligro solidarily liable
with Petron were its findings that (1) the Investigation Committee was created a day after the summons
in NLRC RAB was received, with Maligro no less being the chairman thereof; and (2) the basis for the
charge of insubordination was the private respondent’s alleged making of false accusations against
Maligro.
Those findings, however, cannot justify a finding of personal liability on the part of Maligro
inasmuch as said findings do not point to Maligro’s extreme personal hatred and animosity with the
respondent. It cannot, therefore, be said that Maligro was motivated by malice and bad faith in
connection with private respondent’s dismissal from the service.

CHINA BANKING CORPORATION


vs.
DYNE-SEM ELECTRONICS CORPORATION
G.R. No. 149237. June 11, 2006

FACTS:

On June 19 and 26, 1985, Dynetics, Inc. (Dynetics) and Elpidio O. Lim borrowed a total of
P8,939,000 from petitioner China Banking Corporation evidenced by six promissory notes. The
borrowers failed to pay when the obligations became due prompting the petitioner to institute a
complaint for sum of money against them. Summons was not served on Dynetics, however, because it
had already closed down. An amended complaint was filed by petitioner impleading respondent Dyne-
Sem Electronics Corporation (Dyne-Sem) and its stockholders Vicente Chuidian, Antonio Garcia and
Jacob Ratinoff. According to petitioner, respondent was formed and organized to be Dynetics alter ego
as established by the following circumstances: (a) Dynetics, Inc. and respondent are both engaged in the
same line of business of manufacturing, producing, assembling, processing, importing, exporting,
buying, distributing, marketing and testing integrated circuits and semiconductor devices; (b) the
principal office and factory site of Dynetics, Inc. located at Avocado Road, FTI Complex, Taguig, Metro

59 | P a g e
Law 321_Corporation LAW_ Case Digest

Manila, were used by respondent as its principal office and factory site; (c) respondent acquired some of
the machineries and equipment of Dynetics, Inc. from banks which acquired the same through
foreclosure; (d) respondent retained some of the officers of Dynetics, Inc.

ISSUE:

Whether the Doctrine of Piercing the Veil of Corporate Fiction is applicable in the present case.

RULING:

YES.

The general rule is that a corporation has a personality separate and distinct from that of its
stockholders and other corporations to which it may be connected. This is a fiction created by law for
convenience and to prevent injustice. Nevertheless, being a mere fiction of law, peculiar situations or
valid grounds may exist to warrant the disregard of its independent being and the piercing of the
corporate veil.
The veil of separate corporate personality may be lifted when such personality is used to defeat public
convenience, justify wrong, protect fraud or defend crime; or used as a shield to confuse the legitimate
issues; or when the corporation is merely an adjunct, a business conduit or an alter ego of another
corporation or where the corporation is so organized and controlled and its affairs are so conducted as
to make it merely an instrumentality, agency, conduit or adjunct of another corporation; or when the
corporation is used as a cloak or cover for fraud or illegality, or to work injustice, or where necessary to
achieve equity or for the protection of the creditors. In such cases, the corporation will be considered as
a mere association of persons. The liability will directly attach to the stockholders or to the other
corporation. To disregard the separate juridical personality of a corporation, the wrongdoing must be
proven clearly and convincingly.
In this case, petitioner failed to prove that Dyne-Sem was organized and controlled, and its affairs
conducted, in a manner that made it merely an instrumentality, agency, conduit or adjunct of Dynetics,
or that it was established to defraud Dynetics creditors, including petitioner.

MARUBENI CORPORATION, RYOICHI TANAKA, RYOHEI KIMURA


and SHOICHI ONE
vs.
FELIX LIRAG
G.R. No. 130998. August 10, 2001

FACTS:

Marubeni Corporation is a foreign corporation organized under the laws of Japan. It was doing
business in the Philippines through its duly licensed, wholly owned subsidiary, Marubeni Philippines
Corporation. Petitioners Ryoichi Tanaka, Ryohei Kimura and Shoichi One were officers of Marubeni
assigned to its Philippine branch.
On January 27, 1989, Lirag filed with the RTC of Makati a complaint for specific performance and
damages in the sum of P6M as commission pursuant to an oral consultancy agreement with Marubeni
for obtaining government contracts of various projects. Lirag claimed that on February 2, 1987,
petitioner Ryohei Kimura hired his consultancy group for the purpose of obtaining government contracts
of various projects. The agreement was merely oral because of the mutual trust between Marubeni and
the Lirag family which dates back to the 1960s. One of the projects handled by respondent Lirag, the
Bureau of Post project, amounting to P100,000,000.00 was awarded to the “Marubeni-Sanritsu tandem.
Despite repeated demands of his 6% commission was never paid.
Marubeni claimed that Ryohei Kimura did not have the authority to enter into such agreement in
their behalf. Only the general manager, upon issuance of a SPA by the principal office in Tokyo, Japan,
could enter into any contract in behalf of the corporation. They also claimed that Marubeni never
participated in the Bureau of Post project nor benefited from such project.

ISSUE:

60 | P a g e
Law 321_Corporation LAW_ Case Digest

Whether or not there was a consultancy agreement to make Lirag entitled to commission.

RULING:

NO.

The only basis of Lirag in claiming from Marubeni was because he claims that they are sister
companies since Marubeni was the supplier and contractor of the Sanritsu. Not because two foreign
companies came from the same country and closely worked together on certain projects would the
conclusion arise that one was the conduit of the other, thus piercing the veil of corporate fiction.
The separate personality of the corporation may be disregarded only when the corporation is used
as a cloak or cover for fraud or illegality, or to work injustice, or where necessary for the protection of
creditors. Aside from the self-serving testimony of respondent regarding the existence of a close working
relationship between Marubeni and Sanritsu, there was nothing that would support the conclusion that
Sanritsu was an agent of Marubeni.
Any agreement entered into because of the actual or supposed influence which the party has,
engaging him to influence executive officials in the discharge of their duties, which contemplates the use
of personal influence and solicitation rather than an appeal to the judgment of the official on the merits
of the object sought is contrary to public policy. Consequently, the agreement, assuming that the
parties agreed to the consultancy, is null and void as against public policy. Therefore, it is
unenforceable before a court of justice.

ADALIA B. FRANCISCO and MERRYLAND DEVELOPMENT CORPORATION


vs.
RITA C. MEJIA, as Executrix of ANDREA CORDOVA VDA. DE GUTIERREZ
G.R. No. 141617. August 14, 2001

FACTS:

Gutierrez was the registered owner of a parcel of land which was later subdivided into five lots. In
1964, Gutierrez and Cardale Financing and Realty Corporation executed a Deed of Sale with Mortgage
relating to the four of the five lots for the consideration of P800,000.00. Upon the execution of the deed,
Cardale paid Gutierrez P171,000.00. To secure payment of the balance of the purchase price, Cardale
constituted a mortgage on three of the four parcels of land.
In 1968, owing to Cardale's failure to settle its mortgage obligation, Gutierrez filed a complaint for
rescission of the contract with the Quezon City Regional Trial Court. In 1969, during the pendency of
the rescission case, Gutierrez died and was substituted by her executrix, respondent Rita C. Mejia.
In the meantime, the mortgaged parcels of land became delinquent in the payment of real estate
taxes, which culminated in their levy and auction sale in satisfaction of the tax arrears. The highest
bidder for the three parcels of land was petitioner Merryland Development Corporation, whose President
and majority stockholder is Francisco.

ISSUES:

Whether or not the corporate fiction of Cardale will be pierced.


Whether or not the corporate entity of Merryland must be pierced.

RULING:

YES.

Under the doctrine of piercing the veil of corporate entity, when valid grounds therefore exist, the
legal fiction that a corporation is an entity with a juridical personality separate and distinct from its
members or stockholders may be disregarded. In such cases, the corporation will be considered as a
mere association of persons. The members or stockholders of the corporation will be considered as the
corporation, that is, liability will attach directly to the officers and stockholders. The doctrine applies
when the corporate fiction is used to defeat public convenience, justify wrong, protect fraud, or defend
crime, or when it is made as a shield to confuse the legitimate issues, or where a corporation is the
merealter ego or business conduit of a person, or where the corporation is so organized and controlled

61 | P a g e
Law 321_Corporation LAW_ Case Digest

and its affairs are so conducted as to make it merely an instrumentality, agency, conduit or adjunct of
another corporation.

NO.

Merryland cannot be solidarily liable with Francisco. The only act imputable to Merryland in
relation to the mortgaged properties is that it purchased the same and this by itself is not a fraudulent
or wrongful act. No evidence has been adduced to establish that Merryland was a mere alter ego or
business conduit of Francisco. Time and again it has been reiterated that mere ownership by a single
stockholder or by another corporation of all or nearly all of the capital stock of a corporation is not of
itself sufficient ground for disregarding the separate corporate personality. Neither has it been alleged or
proven that Merryland is so organized and controlled and its affairs are so conducted as to make it
merely an instrumentality, agency, conduit or adjunct of Cardale. Even assuming that the businesses of
Cardale and Merryland are interrelated, this alone is not justification for disregarding their separate
personalities, absent any showing that Merryland was purposely used as a shield to defraud creditors
and third persons of their rights. Thus, Merryland's separate juridical personality must be upheld.
PHILIPPINE NATIONAL BANK & NATIONAL SUGAR DEVELOPMENT CORPORATION (NASUDECO)
vs.
ANDRADA ELECTRIC & ENGINEERING COMPANY
GR No. 142936. April 17, 2002

FACTS:

Respondent is a partnership duly organized, existing, and operating under the laws of the
Philippines is a semi-government corporation duly organized, existing and operating under the laws of
the Philippines; whereas, NASUDECO is also a semi-government corporation and the sugar arm of the
PNB; and the defendant Pampanga Sugar Mills (PASUMIL in short), is a corporation organized, existing
and operating under the 1975 laws of the Philippines; that the plaintiff is engaged in the business of
general construction for the repairs and/or construction of different kinds of machineries and buildings.
On August 26, 1975, PNB acquired the assets of the defendant PASUMIL that were earlier
foreclosed by the DBP. PNB organized the defendant NASUDECO in September, 1975, to take ownership
and possession of the assets and ultimately to nationalize and consolidate its interest in other PNB
controlled sugar mills; that prior to October 29, 1971, the defendant PASUMIL engaged the services of
defendant for electrical rewinding and repair, most of which were partially paid by the defendant
PASUMIL, leaving several unpaid accounts with the plaintiff; that finally, on October 29, 1971, the
plaintiff and the defendant PASUMIL entered into a construction contract.
The defendant PASUMIL and the defendant PNB, and now the defendant NASUDECO, failed and
refused to pay the plaintiff their just, valid and demandable obligation based on the contract. Defendant
prayed that judgment be rendered against the defendants PNB, NASUDECO, and PASUMIL.

ISSUE:

Whether or not the Veil of Corporate Fiction should be pierced in this case.

RULING:

NO.

The absence of the elements in the present case precludes the piercing of the corporate veil.
First, other than the fact that petitioners acquired the assets of PASUMIL, there is no showing that their
control over it warrants the disregard of corporate personalities. Second, there is no evidence that their
juridical personality was used to commit a fraud or to do a wrong; or that the separate corporate entity
was farcically used as a mere alter ego, business conduit or instrumentality of another entity or person.
Third, respondent was not defrauded or injured when petitioners acquired the assets of PASUMIL.
Being the party that asked for the piercing of the corporate veil, respondent had the burden of
presenting clear and convincing evidence to justify the setting aside of the separate corporate
personality rule. However, it utterly failed to discharge this burden; it failed to establish by competent
evidence that petitioner’s separate corporate veil had been used to conceal fraud, illegality or inequity.

62 | P a g e
Law 321_Corporation LAW_ Case Digest

AZCOR MANUFACTURING INC., FILIPINAS PASO and/or ARTURO ZULUAGA/Owner


vs.
NATIONAL LABOR RELATIONS COMMISSION (NLRC) AND CANDIDO CAPULSO
G.R. No. 117963. February 11, 1999

FACTS:

Candido Capluso has been working for petitioner for more than 12 years as a ceramics worker.
On February 1991, Capulso requested to go on sick leave, it appearing that his illness was directly
caused by his occupation. Upon recovering, Capulso was not allowed to resume work and was not
reinstated after having tried five times. He filed a complaint for constructive illegal dismissal and illegal
deduction against AZCOR and Arturo Zuluaga.
AZCOR moved to dismiss the complaint alleging that no employer- employee relationship existed.
Petitioner further added that Capulso became an employee of Fil Paso on March 1990 but voluntarily
resigned after a year as evidenced by a letter of resignation allegedly tendered by Capulso.
The Labor Arbiter dismissed the complaint for lack of merit and ordered AZCOR to refund the
deducted salaries. On Appeal, the NLRC ruled that the Contract of Employment stated that the work to
be done by Capulso was with Fil Paso and added the fact that the latter denied having executed and
signed the said resignation letters. Pending the trial of AZCOR’s petition for Certiorari, Capulso
succumbed to asthma and heart disease.

ISSUE:

Whether the petitioners are jointly liable for backwages in favor of the heirs being separate and
distinct entities.

RULING:

YES.

Capulso was led into believing that while he was working with Filipinas Paso, his real employer
was AZCOR. Petitioners never dealt with him openly and in good faith, nor was he informed of the
developments within the company, i.e., his alleged transfer to Filipinas Paso and the closure of AZCOR's
manufacturing operations beginning 1 March 1990. AZCOR manifested for the first time before the
Court that it had already ceased its business operations. Understandably, Capulso sued AZCOR alone
and was constrained to implead Filipinas Paso as additional respondent only when it became apparent
that the latter also appeared to be his employer.
In the case, the corporate fiction was used as a means to perpetrate a social injustice or as a
vehicle to evade obligations or confuse the legitimate issues. Such corporate fiction would be discarded
and the two (2) corporations would be merged as one, the first being merely considered as the
instrumentality, agency, conduit or adjunct of the other.

EDUARDO CLAPAROLS, ROMULO AGSAM and/or


CLAPAROLS STEEL AND NAIL PLANT
vs.

63 | P a g e
Law 321_Corporation LAW_ Case Digest

COURT OF INDUSTRIAL RELATIONS, ALLIED WORKERS' ASSOCIATION


and/or DEMETRIO GARLITOS, et al.
G.R. No. L-30822. July 31, 1975

FACTS:

In a case filed by private respondents against petitioners for unfair labor practices, CIR held
petitioners liable for reinstatement and back wages from the date of their dismissal up to their actual
reinstatement. Motion for execution was granted and an examination of petitioners’ payrolls and other
records for the computation of the back wages. When respondents returned to work, the company
accountant refused on the ground that there was no order from the plant owner.
This was on the ground that the records of Claparols Steel Corp. (CSC) show that it was
established on July 1, 1957 succeeding the CSNP which ceased operations on June 30, 1957, and that
the CSC stopped operation on Dec. 7, 1962. Petitioners filed an opposition alleging that they cannot
personally reinstate respondents because of the present status of the corporation; back wages should
only be limited to 3 months; and that since it ceased to operate on Dec. 7, 1962, reinstatement should
only be up to that date. Respondents opposed and alleged, among others, that CSNP and CSC is one
and the same corporation controlled by petitioner Claparols, with the latter corporation succeeding the
former.

ISSUE:

Whether or not CSNP and CSC is one and the same corporation.

RULING:

YES.

Respondent Court’s findings that indeed the CSNP, which ceased operation in June 30, 1957,
was succeeded by the CSC effective next day, July 1, 1957 up top December 7, 1962, when the latter
finally ceased to operate, were not disputed by petitioners. It is very clear that the latter was a
continuation and successor of the first entity, and its emergence was skillfully timed to avoid the
financial liability that already attached to its predecessor. Both corporations were owned and controlled
by petitioner Eduardo Claparols and there was no break in the succession and continuity of the same
business. This “avoiding-the-liability” scheme is very patent, considering that 90% of the subscribed
shares of the CSC were owned by Claparols himself, and all the assets of the dissolved CSNP were
turned over to the CSC.

COMMISSIONER OF INTERNAL REVENUE


vs.
NORTON and HARRISON COMPANY
G.R. No. L-17618. August 31, 1964

FACTS:

64 | P a g e
Law 321_Corporation LAW_ Case Digest

Norton and Harrison is a corporation organized to carry on and conduct a general wholesale and
retail mercantile establishment in the Philippines. Jackbilt is, likewise, a corporation organized
primarily for the purpose of making, producing and manufacturing concrete blocks. Norton and Jackbilt
entered into an agreement whereby Norton was made the sole and exclusive distributor of concrete
blocks manufactured by Jackbilt. Pursuant to this agreement, whenever an order for concrete blocks
was received by the Norton & Harrison Co. from a customer, the order was transmitted to Jackbilt
which delivered the merchandise direct to the customer.
Apparently, due to this transaction, the CIR, after conducting an investigation, assessed the
respondent Norton & Harrison for deficiency sales tax making as basis thereof the sales of Norton to the
Public. As Norton and Harrison did not conform with the assessment, the matter was brought to the
Court of Tax Appeals.
The Commissioner of Internal Revenue contends that since Jackbilt was owned and controlled by
Norton & Harrison, the corporate personality of Jackbilt should be disregarded for sales tax purposes,
and the sale of Jackbilt blocks by petitioner to the public must be considered as the original sales from
which the sales tax should be computed. The Norton & Harrison Company contended otherwise -that is,
the transaction subject to tax is the sale from Jackbilt to Norton.

ISSUE:

Whether or not the corporate personality of Norton and Jackbilt be disregarded.

RULING:

YES.

It has been settled that the ownership of all the stocks of a corporation by another corporation
does not necessarily breed an identity of corporate interest between the two companies and be
considered as a sufficient ground for disregarding the distinct personalities. However, in the case at bar,
we find sufficient grounds to support the theory that the separate identities of the two companies should
be disregarded. Among these circumstances are: Norton and Harrison owned all the outstanding stocks
of Jackbilt; of the 15,000 authorized shares of Jackbilt, 14,993 shares belonged to Norton and Harrison
and one each to seven others; Norton constituted Jackbilt's board of directors in such a way as to
enable it to actually direct and manage the other's affairs by making the same officers of the board for
both companies; Norton financed the operations of the Jackbilt, and this is shown by the fact that the
loans obtained from the RFC and Bank of America were used in the expansion program of Jackbilt, to
pay advances for the purchase of equipment, materials rations and salaries of employees of Jackbilt and
other sundry expenses; Norton treats Jackbilt employees as its own. Evidence shows that Norton paid
the salaries of Jackbilt employees and gave the same privileges as Norton employees, an indication that
Jackbilt employees were also Norton's employees. Furthermore services rendered in any one of the two
companies were taken into account for purposes of promotion; Compensation given to board members of
Jackbilt, indicate that Jackbilt is merely a department of Norton. The offices of Norton and Jackbilt are
located in the same compound. Payments were effected by Norton of accounts for Jackbilt and vice
versa. Payments were also made to Norton of accounts due or payable to Jackbilt and vice versa.
Hence, the corporate personality of the two corporations must be disregarded.

CONCEPT BUILDERS, INC.


vs.
THE NATIONAL LABOR RELATIONS COMMISSION
G.R. No. 108734. May 29, 1996

FACTS:

Petitioner, a domestic corporation, with principal office at 355 Maysan Road, Valenzuela, Metro
Manila, is engaged in the construction business. Private respondents were employed by said company as
laborers, carpenters and riggers.
Eventually, respondent’s services were terminated. The Labor Arbiter then rendered judgment
ordering petitioner to reinstate private respondents and to pay them back wages. A writ of execution was
then issued but was partially satisfied because the sheriff reported all the employees inside petitioner's
premises at 355 Maysan Road, Valenzuela, Metro Manila, claimed that they were employees of Hydro
Pipes Philippines, Inc and not by respondent. Subsequently, a certain Dennis Cuyegkeng filed a third-

65 | P a g e
Law 321_Corporation LAW_ Case Digest

party claim with the Labor Arbiter alleging that the properties sought to be levied upon by the sheriff
were owned by Hydro (Phils.), Inc. of which he is the Vice-President.
Private respondents filed a "Motion for Issuance of a Break-Open Order," alleging that HPPI and
petitioner corporation were owned by the same incorporator/stockholders. They also alleged that
petitioner temporarily suspended its business operations in order to evade its legal obligations to them
and that private respondents were willing to post an indemnity bond to answer for any damages which
petitioner and HPPI may suffer because of the issuance of the break-open order.

ISSUE:

Whether or not petitioner corporation and HPPI are one and the same.

RULING:

YES.

It is a fundamental principle of corporation law that a corporation is an entity separate and


distinct from its stockholders and from other corporations to which it may be connected. But, this
separate and distinct personality of a corporation is merely a fiction created by law for convenience and
to promote justice. So, when the notion of separate juridical personality is used to defeat public
convenience, justify wrong, protect fraud or defend crime, or is used as a device to defeat the labor laws,
this separate personality of the corporation may be disregarded or the veil of corporate fiction pierced.
This is true likewise when the corporation is merely an adjunct, a business conduit or an alter ego of
another corporation.
The test in determining the applicability of the doctrine of piercing the veil of corporate fiction is
as follows: a. Control, not mere majority or complete stock control, but complete domination, not only of
finances but of policy and business practice in respect to the transaction attacked so that the corporate
entity as to this transaction had at the time no separate mind, will or existence of its own; b. Such
control must have been used by the defendant to commit fraud or wrong, to perpetuate the violation of a
statutory or other positive legal duty or dishonest and unjust act in contravention of plaintiff's legal
rights; and c.The aforesaid control and breach of duty must proximately cause the injury or unjust loss
complained of.
HPPI is obviously a business conduit of Petitioner Corporation and its emergence was skillfully
orchestrated to avoid the financial liability that already attached to Petitioner Corporation.

COMPLEX ELECTRONICS EMPLOYEES ASSOCIATION (CEEA)


vs.
THE NATIONAL LABOR RELATIONS COMMISSION
G.R. No. 121315. July 19, 1999

FACTS:

Complex was engaged in the manufacture of electronic products. It was actually a subcontractor
of electronic products where its customers gave their job orders, sent their own materials and consigned
their equipment to it. Thus, there was the AMS Line for the Adaptive Micro System, Inc., the Heril Line
for Heril Co., Ltd., the Lite-On Line for the Lite-On Philippines Electronics Co., etc.
The rank and file workers of Complex were organized into a union known as the Complex
Electronics Employees Association, herein referred to as the Union.
Due to its financial reverses, Complex regretfully informed the employees that it was left with no
alternative but to close down the operations of the Lite-On Line. The Union on the other hand filed a
notice of strike with the NCMB. In the evening of April 6, 1992, the machinery, equipment and materials
being used for production at Complex were pulled-out from the company premises and transferred to
the premises of Ionics at Cabuyao, Laguna. The following day, a total closure of company operation was
effected at Complex.
A complaint was, thereafter, filed with the Labor Arbitration Branch of the NLRC for unfair labor
practice. Ionics was impleaded as a party defendant because the officers and management personnel of
Complex were also holding office at Ionics with Lawrence Qua as the President of both companies.

66 | P a g e
Law 321_Corporation LAW_ Case Digest

Complex, on the other hand, averred that since the time the Union filed its notice of strike, there
was a significant decline in the quantity and quality of the products in all of the production lines.
Fearful that the machinery, equipment and materials would be rendered inoperative and unproductive
due to the impending strike of the workers, the customers ordered their pull-out and transfer to Ionics.
Ionics contended that it was an entity separate and distinct from Complex and had been in existence 8
years before the labor dispute arose at Complex. While admitting that Lawrence Qua, the President of
Complex was also the President of Ionics, the latter denied having Qua as their owner since he had no
recorded subscription of P1,200,00.00 in Ionics as claimed by the Union.

ISSUE:

Whether or not Complex and Ionics are one and the same.

RULING:

YES.

Ionics may be engaged in the same business as that of Complex, but this fact alone is not enough
reason to pierce the veil of corporate fiction of the corporation. Well-settled is the rule that a corporation
has a personality separate and distinct from that of its officers and stockholders. This fiction of
corporate entity can only be disregarded in certain cases such as when it is used to defeat public
convenience, justify wrong, protect fraud, or defend crime. To disregard said separate juridical
personality of a corporation, the wrongdoing must be clearly and convincingly established.
As to the additional documentary evidence which consisted of a newspaper clipping filed by
petitioner Union, we agree with respondent Ionics that the photo/newspaper clipping itself does not
prove that Ionics and Complex are one and the same entity. The photo/newspaper clipping merely
showed that some plants of Ionics were recertified to ISO 9002 and does not show that there is a
relation between Complex and Ionics except for the fact that Lawrence Qua was also the president of
Ionics. However, as we have stated above, the mere fact that both of the corporations have the same
president is not in itself sufficient to pierce the veil of corporate fiction of the two corporations.
ROSAURA P. CORDON
vs.
JESUS BALICANTA
A.C. No. 2797. October 4, 2002

FACTS:

When her husband died, herein complainant Rosaura Cordon and her daughter Rosemarie
inherited the properties left by the said decedent. All in all, complainant and her daughter inherited 21
parcels of land located in Zamboanga City. The lawyer who helped her settle the estate of her late
husband was respondent Jesus Balicanta.
Respondent enticed complainant and her daughter to organize a corporation that would develop
the said real properties into a high-scale commercial complex. Relying on these apparently sincere
proposals, complainant and her daughter assigned 19 parcels of land to Rosaura Enterprises,
Incorporated, a newly-formed and duly registered corporation in which they assumed majority
ownership. The subject parcels of land were then registered in the name of the corporation.
Thereafter, respondent single-handedly ran the affairs of the corporation in his capacity as
Chairman of the Board, President, General Manager and Treasurer. The respondent also made
complainant sign a document which turned out to be a voting trust agreement. Respondent likewise
succeeded in making complainant sign a special power of attorney to sell and mortgage some of the
parcels of land she inherited from her deceased husband. She later discovered that respondent
transferred the titles of the properties to a certain Tion Suy Ong who became the new registered owner
thereof. Respondent never accounted for the proceeds of said transfers.
In 1981, respondent, using a spurious board resolution, contracted a loan from the Land Bank of
the Philippines in the amount of P2,220,000 using as collateral 9 of the real properties that the
complainant and her daughter contributed to the corporation. The respondent ostensibly intended to
use the money to construct the Baliwasan Commercial Center.

ISSUE:

Whether or not responden’s acts will bind the Petitioners.

RULING:

67 | P a g e
Law 321_Corporation LAW_ Case Digest

NO.

This Court confirms the duly supported findings of the IBP Board that respondent committed
condemnable acts of deceit against his client. The fraudulent acts he carried out against his client
followed a well thought of plan to misappropriate the corporate properties and funds entrusted to him.
At the very outset, he embarked on his devious scheme by making himself the President, Chairman of
the Board, Director and Treasurer of the corporation, although he knew he was prohibited from
assuming the position of President and Treasurer at the same time.
Also, respondent denies that he acted as Corporate Secretary aside from being the Chairman,
President and Treasurer of the corporation. Yet respondent submitted to the investigating commission
documents which were supposed to be in the official possession of the Corporate Secretary alone such
as the stock and transfer book and minutes of meetings.
After a thorough review of the records, we find that respondent committed grave and serious
misconduct that casts dishonor on the legal profession. His misdemeanors reveal a deceitful scheme to
use the corporation as a means to convert for his own personal benefit properties left to him in trust by
complainant and her daughter. Based on the aforementioned findings, this Court believes that the
gravity of respondent’s offenses cannot be adequately matched by mere suspension as recommended by
the IBP. Instead, his wrongdoings deserve the severe penalty of disbarment, without prejudice to his
criminal and civil liabilities for his dishonest acts.

DELPHER TRADES CORPORATION, and DELPHIN PACHECO


vs.
INTERMEDIATE APPELLATE COURT
G.R. No. L-69259. January 26, 1988

FACTS:

Delfin Pacheco and his sister, Pelagia Pacheco, were the owners of real estate property. The said
co-owners leased to Construction Components International Inc. the same property and providing that
during the existence or after the term of this lease the lessor should he decide to sell the property leased
shall first offer the same to the lessee and the letter has the priority to buy under similar conditions.
Subsequently, lessee assigned its rights and obligations under the contract of lease in favor of Hydro
Pipes Philippines, Inc.
A deed of exchange was executed between Delfin and Pelagia Pacheco and defendant Delpher
Trades Corporation whereby the former conveyed to the latter the leased property for 2,500 shares of
stock of defendant corporation with a total value of P1,500,000.00. On the ground that it was not given
the first option to buy the leased property pursuant to the proviso in the lease agreement, respondent
Hydro Pipes Philippines, Inc., filed an amended complaint for reconveyance of the property in its favor
under conditions similar to those whereby Delpher Trades Corporation acquired the property from
Pelagia Pacheco and Delphin Pacheco.
Respondents on the other hand stated that there was no transfer of ownership over the
properties.

ISSUE:

Whether or not there was an effective transfer of property in this case.

RULING:

NO.

After incorporation, one becomes a stockholder of a corporation by subscription or by purchasing


stock directly from the corporation or from individual owners thereof. In the case at bar, in exchange for
their properties, the Pachecos acquired 2,500 original unissued no par value shares of stocks of the
Delpher Trades Corporation. Consequently, the Pachecos became stockholders of the corporation by
subscription "The essence of the stock subscription is an agreement to take and pay for original
unissued shares of a corporation, formed or to be formed.” It is significant that the Pachecos took no par
value shares in exchange for their properties.
It is to be stressed that by their ownership of the 2,500 no par shares of stock, the Pachecos have
control of the corporation. Their equity capital is 55% as against 45% of the other stockholders, who
also belong to the same family group.

68 | P a g e
Law 321_Corporation LAW_ Case Digest

In effect, the Delpher Trades Corporation is a business conduit of the Pachecos. What they really
did was to invest their properties and change the nature of their ownership from unincorporated to
incorporated form by organizing Delpher Trades Corporation to take control of their properties and at
the same time save on inheritance taxes.
The "Deed of Exchange" of property between the Pachecos and Delpher Trades Corporation
cannot be considered a contract of sale. There was no transfer of actual ownership interests by the
Pachecos to a third party. The Pacheco family merely changed their ownership from one form to another.
The ownership remained in the same hands. Hence, the private respondent has no basis for its claim of
a light of first refusal under the lease contract.

FRANCISCO V. DEL ROSARIO


vs.
NATIONAL LABOR RELATIONS COMMISSION
G.R. No. 85416. July 24, 1990

FACTS:

In POEA Case No. 85-06-0394, the POEA promulgated a decision dismissing the complaint for
money claims for lack of merit. The decision was appealed to the NLRC, which reversed the POEA
decision and ordered Philsa Construction and Trading Co., Inc., the recruiter and Arieb Enterprises, the
foreign employer to jointly and severally pay private respondent their salary differentials and vacation
leave benefits.
A writ of execution was issued by the POEA but it was returned unsatisfied as Philsa was no
longer operating and was financially incapable of satisfying the judgment. Private respondent moved for
the issuance of an alias writ against the officers of Philsa. This motion was opposed by the officers, led
by petitioner, the president and general manager of the corporation.
Petitioner appealed to the NLRC. On September 23, 1988, the NLRC dismissed the appeal on the
theory that the corporate personality of Philsa should be disregarded. According to the NLRC, Philsa
Construction & Trading Co., Inc. and Philsa International Placement & Services Corp are one and the
same because both corporations has the same set of directors and officers. Petitioner's motion for
reconsideration was denied.
Thus, this petition was filed, alleging that the NLRC gravely abused its discretion.

ISSUE:

Whether or not the NLRC acted with grave abuse of discretion.

RULING:

YES.

Under the law a corporation is bestowed juridical personality, separate and distinct from its
stockholders. But when the juridical personality of the corporation is used to defeat public convenience,
justify wrong, protect fraud or defend crime, the corporation shall be considered as a mere association of
persons and its responsible officers and/or stockholders shall be held individually liable. For the same
reasons, a corporation shall be liable for the obligations of a stockholder, or a corporation and its
successor-in-interest shall be considered as one and the liability of the former shall attach to the latter.
But for the separate juridical personality of a corporation to be disregarded, the wrongdoing must
be clearly and convincingly established. It cannot be presumed. Thus, at the time Philsa allowed its
license to lapse in 1985 and even at the time it was delisted in 1986, there was yet no judgment in favor
of private respondent. An intent to evade payment of his claims cannot therefore be implied from the
expiration of Philsa's license and its delisting. Likewise, substantial identity of the incorporators of the
two corporations does not necessarily imply fraud.
In this case, not only has there been a failure to establish fraud, but it has also not been shown
that petitioner is the corporate officer responsible for private respondent's predicament. It must be
emphasized that the claim for differentials and benefits was actually directed against the foreign
employer. Philsa became liable only because of its undertaking to be jointly and severally bound with
the foreign employer, an undertaking required by the rules of the POEA, together with the filing of cash

69 | P a g e
Law 321_Corporation LAW_ Case Digest

and surety bonds, in order to ensure that overseas workers shall find satisfaction for awards in their
favor.

FIRST PHILIPPINE INTERNATIONAL BANK


vs.
COURT OF APPEALS
G.R. No. 115849. January 24, 1996

FACTS:

In the course of its banking operations, the defendant Producer Bank of the Philippines acquired
six parcels of land. The original plaintiffs, Demetrio Demetria and Jose O. Janolo, wanted to purchase
the property and thus initiated negotiations for that purpose.
Negotiations happened between the parties. However, petitioner bank reneged their agreement
because it offered the same lot to different buyers. Plaintiffs then filed a suit for specific performance
with damages against the bank, its Manager Rivers and Acting Conservator Encarnacion. The basis of
the suit was that the transaction had with the bank resulted in a perfected contract of sale.
Subsequently, Henry L. Co, filed a motion to intervene in the trial court, alleging that as owner of
80% of the Bank's outstanding shares of stock, he had a substantial interest in resisting the complaint.
The trial court issued an order denying the motion to intervene on the ground that it was filed after trial
had already been concluded. Henry Co did not appeal the denial of his motion for intervention.
During the pendency of the proceedings in the Court of Appeals, Henry Co and several other
stockholders of the Bank, filed an action purportedly a "derivative suit" with the RTC Branch 134,
against Encarnacion, Demetria and Janolo to declare any perfected sale of the property as
unenforceable and to stop Ejercito from enforcing or implementing the sale. In his answer, Janolo
argued that the Second Case was barred by litis pendentia by virtue of the case then pending in the
Court of Appeals.

ISSUE:

Whether or not the juridical personalities of the two corporations be pierced.

RULING:

YES.

In addition to the many cases where the corporate fiction has been disregarded, we now add the
instant case, and declare herewith that the corporate veil cannot be used to shield an otherwise blatant
violation of the prohibition against forum-shopping. Shareholders, whether suing as the majority in
direct actions or as the minority in a derivative suit, cannot be allowed to trifle with court processes,
particularly where, as in this case, the corporation itself has not been remiss in vigorously prosecuting
or defending corporate causes and in using and applying remedies available to it. To rule otherwise
would be to encourage corporate litigants to use their shareholders as fronts to circumvent the stringent
rules against forum shopping.

FRANCISCO MOTORS

70 | P a g e
Law 321_Corporation LAW_ Case Digest

vs.
CA and SPOUSES GREGORIO and LIBRADA MANUEL
G.R. No. 100812. Jun 25, 1999

Facts:

This case arose from the decision o the trial court granting the counter claim of the herein private
respondents. Such counterclaim is based from the fact that Gregorio Manuel, while he was petitioner’s
Assistant Legal Officer, he represented members of the Francisco family in the intestate estate
proceedings of the late Benita Trinidad. However, even after the termination of the proceedings, his
services were not paid. Said family members, he said, were also incorporators, directors and officers of
petitioner. Hence to counter petitioner’s collection suit, he filed a permissive counterclaim for the unpaid
attorney’s fees.

ISSUE:

Whether or not the petitioner corporation is liable for the attorney’s fee owing to the respondents.

RULING:

NO.

Petitioner argued that being a corporation, it should not be held liable therefore because these
fees were owed by the incorporators, directors and officers of the corporation in their personal capacity
as heirs of Benita Trinidad. Petitioner stressed that the personality of the corporation, vis-à-vis the
individual persons who hired the services of private respondent, is separate and distinct, hence, the
liability of said individuals did not become an obligation chargeable against petitioner.
In this case, the piercing of the corporate veil was not applied because rationale behind piercing a
corporation’s identity in a given case is to remove the barrier between the corporation from the persons
comprising it to thwart the fraudulent and illegal schemes of those who use the corporate personality as
a shield for undertaking certain proscribed activities. However, in the case at bar, instead of holding
certain individuals or persons responsible for an alleged corporate act, the situation has been reversed.
It is the petitioner as a corporation which is being ordered to answer for the personal liability of certain
individual directors, officers and incorporators concerned. Furthermore, according to private respondent
Gregorio Manuel his services were solicited as counsel for members of the Francisco family to represent
them in the intestate proceedings over Benita Trinidad’s estate. These estate proceedings did not involve
any business of petitioner.
The personality of the corporation and those of its incorporators, directors and officers in their
personal capacities ought to be kept separate in this case. The claim for legal fees against the concerned
individual incorporators, officers and directors could not be properly directed against the corporation
without violating basic principles governing corporations.

SOL LAGUIO, RENE LAOLAO, ANNALIZA ENSANDO, EDELIZA ASAS, LILIA MARAY, EVELYN
UNTALAN,* ROSARIO CHICO, REYNALDO GARCIA, MERLITA DE LOS SANTOS,* JOSEPHINE
DERONG,* GEMMA TIBALAO BANTOLO, LUCY ALMONTE,* CRISPINA VANQUARDIA, NARCISA
VENZON, NORMA ELEGANTE,* AMELIA MORENO,* ABNER PETILOS, NARCISO HILAPO, DOLORES
OLAES, MELINDA LLADOC, ERNA AZARCON, and APRIL TOY, INC. WORKERS UNION – ALAB
vs.
NATIONAL LABOR RELATIONS COMMISSION, WELL WORLD TOYS, INC., APRIL TOYS, INC., YU
SHENG LING, JENN L. WANG, EUCLIFF CHENG, CHI SHENG LIN, NENITA C. AGUIRRE, MA.
THERESA R. CADIENTE and GLICERIA R. AGUIRRE

71 | P a g e
Law 321_Corporation LAW_ Case Digest

G.R. No. 108936. October 4, 1996

FACTS:

Private respondent April Toy, Inc. is a domestic corporation, for the purpose of "manufacturing,
importing, exporting, buying , selling, sub-contracting or otherwise dealing in, at wholesale and retail,"
stuffed toys. On December 20, 1989, or after almost a year of operation, April posted a memorandum 2
within its premises and circulated a copy of the same among its employees informing them of its dire
financial condition. April decided to shorten its corporate term "up to February 28, 1990,”
In view of April's cessation of operations, petitioners who initially composed of seventy-seven
employees below filed a complaint for "illegal shutdown/retrenchment/dismissal and unfair labor
practice." On June 21, 1990, petitioners amended their complaint to implead private respondent Well
World Toys, Inc. (Well World for brevity), a corporation also engaged in the manufacture of stuffed toys
for export.
Petitioners further alleged that the original incorporators and principal officers of April were
likewise the original incorporators of Well World, thus both corporations should be treated as one
corporation liable for their claims. The Labor Arbiter found as valid the closure of April, and treated
April and Well World as two distinct corporations.

ISSUE:

Whether or not April and Well World are two distinct corporations.

RULING:

YES.

The two corporations have two different set of officers managing their respective affairs in two
separate offices. It is basic that a corporation is invested by law with a personality separate and distinct
from those of the persons composing it as well as from that of any other legal entity to which it may be
related. Mere substantial identity of the incorporators of the two corporations does not necessarily imply
fraud, 15 nor warrant the piercing of the veil of corporate fiction. In the absence of clear and convincing
evidence that April and Well World's corporate personalities were used to perpetuate fraud, or
circumvent the law said corporations were rightly treated as distinct and separate from each other.

72 | P a g e
Law 321_Corporation LAW_ Case Digest

RUFINA LUY LIM


vs.
COURT OF APPEALS, AUTO TRUCK TBA CORPORATION, SPEED DISTRIBUTING, INC., ACTIVE
DISTRIBUTORS, ALLIANCE MARKETING CORPORATION, ACTION COMPANY, INC.
G.R. No. 124715, January 24, 2000

FACTS:

Petitioner Rufina Luy Lim is the surviving spouse of late Pastor Y. Lim whose estate is the subject
of probate proceedings. The respondent herein is the owner of the properties subject of this. Said
properties were included in the inventory of estate late Pastor Lim. Thus he respondents moved for the
exclusion of said properties which was denied by the trial court. Petitioner contended upon filing an
amended petition that the properties were actually owned by Pastor Lim and the same were registered
under his name, hence they should be included in the inventory of his estate, and that during his
lifetime, he organized and wholly-owned the five corporations, which are the private respondents in the
instant case.

ISSUE:

Whether or not the doctrine of piercing the corporate veil is applicable.

RULING:

NO.

The test in determining the applicability of the doctrine of piercing the veil of corporate fiction is
as follows: 1) Control, not mere majority or complete stock control, but complete domination, not only of
finances but of policy and business practice in respect to the transaction attacked so that the corporate
entity as to this transaction had at the time no separate mind, will or existence of its own; (2) Such
control must have been used by the defendant to commit fraud or wrong, to perpetuate the violation of a
statutory or other positive legal duty, or dishonest and unjust act in contravention of plaintiffs legal
right; and (3) The aforesaid control and breach of duty must proximately cause the injury or unjust loss
complained of. The absence of any of these elements prevents piercing the corporate veil.
In this case there is no showing that the elements are present. Furthermore, it was proven that
said properties were registered in the name of the corporation, hence the same were owned by the
corporation despite the fact that, assuming true, it was Pastor Lim who organized the corporation.

73 | P a g e
Law 321_Corporation LAW_ Case Digest

MATUGUINA INTEGRATED WOOD PRODUCTS, INC.


vs.
The HON. COURT OF APPEALS, DAVAO ENTERPRISES CORPORATION, The HON. MINISTER,
(NOW SECRETARY) of NATURAL RESOURCES AND PHILLIP CO.
G.R. No. 98310. October 24, 1996

FACTS:

On June 28, 1973, the Acting Director of the Bureau of Forest Development issued Provisional
Timber License (PTL) No. 30, covering an area of 5,400 hectares to Ms. Milagros Matuguina who was
then doing business under the name of MLE, a sole proprietorship venture. A portion, covering 1,900
hectares, of the said area was located within the territorial boundary of Gov. Generoso in Mati, Davao
Oriental, and adjoined the timber concession of Davao Enterprises Corporation (DAVENCOR), the
private respondent.
On July 17, 1975, Milagros Matuguina and petitioner MIWPI executed a Deed of Transfer 5
transferring all of the former's rights, interests, ownership and participation in Provincial Timber License
No. 30 to the latter for and in consideration of 148,000 shares of stocks in MIWPI.
On July 28, 1975, pending approval of the request to transfer the PTL to MIWPI, DAVENCOR,
through its Assistant General Manager, complained to the District Forester at Mati, Davao Oriental that
Milagros Matuguina/MLE had encroached into and was conducting logging operations in DAVENCOR's
timber concession.

ISSUE:

Whether or not MLE and MIWPI are separate and distinct corporations.

RULING:

YES.

It is settled that a corporation is clothed with personality separate and distinct from that of the
persons composing it. It may not generally be held liable for that of the persons composing it. It may not
be held liable for the personal indebtedness of its stockholders or those of the entities connected with it.
Conversely, a stockholder cannot be made to answer for any of its financial obligations even if he should
be its president. But when the juridical personality of the corporation is used to defeat public
convenience, justify wrong, protect fraud or defend crime, the corporation shall be considered as a mere
association of persons, and its responsible officers and/or stockholders shall be individually. For the
same reasons, a corporation shall be liable for the obligations of a stockholder, or a corporation and its
successor-in-interest shall be considered as one and the liability of the former shall attach to the latter.
But for the separate juridical personality of a corporation to be disregarded, the wrongdoing must
be clearly and convincingly established. It cannot be presumed.
In the case at bar, there is, insufficient basis for the appellate court's ruling that MIWPI is the
same as Matuguina. The alleged control of Plaintiff Corporation was not evident in any particular
corporate acts of Plaintiff Corporation, wherein Maria Milagros Matuguina Logging Enterprises is using
Plaintiff Corporation, executed acts or powers directly involving Plaintiff Corporation. Also, mere
ownership by a single stockholder or by another corporation of all or nearly all of the capital stocks of
the corporation, is not itself a sufficient warrant for disregarding the fiction of separate personality.

74 | P a g e
Law 321_Corporation LAW_ Case Digest

THE MANILA HOTEL CORP. AND MANILA HOTEL INTL. LTD.


vs.
NATIONAL LABOR RELATIONS COMMISSION, ARBITER CEFERINA J. DIOSANA AND MARCELO G.
SANTOS
G.R. No. 120077. October 13, 2000

FACTS:

MHICL is a corporation duly organized and existing under the laws of Hong Kong. MHC is an
“incorporator” of MHICL, owning 50% of its capital stock. By virtue of a “management agreement” with
the Palace Hotel (Wang Fu Company Limited), MHICL trained the personnel and staff of the Palace Hotel
at Beijing, China.
Respondent Santos accepted an employment offer from Palace Hotel. On November 5, 1988,
respondent Santos left for Beijing, China. He started to work at the Palace Hotel. A year later he received
a letter stating that his employment is being terminated due to business reverses brought about by the
political upheaval in China. On February 20, 1990, respondent Santos filed a complaint for illegal
dismissal.

ISSUE:

Whether or not the doctrine of piercing the corporate veil is available to make MHC liable for
damages.

RULING:

NO.

MHC, as a separate and distinct juridical entity cannot be held liable. True, MHC is an
incorporator of MHICL and owns fifty percent (50%) of its capital stock. However, this is not enough to
pierce the veil of corporate fiction between MHICL and MHC.
Piercing the veil of corporate entity is an equitable remedy. It is resorted to when the corporate
fiction is used to defeat public convenience, justify wrong, protect fraud or defend a crime. It is done
only when a corporation is a mere alter ego or business conduit of a person or another corporation.
In Traders Royal Bank v. Court of Appeals, the court held that “the mere ownership by a single
stockholder or by another corporation of all or nearly all of the capital stock of a corporation is not of
itself a sufficient reason for disregarding the fiction of separate corporate personalities.”
The tests in determining whether the corporate veil may be pierced are: First, the defendant must
have control or complete domination of the other corporation’s finances, policy and business practices
with regard to the transaction attacked. There must be proof that the other corporation had no separate
mind, will or existence with respect the act complained of. Second, control must be used by the
defendant to commit fraud or wrong. Third, the aforesaid control or breach of duty must be the
proximate cause of the injury or loss complained of. The absence of any of the elements prevents the
piercing of the corporate veil.
It is basic that a corporation has a personality separate and distinct from those composing it as
well as from that of any other legal entity to which it may be related. Clear and convincing evidence is
needed to pierce the veil of corporate fiction. In this case, the court found no evidence to show that
MHICL and MHC are one and the same entity.

75 | P a g e
Law 321_Corporation LAW_ Case Digest

SAN JUAN STRUCTURAL AND STEEL FABRICATORS, INC.


vs.
COURT OF APPEALS, MOTORICH SALES CORPORATION, NENITA LEE GRUENBERG, ACL
DEVELOPMENT CORP. and JNM REALTY AND DEVELOPMENT CORP.
G.R. No. 129459. September 29, 1998

FACTS:

A parcel of land was sold by Nenita Lee Gruenberg, the corporate treasurer of defendant
corporation Motorich Sale in favor of San Juan Structural and Steel Fabricators, Inc. However, the latter
failed to execute the necessary Transfer of Rights/Deed of Assignment in favor of plaintiff-appellant.
Hence a case for damages was filed. The defendant corporation questions the validity of the contract
entered by its treasurer in its behalf without authorization from the corporation’s Board.

ISSUE:

Whether or not the doctrine of piercing the veil of corporate fiction be applied to Motorich.

RULING:

NO.

The contract cannot bind Motorich, because it never authorized or ratified such sale. A
corporation is a juridical person separate and distinct from its stockholders or members. Accordingly,
the property of the corporation is not the property of its stockholders or members and may not be sold
by the stockholders or members without express authorization from the corporation’s board of directors.
The corporation may act only through its board of directors, or, when authorized either by its bylaws or
by its board resolution, through its officers or agents in the normal course of business. The general
principles of agency govern the relation between the corporation and its officers or agents, subject to the
articles of incorporation, bylaws, or relevant provisions of law.
As to the piercing of the corporate veil, the same is not applicable. In the present case, the Court
finds no reason to pierce the corporate veil of Respondent Motorich. Petitioner utterly failed to establish
that said corporation was formed, or that it is operated, for the purpose of shielding any alleged
fraudulent or illegal activities of its officers or stockholders; or that the said veil was used to conceal
fraud, illegality or inequity at the expense of third persons, like petitioner.

76 | P a g e
Law 321_Corporation LAW_ Case Digest

TAN BOON BEE & CO., INC.


vs.
THE HONORABLE HILARION U. JARENCIO, PRESIDING JUDGE OF BRANCH XVIII of the Court of
First Instance of Manila, GRAPHIC PUBLISHING, INC., and PHILIPPINE AMERICAN CAN DRUG
COMPANY
G.R. No. L-41337. June 30, 1988

FACTS:

Petitioner herein, doing business under the name and style of Anchor Supply Co., sold on credit
to herein private respondent Graphic Publishing, Inc. (GRAPHIC) paper products amounting to
P55,214.73. On December 20, 1972, GRAPHIC made partial payment by check to petitioner in the total
amount of P24,848.74; and on December 21, 1972, a promissory note was executed to cover the balance
of P30,365.99. In the said promissory note, it was stipulated that the amount will be paid on monthly
installments and that failure to pay any installment would make the amount immediately demandable
with an interest of 12% per annum. On September 6, 1973, for failure of GRAPHIC to pay any
installment, petitioner filed a complaint for collection of Sum of Money.
A decision was rendered and became final and executory, where one (1) unit printing machine
identified as "Original Heidelberg Cylinder Press" Type H 222, NR 78048, found in the premises of
GRAPHIC was levied. However, a third party claim was filed by Philippine American Drug Company
(PADCO), hence after trial the levy was rendered to be without force.

ISSUE:

Whether or not the properties of PADCO could be levied due to the allegation that it is mere an
adjunct or conduit of Graphic.

RULING:

YES.

In the instant case, petitioner's evidence established that PADCO was never engaged in the
printing business; that the board of directors and the officers of GRAPHIC and PADCO were the same;
and that PADCO holds 50% share of stock of GRAPHIC. Petitioner likewise stressed that PADCO's own
evidence shows that the printing machine in question had been in the premises of GRAPHIC since May,
1965, long before PADCO even acquired its alleged title on July 11, 1966 from Capitol Publishing. That
the said machine was allegedly leased by PADCO to GRAPHIC on January 24, 1966, even before PADCO
purchased it from Capital Publishing on July 11, 1966, only serves to show that PADCO's claim of
ownership over the printing machine is not only farce and sham but also unbelievable.
Considering the aforestated principles and the circumstances established in this case, respondent
judge should have pierced PADCO's veil of corporate identity.

77 | P a g e
Law 321_Corporation LAW_ Case Digest

TELEPHONE ENGINEERING & SERVICE COMPANY, INC.


vs.
WORKMEN'S COMPENSATION COMMISSION, PROVINCIAL SHERIFF OF RIZAL and LEONILA
SANTOS GATUS, for herself and in behalf of her minor children, Teresita, Antonina and Reynaldo,
all surnamed GATUS
G.R. No. L-28694. May 13, 1981

FACTS:

Utilities Management Corporation (UMACOR), the sister company of the petitioner hired the late
Pacifica L. Gatus as Purchasing Agent. The latter died due to liver cirrhosis with malignant
degeneration. His widow, respondent Leonila S. Gatus, filed a "Notice and Claim for Compensation"
Workmen's Compensation Section, alleging therein that her deceased husband was an employee of
TESCO, and that he died of liver cirrhosis.
TESCO, in its reply, contended that the cause of the illness contracted by Gatus was in no way
aggravated by the nature of his work. TESCO takes the position that there was no employer-employee
relationship between them, the deceased having been an employee of UMACOR and not of TESCO.

ISSUE:

Whether or not the contentions of TESCO is tenable.

RULING:

NO.

The court ruled that indeed TESCO is estopped from raising the defense of non-existence of
employer-employee relationship because such was raised only in the petition for the first time. It was
considered by the court as a mere afterthought to evade liability. It was also seen that in its initial
pleadings it did not deny that it is the employer of the decedent. Petitioner even admitted that TESCO
and UMACOR are sister companies operating under one single management and housed in the same
building. Although respect for the corporate personality as such, is the general rule, there are
exceptions. In appropriate cases, the veil of corporate fiction may be pierced as when the same is made
as a shield to confuse the legitimate issues.

78 | P a g e
Law 321_Corporation LAW_ Case Digest

BUENAFLOR C. UMALI, MAURICIA M. VDA. DE CASTILLO, VICTORIA M. CASTILLO, BERTILLA C.


RADA, MARIETTA C. ABAÑEZ, LEOVINA C. JALBUENA and SANTIAGO M. RIVERA
vs.
COURT OF APPEALS, BORMAHECO, INC. and PHILIPPINE MACHINERY PARTS MANUFACTURING
CO., INC.
G.R. No. 89561. September 13, 1990

FACTS:

Santiago Rivera is the nephew of plaintiff Mauricia Meer Vda. de Castillo. The Castillo family is
the owners of a parcel of land located in Lucena City which was given as security for a loan from the
Development Banks of the Philippines. For their failure to pay the amortization, foreclosure of the said
property was about to be initiated. This problem was made known to Santiago Rivera, who proposed to
them the conversion into subdivision of the four (4) parcels of land adjacent to the mortgaged property
to raise the necessary fund. The idea was accepted by the Castillo family and to carry out the project, a
Memorandum of Agreement was executed by and between Slobec Realty and Development, Inc.,
represented by its President Santiago Rivera and the Castillo family. In this agreement, Santiago Rivera
obliged himself to pay the Castillo family the sum of P70,000.00 immediately after the execution of the
agreement and to pay the additional amount of P400,000.00 after the property has been converted into
a subdivision. Rivera, armed with the agreement, approached Mr. Modesto Cervantes, President of
defendant Bormaheco, and proposed to purchase from Bormaheco two (2) tractors Model D-7 and D-8.
Subsequently, a Sales Agreement was executed on December 28, 1970, which was accepted by the latter
and executed Sales Agreement. The balance of the consideration was secured by a surety bond from ICP
(Insurance Corporation of the Phil.) which was in turn secured by a mortagage, the properties of the
Castillos.

ISSUE:

Whether or not the doctrine of piercing the veil of corporate fiction is applicable.

RULING:

NO.

Petitioners seek to pierce the veil of corporate entity of Bormaheco, ICP and PM Parts, alleging
that these corporations employed fraud in causing the foreclosure and subsequent sale of the real
properties belonging to petitioners.
In the instant case, petitioners do not seek to impose a claim against the individual members of
the three corporations involved; on the contrary, it is these corporations which desire to enforce an
alleged right against petitioners. Assuming that petitioners were indeed defrauded by private
respondents in the foreclosure of the mortgaged properties, this fact alone is not, under the
circumstances, sufficient to justify the piercing of the corporate fiction, since petitioners do not intend to
hold the officers and/or members of respondent corporations personally liable therefore. Petitioners are
merely seeking the declaration of the nullity of the foreclosure sale, which relief may be obtained without
having to disregard the aforesaid corporate fiction attaching to respondent corporations. Secondly,
petitioners failed to establish by clear and convincing evidence that private respondents were purposely
formed and operated, and thereafter transacted with petitioners, with the sole intention of defrauding
the latter.
The mere fact, therefore, that the businesses of two or more corporations are interrelated is not a
justification for disregarding their separate personalities, absent sufficient showing that the corporate
entity was purposely used as a shield to defraud creditors and third persons of their rights.

79 | P a g e
Law 321_Corporation LAW_ Case Digest

VLASON ENTERPRISES CORPORATION


vs.
COURT OF APPEALS and DURAPROOF SERVICES, represented by its General Manager, Cesar
Urbino Sr.
G.R. Nos. 121662-64. July 6, 1999

FACTS:

Poro Point Shipping Services, then acting as the local agent of Omega Sea Transport Company of
Honduras & Panama, a Panamanian company, (hereafter referred to as Omega), requested permission
for its vessel M/V Star Ace, which had engine trouble, to unload its cargo and to store it at the
Philippine Ports Authority (PPA) compound in San Fernando, La Union while awaiting transhipment to
Hongkong. The request was approved by the Bureau of Customs. Despite the approval, the customs
personnel boarded the vessel when it docked on January 7, 1989, on suspicion that it was the hijacked
M/V Silver Med owned by Med Line Philippines Co., and that its cargo would be smuggled into the
country. The district customs collector seized said vessel and its cargo pursuant to Section 2301, Tariff
and Customs Code.
They entered into a salvage agreement with private respondent to secure and repair the vessel
which was destroyed by the typhoons that hit the province at the agreed consideration of $1 million and
“fifty percent (50%) of the cargo after all expenses, cost and taxes.” Subsequently, the seizure was lifted
for want of fraud. To enforce its preferred salvor’s lien, herein Private Respondent Duraproof Services
filed with the Regional Trial Court of Manila a Petition for Certiorari, Prohibition and Mandamus
assailing the actions of Commissioner Mison and District Collector Sy.

ISSUE:

Whether or not the doctrine of piercing the corporate veil is applicable.

RULING:

NO.

In the present case, Bebero was the secretary of Angliongto, who was president of both VSI and
petitioner, but she was an employee of VSI, not of petitioner. The piercing of the corporate veil cannot
be resorted to when serving summons. Doctrinally, a corporation is a legal entity distinct and separate
from the members and stockholders who compose it. However, when the corporate fiction is used as a
means of perpetrating a fraud, evading an existing obligation, circumventing a statute, achieving or
perfecting a monopoly or, in generally perpetrating a crime, the veil will be lifted to expose the
individuals composing it. None of the foregoing exceptions has been shown to exist in the present case.
Quite the contrary, the piercing of the corporate veil in this case will result in manifest injustice.

80 | P a g e
Law 321_Corporation LAW_ Case Digest

VILLA REY TRANSIT, INC.


vs.
EUSEBIO E. FERRER, PANGASINAN TRANSPORTATION CO., INC. and PUBLIC SERVICE
COMMISSION
EUSEBIO E. FERRER and PANGASINAN TRANSPORTATION CO., INC.

PANGASINAN TRANSPORTATION CO., INC.


vs.
JOSE M. VILLARAMA
G.R. No. L-23893. October 29, 1968

FACTS:

Prior to 1959, Jose M. Villarama was an operator of a bus transportation, under the business
name of Villa Rey Transit, pursuant to certificates of public convenience granted him by the Public
Service Commission (PSC, for short) in Cases Nos. 44213 and 104651, which authorized him to operate
a total of thirty-two (32) units on various routes or lines from Pangasinan to Manila, and vice-versa. On
January 8, 1959, he sold the aforementioned two certificates of public convenience to the Pangasinan
Transportation Company, Inc. (Pantranco), for P350,000.00 with the condition, among others, that the
seller (Villarama) "shall not for a period of 10 years from the date of this sale, apply for any TPU service
identical or competing with the buyer."
Barely three months thereafter, or on March 6, 1959, a corporation called Villa Rey Transit, Inc.
was organized. In less than a month after its registration, it bought five certificates of public
convenience, forty-nine buses, tools and equipment from one Valentin Fernando. Before the PSC could
take final action on said application for approval of sale, however, the Sheriff of Manila, on July 7, 1959,
levied on two of the five certificates of public convenience involved therein pursuant to a writ of
execution issued by the Court of First Instance of Pangasinan in Civil Case No. 13798, in favor of
Eusebio Ferrer, plaintiff, judgment creditor, against Valentin Fernando, defendant, judgment debtor.
The Sheriff made and entered the levy in the records of the PSC. On July 16, 1959, a public sale was
conducted by the Sheriff of the said two certificates of public convenience.

ISSUE:

Whether or not the doctrine of piercing the corporate veil is applicable.

RULING:

YES.

The doctrine that a corporation is a legal entity distinct and separate from the members and
stockholders who compose it is recognized and respected in all cases which are within reason and the
law. 29 When the fiction is urged as a means of perpetrating a fraud or an illegal act or as a vehicle for
the evasion of an existing obligation, the circumvention of statutes, the achievement or perfection of a
monopoly or generally the perpetration of knavery or crime, the veil with which the law covers and
isolates the corporation from the members or stockholders who compose it will be lifted to allow for its
consideration merely as an aggregation of individuals.
Upon the foregoing considerations, the Court so held that the preponderance of evidence have
shown that the Villa Rey Transit, Inc. is an alter ego of Jose M. Villarama, and that the restrictive clause
in the contract entered into by the latter and Pantranco is also enforceable and binding against the said
Corporation. For the rule is that a seller or promissor may not make use of a corporate entity as a
means of evading the obligation of his covenant. Where the Corporation is substantially the alter ego of
the covenantor to the restrictive agreement, it can be enjoined from competing with the covenantee.

 De Facto Corporation

C. ARNOLD HALL and BRADLEY P. HALL, petitioners,


vs.
EDMUNDO S. PICCIO, Judge of the Court of First Instance of Leyte, FRED BROWN, EMMA
BROWN, HIPOLITA CAPUCIONG, in his capacity as receiver of the Far Eastern Lumber and
Commercial Co., Inc., respondents.
G.R. No. L-2598. June 29, 1950

81 | P a g e
Law 321_Corporation LAW_ Case Digest

FACTS:

In 1947, the petitioners and the respondents signed and acknowledged in Leyte, the article of
incorporation of the Far Eastern Lumber and Commercial Co., Inc., organized to engage in a general
lumber business to carry on as general contractors, operators and managers, etc. Attached to the article
was an affidavit of the treasurer stating that 23,428 shares of stock had been subscribed and fully paid
with certain properties transferred to the corporation described in a list appended thereto.
Immediately after the execution of said articles of incorporation, the corporation proceeded to do
business with the adoption of by-laws and the election of its officers.
In 1947, the said articles of incorporation were filed in the office of the SEC for the issuance of
the corresponding certificate of incorporation. Thereafter, pending action on the articles of incorporation
by the SEC, the respondents filed before the Court of First Instance of Leyte a civil case, alleging among
other things that the Far Eastern Lumber and Commercial Co. was an unregistered partnership; that
they wished to have it dissolved because of bitter dissension among the members, mismanagement and
fraud by the managers and heavy financial losses. The petitioners alleged that the court had no
jurisdiction over the civil case decree the dissolution of the company, because it being a de facto
corporation, dissolution thereof may only be ordered in a quo warranto proceeding instituted in
accordance with section 19 of the Corporation Law.

ISSUES:

Whether or not the Far Eastern Lumber and Commercial Co., Inc. is a de facto corporation.

RULING:

NO.

Inasmuch as the Far Eastern Lumber and Commercial Co., is a de facto corporation, section 19
of the Corporation Law applies, and therefore the court had not jurisdiction to take cognizance of said
civil case.
There are least two reasons why this section does not govern the situation. (1) First, not having
obtained the certificate of incorporation, the Far Eastern Lumber and Commercial Co. — even its
stockholders — may not probably claim "in good faith" to be a corporation.
Under our statue it is to be noted that it is the issuance of a certificate of incorporation by the
Director of the Bureau of Commerce and Industry (now SEC) which calls a corporation into being. The
immunity if collateral attack is granted to corporations "claiming in good faith to be a corporation under
this act." Such a claim is compatible with the existence of errors and irregularities; but not with a total
or substantial disregard of the law. Unless there has been an evident attempt to comply with the law the
claim to be a corporation "under this act" could not be made "in good faith."
(2) Second, this is not a suit in which the corporation is a party. This is a litigation between
stockholders of the alleged corporation, for the purpose of obtaining its dissolution. Even the existence
of a de jure corporation may be terminated in a private suit for its dissolution between stockholders,
without the intervention of the state.

 Corporation by Estoppel

INTERNATIONAL EXPRESS TRAVEL & TOUR SERVICES


vs.
HON. COURT OF APPEALS, HENRI KAHN, PHILIPPINE FOOTBALL FEDERATION
G.R. No. 119002. October 19, 2000

FACTS:

Petitioner International Express Travel and Tour Services, Inc., through its managing director,
wrote a letter to the Philippine Football Federation (Federation), through its president private respondent
Henri Kahn, wherein the former offered its services as a travel agency to the latter, which was accepted.
Petitioner secured the airline tickets for the trips of the athletes and officials of the Federation which
amounted to P449,654.83. For failure to pay the unpaid amount after demands, the petitioner filed a
collection case against Henri Kahn in his personal capacity and as President of the Federation and
impleaded the Federation as an alternative defendant. Kahn denied liability and averred that it merely
acted as the agent of the Federation and did not guaranty the payment of the purchased tickets. The
trial court ruled against Kahn.

82 | P a g e
Law 321_Corporation LAW_ Case Digest

ISSUE:

Whether or not Kahn is personally liable.

RULING:

YES.

Kahn avers that he should not be made personally liable because it should be the Federation, as
a corporation having juridical existence, which must be held liable. He merely acted as an agent of the
latter.
The Court was not persuaded. It ruled that under R.A. 3135, and the Department of Youth and
Sports Development under P.D. 604, for a Federation to acquire juridical existence it is a requirement
that the federation must be recognized by the accrediting organization, the Philippine Amateur Athletic
Federation. And Kahn failed to prove that such requirement was complied with by the Federation. It is a
settled principal in corporation law that any person acting or purporting to act on behalf of a
corporation which has no valid existence assumes such privileges and becomes personally liable for
contract entered into or for other acts performed as such agent. 1 As president of the Federation, Henri
Kahn is presumed to have known about the corporate existence or non-existence of the Federation.

83 | P a g e
Law 321_Corporation LAW_ Case Digest

LIM TONG LIM


vs.
PHILIPPINE FISHING GEAR INDUSTRIES, INC.
1999 Nov 3, G.R. No. 136448

FACTS:

On behalf of "Ocean Quest Fishing Corporation," Antonio Chua and Peter Yao entered into a
Contract for the purchase of fishing nets of various sizes from the Philippine Fishing Gear Industries,
Inc. They claimed that they were engaged in a business venture with Petitioner Lim Tong Lim, who
however was not a signatory to the agreement. They, however, failed to pay; hence, private respondent
filed a collection suit against Chua, Yao and Petitioner Lim Tong Lim with a prayer for a writ of
preliminary attachment. The suit was brought against the three in their capacities as general partners,
on the allegation that "Ocean Quest Fishing Corporation" was a nonexistent corporation
Yao and Chua admitted liability while Lim filed his answer. Trial court rendered decision ruling
that Philippine Fishing Gear Industries was entitled to the Writ of Attachment and that Chua, Yao and
Lim, as general partners, were jointly liable to pay respondent.

ISSUE:

Whether or not Lim should be made jointly liable with Yao and Chua.

RULING:

YES.

Lim asserts that he should not be made liable because there was no partnership existing between
them.
The court ruled that there exist a partnership between them. It is clear that Chua, Yao and Lim
had decided to engage in a fishing business, which they started by buying boats worth P3.35 million,
financed by a loan secured from Jesus Lim who was petitioner's brother. In their Compromise
Agreement, they subsequently revealed their intention to pay the loan with the proceeds of the sale of
the boats, and to divide equally among them the excess or loss. These boats, the purchase and the
repair of which were financed with borrowed money, fell under the term "common fund" under Article
1767. The contribution to such fund need not be cash or fixed assets; it could be an intangible like
credit or industry. That the parties agreed that any loss or profit from the sale and operation of the
boats would be divided equally among them also shows that they had indeed formed a partnership.
Moreover, it is clear that the partnership extended not only to the purchase of the boat, but also
to that of the nets and the floats. The fishing nets and the floats, both essential to fishing, were
obviously acquired in furtherance of their business. It would have been inconceivable for Lim to involve
himself so much in buying the boat but not in the acquisition of the aforesaid equipment, without which
the business could not have proceeded.

84 | P a g e
Law 321_Corporation LAW_ Case Digest

MARIANO A. ALBERT
vs.
UNIVERSITY PUBLISHING CO., INC.
G.R. No. L-19118, January 30, 1965

FACTS:

In the original case, the court had awarded P P15,000.00 in favor of the petitioner for damages
arising out of a breach of contract. Such breach of contract arose when the publishing company failed to
pay the petitioner the agreed amount for latter to have the exclusive right to publish his revised
Commentaries on the Revised Penal Code and for his share in previous sales of the book's first edition.
The order became final and executory. A writ of execution was issued against the company, however the
petitioner petitioned for a writ of execution against Jose M. Aruego, as the real defendantstating,
plaintiff's counsel and the Sheriff of Manila discovered that there is no such entity as University
Publishing Co., Inc. and no such entity is registered with the SEC.
This case asks the court whether or not the judgment may be executed against Jose M. Aruego,
supposed President of University Publishing Co., Inc., as the real defendant.

ISSUE:

Whether or not the judgment may be executed against Jose M. Aruego, supposed President of
University Publishing Co., Inc., as the real defendant.

RULING:

NO.

The Court ruled that the doctrine of corporation by estoppel was not applicable. Although the rule
is that a person acting or purporting to act on behalf of a corporation which has no valid existence
assumes such privileges and obligations and becomes personally liable for contracts entered into or for
other acts performed as such agent, in this case, Aruego was not named as a defendant. Since he was
not named, he could not be served and be made liable for the claim because to do so would violate his
right to due process. He was not given the chance to defend himself and be heard during trial.
Wherefore, the order was reversed and set aside and was remanded lower court to hold
supplementary proceedings for the purpose of carrying the judgment into effect against University
Publishing Co., Inc. and/or Jose M. Aruego.

 Non-User of Charter vs. Continuous Inoperation

LOYOLA GRAND VILLAS HOMEOWNERS (SOUTH) ASSOCIATION, INC.


vs.
HON. COURT OF APPEALS
1997 Aug 7, G.R. No. 117188

85 | P a g e
Law 321_Corporation LAW_ Case Digest

FACTS:

LGVHAI was organized as the association of homeowners and residents of the Loyola Grand
Villas. It was registered with the Home Financing Corporation. For unknown reasons, however, LGVHAI
did not file its corporate by-laws. Sometime in 1988, the officers of the LGVHAI tried to register its by-
laws. They failed to do so. They later discovered that there were two other organizations within the
subdivision the North Association and the South Association. According to private respondents, a non-
resident and Soliven himself respectively headed these associations. They also discovered that these
associations had five (5) registered homeowners each who were also the incorporators, directors and
officers thereof. None of the members of the LGVHAI was listed as member of the North Association
while three (3) members of LGVHAI were listed as members of the South Association. When they
inquired as to the status of LGVHAI, the head of the legal department of the HIGC, informed him that
LGVHAI had been automatically dissolved for two reasons. First, it did not submit its by-laws within the
period required by the Corporation Code and, second, there was non-user of corporate charter because
HIGC had not received any report on the association's activities. These prompted the LGVHAI to lodge
complaint with HIGC questioning its act of revoking its certificate of registration without due notice and
hearing and concomitantly prayed for the cancellation of the certificates of registration of the North and
South Associations by reason of the earlier issuance of a certificate of registration in favor of LGVHAI.

ISSUE:

Whether or not the failure of a corporation to file its by-laws within one month from the date of its
incorporation, as mandated by Section 46 of the Corporation Code, result in its automatic dissolution.

RULING:

NO.

Although the Corporation Code requires the filing of by-laws, it does not expressly provide for the
consequences of the non-filing of the same within the period provided for in Section 46. Even under the
express grant of power and authority under Presidential Decree No. 902-A, there can be no automatic
corporate dissolution simply because the incorporators failed to abide by the required filing of by-laws
embodied in Section 46 of the Corporation Code. There is no outright "demise" of corporate existence.
Proper notice and hearing are cardinal components of due process in any democratic institution, agency
or society. In other words, the incorporators must be given the chance to explain their neglect or
omission and remedy the same.

BOARD OF DIRECTORS
 Qualifications/Qualifying Shares

REP. LUIS R. VILLAFUERTE, et al.


vs.
GOV. OSCAR S. MORENO, et al.
G.R. No. 186566, October 2, 2009

FACTS:

As a result of the Tokyo Communique, which unified the feuding Basketball Association of the
Philippines ("BAP") and the newly formed Pilipinas Basketbol ("PB"), the Samahang Basketbol ng
Pilipinas, Inc. ("SBP") was established and its constitutive documents consisting of the Articles of
Incorporation were signed by the five (5) incorporators, which include petitioner Pangilinan. On the
same day, the incorporators likewise passed and signed its by-laws. Subsequently, the three-man panel

86 | P a g e
Law 321_Corporation LAW_ Case Digest

met in Bangkok, Thailand where it forged and executed a Memorandum of Agreement ("Bangkok
Agreement") integrating therein the final terms and conditions of the unity and merger of BAP and PB.
Then came the nomination and election of its transitory officers for the years 2007-2008 the results of
which had led to the proclamation of respondent Villafuerte as Chairman. Petitioner raised its
opposition and did not recognize the election of respondent Villafuerte as Chairman of BAP-SBP on
account of the alleged failure of the latter to qualify for the said position. As a result of this, two
elections were held by the different factions for the positions in the Board of Trustees. Petitioners filed
before the Regional Trial Court of Manila a petition 5 for declaration of nullity of the election of
respondents as members of the Board of Trustees and Officers of BAP-SBP. The trial court rendered
decision in favor of the petitioners.

ISSUE:

Whether or not Villafuerte is qualified as a Director.

RULING:

NO.

Respondents asserted that Villafuerte never assumed the position of Chairman of the BAP-SBP
because he failed to qualify for the same; that before Villafuerte could legally assume the Chairmanship
of BAP-SBP, he must first be elected a member of the Board of Trustees.
As correctly pointed out by CA, petitioner Villafuerte’s nomination must of necessity be
understood as being subject to or in accordance with the qualifications set forth in the By-Laws of the
BAP-SBP. Since the said by-laws require the Chairman of the Board of Trustees to be a trustee himself,
petitioner Villafuerte was not qualified since he had neither been elected nor appointed as one of the
trustees of BAP-SBP. In other words, petitioner Villafuerte never validly assumed the position of
Chairman because he failed in the first place to qualify therefore.

87 | P a g e
Law 321_Corporation LAW_ Case Digest

CONSTANCIO T. BAGUIO
vs.
LAS PALMAS INTERNATIONAL MANPOWER CORPORATION, SPOUSES DONALDO PALMA AND
CONSUELO P. PALMA and CYNTHIA C. CALAPRE
G.R. No. 93417, September 14, 1993

FACTS:

Petitioner claims that he bought 600 shares of stocks from the respondent corporation for the
amount of P60, 000.00. He further avers that the respondents failed, for such a long time, to deliver the
certificates of stocks corresponding to the stocks he bought. He thus sought for the reimbursement of
return of the P60, 000.00 he allegedly paid to the corporation. The respondents denied receiving any
payment from the petitioner. Petitioner forwards the fact that the corporation had adopted a resolution
recognizing him as being a stockholder owning 600 shares, and was further appointed as vice-president,
hence the respondents cannot deny the fact that they have received the payment. There was no receipt
presented because when petitioner asked for one, respondents Palmas assured him that the board
resolution and the secretary's certificate were better evidence of payment than an ordinary receipt. He
was likewise told that the stock certificate would be issued in December 1982, after the board meeting.
Respondents Palmas used the money to pay their employees, whose salaries had not been paid for
several months. The trial court ruled for the petitioner, but the CA reversed the same.

ISSUE:

Whether or not petitioner actually paid respondent Palmas the sum of P60,000.00, the price of
the shares of stock sold to him.

RULING:

NO.

The court ruled that there was no payment. The resolution adopted by the Board does not speak
of any sales transaction and receipt of payment. It merely states that the petitioner was accepted as a
stockholder to the corporation. Even assuming that a transaction between the petitioners and the
spouses Palma transpired, the corporation had nothing to do with the business transactions entered by
its officers in their personal capacity and petitioner.
Furthermore, petitioner cannot claim that being a member of the board of directors and
occupying the position of Vice-President-International necessarily imply that he must have owned duly-
paid shares of stock. The election of a person to the board of directors of a corporation does not
necessarily mean that he has paid for the shares recorded in his name. In most cases, nominee directors
do not pay for the qualifying shares assigned to them. Likewise, the Corporation Code does not require
that one elected or appointed as vice-president of a corporation should be the owner of shares of stock of
the corporation.

88 | P a g e
Law 321_Corporation LAW_ Case Digest

DETECTIVE & PROTECTIVE BUREAU, INC.


vs.
HON. CLORIBEL and FAUSTO S. ALBERTO
1968 Nov 29, G.R. No. L-23428

FACTS:

Plaintiff filed against herein private defendant a complaint for accounting with preliminary
injunction and receivership. It alleged that defendant was managing director of plaintiff corporation from
1952 until January 14, 1964; that in June 1963, defendant illegally seized and took control of all the
assets as well as the books, records, vouchers and receipts of the corporation from the accountant-
cashier, concealed them illegally and refused to allow any member of the corporation to see and examine
the same; that on January 14, 1964, the stockholders, in a meeting, removed defendant as managing
director and elected Jose de la Rosa in his stead; that defendant not only had refused to vacate his office
and to deliver the assets and books to Jose de la Rosa, but also continued to perform unauthorized acts
for and in behalf of plaintiff corporation; that defendant had been required to submit a financial
statement and to render an accounting of his administration from 1952 but defendant has failed to do
so; and that it continued disposing properties of the corporation contrary to a Board resolution.
The writ of preliminary injunction was granted upon posting a bond. However, the respondent
filed a counter-bond which was granted, and the order for preliminary injunction was lifted. Hence,
petition for certiorari under Rule 65 was filed.

ISSUE:
Whether or not a writ of preliminary injunction against respondent should be granted.

RULING:

NO.

Petitioner contended that respondent Alberto had arrogated to himself the powers of the Board of
Directors of the corporation because he refused to vacate the office and surrender the same to Jose de la
Rosa who had been elected managing director by the Board to succeed him. This assertion, however,
was disputed by respondent Alberto who stated that Jose de la Rosa could not be elected managing
director because he did not own any stock in the corporation.
The Court ruled that there is in the record no showing that Jose de la Rosa owned a share of
stock in the corporation. If he did not own any share of stock, certainly he could not be a director
pursuant to the mandatory provision of Section 30 of the Corporation Law, which in part provides: "Sec.
30. Every director must own in his own right at least one share of the capital stock of the stock
corporation of which he is a director, which stock shall stand in his name on the books of the
corporation."
If the managing director-elect was not qualified to become managing director, respondent Fausto
Alberto could not be compelled to vacate his office and cede the same to the managing director-elect
because the by-laws of the corporation provides in Article IV, Section 1 that "Directors shall serve until
the election and qualification of their duly qualified successor."

89 | P a g e
Law 321_Corporation LAW_ Case Digest

GRACE CHRISTIAN HIGH SCHOOL


vs.
THE COURT OF APPEALS, GRACE VILLAGE ASSOCIATION, INC., ALEJANDRO G. BELTRAN, and
ERNESTO L. GO
1997 Oct 23, G.R. No. 108905

FACTS:

Petitioner Grace Christian High School is an educational institution offering preparatory,


kindergarten and secondary courses at the Grace Village in Quezon City. Private respondent Grace
Village Association, Inc., on the other hand, is an organization of lot and/or building owners, lessees
and residents at Grace Village, while private respondents Alejandro G. Beltran and Ernesto L. Go were
its president and chairman of the committee on election. For 15 years the petitioner had been occupying
a permanent seat in the Board of Directors of the respondent. However, the latter decided to “reexamine”
the right of petitioner's representative to continue as an unelected member of the board. As the board
denied petitioner's request to be allowed representation without election, petitioner brought an action for
mandamus in the Home Insurance and Guaranty Corporation. Its action was dismissed by the hearing
officer whose decision was subsequently affirmed by the appeals board. Petitioner appealed to the Court
of Appeals, which in turn upheld the decision of the HIGC's appeals board. Hence this petition for
review.

ISSUE:

Whether or not the petitioner has acquired a vested right to be a permanent director in the
association under the drafted by –laws, but which were not submitted to the members for approval.

RULING:

NO.

The present Corporation Code states that the board of directors of corporations must be elected
from among the stockholders or members. There may be corporations in which there are unelected
members in the board but it is clear that in the examples cited by petitioner the unelected members sit
as ex officio members, i.e., by virtue of and for as long as they hold a particular office. But in the case of
petitioner, there is no reason at all for its representative to be given a seat in the board. Nor does
petitioner claim a right to such seat by virtue of an office held. In fact it was not given such seat in the
beginning. It was only in 1975 that a proposed amendment to the by-laws sought to give it one.
Since the provision in question is contrary to law, the fact that for fifteen years it has not been
questioned or challenged but, on the contrary, appears to have been implemented by the members of
the association cannot forestall a later challenge to its validity. Neither can it attain validity through
acquiescence because, if it is contrary to law, it is beyond the power of the members of the association to
waive its invalidity. For that matter the members of the association may have formally adopted the
provision in question, but their action would be of no avail because no provision of the by-laws can be
adopted if it is contrary to law.
Also, petitioner cannot claim a vested right to sit in the board on the basis of "practice." Practice,
no matter how long continued, cannot give rise to any vested right if it is contrary to law. Even less
tenable is petitioner's claim that its right is "coterminus with the existence of the association."

90 | P a g e
Law 321_Corporation LAW_ Case Digest

RAMON C. LEE and ANTONIO DM. LACDAO


vs.
THE HON. COURT OF APPEALS, SACOBA MANUFACTURING CORP., PABLO GONZALES, JR. and
THOMAS GONZALES
G.R. No. 93695, February 4, 1992

FACTS:

A complaint for a sum of money was filed by the International Corporate Bank, Inc. against the
private respondents who, in turn, filed a third party complaint against ALFA and the petitioners. The
trial court denied the motion to dismiss the 3 rd party complaint filed by petitioners and ordered the
respondents to serve summons to ALFA. Initially the summons was served to ALFA through the DBP as
a consequence of the petitioner's letter informing the court that the summons for ALFA was erroneously
served upon them considering that the management of ALFA had been transferred to the DBP. On the
other hand, the DBP claimed that it was not authorized to receive summons on behalf of ALFA since the
DBP had not taken over the company which has a separate and distinct corporate personality and
existence. Private respondents filed a Manifestation and Motion for the Declaration of Proper Service of
Summons which the trial court granted, and which was opposed by the petitioners contending that
there was improper service of summons because they were no longer officers of ALFA by virtue of a
voting trust agreement.

ISSUE:

Whether or not the petitioners are correct.

RULING:

YES.

The petitioners argue that by virtue of the voting trust agreement the petitioners can no longer be
considered directors of ALFA. They cited that to be directors, the Corporation Code requires that it must
own at least 1 one (1) share of the capital stock of the corporation of which he is a director which share
shall stand in his name on the books of the corporation. The voting trust agreement effectively
transferred to DBP, as the trustee, legal ownership of the stock covered by the agreement and the latter
became the stockholder of record with respect to the said shares of stocks. Since the petitioners no
longer had in their names even a single share in the corporation, they ceased to be qualified as
directors, hence they are no longer authorized to receive summons. Being so, the service of summons
upon the petitioners was invalid.

 Disqualifications

ENRIQUE P. BRIAS Y ROXAS


vs.
JOHN S. HORD, et al.

91 | P a g e
Law 321_Corporation LAW_ Case Digest

1913 Feb 5, G.R. No. 8387

FACTS:

The petitioner was a duly elected member of the Board of BPI. When he requested before Hord,
the President of the company, an examination of the books and finances of the company, the same was
denied, even after repeated demands. Thereafter, he alleged that the respondents made it appear that
the petitioner had tendered a resignation and declared that his position was vacant. Hence the latter
filed this complaint demanding that he be reinstated from his former office.

ISSUE:

Whether or not the petitioner is entitled to the relief sought.

HELD:

YES.

Based from the documentary and testimonial evidence there is no clear showing that the
petitioner had actually resigned. The testimonies of the respondents posed several and fatal
inconsistencies while the testimony of the petitioner more or less proves what really transpired during
the meeting. With these, the petitioner is still entitled to his position and his request for examination of
the corporate books must be granted.

 Voting

WOLFGANG AURBACH, et al.


vs.
SANITARY WARES MANUFACTURING CORPORATION, et al.
1989 Dec 15, G.R. No. 75875

FACTS:

92 | P a g e
Law 321_Corporation LAW_ Case Digest

In 1961, Saniwares, a domestic corporation was incorporated for the primary purpose of
manufacturing and marketing sanitary wares. One of the incorporators, Mr. Baldwin Young went abroad
to look for foreign partners, European or American who could help in its expansion plans.
ASI, a foreign corporation domiciled in Delaware, United States entered into an Agreement with
Saniwares and some Filipino investors whereby ASI and the Filipino investors agreed to participate in
the ownership of an enterprise which would engage primarily in the business of manufacturing in the
Philippines and selling here and abroad vitreous china and sanitary wares. The parties agreed that the
business operations in the Philippines shall be carried on by an incorporated enterprise and that the
name of the corporation shall initially be "Sanitary Wares Manufacturing Corporation."
The conflict arose when there had dissentions from ASI for the proposed export expansion by the
other stockholders. When the next annual election of the Board came, further conflicts arose on the
manner of voting, it resulted to the uncertainty as to who were duly elected. The contending groups of
Lagdameo Group and ASI Group claim claimed to be the legitimate directors of the corporation.

ISSUE:

Whether or not Petitioners were the duly elected members of the Board.

HELD:

NO.

The Court ruled that Wolfgang Aurbach, John Griffin, David P Whittingham, Ernesto V.
Lagdameo, Baldwin Young, Raul A. Boncan, Ernesto R. Lagdameo, Jr., Enrique Lagdameo, and George
F. Lee as the duly elected directors of Saniwares at the March 8, 1983 annual stockholders’ meeting
were the duly elected members of the Board. Under their agreement, both parties were given the right
their shares cumulatively. ASI, however, should not be allowed to interfere in the voting within the
Filipino group. Otherwise, ASI would be able to designate more than the three directors it is allowed to
designate under the Agreement, and may even be able to get a majority of the board seats, a result
which is clearly contrary to the contractual intent of the parties. The foreign Group (ASI) was limited to
designate three directors . This is the allowable participation of the ASI Group. Hence, in future
dealings, this limitation of six to three board seats should always be maintained as long as the joint
venture agreement exists considering that in limiting 3 board seats in the 9-man board of directors there
are provisions already agreed upon and embodied in the parties' Agreement to protect the interests
arising from the minority status of the foreign investors.

BATAAN SHIPYARD & ENGINEERING CO., INC. (BASECO)


vs.
PRESIDENTIAL COMMISSION ON GOOD GOVERNMENT, et al.
1987 May 27, G.R. No. 75885

FACTS:

This case arose from a sequestration order issued by the PCGG under authority given by the
president. Such sequestration order was sent and received by petitioner. Pursuant to this sequestration
orders, take over orders were also issued to protect public interest and to prevent the disposal or
dissipation of business enterprises and properties taken over by the government of the Marcos
Administration or by entities or persons close to former President Marcos, until the transactions leading
to such acquisition by the latter can be disposed of by the appropriate authorities. However, among
other facts, the petitioner questions the exercise of PCGGs right of ownership and management when it
terminated several contracts without the consent of both parties, to enter contracts, and to operate its
quarry business, and especially its right ot vote during stockholders’ meetings.

ISSUE:

93 | P a g e
Law 321_Corporation LAW_ Case Digest

Whether or not PCGG may vote in stockholders’ meetings.

RULING:

YES.

PCGG may properly exercise the prerogative to vote sequestered stock of corporations, granted to
it by the President of the Philippines through a Memorandum dated June 26, 1986. That Memorandum
authorizes the PCGG, pending the outcome of proceedings to determine the ownership of sequestered
shares of stock, to vote such shares of stock as it may have sequestered in corporations at all
stockholders' meetings called for the election of directors, declaration of dividends, amendment of the
Articles of Incorporation, etc. Moreover, in the case at bar, there was adequate justification to vote the
incumbent directors out of office and elect others in their stead because the evidence showed prima facie
that the former were just tools of President Marcos and were no longer owners of any stock in the firm, if
they ever were at all.

94 | P a g e
Law 321_Corporation LAW_ Case Digest

 Report on Election

PREMIUM MARBLE RESOURCES, INC.


vs.
THE COURT OF APPEALS and INTERNATIONAL CORPORATE BANK
1996 Nov 4, G.R. No. 96551

FACTS:

Herein petitioner filed a case for damages against respondent for allowing clearance of checks by
unauthorized officers of the former, to the former’s prejudice. However this case was opposed by some
members of the petitioner on the ground that the filing of the complaint was not authorized by the
Board. Hence, a resolution of this case was necessary to litigate the claim of the petitioner for damages
against the respondent bank.

ISSUE:

Whether or not the filing was authorized by a duly constituted Board of Directors of the petitioner
corporation.

RULING:

NO.

The petitioners asserted that the Board authorized such filing. However, from the records of the
case as well as that of the corporation, no evidence was seen and shown that the results of the election
where the supposed members of the Board who allegedly authorized the filing were filed with the
Securities and Exchange Commission. The Corporation Code mandates that within thirty (30) days after
the election of the directors, trustees and officers of the corporation, the secretary, or any other officer of
the corporation, shall submit to the Securities and Exchange Commission, the names, nationalities and
residences of the directors, trustees and officers elected. Failure to comply with such requirement, the
elected members cannot be considered as the duly constituted and elected members of the Board.
Hence, being not duly constituted, the filing of the case was not authorized by the Board.

 Term of Office/Holdover

DR. HANS CHRISTIAN M. SEÑERES


vs.

95 | P a g e
Law 321_Corporation LAW_ Case Digest

COMMISSION ON ELECTIONS and MELQUIADES A. ROBLES


G.R. No. 178678, April 16, 2009

FACTS:

Private respondent Robles was elected president and chairperson of Buhay, a party-list group
duly registered with COMELEC. The constitution of BUHAY provides for a three-year term for all its
party officers, without re-election. Robles again signed and filed a Certificate of Nomination of BUHAY’s
nominees for the 2007 elections, however such certificate was denied by petitioner alleging that he was
the acting president and secretary-general of BUHAY, having assumed that position since August 17,
2004 when Robles vacated the position. Señeres further claimed that the nominations made by Robles
were, for lack of authority, null and void owing to the expiration of the latter’s term as party president.
On May 10, 2007, the National Council of BUHAY adopted a resolution expelling Señeres as party
member for his act of submitting a Certificate of Nomination for the party. Subsequently, Robles was
adjudged as the duly authorized representative of Buhay. Aggrieved, petitioner filed this complaint.

ISSUE:

Whether or not respondent Robles is the duly authorized representative of BUHAY.

RULING:

YES.

Petitioner Señeres maintains that at the time the Certificate of Nomination was submitted, Robles’
term as President of BUHAY had already expired, thus effectively nullifying the Certificate of Nomination
and the nomination process.
The Court was mot amenable. As a general rule, officers and directors of a corporation hold over
after the expiration of their terms until such time as their successors are elected or appointed. Sec. 23 of
the Corporation Code contains a provision to this effect, thus: the board of directors or trustees to be
elected from among the holders of stocks, or where there is no stock, from among the members of the
corporation, who shall hold office for one (1) year until their successors are elected and qualified.
The holdover doctrine accords validity to what would otherwise be deemed as dubious corporate
acts and gives continuity to a corporate enterprise in its relation to outsiders. The voting members of
BUHAY duly elected Robles as party President in October 1999. And although his regular term as such
President expired in October 2002, no election was held to replace him and the other original set of
officers. Further, the constitution and by-laws of BUHAY do not expressly or impliedly prohibit a hold-
over situation. As such, since no successor was ever elected or qualified, Robles remained the President
of BUHAY in a "hold-over" capacity.

 How Removed

LEON J. LAMBERT
vs.
T. J. FOX
1914 Jan 29, G.R. No. 7991

FACTS:

Due to financial crisis the petitioner and the defendant were able to acquire the bulk of the stocks
of John R. Edgar & Co. as the latter’s creditors. Hence, upon incorporating said company, the parties
entered into an agreement that either of them will not sell or transfer their respective shares till after
one year from the date of agreement. However, less than a year, defendant Fox sold his stock in the said

96 | P a g e
Law 321_Corporation LAW_ Case Digest

corporation to E. D. McCullough of the firm of E. C. McCullough & Co. of Manila, a strong competitor of
the said John R. Edgar & Co., Inc. This sale was made by the defendant against the protest of the
plaintiff and with the warning that he would be held liable under the contract hereinabove set forth and
in accordance with its terms. In fact, the defendant Fox offered to sell his shares of stock to the plaintiff
for the same sum that McCullough was paying for them less P1, 000, the penalty specified in the
contract.
The trial Court rendered judgment in favor of defendant.

ISSUE:

Whether or not the stipulation not to sell is valid.

RULING:

YES.

The suspension of the power to sell has a beneficial purpose, results in the protection of the
corporation as well as of the individual parties to the contract, and is reasonable as to the length of time
of the suspension.
The intention of parties to a contract must be determined, in the first instance, from the words of
the contract itself. It is to be presumed that persons mean what they say when they speak plain English.
Interpretation and construction should by the instruments last resorted to by a court in determining
what the parties agreed to. Where the language used by the parties is plain, then construction and
interpretation are unnecessary and, if used, result in making a contract for the parties.
In this jurisdiction, there is no difference between a penalty and liquidated damages, so far as
legal results are concerned. Whatever differences exists between them as a matter of language, they are
treated the same legally. In either case the party to whom payment is to be made is entitled to recover
the sum stipulated without the necessity of proving damages. Indeed one of the primary purposes in
fixing a penalty or in liquidating damages is to avoid such necessity.

 How Vacancy Filled

VALLE VERDE COUNTRY CLUB, INC., et al.


vs.
VICTOR AFRICA
G.R. No. 151969, September 4, 2009

FACTS:

During the Annual Stockholders’ Meeting of petitioner Valle Verde Country Club, Inc. (VVCC), the
following were elected as members of the VVCC Board of Directors: Ernesto Villaluna, Jaime C.
Dinglasan, Eduardo Makalintal, Francisco Ortigas III, Victor Salta, Amado M. Santiago, Jr., Fortunato
Dee, Augusto Sunico, and Ray Gamboa. In the years 1997, 1998, 1999, 2000, and 2001, however, the
requisite quorum for the holding of the stockholders’ meeting could not be obtained. Consequently, the
above-named directors continued to serve in the VVCC Board in a hold-over capacity. Two of the said
members resigned (Makalintal and Dinglasan). After the resignation of Dinglasan, Eric Roxas was
elected. Makalintal was replaced by Jose Ramirez.
Respondent Africa, a member of VVCC, questioned the election of Roxas and Ramirez as members
of the VVCC Board with the Securities and Exchange Commission (SEC) and the Regional Trial Court.

97 | P a g e
Law 321_Corporation LAW_ Case Digest

Africa alleged that the election of Roxas was contrary to Section 29, in relation to Section 23, of the
Corporation Code of the Philippines. The respective trial courts ruled in favor of Africa.

ISSUE:

Whether or not the elections were valid.

RULING:

YES.

Section 23of the Corporation Code declares that "the board of directors shall hold office for one (1)
year until their successors are elected and qualified," we construe the provision to mean that the term of
the members of the board of directors shall be only for one year; their term expires one year after
election to the office. The holdover period – that time from the lapse of one year from a member’s election
to the Board and until his successor’s election and qualification – is not part of the director’s original
term of office, nor is it a new term; the holdover period, however, constitutes part of his tenure.
Corollary, when an incumbent member of the board of directors continues to serve in a holdover
capacity, it implies that the office has a fixed term, which has expired, and the incumbent is holding the
succeeding term.
After the lapse of one year from his election as member of the VVCC Board in 1996, Makalintal’s
term of office is deemed to have already expired. That he continued to serve in the VVCC Board in a
holdover capacity cannot be considered as extending his term. This holdover period, however, is not to
be considered as part of his term, which, as declared, had already expired.
With the expiration of Makalintal’s term of office, a vacancy resulted which, by the terms of
Section 29of the Corporation Code, must be filled by the stockholders of VVCC in a regular or special
meeting called for the purpose. As correctly pointed out by the RTC, when remaining members of the
VVCC Board elected Ramirez to replace Makalintal, there was no more unexpired term to speak of, as
Makalintal’s one-year term had already expired. Pursuant to law, the authority to fill in the vacancy
caused by Makalintal’s leaving lies with the VVCC’s stockholders, not the remaining members of its
board of directors.

 How Compensated

GABRIEL C. SINGSON, et al.


vs.
COMMISSION ON AUDIT
G.R. No. 159355, August 9, 2010

FACTS:

Petitioners are the members of the Board of Philippine International Convention Center, Inc.
(PICCI). By virtue of the PICCI By-Laws, petitioners were authorized to receive P1,000.00 per diem each
for every meeting attended. An amended resolution further granted the Members of the additional
monthly RATA, in the amount of P1,500.00, to each of the petitioners. However, payment for such
grants were denied. The disallowance was questioned but it was upheld by herein respondent. Hence
this petition.

ISSUE:

Whether or not the grant of the compensation as well as the monthly RATA are valid.

RULING:

NO.

Section 30 of the Corporation Code, which authorizes the stockholders to grant compensation to
its directors, states: In the absence of any provision in the by-laws fixing their compensation, the
directors shall not receive any compensation, as such directors, except for reasonable per diems;

98 | P a g e
Law 321_Corporation LAW_ Case Digest

Provided, however, that any such compensation (other than per diems) may be granted to directors by
the vote of the stockholders representing at least a majority of the outstanding capital stock at a regular
or special stockholders’ meeting. In no case shall the total yearly compensation of directors, as such
directors, exceed ten (10%) percent of the net income before income tax of the corporation during the
preceding year.
From this, it is clear that the directors of a corporation shall not receive any compensation for
being members of the board of directors, except for reasonable per diems. The two instances where the
directors are to be entitled to compensation shall be when it is fixed by the corporation’s by-laws or
when the stockholders, representing at least a majority of the outstanding capital stock, vote to grant
the same at a regular or special stockholder’s meeting, subject to the qualification that, in any of the two
situations, the total yearly compensation of directors, as such directors, shall in no case exceed ten
(10%) percent of the net income before income tax of the corporation during the preceding year.
In this regard, the Court upholds the findings of respondent that petitioners’ right to
compensation as members of the PICCI Board of Directors is limited only to per diem of P1,000.00 for
every meeting attended, by virtue of the PICCI By-Laws.

99 | P a g e
Law 321_Corporation LAW_ Case Digest

WESTERN INSTITUTE OF TECHNOLOGY, INC.,


vs.
HOMERO L. VILLASIS, DIMAS ENRIQUEZ, PRESTON F. VILLASIS & REGINALD F. VILLASIS v.
RICARDO T. SALAS, et al.
1997 Aug 21, G.R. No. 113032

FACTS:

The petitioners are the minority stockholders, while the respondents are the majority
stockholders of the corporation. The petitioners alleged that a meeting was held in its principal office
and a prior notice was distributed to the members. Subsequently a resolution was passed granting
monthly compensation for services rendered by its officers.
A complaint was filed against the respondents by the petitioners contending that the grant of
compensation is prohibited. The trial court rendered decision in favor of the respondents.

ISSUE:

Whether or not the grant of compensation was valid.

RULING:

YES.

Under section 30, there are two (2) ways by which members of the board can be granted
compensation apart from reasonable per diems: (1) when there is a provision in the by-laws fixing their
compensation; and (2) when the stockholders representing a majority of the outstanding capital stock at
a regular or special stockholders' meeting agree to give it to them. The proscription under said section
pertains to compensations granted to members of the Board. But they are not prohibited to be
compensated if these members of the Board act as officers of the corporation, more particularly as
Chairman, Vice-Chairman, Treasurer and Secretary of Western Institute of Technology.

100 | P a g e
Law 321_Corporation LAW_ Case Digest

CENTRAL COOPERATIVE EXCHANGE, INC.


vs.
CONCORDIO TIBE, SR. and THE HONORABLE COURT OF APPEALS
1970 Jun 30, G.R. No. L-27972

FACTS:

The petitioner is a national federation of farmers' cooperative marketing associations, or


FACOMAS, scattered throughout the country. Under its by- laws "The compensation, if any, and the per
diems for attendance at meetings of the members of the Board of Directors shall be determined by the
members at any annual meeting in special meeting of the Exchange called for the purpose."
In the annual stockholders’ meeting it was resolved that the members of the Board of Directors
attending the CCE board meetings be entitled to actual transportation expenses plus the per diems of
P30.00 and actual expenses while waiting. In this regard, Tibe collected the said amounts however the
petitioner refused to give on the ground that the resolutions are invalid. The trial court ruled in favor of
Tibe.

ISSUE:

Whether or not the resolutions are valid.

RULING:

NO.

The Court ruled that resolutions are contrary to the By-Laws of the federation and, therefore, are
not within the power of the board of directors to enact. The By-Laws, in the aforequoted Section 8,
explicitly reserved unto the stockholders the power to determine the compensation of members of the
board of directors, and the stockholders did restrict such compensation to "actual transportation
expenses plus the per diems of P30.00 and actual expenses while waiting." Even without the express
reservation of said power, the directors are not entitled to compensation under the Corporation Code.
The directors, in assigning themselves additional duties, such as the visitation of FACOMAS,
acted within their power, but, by voting for themselves compensation for such additional duties, they
acted in excess of their authority, as expressed in the By-Laws.

LINGAYEN GULF ELECTRIC POWER COMPANY, INC.


vs.
IRINEO BALTAZAR
G.R. No. L-4824, June 30, 1953

FACTS:

101 | P a g e
Law 321_Corporation LAW_ Case Digest

The respondent subscribed stocks of the petitioner. After paying several amount, the respondent
failed to pay its outstanding balance, even after a demand made by the corporation. The latter hence
opted to collect the unpaid balance of the subscription made. However, the respondent refused to pay on
the contention that he has been released from his liability under Resolution No. 17. Furthermore, he
countered that, as the President of the corporation, he was entitled to compensation.
The trial court rendered judgment in favor of respondent.

ISSUE:

Whether or not the respondent is entitled to compensation.

RULING:

NO.

It is clear that he is not entitled to the same. The by-laws of the company are silent as to the
salary of the President. And, while resolutions of the incorporators and stockholders provide salaries for
the general manager, secretary-treasurer and other employees, there was no provision for the salary of
the President. On the other hand, other resolutions provide for per diems to be paid to the President and
the directors of each meeting attended, P10 for the President and P8 for each director, which were later
increased to P25 and P15, respectively. This leads to the conclusions that the President and the board of
directors were expected to serve without salary, and that the per diems paid to them were sufficient
compensation for their services. Furthermore, for defendant's several years of service as President and
up to the filing of the action against him, he never filed a claim for salary.

 Authority of the Board of Directors

LA BUGAL-B'LAAN TRIBAL ASSOCIATION, INC., et al.


vs.
VICTOR O. RAMOS, Secretary, Department of Environment and Natural Resources (DENR);
HORACIO RAMOS, Director, Mines and Geosciences Bureau (MGB-DENR); RUBEN TORRES,
Executive Secretary; and WMC (PHIL.), INC.
G.R. No. 127882. December 1, 2004

FACTS:

102 | P a g e
Law 321_Corporation LAW_ Case Digest

On July 25, 1987, then President Corazon C. Aquino issued Executive Order (E.O.) No.
279authorizing the DENR Secretary to accept, consider and evaluate proposals from foreign-owned
corporations or foreign investors for contracts or agreements involving either technical or financial
assistance for large-scale exploration, development, and utilization of minerals, which, upon appropriate
recommendation of the Secretary, the President may execute with the foreign proponent.
Subsequently, then President Fidel V. Ramos approved R.A. No. 7942 to "govern the exploration,
development, utilization and processing of all mineral resources."R.A. No. 7942 defines the modes of
mineral agreements for mining operations,outlines the procedure for their filing and approval,
assignment/transferand withdrawal, and fixes their terms. Similar provisions govern financial or
technical assistance agreements.
On August 15, 1995, then DENR Secretary Victor O. Ramos issued DENR Administrative Order (DAO)
No. 95-23, s. 1995, otherwise known as the Implementing Rules and Regulations of R.A. No. 7942. This
was later repealed by DAO No. 96-40, s. 1996 which was adopted on December 20, 1996.
On January 10, 1997, counsels for petitioners sent a letter to the DENR Secretary demanding
that the DENR stop the implementation of R.A. No. 7942 and DAO No. 96-40.

ISSUE:

Whether or not the law in force when the WMC FTAA was executed, not come into effect.

RULING:

YES.

It bears noting that there is nothing in E.O. No. 200 that prevents a law from taking effect on a
date other than even before the 15-day period after its publication. Where a law provides for its own date
of effectivity, such date prevails over that prescribed by E.O. No. 200. Indeed, this is the very essence of
the phrase "unless it is otherwise provided" in Section 1 thereof. Section 1, E.O. No. 200, therefore,
applies only when a statute does not provide for its own date of effectivity.
As noted, "service contracts" is a term that assumes different meanings to different people. The
commissioners may have been using the term loosely, and not in its technical and legal sense, to refer,
in general, to agreements concerning natural resources entered into by the Government with foreign
corporations. These loose statements do not necessarily translate to the adoption of the 1973
Constitution provision allowing service contracts.
In any case, the constitutional provision allowing the President to enter into FTAAs with foreign-
owned corporations is an exception to the rule that participation in the nation's natural resources is
reserved exclusively to Filipinos. Accordingly, such provision must be construed strictly against their
enjoyment by non-Filipinos. As Commissioner Villegas emphasized, the provision is "very
restrictive."Commissioner Nolledo also remarked that "entering into service contracts is an exception to
the rule on protection of natural resources for the interest of the nation and, therefore, being an
exception, it should be subject, whenever possible, to stringent rules." Indeed, exceptions should be
strictly but reasonably construed.
SHIPSIDE INCORPORATED
vs.
THE HON. COURT OF APPEALS, HON. REGIONAL TRIAL COURT, BRANCH 26 & the REPUBLIC OF
THE PHILIPPINES
G.R. No. 143377, February 20, 2001

FACTS:

Originally four lots were owned Rafael Galvez. He subsequently sold lot 1 and 4 in favor of
Filipina Mamaril, Cleopatra Llana, Regina Bustos, and Erlinda Balatbat in a deed of sale. Mamaril later
sold lot 1 to Lepanto Consolidated Mining Company. Later, unknown to Lepanto, the RTC declared the
OCT registered in the name of Galvez as null and void and ordered the cancellation thereof. On October
28, 1963, Lepanto Consolidated Mining Company sold to herein petitioner Lots No. 1 and 4. Meanwhile
the decision of the CA became final and executory and a writ was issued, however said writ remained
unsatisfied for 24 years.
Office of the Solicitor General filed a complaint for revival of judgment and cancellation of titles
before the Regional Trial Court of the First Judicial Region against the successors of Galvez and herein
petitioner and its motion for reconsideration was likewise turned down. The CA affirmed the same,
hence this petition.

ISSUE:

103 | P a g e
Law 321_Corporation LAW_ Case Digest

Whether or not the filing of the petition was authorized by the BOD of petitioner.

RULING:

YES.

The Court of Appeals dismissed the petition for certiorari on the ground that Lorenzo Balbin, the
resident manager for petitioner, who was the signatory in the verification and certification on non-forum
shopping, failed to show proof that he was authorized by petitioner's board of directors to file such a
petition.
It was clear from the record that when the general manager filed the petition, there was no proof
attached as to the authorization by the Board. However, when the petitioner filed its motion for
reconsideration a resolution or secretary’s certification stating that that on October 11, 1999, or ten
days prior to the filing of the petition, Balbin had been authorized by petitioner's board of directors to
file said petition. The Court accepted this certification, although belatedly presented, as a valid
authorization. The Court was reiterated that belated submission of a verification is allowed the same
being not a mandatory and jurisdictional requirement, and as to the non-forum shopping the same was
considered to be valid because the case of the petitioner must be litigated based on its merit and must
not be dismissed based on technical and procedural infirmities, which were actually cured.

104 | P a g e
Law 321_Corporation LAW_ Case Digest

ABS-CBN
vs.
COURT OF APPEALS
G.R. No. 128690. January 21, 1999

FACTS:

In 1990, ABS CBN and Viva executed a Film Exhibition Agreement whereby Viva gave ABS CBN
an exclusive right to exhibit some Viva films. Said agreement contained a stipulation that ABS shall
have the right of first refusal to the next 24 Viva films for TV telecast, provided that such right shall be
exercised by ABS from the actual offer in writing. Hence, through this agreement, Viva offered ABS a list
of 36 films from which ABS may exercise its right of first refusal. ABS however, through VP Concio, did
not accept the list since she could only tick off 10 films. This rejection was embodied in a letter. In
1992, Viva again approached ABS with a list consisting of 52 original films where Viva proposed to sell
these airing rights for P60M. Viva’s Vic del Rosario and ABS’ general manager Eugenio Lopez III met at
the Tamarind Grill to discuss this package proposal. What transcribed at that meeting was subject to
conflicting versions.
According to Lopez, he and del Rosario agreed that ABS was granted exclusive film rights to 14
films for P36M, and that this was put in writing in a napkin, signed by Lopez and given to del Rosario.
On the other hand, del Rosario denied the existence of the napkin in which Lopez wrote something, and
insisted that what he and Lopez discussed was Viva’s film package of the 52 original films for P60M
stated above, and that Lopez refused said offer, allegedly signifying his intent to send a counter
proposal. When the counter proposal arrived, Viva’s BoD rejected it; hence, he sold the rights to the 52
original films to RBS.
Thus, ABS filed before RTC a complaint for specific performance with prayer for TRO against RBS
and Viva. RTC issued the TRO enjoining the airing of the films subject of controversy. After hearing,
RTC rendered its decision in favor of RBS and Viva contending that there was no meeting of minds on
the price and terms of the offer. The agreement between Lopez and del Rosario was subject to Viva BoD
approval, and since this was rejected by the board, then, there was no basis for ABS’ demand that a
contract was entered into between them. That the 1990 Agreement with the right of first refusal was
already exercised by Ms. Concio when it rejected the offer, and such 1990 Agreement was an entirely
new contract other than the 1992 alleged agreement at the Tamarind Grill.

ISSUE:

Whether or not there was a perfected contract between Lopez and del Rosario.

RULING:

NO.

Contracts that are consensual in nature are perfected upon mere meeting of the minds. Once
there is concurrence between the offer and the acceptance upon the subject matter, consideration, and
terms of payment, a contract is produced. The offer must be certain. To convert the offer into a
contract, the acceptance must be absolute and must not qualify the terms of the offer; it must be plain,
unequivocal, unconditional, and without variance of any sort from the proposal. A qualified acceptance,
or one that involves a new proposal, constitutes a counter offer and is a rejection of the original offer.
Consequently, when something is desired which is not exactly what is proposed in the offer, such
acceptance is not sufficient to generate consent because any modification or variation from the terms of
the offer annuls the offer.
In the case at bar, when Del Rosario met with Lopez at the Tamarind Grill, the package of 52
films was Viva’s offer to enter into a new Exhibition Agreement. But ABS, through its counter proposal
sent to Viva, actually made a counter offer. Clearly, there was no acceptance. The acceptance should
be unqualified.
ASSET PRIVATIZATION TRUSTS
vs.
COURT OF APPEALS, JESUS S. CABARRUS, SR., et al.
G.R. No. 121171. December 29, 1998

FACTS:

By virtue of an agreement, the exclusive right to explore, develop and exploit nickel, cobalt and
other minerals in the Surigao mineral reservation was granted to Marinduque Mining and Industrial

105 | P a g e
Law 321_Corporation LAW_ Case Digest

Corporation (MMIC), with respondent Jesus S. Cabarrus, Sr. as President and among its original
stockholders.
The Philippine Government undertook to support the financing of MMIC by purchase of MMIC
debenture bonds and extension of guarantees. Subsequently, MMIC, PNB and DBP executed a Mortgage
Trust Agreement 3 whereby MMIC, as mortgagor, agreed to constitute a mortgage in favor or PNB and
DBP as mortgagees, over all MMIC's assets. MMIC defaulted in its loans with the bank amounting to
more or less P21M. Thus, a financial restructuring plan (FRP) designed to reduce MMIC's interest
expense through debt conversion to equity was drafted and was approved by the Board of Directors of
the MMIC. However, the proposed FRP had never been formally adopted, approved or ratified by either
PNB or DBP.
Hence, the properties were foreclosed with PNB as the lone bidder and were assigned to three
newly formed corporations, namely, Nonoc Mining Corporation, Maricalum Mining and Industrial
Corporation, and Island Cement Corporation. In 1986, these assets were transferred to the Asset
Privatization Trust (APT). , Jesus S. Cabarrus, Sr., together with the other stockholders of MMIC, filed a
derivative suit against DBP and PNB before the RTC of Makati, Branch 62, for Annulment of
Foreclosures, Specific Performance and Damages. The parties entered into a compromise agreement.
However, when submitted for approval, the Arbitration Committee rendered judgment in favor of MMIC.

ISSUE:

Whether or not there was a valid foreclosure of the properties.

RULING:

YES.

The FRP was not adopted by PNB and DBP hence there was no valid restructuring program
undertaken and the option of the banks to foreclose the properties were never divested. Although there
were allegations that representatives of PNB and DBP were part of the drafting of the FRP there was no
showing that the representatives of PNB and DBP in MMIC even had the requisite authority to enter into
a debt-for-equity swap. And if they had such authority, there was no showing that the banks, through
their board of directors, had ratified the FRP. Hence, without such proof, the Court ruled that those
representatives, singly or collectively, are not themselves PNB or DBP. They are individuals with
personalities separate and distinct from the banks they represent. PNB and DBP have different boards
with different members who may have different decisions. And estoppel cannot be used to impose upon
them the decision of the board of another company. Otherwise the rights of entirely separate distinct
and autonomous legal entities like PNB and DBP with thousands of stockholders will be suppressed and
rendered nugatory. Wherefore, the decisions of the appellate court were reversed.

BA SAVINGS BANK
vs.
ROGER T. SIA, TACIANA U. SIA and JOHN DOE
G.R. No. 131214, July 27, 2000

FACTS:

A petition for certiorari was filed by herein petitioner bank. However, the CA denied due course
the same on the ground that the certificate of non-forum shopping was signed by a lawyer. A Motion for
Reconsideration was subsequently filed by the petitioner, attached to which was a BA Savings Bank
Corporate Secretary’s Certificate. The Certificate showed that the petitioner’s Board of Directors
approved a Resolution on May 21, 1996, authorizing the petitioner’s lawyers to represent it in any action
or proceeding before any court, tribunal or agency; and to sign, execute and deliver the Certificate of
Non-forum Shopping, among others. Said motion was denied on the ground that Supreme Court Revised
Circular No. 28-91 requires that it is the petitioner, not the counsel, who must certify under oath to all
of the facts and undertakings required therein.

106 | P a g e
Law 321_Corporation LAW_ Case Digest

ISSUE:

Whether or not the CA was correct.

RULING:

YES.

A corporation exercises powers through its board of directors and/or its duly authorized officers
and agents. Physical acts, like the signing of documents, can be performed only by natural persons duly
authorized for the purpose by corporate bylaws or by a specific act of the board of directors. In this case,
the corporation’s board of directors issued a Resolution specifically authorizing its lawyers “to act as
their agents in any action or proceeding before the Supreme Court, the Court of Appeals, or any other
tribunal or agency and to sign, execute and deliver in connection therewith the necessary pleadings,
motions, verification, affidavit of merit, certificate of non-forum shopping and other instruments
necessary for such action and proceeding.” The Resolution was sufficient to vest such persons with the
authority to bind the corporation and was specific enough as to the acts they were empowered to do.
Circular 28-91 requires the parties themselves to sign the certificate of non-forum shopping.
However, such requirement cannot be imposed on artificial persons, like corporations, for the simple
reason that they cannot personally do the task themselves. In this case, the corporation very well
exercised its power to authorize a representative to act on its behalf.

107 | P a g e
Law 321_Corporation LAW_ Case Digest

ALFREDO MONTELIBANO and ALEJANDRO MONTELIBANO


vs.
THE HON. COURT OF APPEALS and BACOLOD-MURCIA MILLING COMPANY, INC.
G.R. No. 85757, July 8, 1991

FACTS:

Alfredo and Alejandro Montelibano, together with other planters, entered into contracts with
Bacolod-Murcia Milling Co., Inc., for the milling of sugar cane at a sharing ratio of 55% for the planters
and 45% for the miller. The contracts were to be in force for thirty (30) years starting with the 1920-21
crops. A proposal was made to amend the milling contracts by increasing the planters' share to 60% of
the manufactured sugar and molasses and giving them other concessions besides, but the term of the
contracts was extended to 45 years instead of 30. On August 30, 1936, the milling company's Board of
Directors adopted a resolution granting further concessions to the planters over and above those
contained in the amended milling contract. Subsequently, the Montelibanos sued the milling company
alleging that the three other centrals in the province were granting increased participation to their
planters; therefore, pursuant to paragraph 9 of the August 20, 1936 Resolution, Bacolod-Murcia Milling
Co., Inc. was obligated to grant similar concessions to the Montelibanos.
The milling company opposed the claim on the ground that, among others, it was a donation
which was not within the power of the Board of Directors to grant. The trial court dismissed the action,
but on appeal to the Supreme Court reversed the lower court.

ISSUE:

Whether or not the reversal was proper.

RULING:

YES.

The Court ruled that the August 20, 1936 resolution, passed in good faith by the board of
directors, was valid and binding and formed an integral part of the amended milling contracts, the
milling company having agreed to give concessions to the planters, precisely to induce them to agree to
an extension of their contracts.
Petitioner filed two motions for reconsideration; however, the doctrine of res judicata had set in.
Wherefore, the appeal was denied.

ANTHONY POWERS, et al.


vs.
DONALD I. MARSHALL, et al.
G.R. No. L-48064, May 9, 1988

FACTS:

108 | P a g e
Law 321_Corporation LAW_ Case Digest

A letter was passed to the members of the International School, Inc. indicating that that the
Board of Trustees had decided to embark on a program to construct new buildings and remodel existing
ones to accommodate the increasing enrollment in the school, and that it was necessary for the school
to raise P35,000,000.00 for this purpose. The Board intended to raise the needed funds primarily
through subscriptions to capital notes and prepayment certificates, and any deficiency from these
sources would be covered by collecting a so-called "development fees" of P2,625 from each enrollee
starting with the school year 1975-1976 and continuing up to the school year 1986-1987. An
implementing letter was issued indicating therein that the conditions stated are pre-enrollment
requirements.
Fourteen (14) plaintiffs, all associate members of the International School, Inc., brought an action
for injunction in the Court of First Instance of Rizal. A TRO was granted in favor of the defendants.
Subsequently a decision was rendered dismissing the complaint for lack of cause of action.

ISSUE:

Whether or not the actions of the BOT are valid.

RULING:

YES.

Section 2 (b) of P.D. No. 732 granting certain rights to the International School, Inc., expressly
authorized the Board of Trustees "upon consultation with the Secretary of Education and Culture, ... to
determine the amount of fees and assessments which may be reasonably imposed upon its students, to
maintain or conform to the school standard of education." Such consultation had been made with the
Secretary of Education and Culture who expressed his conformity with the reasonableness of the
assessment of P2,625.00 per student for the whole school year to carry out its development program.
Since the collection of the development fee had been approved by the Board of Trustees of the
International School, Inc., it was a valid exercise of corporate power by the Board, and said assessment
was binding upon all the members of the corporation.

PREMIUM MARBLE RESOURCES, INC.


vs.
THE COURT OF APPEALS and INTERNATIONAL CORPORATE BANK
1996 Nov 4, G.R. No. 96551

FACTS:

Herein petitioner filed a case for damages against respondent for allowing clearance of checks by
unauthorized officers of the former, to the former’s prejudice. However this case was opposed by some
members of the petitioner on the ground that the filing of the complaint was not authorized by the
Board. Hence, a resolution of this case was necessary to litigate the claim of the petitioner for damages
against the respondent bank.

109 | P a g e
Law 321_Corporation LAW_ Case Digest

ISSUE:

Whether or not the filing was authorized by a duly constituted Board of Directors of the petitioner
corporation.

RULING:

NO.

The petitioners asserted that the Board authorized such filing. However, from the records of the
case as well as that of the corporation’s, no evidence was seen and shown that the results of the election
where the supposed members of the Board who allegedly authorized the filing were filed with the
Securities and Exchange Commission. The Corporation Code mandates that within thirty (30) days after
the election of the directors, trustees and officers of the corporation, the secretary, or any other officer of
the corporation, shall submit to the Securities and Exchange Commission, the names, nationalities and
residences of the directors, trustees and officers elected. Failure to comply with such requirement, the
elected members cannot be considered as the duly constituted and elected members of the Board.
Hence, being not duly constituted, the filing of the case was not authorized by the Board.

110 | P a g e
Law 321_Corporation LAW_ Case Digest

J. F. RAMIREZ
vs.
THE ORIENTALIST CO., and RAMON J. FERNANDEZ
G.R. No. 11897, September 24, 1918

FACTS:

Orientalist Companywas engaged in the business of maintaining and conducting a theatre in the
city of Manila for the exhibition of cinematographic films. Later on it accepted an offer from Jose
Ramirez, the son of herein petitioner that the latter will supply films that will be managed by the
respondent. However, when the films arrived, Orientalist was without fund to pay the cost and expenses
incident to each shipment. In effect the company’s president B. Hernandez paid said obligations and
treated the films by him as his own property; and they in fact never came into the actual possession of
the Orientalist Company as owner at all, though it is true Hernandez rented the films to the Orientalist
Company and they were exhibited by it in the Oriental Theater under an arrangement which was made
between him and the theater's manager. However, subsequent deliveries were no longer paid by any of
the concerned party.

ISSUE:

Whether or not the contract was entered into with the authorization of its Board.

RULING:

YES.

Although there were no evidence as to the authority of Ramon Fernandez to enter into said
contract, the Court had observed that when the defendant corporation failed to question the validity of
the contract, it resulted to eliminating the question of his authority from the case. This is a case where
an officer of a corporation has made a contract in its name, that the corporation should be required, if it
denies his authority, to state such defense in its answer. By this means the plaintiff is apprised of the
fact that the agent's authority is contested; and he is given an opportunity to adduce evidence showing
either that the authority existed or that the contract was ratified and approved. Failure to question such
timely and appropriately question such authority results to the admission of such fact.

 Corporate Officers/Meaning of “Office” vis-à-vis Employment

RENATO REAL
vs.
SANGU PHILIPPINES, INC. and/ or KIICHI ABE
G.R. No. 168757, January 19, 2011

111 | P a g e
Law 321_Corporation LAW_ Case Digest

FACTS:

Petitioner Renato Real was the Manager of Respondent Corporation Sangu Philippines, Inc.
engaged in the business of providing manpower for general services, like janitors, janitresses and other
maintenance personnel, to various clients. Subsequently, the janitors, etc employed by the latter filed
their respective complaint for illegal dismissal. Petitioner, on the other hand, was removed from his
position via Board resolution, and thus also filed an illegal dismissal case against the corporation. The
LA ruled for the petitioner and his co-complainants, which was reversed by the NLRC. The ruling of the
NLRC was affirmed by the CA.

ISSUE:

Whether or not petitioner is a corporate officer of the corporation.

RULING:

NO.

He is not a corporate officer. It has been consistently held that an ‘office’ is created by the charter
of the corporation and the officer is elected (or appointed) by the directors or stockholders. Clearly here,
respondents failed to prove that petitioner was appointed by the board of directors. Although they had
been reiterating that the petitioner was employed as a manager, there was no indication as to how he
was put into such position. For respondents’ failure to substantiate its claim the petitioner was deemed
to be not a coporate officer, hence jurisdiction properly lies with the LA.

MATLING INDUSTRIAL AND COMMERCIAL CORPORATION, RICHARD K. SPENCER, CATHERINE


SPENCER, AND ALEX MANCILLA
vs.
RICARDO R. COROS
G.R. No. 157802, October 13, 2010

FACTS:

The respondent filed a case for illegal dismissal against Matling and some of its corporate officers
(petitioners) in the NLRC. He was the Vice President for Finance and Administration when he was
dismissed. The petitioner opposed said complaint on the ground that the issue at hand is an intra-

112 | P a g e
Law 321_Corporation LAW_ Case Digest

corporate dispute which falls under the jurisdiction of the SEC. The respondent opposed the same
contending that his being in the position was doubtful that he had not been formally elected as such.
The LA ruled in favor of petitioner but the same was reversed by the NLRC, and to which the CA
affirmed.

ISSUE:

Whether or not respondent is a corporate officer.

RULING:

NO.

The petitioners contend that the position of Vice President for Finance and Administration was a
corporate office, having been created by Matling’s President pursuant to the by-Law.
However, the Court explained that an "office" is created by the charter of the corporation and the
officer is elected by the directors or stockholders. On the other hand, an employee occupies no office and
generally is employed not by the action of the directors or stockholders but by the managing officer of
the corporation who also determines the compensation to be paid to such employee. In this case,
respondent was appointed vice president for nationwide expansion by Malonzo, petitioner’'s general
manager, not by the board of directors of petitioner. Also his compensation was paid by Malonzo. Thus,
respondent was an employee, not a "corporate officer.
Also, the Board of Directors of Matling could not validly delegate the power to create a corporate
office to the President, in light of Section 25 of the Corporation Code requiring the Board of Directors
itself to elect the corporate officers. Verily, the power to elect the corporate officers was a discretionary
power that the law exclusively vested in the Board of Directors, and could not be delegated to
subordinate officers or agents.The office of Vice President for Finance and Administration created by
Matling’s President pursuant to the by- law was an ordinary, not a corporate, office.

113 | P a g e
Law 321_Corporation LAW_ Case Digest

MANILA METAL CONTAINER CORPORATION, REYNALDO C. TOLENTINO


vs.
PHILIPPINE NATIONAL BANK, DMCI-PROJECT DEVELOPERS, INC.
G.R. No. 166862, December 20, 2006

FACTS:

Petitioner is the owner of a parcel of land. To secure a loan he obtained from PNB he executed a
REM over said land. For failure to pay the loan, PNB sought the foreclosure of the REM. After the public
auction, the petitioner requested PNB to grant him an extension to redeem the property. He failed to
redeem the property. Later on, the PNB agreed to let the petitioner purchase the property for a certain
amount, and a downpayment was then given. Subsequently, however, the bank informed the petitioner
that it was increasing the purchase price. Hence, a case was filed by petitioner.

ISSUE:

Whether or not the letter by the respondent accepting the petitioner’s offer was valid.

RULING:

NO.

There is no evidence that the SAMD was authorized by respondent's Board of Directors to accept
petitioner's offer and sell the property. Any acceptance by the SAMD of petitioner's offer would not bind
respondent. A corporation can only execute its powers and transact its business through its Board of
Directors and through its officers and agents when authorized by a board resolution or its by-laws.
Absent such valid delegation/authorization, the rule is that the declarations of an individual director
relating to the affairs of the corporation, but not in the course of, or connected with the performance of
authorized duties of such director, is held not binding on the corporation.

BIENVENIDO ONGKINGCO, as President and GALERIA DE MAGALLANES CONDOMINIUM


ASSOCIATION, INC.
vs.
NATIONAL LABOR RELATIONS COMMISSION and FEDERICO B. GUILAS
G.R. No. 119877, March 31, 1997

FACTS:

114 | P a g e
Law 321_Corporation LAW_ Case Digest

Petitioner Galeria de Magallanes Condominium Association, Inc. is a non-stock, non-profit


corporation with a primary purpose of holding title to the common areas of the Galeria de Magallanes
Condominium Project and to manage and administer the same for the use and convenience of the
residents and/or owners. Petitioner Bienvenido Ongkingco was the president of Galeria at the time
private respondent filed his complaint. Subsequently, Galeria's Board of Directors appointed private
respondent Federico B. Guilas as Administrator/Superintendent. Respondent, however, was no longer
re-appointed as Administrator; hence he filed a case for illegal dismissal. Petitioners filed a motion to
dismiss alleging that it is the SEC, and not the labor arbiter, which has jurisdiction over the subject
matter of the complaint. The LA granted the motion to dismiss, which decision was reversed by the
NLRC.

ISSUE:

Whether or not respondent was a corporate officer.

RULING:

YES.

Private respondent is an officer of Petitioner Corporation and not its mere employee. The by-laws
of the Galeria de Magallanes Condominium Association specifically include the
Superintendent/Administrator in its roster of corporate officers. He was appointed directly by the Board
of Directors not by any managing officer of the corporation and his salary was, likewise, set by the same
Board. Having thus determined, his dismissal or non-appointment is clearly an intra-corporate matter
and jurisdiction, therefore, properly belongs to the SEC and not the NLRC. Despite not being elected,
P.D. 902-A Sec. 5(c) expressly covers both election and appointment of corporate directors, trustees,
officers and managers.

115 | P a g e
Law 321_Corporation LAW_ Case Digest

ANDRES LAO
vs.
COURT OF APPEALS, THE ASSOCIATED ANGLO-AMERICAN TOBACCO CORPORATION and
ESTEBAN CO,
G.R. No. 47013, February 17, 2000

FACTS:

On April 6, 1965, The Associated Anglo-American Tobacco Corporation entered into a "Contract of
Sales Agent" with Andres Lao. Under the contract, Lao agreed to sell cigarettes manufactured and
shipped by the Corporation to his business. Lao would in turn remit the sales proceeds to the
Corporation. For his services, Lao would receive commission depending on the kind of cigarettes sold,
fixed monthly salary, and operational allowance. For several months, Lao had religiously complied with
his obligations, however until about seven (7) months later, Lao failed to accomplish his monthly sales
report. Re was reminded of his enormous accounts and the difficulty of obtaining a tally thereon despite
Lao's avowal of regular remittances of his collections. Because of this he was summoned to the office of
the corporation where his liability amounted to P525,053.47. The corporation stopped providing Lao
with the products. Subsequently, Andres, Jose and Tomas Lao brought a complaint for accounting and
damages with writ of preliminary injunction against the Corporation. The CFI rendered judgment in
favor of petitioners. Meanwhile, on June 24, 1974 and during the pendency of Civil Case No. 4452,
Esteban Co, representing the Corporation as its new vice-president, filed an affidavit of complaint
alleging that Lao failed to remit the amount of P224,585.82 which he allegedly misappropriated and
converted to his personal use, which resulted to the filing of an estafa case against Lao. In turn Lao filed
a case for malicious prosecution against the corporation. In this case, Co was made liable solidarily with
the corporation for malicious prosecution.

ISSUE:

Whether or not Co should be made liable solidarily with the corporation for malicious
prosecution.

RULING:

YES.

Co asserted that he should not be held jointly and severally liable with the Corporation because
in filing the affidavit-complaint against respondent Lao, he was acting as the executive vice-president of
the Corporation and his action was within the scope of his authority as such corporate officer. Based
from the records, Co was the vice-president of the corporation when he filed the affidavit- complaint. The
corporation failed tomake an issue out of his authority to file said case. Upon well-established principles
of pleading, lack of authority of an officer of a corporation to bind it by contract executed by him in its
name, is a defense which should have been specially pleaded by the Corporation
The Corporation's failure to interpose such a defense could only mean that the filing of the
affidavit-complaint by petitioner Co was with the consent and authority of the Corporation. In the same
vein, petitioner Co may not be held personally liable for acts performed in pursuance of an authority and
therefore, holding him solidarily liable with the Corporation for the damages awarded to respondent Lao
does accord with law and jurisprudence.

MITA PARDO DE TAVERA


vs.
PHILIPPINE TUBERCULOSIS SOCIETY, INC., et al
GR. No. L-48928, February 25, 1982

FACTS:

116 | P a g e
Law 321_Corporation LAW_ Case Digest

The plaintiff alleges that she is a doctor of Medicine by profession and a recognized specialist in
the treatment of tuberculosis and that she was a member of the Board of Directors of the defendant
Society, in representation of the Philippine Charity Sweepstakes Office. She was also duly appointed as
Executive Secretary of the Society. However, she was removed from such position without any cause.
This was denied by the respondents. The appellate courts rendered decision in favor of respondents.

ISSUE:

Whether or not the petitioner is an officer of the corporation.

RULING:

NO.

The Court ruled that there was no clear indication that the petitioner was appointed to a permanent
position. Although the minutes of the organizational meeting show that the Chairman mentioned the
need of appointing a "permanent" Executive Secretary, such statement alone cannot characterize the
appointment of petitioner without a contract of employment definitely fixing her term. Without such
term, the appointment was deemed to be temporary, and is subject to the pleasure of the Board or of the
appointing body. Hence, when the Board opts to replace the incumbent, technically there is no removal
but only expiration of term and in an expiration of term, there is no need of prior notice, due hearing or
sufficient grounds before the incumbent can be separated from office.

 Corporate Officers; Qualifications and Disqualifications; Authority and


Liabilities

MATLING INDUSTRIAL AND COMMERCIAL CORPORATION, RICHARD K. SPENCER, CATHERINE


SPENCER, AND ALEX MANCILLA
vs.
RICARDO R. COROS
G.R. No. 157802, October 13, 2010

FACTS:

117 | P a g e
Law 321_Corporation LAW_ Case Digest

The respondent filed a case for illegal dismissal against Matling and some of its corporate officers
(petitioners) in the NLRC. He was the Vice President for Finance and Administration when he was
dismissed. The petitioner opposed said complaint on the ground that the issue at hand is an intra-
corporate dispute which falls under the jurisdiction of the SEC. The respondent opposed the same
contending that his being in the position was doubtful that he had not been formally elected as such.
The LA ruled in favor of petitioner but the same was reversed by the NLRC, and to which the CA
affirmed.

ISSUE:

Whether or not respondent is a corporate officer.

RULING:

NO.

The petitioners contend that the position of Vice President for Finance and Administration was a
corporate office, having been created by Matling’s President pursuant to the by-Law.
However, the Court explained that an "office" is created by the charter of the corporation and the
officer is elected by the directors or stockholders. On the other hand, an employee occupies no office and
generally is employed not by the action of the directors or stockholders but by the managing officer of
the corporation who also determines the compensation to be paid to such employee. In this case,
respondent was appointed vice president for nationwide expansion by Malonzo, petitioner’'s general
manager, not by the board of directors of petitioner. Also his compensation was paid by Malonzo. Thus,
respondent was an employee, not a "corporate officer.
Also, the Board of Directors of Matling could not validly delegate the power to create a corporate
office to the President, in light of Section 25 of the Corporation Code requiring the Board of Directors
itself to elect the corporate officers. Verily, the power to elect the corporate officers was a discretionary
power that the law exclusively vested in the Board of Directors, and could not be delegated to
subordinate officers or agents.The office of Vice President for Finance and Administration created by
Matling’s President pursuant to the by- law was an ordinary, not a corporate, office.
In this case the Court enumerated the minimum set of officers who are the president, vice
president, secretary and treasurer and modern corporation statutes usually designate them as the
officers of the corporation. However, other offices are sometimes created by the charter or by-laws of a
corporation, or the board of directors may be empowered under the by-laws of a corporation to create
additional offices as may be necessary.

LESLIE OKOL
vs.
SLIMMERS WORLD INTERNATIONAL, BEHAVIOR MODIFICATIONS, INC., and RONALD JOSEPH
MOY
G.R. No. 160146, December 11, 2009

FACTS:

Leslie Okol, a Vice President of Slimmers World, was terminated from employment after an
incident with the Bureau of Customs regarding equipment belonging to/consigned to Slimmers World.
As such, Okolfiled a complaint with the Arbitration branch of the NLRC against Slimmers World for
illegal suspension, illegal dismissal, unpaid commissions, damages, and attorney’s fees, with prayer for
reinstatement and payment of backwages.
Slimmers World filed a Motion to Dismiss the case, asserting that the NLRC had no jurisdiction
over the subject matter of the complaint. Slimmers World’s motion was sustained, with the labor arbiter
ruling that since Okol was the vice president at the time of her dismissal, being a corporate officer, the
dispute was an intra-corporate controversy falling outside the jurisdiction of the arbitration branch. On

118 | P a g e
Law 321_Corporation LAW_ Case Digest

appeal, the NLRC reversed the LA decision and ordered Slimmers World to reinstate Okol. The CA
subsequently set aside the NLRC decision and ruled that the case was an intra-corporate controversy,
and falls within the jurisdiction of the regular courts pursuant to RA 8799.

ISSUE:

Whether Okol was a corporate officer of Slimmers World.

RULING:

YES.

Okol was a corporate officer at the time of her dismissal. According to the Amended By-Laws
of Slimmers World which enumerate the power of the board of directors as well as the officers of the
corporation, the general management of the corporation shall be vested in a board of five directors who
shall be stockholders and who shall be elected annually by the stockholders and who shall serve until
the election and qualification of their successors and like the Chairman of the Board and the President,
the Vice President shall be elected by the Board of Directors from its own members. The Vice President
shall be vested with all the powers and authority and is required to perform all the duties of the
President during the absence of the latter for any cause. The Vice President will perform such duties as
the Board of Directors may impose upon him from time to time. This clearly shows that Okol was a
director and officer of Slimmers World.
An office is created by the charter of the corporation and the officer is elected by the directors and
stockholders. On the other hand, an employee usually occupies no office and generally is employed not
by action of the directors or stockholders but by the managing officer of the corporation who also
determines the compensation to be paid to such employee.

GLORIA V. GOMEZ
vs.
PNOC DEVELOPMENT AND MANAGEMENT CORPORATION (PDMC) - (formerly known as FILOIL
DEVELOPMENT AND MANAGEMENT CORPORATION [FDMC])
G.R. No. 174044, November 27, 2009

FACTS:

Petitioner Gloria V. Gomez used to work as Manager of the Legal Department of Petron
Corporation, then a government-owned corporation. With Petron’s privatization, she availed of the
company’s early retirement program and left that organization on April 30, 1994. On the following day,
May 1, 1994, however, Filoil Refinery Corporation (Filoil), also a government-owned corporation,
appointed her its corporate secretary and legal counsel, with the same managerial rank, compensation,
and benefits that she used to enjoy at Petron. However, the privatization did not materialize so Gomez
continued to serve as corporate secretary of respondent PDMC.
On March 29, 1999 the new board of directors of respondent PDMC removed petitioner Gomez as
corporate secretary. Further, at the board’s meeting on October 21, 1999 the board questioned her
continued employment as administrator. In answer, she presented the former president’s May 24, 1998
letter that extended her term. Dissatisfied with this, the board sought the advice of its legal department,
which expressed the view that Gomez’s term extension was an ultra vires act of the former president. It
reasoned that, since her position was functionally that of a vice-president or general manager, her term
could be extended under the company’s by-laws only with the approval of the board. The legal
department held that her “de facto” tenure could be legally put to an end. Petitioner Gomez for her part
conceded that as corporate secretary, she served only as a corporate officer. But, when they named her

119 | P a g e
Law 321_Corporation LAW_ Case Digest

administrator, she became a regular managerial employee. Consequently, the respondent PDMC’s
board did not have to approve either her appointment as such or the extension of her term in 1998.

ISSUE:

Whether or not Gomez is an ordinary employee whose complaint is within the jurisdiction of the
NLRC.

RULING:

YES.

The relationship of a person to a corporation, whether as officer or agent or employee, is not


determined by the nature of the services he performs but by the incidents of his relationship with the
corporation as they actually exist. That the employee served concurrently as corporate secretary for a
time is immaterial. A corporation is not prohibited from hiring a corporate officer to perform services
under circumstances which will make him an employee. Indeed, it is possible for one to have a dual role
of officer and employee. NLRC has jurisdiction over a complaint filed by one who served both as
corporate officer and employee, when the money claims were made as an employee and not as a
corporate officer.

E. B. VILLAROSA & PARTNER CO., LTD.


vs.
HON. HERMINIO I. BENITO, in his capacity as Presiding Judge, RTC, Branch 132, Makati City and
IMPERIAL DEVELOPMENT CORPORATION
G.R. No. 136426, August 6, 1999

FACTS:

Petitioner E.B. Villarosa & Partner Co., Ltd. is a limited partnership. Petitioner and private
respondent executed a Deed of Sale with Development Agreement wherein the former agreed to develop
certain parcels of land belonging to the latter into a housing subdivision for the construction of low cost
housing units. They further agreed that in case of litigation regarding any dispute arising therefrom, the
venue shall be in the proper courts of Makati. Subsequently, a complaint for breach of contract was filed
by the respondent against the plaintiff allegedly for failure of the latter to comply with its contractual
obligation in that, other than a few unfinished low cost houses, there were no substantial developments.
Summons, together with the complaint, were served upon the defendant, through its Branch Manager
Engr. Wendell Sabulbero. The respondent moved for dismissal on the ground that there was improper
service of summons.
The trial court rendered decision denying the motion to dismiss. Hence this petition.

ISSUE:

Whether or not there was imporoper service of summons.

RULING:

YES.

Section 13, Rule 14 of the Rules of Court which provided that: Service upon private domestic
corporation or partnership, If the defendant is a corporation organized under the laws of the Philippines

120 | P a g e
Law 321_Corporation LAW_ Case Digest

or a partnership duly registered, service may be made on the president, manager, secretary, cashier,
agent, or any of its directors.
The Court ruled that under such provision, it is clear upon whom the service of summons should
be made. The designation of persons or officers who are authorized to accept summons for a domestic
corporation or partnership is now limited and more clearly specified. The rule now states "general
manager" instead of only "manager"; "corporate secretary" instead of "secretary"; and "treasurer" instead
of "cashier." The phrase "agent, or any of its directors" is conspicuously deleted in the new rule. In this
case, since the summons was served upon a branch manager, who is not authorized to accept the same,
there was improper service of summons.

121 | P a g e
Law 321_Corporation LAW_ Case Digest

SUPREME STEEL PIPE CORPORATION and REGAN SY


vs. ROGELIO BARDAJE
G.R. No. 170811, April 24, 2007

FACTS:

Petitioner Supreme Steel Pipe Corporation (SSPC) was primarily engaged in the business of
manufacturing steel pipes. It employed respondent Rogelio Bardaje as a warehouseman on March 14,
1994. SSPC employees were required to wear a uniform (a yellow t-shirt with a logo and the marking
"Supreme") while at work.
Due to an incident, his employment was terminated on the ground of multiple infractions of
company rules. He thus filed a case for illegal dismissal. The LA ruled for the respondent, while the
NLRC reversed said decision. The CA ruled favoring the LA.

ISSUE:

Whether or not Regan Sy, the president of SSPC, may be held solidarily liable with the latter.

RULING:

NO.

It appears that respondent impleaded SSPC President Regan Sy only because he is an


officer/agent of the company. However, the court ruled that he cannot be made solidarily liable because
for the termination of respondent’s employment, since there is no showing that the dismissal was
attended with malice or bad faith. The rule still stand that the liabilities of a corporation should not be
directly imputed to its officers and it shall be borne entirely by the corporation itself.

122 | P a g e
Law 321_Corporation LAW_ Case Digest

CAGAYAN VALLEY DRUG CORPORATION


vs.
COMMISSIONER OF INTERNAL REVENUE
G.R. No. 151413, February 13, 2008

FACTS:

Petitioner is a duly licensed retailer of medicine and other pharmaceutical products. In


compliance with Revenue Regulation No. (RR) 2-94, petitioner treated the 20% sales discounts granted
to qualified senior citizens in 1995 as deductions from the gross sales in order to arrive at the net sales,
instead of treating them as tax credit as provided by Section 4 of RA 7432. however, petitioner filed with
the Bureau of Internal Revenue (BIR) a claim for tax refund/tax credit of the full amount of the 20%
sales discount it granted to senior citizens for the year 1995, allegedly totalling to P123,083.00. Because
of the BIR’s inaction, the petitioner filed a petition for review before the CTA, which denied its claim. The
CA also denied the same on procedural grounds, such that the person who signed the verification and
certification of absence of forum shopping, a certain Jacinto J. Concepcion, President of petitioner, failed
to adduce proof that he was duly authorized by the board of directors to do so.

ISSUE:

Whether petitioner’s president can sign the subject verification and certification without the
approval of its Board of Directors.

RULING:

YES.

In several cases the court has recognized the authority of some corporate officers to sign the
verification and certification against forum shopping. In these cases, the court allowed the: (1) the
Chairperson of the Board of Directors, (2) the President of a corporation, (3) the General Manager or
Acting General Manager, (4) Personnel Officer, and (5) an Employment Specialist in a labor case, to sign
said documents, without need of a board resolution.
Also in this case, an authorization was belatedly submitted. Although belated, the court still
accepts it as a valid and which it had cured the procedural infirmities of the case.

SALOME PABON and VICENTE CAMONAYAN


vs.
NATIONAL LABOR RELATIONS COMMISSION and SENIOR MARKETING CORPORATION
G.R. No. 120457 September 24, 1998

FACTS:

123 | P a g e
Law 321_Corporation LAW_ Case Digest

On May 24, 1994 and June 22, 1994, complaints for illegal dismissal and non-payment of
benefits were filed by petitioners Salome Pabon and Vicente Camonayan against private respondent
Senior Marketing Corporation (SMC) and its Field Manager, R-Jay Roxas Summons and notices of
hearings were sent to Roxas at private respondent's provincial office in 13 Valley Homes, Patul Road,
Santiago, Isabela which were received by its bookkeeper, Mina Villanueva.
On September 15, 1994, the Labor Arbiter rendered a judgment by default after finding that
private respondent tried to evade all the summons and orders of hearing by refusing to claim all the
registered mail addressed to it.

ISSUE:

Whether or not Petitioners herein are authorized to receive summons in behalf of the corporation.

RULING:

YES.

Bookkeeper can be considered as an agent of private respondent corporation within the purview
of Section 13, Rule 14 of the old Rules of Court. The rationale of all rules with respect to service of
process on a corporation is that such service must be made to an agent or a representative so integrated
with the corporation sued as to make it a priori supposable that he will realize his responsibilities and
know what he should do with any legal papers served on him. The bookkeeper's task is one under
consideration. The job of a bookkeeper is so integrated with the corporation that his regular recording of
the corporation's "business accounts" and "essential facts about the transactions of a business or
enterprise" safeguards the corporation from possible fraud being committed adverse to its own corporate
interest.
Although it may be true that the service of summons was made on a person not authorized to
receive the same in behalf of the petitioner, nevertheless since it appears that the summons and
complaint were in fact received by the corporation through its said clerk, the Court finds that there was
substantial compliance with the rule on service of summons. Indeed the purpose of said rule as above
stated to assure service of summons on the corporation had thereby been attained. The need for speedy
justice must prevail over technicality.

VLASON ENTERPRISES CORPORATION


vs.
COURT OF APPEALS and DURAPROOF SERVICES, represented by its General Manager, Cesar
Urbino Sr.
G.R. Nos. 121662-64. July 6, 1999

FACTS:

Poro Point Shipping Services, then acting as the local agent of Omega Sea Transport Company of
Honduras & Panama, a Panamanian company, (hereafter referred to as Omega), requested permission
for its vessel M/V Star Ace, which had engine trouble, to unload its cargo and to store it at the
Philippine Ports Authority (PPA) compound in San Fernando, La Union while awaiting transshipment to
Hong Kong. The request was approved by the Bureau of Customs. Despite the approval, the customs

124 | P a g e
Law 321_Corporation LAW_ Case Digest

personnel boarded the vessel when it docked on January 7, 1989, on suspicion that it was the hijacked
M/V Silver Med owned by Med Line Philippines Co., and that its cargo would be smuggled into the
country. The district customs collector seized said vessel and its cargo pursuant to Section 2301, Tariff
and Customs Code.
They entered into a salvage agreement with private respondent to secure and repair the vessel
which was destroyed by the typhoons that hit the province at the agreed consideration of $1 million and
“fifty percent (50%) of the cargo after all expenses, cost and taxes.” Subsequently, the seizure was lifted
for want of fraud.

ISSUE:

Whether or not summons was properly served with Vlason Corporation.

RULING:

NO.

Sec 4 and 5 of the Rules of Court ideally requires a movant to address and serve on the counsel of
the adverse party the notice of hearing of its motion. Service of a copy of a motion must contain a notice
of the time and the place of hearing. There are, however, exceptions to the rule: Where a rigid
application will result in a manifest failure or miscarriage of justice, especially if a party successfully
shows that the alleged defect in the questioned final and executory judgment is not apparent on its face
or from the recitals contained therein; Where the interest of substantial justice will be served; Where the
resolution of the motion is addressed solely to the sound and judicious discretion of the court; Where
the injustice to the adverse party is not commensurate to the degree of his failure to comply with
prescribed procedure
In this case, Vlason was not informed of any cause of action against it. It was not validly
summoned. Its vessels that it used for its salvaging business was levied upon and sold in execution to
satisfy a supposed judgment against it. To allow this to happen simply because of its failure to comply
with the notice requirement would result into manifest injustice.

Prime White Cement Corporation


vs.
Intermediate Appellate Court
GR 68555, 19 March 1993

FACTS:

On or about 16 July 1969, Alejandro Te and Prime White Cement Corporation (PWCC) thru its
President, Mr. Zosimo Falcon and Justo C. Trazo, as Chairman of the Board, entered into a dealership
agreement whereby Te was obligated to act as the exclusive dealer and/or distributor of PWCC of its
cement products in the entire Mindanao area for a term of 5 years.
Right after Te entered into the dealership agreement, he placed an advertisement in a national,
circulating newspaper the fact of his being the exclusive dealer of PWWC's white cement products in
Mindanao area, more particularly, in the Manila Chronicle dated 16 August 1969 and was even
congratulated by his business associates, so much so, he was asked by some of his businessmen friends
and close associates if they can be his sub-dealer in the Mindanao area.

ISSUE:

125 | P a g e
Law 321_Corporation LAW_ Case Digest

Whether the "dealership agreement" referred by the President and Chairman of the Board of
PWCC is a valid and enforceable contract.

RULING:

NO.

The “dealership agreement” is not valid and unenforceable. Under the Corporation Law, which
was then in force at the time the case arose, as well as under the present Corporation Code, all
corporate powers shall be exercised by the Board of Directors, except as otherwise provided by law.
Although it cannot completely abdicate its power and responsibility to act for the juridical entity, the
Board may expressly delegate specific powers to its President or any of its officers.
In the absence of such express delegation, a contract entered into by its President, on behalf of
the corporation, may still bind the corporation if the board should ratify the same expressly or impliedly.
Implied ratification may take various forms — like silence or acquiescence; by acts showing approval or
adoption of the contract; or by acceptance and retention of benefits flowing therefrom. Furthermore,
even in the absence of express or implied authority by ratification, the President as such may, as a
general rule, bind the corporation by a contract in the ordinary course of business, provided the same is
reasonable under the circumstances. These rules are basic, but are all general and thus quite flexible.
They apply where the President or other officer, purportedly acting for the corporations, is dealing with a
third person, i.e., a person outside the corporation. The situation is quite different where a director or
officer is dealing with his own corporation. Herein, Te was not an ordinary stockholder; he was a
member of the Board of Directors and Auditor of the corporation as well. He was what is often referred
to as a "self-dealing" director.

LOUIS VUITTON S.A.


vs.
JUDGE FRANCISCO DIAZ VILLANUEVA, presiding Judge, Branch 36, The Metropolitan Trial Court
at Quezon City, Metro Manila
A.M. No. MTJ-92-643 November 27, 1992

FACTS:

In Criminal Case No. XXXVI-62431, entitled "People of the Philippines vs. Jose V. Rosario", Louis
Vuitton, S.A. accused the latter of unfair competition as defined by paragraph 1 of Article 189, Revised
Penal Code.
Complainant also assailed respondent judge's findings that there was no unfair competition
because the elements of the crime were not met, and that he seized articles did not come close to the
appearance of a genuine Louis Vuitton product, the counterfeit items having been poorly, done.

ISSUE:

Whether or not respondent judge is guilty of knowingly rendering a manifestly unjust judgment.

RULING:

NO.

The ground which was relied upon by the trial court in acquitting the accused finds basis in the
well-settled doctrine that a corporation has a distinct personality from that of its stockholders/owners.

126 | P a g e
Law 321_Corporation LAW_ Case Digest

A corporation is vested by law with a personality of its own, separate and distinct from that of its
stockholders and from that of its officers who manage and run its affairs. This decision is assailed to be
unjust mainly because it did not consider the Prosecution's Memorandum with Motion and Motion for
Early Resolution filed by private prosecutor, herein complainant, on February 8, 1991 and February 11,
1991, respectively. According to complainant, had respondent judge taken the former motion into
account, he would not have acquitted the accused, Jose V. Rosario. Instead, he would have been held
guilty for giving others an opportunity engage in unfair competition as prescribed by Article 189 of the
Revised Penal Code.
In the first place, it would not have made any difference because Jose v. Rosario was charged as
owner/proprietor. COD is not a single proprietorship but one that is run and owned by a corporation,
Rosario Bros., Inc., of which the accused is stockholder and Executive Vice-President. A stockholder
generally does not have a hand in the management of the corporate affairs. On the other hand, the Vice-
President had no inherent power to bind the corporation.
As general rule, his duties must be specified in the by-laws. In the criminal case, the information
did not specify his duties as Executive Vice-President. The trial court had no basis for holding that as
such, the accused entered into a contract with the concessionaire thereby giving the latter an
opportunity to practice unfair competition. Whereas, Section 23 of the Corporation Code is explicit that
the directors, acting as a body, exercise corporation powers and conduct the corporation's business.

 Doctrine of Apparent Authority

VIOLETA BANATE et. al.


vs.
PHILIPPINE COUNTRYSIDE RURAL BANK
GR 163825, 13 July 2010

FACTS:

Sometime in November 1997 the spouses Maglasang and the spouses Cortel asked PCRB’s
permission to sell the properties which they mortgaged with the bank. They likewise requested that the
said properties be released from the mortgage since the two other loans were adequately secured by the
other mortgages. The spouses Maglasang and the spouses Cortel claimed that the PCRB, acting through
its Branch Manager, Pancrasio Mondigo, verbally agreed to their request but required first the full
payment of the subject loan. They thereafter sold to petitioner Violeta Banate the subject properties for
P1,750,000.00 and used the amount to pay the subject loan with PCRB. After settling the subject loan,
PCRB gave the owner’s duplicate certificate of title of Lot 12868-H-3-C to Banate, who was able to
secure a new title in her name. It, however, carried the mortgage lien in favor of PCRB, prompting the
petitioners to request from PCRB a Deed of Release of Mortgage. As PCRB refused to comply with the
petitioners’ request, the petitioners instituted an action for specific performance before the RTC to
compel PCRB to execute the release deed.
Accordingly, PCRB claimed that full payment of the three loans, obtained by the spouses
Maglasang, was necessary before any of the mortgages could be released; the settlement of the subject
loan merely constituted partial payment of the total obligation. Thus, the payment does not authorize
the release of the subject properties from the mortgage lien.

ISSUE:

Whether or not Mondigo, as branch manager of PCRB, has the authority to modify the original
mortgage contract on behalf of the company.

RULING:

127 | P a g e
Law 321_Corporation LAW_ Case Digest

NO.

He is not authorized to modify the mortgage contract that would in effect cause novation. Under
the doctrine of apparent authority, acts and contracts of the agent, as are within the apparent scope of
the authority conferred on him, although no actual authority to do such acts or to make such contracts
has been conferred, bind the principal. The principal’s liability, however, is limited only to third persons
who have been led reasonably to believe by the conduct of the principal that such actual authority
exists, although none was given. In other words, apparent authority is determined only by the acts of
the principal and not by the acts of the agent. There can be no apparent authority of an agent without
acts or conduct on the part of the principal; such acts or conduct must have been known and relied
upon in good faith as a result of the exercise of reasonable prudence by a third party as claimant, and
such acts or conduct must have produced a change of position to the third party’s detriment.
In the present case, the decision of the trial court was utterly silent on the manner by which
PCRB, as supposed principal, has “clothed” or “held out” its branch manager as having the power to
enter into an agreement, as claimed by petitioners. No proof of the course of business, usages and
practices of the bank about, or knowledge that the board had or is presumed to have of, its responsible
officers’ acts regarding bank branch affairs, was ever adduced to establish the branch manager’s
apparent authority to verbally alter the terms of mortgage contracts. Neither was there any allegation,
much less proof, that PCRB ratified Mondigo’s act or is estopped to make a contrary claim.

SARGASSO CONSTRUCTION
vs.
PHILIPPINE PORTS AUTHORITY
GR 170530, 05 July 2010

FACTS:

Plaintiff Sargasso Construction and Development Corporation, Pick and Shovel, Inc. and Atlantic
Erectors, Inc., a joint venture, was awarded the construction of Pier 2 and the rock causeway (R.C. Pier
2) for the port of San Fernando, La Union, after a public bidding conducted by the defendant PPA.
Plaintiff offered to undertake the reclamation between the Timber Pier and Pier 2 of the Port of
San Fernando, La Union, as an extra work to its existing construction of R.C. Pier 2 and Rock Causeway
for a price of P36,294,857.03. The offer was unacceptable to PPA and thereafter asked for its reduction
to P30,794,230.89.
On August 26, 1993, a Notice of Award signed by PPA General Manager Rogelio Dayan was sent
to plaintiff for the phase I Reclamation Contract in the amount of P30,794,230.89 and instructing it to
“enter into and execute the contract agreement with this Office” and to furnish the documents
representing performance security and credit line. PPA Management further set a condition [that] “the
acceptance by the contractor that mobilization/demobilization cost shall not be included in the contract
and that escalation shall be reckoned upon approval of the Supplemental Agreement. Hence, then
General Manager Carlos L. Agustin presented for consideration by the PPA Board of Directors the
contract proposal for the reclamation project.
At its meeting held on September 9, 1994, the Board decided not to approve the contract
proposal. The Board noted that the Pier 2 Project was basically for the construction of a pier while the
supplemental agreement refers to reclamation. Thus there is no basis to compare the terms and
conditions of the reclamation project with the original contract (Pier 2 Project) of Sargasso.” Plaintiff filed
a complaint for specific performance and damages.

ISSUE:

Whether or not the Notice of Award made by the General Manager of PPA binding with the entity.

RULING:

NO.

There is no perfected contract between the parties in this case. Likewise, the General Manager of
the PPA is not authorized to enter into contracts on behalf of the agency. Petitioner’s invocation of the
doctrine of apparent authority is misplaced. This doctrine, in the realm of government contracts, has
been restated to mean that the government is NOT bound by unauthorized acts of its agents, even
though within the apparent scope of their authority. Under the law on agency, however, “apparent
authority” is defined as the power to affect the legal relations of another person by transactions with

128 | P a g e
Law 321_Corporation LAW_ Case Digest

third persons arising from the other’s manifestations to such third person such that the liability of the
principal for the acts and contracts of his agent extendsto those which are within the apparent scope of
the authority conferred on him, although no actual authority to do such acts or to make such contracts
has been conferred.
The existence of apparent authority may be ascertained through (1) the general manner in which
the corporation holds out an officer or agent as having the power to act or, in other words, the apparent
authority to act in general, with which it clothes him; or (2) the acquiescence in his acts of a particular
nature, with actual or constructive knowledge thereof, whether within or beyond the scope of his
ordinary powers. It requires presentation of evidence of similar act(s) executed either in its favor or in
favor of other parties. In this case, not a single act of respondent, acting through its Board of Directors,
was cited as having clothed its general manager with apparent authority to execute the contract with it.
ASSOCIATED BANK (now UNITED OVERSEAS BANK [PHILS.])
vs.
SPOUSES RAFAEL and MONALIZA PRONSTROLLER,
G.R. No. 148444, July 14, 2008

FACTS:

On April 21, 1988, spouses Vaca executed a Real Estate Mortgage (REM) in favor of the petitioner
over their parcel of residential land located at Quezon City. For failure of the spouses Vaca to pay their
obligation, the subject property was sold at public auction with the petitioner as the highest bidder. The
spouses Vaca, however, commenced an action for the nullification of the real estate mortgage and the
foreclosure sale. During the pendency of the aforesaid cases, Respondents Rafael and Monaliza
Pronstroller offered to purchase the property for P7,500,000.00. Said offer was made through Atty. Jose
Soluta, Jr., petitioner’s Vice-President, Corporate Secretary and a member of its Board of Directors.
Petitioner accepted respondents’ offer of P7.5 million. Consequently, respondents paid petitioner
P750,000.00, or 10% of the purchase price, as down payment. On March 18, 1993, petitioner, through
Atty. Soluta, and respondents, executed a Letter-Agreement.

ISSUE:

Whether or not the petitioner is bound by the July 14, 1993 Letter-Agreement signed by Atty.
Soluta under the doctrine of apparent authority.

RULING:

YES.

The general rule is that, in the absence of authority from the board of directors, no person, not
even its officers, can validly bind a corporation. The power and responsibility todecide whether the
corporation should enter into a contract that will bind the corporation islodged in the board of directors.
However, just as a natural person may authorize another to do certain acts for and on his behalf, the
board may validly delegate some of its functions and powers to officers, committees and agents. The
authority of such individuals to bind the corporation is generally derived from law, corporate bylaws or
authorization from the board, either expressly or impliedly, by habit, custom, or acquiescence, in the
general course of business.The authority of a corporate officer or agent in dealing with third persons
may be actual or apparent.
The doctrine of “apparent authority,” with special reference to banks, had long been recognized in
this jurisdiction. Apparent authority is derived not merely from practice. Its existence may be
ascertained through 1) the general manner in which the corporation holds out an officer or agent as
having the power to act, or in other words, the apparent authority to act in general, with which it clothes
him; 2) the acquiescence in his acts of a particular nature, with actual or constructive knowledge
thereof, within or beyond the scope of his ordinary powers. Accordingly, the authority to act for and to
bind a corporation may be presumed from acts of recognition in other instances, wherein the power was
exercised without any objection from its board or shareholders.

129 | P a g e
Law 321_Corporation LAW_ Case Digest

EMILIANO ACUÑA
vs.
BATAC PRODUCERS
GR L-20333, 30 June 1963

FACTS:

Acuña entered into a contract with Batac wherein he agreed to advance P20,000.00 to the
company for its tobacco planting and drying, provided that he shall be assigned as the company’s
representative in Manila and supervise the transport and delivery of the goods in the said place. Batac’s
Board of Directors are amenable with the idea and thereafter issued a resolution authorizing Manager
Leon Verano to enter into the agreement on behalf of the corporation.
The necessary contract between Acuña and Verano was entered into, with some of the Board of
Directors acting as witness. Acuña then inquired if the contract needs to be ratified by the Board, in
which the counsel for Batac answered in the negative. Acuña thereafter proceeded to perform his part of
the contrac, including the advancement of the amount promised, which was accepted by Batac.
Batac’s BoD, however, disapproved the contract. Acuña insisted on its performance, but the
corporation refused, stating that the contract is not binding by reason that it was not ratified by the
board.

ISSUE:

Whether or not the contract between Acuña and Verano is binding with the corporation.

RULING:

YES.

A perusal of the complaint reveals that it contains sufficient allegations indicating such approval
or at least subsequent ratification. On the first point note the following averments: that on May 9th the
plaintiff met with each and all of the individual defendants (who constituted the entire Board of
Directors) and discussed with them extensively the tentative agreement and he was made to understand
that it was acceptable to them, except as to plaintiff's remuneration; that it was finally agreed between
plaintiff and all said Directors that his remuneration would be P0.30 per kilo (of tobacco); and that after
the agreement was formally executed he was assured by said Directors that there would be no need of
formal approval by the Board. It should be noted in this connection that although the contract required
such approval it did not specify just in what manner the same should be given.
On the question of ratification the complaint alleges that plaintiff delivered to the defendant
corporation the sum of P20,000.00 as called for in the contract; that he rendered the services he was
required to do; that he furnished said defendant 3,000 sacks at a cost of P6,000.00 and advanced to it
the further sum of P5,000.00; and that he did all of these things with the full knowledge, acquiescence
and consent of each and all of the individual defendants who constitute the Board of Directors of the
defendant corporation. There is abundant authority in support of the proposition that ratification may
be expressed or implied, and that implied ratification may take diverse forms, such as by silence or
acquiescence; by acts showing approval or adoption of the contract; or by acceptance and retention of
benefits flowing therefrom.

130 | P a g e
Law 321_Corporation LAW_ Case Digest

BOARD OF LIQUIDATORS
vs.
HEIRS OF MAXIMO KALAW
GR L-18805, 14 August 1967

FACTS:

National Coconut Corporation (NACOCO) is with Maximo Kalaw as its General Manager and
Chairman of the BOD. Under his tenure NACOCO entered into different contracts involving the trade of
coconuts. It failed, however, due to natural calamities that greatly affected the production of coconuts.
This led to some customers of NACOCO suing the corporation for undelivered coconuts due to them
under the contracts that they signed. This was settled by NACOCO by paying the customers.
Thereafter, NACOCO seeks to recover the above sum of P1,343,274.52 from general manager and
board chairman Maximo M. Kalaw, and directors Juan Bocar, Casimiro Garcia and Leonor Moll. It
charges Kalaw with negligence under Article 1902 of the old Civil Code (now Article 2176, new Civil
Code); and defendant board members, including Kalaw, with bad faith and/or breach trust for having
approved the contracts.

ISSUE:

Whether or not Kalaw may be held liable by NACOCO for the debts the corporation incurred
under his administration.

RULING:

NO.

They were done with implied authority from the BOD. These previous contracts, it should be
stressed, were signed by Kalaw without prior authority from the board. Said contracts were known all
along to the board members. Nothing was said by them. The aforesaid contracts stand to prove one
thing. Obviously NACOCO board met the difficulties attendant to forward sales by leaving the adoption
of means to end, to the sound discretion of NACOCO's general manager Maximo M. Kalaw.
Settled jurisprudence has it that where similar acts have been approved by the directors as a
matter of general practice, custom, and policy, the general manager may bind the company without
formal authorization of the board of directors. In varying language, existence of such authority is
established, by proof of the course of business, the usages and practices of the company and by the
knowledge which the board of directors has, or must be presumed to have, of acts and doings of its
subordinates in and about the affairs of the corporation.
Authorities, great in number, are one in the idea that "ratification by a corporation of an
unauthorized act or contract by its officers or others relates back to the time of the act or contract
ratified, and is equivalent to original authority;" and that "[t]he corporation and the other party to the
transaction are in precisely the same position as if the act or contract had been authorized at the time."
The language of one case is expressive: "The adoption or ratification of a contract by a corporation is
nothing more nor less than the making of an original contract. The theory of corporate ratification is
predicated on the right of a corporation to contract, and any ratification or adoption is equivalent to a
grant of prior authority.

131 | P a g e
Law 321_Corporation LAW_ Case Digest

TRINIDAD FRANCISCO
vs.
GOVERNMENT SERVICE INSURANCE SYSTEM
GR L-18287, 30 March 1963

FACTS:

On 10 October 1956, the plaintiff, Trinidad J. Francisco, in consideration of a loan in the amount
of P400,000.00, out of which the sum of P336,100.00 was released to her, mortgaged in favor of the
defendant, GSIS, a parcel of land containing an area of 18,232 square meters, with twenty-one (21)
bungalows, known as Vic-Mari Compound, located at Baesa, Quezon City, payable within ten (10) years
in monthly installments of P3,902.41, and with interest of 7% per annum compounded monthly.
On 6 January 1959, the System extrajudicially foreclosed the mortgage on the ground that up to
that date the plaintiff-mortgagor was in arrears on her monthly installments in the amount of
P52,000.00. Payments made by the plaintiff at the time of foreclosure amounted to P130,000.00. The
System itself was the buyer of the property in the foreclosure sale.
Trinidad’s father, Vicente Francisco, offered to pay part of the arrears with P30,000.00 with the
remainder of the arrears plus the balance of the loan shall be paid with the proceeds of the rents of the
property which shall be administered by GSIS. GSIS informed the plaintiff through a telegraph made its
General Manager Rodolfo Andal and accepted the tender of P30,000.00 but did not take over the
administration of the property. Meanwhile, Trinidad received the rents amounting to P44,000.00 and
remitted the same to GSIS, who issued a receipt thereto.
GSIS, however, notified the appellant that the time for redemption is about to expire and
requested the Franciscos to offer them a system of payment. Trinidad questioned the action, standing by
the validity of the proposal they made with GSIS, whose acceptance of the payment amounted to
estoppel. he defendant answered, pleading that the binding acceptance of Francisco's offer was the
resolution of the Board, and that Andal's telegram, being erroneous, should be disregarded.

ISSUE:

Whether or not the approval of the proposal made by GSIS is valid.

RULING:

YES.

The terms of the offer were clear, and over the signature of defendant's general manager, Rodolfo
Andal, plaintiff was informed telegraphically that her proposal had been accepted. There was nothing in
the telegram that hinted at any anomaly, or gave ground to suspect its veracity, and the plaintiff,
therefore, cannot be blamed for relying upon it. There is no denying that the telegram was within
Andal's apparent authority, but the defense is that he did not sign it, but that it was sent by the Board
Secretary in his name and without his knowledge. Assuming this to be true, how was appellee to know
it? Corporate transactions would speedily come to a standstill were every person dealing with a
corporation held duty-bound to disbelieve every act of its responsible officers, no matter how regular
they should appear on their face
If a private corporation intentionally or negligently clothes its officers or agents with apparent
power to perform acts for it, the corporation will be estopped to deny that such apparent authority is
real, as to innocent third persons dealing in good faith with such officers or agents. Knowledge of facts
acquired or possessed by an officer or agent of a corporation in the course of his employment, and in
relation to matters within the scope of his authority, is notice to the corporation, whether he
communicates such knowledge or not.

RURAL BANK OF MILAOR (CAMARINES SUR)


vs.
FRANCISCA OCFEMIA, et. al.
GR 137686, 08 February 2000

FACTS:

132 | P a g e
Law 321_Corporation LAW_ Case Digest

The spouses Felicisimo Ocfemia and Juanita Arellano Ocfemia were not able to redeem the
mortgaged properties consisting of seven (7) parcels of land from Milaor and so the mortgage was
foreclosed and thereafter ownership thereof was transferred to the bank. Out of the seven (7) parcels
that were foreclosed, five (5) of them are in the possession of the Ocfemias because these were sold by
the [petitioner] bank to the parents of Marife Ocfemia Niño as evidenced by a Deed of Sale executed in
January 1988.
Marife went to the Register of Deeds of Camarines Sur with the Deed of Sale (Exh. C) in order to
have the same registered. The Register of Deeds, however, informed her that the document of sale
cannot be registered without a board resolution of the Bank. Marife then went to the bank, showed to it
the Deed of Sale, the tax declaration and receipt of tax payments and requested the bank for a board
resolution so that the property can be transferred to the name of Marife’s parents Renato Ocfemia and
Francisca Ocfemia.
The bank, after requiring so many requirements and making so many alibis to Marife, refused to
issue the board resolution. It claims that its bank manager Fe Tena did not have authority to sell the
properties to the Ocfemias therefore rendering the deed of sale invalid.

ISSUE:

Whether or not the bank manager has authority to act on behalf of the bank.

RULING:

YES.

There was an apparent authority bestowed with Tena. The bank acknowledged, by its own acts or
failure to act, the authority of Fe S. Tena to enter into binding contracts. After the execution of the Deed
of Sale, respondents occupied the properties in dispute and paid the real estate taxes due thereon. If the
bank management believed that it had title to the property, it should have taken some measures to
prevent the infringement or invasion of its title thereto and possession thereof.
Likewise, Tena had previously transacted business on behalf of the bank, and the latter had
acknowledged her authority. A bank is liable to innocent third persons where representation is made in
the course of its normal business by an agent like Manager Tena, even though such agent is abusing
her authority. Clearly, persons dealing with her could not be blamed for believing that she was
authorized to transact business for and on behalf of the bank.
In this light, the bank is estopped from questioning the authority of the bank manager to enter
into the contract of sale. If a corporation knowingly permits one of its officers or any other agent to act
within the scope of an apparent authority, it holds the agent out to the public as possessing the power
to do those acts; thus, the corporation will, as against anyone who has in good faith dealt with it
through such agent, be estopped from denying the agent’s authority.
More so, the bank is in default for failing to answer the complaint of the Ocfemias within the
reglamentary period without any justifiable excuse.

 Personal Liability of Directors and Other Corporate Officers

EVER ELECTRICAL MANUFACTURING, INC., (EEMI) and VICENTE GO


vs.
SAMAHANG MANGGAGAWA NG EVER ELECTRICAL/NAMAWU LOCAL
G.R. No. 194795. June 13, 2012.

FACTS:

EEMI is a corporation engaged in the business of manufacturing electrical parts and supplies.
EEMI was one of those who suffered huge losses during the Asian financial crisis. In 1996, EEMI
obtained a loan from UCPB. As security, EEMI’s land and improvements, including the factory, were
mortgaged to UCPB. EEMI’s business suffered further losses due to the continued entry of cheaper
goods from China and other Asian countries. Later, Orient Bank, where EEMI invested 500MIO, went
out of business. As aresult, EEMI defaulted on its loan with UCPB. In an attempt to save the company,

133 | P a g e
Law 321_Corporation LAW_ Case Digest

EEMI entered into a dacion en pago arrangement with UCPB which, in effect, transferred ownership of
the company’s property to UCPB. Since UCPB’s policy prohibited EEMI from leasing the premises
directly with UCPB, UCPB agreed to lease it to an affiliate corporation, EGO Electrical Supply Co, Inc.
(EGO), for and in behalf of EEMI. However, UCPB later instituted an unlawful detainer suit against
EGO, wherein UCPB won. The Sheriff implemented the writ of execution by closing the premises and, as
a result, EEMI’s employees were prevented from entering the factory. Aggrieved, UNION filed a complaint
for illegal dismissal with prayer for payment of 13th month pay, separation pay, damages, and
attorney’s fees.

ISSUE:

Whether or not Vicente Go should be held solidarily liable with EEMI.

RULING:

NO.

As a general rule, corporate officers should not be held solidarily liable with the corporation for
separation pay for it is settled that a corporation is invested by law with a personality separate and
distinct from those of the persons composing it as well as from that of any other legal entity to which it
may be related. Mere ownership by a single stockholder or by another corporation of all or nearly all of
the capital stock of a corporation is not of itself sufficient ground for disregarding the separate corporate
personality.
A corporation is invested by law with a personality separate and distinct from those of the
persons composing it as well as from that of any other legal entity to which it may be related. Corporate
directors and officers become solidarily liable with the corporation for the termination of employees done
with malice or bad faith. It stressed that bad faith does not connote bad judgment or negligence; it
imports a dishonest purpose or some moral obliquity and conscious doing of wrong; it means breach of
a known duty through some motive or interest or ill will; it partakes of the nature of fraud.

HARPOON MARINE SERVICES, Inc. and JOSE LIDO T. ROSIT


vs.
FERNAN H. FRANCISCO
G.R. No. 167751. March 2, 2011

FACTS:

Respondent averred that he was unceremoniously dismissed by petitioner Rosit. He was informed
that the company could no longer afford his salary and that he would be paid his separation pay and
accrued commissions. Respondent nonetheless continued to report for work. A few days later, however,
he was barred from entering the company premises. Relying on the promise of petitioner Rosit,
respondent went to the office on June 30, 2001 to receive his separation pay and commissions, but
petitioner Rosit offered only his separation pay. Respondent refused to accept it and also declined to
sign a quitclaim. After several unheeded requests, respondent, through his counsel, sent a demand
letter dated September 24, 2001 to petitioners asking for payment of P70,000.00, which represents his
commissions for the seven boats constructed and repaired by the company under his supervision. In a
letter-reply dated September 28, 2001, petitioners denied that it owed respondent any commission,
asserting that they never entered into any contract or agreement for the payment of commissions.
Hence, on October 24, 2001, respondent filed an illegal dismissal complaint praying for the payment of
his backwages, separation pay, unpaid commissions, moral and exemplary damages and attorney’s fees.

ISSUE:

134 | P a g e
Law 321_Corporation LAW_ Case Digest

Whether or not the President is solidarily liable with the corporation.

RULING:

NO.

Though the Court found that Respondent was illegally dismissed, it held that the President of the
Petitioner Corporation should not be held solidarily liable with Petitioner Corporation. Obligations
incurred by corporate officers, acting as such corporate agents, are not theirs but the direct
accountabilities of the corporation they represent. Thus, they should not be generally held jointly and
solidarily liable with the corporation. The general rule is grounded on the theory that a corporation has
a legal personality separate and distinct from the persons comprising it.
As exceptions to the general rule, solidary liability may be imposed: (1) When directors and
trustees or, in appropriate cases, the officers of a corporation –(a) vote for or assent to [patently]
unlawful acts of the corporation; (b) act in bad faith or with gross negligence in directing the corporate
affairs; (c) are guilty of conflict of interest to the prejudice of the corporation, its stockholders or
members, and other persons; (2) When the director or officer has consented to the issuance of watered
stock or who, having knowledge thereof, did not forthwith file with the corporate secretary his written
objection thereto; (3) When a director, trustee or officer has contractually agreed or stipulated to hold
himself personally and solidarily liable with the corporation; (4) When a director, trustee or officer is
made, by specific provision of law, personally liable for his corporate action. To warrant the piercing of
the veil of corporate fiction, the officer’s bad faith or wrongdoing must be established clearly and
convincingly as bad faith is never presumed.

ARNEL TY, et al.


vs.
NBI AGENT MARTIN JAMIL, et al.
GR 182147, 15 December 2010

FACTS:

Acting on a complaint filed by various LPG manufacturers, the NBI conducted a surveillance on
the activities of Omni Gas, which is being accused of selling LPG tanks without the required permit and
below its required standards. Agents De Jemil and Kawada attested to conducting surveillance of Omni
and brought eight branded LPG cylinders of Shellane, Petron Gasul, Totalgaz, and Superkalan Gaz to
Omni for refilling. The branded LPG cylinders were refilled, for which agents paid P1,582 as evidenced
by Sales Invoice No. 90040issued by Omni on April 15, 2004. The refilled LPG cylinders were without
LPG valve seals and one of the cylinders was actually underfilled, as found by LPG Inspector Noel N.
Navio of the Liquefied Petroleum Gas Industry Association (LPGIA) who inspected the eight branded LPG
cylinders.
The NBI's test-buy yielded positive results for violations of BP 33, Section 2(a) in relation to Secs.
3(c) and 4, i.e., refilling branded LPG cylinders without authority; and Sec. 2(c) in relation to Sec. 4, i.e.,
under delivery or under filling of LPG cylinders.
Petitioners Arnel Ty, Marie Antonette Ty, Jason Ong, Willy Dy, and Alvin Ty questioned the case
against that, claiming that being mere directors, they are not liable to the case filed against Omni for
they are not in charge of the management of the said entity.

ISSUE:

Whether or not petitioners may be held liable for the actions of Omni.

RULING:

NO.

135 | P a g e
Law 321_Corporation LAW_ Case Digest

Only Arnel Ty may be held liable in his capacity as president of Omni, but not the other directors.
The corporate powers of a corporation are reposed in the board of directors under the first paragraph of
Sec. 23 of the Corporation Code, it is of common knowledge and practice that the board of directors is
not directly engaged or charged with the running of the recurring business affairs of the corporation.
Depending on the powers granted to them by the Articles of Incorporation, the members of the board
generally do not concern themselves with the day-to-day affairs of the corporation, except those
corporate officers who are charged with running the business of the corporation and are concomitantly
members of the board, like the President. Section 25of the Corporation Code requires the president of a
corporation to be also a member of the board of directors.
Evidently, petitioner Arnel, as President, who manages the business affairs of Omni, can be held
liable for probable violations by Omni of BP 33, as amended. The fact that petitioner Arnel is ostensibly
the operations manager of Multi-Gas Corporation, a family owned business, does not deter him from
managing Omni as well. It is well-settled that where the language of the law is clear and unequivocal, it
must be taken to mean exactly what it says. As to the other petitioners, unless otherwise shown that
they are situated under the catch-all "such other officer charged with the management of the business
affairs," they may not be held liable under BP 33, as amended, for probable violations. Consequently,
with the exception of petitioner Arnel, the charges against other petitioners must perforce be dismissed
or dropped.
Also, under BP 33 (which regulates the production and sale of LPG), Directors are not among
those enumerated as criminally liable for the acts of the corporation.

QUEENSLAND-TOKYO COMMODITIES et al.


vs.
THOMAS GEORGE
GR 172727, 08 September 2010

FACTS:

QTCI is a duly licensed broker engaged in the trading of commodity futures. In 1995, Guillermo
Mendoza, Jr. (Mendoza) and Oniler Lontoc (Lontoc) of QTCI met with respondent Thomas George
(respondent), encouraging the latter to invest with QTCI. On July 7, 1995, upon Mendoza's prodding,
respondent finally invested with QTCI. On the same day, Collado, in behalf of QTCI, and respondent
signed the Customer's Agreement. Forming part of the agreement was the Special Power of Attorney
executed by respondent, appointing Mendoza as his attorney-in-fact with full authority to trade and
manage his account.
On June 20, 1996, the Securities and Exchange Commission (SEC) issued a Cease-and-Desist
Order against QTCI. Alarmed by the issuance of the CDO, respondent demanded from QTCI the return
of his investment, but it was not heeded.
QTCI claimed that they were not aware of, nor were they privy to, any arrangement which
resulted in the account of respondent being handled by unlicensed brokers. They pointed out that
respondent transacted business with QTCI for almost a year, without questioning the license or the
authority of the traders handling his account, rendering him estopped. It was only after it became
apparent that QTCI could no longer resume its business transactions by reason of the CDO that
respondent raised the alleged lack of authority of the brokers or traders handling his account.

ISSUE:

Whether or not QTCI should be held liable for the loss incurred by George in the investment he
made with the corporation.

RULING:

YES.

It recognized Mendoza and Collado as its brokers. Petitioners did not object to, and in fact
recognized, Mendoza's appointment as respondent's attorney-in-fact. Collado, in behalf of QTCI,
concluded the Customer's Agreement despite the fact that the appointed attorney-in-fact was not a
licensed dealer. Worse, petitioners permitted Mendoza to handle respondent's account.

136 | P a g e
Law 321_Corporation LAW_ Case Digest

Doctrine dictates that a corporation is invested by law with a personality separate and distinct
from those of the persons composing it, such that, save for certain exceptions, corporate officers who
entered into contracts in behalf of the corporation cannot be held personally liable for the liabilities of
the latter. Personal liability of a corporate director, trustee, or officer, along (although not necessarily)
with the corporation, may validly attach, as a rule, only when - (1) he assents to a patently unlawful act
of the corporation, or when he is guilty of bad faith or gross negligence in directing its affairs, or when
there is a conflict of interest resulting in damages to the corporation, its stockholders, or other persons;
(2) he consents to the issuance of watered down stocks or who, having knowledge thereof, does not
forthwith file with the corporate secretary his written objection thereto; (3) he agrees to hold himself
personally and solidarily liable with the corporation; or (4) he is made by a specific provision of law
personally answerable for his corporate action.
Romeo Lau, as president of [petitioner] QTCI, cannot feign innocence on the existence of these
unlawful activities within the company, especially so that Collado, himself a ranking officer of QTCI, is
involved in the unlawful execution of customers orders. Lau, being the chief operating officer, cannot
escape the fact that had he exercised a modicum of care and discretion in supervising the operations of
QTCI, he could have detected and prevented the unlawful acts of Collado and Mendoza.

WENSHA SPA CENTER and/or XU ZHI JIE


vs.
LORETA YUNG
GR 185122, 16 August 2010

FACTS:

Loreta stated that she used to be employed by Manmen Services Co., Ltd. where Xu was a client.
Xu was apparently impressed by Loreta's performance. After he established Wensha, he convinced
Loreta to transfer and work at Wensha. Loreta was initially reluctant to accept Xu's offer because her
job at Manmen was stable and she had been with Manmen for seven years. But Xu was persistent and
offered her a higher pay. Enticed, Loreta resigned from Manmen and transferred to Wensha as Xu's
personal assistant and interpreter. Loreta introduced positive changes to Wensha which resulted in
increased business. This pleased Xu so that she was promoted to the position of Administrative
Manager.
Wensha and Xu denied illegally terminating Loreta's employment. They claimed that two months
after Loreta was hired, they received various complaints against her from the employees so that on
August 10, 2004, they advised her to take a leave of absence for one month while they conducted an
investigation on the matter. Based on the results of the investigation, they terminated Loreta's
employment on August 31, 2004 for loss of trust and confidence.
The Court ruled that indeed Loreta was illegally dismissed because Wensha failed to substantially
prove its claim that she committed wrongdoings with Wensha’s employees, and that Loreta’s testimony
as to her termination because her feng shui aura does not match that of Xu is consistent. Xu failed to
duly prove a valid ground for the loss of trust and confidence with Loreta. Question lies if Xu should be
held liable together with Wensha.

ISSUE:

Whether or not Xu is liable together with the corporation.

RULING:

NO.

Xu is not liable together with the corporation. Elementary is the rule that a corporation is
invested by law with a personality separate and distinct from those of the persons composing it and
from that of any other legal entity to which it may be related. "Mere ownership by a single stockholder or
by another corporation of all or nearly all of the capital stock of a corporation is not of itself sufficient
ground for disregarding the separate corporate personality."
In labor cases, corporate directors and officers may be held solidarily liable with the corporation
for the termination of employment only if done with malice or in bad faith.Bad faith does not connote
bad judgment or negligence; it imports a dishonest purpose or some moral obliquity and conscious
doing of wrong; it means breach of a known duty through some motive or interest or ill will; it partakes
of the nature of fraud.
There is no finding of bad faith or malice on the part of Xu. There is, therefore, no justification
for such a ruling. To sustain such a finding, there should be an evidence on record that an officer or

137 | P a g e
Law 321_Corporation LAW_ Case Digest

director acted maliciously or in bad faith in terminating the services of an employee.Moreover, the
finding or indication that the dismissal was effected with malice or bad faith should be stated in the
decision itself.

CEBU MACTAN MEMBERS CENTER


vs.
MASAHIRO TSUKAHARA
GR 159624, 17 July 2009

FACTS:

In February 1994, petitioner Cebu Mactan Members Center, Inc. (CMMCI), through Mitsumasa
Sugimoto (Sugimoto), the President and Chairman of the Board of Directors of CMMCI, obtained a loan
amounting to P6,500,000 from respondent Masahiro Tsukahara. As payment for the loan, CMMCI
issued seven postdated checks of CMMCI payable to Tsukahara. On 13 April 1994, CMMCI, through
Sugimoto, obtained another loan amounting to P10,000,000 from Tsukahara. Sugimoto executed and
signed a promissory note in his capacity as CMMCI President and Chairman, as well as in his personal
capacity.
Upon maturity, the seven checks were presented for payment by Tsukahara, but the same were
dishonored by PNB, the drawee bank. After several failed attempts to collect the loan amount totaling
P16,500,000, Tsukahara filed the instant case for collection of sum of money against CMMCI and
Sugimoto.
Tsukahara alleged that the amount of P16,500,000 was used by CMMCI for the improvement of
its beach resort, which included the construction of a wave fence, the purchase of air conditioners and
curtains, and the provision of salaries of resort employees. He also asserted that Sugimoto, as the
President of CMMCI, "has the power to borrow money for said corporation by any legal means
whatsoever and to sign, endorse and deliver all checks and promissory notes on behalf of the
corporation."
CMMCI, on the other hand, denied borrowing the amount from Tsukahara, and claimed that both
loans were personal loans of Sugimoto. The company also contended that if the loans were those of
CMMCI, the same should have been supported by resolutions issued by CMMCI's Board of Directors.

ISSUE:

Whether or not CMMCI is liable to Tsukahara.

RULING:

YES.

It is because Sugimoto’s actions are binding with CMMCI. A corporation, being a juridical entity,
may act through its board of directors which exercises almost all corporate powers, lays down all
corporate business policies, and is responsible for the efficiency of management. The general rule is
that, in the absence of authority from the board of directors, no person, not even its officers, can validly
bind a corporation.
In this case, the corporate by-laws of CMMCI explicitly empowered the President to enter into loans with
third persons on behalf of the corporation without the necessity of a board resolution. By-laws of a
corporation should be construed and given effect according to the general rules governing the
construction of contracts. They, as the self-imposed private laws of a corporation, have, when valid,
substantially the same force and effect as laws of the corporation, as have the provisions of its charter
insofar as the corporation and the persons within it are concerned. They are in effect written into the
charter and in this sense; they become part of the fundamental law of the corporation. And the
corporation and its directors (or trustees) and officers are bound by and must comply with them.
The corporation is now estopped from denying the authority of its president to bind the former
into contractual relations.

138 | P a g e
Law 321_Corporation LAW_ Case Digest

ARMANDO DAVID
vs.
NFLU, MARIVELES APPAREL CORPORATION LABOR UNION
GR 148263, 148271-72, 21 April 2009

FACTS:

MAC hired David as IMPEX and Treasury Manager on 16 September 1988. David began serving
as MAC's President in May 1990. David served as President in the nature of a nominee as he did not
own any of MAC's shares. David tendered his irrevocable resignation from MAC on 30 September 1993.
David's resignation was made effective on 15 October 1993.
In a complaint for illegal dismissal dated 12 August 1993, National Federation of Labor Unions
(NAFLU) and Mariveles Apparel Corporation Labor Union (MACLU) alleged that MAC ceased operations
on 8 July 1993 without prior notice to its employees. MAC allegedly gave notice of its closure on the
same day that it ceased operations. MACLU and NAFLU further alleged that, at the time of MAC's
closure, employees who had rendered one to two weeks work were not paid their corresponding salaries.
Atty. Joshua Pastores, as MAC's counsel, submitted a position paper dated 21 February 1994
and argued that Carag and David should not be held liable because MAC is owned by a consortium of
banks. Carag's and David's ownership of MAC shares only served to qualify them to serve as officers in
MAC.

ISSUE:

Whether or not David may be held liable for the illegal dismissal of MAC employees.

RULING:

NO.

It is improper to hold David liable for MAC's obligations to its employees. However, Article 212(e)
of the Labor Code, by itself, does not make a corporate officer personally liable for the debts of the
corporation because Section 31 of the Corporation Code is still the governing law on personal liability of
officers for the debts of the corporation. Section 31 of the Corporation Code provides: Directors or
trustees who willfully and knowingly vote for or assent to patently unlawful acts of the corporation or
who are guilty of gross negligence or bad faith in directing the affairs of the corporation or acquire any
personal or pecuniary interest in conflict with their duty as such directors, or trustees shall be liable
jointly and severally for all damages resulting therefrom suffered by the corporation, its stockholders or
members and other persons.
There was no showing of David willingly and knowingly voting for or assenting to patently unlawful acts
of the corporation, or that David was guilty of gross negligence or bad faith.
Also, the NLRC never gained jurisdiction over David for there was an invalid service of summons.

HILARIO SORIANO AND ROSARIO ILAGAN


vs.
PEOPLE, BANGKO SENTRAL, PDIC
GR 159517-18, 30 June 2009

FACTS:

Hilario P. Soriano (Soriano) and Rosalinda Ilagan (Ilagan) were the President and General
Manager, respectively, of the Rural Bank of San Miguel (Bulacan), Inc. (RBSM). Allegedly, on June 27,

139 | P a g e
Law 321_Corporation LAW_ Case Digest

1997 and August 21, 1997, during their incumbency as president and manager of the bank, petitioners
indirectly obtained loans from RBSM. They falsified the loan applications and other bank records, and
made it appear that Virgilio J. Malang and Rogelio Mañaol obtained loans of P15,000,000.00 each, when
in fact they did not.
Criminal charges were filed against them. They sought for its dismissal because their action does
not amount to any criminal action, and if it does, it will only render them liable civilly. Also, their single
act could not amount to multiple offenses.

ISSUE:

Whether or not the petitions may be held liable for their actions.

RULING:

YES.

They committed grave abuse of discretion in the exercise of their duties. As aptly pointed out by
the BSP in its memorandum, there are differences between the two (2) offenses. A DOSRI violation
consists in the failure to observe and comply with procedural, reportorial or ceiling requirements
prescribed by law in the grant of a loan to a director, officer, stockholder and other related interests in
the bank, i.e. lack of written approval of the majority of the directors of the bank and failure to enter
such approval into corporate records and to transmit a copy thereof to the BSP supervising department.
The elements of abuse of confidence, deceit, fraud or false pretenses, and damage, which are essential to
the prosecution for estafa, are not elements of a DOSRI violation. The filing of several charges against
Soriano was, therefore, proper.

CEBU COUNTRY CLUB et al.


vs. RICARDO ELIZAGAQUE
GR 160273, 18 January 2008

FACTS:

Sometime in 1987, San Miguel Corporation, a special company proprietary member of CCCI,
designated respondent Ricardo F. Elizagaque, its Senior Vice President and Operations Manager for the
Visayas and Mindanao, as a special non-proprietary member. The designation was thereafter approved
by the CCCI’s Board of Directors.
In 1996, respondent filed with CCCI an application for proprietary membership. As the price of a
proprietary share was around the P5 million range, Benito Unchuan, then president of CCCI, offered to
sell respondent a share for only P3.5 million. Respondent, however, purchased the share of a certain

140 | P a g e
Law 321_Corporation LAW_ Case Digest

Dr. Butalid for only P3 million. Consequently, on September 6, 1996, CCCI issued Proprietary
Ownership Certificate No. 1446 to respondent. On August 1, 1997, respondent received a letter from
Julius Z. Neri, CCCI’s corporate secretary, informing him that the Board disapproved his application for
proprietary membership.
On August 6, 1997, Edmundo T. Misa, on behalf of respondent, wrote CCCI a letter of
reconsideration. As CCCI did not answer, respondent, on October 7, 1997, wrote another letter of
reconsideration. Still, CCCI kept silent. On November 5, 1997, respondent again sent CCCI a letter
inquiring whether any member of the Board objected to his application. Again, CCCI did not reply.

ISSUE:

Whether or not the Board of Directors of Cebu Country Club are liable to Elizagawue for
damages.

RULING:

YES.

There is bad faith among the members of the board. As shown by the records, the Board adopted
a secret balloting known as the “black ball system” of voting wherein each member will drop a ball in the
ballot box. A white ball represents conformity to the admission of an applicant, while a black ball
means disapproval. Pursuant to Section 3(c), as amended, cited above, a unanimous vote of the
directors is required. When respondent’s application for proprietary membership was voted upon during
the Board meeting on July 30, 1997, the ballot box contained one (1) black ball. Thus, for lack of
unanimity, his application was disapproved.
Obviously, the CCCI Board of Directors, under its Articles of Incorporation, has the right to
approve or disapprove an application for proprietary membership. But such right should not be
exercised arbitrarily.
It is thus clear that respondent was left groping in the dark wondering why his application was
disapproved. He was not even informed that a unanimous vote of the Board members was required.
When he sent a letter for reconsideration and an inquiry whether there was an objection to his
application, petitioners apparently ignored him. Certainly, respondent did not deserve this kind of
treatment. Having been designated by San Miguel Corporation as a special non-proprietary member of
CCCI, he should have been treated by petitioners with courtesy and civility. At the very least, they
should have informed him why his application was disapproved.
The exercise of a right, though legal by itself, must nonetheless be in accordance with the proper
norm. When the right is exercised arbitrarily, unjustly or excessively and results in damage to another,
a legal wrong is committed for which the wrongdoer must be held responsible.

CALTEX (CHEVRON) PHILIPPINES, INC.


vs.
NLRC and ROMEO STO. TOMAS
GR 159641, 15 October 2007

FACTS:

In a letter dated October 21, 1996, Caltex informed the Department of Labor and Employment
(DOLE) of its plan to implement a redundancy program in its Marketing Division and some departments
in its Batangas Refinery for the period starting October 1996 to December 1998. The letter alleged that
the redundancy program is a response to the market situation which constrained petitioner to
rationalize and simplify its business processes.
Santo Tomas was notified of his termination effective July 31, 1997 due to the redundancy of his
position and awarded him a separation package in the amount of P559,458.90. On June 8, 1998,
respondent filed with the Labor Arbiter a complaint for illegal dismissal against petitioner and its
President and Chief Executive Officer, Mr. Clifton Hon. Private respondent alleged that: being
petitioner’s regular employee, he is entitled to security of tenure; he did not commit any serious
misconduct, willful disobedience, gross and habitual neglect of duty or fraud and willful breach of trust
to warrant the penalty of dismissal from employment; there was no independent proof or evidence
presented by petitioner to substantiate its claim of redundancy nor was he afforded due process as he
was not given any opportunity to present his side; he was dismissed due to his active participation in

141 | P a g e
Law 321_Corporation LAW_ Case Digest

union activities; petitioner opened positions for hiring some of which offered jobs that are the same as
what private respondent was performing; petitioner failed to give written notice to him and DOLE at
least one month before the intended date of termination as required by the Labor Code.

ISSUE:

Whether or not Santo Tomas was illegally dismissed.

RULING:

YES.

Caltex failed to prove the necessity of the redundancy program. It is the rule that the
characterization of an employee’s services as no longer necessary or sustainable, and therefore, properly
terminable, is an exercise of business judgment on the part of the employer, and that the wisdom or
soundness of such characterization or decision is not subject to discretionary review. However, such
characterization may be rejected if the same is found to be in violation of law or is arbitrary or
malicious.
In the instant case, there was no substantial evidence presented by petitioner to justify private
respondent's dismissal due to redundancy. As correctly found by the CA, petitioner’s evidence to show
redundancy merely consisted of a copy of petitioner’s letter to the DOLE informing the latter of its
intention to implement a redundancy program and nothing more. The letter which merely stated that
petitioner undertook a review, restructuring and streamlining of its organization which resulted in
consolidation, abolition and outsourcing of certain functions; and which resulted in identified and
redundant positions instead of simplifying its business process restructuring, does not satisfy the
requirement of substantial evidence, that is, the amount of evidence which a reasonable mind might
accept as adequate to justify a conclusion.
Petitioner failed to demonstrate the superfluity of private respondent’s position as there was
nothing in the records that would establish any concrete and real factors recognized by law and relevant
jurisprudence, such as overhiring of workers, decreased volume of business, or dropping of a particular
product line or service activity previously manufactured or undertaken by the enterprise, which were
adopted by petitioner in implementing the redundancy program.

142 | P a g e
Law 321_Corporation LAW_ Case Digest

ATRIUM MANAGEMENT CORPORATION


vs.
COURT OF APPEALS, E.T. HENRY AND CO., et al.
GR 109491, 28 February 2001

FACTS:

Hi-Cement Corporation through its corporate signatories, petitioner Lourdes M. de Leon,


treasurer, and the late Antonio de las Alas, Chairman, issued checks in favor of E.T. Henry and Co. Inc.,
as payee. E.T. Henry and Co., Inc., in turn, endorsed the four checks to petitioner Atrium Management
Corporation for valuable consideration. Upon presentment for payment, the drawee bank dishonored all
four checks for the common reason "payment stopped". Atrium, thus, instituted an action after its
demand for payment of the value of the checks was denied.
At the trial, Atrium presented as its witness Carlos C. Syquia who testified that in February 1981,
Enrique Tan of E.T. Henry approached Atrium for financial assistance, offering to discount four RCBC
checks in the total amount of P2 million, issued by Hi-Cement in favor of E.T. Henry. Atrium agreed to
discount the checks, provided it be allowed to confirm with Hi-Cement the fact that the checks
represented payment for petroleum products which E.T. Henry delivered to Hi-Cement. Carlos C. Syquia
identified two letters, dated February 6, 1981 and February 9, 1981 issued by Hi-Cement through
Lourdes M. de Leon, as treasurer, confirming the issuance of the four checks in favor of E.T. Henry in
payment for petroleum products.
Lourdes M. de Leon claimed she is not solidarilly liable with Hi-Cement for the amount of the
check and that Atrium was an ordinary holder, not a holder in due course of the rediscounted checks.

ISSUE:

Whether or not de Leon may be held liable.

RULING:

YES.

Due to negligence. Lourdes M. de Leon and Antonio de las Alas as treasurer and Chairman of Hi-
Cement were authorized to issue the checks. However, Ms. de Leon was negligent when she signed the
confirmation letter requested by Mr. Yap of Atrium and Mr. Henry of E.T. Henry for the rediscounting of
the crossed checks issued in favor of E.T. Henry. She was aware that the checks were strictly endorsed
for deposit only to the payee's account and not to be further negotiated. What is more, the confirmation
letter contained a clause that was not true, that is, "that the checks issued to E.T. Henry were in
payment of Hydro oil bought by Hi-Cement from E.T. Henry". Her negligence resulted in damage to the
corporation. Hence, Ms. de Leon may be held personally liable therefor.
"Personal liability of a corporate director, trustee or officer along (although not necessarily) with
the corporation may so validly attach, as a rule, only when: He assents (a) to a patently unlawful act of
the corporation, or (b) for bad faith or gross negligence in directing its affairs, or (c) for conflict of
interest, resulting in damages to the corporation, its stockholders or other persons; He consents to the
issuance of watered down stocks or who, having knowledge thereof, does not forthwith file with the
corporate secretary his written objection thereto; He agrees to hold himself personally and solidarily
liable with the corporation; or He is made, by a specific provision of law, to personally answer for his
corporate action.
However, as to the claim of Atrium, it cannot be upheld because it is not a holder of the check in
due course due to the fact that the same was crossed in favor of E.T. Henry, and therefore only payable
to the latter’s account.

ARB CONSTRUCTION and MARK MOLINA


vs.
COURT OF APPEALS, TBS SECURITY AND INVESTIGATION
GR 126554, 31 May 2000

FACTS:

143 | P a g e
Law 321_Corporation LAW_ Case Digest

On 15 August 1993 TBS Security and Investigation Agency (TBSS) entered into two (2) Service
Contracts with ARBC wherein TBSS agreed to provide and post security guards in the five (5)
establishments being maintained by ARBC. The contract shall be effective for one (1) year and shall be
considered renewed for the same period unless the same is terminated after a notice is given to the
parties thirty (30) days in advance.
In a letter dated 23 February 1994 ARBC informed TBSS of its desire to terminate the Service
Contracts effective thirty (30) days after receipt of the letter. Also, in a letter dated 22 March 1994,
ARBC through its Vice President for Operations, Mark Molina, informed TBSS that it was replacing its
security guards with those of Global Security Investigation Agency (GSIA).
In response to both letters, TBSS informed ARBC that the latter could not preterminate the
Service Contracts nor could it post security guards from GSIA as it would run counter to the provisions
of their service contracts. Nevertheless, Molina decreased the security guards to only one (1) as a right
provided under the service contract. TBSS thereafter filed a case for breach of contract against ARBC
and Mark Molina.

ISSUE:

Whether or not Mark Molina should be held liable together with ARBC.

RULING:

NO.

He merely acted within his capacity as an officer of the corporation. It is basic that a corporation
is invested by law with a personality separate and distinct from those of the persons composing it as
well as from that of any other legal entity to which it may be related. As a general rule, a corporation
may not be made to answer for acts or liabilities of its stockholders or those of the legal entities to which
it may be connected and vice versa. However, the veil of corporate fiction may be pierced when it is used
as a shield to further an end subversive of justice; or for purposes that could not have been intended by
the law that created it; or to defeat public convenience, justify wrong, protect fraud, or defend crime; or
to perpetuate deception; or as an alter ego, adjunct or business conduit for the sole benefit of the
stockholders.
On the basis hereof, petitioner Molina could not be held jointly and severally liable for any
obligation which petitioner ARBC may be held accountable for, absent any proof of bad faith or malice
on his part. Corollarily, it is also incorrect on the part of the Court of Appeals to conclude that there was
a sufficient cause of action against Molina as to make him personally liable for his actuations as Vice
President for Operations of ARBC.

RUFINA LUY LIM


vs.
COURT OF APPEALS, AUTO TRUCK TBA CORP., et al.
GR 124715, 24 January 2000

FACTS:

On 11 June 1994, Pastor Y. Lim died intestate. Herein petitioner, as surviving spouse and duly
represented by her nephew George Luy, filed on 17 March 1995, a joint petition for the administration of
the estate of Pastor Y. Lim before the Regional Trial Court of Quezon City.
Private respondent corporations, whose properties were included in the inventory of the estate of
Pastor Y. Lim, then filed a motion6 for the lifting of lis pendens and motion for exclusion of certain
properties from the estate of the decedent.

144 | P a g e
Law 321_Corporation LAW_ Case Digest

Although the defendant corporations dealt and engaged in business with the public as
corporations, all their capital, assets and equity were however, personally owned by the late Pastor Y
Lim. Hence the alleged stockholders and officers appearing in the respective articles of incorporation of
the above business entities were mere dummies of Pastor Y. Lim, and they were listed therein only for
purposes of registration with the Securities and Exchange Commission.
Petitioner argues that the parcels of land covered under the Torrens system and registered in the
name of private respondent corporations should be included in the inventory of the estate of the
decedent Pastor Y. Lim, alleging that after all the determination by the probate court of whether these
properties should be included or not is merely provisional in nature, thus, not conclusive and subject to
a final determination in a separate action brought for the purpose of adjudging once and for all the issue
of title.

ISSUE:

Whether or not the properties of the corporation-defendants be included in the estate of the
deceased.

RULING:

NO.

They hold separate personalities from the deceased. In as much as the real properties included in
the inventory of the estate of the late Pastor Y. Lim are in the possession of and are registered in the
name of private respondent corporations, which under the law possess a personality separate and
distinct from their stockholders, and in the absence of any cogency to shred the veil of corporate fiction,
the presumption of conclusiveness of said titles in favor of private respondents should stand
undisturbed.
Notwithstanding that the real properties were duly registered under the Torrens system in the
name of private respondents, and as such were to be afforded the presumptive conclusiveness of title,
the probate court obviously opted to shut its eyes to this gleamy fact and still proceeded to issue the
impugned orders.
Mere ownership by a single stockholder or by another corporation of all or nearly all of the capital
stock of a corporation is not of itself a sufficient reason for disregarding the fiction of separate corporate
personalities.

ADALIA FRANCISCO and MERRYLAND DEVELOPMENT


vs. RITA MEJIA
GR 141617, 14 August 2001

FACTS:

On 21 December 1964, Gutierrez and Cardale Financing and Realty Corporation (Cardale)
executed a Deed of Sale with Mortgage for the consideration of P800,000.00 to be paid in several
installments within five years from the date of the deed, at an interest of nine percent per annum “based
on the successive unpaid principal balances.” Thereafter, the titles of Gutierrez were cancelled new ones
were issued in favor of Cardale.
On 26 August 1968, owing to Cardale’s failure to settle its mortgage obligation, Gutierrez filed a
complaint for rescission of the contract. During the pendency of the rescission case, Gutierrez died and
was substituted by her executrix, respondent Rita C. Mejia (Mejia). However, Cardale, which was
represented by petitioner Adalia B. Francisco (Francisco) in her capacity as Vice-President and
Treasurer of Cardale, lost interest in proceeding with the presentation of its evidence and the case
lapsed into inactive status for a period of about fourteen years.
In the meantime, the mortgaged parcels of land became delinquent in the payment of real estate
taxes, which culminated in their levy and auction sale in satisfaction of the tax arrears. The highest

145 | P a g e
Law 321_Corporation LAW_ Case Digest

bidder for the three parcels of land was petitioner Merryland Development Corporation, whose President
and majority stockholder is Francisco.
Mejia filed for damages against Francisco who controlled Cardale and Merryland and that she
had employed fraud by intentionally causing Cardale to default in its payment of real property taxes on
the mortgaged properties so that Merryland could purchase the same by means of a tax delinquency
sale.

ISSUE:

Whether or not Francisco is liable for damages.

RULING:

YES.

The totality of the circumstances appertaining conduce to the inevitable conclusion that
petitioner Francisco acted in bad faith. The events leading up to the loss by the Gutierrez estate of its
mortgage security attest to this. It has been established that Cardale failed to comply with its obligation
to pay the balance of the purchase price for the four parcels of land it bought from Gutierrez. This
prompted Gutierrez to file an action for rescission of the Deed of Sale with Mortgage, but the case
dragged on for about fourteen years when Cardale, as represented by Francisco, who was Vice-President
and Treasurer of the same, lost interest in completing its presentation of evidence
That Merryland acquired the property at the public auction only serves to shed more light upon
Francisco’s fraudulent purposes. Based on the findings of the Court of Appeals, Francisco is the
controlling stockholder and President of Merryland. Thus, aside from the instrumental role she played
as an officer of Cardale, in evading that corporation’s legitimate obligations to Gutierrez, it appears that
Francisco’s actions were also oriented towards securing advantages for another corporation in which she
had a substantial interest.
Under the doctrine of piercing the veil of corporate entity, when valid grounds therefore exist, the
legal fiction that a corporation is an entity with a juridical personality separate and distinct from its
members or stockholders may be disregarded. In such cases, the corporation will be considered as a
mere association of persons. The members or stockholders of the corporation will be considered as the
corporation, that is, liability will attach directly to the officers and stockholders.

DEVELOPMENT BANK OF THE PHILIPPINES


vs.
COURT OF APPEALS, REMINGTON INDUSTRIAL SALES
GR 126200, 16 August 2001

FACTS:

Between July 1981 and April 1984, Marinduque Mining entered into 3 mortgage agreements with
PNB and DBP involving its real properties located in Surigao del Norte, Negros Occidental, and Rizal, as
well as its equipments located therein. Marinduque failed to pay its loans, causing the foreclosure of the
said mortgages. PNB and DBP thereafter gained control of the said properties.
In the meantime, between July 16, 1982 to October 4, 1983, Marinduque Mining purchased and
caused to be delivered construction materials and other merchandise from Remington Industrial Sales
Corporation. The purchases remained unpaid as of August 1, 1984 when Remington filed a complaint
for a sum of money and damages against Marinduque Mining for the value of the unpaid construction
materials and other merchandise purchased by Marinduque Mining, as well as interest, attorney’s fees
and the costs of suit.
Remington’s original complaint was amended to include PNB, DBP, Maricalum Mining
Corporation and Island Cement Corporation as co-defendants. Remington asserted that Marinduque
Mining, PNB, DBP, Nonoc Mining, Maricalum Mining and Island Cement must be treated in law as one
and the same entity by disregarding the veil of corporate fiction since the personnel, key officers and
rank-and-file workers and employees of co-defendants NMIC, Maricalum and Island Cement creations of
co-defendants PNB and DBP were the personnel of co-defendant MMIC such that practically there has
only been a change of name for all legal purpose and intents.

ISSUE:

Whether or not the take over of PNB and DBP over Marinduque Mining is in bad faith.

146 | P a g e
Law 321_Corporation LAW_ Case Digest

RULING:

NO.

Their actions are mandated under the law. Where the corporations have directors and officers in
common, there may be circumstances under which their interest as officers in one company may
disqualify them in equity from representing both corporations in transactions between the two. Thus,
where one corporation was ‘insolvent and indebted to another, it has been held that the directors of the
creditor corporation were disqualified, by reason of self-interest, from acting as directors of the debtor
corporation in the authorization of a mortgage or deed of trust to the former to secure such
indebtedness In the same manner that when the corporation is insolvent, its directors who are its
creditors cannot secure to themselves any advantage or preference over other creditors. They cannot
thus take advantage of their fiduciary relation and deal directly with themselves, to the injury of others
in equal right.
Directors of insolvent corporation, who are creditors of the company, can not secure to
themselves any preference or advantage over other creditors in the payment of their claims. It is not
good morals or good law. The governing body of officers thereof are charged with the duty of conducting
its affairs strictly in the interest of its existing creditors, and it would be a breach of such trust for them
to undertake to give any one of its members any advantage over any other creditors in securing the
payment of his debts in preference to all others. When validity of these mortgages, to secure debts
upon which the directors were indorsers, was questioned by other creditors of the corporation, they
should have been classed as instruments rendered void by the legal principle which prevents directors of
an insolvent corporation from giving themselves a preference over outside creditors.

AMERICAN HOSPITAL SUPPLIES/PHILIPPINES et al.


vs.
COURT OF APPEALS, ALFONSO BAYANI
GR 111807, 14 June 1996

FACTS:

American Hospital Supplies was engaged in the sale and manufacture of medicines and
pharmaceuticals in the country and did substantial business with government hospitals. On 1 June
1970 it hired Alfonso Bayani as an Area Manager for Visayas and Mindanao, and later appointed him
Manager of its Cebu branch. On 30 January 1978 private respondent was dismissed from the service. At
that time he was receiving a monthly compensation of P3,180.00.
On 5 May 1978 private respondent filed a complaint for damages before the trial court alleging
that in the course of their business petitioners were directly encouraging, abetting and promoting
bribery in the guise of "commissions," "entertainment expenses" and "representation expenses" which
were given to various government hospital officials in exchange for favorable recommendations,
approvals and actual purchases of medicines and pharmaceuticals. For his refusal to take direct and
personal hand in giving "bribe money" he was dismissed. He then implicated AHS President Gervacio
Amistoso and Vice President Constancio Halili as responsible for his illegal dismissal.

ISSUE:

Whether or not Amistoso and Halili be held solidarily liable with the corporation.

RULING:

NO.

Corporate officers are not personally liable for money claims of discharged corporate employees
unless they acted with evident malice and bad faith in terminating their employment. In the case at bar,
while petitioners Amistoso and Halili may have had a hand in the relief of respondent. Bayani, there are
no indications of malice and bad faith on their part. We take exception to the conclusion of respondent
Court of Appeals that "the manner by which Halili and Amistoso acted is characterized by bad faith and
malice, thus binding them personally liable to plaintiff-appellee,'' On the contrary it is apparent that the
relief order was a business judgment on the part of the officers, with the best interest of the corporation
in mind, based on their opinion that respondent Bayani had failed to perform the duties expected of

147 | P a g e
Law 321_Corporation LAW_ Case Digest

him. Hence both the trial court and respondent Court of Appeals committed a reversible error in holding
petitioners Amistoso and Halili jointly and solidarily liable with Petitioner Corporation.

COMPLEX ELECTRONICS EMPLOYEES ASSOCIATION


vs.
NLRC, et al.
GR 121315, 19 July 1999

FACTS:

Complex informed its Lite-On personnel that a request from Lite On Philippines to lower their
selling price by 10% was not feasible as they were already incurring losses at the present prices of their
products. Under such circumstances, Complex regretfully informed the employees that it was left with
no alternative but to close down the operations of the Lite-On Line. The Union, however, decried the
decision and voted to declare a strike. Labor unrest within the company eventually ensued.
In the evening of April 6, 1992, the machinery, equipment and materials being used for
production at Complex were pulled-out from the company premises and transferred to the premises of
Ionics Circuit, Inc. at Cabuyao, Laguna. The following day, a total closure of company operation was
effected at Complex.
A complaint was, thereafter, filed with the Labor Arbitration Branch of the NLRC for unfair labor
practice, illegal closure/illegal lockout, money claims for vacation leave, sick leave, unpaid wages, 13th
month pay, damages and attorney's fees. Ionics was impleaded as a party defendant because the officers
and management personnel of Complex were also holding office at Ionics with Lawrence Qua as the
President of both companies.
Ionics contended that it was an entity separate and distinct from Complex and had been in
existence since July 5, 1984 or eight (8) years before the labor dispute arose at Complex. Like Complex,
it was also engaged in the semi-conductor business where the machinery, equipment and materials
were consigned to them by their customers. While admitting that Lawrence Qua, the President of
Complex was also the President of Ionics, the latter denied having Qua as their owner since he had no
recorded subscription of P1,200,000.00 in Ionics as claimed by the Union.

ISSUE:

Whether or not Lawrence Qua should be held liable for the alleged illegal transfer of machineries
of Complex to Ionics.

RULING:

NO.

It is settled that in the absence of malice or bad faith, a stockholder or an officer of a corporation
cannot be made personally liable for corporate liabilities. The fact that the pull-out of the machinery,
equipment and materials was effected during nighttime is not per se an indicia of bad faith on the part
of respondent Qua since he had no other recourse, and the same was dictated by the prevailing mood of
unrest as the laborers were already vandalizing the equipment, bent on picketing the company premises
and threats to lock out the company officers were being made. Such acts of respondent Qua were, in
fact, made pursuant to the demands of Complex's customers who were already alarmed by the pending
labor dispute and imminent strike to be stage by the laborers, to have their equipment, machinery and

148 | P a g e
Law 321_Corporation LAW_ Case Digest

materials pull out of Complex. As such, these acts were merely done pursuant to his official functions
and were not, in any way, made with evident bad faith.
As to the juridical personality of the corporations, Ionics may be engaged in the same business as
that of Complex, but this fact alone is not enough reason to pierce the veil of corporate fiction of the
corporation. Well-settled is the rule that a corporation has a personality separate and distinct from that
of its officers and stockholders. Likewise, mere ownership by a single stockholder or by another
corporation of all or nearly all of the capital stock of a corporation is not of itself sufficient ground for
disregarding the separate corporate personality.

ERNESTINA CRISOLOGO-JOSE
vs.
COURT OF APPEALS, RICARDO SANTOS, JR.
GR 80599, 15 September, 1989

FACTS:

In 1980, Ricardo S. Santos, Jr. was the vice-president of Mover Enterprises, Inc. in-charge of
marketing and sales; and the president of the said corporation was Atty. Oscar Z. Benares. On April 30,
1980, Atty. Benares, in accommodation of his clients, the spouses Jaime and Clarita Ong, a check
drawn against Traders Royal Bank, dated June 14, 1980, in the amount of P45,000.00 payable to
Ernestina Crisologo-Jose. Since the check was under the account of Mover Enterprises, Inc., the same
was to be signed by its president, Atty. Oscar Z. Benares, and the treasurer of the said corporation.
However, since at that time, the treasurer of Mover Enterprises was not available, Atty. Benares
prevailed upon Santos, Jr., to sign the aforesaid check as an alternate signatory, who did sign the same.
It appears that the check to Crisologo-Jose in consideration of the waiver or quitclaim by said
defendant over a certain property which the Government Service Insurance System (GSIS) agreed to sell
to the clients of Atty. Oscar Benares, the spouses Jaime and Clarita Ong, with the understanding that
upon approval by the GSIS of the compromise agreement with the spouses Ong, the check will be
encashed accordingly. However, since the compromise agreement was not approved within the expected
period of time, the aforesaid check was replaced by Atty. Benares with another Traders Royal Bank
check dated August 10, 1980, in the same amount. This replacement check was also signed by Atty.
Benares and by Santos, Jr. When Jose deposited this replacement check with her account, it was
dishonored for insufficiency of funds. A subsequent redepositing of the said check was likewise
dishonored by the bank for the same reason.

ISSUE:

Whether or not Movers Enterprises should be held liable to the bounced checks which are
personal liabilities of Atty. Bañares.

RULING:

NO.

The provision of the Negotiable Instruments Law which holds an accommodation party liable on
the instrument to a holder for value, although such holder at the time of taking the instrument knew
him to be only an accommodation party, does not include nor apply to corporations which are
accommodation parties. This is because the issue or indorsement of negotiable paper by a corporation
without consideration and for the accommodation of another is ultra vires. Hence, one who has taken
the instrument with knowledge of the accommodation nature thereof cannot recover against a
corporation where it is only an accommodation party.
By way of exception, an officer or agent of a corporation shall have the power to execute or
indorse a negotiable paper in the name of the corporation for the accommodation of a third person only
if specifically authorized to do so.. Since such accommodation paper cannot thus be enforced against
the corporation, especially since it is not involved in any aspect of the corporate business or operations,
the inescapable conclusion in law and in logic is that the signatories thereof shall be personally liable
therefor, as well as the consequences arising from their acts in connection therewith.
Instead, Jose should direct her claim against Bañares and Santos. Santos, however, is exculpated
from criminal liability under BP 22 for he successfully and legally consigned the amount of the check
with the Court within the reglamentary period.

149 | P a g e
Law 321_Corporation LAW_ Case Digest

FCY CONSTRUCTION GROUP and FRANCIS YU


vs.
COURT OF APPEALS, HON. JOSE DE LA RAMA
GR 123358, 01 February 2000

FACTS:

On June 29, 1993, Ley Construction and Development Corporation filed a complaint for collection
of a sum of money with application for preliminary attachment against petitioner FCY Construction
Group, Inc. and Francis C. Yu. Ley alleged that it had a joint venture agreement with petitioner FCY
Construction Group, Inc. (wherein petitioner Francis C. Yu served as President) over the Tandang Sora
Commonwealth Flyover government project for which it had provided funds and construction materials.
The Complaint was filed in order to compel petitioners to pay its half share in the collections received in
the project as well as those yet to be received therein. In support of its application for a writ of
attachment, private respondent alleged that petitioners were guilty of fraud in incurring the obligation
and had fraudulently misapplied or converted the money paid them, to which it had an equal share.
FCY denied the allegation, and also moved for the dropping of Francis Yu as one of the
defendants, claiming that the hornbook law that corporate personality is a shield against personal
liability of its officers.

ISSUE:

Whether or not Francis Yu may be held solidarily liable with FCY.

RULING:

NO.

FCY has a separate juridical entity from that of Francis. Francis Yu cannot be made liable in his
individual capacity if he indeed entered into and signed the contract in his official capacity as President,
in the absence of stipulation to that effect, due to the personality of the corporation being separate and
distinct from the persons composing it. However, while Yu cannot be held solidarily liable with petitioner
corporation merely because he is the President thereof and was involved in the transactions with private
corporation, there exists instances when corporate officers may be held personally liable for corporate
acts.
Personal liability of a corporate director, trustee or officer along (although not necessarily) with
the corporation may so validly attach, as a rule. The attendance of these circumstances, however,
cannot be determined at this stage and should properly be threshed out during the trial on the merits.
Also, there was no fraud on the part of FCY in the performance of its obligations with Ley, therefore
rendering attachment as improper.

RICARDO LLAMADO
vs.
COURT OF APPEALS, PEOPLE OF THE PHILIPPINES
GR 99032, 26 March 1997

FACTS:

150 | P a g e
Law 321_Corporation LAW_ Case Digest

Private complainant, Leon Gaw, delivered to accused the amount of P180,000.00, with the
assurance of Aida Tan, the secretary of the accused in the corporation, that it will be repaid on 4
November 1983. Upon delivery of the money, accused Ricardo Llamado took it and placed it inside a
deposit box. Accused Jacinto Pascual and Ricardo Llamado signed Philippine Trust Company Check No.
047809, postdated 4 November 1983, in the amount of P186,500.00 in the presence of private
complainant.
The aforesaid check was issued in payment of the cash money delivered to the accused by private
complainant, plus interests thereon for sixty (60) days in the amount of P6,500.00.
On 4 November 1983, private complainant deposited the check in his current account with the
Equitable Banking Corporation which later informed the complainant that said check was dishonored
by the drawee bank because payment was stopped, and that the check was drawn against insufficient
funds. Private complainant was also notified by the Equitable Banking Corporation that his current
account was debited for the amount of P186,500.00 because of the dishonor of the said check.
Private complainant returned to Aida Tan to inform her of the dishonor of the check. Aida Tan
received the check from private complainant with the assurance that she will have said check changed
with cash. However, upon his return to Aida Tan, the latter informed him that she had nothing to do
with the check.
Llamado alleges that he should not be held personally liable for the amount of the check because
it was a check of the Pan Asia Finance Corporation and he signed the same in his capacity as Treasurer
of the corporation.

ISSUE:

Whether or not Llamado should be held liable under BP 22.

RULING:

YES.

He is mere act of signing the check held him liable under BP 22. Where the check is drawn by a
corporation, company or entity, the person or persons who actually signed the check in behalf of such
drawer shall be liable under this Act.

MAM REALTY CORPORATION


vs.
NLRC, CELSO BALBASTRO
GR 114787, 02 June 1995

FACTS:

Celso B. Balbastro filed a case against MAM Realty Development Corporation ("MAM") and its Vice
President Manuel P. Centeno, for unfair labor practice in violation of the Labor Code. Balbastro alleged
that he was employed by MAM as a pump operator in 1982 and had since performed such work at its
Rancho Estate, Marikina, Metro manila.

151 | P a g e
Law 321_Corporation LAW_ Case Digest

MAM countered that Balbastro had previously been employed by Francisco Cacho and co., Inc.,
the developer of Rancho Estates. Sometime in May 1982, his services were contracted by MAM for the
operation of the Rancho Estates' water pump. He was engaged, however, not as an employee, but as a
service contractor, at an agreed fee of P1,590.00 a month. Similar arrangements were likewise entered
into by MAM with one Rodolfo Mercado and with a security guard of Rancho Estates III Homeowners'
Association. Under the agreement, Balbastro was merely made to open and close on a daily basis the
water supply system of the different phases of the subdivision in accordance with its water rationing
scheme. He worked for only a maximum period of three hours a day, and he made use of his free time
by offering plumbing services to the residents of the subdivision. He was not at all subject to the control
or supervision of MAM for, in fact, his work could so also be done either by Mercado or by the security
guard. On 23 May 1990, prior to the filing of the complaint, MAM executed a Deed of Transfer, 1
effective 01 July 1990, in favor of the Rancho Estates Phase III Homeowners Association, Inc., conveying
to the latter all its rights and interests over the water system in the subdivision.
NLRC found the corporation guilty as charged, and likewise held Centeno liable together with said
corporation.

ISSUE:

Whether or not Centeno should be held liable together with MAM Realty.

RULING:

NO.

A corporation, being a juridical entity, may act only through its directors, officers and employees.
Obligations incurred by them, acting as such corporate agents, are not theirs but the direct
accountabilities of the corporation they represent. True, solidarily liabilities may at times be incurred
but only when exceptional circumstances. In labor cases, for instance, the Court has held corporate
directors and officers solidarily liable with the corporation for the termination of employment of
employees done with malice or in bad faith.
In the case at bench, there is nothing substantial on record that can justify, prescinding from the
foregoing, petitioner Centeno's solidary liability with the corporation. Nothing states that he acted in
bad faith.
Although the Court found that there is an employer-employee relationship between Balbastro and
MAM Realty, the case was remanded to NLRC for the recomputation of Balbastro’s monetary awards,
such as backwages and wage differentials.

SERGIO NAGUIAT
vs.
CLARK FIELD TAXI, INC.
GR 116123, 13 March 1997

FACTS:

Petitioner CFTI held a concessionaire's contract with the Army Air Force Exchange Services
("AAFES") for the operation of taxi services within Clark Air Base. Sergio F. Naguiat was CFTI's
president, while Antolin T. Naguiat was its vice-president. Like Sergio F. Naguiat Enterprises,
Incorporated ("Naguiat Enterprises"), a trading firm, it was a family-owned corporation.
Individual respondents were previously employed by CFTI as taxicab drivers. Due to the phase-
out of the US military bases in the Philippines, from which Clark Air Base was not spared, the AAFES
was dissolved, and the services of individual respondents were officially terminated on November 26,
1991.
The AAFES Taxi Drivers Association ("drivers' union"), through its local president, Eduardo
Castillo, and CFTI held negotiations as regards separation benefits that should be awarded in favor of
the drivers. They arrived at an agreement that the separated drivers will be given P500.00 for every year

152 | P a g e
Law 321_Corporation LAW_ Case Digest

of service as severance pay. Most of the drivers accepted said amount in December 1991 and January
1992. However, individual respondents herein refused to accept theirs.
Instead, after disaffiliating themselves from the drivers' union and filed a complaint against
"Sergio F. Naguiat doing business under the name and style Sergio F. Naguiat Enterprises, Inc., and
CFTI with Antolin T. Naguiat as vice president and general manager, as party respondent.

ISSUE:

Whether or not Sergio Naguiat may be held liable for the claims instituted by the taxi drivers
against his company.

RULING:

YES.

As provided for under the fifth paragraph of Section 100 of the Corporation Code specifically
imposes personal liability upon the stockholder actively managing or operating the business and affairs
of the close corporation.
In fact, in posting the surety bond required by this Court for the issuance of a temporary
restraining order enjoining the execution of the assailed NLRC Resolutions, only Sergio F. Naguiat, in
his individual and personal capacity, principally bound himself to comply with the obligation
thereunder, i.e., "to guarantee the payment to private respondents of any damages which they may
incur by reason of the issuance of a temporary restraining order sought, if it should be finally adjudged
that said principals were not entitled thereto.
The Court here finds no application to the rule that a corporate officer cannot be held solidarily
liable with a corporation in the absence of evidence that he had acted in bad faith or with malice. In the
present case, Sergio Naguiat is held solidarily liable for corporate tort because he had actively engaged
in the management and operation of CFTI, a close corporation.
Antolin Naguiat, however, could not be held liable. Although he carried the title of "general
manager" as well, it had not been shown that he had acted in such capacity. Furthermore, no evidence
on the extent of his participation in the management or operation of the business was preferred. In this
light, he cannot be held solidarily liable for the obligations of CFTI and Sergio Naguiat to the private
respondents.

PROGRESS HOMES and ERMELO ALMEDA


vs.
NLRC, et al.
GR 106212, 07 March 1997

FACTS:

Private respondents allegedly were among the workers employed by Progress Homes in their
construction and development of the subdivision from 1986 to 1988. They were paid varying salaries.
Forty of these workers, including private respondents, filed before the NLRC Arbitration Branch a
petition for reinstatement, salary adjustment, ECOLA, overtime pay and 13th month pay. Petitioners
amicably settled the case with thirty-three of the laborers, leaving private respondents as the only
claimants. Private respondents alleged that they worked as laborers and carpenters for 8.5 hours a day
at a salary below the minimum wage and that when they demanded payment of the benefits due them,
they were summarily dismissed and barred from entering the workplace. It also denied that private
respondents were regular employees claiming that they were only project employees and that there was
no employer-employee relationship between them.
The Labor Arbiter and the NLRC ruled that Progress is liable to the respondents, with Almeda
jointly and severally liable.

ISSUE:

Whether or not Almeda may be held liable.

RULING:

153 | P a g e
Law 321_Corporation LAW_ Case Digest

NO.

It amounted to grave abuse of discretion. The Court has held that corporate directors and officers
are solidarily liable with the corporation for the termination of employment of employees only if the
termination is done with malice or in bad faith.
The Labor Arbiter's decision failed to disclose why Almeda was made personally liable. There
appears no evidence on record that he acted maliciously or in bad faith in terminating the services of
private respondents. Almeda, therefore, should not have been made personally answerable for the
payment of private respondents' salaries.
The decision of the Labor Arbiter and the NLRC, however, should be set aside because of denial of
due process on the part of Progress Homes.

REAHS CORP., SEVERO CASTULO, et al.


vs.
NLRC, BONIFACIO RED, et al.
GR 117473, 15 April 1997

FACTS:

Private respondents sued Reahs Corp. for unfair labor practice and illegal dismissal. They claim
that they were unlawfully dismissed and were not awarded nor given any separation pay.
On the other hand, respondents allege that sometime in 1986, a certain Ms Soledad Domingo, the
sole proprietress and operator of Rainbow Sauna located at 316 Araneta Avenue, Quezon City, offered to
sell her business to respondent Reah's Corporation After the sale, all the assets of Ms Domingo were
turned over to respondent Reah's, which put a sing-along coffee shop and massage clinic; that
complainant Red started his employment on the first week of December 1988 as a room boy at
P50.00/day and was given living quarters inside the premises as he requested; that sometime in March
1989, complainant Red asked permission to go to Bicol for a period of ten (10) days, which was granted,
and was given an advance money of P1,200.00 to bring some girls from the province to work as
attendants at the respondent's massage clinic, that it was only on January 1, 1990 that complainant
Red returned and was re-hired under the same terms and conditions of his previous employment with
the understanding that he will have to refund the P1,200.00 cash advance given to him; that due to poor
business, increase in the rental cost and the failure of Meralco to reconnect the electrical services in the
establishment, it suffered losses leading to its closure.
The NLRC ruled in favor of respondents. Together with the corporation, the NLRC also held
Castulo, Romeo Pascua, and Daniel Valenzuela solidarily liable due to their capacity as Chairman,
Board Member and Accountant, and Acting Manager, respectively.

ISSUE:

Whether or not Pascua, Castulo, and Valenzuela, may be held liable.

RULING:

154 | P a g e
Law 321_Corporation LAW_ Case Digest

YES.

They acted in bad faith in dismissing the respondents. As a general rule established by legal
fiction, the corporation has a personality separate and distinct from its officers, stockholders and
members. Hence, officers of a corporation are not personally liable for their official acts unless it is
shown that they have exceeded their authority. This fictional veil, however, can be pierced by the very
same law which created it when "the notion of the legal entity is used as a means to perpetrate fraud, an
illegal act, as a vehicle for the evasion of an existing obligation, and to confuse legitimate issues". Under
the Labor Code, for instance, when a corporation violates a provision declared to be penal in nature, the
penalty shall be imposed upon the guilty officer or officers of the corporation.
In the case at bar, the thrust of petitioners' arguments was aimed at confining liability solely to
the corporation, as if the entity were an automaton designed to perform functions at the push of a
button. The issue, however, is not limited to payment of separation pay under Article 283 but also
payment of labor standard benefits such as underpayment of wages, holiday pay and 13th month pay to
two of the private respondents. While there is no sufficient evidence to conclude that petitioners have
indiscriminately stopped the entity's business, at the same time, petitioners have opted to abstain from
presenting sufficient evidence to establish the serious and adverse financial condition of the company.

BENJAMIN SANTOS
vs.
NLRC, MELVIN MILLENA
GR 101699, 13 March 1996

FACTS:

Melvin Millena, on 01 October 1985, was hired to be the project accountant for Mana Mining and
Development Corp.’s mining operations in Gatbo, Bacon, Sorsogon. On 12 August 1986, private
respondent sent to Mr. Gil Abaño, the MMDC corporate treasurer, a memorandum calling the latter's
attention to the failure of the company to comply with the withholding tax requirements of, and to make
the corresponding monthly remittances to, the Bureau of Internal Revenue ("BIR") on account of delayed
payments of accrued salaries to the company's laborers and employees.
Albaño responded that the mining operations in Sorsogon shall be stopped pending the end of the
wet season and the normalization of the peace and order situation in the province. Therefore, MMDC is
dispensing the services of Millena because of lack of work load.
Private respondent expressed "shock" over the termination of his employment. He complained
that he would not have resigned from the Sycip, Gorres & Velayo accounting firm, where he was already
a senior staff auditor, had it not been for the assurance of a "continuous job" by MMDC's Engr.
Rodillano E. Velasquez. Private respondent requested that he be reimbursed the "advances" he had
made for the company and be paid his "accrued salaries/claims
With his demands left unheeded, Millena filed a complaint for illegal dismissal, unpaid salaries,
13th month pay, overtime pay, separation pay and incentive leave pay against MMDC and its two top
officials, namely, herein petitioner Benjamin A. Santos (the President) and Rodillano A. Velasquez (the
executive vice-president).

ISSUE:

Whether or not the impleaded officials of MMDC may be held liable.

RULING:

NO.

It was not proven that they acted in bad faith. A corporation is a juridical entity with legal
personality separate and distinct from those acting for and in its behalf and, in general, from the people
comprising it. Nevertheless, being a mere fiction of law, peculiar situations or valid grounds can exist to
warrant, albeit done sparingly, the disregard of its independent being and the lifting of the corporate
veil. The Court also has collated the settled instances when, without necessarily piercing the veil of
corporate fiction, personal civil liability can also be said to lawfully attach to a corporate director, trustee
or officer.

155 | P a g e
Law 321_Corporation LAW_ Case Digest

The case of petitioner is way off these exceptional instances. It is not even shown that petitioner
has had a direct hand in the dismissal of private respondent enough to attribute to him (petitioner) a
patently unlawful act while acting for the corporation. It is undisputed that the termination of
petitioner's employment has, instead, been due, collectively, to the need for a further mitigation of
losses, the onset of the rainy season, the insurgency problem in Sorsogon and the lack of funds to
further support the mining operation in Gatbo.

JOSE SIA
vs.
PEOPLE OF THE PHILIPPINES
GR L-30896, 28 April 1983

FACTS:

Jose O. Sia sometime prior to 24 May, 1963, was General Manager of the Metal Manufacturing
Company of the Philippines, Inc. engaged in the manufacture of steel office equipment; on 31 May,
1963, because his company was in need of raw materials to be imported from abroad, he applied for a
letter of credit to import steel sheets from Mitsui Bussan Kaisha, Ltd. of Tokyo, Japan, the application
being directed to the Continental Bank, herein complainant, and his application having been approved,
the letter of credit was opened on 5 June, 1963 in the amount of $18,300. The goods arrived sometime
in July, 1963 according to accused himself, now from here on there is some debate on the evidence;
according to Complainant Bank, there was permitted delivery of the steel sheets only upon execution of
a trust receipt, while according to the accused, the goods were delivered to him sometime before he
executed that trust receipt in fact they had already been converted into steel office equipment by the
time he signed said trust receipt. But there is no question - and this is not debated — that the bill of
exchange issued for the purpose of collecting the unpaid account thereon having fallen due neither
accused nor his company having made payment thereon notwithstanding demands, and the accounts
having reached the sum in pesos of P46,818.68 after deducting his deposit valued at P28,736.47.

ISSUE:

Whether or not Sia should be held liable.

RULING:

NO.

The bank is transacting with Metal Manufacturing and not with him. The case cited by the Court
of Appeals in support of its stand - Tan Boon Kong case, supra - may however not be squarely
applicable to the instant case in that the corporation was directly required by law to do an act in a given
manner, and the same law makes the person who fails to perform the act in the prescribed manner
expressly liable criminally. The performance of the act is an obligation directly imposed by the law on
the corporation. Since it is a responsible officer or officers of the corporation who actually perform the
act for the corporation, they must of necessity be the ones to assume the criminal liability; otherwise
this liability as created by the law would be illusory, and the deterrent effect of the law, negated.
In the present case, a distinction is to be found with the Tan Boon Kong case in that the act
alleged to be a crime is not in the performance of an act directly ordained by law to be performed by the
corporation. The act is imposed by agreement of parties, as a practice observed in the usual pursuit of a
business or a commercial transaction. The offense may arise, if at all, from the peculiar terms and
condition agreed upon by the parties to the transaction, not by direct provision of the law. The intention
of the parties, therefore, is a factor determinant of whether a crime was committed or whether a civil
obligation alone intended by the parties. With this explanation, the distinction adverted to between the
Tan Boon Kong case and the case at bar should come out clear and meaningful. In the absence of an
express provision of law making the petitioner liable for the criminal offense committed by the
corporation of which he is a president as in fact there is no such provisions in the Revised Penal Code

156 | P a g e
Law 321_Corporation LAW_ Case Digest

under which petitioner is being prosecuted, the existence of a criminal liability on his part may not be
said to be beyond any doubt. In all criminal prosecutions, the existence of criminal liability for which the
accused is made answerable must be clear and certain. The maxim that all doubts must be resolved in
favor of the accused is always of compelling force in the prosecution of offenses.

TRAMAT MERCANTILE, INC. and DAVID HONG


vs.
COURT OF APPEALS, MELCHOR DE LA CUESTA
GR 111008, 07 November 1994

FACTS:

On 09 April 1984, Melchor de la Cuesta, doing business under the name and style of "Farmers
Machineries," sold to Tramat Mercantile, Inc. one unit Hinomoto Tractor Model MB 1100D powered by a
13 H.P. diesel engine. In payment, David Ong, Tramat's president and manager, issued a check for
P33,500.00 (apparently replacing an earlier postdated check for P33,080.00). Tramat, in turn, sold the
tractor, together with an attached lawn mower fabricated by it, to the Metropolitan Waterworks and
Sewerage System/NAWASA for P67,000.00. David Ong caused a stop payment of the check when
NAWASA refused to pay the tractor and lawn mower after discovering that, aside from some stated
defects of the attached lawn mower, the engine (sold by de la Cuesta) was a reconditioned unit.
On 28 May 1985, de la Cuesta filed an action for the recovery of P33,500.00, as well as attorney's
fees of P10,000.00, and the costs of suit. Ong, in his answer, averred, among other things, that de la
Cuesta had no cause of action; that the questioned transaction was between plaintiff and Tramat
Mercantile, Inc., and not with Ong in his personal capacity; and that the payment of the check was
stopped because the subject tractor had been priced as a brand new, not as a reconditioned unit.

ISSUE:

Whether or not Ong should be held liable for the unpaid tractor.

RULING:

NO.

It is an error to hold David Ong jointly and severally liable with TRAMAT to de la Cuesta under
the questioned transaction. Ong had acted, not in his personal capacity, but as an officer of a
corporation, TRAMAT, with a distinct and separate personality. As such, it should only be the
corporation, not the person acting for and on its behalf, that properly could be made liable thereon.
Tramat, however, should be held liable for the unpaid tractor because at the time of the
purchase, the appellants did not reveal to the appellee the true purpose for which the tractor would be
used. Granting that the appellants informed the appellee that they would be reselling the unit to the
MWSS, an entity admittedly not engaged in farming, and that they ordered the tractor without the power
tiller, an indispensable accessory if the tractor would be used in farming, these in themselves would not
constitute the required implied notice to the appellee as seller.

 Self-Dealing Director/Officer

REPUBLIC OF THE PHILIPPINES

157 | P a g e
Law 321_Corporation LAW_ Case Digest

vs.
EDUARDO CONJUANGCO
GR 166859, 12 April 2011

FACTS:

Danding Cojuangco is being accused of using public funds to finance his acquisition of shares in
the San Miguel Corporation. Through the coconut levy fund, was is being accused of buying out
shareholders in the corporation in order to become a substantial shareholder himself. To carry out his
scheme, he used dummy shareholders who shall be trustors of the shares on his behalf.
Contention rises on his culpability as a public official during the time that he bought the shares.
It is claimed by the Sandiganbayan that he was able to amass vast shares of the corporation through
the use of the coconut levy fund, which is public in nature. Therefore, it but apparent that he be held
liable for his actions in taking control of the corporation.

ISSUE:

Whether or not Conjuangco illegally used ill-gotten wealth to buy shares of SMC.

RULING:

NO.

The funds are in fact loaned from UCPB, which was organized as a depositary of the coconut levy
funds of the corporation. Also, the Government failed to adduce substantial evidence linking Cojuangco
to the use of Marcos ill-gotten wealth.
All these judicial pronouncements demand two concurring elements to be present before assets or
properties were considered as ill-gotten wealth, namely: (a) they must have “originated from the
government itself,” and (b) they must have been taken by former President Marcos, his immediate
family, relatives, and close associates by illegal means.
But settling the sources and the kinds of assets and property covered by E.O. No. 1 and related
issuances did not complete the definition of ill-gotten wealth. The further requirement was that the
assets and property should have been amassed by former President Marcos, his immediate family,
relatives, and close associates both here and abroad. In this regard, identifying former President Marcos,
his immediate family, and relatives was not difficult, but identifying other persons who might be the
close associates of former President Marcos presented an inherent difficulty, because it was not fair and
just to include within the term close associates everyone who had had any association with President
Marcos, his immediate family, and relatives.
It does not suffice, as in this case, that the respondent is or was a government official or employee
during the administration of former Pres. Marcos. There must be a prima facie showing that the
respondent unlawfully accumulated wealth by virtue of his close association or relation with former
Pres. Marcos and/or his wife. This is so because otherwise the respondent’s case will fall under existing
general laws and procedures on the matter.

CHARLES W. MEAD
vs.
E.C. MCCULLOUGH, et. al.
GR 6217, 26 December 2011

FACTS:

On March 15, 1902, the plaintiff (Mead will be referred to as the plaintiff in this opinion unless it
is otherwise stated) and the defendant organized the "Philippine Engineering and Construction
Company.

158 | P a g e
Law 321_Corporation LAW_ Case Digest

Shortly after the organization, the directors held a meeting and elected the plaintiff as general
manager. The plaintiff held this position with the company for nine months, when he resigned to accept
the position of engineer of the Canton and Shanghai Railway Company.
The contract and work undertaken by the company during the management of Mead were the
wrecking contract with the Navy Department at Cavite for the raising of the Spanish ships sunk by
Admiral Dewey; the contract for the construction of certain warehouses for the quartermaster
department; the construction of a wharf at Fort McKinley for the Government; The supervision of the
construction of the Pacific Oriental Trading Company's warehouse; and some other odd jobs not
specifically set out in the record.
Shortly after the plaintiff left the Philippine Islands for China, the other directors, the defendants
in this case, held a meeting on December 24, 1903, for the purpose of discussing the condition of the
company at that time and determining what course to pursue.
The assignees of the wrecking contract, including McCullough, formed was not known as the
"Manila Salvage Association." This association paid to McCullough $15,000 Mexican Currency cash for
the assignment of said contract. In addition to this payment, McCullough retained a one-sixth interest
in the new company or association.

ISSUE:

Whether or not the respondents are self-dealing directors.

RULING:

NO.

While a corporation remains solvent, there is no reason why a director or officer, by the authority
of a majority of the stockholders or board of managers, may not deal with the corporation, loan it money
or buy property from it, in like manner as a stranger. So long as a purely private corporation remains
solvent, its directors are agents or trustees for the stockholders. They owe no duties or obligations to
others. But the moment such a corporation becomes insolvent, its directors are trustees of all the
creditors, whether they are members of the corporation or not, and must manage its property and assets
with strict regard to their interest; and if they are themselves creditors while the insolvent corporation is
under their management, they will not be permitted to secure to themselves by purchasing the corporate
property or otherwise any personal advantage over the other creditors. Nevertheless, a director or officer
may in good faith and for an adequate consideration purchase from a majority of the directors or
stockholders the property even of an insolvent corporation, and a sale thus made to him is valid and
binding upon the minority.

PRIME WHITE CEMENT


vs.
INTERMEDIATE APPELLATE COURT, ALEJANDRO TE
GR 68555, 19 March 1993

FACTS:

On the 16th day of July, 1969, plaintiff and defendant corporation thru its President, Mr. Zosimo
Falcon and Justo C. Trazo, as Chairman of the Board, entered into a dealership agreement (Exhibit A)
whereby said plaintiff was obligated to act as the exclusive dealer and/or distributor of the said
defendant corporation of its cement products in the entire Mindanao area.
Prime, however, amended the agreement made with Te, forcing the latter to demand the
performance of the conditions stated in the original contract. Aside from that, Prime entered into a
dealership contract with Napoleon Co, therefore violating the exclusive rights of Te in Mindanao. Te
thereafter filed for specific performance against Prime.
Prime questioned the validity of the contract, claiming it is null and void due to the fact that Te is
a Director and the Auditor of the cement company.

159 | P a g e
Law 321_Corporation LAW_ Case Digest

ISSUE:

Whether or not the dealership contract between Prime and Te is valid.

RULING:

NO.

The requisites for the approval of a contract with a ‘self dealing director’ was not satisfied. A
director of a corporation holds a position of trust and as such, he owes a duty of loyalty to his
corporation. In case his interests conflict with those of the corporation, he cannot sacrifice the latter to
his own advantage and benefit. As corporate managers, directors are committed to seek the maximum
amount of profits for the corporation. This trust relationship "is not a matter of statutory or technical
law. It springs from the fact that directors have the control and guidance of corporate affairs and
property and hence of the property interests of the stockholders."
A director's contract with his corporation is not in all instances void or voidable. If the contract is
fair and reasonable under the circumstances, it may be ratified by the stockholders provided a full
disclosure of his adverse interest is made.
Granting arguendo that the "dealership agreement" involved here would be valid and enforceable
if entered into with a person other than a director or officer of the corporation, the fact that the other
party to the contract was a Director and Auditor of the petitioner corporation changes the whole
situation. First of all, the contract was neither fair nor reasonable. The "dealership agreement" entered
into in July, 1969, was to sell and supply to respondent Te 20,000 bags of white cement per month, for
five years starting September, 1970, at the fixed price of P9.70 per bag. Respondent Te is a businessman
himself and must have known, or at least must be presumed to know, that at that time, prices of
commodities in general, and white cement in particular, were not stable and were expected to rise. At
the time of the contract, petitioner corporation had not even commenced the manufacture of white
cement, the reason why delivery was not to begin until 14 months later. He must have known that
within that period of six years, there would be a considerable rise in the price of white cement. In fact,
respondent Te's own Memorandum shows that in September, 1970, the price per bag was P14.50, and
by the middle of 1975, it was already P37.50 per bag. Despite this, no provision was made in the
"dealership agreement" to allow for an increase in price mutually acceptable to the parties. Instead, the
price was pegged at P9.70 per bag for the whole five years of the contract. Fairness on his part as a
director of the corporation from whom he was to buy the cement, would require such a provision.

 Contracts Between Corporations with Interlocking Directors

PEDRO PALTING
vs.
SAN JOSE PETROLEUM, INC.
GR L-14441, 17 December 1966

FACTS:

San Jose Petroleum filed with the Philippine Securities and Exchange Commission a sworn
registration statement, for the registration and licensing for sale in the Philippines Voting Trust
Certificates representing 2,000,000 shares of its capital stock with a par value of $0.35 a share, at P1.00
per share
Pedro R. Palting and others, allegedly prospective investors in the shares of San Jose Petroleum,
filed with the Securities and Exchange Commission an opposition to the registration and licensing of the
securities on the grounds that (1) the tie-up between the issuer, San Jose Petroleum, a Panamanian
corporation, and San Jose Oil, a domestic corporation, violates the Constitution of the Philippines, the
Corporation Law and the Petroleum Act of 1949; (2) the issuer has not been licensed to transact
business in the Philippines; (3) the sale of the share of the issuer is fraudulent, and works or tends to
work a fraud upon Philippine purchasers; and (4) the issuer as an enterprise, as well as its business, is
based upon unsound business principles.

ISSUE:

Whether or not San Jose Petroleum can validly engage in business in the Philippines.

RULING:

160 | P a g e
Law 321_Corporation LAW_ Case Digest

NO.

It does not have the required percentage of Filipino capital to validly exercise its business in the
Philippines. In the two lists of stockholders, there is no indication of the citizenship of these
stockholders, or of the total number of authorized stocks of each corporation for the purpose of
determining the corresponding percentage of these listed stockholders in relation to the respective
capital stock of said corporation.
These provisions are in direct opposition to the corporation law and corporate practices in this
country. These provisions alone would outlaw any corporation locally organized or doing business in
this jurisdiction. Consider the unique and unusual provision that no contract or transaction between
the company and any other association or corporation shall be affected except in case of fraud, by the
fact that any of the directors or officers of the company may be interested in or are directors or officers
of such other association or corporation; and that none of such contracts or transactions of this
company with any person or persons, firms, associations or corporations shall be affected by the fact
that any director or officer of this company is a party to or has an interest in such contract or
transaction or has any connection with such person or persons, firms, associations or corporations: and
that any and all persons who may become directors or officers of this company are hereby relieved of all
responsibility which they would otherwise incur by reason of any contract entered into which this
company either for their own benefit, or for the benefit of any person, firm, association or corporation in
which they may be interested.

DEVELOPMENT BANK OF THE PHILIPPINES


vs.
COURT OF APPEALS, REMINGTON INDUSTRIAL SALES
GR 126200, 16 August 2001

FACTS:

Between July 1981 and April 1984, Marinduque Mining entered into 3 mortgage agreements with
PNB and DBP involving its real properties located in Surigao del Norte, Negros Occidental, and Rizal, as
well as its equipment located therein. Marinduque failed to pay its loans, causing the foreclosure of the
said mortgages. PNB and DBP thereafter gained control of the said properties.
In the meantime, between July 16, 1982 to October 4, 1983, Marinduque Mining purchased and
caused to be delivered construction materials and other merchandise from Remington Industrial Sales
Corporation. The purchases remained unpaid as of August 1, 1984 when Remington filed a complaint
for a sum of money and damages against Marinduque Mining for the value of the unpaid construction
materials and other merchandise purchased by Marinduque Mining, as well as interest, attorney’s fees
and the costs of suit.
Remington’s original complaint was amended to include PNB, DBP, Maricalum Mining
Corporation (Maricalum Mining) and Island Cement Corporation (Island Cement) as co-defendants.
Remington asserted that Marinduque Mining, PNB, DBP, Nonoc Mining, Maricalum Mining and Island
Cement must be treated in law as one and the same entity by disregarding the veil of corporate fiction
since the personnel, key officers and rank-and-file workers and employees of co-defendants NMIC,
Maricalum and Island Cement creations of co-defendants PNB and DBP were the personnel of co-
defendant MMIC such that practically there has only been a change of name for all legal purpose and
intents.

ISSUE:

Whether or not the takeover of PNB and DBP over Marinduque Mining is in bad faith.

RULING:

NO.

161 | P a g e
Law 321_Corporation LAW_ Case Digest

Their actions are mandated under the law. Where the corporations have directors and officers in
common, there may be circumstances under which their interest as officers in one company may
disqualify them in equity from representing both corporations in transactions between the two. Thus,
where one corporation was ‘insolvent and indebted to another, it has been held that the directors of the
creditor corporation were disqualified, by reason of self-interest, from acting as directors of the debtor
corporation in the authorization of a mortgage or deed of trust to the former to secure such
indebtedness In the same manner that when the corporation is insolvent, its directors who are its
creditors cannot secure to themselves any advantage or preference over other creditors. They cannot
thus take advantage of their fiduciary relation and deal directly with themselves, to the injury of others
in equal right. If they do, equity will set aside the transaction at the suit of creditors of the corporation
or their representatives, without reference to the question of any actual fraudulent intent on the part of
the directors, for the right of the creditors does not depend upon fraud in fact, but upon the violation of
the fiduciary relation to the directors.
Directors of insolvent corporation, who are creditors of the company, can not secure to
themselves any preference or advantage over other creditors in the payment of their claims. It is not
good morals or good law. The governing body of officers thereof are charged with the duty of conducting
its affairs strictly in the interest of its existing creditors, and it would be a breach of such trust for them
to undertake to give any one of its members any advantage over any other creditors in securing the
payment of his debts in preference to all others.

 Disloyalty

JOHN GOKONGWEI
vs.
SEC, ANDRES SORIANO, et al.
GR L-45911, 11 April 1979

FACTS:

Gokonwei alleged that on September 18, 1976, individual respondents amended by bylaws of San
Miguel Corporation, basing their authority to do so on a resolution of the stockholders adopted on
March 13, 1961, when the outstanding capital stock of respondent corporation was only
P70,139.740.00, divided into 5,513,974 common shares at P10.00 per share and 150,000 preferred
shares at P100.00 per share. At the time of the amendment, the outstanding and paid up shares totalled
30,127,043, with a total par value of P301,270,430.00. It was contended that according to section 22 of
the Corporation Law and Article VIII of the by-laws of the corporation, the power to amend, modify,
repeal or adopt new by-laws may be delegated to the Board of Directors only by the affirmative vote of
stockholders representing not less than 2/3 of the subscribed and paid up capital stock of the
corporation, which 2/3 should have been computed on the basis of the capitalization at the time of the
amendment. Since the amendment was based on the 1961 authorization, petitioner contended that the
Board acted without authority and in usurpation of the power of the stockholders.
It was claimed that prior to the questioned amendment, petitioner had all the qualifications to be
a director of respondent corporation, being a substantial stockholder thereof; that as a stockholder,
petitioner had acquired rights inherent in stock ownership, such as the rights to vote and to be voted
upon in the election of directors; and that in amending the by-laws, respondents purposely provided for
petitioner's disqualification and deprived him of his vested right as afore-mentioned, hence the amended
by-laws are null and void.

ISSUE:

Whether or not SMC’s BoD acted in bad faith in making the amendment which disqualified
Gokongwei from being elected as Director.

RULING:

NO.

SMC is merely protecting its interest from Gokongwei, who owns companies in direct competition
with SMC’s business. Although in the strict and technical sense, directors of a private corporation are
not regarded as trustees, there cannot be any doubt that their character is that of a fiduciary insofar as
the corporation and the stockholders as a body are concerned. As agents entrusted with the
management of the corporation for the collective benefit of the stockholders, they occupy a fiduciary

162 | P a g e
Law 321_Corporation LAW_ Case Digest

relation, and in this sense the relation is one of trust. It springs from the fact that directors have the
control and guidance of corporate affairs and property; hence of the property interests of the
stockholders. Equity recognizes that stockholders are the proprietors of the corporate interests and are
ultimately the only beneficiaries thereof
It is obviously to prevent the creation of an opportunity for an officer or director of San Miguel
Corporation, who is also the officer or owner of a competing corporation, from taking advantage of the
information which he acquires as director to promote his individual or corporate interests to the
prejudice of San Miguel Corporation and its stockholders, that the questioned amendment of the by-
laws was made.
Certainly, where two corporations are competitive in a substantial sense, it would seem
improbable, if not impossible, for the director, if he were to discharge effectively his duty, to satisfy his
loyalty to both corporations and place the performance of his corporation duties above his personal
concerns.

ELEANOR ERICA STRONG, ET AL.


vs.
FRANCISCO GUTIERREZ REPIDE
G.R. No. L-7154. February 21, 1912

FACTS:

Eleanor Strong was the owner of 800 shares of the capital stock of Philippine Sugar Estate
Development Company. Gutierrez Rapide, owner of three-fourths shares of the company’s stock , 1 of
the 5 directors of the company and was elected by the board as administrator general of such company,
took steps to purchase the 800 shares owned by Strong, which he knew were in the possession of F.
Stuart Jones, as her agent. Instead of seeing Jones, who had an office next door, Repide employed one
Kauffman. Kaufmann, in turn, employed Mr. Sloan, a broker, to purchase the stock for him. Kauffman
told Sloan that the stock to be purchased was for a member of his wife’s family. This action by Repide
was due to the negotiations initiated by the government where the latter will purchase the company’s
lands (together with other friar lands) at a price which greatly enhance the value of the stock.
As a result of the negotiations, Jones, assuming he had the power and without consulting Strong,
sold the 800 shares. Strong filed a case to recover the shares from Repide on the ground that the shares
had been sold and delivered by Strong’s agent without authority to do so and on the ground that Repide
fraudulently concealed from Strong’s agent the facts affecting the value of the stock so sold and
delivered.

ISSUE:

Whether or notRepide, acting in good faith, has the duty to disclose to the agent of Strong the
facts bearing upon or which might affect the value of the stocks.

RULING:

YES.

The Court ruled that there is no relationship of a fiduaciary nature exists between a director and
a shareholder in a business corporation. There are cases, however, where, by reason of special facts,
such duty of a director to disclose to a shareholder the knowledge which he may possess regarding the
value of the shares of the company before he purchases any from a shareholder. Some special facts are
present in this case such as the fact the Repide is not only a director of the corporation but an owner of
three-fourths shares of its stock. He was the chief negotiator for the sale of all the lands and was acting
substantially as the agent of the shareholders by reason of his ownership of the shares. Thus, a
purchase of stock in a corporation by a director and owner of three-fourths of the entire capital stock,
who was also administrator general of the company and engaged in the negotiations which finally led to
the sale of company’s lands to the government at a price which greatly enhanced the value of the stock,
was fraudulent as procured by insidious machination where he employed an agent to make the
purchase, concealing both his identity as purchaser and his knowledge of the state of the negotiations
and their probable successful result

163 | P a g e
Law 321_Corporation LAW_ Case Digest

 Watered Stocks

LIRAG TEXTILE MILLS and BASILIO LIRAG


vs.
SSS, HON. PACIFICO DE CASTRO
GR L-33205, 31 August 1987

FACTS:

That on September 4, 1961, the SSS and Lirag Textile Mills, Inc. and Basilio Lirag entered into a
Purchase Agreement under which the plaintiff agreed to purchase from the said defendant preferred
shares of P1,000,000.00 subject to the conditions set forth in such agreement. Pursuant to the
Purchase Agreement of September 4, 1961, SSS, on January 31, 1962, paid Lirag Textile Mills, Inc. the
sum of P500,000.00 for which the said defendant issued to plaintiff 5,000 preferred shares with a par
value of P100.00 per share.
To guarantee the redemption of the stocks purchased by the plaintiff, the payment of dividends,
as well as the other obligations of the Lirag Textile Mills, Basilio signed the Purchase Agreement of
September 4, 1961 not only as president of the defendant corporation, but also as surety so that should
the Lirag Textile Mills, Inc. fail to perform any of its obligations in the said Purchase Agreement, the
surety shall immediately pay to the vendee the amounts then outstanding.
Notwithstanding such letters of demand to the defendant Basilio L. Lirag, Stock Certificates Nos.
128 and 139 issued to plaintiff are still unredeemed and no dividends have been paid on said stock
certificates;

ISSUE:

Whether or not Lirag Textile is liable to SSS.

RULING:

YES.

It failed to comply with its contractual stipulations. The Purchase Agreement is, indeed, a debt
instrument. Its terms and conditions unmistakably show that the parties intended the repurchase of the
preferred shares on the respective scheduled dates to be an absolute obligation which does not depend
upon the financial ability of petitioner corporation. This absolute obligation on the part of petitioner
corporation is made manifest by the fact that a surety was required to see to it that the obligation is
fulfilled in the event of the principal debtor's inability to do so. The unconditional undertaking of
petitioner corporation to redeem the preferred shares at the specified dates constitutes a debt which is
defined "as an obligation to pay money at some fixed future time, or at a time which becomes definite
and fixed by acts of either party and which they expressly or impliedly, agree to perform in the contract.
A stockholder sinks or swims with the corporation and there is no obligation to return the value
of his shares by means of repurchase if the corporation incurs losses and financial reverses, much less
guarantee such repurchase through a surety.

164 | P a g e
Law 321_Corporation LAW_ Case Digest

RICARDO NAVA
vs.
PEERS MARKETING CORP., RENATO CUSI and AMPARO CUSI
GR L-28120, 25 November 1976

FACTS:

Teofilo Po as an incorporator subscribed to eighty shares of Peers Marketing Corporation at one


hundred pesos a share or a total par value of eight thousand pesos. Po paid two thousand pesos or
twenty-five percent of the amount of his subscription. No certificate of stock was issued to him or, for
that matter, to any incorporator, subscriber or stockholder.
On April 2, 1966 Po sold to Ricardo A. Nava for two thousand pesos twenty of his eighty shares.
In the deed of sale Po represented that he was "the absolute and registered owner of twenty shares" of
Peers Marketing Corporation.
Nava requested the officers of the corporation to register the sale in the books of the corporation.
The request was denied because Po has not paid fully the amount of his subscription. Nava was
informed that Po was delinquent in the payment of the balance due on his subscription and that the
corporation had a claim on his entire subscription of eighty shares which included the twenty shares
that had been sold to Nava.

ISSUE:

Whether or not Peers may be compelled by mandamus to register the stocks in Nava’s name.

RULING:

NO.

There’s no certificate of stock issued in favor of Po. Shares of stock may be transferred by delivery
to the transferee of the certificate properly indorsed. "Title may be vested in the transferee by delivery of
the certificate with a written assignment or indorsement thereof" There should be compliance with the
mode of transfer prescribed by law.
The usual practice is for the stockholder to sign the form on the back of the stock certificate. The
certificate may thereafter be transferred from one person to another. If the holder of the certificate
desires to assume the legal rights of a shareholder to enable him to vote at corporate elections and to
receive dividends, he fills up the blanks in the form by inserting his own name as transferee. Then he
delivers the certificate to the secretary of the corporation so that the transfer may be entered in the
corporation's books. The certificate is then surrendered and a new one issued to the transferee.
That procedure cannot be followed in the instant case because, as already noted, the twenty
shares in question are not covered by any certificate of stock in Po's name. Moreover, the corporation
has a claim on the said shares for the unpaid balance of Po's subscription. A stock subscription is a
subsisting liability from the time the subscription is made. The subscriber is as much bound to pay his
subscription as he would be to pay any other debt. The right of the corporation to demand payment is
no less incontestable.
In this case no stock certificate was issued to Po. Without the stock certificate, which is the
evidence of ownership of corporate stock, the assignment of corporate shares is effective only between
the parties to the transaction.

 Derivative Suit: Remedies to Enforce Personal Liability

JUANITO ANG, for and in behalf of SUNRISE MARKETING (BACOLOD), INC.


vs.
SPOUSES ROBERTO and RACHEL ANG
G.R. No. 201675. June 19, 2013

165 | P a g e
Law 321_Corporation LAW_ Case Digest

FACTS:

Sps. Roberto and Rachel Ang took over the active management of [SMBI]. Through the
employment of sugar coated words, they were able to successfully manipulate the stocks sharings
between themselves at 50-50 under the condition that the procedures mandated by the Corporation
Code on increase of capital stock be strictly observed (valid Board Meeting). No such meeting of the
Board to increase capital stock materialized. It was more of an accommodation to buy peace.
Juanito claimed that payments to Nancy and Theodore ceased sometime after 2006. On 24
November 2008, Nancy and Theodore, through their counsel here in the Philippines, sent a demand
letter to "Spouses Juanito L. Ang/Anecita L. Ang and Spouses Roberto L. Ang/Rachel L. Ang" for
payment of the principal amounting to $1,000,000.00 plus interest at ten percent (10%) per annum, for
a total of $2,585,577.37 within ten days from receipt of the letter. 12 Roberto and Rachel then sent a
letter to Nancy and Theodore’s counsel on 5 January 2009, saying that they are not complying with the
demand letter because they have not personally contracted a loan from Nancy and Theodore.

ISSUE:

Whether or not the Honorable Court of Appeals erred in ordering the dismissal of the Complaint
on the ground that the case is not a derivative suit.

RULING:

NO.

The Complaint is not a derivative suit. A derivative suit is an action brought by a stockholder on
behalf of the corporation to enforce corporate rights against the corporation’s directors, officers or other
insiders. Under Sections 23 and 36 of the Corporation Code, the directors or officers, as provided under
the by-laws, have the right to decide whether or not a corporation should sue. Since these directors or
officers will never be willing to sue themselves, or impugn their wrongful or fraudulent decisions,
stockholders are permitted by law to bring an action in the name of the corporation to hold these
directors and officers accountable. In derivative suits, the real party ininterest is the corporation, while
the stockholder is a mere nominal party.

LISAM ENTERPRISES, INC. represented by LOLITA A. SORIANO, and LOLITA A. SORIANO


vs.
BANCO DE ORO UNIBANK, INC. (formerly PHILIPPINE COMMERCIAL INTERNATIONAL
BANK),* LILIAN S. SORIANO, ESTATE OF LEANDRO A. SORIANO, JR., REGISTER OF DEEDS OF
LEGASPI CITY, and JESUS L. SARTE
G.R. No. 143264 April 23, 2012

FACTS:

On August 13, 1999, petitioners filed a Complaint against respondents for Annulment of
Mortgage with Prayer for Temporary Restraining Order & Preliminary Injunction with Damages with the

166 | P a g e
Law 321_Corporation LAW_ Case Digest

RTC of Legaspi City. Petitioner Lolita A. Soriano alleged that she is a stockholder of petitioner Lisam
Enterprises, Inc. (LEI) and a member of its Board of Directors, designated as its Corporate Secretary.
On or about 28 March 1996, defendant Lilian S. Soriano and the late Leandro A. Soriano, Jr., as
husband and wife Spouses Soriano, in their personal capacity and for their own use and benefit,
obtained a loan from defendant PCIB (Legaspi Branch) (Banco de Oro Unibank, Inc.) in the total amount
of P20 Million.

ISSUE:

Whether or not the case will prosper.

RULING:

YES.

The courts should be liberal in allowing amendments to pleadings to avoid a multiplicity of suits
and in order that the real controversies between the parties are presented, their rights determined, and
the case decided on the merits without unnecessary delay. This liberality is greatest in the early stages
of a lawsuit, especially in this case where the amendment was made before the trial of the case, thereby
giving the petitioners all the time allowed by law to answer and to prepare for trial.
The Court enumerated the requisites for filing a derivative suit, as follows: a) the party bringing
the suit should be a shareholder as of the time of the act or transaction complained of, the number of
his shares not being material; b) he has tried to exhaust intra-corporate remedies, i.e., has made a
demand on the board of directors for the appropriate relief but the latter has failed or refused to heed
his plea; and c) the cause of action actually devolves on the corporation, the wrongdoing or harm having
been, or being caused to the corporation and not to the particular stockholder bringing the suit.
A reading of the amended complaint will reveal that all the foregoing requisites had been alleged
therein. Hence, the amended complaint remedied the defect in the original complaint and now
sufficiently states a cause of action.

STRATEGIC ALLIANCE DEV. CORPORATION


vs.
RADSTOCK SECURITIES
GR 178158, 04 December 2009

FACTS:

CCDCP Mining Corporation (CDCP Mining), an affiliate of CDCP, obtained loans from Marubeni
Corporation of Japan (Marubeni). A CDCP official issued letters of guarantee for the loans although
there was no CDCP Board Resolution authorizing the issuance of such letters of guarantee. CDCP
Mining secured the Marubeni loans when CDCP and CDCP Mining were still privately owned and
managed.
In 1983, CDCP’s name was changed to Philippine National Construction Corporation (PNCC) in
order to reflect that the Government already owned 90.3% of PNCC and only 9.70% is under private
ownership. Meanwhile, the Marubeni loans to CDCP Mining remained unpaid.
On 20 October 2000 and 22 November 2000, the PNCC Board of Directors (PNCC Board) passed
Board Resolutions admitting PNCC’s liability to Marubeni. Previously, for two decades the PNCC Board
consistently refused to admit any liability for the Marubeni loans.

167 | P a g e
Law 321_Corporation LAW_ Case Digest

In January 2001, Marubeni assigned its entire credit to Radstock Securities Limited (Radstock), a
foreign corporation. Radstock immediately sent a notice and demand letter to PNCC.
PNCC and Radstock entered into a Compromise Agreement. Under this agreement, PNCC shall
pay Radstock the reduced amount of P6,185,000,000.00 in full settlement of PNCC’s guarantee of CDCP
Mining’s debt allegedly totaling P17,040,843,968.00 (judgment debt as of 31 July 2006). To satisfy its
reduced obligation, PNCC undertakes to (1) "assign to a third party assignee to be designated by
Radstock all its rights and interests" to the listed real properties of PNCC; (2) issue to Radstock or its
assignee common shares of the capital stock of PNCC issued at par value which shall comprise 20% of
the outstanding capital stock of PNCC; and (3) assign to Radstock or its assignee 50% of PNCC’s 6%
share, for the next 27 years, in the gross toll revenues of the Manila North Tollways Corporation.
Luis Sison, a stockholder and former PNCC President and Chairman, filed a derivative suit
questioning the legality of the compromise agreement.

ISSUE:

Whether or not Sison’s derivative suit is valid.

RULING:

YES.

Sison has legal standing to challenge the Compromise Agreement. Although there was no
allegation that Sison filed the case as a derivative suit in the name of PNCC, it could be fairly deduced
that Sison was assailing the Compromise Agreement as a stockholder of PNCC.
A derivative action is a suit by a stockholder to enforce a corporate cause of action. Under the
Corporation Code, where a corporation is an injured party, its power to sue is lodged with its board of
directors or trustees. However, an individual stockholder may file a derivative suit on behalf of the
corporation to protect or vindicate corporate rights whenever the officials of the corporation refuse to
sue, or are the ones to be sued, or hold control of the corporation. In such actions, the corporation is the
real party-in-interest while the suing stockholder, on behalf of the corporation, is only a nominal party.
In this case, the PNCC Board cannot conceivably be expected to attack the validity of the
Compromise Agreement since the PNCC Board itself approved the Compromise Agreement. In fact, the
PNCC Board steadfastly defends the Compromise Agreement for allegedly being advantageous to PNCC.

ANTHONY YU et al.
vs.
JOSEPH YUKAYGUAN et al.
GR 177549, 18 June 2009

FACTS:

Petitioners and the respondents were all stockholders of Winchester Industrial Supply, Inc. On 15
October 2002, respondents filed against petitioners a verified Complaint forAccounting, Inspection of
Corporate Books and Damages through Embezzlement and Falsification of Corporate Records and
Accounts2[6] before the RTC of Cebu. The said Complaint was filed by respondents, in their own behalf
and as a derivative suit on behalf of Winchester, Inc., and was docketed as SRC Case No. 022-CEB. The
factual background of the Complaint was stated in the attached Affidavit executed by respondent
Joseph.
According to respondents, Winchester, Inc. was established and incorporated on 12 September
1977, with petitioner Anthony as one of the incorporators, holding 1,000 shares of stock worth
P100,000.00. Petitioner Anthony paid for the said shares of stock with respondent Joseph’s money,
thus, making the former a mere trustee of the shares for the latter.
The case at bar was initiated before the RTC by respondents as a derivative suit, on their own
behalf and on behalf of Winchester, Inc., primarily in order to compel petitioners to account for and
reimburse to the said corporation the corporate assets and funds which the latter allegedly
misappropriated for their personal benefit.

ISSUE:

168 | P a g e
Law 321_Corporation LAW_ Case Digest

Whether or not the derivative suit is valid.

RULING:

YES.

The general rule is that where a corporation is an injured party, its power to sue is lodged with its
board of directors or trustees. Nonetheless, an individual stockholder is permitted to institute a
derivative suit on behalf of the corporation wherein he holds stocks in order to protect or vindicate
corporate rights, whenever the officials of the corporation refuse to sue, or are the ones to be sued, or
hold the control of the corporation. In such actions, the suing stockholder is regarded as a nominal
party, with the corporation as the real party in interest. A derivative action is a suit by a shareholder to
enforce a corporate cause of action. The corporation is a necessary party to the suit. And the relief which
is granted is a judgment against a third person in favor of the corporation.
Glaringly, a derivative suit is fundamentally distinct and independent from liquidation
proceedings. They are neither part of each other nor the necessary consequence of the other. There is
totally no justification for the Court of Appeals to convert what was supposedly a derivative suit
instituted by respondents, on their own behalf and on behalf of Winchester, Inc. against petitioners, to a
proceeding for the liquidation of Winchester, Inc.
While it may be true that the parties earlier reached an amicable settlement, in which they agreed
to already distribute the assets of Winchester, Inc., and in effect liquidate said corporation, it must be
pointed out that respondents themselves repudiated said amicable settlement before the RTC, even after
the same had been partially implemented; and moved that their case be set for pre-trial. Attempts to
again amicably settle the dispute between the parties before the Court of Appeals were unsuccessful.
VIRGINIA GOCHAN et al.
vs.
RICHARD YOUNG, et al.
GR 131889, 12 March 2001

FACTS:

Felix Gochan and Sons Realty Corporation (Gochan Realty, for brevity) was registered with the
SEC on June, 1951, with Felix Gochan, Sr., Maria Pan Nuy Go Tiong, Pedro Gochan, Tomasa Gochan,
Esteban Gochan and Crispo Gochan as its incorporators.
Felix Gochan Sr.'s daughter, Alice, mother of respondents, inherited 50 shares of stock in Gochan
Realty from the former. She died in 1955, leaving the 50 shares to her husband, John Young, Sr.
In 1962, the Regional Trial Court of Cebu adjudicated 6/14 of these shares to her children,
Richard Young, David Young, Jane Young Llaban, John Young Jr., Mary Young Hsu and Alexander
Thomas Young.
Having earned dividends, these stocks numbered 179 by 20 September 1979. Five days later (25
September), at which time all the children had reached the age of majority, their father John Sr.,
requested Gochan Realty to partition the shares of his late wife by cancelling the stock certificates in his
name and issuing in lieu thereof, new stock certificates in the names of the children.

ISSUE:

Whether or not respondents have the legal personality to file a derivative suit on behalf of the
corporation.

RULING:

NO.

Where corporate directors have committed a breach of trust either by their frauds, ultra vires
acts, or negligence, and the corporation is unable or unwilling to institute suit to remedy the wrong, a
single stockholder may institute that suit, suing on behalf of himself and other stockholders and for the
benefit of the corporation, to bring about a redress of the wrong done directly to the corporation and
indirectly to the stockholders.
In the present case, the Complaint alleges all the components of a derivative suit. The allegations
of injury to the Spouses Uy can coexist with those pertaining to the corporation. The personal injury
suffered by the spouses cannot disqualify them from filing a derivative suit on behalf of the corporation.
It merely gives rise to an additional cause of action for damages against the erring directors. This cause
of action is also included in the Complaint filed before the SEC.

169 | P a g e
Law 321_Corporation LAW_ Case Digest

The Spouses Uy have the capacity to file a derivative suit in behalf of and for the benefit of the
corporation. The reason is that, as earlier discussed, the allegations of the Complaint make them out as
stockholders at the time the questioned transaction occurred, as well as at the time the action was filed
and during the pendency of the action.
As to the Intestate Estate of John Young, Sr., permitting an executor or administrator to
represent or to bring suits on behalf of the deceased, do not prohibit the heirs from representing the
deceased. These rules are easily applicable to cases in which an administrator has already been
appointed. But no rule categorically addresses the situation in which special proceedings for the
settlement of an estate have already been instituted, yet no administrator has been appointed. In such
instances, the heirs cannot be expected to wait for the appointment of an administrator; then wait
further to see if the administrator appointed would care enough to file a suit to protect the rights and
the interests of the deceased; and in the meantime do nothing while the rights and the properties of the
decedent are violated or dissipated.

WESTERN INSTITUTE OF TECHNOLOGY, et al.


vs.
RICARDO SALAS
GR 113032, 21 August 1997

FACTS:

The minority stockholders of WIT, sometime on June 1, 1986 in the principal office of WIT at La
Paz, Iloilo City, a Special Board Meeting was held. In attendance were other members of the Board
including one of the petitioners Reginald Villasis. Prior to aforesaid Special Board Meeting, copies of
notice thereof, dated May 24, 1986, were distributed to all Board Members, regarding the compensation
of the school’s officers, which was eventually passed.
A few years later, that is, on March 13, 1991, petitioners Homero Villasis, Prestod Villasis,
Reginald Villasis and Dimas Enriquez filed an affidavit-complaint against private respondents before the
Office of the City Prosecutor of Iloilo, as a result of which two (2) separate criminal informations, one for
falsification of a public document under Article 171 of the Revised Penal Code and the other for estafa
under Article 315, par. 1(b) of the RPC, were filed before Branch 33 of the Regional Trial Court of Iloilo
City. The charge for falsification of public document was anchored on the private respondents'
submission of WIT's income statement for the fiscal year 1985-1986 with the Securities and Exchange
Commission (SEC) reflecting therein the disbursement of corporate funds for the compensation of
private respondents based on Resolution No. 4, series of 1986, making it appear that the same was
passed by the board on March 30, 1986, when in truth, the same was actually passed on June 1, 1986,
a date not covered by the corporation's fiscal year 1985-1986 (beginning May 1, 1985 and ending April
30, 1986).
WIT questioned the legal standing of the petitioners to sue on its behalf, claiming it did not give
them authority to do do. Petitioner, however, contended that the case is a derivative suit.

ISSUE:

Whether or not the case at bar is a derivative suit.

RULING:

NO.

A derivative suit is an action brought by minority shareholders in the name of the corporation to
redress wrongs committed against it, for which the directors refuse to sue. It is a remedy designed by
equity and has been the principal defense of the minority shareholders against abuses by the majority.
Here, however, the case is not a derivative suit but is merely an appeal on the civil aspect of Criminal
Cases Nos. 37097 and 37098 filed with the RTC of Iloilo for estafa and falsification of public document.
Among the basic requirements for a derivative suit to prosper is that the minority shareholder who is
suing for and on behalf of the corporation must allege in his complaint before the proper forum that he
is suing on a derivative cause of action on behalf of the corporation and all other shareholders similarly
situated who wish to join. This is necessary to vest jurisdiction upon the tribunal in line with the rule
that it is the allegations in the complaint that vests jurisdiction upon the court or quasi-judicial body
concerned over the subject matter and nature of the action. This was not complied with by the
petitioners either in their complaint before the court a quo nor in the instant petition which, in part,
merely states that "this is a petition for review on certiorari on pure questions of law to set aside a
portion of the RTC decision in Criminal Cases Nos. 37097 and 37098" since the trial court's judgment of

170 | P a g e
Law 321_Corporation LAW_ Case Digest

acquittal failed to impose any civil liability against the private respondents. By no amount of equity
considerations, if at all deserved, can a mere appeal on the civil aspect of a criminal case be treated as a
derivative suit.

FIRST PHILIPPINE INTERNATIONAL BANK


vs.
COURT OF APPEALS, CARLOS EJERCITO, et al.
GR 115849, 24 January 1996

FACTS:

Janolo and Demetria and Producers Bank, through its bank manager Mercurio Rivera, entered
into a contract to sell involving parcels of land in Laguna owned by the bank. The sale, however, was
disapproved by the bank’s conservator (the bank is under receivership). Ejercito insisted that there was
already a perfected contract between him and the bank, considering that the offer that he made was
already approved by the bank’s board of directors. He then instituted a case for specific performance
against the bank.
On July 11, 1992, during the pendency of the proceedings in the Court of Appeals, Henry Co and
several other stockholders of the Bank filed an action, purportedly a “derivative suit”, against
Encarnacion, Demetria and Janolo “to declare any perfected sale of the property as unenforceable and to
stop Ejercito from enforcing or implementing the sale.” In his answer, Janolo argued that the Second
Case was barred by litis pendentia by virtue of the case then pending in the Court of Appeals. During
the pre-trial conference in the Second Case, plaintiffs filed a Motion for Leave of Court to Dismiss the
Case Without Prejudice. “Private respondent opposed this motion on the ground, among others, that
plaintiff’s act of forum shopping justifies the dismissal of both cases, with prejudice.

ISSUE:

Whether or not the case filed by the stockholders of the bank is a ‘derivative suit’.

RULING:

NO.

An individual stockholder is permitted to institute a derivative suit on behalf of the corporation


wherein he holds stock in order to protect or vindicate corporate rights, whenever the officials of the
corporation refuse to sue, or are the ones to be sued or hold the control of the corporation. In such
actions, the suing stockholder is regarded as a nominal party, with the corporation as the real party in
interest.
In the face of the damaging admissions taken from the complaint, petitioners, quite strangely,
sought to deny that the Second Case was a derivative suit, reasoning that it was brought, not by the
minority shareholders, but by Henry Co et al., who not only own, hold or control over 80% of the
outstanding capital stock, but also constitute the majority in the Board of Directors of petitioner Bank.
That being so, then they really represent the Bank. So, whether they sued “derivatively” or directly,
there is undeniably an identity of interests/entity represented.

COMMART PHILIPPINES, INC.


vs.

171 | P a g e
Law 321_Corporation LAW_ Case Digest

SEC, ALICE MAGLUTAC


GR 85318, 03 June 1991

FACTS:

Commart (Phils.), Inc., (Commart for short) is a corporation organized by two brothers, Jesus and
Mariano Maglutac, to engage in the brokerage business for the importation of fertilizers and other
products/commodities.
Sometime in June 1984, the two brothers agreed to go their separate ways, with Mariano being
persuaded to sell to Jesus his shareholdings in Commart amounting to 25% of the outstanding capital
stock. As part of the deal, a "Cooperative Agreement" was signed, between Commart (represented by
Jesus) and Mariano, in which, among others, Commart ceded to Mariano or to an "acceptable entity" he
may create, a portion of its business, with a pledge of mutual cooperation for a certain period so as to
enable Mariano to get his own corporation off the ground, so to speak.
Mariano's wife, Alice M. Maglutac, has been for years a stockholder and director of Commart, did not
dispose of her shareholdings, and thus continued as such even after the sale of Mariano's equity.
As broker and indentor, Commart's principal income came from commissions paid to it in U.S.
dollars by foreign suppliers of fertilizers and other commodities imported by Planters Products, Inc. and
other local importers.

ISSUE:

Whether or not Alice has the legal standing to file the derivative suit.

RULING:

YES.

A derivative suit has been the principal defense of the minority shareholder against abuses by the
majority. It is a remedy designed by equity for those situations where the management, through fraud,
neglect of duty, or other cause, declines to take the proper and necessary steps to assert the
corporation's rights. Indeed, to grant to Commart the light of withdrawing or dismissing the suit, at the
instance of majority stockholders and directors who themselves are the persons alleged to have
committed breaches of trust against the interest of the corporation, would be to emasculate the right of
minority stockholders to seek redress for the corporation. To consider the Notice of Dismissal filed by
Commart as quashing the complaint filed by Alice Maglutac in favor of the corporation would be to
defeat the very nature and function of a derivative suit and render the right to institute the action
illusory.

ELTON CHASE
vs.
DR. VICTOR BUENCAMINO
GR L-20395, 13 May 1985

FACTS:

172 | P a g e
Law 321_Corporation LAW_ Case Digest

Plaintiff Elton Chase, on the other hand, was the owner of Production Manufacturing Company,
of Portland, Oregon, USA, a corporation primarily dedicated to the operation of a machine shop and
heat-treating plant for the production of tractor parts.
Sometime in 1954, Chase was notified by the Highway Commission of the State of Oregon that
his factory was going to be in the path of a proposed highway. He was then advised to sell or face
expropriation and warned to remove his plant within a year. His distributor Craig Carrol told him of a
Dr. Buencamino of Manila who he said was interested in establishing a manufacturing plant in the
Philippines. Craig Carrol contacted Buencamino who told him to contact his associate William Cranker
in the United States. 8 Thereafter, a series of negotiations took place both here in Manila, and in the
United States, between Chase on the one hand, and Cranker and Buencamino, on the other, for the
purchase of Chase's factory (Production Manufacturing Company) and the establishment of a new
factory in Manila which was to be called the American Machinery Engineering Parts, Inc. (Amparts for
short). These negotiations culminated in a final agreement to the effect that - Elton Chase was to be paid
One Hundred Thousand Dollars ($100,000.00) and he would also be given a one-third interest in
Amparts, with the other two, Dr. Buencamino and Cranker, as the owner of the other two-thirds (2/3)
interest, 1/3 interest each; that in exchange for said $100,000.00 and the 1/3 interest, Chase was to
transfer to Amparts his tractor plant, ship his machineries to Manila, assuming all costs of dismantling,
preserving and crating for shipment to Manila, install said machineries at Amparts plant with the aid of
five technicians and finally, he has to be the production manager of Amparts.
Chase had shipped his machineries and had them installed in the Amparts plant in Pasig, Rizal.
Amparts then began operation with Dr. Buencamino as President, William Cranker as Manager and
Elton Chase as Production Manager. For sometime the three maintained harmonious relations but later
on distrust came in until finally Chase tendered his letter of resignation as Production Manager. He then
filed a derivative suit against Buencamino and Chase, who allegedly stole from the corporation. He
sought for the dissolution of the corporation.

ISSUE:

Whether or not the corporation may be dissolved.

RULING:

NO.

The case is of derivative in nature, therefore, it was filed for the benefit of the corporation. The
Court grant a dissolution because the action is a derivative one for the benefit of Amparts and not for
the personal benefit of Chase, and Amparts can not be benefited by its extinction; as to the ouster of Dr.
Buencamino from management, it should not be forgotten that Dr. Buencamino is not only a manager,
but is in fact 2/3 owner of Amparts and to oust him from management would amount to his
disenfranchisement as owner of the majority of the enterprise apart from the fact that it is also
established in the proofs that Amparts is already picking up and has been a going concern after Cranker
left unto him the direction of its affairs; the Court therefore having in mind all these finds that the
solution most equitable and just would be to limit its decision to imposing a monetary judgment upon
the guilty parties for the benefit of Amparts.

SAN MIGUEL CORPORATION, represented by EDUARDO DE LOS ANGELES


vs.
ERNEST KAHN, ANDRES SORIANO III, BENIGNO TODA, JR., ANTONIO ROXAS, ANTONIO PRIETO,
FRANCISCO EIZMENDI, JR., EDUARDO SORIANO, RALPH KAHN and RAMON DEL ROSARIO, JR.
G.R. No. 85339. August 11, 1989

FACTS:

33,133,266 shares of the outstanding capital stock of SMC were acquired 14 other
corporations, and were placed under a Voting Trust Agreement in favor of the late Andres Soriano, Jr.
However, 33,133,266 SMC shares were sequestered by the PCGG, on the ground that the stock belonged
to Eduardo Cojuangco, Jr., allegedly a close associate and dummy of former President Marcos. SMC
promptly suspended payment of the other installments of the price to the 14 seller corporations.
On December, 1986, the SMC Board, by Resolution No. 86-122, "decided to assume the loans
incurred by Neptunia for the down payment ((P500M)) on the 33,133,266 shares." The Board opined
that there was "nothing illegal in this assumption (of liability for the loans)," since Neptunia was "an

173 | P a g e
Law 321_Corporation LAW_ Case Digest

indirectly wholly owned subsidiary of SMC," there "was no additional expense or exposure for the SMC
Group, and there were tax and other benefits which would redound to the SMC group of companies.
However, at the meeting of the SMC Board, Eduardo de los Angeles, one of the PCGG representatives in
the SMC board, impugned said Resolution No. 86-122.

ISSUE:

Whether or not de los Angeles can file a derivative suit in behalf of the corporation.

RULING:

YES.

The Court ruled that it is claimed that since de los Angeles 20 shares represent only .00001644%
of the total number of outstanding shares (1 21,645,860), he cannot be deemed to fairly and adequately
represent the interests of the minority stockholders. The implicit argument — that a stockholder, to be
considered as qualified to bring a derivative suit, must hold a substantial or significant block of stock —
finds no support whatever in the law. The requisites for a derivative suit are as follows: (a) the party
bringing suit should be a shareholder as of the time of the act or transaction complained of, the number
of his shares not being material; (b) he has tried to exhaust intra-corporate remedies, i.e., has made a
demand on the board of directors for the appropriate relief but the latter has failed or refused to heed
his plea; and (c) the cause of action actually devolves on the corporation, the wrongdoing or harm having
been, or being caused to the corporation and not to the particular stockholder bringing the suit.
The bona fide ownership by a stockholder of stock in his own right suffices to invest him with
standing to bring a derivative action for the benefit of the corporation. The number of his shares is
immaterial since he is not suing in his own behalf, or for the protection or vindication of his own
particular right, or the redress of a wrong committed against him, individually, but in behalf and for the
benefit of the corporation.

HARRIE S. EVERETT, CRAL G. CLIFFORD, ELLIS H. TEAL and GEORGE W. ROBINSON


vs.
THE ASIA BANKING CORPORATION, NICHOLAS E. MULLEN, ERIC BARCLAY, ALFRED F. KELLY,
JOHN W. MEARS and CHARLES D. MACINTOSH
G.R. No. L-25241. November 3, 1926

FACTS:

In order more effectually to plunder the Company and to defraud these plaintiffs the said
defendants, Mullen, Barclay, Mears and Macintosh, made, executed and filed in the Bureau of
Commerce and Industry of the Philippine Islands, articles of incorporation of a corporation called the
"Philippine Motors Corporation," having its principal office in the City of Manila, a capital stock of
P25,000, of which the sum of P5,000, was alleged to have been subscribed and paid as follows: the
defendant Barclay P200, defendant Mears P1,200, defendant Kelly P1,200, defendant Macintosh P1,200,
defendant Mullen P1,200, the treasurer thereof being the defendant Mears. And these plaintiffs beg
leave to refer to the original articles of Incorporation on file in the said Bureau for greater certainty.
That at the time of such incorporation each and every one of the last above named defendants
was an officer or employee of the defendant Bank. That these plaintiffs have nor information nor means
of obtaining information as to whether the money alleged to have been described by them for their
shares of stock was of their personal funds and property or whether it was money furnished them by the
Bank of purpose moneys such incorporation was a fraud upon these plaintiffs for the reason that it was
intended for the sole purpose of taking over the assets of the Company and said defendants were
enabled to effectuate such intent by reason of their positions as officers and employees of the Bank.

ISSUE:

174 | P a g e
Law 321_Corporation LAW_ Case Digest

Whether or not plaintiffs have the capacity to sue.

RULING:

YES.

Invoking the well-known rule that shareholders cannot ordinarily sue in equity to redress wrongs
done to the corporation, but that the action must be brought by the Board of Directors, the appellees
argue — and the court below held — that the corporation Teal and Company is a necessary party
plaintiff and that the plaintiff stockholders, not having made any demand on the Board to bring the
action, are not the proper parties plaintiff. But, like most rules, the rule in question has its exceptions.
It is alleged in the complaint and, consequently, admitted through the demurrer that the corporation
Teal and Company is under the complete control of the principal defendants in the case, and, in these
circumstances, it is obvious that a demand upon the Board of Directors to institute an action and
prosecute the same effectively would have been useless, and the law does not require litigants to
perform useless acts.
The conclusion of the court below that the plaintiffs, not being stockholders in the Philippine
Motors Corporation, had no legal right to proceed against that corporation in the manner suggested in
the complaint evidently rest upon a misconception of the character of the action. In this proceeding it
was necessary for the plaintiffs to set forth in full the history of the various transactions which
eventually led to the alleged loss of their property and, in making a full disclosure, references to the
Philippine Motors Corporation appear to have been inevitable. It is to be noted that the plaintiffs seek no
judgment against the corporation itself at this stage of the proceedings.

175 | P a g e
Law 321_Corporation LAW_ Case Digest

RICARDO L. GAMBOA, LYDIA R. GAMBOA, HONORIO DE 1A RAMA, EDUARDO DE LA RAMA, and


the HEIRS OF MERCEDES DE LA RAMA-BORROMEO
vs.
HON. OSCAR R. VICTORIANO as Presiding Judge of the Court of First Instance of Negros
Occidental, Branch II, BENJAMIN LOPUE, SR., BENJAMIN LOPUE, JR., LEONITO LOPUE, and
LUISA U. DACLES
G.R. No. L-40620. May 5, 1979

FACTS:

The herein petitioners were sued by herein defendants to nullify the issuance of 823 shares of
stock of the Inocentes de la Rama, Inc. in favor of the petitioners.
On April 4, 1972, the respondents, are the owners of 1,328 shares of stock of the Inocentes de la
Rama, Inc., a domestic corporation, with an authorized capital stock of 3,000 shares, with a par value of
P100.00 per share, 2,177 of which were subscribed and issued, thus leaving 823 shares unissued. Then
President and Vice-President of the corporation, respectively, the defendants Mercedes R. Borromeo,
Honorio de la Rama, and Ricardo Gamboa, remaining members of the board of directors of the
corporation, in order to forestall the takeover by the plaintiffs of the afore-named corporation,
surreptitiously met and elected Ricardo L. Gamboa and Honorio de la Rama as president and vice-
president of the corporation, respectively, and passed a resolution authorizing the sale of the 823
unissued shares of the corporation to the defendants, at par value, after which the petitioners were
elected to the board of directors of the corporation.
The respondents claimed that the sale of the unissued 823 shares of stock of the corporation was
in violation of the plaintiffs' and pre-emptive rights and made without the approval of the board of
directors representing 2/3 of the outstanding capital stock, and is in disregard of the strictest relation of
trust existing between the defendants, as stockholders. The respondents prayed that a writ of
preliminary injunction be issued restraining the defendants from committing, or continuing the
performance of an act tending to prejudice, diminish or otherwise injure the plaintiffs' rights in the
corporate properties and funds of the corporation, and from disposing, transferring, selling, or otherwise
impairing the value of the 823 shares of stock illegally issued. The respondent court granted the prayer.

ISSUES:

Whether or not the proper action is a derivative suit.

RULING:

YES.

An individual stockholder is permitted to institute a derivative suit on behalf of the corporation


wherein he holds stock in order to protect or vindicate corporate rights, whenever the officials of the
corporation refuse to sue, or are the ones to be sued or hold the control of the corporation. In such
actions, the suing stockholder is regarded as a nominal party, with the corporation as the real party in
interest. 12 In the case at bar, however, the plaintiffs are alleging and vindicating their own individual
interests or prejudice, and not that of the corporation. At any rate, it is yet too early in the proceedings
since the issues have not been joined. Besides, misjoinder of parties is not a ground to dismiss an
action.

CATALINA R. REYES
vs.
HON. BIENVENIDO A. TAN, as Judge of the Court of First Instance of Manila, Branch XIII and
FRANCISCA R. JUSTINIANI
G.R. No. L-16982. September 30, 1961

FACTS:

176 | P a g e
Law 321_Corporation LAW_ Case Digest

The corporation, Roxas-Kalaw Textile Mills, Inc., was organized on June 5, 1954 by defendants
Cesar K. Roxas, Adelia K. Roxas, Benjamin M. Roxas, Jose Ma. Barcelona and Morris Wilson, for and on
behalf of the following primary principals with the following shareholdings: Adelia K. Roxas, 1200 Class
A shares; I. Sherman, 900 Class A shares; Robert W. Born, 450 Class A shares and Morris Wilson, 450
Class A shares; that the respondent holds both Class A and Class B shares and number and value
thereof are is follows: Class A — 50 shares, Class B — 1,250 shares.
On May 8, 1957, the Board of Directors approved a resolution designating one Dayaram as co-
manager and Morris Wilson was likewise designated as co-manager with responsibilities for the
management of the factory only’. An office in New York was opened for the purpose of supervising
purchases, which purchases must have the unanimous agreement of Cesar K. Roxas, New York resident
member of the board of directors, Robert Born and Wadhumal Dalamal or their respective
representatives. Several purchases aggregating $289,678.86 were made in New York for raw materials
and shipped to the Philippines, which shipment were found out to consist not of raw materials but
already finished products, for which reasons the Central Bank of the Philippines stopped all dollar
allocations for raw materials for the corporation which necessarily led to the paralyzation of the
operation of the textile mill and its business.

ISSUES:

Whether or not a derivative suit will prosper.

RULING:

NO.

The claim that respondent Justiniani did not take steps to remedy the illegal importation for a
period of two years is without merit. During that period of time respondent had the right to assume and
expect that the directors would remedy the anomalous situation of the corporation brought about by
their own wrong doing. Only after such period of time had elapsed could respondent conclude that the
directors were remiss in their duty to protect the corporation property and business. The fraud
consisted in importing finished textile instead of raw cotton for the textile mill; the fraud, therefore, was
committed by the manager of the business and was consented to by the directors, evidently beyond
reach of respondent as treasurer for that period.
The directors permitted the fraudulent transaction to go unpunished and nothing appears to have
been done to remove the erring purchasing managers. In a way the appointment of a receiver may have
been thought of by the court below so that the dollar allocation for raw material may be revived and the
textile mill placed on an operating basis.

CANDIDO PASCUAL
vs.
EUGENIO DEL SAZ OROZCO, ET AL.
G.R. No. L-5174. March 17, 1911

FACTS:

This action was brought by the plaintiff Pascual, in his own right as a stockholder of the bank, for
the benefit of the bank, and all the other stockholders thereof. The Banco Español-Filipino is a banking
corporation, constituted as such by royal decree of the Crown of Spain in the year 1854, the original
grant having been subsequently extended and modified by royal decree of July 14, 1897, and by Act No.
1790 of the Philippine Commission.

177 | P a g e
Law 321_Corporation LAW_ Case Digest

It is alleged in the amended complaint that the only compensation contemplated or provided for
the managing officers of the bank was a certain per cent of the net profits resulting from the bank's
operations, as set forth in article 30 of its reformed charter or statutes.
The gist of the first and second causes of action is as follows: The defendants constitute a
majority of the present board of directors of the bank, who alone can authorize an action against them
in the name of the corporation. It appears that during the years 1903, 1904, 1905, and 1907 the
defendants and appellees, without the knowledge, consent, or acquiescence of the stockholders,
deducted their respective compensation from the gross income instead of from the net profits of the
bank, thereby defrauding the bank and its stockholders of approximately P20,000 per annum.
The second cause of action sets forth that defendants' and appellees' immediate predecessors in office in
the bank during the years 1899, 1900, 1901, and 1902, committed the same illegality as to their
compensation as is charged against the defendants themselves. In the four years immediately following
the year 1902, the defendants and appellees were the only officials or representatives of the bank who
could and should investigate and take action in regard to the sums of money thus fraudulently
appropriated by their predecessors. They were the only persons interested in the bank who knew of the
fraudulent appropriation by their predecessors.
The court below sustained the demurrer as to the first and second causes of action on the ground that
in actions of this character the plaintiff must aver in his complaint that he was the owner of stock in the
corporation at the time of the occurrences complained of, or else that the stock has since devolved upon
him by operation of law.

ISSUE:

Whether or not the petitioner has a cause of action to file a derivative suit.

RULING:

YES.

As to the first cause of action: In suits of this character the corporation itself and not the plaintiff
stockholder is the real party in interest. The rights of the individual stockholder are merged into that of
the corporation. It is a universally recognized doctrine that a stockholder in a corporation has no title
legal or equitable to the corporate property; that both of these are in the corporation itself for the benefit
of all the stockholders. So it is clear that the plaintiff, by reason of the fact that he is a stockholder in
the bank (corporation) has a right to maintain a suit for and on behalf of the bank, but the extent of
such a right must depend upon when, how, and for what purpose he acquired the shares which he now
owns.
As to the Second cause of action: It affirmatively appears from the complaint that the plaintiff was
not a stockholder during any of the time in question in this second cause of action. Upon the question
whether or not a stockholder can maintain a suit of this character upon a cause of action pertaining to
the corporation when it appears that he was not a stockholder at the time of the occurrence of the acts
complained of and upon which the action is based, the authorities do not agree.
POWERS OF CORPORATION
 Theory of Special Capacities v. Theory of General Capacities

ACEBEDO OPTICAL COMPANY, INC.


vs.
THE HONORABLE COURT OF APPEALS, Hon. MAMINDIARA MANGOTARA, in his capacity as
Presiding Judge of the RTC, 12th Judicial Region, Br. 1, Iligan City; SAMAHANG OPTOMETRIST
Sa PILIPINAS — Iligan City Chapter, LEO T. CAHANAP, City Legal Officer, and Hon. CAMILO P.
CABILI, City Mayor of Iligan
G.R. No. 100152. March 31, 2000

FACTS:

Petitioner applied with the Office of the City Mayor of Iligan for a business permit. After
consideration of petitioner's application and the opposition interposed thereto by local optometrists,
respondent City Mayor issued Business Permit No. 5342 subject to the following conditions that since it
is a corporation, Acebedo cannot put up an optical clinic but only a commercial store; it cannot examine
and/or prescribe reading and similar optical glasses for patients, because these are functions of optical
clinics; it cannot sell reading and similar eyeglasses without a prescription having first been made by an
independent optometrist (not its employee) or independent optical clinic and can only sell directly to the

178 | P a g e
Law 321_Corporation LAW_ Case Digest

public, without need of a prescription, Ray-Ban and similar eyeglasses; it cannot advertise optical lenses
and eyeglasses, but can advertise Ray-Ban and similar glasses and frames; and is allowed to grind
lenses but only upon the prescription of an independent optometrist.
Private respondent Samahan ng Optometrist Sa Pilipinas (SOPI), Iligan Chapter, through its
Acting President, Dr. Frances B. Apostol, lodged a complaint against the petitioner before the Office of
the City Mayor, alleging that Acebedo had violated the conditions set forth in its business permit and
requesting the cancellation and/or revocation of such permit.

ISSUES:

Whether or not the act of the Respondent Mayor was lawful.

RULING:

NO.

The authority of city mayors to issue or grant licenses and business permits is beyond cavil.
However, the power to grant or issue licenses or business permits must always be exercised in
accordance with law, with utmost observance of the rights of all concerned to due process and equal
protection of the law. In the case under consideration, the business permit granted by respondent City
Mayor to petitioner was burdened with several conditions.
Distinction must be made between the grant of a license or permit to do business and the
issuance of a license to engage in the practice of a particular profession. The first is usually granted by
the local authorities and the second is issued by the Board or Commission tasked to regulate the
particular profession. A business permit authorizes the person, natural or otherwise, to engage in
business or some form of commercial activity. A professional license, on the other hand, is the grant of
authority to a natural person to engage in the practice or exercise of his or her profession.
A business permit is issued primarily to regulate the conduct of business and the City Mayor
cannot, through the issuance of such permit, regulate the practice of a profession, like that of
optometry. Such a function is within the exclusive domain of the administrative agency specifically
empowered by law to supervise the profession, in this case the Professional Regulations Commission
and the Board of Examiners in Optometry.

 Express, Implied and Incidental Powers, Distinguished

PILIPINAS LOAN COMPANY, INC.


vs.
HON. SECURITES AND EXCHANGE COMMISSION AND FILIPINAS PAWNSHOP, INC.
G.R. No. 104720. April 4, 2001

FACTS:

Private respondent Filipinas Pawnshop, Inc. is a duly organized corporation registered with the
Securities and Exchange Commission on February 9, 1959. The articles of incorporation of private
respondent states that its primary purpose is to extend loans at legal interest on the security of either
personal properties or on the security of real properties, and to finance installment sales of motor
vehicles, home appliances and other chattels.
Petitioner is a lending corporation duly registered with the SEC on July 27, 1989. Based on its
articles of incorporation, the primary purpose of petitioner is “to act as a lending investor or, otherwise,
to engage in the practice of lending money or extending loans on the security of real or personal,
tangible or intangible properties whether as pledge, real or chattel mortgage or otherwise, xxx without
however, engaging in pawnbroking as defined under PD 114."
Private respondent filed a complaint with the Prosecution and Enforcement Department (PED) of
the SEC and alleged that: (1) petitioner, contrary to the restriction set by the Commission, has been
operating and doing business as a pawnbroker, pawnshop or "sanglaan" in the same neighborhood
where private respondent has had its own pawnshop for 30 years in violation of its primary purpose and
without the imprimatur of the Central Bank to engage in the pawnshop business thereby causing unjust
and unfair competition with private respondent. Petitioner denied that it is engaged in the pawnshop
business, alleging that it is a lending investor duly registered with the Central Bank.

ISSUES:

179 | P a g e
Law 321_Corporation LAW_ Case Digest

Whether or not petitioner violated its primary franchise.

RULING:

YES.

A corporation, under the Corporation Code, has only such powers as are expressly granted to it
by law and by its articles of incorporation, those which may be incidental to such conferred powers,
those reasonably necessary to accomplish its purposes and those which may be incident to its existence.
In the case at bar, the limit of the powers of petitioner as a corporation is very clear, it is
categorically prohibited from "engaging in pawnbroking as defined under PD 114". Hence, in
determining what constitutes pawnbrokerage, the relevant law to consider is PD 114.
Indispensable therefore to the determination of whether or not petitioner had violated its articles
of incorporation, was an inquiry by the SEC if petitioner was holding out itself to the public as a
pawnshop. It must be stressed that the determination of whether petitioner violated PD 114 was merely
incidental to the regulatory powers of the SEC, to see to it that a corporation does not go beyond the
powers granted to it by its articles of incorporation.

LUNETA MOTOR COMPANY


vs.
A.D. SANTOS, INC., ET AL.
G.R. No. L-17716. July 31, 1962

FACTS:

On December 31, 1941, to secure payment of a loan evidenced by a promissory note executed by
Nicolas Concepcion and guaranteed by one Placido Esteban in favor of petitioner, Concepcion executed a
chattel mortgage covering the above mentioned certificate in favor of petitioner.
Thereafter, he constituted a second mortgage on the same certificate to secure payment of a
subsequent loan obtained by Concepcion from the Rehabilitation Finance Corporation (now
Development Bank of the Philippines). This second mortgage was approved by the respondent
Commission, subject to the mortgage lien in favor of petitioner. The certificate was later sold to
Francisco Benitez, Jr., who resold it to Rodi Taxicab Company. Both sales were made with assumption
of the mortgage in favor of the RFC, and were also approved provisionally by the Commission, subject to
petitioner's lien.
On October 10, 1953 petitioner filed an action to foreclose the chattel mortgage executed in its
favor by Concepcion. While the above case was pending, the RFC also instituted foreclosure proceedings
on its second chattel mortgage, and as a result of the decision in its favor therein rendered, the
certificate of public convenience was sold at public auction in favor of Amador D. Santos for P24,010.00
on August 31, 1956. Santos immediately applied with the Commission for the approval of the sale, and
the same was approved on January 26, 1957, subject to the mortgage lien in favor of petitioner.

ISSUE:

Whether or not the purpose for which petitioner was organized and the transaction of its lawful
business reasonably and necessarily requires acquisition and holds the certificate and operates as a
common carrier by land.

RULING:

NO.

Under Section 13 (5) of the Corporation Law, a corporation created thereunder may purchase,
hold, etc., and otherwise deal in such real and personal property is the purpose for which the
corporation was formed may permit, and the transaction of its lawful business may reasonably and
necessarily require.

180 | P a g e
Law 321_Corporation LAW_ Case Digest

Petitioner’s corporate purposes are to carry on a general mercantile and commercial business,
etc., and that it is authorized in its articles of incorporation to operate and otherwise deal in and
concerning automobiles and automobile accessories' business in all its multifarious ramification and to
operate, etc., and otherwise dispose of vessels and boats, etc., and to own and operate steamship and
sailing ships and other floating craft and deal in the same and engage in the Philippine Islands and
elsewhere in the transportation of persons, merchandise and chattels by water; all this incidental to the
transportation of automobiles.
The Court finds that Petitoner’s articles of incorporation are precisely the best evidence that it has
no authority at all to engage in the business of land transportation and operate a taxicab service. That it
may operate and otherwise deal in automobiles and automobile accessories; that it may engage in the
transportation of persons by water does not mean that it may engage in the business of land
transportation — an entirely different line of business. If it could not thus engage in the line of business,
it follows that it may not acquire a certificate of public convenience to operate a taxicab service, such as
the one in question, because such acquisition would be without purpose and would have no necessary
connection with petitioner's legitimate business.
TERESA ELECTRIC AND POWER CO., INC.
vs.
PUBLIC SERVICE COMMISSION and FILIPINAS CEMENT CORPORATION
G.R. No. L-21804. September 25, 1967

FACTS:

The petitioner Teresa Electric Light and Power Co., Inc. is a domestic corporation operating an
electric plant in Teresa, Rizal, under a subsisting certificate of public convenience and necessity issued
on June 2, 1960, while the respondent Filipinas is likewise a domestic corporation engaged in the
manufacture and sale of cement.
On May 24, 1962, Filipinas filed an application with the Public Service Commission for a
certificate of public convenience to install, maintain and operate an electric plant in sitio Kaysapon of
barrio Pamanaan, municipality of Teresa, Rizal, for the purpose of supplying electric power and light to
its cement factory and its employees living within its compound.
Petitioner opposed alleging that it is the duly authorized operator of an electric light, heat and
power service in Teresa, Rizal and that Filipinas is not authorized by its articles of incorporation to
operate an electric plant; that the Municipal Council of Teresa had not authorized it either to operate the
proposed service since Filipinas' principal business does not come within the jurisdiction of the
respondent Commission.

ISSUES:

Whether or not under its articles of incorporation Filipinas is authorized to operate and maintain
an electric plant.

RULING:

YES.

The Articles of Incorporation of Filipinas (paragraph 7) provide for authority to secure from any
governmental, state, municipality, or provincial, city or other authority, and to utilize and dispose of in
any lawful manner, rights, powers, privileges, franchises and concessions — obviously necessary or at
least related to the operation of its cement factory. Moreover, said Articles of Incorporation also provide
that the corporation may generally perform any and all acts connected with the business of
manufacturing Portland cement or arising therefrom or incidental thereto.
It cannot be denied that the operation of an electric light, heat and power plant is necessarily connected
with the business of manufacturing cement. Moreover, it has been established in this case that
petitioner was in no condition to supply the power needs of Filipinas, because its load capacity was only
200 kilowatts while Filipinas was in need of 6,000 Kilowatts power to operate its cement factory.

181 | P a g e
Law 321_Corporation LAW_ Case Digest

ANTHONY POWERS, BERTEL FASSNACHT, RICHARD I GUARDIAN, JOANN KELLY, LANDLESS,


AMADO MACASAET, JAVIER MACICIORATUSHI NAKAI KAY NG, JAMES ROBERSON, FREDERICK
SEGGERMAN, ARTHUR YANG, EZRA TOEG, ISIDRO CO, In behalf of themselves and 316 other
Associate Members all other Associate Members similarly situated, and in behalf of and for the
benefit of the INTERNATIONAL SCHOOL, INC.
vs.
DONALD I. MARSHALL, CHARLES ANGEVINE, CARLOS D. ARGUELLES, BRYCE F. BASTIAN,
GABRIEL DIMACHE, JOSE FLORENTO, JAMES T. HODGE, ROSEMARY IYAS, EUSEBIO R.
LUZURIAGA, THOMAS C. NIBLOCK Board of Trustees of the International School, Inc., and MAX
SNYDER Superintendent, International School, Inc.
G.R. No. L-48064. May 9, 1988

FACTS:

On July 16, 1975, the fourteen (14) plaintiffs, all associate members of the International School,
Inc., brought an action for injunction against the ten (10) members of the Board of Trustees of the
school, praying that said Trustees be enjoined from collecting a "development fee" of P2,625.00 per
child-enrollee per school year for a period of twelve (12) years, beginning with the school year 1975-
1976, as a pre-requisite for re-enrollment in said school.
The suit was precipitated by a letter addressed to the parents of the students, giving notice that
the Board of Trustees had decided to embark on a program to construct new buildings and remodel
existing ones. The Board intended to raise the needed funds primarily through subscriptions to capital
notes and prepayment certificates, and any deficiency from these sources would be covered by collecting
a so-called "development fees" of P2,625 from each enrollee starting with the school year 1975-1976 and
continuing up to the school year 1986-1987.
The trial court issued an order temporarily restraining the defendants or their authorized
representatives and agents from executing and/or enforcing in any manner the development program
and after the submission of the parties' memoranda the trial court issued an order dismissing the
complaint for lack of valid cause of action

ISSUE:

Whether or not the Board of Trustees of the International School was authorized to adopt the
development plan for which the disputed fee was being collected from the students.

RULING:

YES.

Section 2 of Article 3 of the By-Laws of the International School, Inc. provides: The Board of
Trustees, in addition to the powers conferred by these By-Laws, shall have the right to such powers and
do such acts as may be lawfully exercised or performed by the corporation, subject to applicable laws
and to the provisions of the articles of incorporation and the By-Laws.
Section 2 (b) of P.D. No. 732 granting certain rights to the International School, Inc., expressly
authorized the Board of Trustees, upon consultation with the Secretary of Education and Culture, to
determine the amount of fees and assessments which may be reasonably imposed upon its students, to
maintain or conform to the school standard of education." Such consultation had been made with the
Secretary of Education and Culture who expressed his conformity with the reasonableness of the
assessment of P2,625.00 per student for the whole school year to carry out its development program.
Since the collection of the development fee had been approved by the Board of Trustees of the
International School, Inc., it was a valid exercise of corporate power by the Board, and said assessment
was binding upon all the members of the corporation.

 Power to Have/Use Corporate Name and Seal

182 | P a g e
Law 321_Corporation LAW_ Case Digest

LAUREANO INVESTMENT & DEVELOPMENT CORPORATION


vs.
THE HONORABLE COURT OF APPEALS and BORMAHECO, INC.
G.R. No. 100468. May 6, 1997

FACTS:

The Spouses Reynaldo Laureano and Florence Laureano are majority stockholders of petitioner
Corporation who entered into a series of loan and credit transactions with Philippine National
Cooperative Bank. To secure payment of the loans, they executed Deeds of Real Estate Mortgage dated
December 11, 1962, January 9, 1963, July 2, 1963 and September 5, 1964, for the following amounts:
P100,000.00, P20,000.00, P70,000.00 and P13,424.04, respectively. Spouses Laureano failed to pay
their indebtedness, thus PNCB applied for extrajudicial foreclosure of the real estate mortgages. The
bank was the purchaser of the properties in question in the foreclosure sale and titles thereof were
consolidated in PNCB's name on February 20, 1984. PNCB did not secure a writ of possession nor did it
file ejectment proceedings against the Laureano spouses, because there were then pending cases
involving the titles of ownership of subject two lots, which are situated at Bel-Air Subdivision, Makati,
Metro Manila.
Private respondent Bormaheco, Inc. became the successor of the obligations and liabilities of
PNCB over subject lots by virtue of a Deed of Sale/Assignment. Transfer Certificate of Title Nos. 157724
and 157725 over the lots in question were issued on October 12, 1988 in the name of Bormaheco.
Five (5) days after securing titles over the said properties, Petitioner Corporation filed on January
18, 1989 its Motion for Intervention and to Admit Attached Complaint in Intervention in said.
Respondent Bormaheco filed its Motion to Strike out the Complaint in Intervention and all related
pleadings filed by LIDECO Corporation. The motion was granted stating that Intervenor LIDECO
Corporation and LAUREANO INVESTMENT AND DEVELOPMENT CORPORATION are two (2) separate
and distinct entities, therefore, no way whatsoever that LIDECO Corporation's interests will be adversely
affected by the outcome of the instant case. Thus, intervenor LIDECO lacks personality to intervene in
the instant

ISSUES:

Whether or not Respondent Bormaheco, Inc. is estopped from contesting the legal personality to
sue of "Lideco Corporation".

RULING:

NO.

Bad faith implies a conscious and intentional design to do a wrongful act for a dishonest purpose
or moral obliquity; bad faith contemplates a state of mind affirmatively operating with furtive design or
ill will.
Other than its bare allegations that private respondent acted in bad faith, petitioner failed to
show that the former acted consciously and deliberately to achieve a dishonest purpose or moral
obliquity, or was motivated by ill will. Rather, as discussed above, no false representation was contrived
nor concealment made by private respondent. Neither did it deliberately convey facts other than, or
inconsistent with, what it now asserts and upon which petitioner had relied or acted upon due to the
representations of private respondent.

 Power to Sue and be Sued

TAM WING TAK


vs.
HON. RAMON P. MAKASIAR (in his Capacity as Presiding Judge of the Regional Trial Court of
Manila, Branch 35) and ZENON DE GUIA (in his capacity as Chief State Prosecutor)
G.R. No. 122452. January 29, 2001

FACTS:

183 | P a g e
Law 321_Corporation LAW_ Case Digest

On November 11, 1992, petitioner, in his capacity as director of Concord-World Properties, Inc.,
(Concord for brevity), a domestic corporation, filed an affidavit-complaint with the Quezon City
Prosecutor's Office, charging Vic Ang Siong with violation of B.P. Blg. 22 alleging that a check for the
amount of P83,550,000.00, issued by Vic Ang Siong in favor of Concord, was dishonored when
presented for encashment.
Vic Ang Siong sought the dismissal of the case on two grounds: First, that petitioner had no
authority to file the case on behalf of Concord, the payee of the dishonored check, since the firm's board
of directors had not empowered him to act on its behalf. Second, he and Concord had already agreed to
amicably settle the issue after he made a partial payment of P19,000,000.00 on the dishonored check.
The City Prosecutor dismissed the case. Petitioner moved for reconsideration but the City Prosecutor
denied such. On November 8, 1994, petitioner appealed the dismissal of his complaint and the Chief
State Prosecutor dismissed the appeal for having been filed out of time.

ISSUES:

Whether or not petitioner is the proper party to institute the case.

RULING:

NO.

In general, mandamus may be resorted to only where one's right is founded clearly in law and not
when it is doubtful. The exception is to be found in criminal cases where mandamus is available to
compel the performance by the public prosecutor of an ostensibly discretionary function, where by
reason of grave abuse of discretion on his part, he willfully refuses to perform a duty mandated by law.
Thus, mandamus may issue to compel a prosecutor to file information when he refused to do so in spite
of the prima facie evidence of guilt.
First, with respect to the agreement between Concord and Victor Ang Siong to amicably settle
their difference, we find this resort to an alternative dispute settlement mechanism as not contrary to
law, public policy, or public order. Efforts of parties to solve their disputes outside of the courts are
looked on with favor, in view of the clogged dockets of the judiciary.
Second, it is not disputed in the instant case that Concord, a domestic corporation, was the payee
of the bum check, not petitioner. Therefore, it is Concord, as payee of the bounced check, which is the
injured party. Since petitioner was neither a payee nor a holder of the bad check, he had neither the
personality to sue nor a cause of action against Vic Ang Siong.
Petitioner failed to show any proof that he was authorized or deputized or granted specific powers
by Concord's board of director to sue Victor Ang Siong for and on behalf of the firm. Petitioner as a
minority stockholder and member of the board of directors had no such power or authority to sue on
Concord's behalf. Nor can we uphold his act as a derivative suit. For a derivative suit to prosper, it is
required that the minority stockholder suing for and on behalf of the corporation must allege in his
complaint that he is suing on a derivative cause of action on behalf of the corporation and all other
stockholders similarly situated who may wish to join him in the suit.

NORA A. BITONG
vs.
COURT OF APPEALS (FIFTH DIVISION), EUGENIA D. APOSTOL, JOSE A. APOSTOL, MR. & MS.
PUBLISHING CO., LETTY J. MAGSANOC, AND ADORACION G. NUYDA
G.R. No. 123553. July 13, 1998
NORA A. BITONG
vs.
COURT OF APPEALS (FIFTH DIVISION) and EDGARDO B. ESPIRITU
(CA-G.R. No. 33873) July 13, 1998

FACTS:

Bitong alleged that she was the treasurer and member of the BoD of Mr. & Mrs. Corporation. She
filed a complaint with the SEC to hold respondent spouses Apostol liable for fraud, misrepresentation,
disloyalty, evident bad faith, conflict of interest and mismanagement in directing the affairs of the
corporation to the prejudice of the stockholders. She alleges that certain transactions entered into by
the corporation were not supported by any stockholder’s resolution. The complaint sought to enjoin
Apostol from further acting as president-director of the corporation and from disbursing any money or
funds.

184 | P a g e
Law 321_Corporation LAW_ Case Digest

Apostol contends that Bitong was merely a holder-in-trust of the JAKA shares of the corporation,
hence, not entitled to the relief she prays for. SEC Hearing Panel issued a writ enjoining Apostol. After
hearing the evidence, SEC Hearing Panel dissolved the writ and dismissed the complaint filed by Bitong.
Bitong appealed to the SEC en banc which reversed SEC Hearing Panel decision. Apostol filed petition
for review with the CA. CA reversed SEC en banc ruling holding that Bitong was not the owner of any
share of stock in the corporation and therefore, not a real party in interest to prosecute the complaint.

ISSUE:

Whether or not Bitong was the real party in interest.

RULING:

NO.

It could be gleaned that Bitong was not a bona fide stockholder of the corporation. Several
corporate documents disclose that the true party in interest was JAKA. Although her buying of the
shares were recorded in the Stock and Transfer Book of the corporation, and as provided by Sec. 63 of
the Corp Code that no transfer shall be valid except as between the parties until the transfer is recorded
in the books of the corporation, and upon its recording the corporation is bound by it and is estopped to
deny the fact of transfer of said shares, this provision is not conclusive even against the corporation but
are prima facie evidence only.
Parol evidence may be admitted to supply the omissions in the records, explain ambiguities, or
show what transpired where no records were kept, or in some cases where such records were
contradicted.
The certificate of stock itself once issued is a continuing affirmation or representation that the
stock described therein is valid and genuine and is at least prima facie evidence that it was legally
issued in the absence of evidence to the contrary. However, this presumption may be rebutted. However,
the books and records of a corporation are not conclusive even against the corporation but are prima
facie evidence only. The effect of entries in the books of the corporation which purport to be regular
records of the proceedings of its board of directors or stockholders can be destroyed by testimony of a
more conclusive character than mere suspicion that there was an irregularity in the manner in which
the books were kept.

SPECIAL SERVICES CORPORATION


vs.
CENTRO LA PAZ (SAMAHANG ESPIRITISTA SA LUNDUYANG LA PAZ), A CHAPTER OF UNION
ESPIRITISTA CRISTIANA DE FILIPINAS, INC.
G.R. No. L-44100. April 28, 1983

FACTS:

On October 10, 1972, judgment was rendered in favor of petitioner against one Alejandro
Estudillo in the amount of P94,727.52, in an action for Replevin with Sum of Money and a writ of
execution was thereafter issued but which has remained unsatisfied.
By virtue of an alias writ of execution issued on December 15, 1972, the Sheriff of Manila caused
the annotation of a notice of levy on Transfer Certificate of Title No. 51837, in respect of the rights,
interest and participation of said Alejandro Estudillo, one of the registered owners indicated in said title.
On July 23, 1973, "Centro La Paz (Samahang Espiritista sa Lunduyang La Paz) a Chapter of
Union Espiritista Cristiana de Filipinas, Inc.," as plaintiff, instituted for Damages and Preliminary
Injunction against herein petitioner and the Sheriff of Manila with the Court of First Instance, Branch
IV, Manila, the same Court which rendered judgment in the replevin case. CENTRO reiterated ownership
of the properties in question and emphasized that the registered owners thereof had publicly
acknowledged their possession of said properties in the concept of trustees.

ISSUE:

Whether or not Centro La Paz which is merely a Chapter of Union Espiritista de Filipinas, Inc.
has a juridical personality of its own in accordance with the provisions of our laws.

RULING:

185 | P a g e
Law 321_Corporation LAW_ Case Digest

YES.

Although it was CENTRO that was actively prosecuting the case, in substance, it was
representing the mother organization, the Union Espiritista Cristiana de Filipinas, Inc., which is the real
party in interest and is itself named in the Complaint. It is an organization that is duly registered with
the Securities and Exchange Commission, and thus possessed of a juridical personality to sue and be
sued.
Admittedly, the trust was not registered in accordance with section 65 of Act 496 (the former
Land Registration Law). The absence of said registration, however, cannot be taken against CENTRO
inasmuch as, if the public auction sale had actually been held, with petitioner as the successful buyer,
petitioner could not have been considered a purchaser for value and in good faith at said sale since it
had knowledge of CENTRO's claim, particularly when the latter had filed a third-party-claim with the
Sheriff of Manila before the scheduled auction sale, which knowledge was equivalent to registration of
the several "Acknowledgments" in the Registry of Deeds.
The conclusion follows that inasmuch as Estudillo has no interest in the properties in question,
there is nothing that petitioner can levy upon. The power of a Court in the execution of its judgment
extends only over properties unquestionably belonging to the judgment debtor.
R. TRANSPORT CORPORATION
vs.
HON. COURT OF APPEALS, Former 15th Division, Manila, HON. SALVADOR S. ABAD SANTOS, as
Presiding Judge, Regional Trial Court of, Metro Manila, Branch 65 and FLOSERIDA L.
CASTAÑEDA
G.R. No. 111187. February 1, 1995

FACTS:

On November 22, 1991, private respondent filed a complaint for damages arising from breach of
contract of carriage against petitioner. In an Order dated January 28, 1991, the trial court upon ex
parte motion of private respondent, declared petitioner in default and appointed a commissioner to
receive evidence ex parte. Petitioner filed a Motion to Dismiss and to Stop Ex Parte Reception of
Evidence. It asserted that it was not properly served with summons and consequently, the trial court did
not acquire jurisdiction over its person. It argued that none of the officers enumerated in Section 13,
Rule 14 of the Revised Rules of Court (namely, the corporation's president, manager, secretary, cashier,
agent or any of its directors) received any summons. The trial court denied petitioner's motion and
allowed private respondent to adduce its evidence ex parte.
The Court of Appeals dismissed the petitioner’s petition for certiorari ruling that the trial court
did not commit any grave abuse of discretion in declaring the petitioner in default and in denying
petitioner's motion for reconsideration.

ISSUE:

Whether or not there was valid service of summons.

RULING:

YES.

As a general rule, service of summons must be made on the persons named in Section 13, Rule
14 of the Revised Rules of Court which provides: If the defendant is a corporation organized under the
laws of the Philippines or a partnership duly registered, service may be made on the president, manager,
secretary, cashier, agent or any of its directors.
Thus service on persons other than those mentioned in said Rule has been held as improper.
Through the years, the rule on service of summons has been liberalized. Such liberalization is to give life
to the rationale behind Section 13 of Rule 14. Service of summons on persons other than those
enumerated in Section 13 of Rule 14 have been held proper on the theory that those persons served
were holding positions of responsibility and could appreciate the importance of the papers handed them,
and could be expected to deliver the papers to the proper officer. These individuals were considered
"agents" within the contemplation of Section 13 of Rule 14. Thus, the Court holds that service of
summons on petitioner's Operations Manager was valid. He is an officer who may be relied upon to
appreciate the importance of the papers served on him.
The fact that service was made at petitioner's bus terminal at the address stated in the summons
and not at its office in Makati does not render the service of summons invalid. Petitioner is engaged in

186 | P a g e
Law 321_Corporation LAW_ Case Digest

the transportation business, operating over 100 buses. Its central bus terminal is located at Sucat,
Parañaque, from where it conducts the bulk of its business. It was at that terminal where petitioner's
Operations Manager was found and upon whom service was made.

 Power to Acquire, Dispose, Encumber Property

THE DIRECTOR OF LANDS


vs.
THE HONORABLE COURT OF APPEALS and IGLESIA NI CRISTO
G.R. No. L-56613. March 14, 1988

FACTS:

On November 28, 1973, private respondent Iglesia ni Cristo filed an application for registration in
its name of a parcel of land with an area of 379 square meters located at Poblacion, Municipality of
Amadeo, Cavite. In its application, private respondent alleged inter alia that it was the owner in fee
simple of the land afore-described, having acquired title thereto by virtue of a Deed of Absolute Sale
executed in 1947 by Aquelina de la Cruz in its favor and that applicant. Private respondent prayed that
should the Land Registration Act not be applicable, the provisions of Chapter VIII of Commonwealth Act
No. 141, as amended by Republic Act No. 6236 be applied as applicant and its predecessors-in-interest
had been in possession of the land for more than thirty [30] years and had introduced improvements
thereon.
The Republic of the Philippines, represented by the Director of Lands, opposed the application on
the following grounds: 1] the applicant and its predecessors-in-interest did not possess sufficient title to
acquire ownership in fee simple of the parcel of land applied for; 2] neither the applicant nor its
predecessors-in-interest have been in open, continuous, exclusive and notorious possession and
occupation of the land in question; and, 3] the subject parcel of land is a portion of the public domain
not subject to private appropriation.

ISSUES:

Whether or not the respondent is prohibited from acquiring private land as provided under the
Constitution.

RULING:

YES.

Taking the year 1936 as the reckoning point, the 30-year period of open, continuous, exclusive
and notorious possession and occupation required by law was completed in 1966. The completion by
private respondent of this statutory 30-year period has dual significance in the light of Section 48[b] of
Commonwealth Act No. 141, as amended and prevailing jurisprudence: [1] at this point, the land in
question ceased by operation of law to be part of the public domain; and [2] private respondent could
have its title thereto confirmed through the appropriate proceedings as under the Constitution then in
force, private corporations or associations were not prohibited from acquiring public lands, but merely
prohibited from acquiring, holding or leasing such type of land in excess of 1,024 hectares.
If in 1966, the land in question was converted ipso jure into private land, it remained so in 1974
when the registration proceedings were commenced. This being the case, the prohibition under the 1973
Constitution would have no application. Otherwise construed, if in 1966, private respondent could have
its title to the land confirmed, then it had acquired a vested right thereto, which the 1973 Constitution
can neither impair nor defeat.

187 | P a g e
Law 321_Corporation LAW_ Case Digest

 Power to Make Donations

MARIA CLARA PIROVANO ET AL.


vs.
THE DE LA RAMA STEAMSHIP CO.
G.R. No. L-5377. December 29, 1954

FACTS:

Plaintiffs herein are the minor children of the late Enrico Pirovano represented by their mother
and judicial guardian Estefania R. Pirovano. They seek to enforce certain resolutions adopted by the
Board of Directors and stockholders of the defendant company giving to said minor children of the
proceeds of the insurance policies taken on the life of their deceased father Enrico Pirovano with the
company as beneficiary. Defendant's main defense is: that said resolutions and the contract executed
pursuant thereto are ultra vires, and, if valid, the obligation to pay the amount given is not yet due and
demandable.
Plaintiff-appellant Pirovano is the owner of 3424 shares of stocks in defendant-appellee
Corporation which declared a dividend of P100 per share. Appellant wants to recover from appellee the
sum of P221, 975 after deducting the sum of P120, 424 which she had withdrawn or received from
appellee for advances she received after the death of her father, the late Esteban de la Rama.
Appellant’s theory is that the cash advances to her for her personal use and that of her children
were assumed by Esteban de la Rama. She claims that the advances made to her by appellees were
debited from the account of Hijos de I. de la Rama, another corporation practically owned by Esteban de
la Rama. She further claims that the appellee can only deduct from the amount of dividend she is
entitled to, the amount of cash advances which was not assumed by her father. The withdrawals by the
appellant were made during the period 1940 to 1949 during which the appellee made a deed of trust
with Hijos. The deed of trust was made to circumvent the prohibition of declaring dividends during the
period.

ISSUE:

Whether or not the donation made by the corporation of the proceeds of the insurance is a valid
act.

RULING:

YES.

The Articles of Incorporation of Dela Rama Steamship provided that under (g) the company may
invest and deal with moneys of the company not immediately required, in such a manner as from time
to time may be determined, and under (i)… to lend money or to aid in any other manner any person
association, or corporation of which any obligation or in which any interest is held by the corporation or
in the affairs of prosperity of which the corporation has a lawful interest.
The corporation was thus given broad and almost unlimited powers to carry out the purposes for
which it was organized. The word “deal” is broad enough to include any manner of disposition, and thus
the donation comes within the scope of this broad power. The company was in fact very much solvent as
it was able to declare and issue dividends to its stockholders, and shows that the excess funds which
were not needed by the company which was donated to the children was justified under the AOI. Under
the second broad power, the corporation knew well its scope such that none lifted a finger to dispute its
validity. The company gave the donation not only because it was indebted to him but also because it
was fit and proper to make provisions for the children and out of a sense of gratitude.

 To Increase or Decrease Capital Stock

MADRIGAL & COMPANY, INC.


vs.

188 | P a g e
Law 321_Corporation LAW_ Case Digest

HON. RONALDO B. ZAMORA, PRESIDENTIAL ASSISTANT FOR LEGAL AFFAIRS, THE HON.
SECRETARY OF LABOR, and MADRIGAL CENTRAL OFFICE EMPLOYEES UNION
G.R. No. L-48237. June 30, 1987

MADRIGAL & COMPANY, INC.


vs.
HON. MINISTER OF LABOR and MADRIGAL CENTRAL OFFICE EMPLOYEES UNION
No. L-49023. June 30, 1987

FACTS:

The petitioner was engaged, among several other corporate objectives, in the management of Rizal
Cement Co., Inc.Admittedly, the petitioner and Rizal Cement Co., Inc. are sister companies.Both are
owned by the same or practically the same stockholders.On December 28, 1973, the respondent, the
Madrigal Central Office Employees Union, sought for the renewal of its collective bargaining agreement
with the petitioner, which was due to expire on February 28, 1974.Specifically, it proposed a wage
increase of P200.00 a month, an allowance of P100.00 a month, and other economic benefits.The
petitioner, however, requested for a deferment in the negotiations.
On July 29, 1974, by an alleged resolution of its stockholders, the petitioner reduced its capital
stock from 765,000 shares to 267,366 shares.This was effected through the distribution of the
marketable securities owned by the petitioner to its stockholders in exchange for their shares in an
equivalent amount in the corporation.On August 22, 1975, by yet another alleged stockholders' action,
the petitioner reduced its authorized capitalization from 267,366 shares to 110,085 shares, again,
through the same scheme.

ISSUE:

Whether or not the decrease in the ACS of petitioner is valid.

RULING:

NO.

The Court ruled that what clearly emerges from the recorded facts is that the petitioner, awash
with profits from its business operations but confronted with the demand of the union for wage
increases, decided to evade its responsibility towards the employees by a devised capital reduction.
While the reduction in capital stock created an apparent need for retrenchment, it was, by all
indications, just a mask for the purge of union members, who, by then, had agitated for wage increases.
As such shareholder, the dividends paid to it were its own money, which may then be available
for wage increments. It is not a case of a corporation distributing dividends in favor of its stockholders,
in which case, such dividends would be the absolute property of the stockholders and hence, out of
reach by creditors of the corporation. Here, the petitioner was acting as stockholder itself, and in that
case, the right to a share in such dividends, by way of salary increases, may not be denied its
employees.
Accordingly, the Court is convinced that the petitioner's capital reduction efforts were, to begin
with, a subterfuge, a deception as it were, to camouflage the fact that it had been making profits, and
consequently, to justify the mass layoff in its employee ranks, especially of union members.

PHILIPPINE TRUST COMPANY, as assignee in insolvency of "La Cooperativa Naval Filipina"


vs.
MARCIANO RIVERA
G.R. No. L-19761. January 29, 1923

FACTS:

In 1918 the Cooperativa Naval Filipina was duly incorporated under the laws of the Philippine
Islands, with a capital of P100,000, divided into one thousand shares of a par value of P100 each.
Among the incorporators of this company was numbered the defendant Mariano Rivera, who subscribed
for 450 shares representing a value of P45,000, the remainder of the stock being taken by other
persons.
In the course of time the company became insolvent and went into the hands of the Philippine
Trust Company, as assignee in bankruptcy; and by it this action was instituted to recover one-half of
the stock subscription of the defendant, which admittedly has never been paid.

189 | P a g e
Law 321_Corporation LAW_ Case Digest

The reason given for the failure of the defendant to pay the entire subscription is a meeting of its
stockholders occurred, at which a resolution was adopted to the effect that the capital should be
reduced by 50 per centum and the subscribers released from the obligation to pay any unpaid balance
of their subscription in excess of 50 per centum of the same.

ISSUE:

Whether or not the reduction of the company’s capital by 50 per centum and the subscribers
released from the obligation to pay any unpaid balance of their subscription in excess of 50 per centum
is valid.

RULING:

NO.

The Court ruled that defendant was still liable for the unpaid balance of his subscription. It is
established doctrine that subscription to the capital of a corporation constitute a find to which creditors
have a right to look for satisfaction of their claims and that the assignee in insolvency can maintain an
action upon any unpaid stock subscription in order to realize assets for the payment of its debts.
A corporation has no power to release an original subscriber to its capital stock from the
obligation of paying for his shares, without a valuable consideration for such release; and as against
creditors a reduction of the capital stock can take place only in the manner an under the conditions
prescribed by the statute or the charter or the articles of incorporation. Moreover, strict compliance with
the statutory regulations is necessary. In the case before us the resolution releasing the shareholders
from their obligation to pay 50 per centum of their respective subscriptions was an attempted
withdrawal of so much capital from the fund upon which the company's creditors were entitled
ultimately to rely and, having been effected without compliance with the statutory requirements, was
wholly ineffectual.

 To Deny Pre-Emptive Rights

DATU TAGORANAO BENITO


vs.
SECURITIES AND EXCHANGE COMMISSION and JAMIATUL PHILIPPINE-AL ISLAMIA, INC.
G.R. No. L-56655. July 25, 1983

FACTS:

On February 6, 1959, the Articles of Incorporation of respondent Jamiatul Philippine-Al Islamia,


Inc. (originally Kamilol Islam Institute, Inc.) were filed with the Securities and Exchange Commission
(SEC) and were approved on December 14, 1962. The corporation had an authorized capital stock of
P200,000.00 divided into 20,000 shares at a par value of P10.00 each. Of the authorized capital stock,
8,058 shares worth P80,580.00 were subscribed and fully paid for. Petitioner Datu Tagoranao Benito
subscribed to 460 shares worth P4,600.00.
On October 28, 1975, the respondent corporation filed a certificate of increase of its capital stock
from P200,000.00 to P1,000,000.00. Thus, P110,980.00 worth of shares were subsequently issued by
the corporation from the unissued portion of the authorized capital stock of P200,000.00. Of the
increased capital stock of P1,000,000.00, P160,000.00 worth of shares were subscribed by Mrs. Fatima
A. Ramos, Mrs. Tarhata A. Lucman and Mrs. Moki-in Alonto.
Petitioner Datu Tagoranao filed a petition alleging that the additional issue (worth P110,980.00)
of previously subscribed shares of the corporation was made in violation of his pre-emptive right to said

190 | P a g e
Law 321_Corporation LAW_ Case Digest

additional issue and that the increase in the authorized capital stock of the corporation from
P200,000.00 to P1,000,000.00 was illegal considering that the stockholders of record were not notified
of the meeting wherein the proposed increase was in the agenda.
Respondents denied the material allegations of the petition and claimed that petitioner has no
cause of action and that the stock certificates covering the shares alleged to have been sold to petitioner
were only given to him as collateral for the loan of Domocao Alonto and Moki-in Alonto. The SEC
affirmed the sale.

ISSUE:

Whether or not the issuance of the unissued shares was subject to the pre-emptive right of the
stockholders.

RULING:

NO.

The Court held that the questioned issuance of the unsubscribed portion of the capital stock
worth P110,980.00 is not invalid even if assuming that it was made without notice to the stockholders
as claimed by petitioner. The power to issue shares of stocks in a corporation is lodged in the board of
directors and no stockholders' meeting is necessary to consider it because additional issuance of shares
of stocks does not need approval of the stockholders.
Petitioner bewails the fact that in view of the lack of notice to him of such subsequent issuance,
he was not able to exercise his right of pre-emption over the unissued shares. However, the general rule
is that pre-emptive right is recognized only with respect to new issue of shares, and not with respect to
additional issues of originally authorized shares. This is on the theory that when a corporation at its
inception offers its first shares, it is presumed to have offered all of those which it is authorized to issue.
An original subscriber is deemed to have taken his shares knowing that they form a definite
proportionate part of the whole number of authorized shares. When the shares left unsubscribed are
later re-offered, he cannot therefore claim a dilution of interest.

PEDRO LOPEZ DEE


vs.
SECURITIES AND EXCHANGE COMMISSION, HEARING OFFICER EMMANUEL SISON, NAGA
TELEPHONE CO., INC., COMMUNICATION SERVICES, INC., LUCIANO MAGGAY, AUGUSTO
FEDERIS, NILDA RAMOS, FELIPA JAVALERA, DESIDERIO SAAVEDRA
G.R. No. L-60502. July 16, 1991

JUSTINO DE JESUS, SR., PEDRO LOPEZ DEE, JULIO LOPEZ DEE, and VICENTE TORDILLA, JR.
vs.
INTERMEDIATE APPELLATE COURT, LUCIANO MAGGAY, NILDA I. RAMOS, DESIDERIO
SAAVEDRA, AUGUSTO FEDERIS, ERNESTO MIGUEL, COMMUNICATION SERVICES, INC., and
NAGA TELEPHONE COMPANY, INC.
G.R. No. L-63922. July 16, 1991

FACTS:

Naga Telephone Company, Inc. was organized in 1954, the authorized capital was P100,000.00.
In 1974 Naga Telephone Co., Inc. (Natelco) decided to increase its authorized capital to P3,000,000.00.
As required by the Public Service Act, Natelco filed an application for the approval of the increased
authorized capital with the then Board of Communications on which a decision was rendered approving
the application subject to certain conditions, among which was: That the issuance of the shares of
stocks will be for a period of one year from the date hereof, "after which no further issues will be made
without previous authority from this Board."
Natelco filed its Amended Articles of Incorporation with the Securities and Exchange Commission.
When the amended articles were filed with the SEC, the original authorized capital of P100,000.00 was
already paid. Of the increased capital of P2,900,000.00 the subscribers subscribed to P580,000.00 of
which P145,000 was fully paid.

ISSUE:

191 | P a g e
Law 321_Corporation LAW_ Case Digest

Whether or not Natelco stockholders have a right of preemption to the 113,800 shares in
question.

RULING:

NO.

The general rule is that pre-emptive right is recognized only with respect to new issues of shares,
and not with respect to additional issues of originally authorized shares. This is on the theory that when
a corporation at its inception offers its first shares, it is presumed to have offered all of those which it is
authorized to issue. An original subscriber is deemed to have taken his shares knowing that they form a
definite proportionate part of the whole number of authorized shares. When the shares left
unsubscribed are later re-offered, he cannot therefore claim a dilution of interest.
Thus, the questioned issuance of the 113,800 stocks is not invalid even assuming that it was made
without notice to the stockholders as claimed by the petitioner. The power to issue shares of stocks in a
corporation is lodged in the board of directors and no stockholders meeting is required to consider it
because additional issuance of shares of stocks does not need approval of the stockholders.
Consequently, no pre-emptive right of Natelco stockholders was violated by the issuance of the 113,800
shares to CSI.
Accordingly, it is clear that since the trial judge in the lower court did not have jurisdiction in
issuing the questioned restraining order, disobedience thereto did not constitute contempt, as it is
necessary that the order be a valid and legal one. It is an established rule that the court has no
authority to punish for disobedience of an order issued without authority.
PRESIDENTIAL COMMISSION ON GOOD GOVERNMENT
vs.
HON. BENJAMIN M. AQUINO, JR., as Presiding Judge, Regional Trial Court, NCJR Branch LXXII
Malabon, Metro Manila, and MARCELO FIBERGLASS CORPORATION
G.R. No. 77816. June 30, 1988

MARCELO FIBERGLASS CORPORATION


vs.
PRESIDENTIAL COMMISSION ON GOOD GOVERNMENT
G.R. No. 78753. June 30, 1988

FACTS:

On June 18, 1982, Edward T. Marcelo as president of private respondent Marcelo Fiberglass
Corporation [hereinafter referred to as MFC] entered into a Contract to Buy and Sell with the Philippine
Navy represented by Rear Admiral Simeon M. Alejandro, then Flag Officer in Command, for the
construction and delivery by the former of fifty-five 551 units of fiberglass high-speed patrol boats at
P7,200,000 each plus spare parts amounting to P29,700,000 for a total contract price of P425,700,000.
It was stipulated in the contract that the patrol boats would be delivered within thirty-six [36] months
from the date the Philippine Navy pays to private respondent the stipulated down payment of thirty per
cent [30%] of the contract price.
To facilitate funding of the initial down payment agreed upon under the contract, MFC through
Edward Marcelo, secured presidential approval for the issuance of a guarantee 'by the national
government in acquiring either a foreign currency loan in behalf of the Philippine Navy with a foreign
bank or offshore banking unit, or a peso term loan to be negotiated by the Philippine National Bank,
also, in behalf of the Philippine Navy.

ISSUES:

Whether or not the issuance and implementation of the writ of sequestration violates the
constitutional rights of private respondent against impairment of obligation of contracts and deprivation
of property without due process of law.

RULING:

NO.

The Court sustains petitioner's stand and holds that regional trial courts and the Court of
Appeals for that matter have no jurisdiction over the Presidential Commission on Good Government in
the exercise of its powers under the applicable Executive Orders and Article XVIII, Section 26 of the

192 | P a g e
Law 321_Corporation LAW_ Case Digest

Constitution and therefore may not interfere with and restrain or set aside the orders and actions of tile
Commission.
Under Section 2 of the President's Executive Order No. 14 issued on May 7, 1986, all cases of the
Commission regarding "the Funds, Moneys, Assets and Properties Illegally Acquired or Misappropriated
by Former President Ferdinand Marcos, Mrs. Imelda Romualdez Marcos, then, Close Relatives,
Subordinates, Business Associate, Dummies, Agents or Nominees' whether civil or criminal.
The attempt to remove special civil actions from the Sandiganbayan's exclusive jurisdiction is of
no avail if they similarly involve the powers and functions of the Presidential Commission on Good
Government.
The matters involved in these cases are orders of the PCGG issued in the exercise of its powers
and functions for they involve the sequestration of the assets of private respondent Marcelo Fiberglass
Corporation and Edward T. Marcelo, its president. The propriety of said sequestration and any incident
arising from, incidental or related to such sequestration is within the exclusive jurisdiction of the
Sandiganbayan.

REPUBLIC OF THE PHILIPPINES


vs.
SANDIGANBAYAN (3RD DIVISION), JOSE L. AFRICA, UNIMOLCO, ROBERTO BENEDICTO, ANDRES
AFRICA and SMART COMMUNICATIONS
G.R. No. 128606. December 4, 2000

FACTS:

Eastern Telecommunications Philippines, Inc. (ETPI) was one of the corporations sequestered by
the Presidential Commission on Good Government (PCGG). Among its stockholders were Roberto S.
Benedicto and Universal Molasses Corporation (UNIMOLCO).
Sometime in 1990, PCGG and Benedicto entered into a compromise agreement whereby
Benedicto ceded to the government 204,000 shares of stock in ETPI, representing his fifty-one percent
(51%) equity therein. The other forty-nine percent (49%), consisting of 196,000 shares of stock, were
released from sequestration and adjudicated by final judgment to Benedicto and UNIMOLCO.
Furthermore, the government agreed to withdraw the cases filed against Benedicto and free him from
further criminal prosecution.
Meanwhile, on motion of petitioner, through the PCGG, the Sandiganbayan issued a Resolution,
dated May 7, 1996, authorizing the entry in the Stock and Transfer Book of ETPI of the transfer of
ownership of 204,000 shares of stock to petitioner, to be taken out of the shareholdings of UNIMOLCO.
PCGG issued Resolution No. 96-142 enjoining all stockholders of ETPI from selling shares of
stock therein without the written conformity of the PCGG. Subsequently, on July 24, 1996, UNIMOLCO
and Smart Communications executed a Deed of Absolute Sale whereby UNIMOLCO sold its 196,000
shares of stock in ETPI to Smart. Prior to the sale, Smart was not a stockholder of ETPI.

ISSUE:

Whether or not petitioner was denied of his pre-emptive right because of the defective notice.

RULING:

NO.

The records of the case clearly show that the written notice by UNIMOLCO, the Offeror, of its
intention to sell its 196,000 shares of stock was duly received on April 24, 1996 by the President and
Chairman of the Board of ETPI.
Moreover, the purpose of the notice requirement in Article 10 of the ETPI Articles of Incorporation
is to give the stockholders knowledge of the intended sale of shares of stock of the corporation, in order
that they may exercise their preemptive right. Where it is shown that a stockholder had actual
knowledge of the intended sale within the period prescribed to exercise the right, the notice requirement
had been sufficiently met.
In the case at bar, PCGG had actual knowledge of UNIMOLCO’s offer to sell its shares of stock. In
fact, it issued Resolution No. 96-142 enjoining the sale of the said shares of stock to Smart. Petitioner,
thus, cannot feign lack of notice. PCGG had no more authority to enjoin the sale of UNIMOLCO’s
196,000 shares of stock, as it endeavored to do in Resolution No. 96-142. As correctly found by the
Sandiganbayan, since the 196,000 shares of stock had already been adjudicated by final judgment to
Benedicto and UNIMOLCO, PCGG could no longer exercise power and authority over the same.

193 | P a g e
Law 321_Corporation LAW_ Case Digest

 To Sell Or Otherwise Dispose Of All or Substantially All Of Corporate Assets

PHILIPPINE NATIONAL BANK & NATIONAL SUGAR DEVELOPMENT CORPORATION


vs.
ANDRADA ELECTRIC & ENGINEERING COMPANY
G.R. No. 142936. April 17, 2002

FACTS:

The Plaintiff (herein respondent) alleged that it is a partnership duly organized, existing, and
operating under the laws of the Philippines, while the herein petitioner Philippine National Bank (PNB),
is a semi-government corporation duly organized, existing and operating under the laws of the
Philippines; whereas, the other defendant, the National Sugar Development Corporation (NASUDECO),
is also a semi-government corporation and the sugar arm of the PNB; and the defendant Pampanga
Sugar Mills (PASUMIL), is a corporation organized, existing and operating under the 1975 laws of the
Philippines.
The respondent is engaged in the business of general construction for the repairs and/or
construction of different kinds of machineries and buildings. On August 26, 1975, the defendant PNB
acquired the assets of the defendant PASUMIL that were earlier foreclosed by the Development Bank of
the Philippines (DBP) under LOI No. 311. But prior to October 29, 1971, the defendant PASUMIL
engaged the services of plaintiff for electrical rewinding and repair, most of which were partially paid by
the defendant PASUMIL, leaving several unpaid accounts with the plaintiff; that finally, on October 29,
1971, the plaintiff and the defendant PASUMIL entered into a contract.
Out of the total obligation of P777,263.80, the defendant PASUMIL had paid only P264,000.00,
leaving an unpaid balance of P513,263.80. Petitioners herein failed and refused to pay the plaintiff their
just, valid and demandable obligation; that the President of the NASUDECO is also the Vice-President of
the PNB, inasmuch as PNB and NASUDECO now owned and possessed the assets of the defendant
PASUMIL. Accordingly, the plaintiff prayed that judgment be rendered against the defendants PNB,
NASUDECO, and PASUMIL, jointly and severally.

ISSUE:

Whether or not petitioners should be held liable for the corporate debts of PASUMIL for taking
over of the latter’s foreclosed assets.

RULING:

NO.

As a rule, a corporation that purchases the assets of another will not be liable for the debts of the
selling corporation, provided the former acted in good faith and paid adequate consideration for such
assets, except when any of the following circumstances is present: (1) where the purchaser expressly or
impliedly agrees to assume the debts, (2) where the transaction amounts to a consolidation or merger of
the corporations, (3) where the purchasing corporation is merely a continuation of the selling
corporation, and (4) where the transaction is fraudulently entered into in order to escape liability for
those debts.
Equally well-settled is the principle that the corporate mask may be removed or the corporate veil
pierced when the corporation is just an alter ego of a person or of another corporation. For reasons of
public policy and in the interest of justice, the corporate veil will justifiably be impaled only when it
becomes a shield for fraud, illegality or inequity committed against third persons.
ISLAMIC DIRECTORATE OF THE PHILIPPINES, MANUEL F. PEREA and SECURITIES & EXCHANGE
COMMISSION
vs.
COURT OF APPEALS and IGLESIA NI CRISTO
G.R. No. 117897. May 14, 1997

FACTS:

194 | P a g e
Law 321_Corporation LAW_ Case Digest

Petitioner IDP-Tamano Group alleges that sometime in 1971, Islamic leaders of all Muslim major
tribal groups in the Philippines headed by Dean Cesar Adib Majul organized and incorporated the
ISLAMIC DIRECTORATE OF THE PHILIPPINES (IDP), the primary purpose of which is to establish an
Islamic Center in Quezon City for the construction of a "Mosque (prayer place), Madrasah (Arabic
School), and other religious infrastructures" so as to facilitate the effective practice of Islamic faith in the
area.
The Libyan government donated money to the IDP to purchase land at Culiat, Tandang Sora,
Quezon City, to be used as a Center for the Islamic populace. The land, with an area of 49,652 square
meters, was covered by two titles: Transfer Certificate of Title Nos. RT-26520 (176616) and RT-26521
(170567), both registered in the name of IDP.
According to the petitioner, in 1972, after the purchase of the land by the Libyan government in
the name of IDP, Martial Law was declared by the late President Ferdinand Marcos. Thereafter, two
Muslim groups sprung, the Carpizo Group and the Abbas Group, both groups claimed to be the
legitimate IDP. Significantly, on October 3, 1986, the SEC, in a suit between these two contending
groups, came out with a Decision in SEC Case No. 2687 declaring the election of both the Carpizo
Group and the Abbas Group as IDP board members to be null and void.

ISSUE:

Whether or not the Deed of Sale executed by Carpizo Group is valid.

RULING:

NO.

This is precisely what the SEC did in SEC Case No. 4012 when it adjudged the election of the
Carpizo Group to the IDP Board of Trustees to be null and void. Consequently, the Carpizo Group is
bereft of any authority whatsoever to bind IDP in any kind of transaction including the sale or
disposition of ID property.
Nothing thus becomes more settled than that the IDP-Carpizo Group with whom private
respondent INC contracted is a fake Board. Premises considered, all acts carried out by the Carpizo
Board, particularly the sale of the Tandang Sora property, allegedly in the name of the IDP.
The Carpizo Group-INC sale is further deemed null and void ab initio because of the Carpizo
Group's failure to comply with Section 40 of the Corporation Code pertaining to the disposition of all or
substantially all assets of the corporation.
The Tandang Sora property, appears from the records, constitutes the only property of the IDP.
Hence, its sale to a third-party is a sale or disposition of all the corporate property and assets of IDP
falling squarely within the contemplation of the foregoing section. For the sale to be valid, the majority
vote of the legitimate Board of Trustees, concurred in by the vote of at least 2/3 of the bona fide
members of the corporation should have been obtained.
These twin requirements were not met as the Carpizo Group which voted to sell the Tandang Sora
property was a fake Board of Trustees, and those whose names and signatures were affixed by the
Carpizo Group together with the sham Board Resolution authorizing the negotiation for the sale were,
from all indications, not bona fide members of the IDP as they were made to appear to be. All told, the
disputed Deed of Absolute Sale executed by the fake Carpizo Board and private respondent INC was
intrinsically void ab initio.
THE EDWARD J. NELL COMPANY
vs.
PACIFIC FARMS, INC.
G.R. No. L-20850. November 29, 1965

FACTS:

On October 9, 1958, appellant secured against Insular Farms, Inc. a judgment for the sum of
P1,853.80 representing the unpaid balance of the price of a pump sold by appellant to Insular Farms
with interest on said sum. A writ of execution, issued after the judgment had become final, was, on
August 14, 1959, returned unsatisfied, stating that Insular Farms had no leviable property. Appellant
then filed with said court the present action against Pacific Farms, Inc. for the collection of the judgment
aforementioned, upon the theory that appellee is the alter ego of Insular Farms, which appellee has
denied.
In due course, the municipal court rendered judgment dismissing petitioner’s complaint.
Defendant appealed, with the same result, to the court of First Instance and, subsequently, to the Court

195 | P a g e
Law 321_Corporation LAW_ Case Digest

of Appeals. Hence this appeal by certiorari, upon the ground that the Court of Appeals had erred in not
holding the defendant liable for said unpaid obligation of the Insular Farms.

ISSUE:

Whether or not the defendant is liable for the unpaid obligation of the Insular Farms.

RULING:

NO.

Generally where one corporation sells or otherwise transfers all of its assets to another
corporation, the latter is not liable for the debts and liabilities of the transferor, except: (1) where the
purchaser expressly or impliedly agrees to assume such debts; (2) where the transaction amounts to a
consolidation or merger of the corporations; (3) where the purchasing corporation is merely a
continuation of the selling corporation; and (4) where the transaction is entered into fraudulently in
order to escape liability for such debts."
In the case at bar, there is neither proof nor allegation that defendant had expressly or impliedly
agreed to assume the debt of Insular Farms in favor of petitioner, or that the defendant is a continuation
of Insular Farms, or that the sale of either the shares of stock or the assets of Insular Farms to the
defendant had been entered into fraudulently, in order to escape liability for the debt of the Insular
Farms in favor of petitioner. Moreover, defendant purchased the shares of stock of Insular Farms as the
highest bidder at an auction sale held at the instance of a bank to which said shares had been pledged
as security for an obligation of Insular Farms in favor of said bank. It has also been established that the
defendant had paid P285,126.99 for said shares of stock, apart from the sum of P10,000.00 it, likewise,
paid for other assets of Insular Farms.

JULIETA V. ESGUERRA
vs.
COURT OF APPEALS and SURESTE PROPERTIES, INC.
G.R. No. 119310. February 3, 1997

FACTS:

Julieta Esguerra filed a complaint for administration of conjugal partnership or separation of


property against her husband Vicente Esguerra, Jr. and V. Esguerra Construction Co., Inc. (VECCI) and
other family corporations as defendants before the trial court.
The parties entered into a compromise agreement. By virtue of said agreement, Esguerra Bldg. I
was sold and the net proceeds distributed according to the agreement. The controversy arose with
respect to Esguerra Building II. Herein petitioner started claiming one-half of the rentals of the said
building which VECCI refused. Thus, petitioner filed a motion with respondent court praying that VECCI
be ordered to remit one-half of the rentals to her. The trial court ruled in favor of petitioner.
Meanwhile, Esguerra Bldg. II was sold to private respondent Sureste Properties. Inc. for
P150,000,000.00 prompting Julieta V. Esguerra to file a motion seeking the nullification of the sale on
the ground that VECCI is not the lawful and absolute owner thereof and that she has not been notified
nor consulted as to the terms and conditions of the sale. The trial court ruled that the sale to Sureste
was valid.

ISSUES:

Whether or not the sale of Esguerra Building II is a valid exercise of corporate power.

RULING:

YES.

196 | P a g e
Law 321_Corporation LAW_ Case Digest

VECCI's sale of all the properties mentioned in the judicially-approved compromise agreement
was done on the basis of its Corporate Secretary's Certification of these two resolutions. The partial
decision did not require any further board or stockholder resolutions to make VECCI's sale of these
properties valid. Being regular on its face, the Secretary's Certification was sufficient for private
respondent Sureste Properties, Inc. to rely on. It did not have to investigate the truth of the facts
contained in such certification. Otherwise, business transactions of corporations would become
tortuously slow and unnecessarily hampered. Ineluctably, VECCI's sale of Esguerra Building II to
private respondent was not ultra vires but a valid execution of the trial court's partial decision.
Based on the foregoing, the sale is also deemed to have satisfied the requirements of Section 40 of
the Corporation Code.

LOPEZ REALTY, INC., AND ASUNCION LOPEZ GONZALES


vs.
FLORENTINA FONTECHA, ET AL., AND THE NATIONAL LABOR RELATIONS COMMISSION
G.R. No. 76801. August 11, 1995

FACTS:

Lopez Realty, Inc., is a corporation engaged in real estate business, while petitioner Asuncion
Lopez Gonzales is one of its majority shareholders. Sometime in 1978, Arturo Lopez submitted a
proposal relative to the distribution of certain assets of Petitioner Corporation among its three (3) main
shareholders. The proposal had three (3) aspects, viz: (1) the sale of assets of the company to pay for its
obligations; (2) the transfer of certain assets of the company to its three (3) main shareholders, while
some other assets shall remain with the company; and (3) the reduction of employees with provision for
their gratuity pay. The proposal was deliberated upon and approved in a special meeting of the board of
directors held on April 17, 1978.
It appears that petitioner corporation approved two (2) resolutions providing for the gratuity pay
of its employees, viz: (a) Resolution No. 6, Series of 1980 resolving to set aside, twice a year, a certain
sum of money for the gratuity pay of its retiring employees and to create a Gratuity Fund for the said
contingency; and (b) Resolution No. 10, Series of 1980, setting aside the amount of P157,750.00 as
Gratuity Fund covering the period from 1950 up to 1980.
On August 17, 1981, the remaining members of the Board of Directors, namely: Rosendo de Leon,
Benjamin Bernardino, and Leo Rivera, convened a special meeting and passed a resolution which
provides that: (a) Those who will be laid off be given the full amount of gratuity; (b) Those who will be
retained will receive 25% of their gratuity (pay) due on September 1, 1981, and another 25% on January 1,
1982, and 50% to be retained by the office in the meantime.
Private respondents were the retained employees of petitioner corporation. In a letter, dated
August 31, 1981, private respondents requested for the full payment of their gratuity pay. Their request
was granted in a special meeting held on September 1, 1981.

ISSUES:

Whether or not the subject resolutions requires for their validity stockholders’ approval.

RULING:

YES.

The Court is not persuaded that the subject resolutions had no force and effect in view of the
non-approval thereof during the Annual Stockholders' Meeting held on March 1, 1982. To strengthen
their position, petitioners cite section 28 1/2 of the Corporation Law (Section 40 of the Corporation
Code).
The cited provision is not applicable to the case at bench as it refers to the sale, lease, exchange
or disposition of all or substantially all of the corporation's assets, including its goodwill. In such a case,
the action taken by the board of directors requires the authorization of the stockholders on record.
It will be observed that, except for Arturo Lopez, the stockholders of petitioner corporation also sit
as members of the board of directors. Under the circumstances in field, it will be illogical and
superfluous to require the stockholders' approval of the subject resolutions. Thus, even without the
stockholders' approval of the subject resolutions, petitioners are still liable to pay private respondents'
gratuity pay.
Petition is dismissed.

197 | P a g e
Law 321_Corporation LAW_ Case Digest

 To Invest Corporate Funds In Another Corporation or Business

JOHN GOKONGWEI, JR.


vs.
SECURITIES AND EXCHANGE COMMISSION, ANDRES M. SORIANO, JOSE M. SORIANO, ENRIQUE
ZOBEL, ANTONIO ROXAS, EMETERIO BUNAO, WALTHRODE B. CONDE, MIGUEL ORTIGAS,
ANTONIO PRIETO, SAN MIGUEL CORPORATION, EMIGDIO TANJUATCO, SR., and EDUARDO R.
VISAYA
G.R. No. L-45911. April 11, 1979

FACTS:

On October 22, 1976, petitioner, as stockholder of respondent San Miguel Corporation, filed with
the Securities and Exchange Commission (SEC) a petition for "declaration of nullity of amended by-laws,
cancellation of certificate of filing of amended by- laws, injunction and damages with prayer for a
preliminary injunction" against the majority of the members of the Board of Directors and San Miguel
Corporation as an unwilling petitioner.
Petitioner alleged that on September 18, 1976, individual respondents amended by bylaws of the
corporation, basing their authority to do so on a resolution of the stockholders adopted on March 13,
1961, when the outstanding capital stock of respondent corporation was only P70,139.740.00, divided
into 5,513,974 common shares at P10.00 per share and 150,000 preferred shares at P100.00 per share.
At the time of the amendment, the outstanding and paid up shares totalled 30,127,047 with a total par
value of P301,270,430.00. It was contended that according to section 22 of the Corporation Law and
Article VIII of the by-laws of the corporation, the power to amend, modify, repeal or adopt new by-laws
may be delegated to the Board of Directors only by the affirmative vote of stockholders representing not
less than 2/3 of the subscribed and paid up capital stock of the corporation, which 2/3 should have
been computed on the basis of the capitalization at the time of the amendment. Since the amendment
was based on the 1961 authorization, petitioner contended that the Board acted without authority and
in usurpation of the power of the stockholders.

ISSUES:

Whether or not respondent SEC committed grave abuse of discretion in allowing discussion of
Item 6 of the Agenda of the Annual Stockholders' Meeting on May 10, 1977, and the ratification of the
investment in a foreign corporation of the corporate funds, allegedly in violation of section 17-1/2 of the
Corporation Law.

RULING:

NO.

Section 17-1/2 of the Corporation Law allows a corporation to "invest its funds in any other
corporation or business or for any purpose other than the main purpose for which it was organized"
provided that its Board of Directors has been so authorized by the affirmative vote of stockholders
holding shares entitling them to exercise at least two-thirds of the voting power. If the investment is
made in pursuance of the corporate purpose, it does not need the approval of the stockholders. It is only
when the purchase of shares is done solely for investment and not to accomplish the purpose of its
incorporation that the vote of approval of the stockholders holding shares entitling them to exercise at
least two-thirds of the voting power is necessary.
As stated by Respondent Corporation, the purchase of beer manufacturing facilities by SMC was an
investment in the same business stated as its main purpose in its Articles of Incorporation, which is to
manufacture and market beer. It appears that the original investment was made in 1947-1948, when
SMC, then San Miguel Brewery, Inc., purchased a beer brewery in Hongkong (Hongkong Brewery &
Distillery, Ltd.) for the manufacture and marketing of San Miguel beer thereat. Restructuring of the
investment was made in 1970-1971 thru the organization of SMI in Bermuda as a tax free
reorganization.
RAMON DE LA RAMA et.al
vs.
MA-AO SUGAR CENTRAL CO., INC., J. AMADO ARANETA, MRS. RAMON S. ARANETA, ROMUALDO
M. ARANETA, and RAMON A. YULO

198 | P a g e
Law 321_Corporation LAW_ Case Digest

G.r.No. L-17504 & l-17506; February 28, 1969

FACTS:

In 1950 the MSCCI through its President, J. Amado, subscribed for P300k worth of capital stock
of the Philippine Fiber Processing Co., Inc. (PFPC). Payments of the subscription were made on 3
installments, but at the time the first two payments were made there was no board resolution
authorizing the investment; and that it was only on November 26, 1951, that J. Amado was so
authorized by the BOD, by the way, making the third payment made in March 1952 authorized.
In addition, 355k shares of PFPC, owned by Luzon Industrial Corporation (LIC) were transferred
on May 31, 1952, to MSCCI. Again, the investment was made without prior board resolution, the
authorizing resolution having been subsequently approved only on June 4, 1952. A derivative suit was
filed by 4 minority SHs of MSCCI which stated 5 causes of action: (1) for alleged illegal and ultra-vires
acts consisting of self-dealing, irregular loans, and unauthorized investments; (2) for alleged gross
mismanagement; (3) for alleged forfeiture of corporate rights warranting dissolution; (4) for alleged
damages and attorney's fees; and (5) for receivership.

ISSUE:

Whether or not a corporation can invest in another corporation.

RULING:

YES.

The law requiring the votes does not apply in the case because of MSCCI’s contention that since
said PFPC was engaged in the manufacture of sugar bags it was perfectly legitimate for MSCCI either to
manufacture sugar bags or invest in another corporation engaged in said manufacture.
SC also quoted the interpretation of Professor Guevara, a well-known authority in Commercial
Law: A private corporation, in order to accomplish its purpose as stated in its articles of incorporation,
and subject to the limitations imposed by the Corporation Law, has the power to acquire, hold,
mortgage, pledge or dispose of shares, bonds, securities, and other evidences of indebtedness of any
domestic or foreign corporation. Such an act, if done in pursuance of the corporate purpose, does not
need the approval of the stockholders; but when the purchase of shares of another corporation is done
solely for investment and not to accomplish the purpose of its incorporation, the vote of approval of the
stockholders is necessary. In any case, the purchase of such shares or securities must be subject to the
limitations established by the Corporation Law; namely, (a) that no agricultural or mining corporation
shall in anywise be interested in any other agricultural or mining corporation; or (b) that a non-
agricultural or non-mining corporation shall be restricted to own not more than 15% of the voting stock
of any agricultural or mining corporation; and (c) that such holdings shall be solely for investment and
not for the purpose of bringing about a monopoly in any line of commerce or combination in restraint of
trade."
Power to invest corporate funds - A private corporation has the power to invest its corporate
funds 'in any other corporation or business, or for any purpose other than the main purpose for which it
was organized,' provided that 'its board of directors has been so authorized in a resolution by the
affirmative vote of stockholders holding shares in the corporation entitling them to exercise at least two-
thirds of the voting power on such a proposal at a stockholders' meeting called for that purpose,' and
provided further, that no agricultural or mining corporation shall in anywise be interested in any other
agricultural or mining corporation. When the investment is necessary to accomplish its purpose or
purposes as stated in its articles of incorporation, the approval of the stockholders is not necessary."
 To Acquire Own Shares

BOMAN ENVIRONMENTAL DEVELOPMENT CORPORATION


vs.
HON. COURT OF APPEALS and NILCAR Y. FAJILAN
G.R. No. 77860. November 22, 1988

FACTS:

On May 7, 1984, respondent Nilcar Y. Fajilan offered in writing to resign as President and
Member of the Board of Directors of petitioner, Boman Environmental Development Corporation
(BEDECO), and to sell to the company all his shares, rights, and interests therein for P 300,000 plus the
transfer to him of the company's Isuzu pick-up truck which he had been using.

199 | P a g e
Law 321_Corporation LAW_ Case Digest

At a meeting of the Board of Directors of BEDECO, Fajilan's resignation as president was


accepted and new officers were elected. Fajilan's offer to sell his shares back to the corporation was
approved, the Board promising to pay for them on a staggered basis from July 15, 1984 to December 15,
1984.
A promissory note was signed by BEDECO'S new president, Alfredo Pangilinan, in the presence of
two directors, committing BEDECO to pay him P300,000 over a six-month period from July 15, 1984 to
December 15, 1984. However, BEDECO paid only P50,000 on July 15, 1984 and another P50,000 on
August 31, 1984 and defaulted in paying the balance of P200,000. On April 30, 1985, Fajilan filed a
complaint in the Regional Trial Court of Makati for collection of that balance from BEDECO.

ISSUES:

Whether or not Petitioner Corporation can acquire its own shares.

RULING:

YES.

The provisions of the Corporation Code should be deemed written into the agreement between the
corporation and the stockholders even if there is no express reference to them in the promissory note.
The principle is well settled that an existing law enters into and forms part of a valid contract without
need for the parties' expressly making reference to it.
The requirement of unrestricted retained earnings to cover the shares is based on the trust fund
doctrine which means that the capital stock, property and other assets of a corporation are regarded as
equity in trust for the payment of corporate creditors. The reason is that creditors of a corporation are
preferred over the stockholders in the distribution of corporate assets. There can be no distribution of
assets among the stockholders without first paying corporate creditors. Hence, any disposition of
corporate funds to the prejudice of creditors is null and void.

C. H. STEINBERG, as Receiver of the Sibuguey Trading Company, Incorporated


vs.
GREGORIO VELASCO, ET AL.
G.R. No. L-30460. March 12, 1929

FACTS:

Plaintiff is the receiver of the Sibuguey Trading Company, a domestic corporation. The defendants
are residents of the Philippine Islands. It is alleged that the defendants, Gregorio Velasco, as president,
Felix del Castillo, as vice-president, Andres L. Navallo, as secretary-treasurer, and Rufino Manuel, as
director of Trading Company, at a meeting of the board of directors, approved and authorized various
lawful purchases already made of a large portion of the capital stock of the company from its various
stockholders with total amount of the capital stock unlawfully purchased was P3,300. At the time of
such purchase, the corporation had accounts payable amounting to P13,807.50, most of which were
unpaid at the time petition for the dissolution of the corporation was its financial condition, in
contemplation of an insolvency and dissolution. That on September 11, 1923, when the petition was
filed for its dissolution upon the ground that it was insolvent, its accounts payable amounted to
P9,241.19, and its accounts receivable P12,512.47, or an apparent asset of P3,271.28 over and above its
liabilities.

200 | P a g e
Law 321_Corporation LAW_ Case Digest

ISSUE:

Whether or not the Petition Corporation can acquire its own shares.

RULING:

NO.

It is, indeed, peculiar that the action of the board in purchasing the stock from the corporation
and in declaring the dividends on the stock was all done at the same meeting of the board of directors,
and it appears in those minutes that the both Ganzon and Mendaros were formerly directors and
resigned before the board approved the purchase and declared the dividends, and that out of the whole
330 shares purchased, Ganzon, sold 100 and Mendaros 200, or a total of 300 shares out of the 330,
which were purchased by the corporation, and for which it paid P3,300.
In other words, the directors were permitted to resign so that they could sell their stock to the
corporation. As stated, the authorized capital stock was P20,000 divided into 2,000 shares of the par
value of P10 each, which only P10,030 was subscribed and paid. Deducting the P3,300 paid for the
purchase of the stock, there would be left P7,000 of paid up stock, from which deduct P3,000 paid in
dividends, there would be left P4,000 only. In this situation and upon this state of facts, it is very
apparent that the directors did not act in good faith or that they were grossly ignorant of their duties.
Creditors of a corporation have the right to assume that so long as there are outstanding debts
and liabilities, the board of directors will not use the assets of the corporation to purchase its own stock,
and that it will not declare dividends to stockholders when the corporation is insolvent.

 Dividend: Kinds: Cash, Stock, Property, Scrip


 Declaration, Payment and Record Dates

IMELDA O. COJUANGCO, PRIME HOLDINGS, INC., AND THE ESTATE OF RAMON U. COJUANGCO
vs.
SANDIGANBAYAN, REPUBLIC OF THE PHILIPPINES, AND THE SHERIFF OF SANDIGANBAYAN
G.R. No. 183278. April 24, 2009

FACTS:

On July 16, 1987, respondent Republic of the Philippines filed before the Sandiganbayan a
"Complaint for Reconveyance, Reversion, Accounting, Restitution and Damages," praying for the
recovery of alleged ill-gotten wealth from the late President Marcos and former First Lady Imelda Marcos
and their cronies, including some 2.4 million shares of stock in the Philippine Long Distance Telephone
Company (PLDT) allegedly registered in the name of Prime Holdings, Inc. (Prime Holdings).
The Sandiganbayan dismissed the complaint with respect to the recovery of the PLDT shares,
hence, the Republic appealed to this Court, docketed as G.R. No. 153459, which appeal was later
consolidated with pending cases of similar import – G.R. Nos. 149802, 150320, and 150367.
The Decision became final and executory on October 26, 2006, hence, the Republic filed on
November 20, 2006 with the Sandiganbayan a Motion for the Issuance of a Writ of Execution, praying
for the cancellation of the 111,415 shares/certificates of stock registered in the name of Prime Holdings
and the annotation of the change of ownership on PTIC’s Stock and Transfer Book. The Republic further
prayed for the issuance of an order for PTIC to account for all cash and stock dividends declared and/or
issued by PLDT in favor of PTIC from 1986 up to the present including compounded interests
appurtenant thereto.

201 | P a g e
Law 321_Corporation LAW_ Case Digest

The Sandiganbayan granted the Motion for the Issuance of a Writ of Execution with respect to the
reconveyance of the shares, but denied the prayer for accounting of dividends. On Motion for
Reconsideration of the Republic, the Sandiganbayan, by the first assailed Resolution dated November 7,
2007, directed PTIC to deliver the cash and stock dividends pertaining to the 111,415 shares, including
compounded interests, ratiocinating that the same were covered by this Court’s Decision in G.R. No.
153459, since the Republic was therein adjudged the owner of the shares and, therefore, entitled to the
fruits thereof.

ISSUES:

Whether or not the Sandiganbayan gravely abused its discretion in ordering the accounting,
delivery, and remittance to the Republic of the stock, cash, and property dividends pertaining to the
111,415 PTIC shares of Prime Holdings.

Whether or not the Republic, having transferred the shares to a third party, is entitled to the
dividends, interests, and earnings thereof.

RULING:

NO.

The term "dividend" in its technical sense and ordinary acceptation is that part or portion of the
profits of the enterprise which the corporation, by its governing agents, sets apart for ratable division
among the holders of the capital stock. It is a payment to the stockholders of a corporation as a return
upon their investment and the right thereto is an incident of ownership of stock.
In directing the reconveyance to the Republic of the 111,415 shares of PLDT stock owned by PTIC
in the name of Prime Holdings, the Court declared the Republic as the owner of said shares and,
necessarily, the dividends and interests accruing thereto.
Contrary to petitioners’ contention, while the general rule is that the portion of a decision that
becomes the subject of execution is that ordained or decreed in the dispositive part thereof, there are
recognized exceptions to this rule, viz: (a).where there is ambiguity or uncertainty, the body of the
opinion may be referred to for purposes of construing the judgment, because the dispositive part of a
decision must find support from the decision’s ratio decidendi; and (b).where extensive and explicit
discussion and settlement of the issue is found in the body of the decision.
In G.R. No. 153459, although the inclusion of the dividends, interests, and earnings of the
111,415 PTIC shares as belonging to the Republic was not mentioned in the dispositive portion of the
Court’s Decision, it is clear from its body that what was being adjudicated in favor of the Republic was
the whole block of shares and the fruits thereof, said shares having been found to be part of the
Marcoses’ ill-gotten wealth, and therefore, public money.
It would be absurd to award the shares to the Republic as their owner and not include the
dividends and interests accruing thereto. An owner who cannot exercise the "juses" or attributes of
ownership -- the right to possess, to use and enjoy, to abuse or consume, to accessories, to dispose or
alienate, to recover or vindicate, and to the fruits - is a crippled owner.

202 | P a g e
Law 321_Corporation LAW_ Case Digest

 Limitation on Retention of Surplus Profits

C. H. STEINBERG, as Receiver of the Sibuguey Trading Company, Incorporated


vs.
GREGORIO VELASCO, ET AL.
G.R. No. L-30460. March 12, 1929

FACTS:

Plaintiff is the receiver of the Sibuguey Trading Company, a domestic corporation. The defendants
are residents of the Philippine Islands. It is alleged that the defendants, Gregorio Velasco, as president,
Felix del Castillo, as vice-president, Andres L. Navallo, as secretary-treasurer, and Rufino Manuel, as
director of Trading Company, at a meeting of the board of directors, approved and authorized various
lawful purchases already made of a large portion of the capital stock of the company from its various
stockholders with total amount of the capital stock unlawfully purchased was P3,300. At the time of
such purchase, the corporation had accounts payable amounting to P13,807.50, most of which were
unpaid at the time petition for the dissolution of the corporation was its financial condition, in
contemplation of an insolvency and dissolution. That on September 11, 1923, when the petition was
filed for its dissolution upon the ground that it was insolvent, its accounts payable amounted to
P9,241.19, and its accounts receivable P12,512.47, or an apparent asset of P3,271.28 over and above its
liabilities.

ISSUE:

Whether or not the Corporation acted in bad faith in acquiring its own shares of stocks.

RULING:

YES.

There is no stipulation or finding of facts as to what was the actual cash value of its accounts
receivable. Neither is there any stipulation that those accounts or any part of them ever have been or
will be collected, and it does appear that after his appointment on February 28, 1924, the receiver made
a diligent effort to collect them, and that he was unable to do so, and it also appears from the minutes of
the board of directors that the president and manager "recommended that P3,000 — out of the surplus
account to be set aside for dividends payable, and that payments be made in installments so as not to
effect the financial condition of the corporation."
It is very apparent that on June 24, 1922, the board of directors acted on assumption that,
because it appeared from the books of the corporation that it had accounts receivable of the face value
of P19,126.02, therefore it had a surplus over and above its debts and liabilities. Thus, in the purchase

203 | P a g e
Law 321_Corporation LAW_ Case Digest

of its own stock to the amount of P3,300 and in declaring the dividends to the amount of P3,000, the
real assets of the corporation were diminished P6,300. The corporation did not then have an actual
bona fide surplus from which the dividends could be paid, and that the payment of them in full at the
time would "affect the financial condition of the corporation."
Creditors of a corporation have the right to assume so long as there are outstanding debts and
liabilities, the board of directors will not use the assets of the corporation to purchase its own stock, and
that it will not declare dividends to stockholders when the corporation is insolvent.

NIELSON & COMPANY, INC.


vs.
LEPANTO CONSOLIDATED MINING COMPANY
G.R. NO. L-21601 DECEMBER 28, 1968

FACTS:

Lepanto entered into a contract with Nielson wherein the latter was to manage and operate the
mining properties of the former claiming to be a contract of agency. However, Nielson claims that the
agreement is not one of agency.The phrase "Both parties to this agreement fully recognize that the terms
of this agreement are made possible only because of the faith and confidence of the officials of each
company have in the other" in paragraph XI of the management contract does not qualify the relation
between Lepanto and Nielson as that of principal and agent based on trust and confidence, such that
the contractual relation may be terminated by the principal at any time that the principal loses trust
and confidence in the agent. Rather, that phrase simply implies the circumstance that brought about
the execution of the management contract. In the annual report for 1936, submitted by Mr. C. A. Dewit,
President of Lepanto, to its stockholders, under date of March 15, 1937, it states that instead of giving a
monthly compensation to Nielson such was modified by giving the amount of P2,500 plus 10% of cash
value of the dividends declared and paid by Lepanto.
The Court ruled that from the foregoing statements in the annual report for 1936, and from the
provision of paragraph XI of the Management contract, that the employment by Lepanto of Nielson to
operate and manage its mines was principally in consideration of the know-how and technical services
that Nielson offered Lepanto. The contract thus entered into pursuant to the offer made by Nielson and
accepted by Lepanto was a "detailed operating contract".

ISSUE:

Whether or not Nielson is entitled to receive shares of stock forming part of the stock dividend of
Lepanto in lieu of the cash value of the dividends declared by Lepanto during the Japanese occupation.

RULING:

NO.

Shares of stock are given the special name "stock dividends" only if they are issued in lieu of
undistributed profits. If shares of stocks are issued in exchange of cash or property then those shares
do not fall under the category of "stock dividends".
A corporation may legally issue shares of stock in consideration of services rendered to it by a person
not a stockholder, or in payment of its indebtedness. A share of stock issued to pay for services rendered
is equivalent to a stock issued in exchange of property, because services are equivalent to
property.Likewise a share of stock issued in payment of indebtedness is equivalent to issuing a stock in
exchange for cash.
In other words, it is the shares of stock that are originally issued by the corporation and forming
part of the capital that can be exchanged for cash or services rendered, or property; that is, if the
corporation has original shares of stock unsold or unsubscribed, either coming from the original
capitalization or from the increased capitalization. Those shares of stock may be issued to a person who
is not a stockholder, or to a person already a stockholder in exchange for services rendered or for cash
or property. But a share of stock coming from stock dividends declared cannot be issued to one who is
not a stockholder of a corporation.

204 | P a g e
Law 321_Corporation LAW_ Case Digest

Thus, stock dividends cannot be issued to a person who is not a stockholder in payment of
services rendered. And so, in the case at bar, Nielson cannot be paid in shares of stock which form part
of the stock dividends of Lepanto for services it rendered under the management contract.

COMMISSIONER OF INTERNAL REVENUE


vs.
MANNING, MCDONALD, SIMMONS
AUGUST 06, 1975

FACTS:

In 1952 the MANTRASCO had an authorized capital stock of P2,500,000 divided into 25,000
common shares; 24,700 of these were owned by Julius S. Reese, and the rest, at 100 shares each, by
the three respondents.
In view of Reese's desire that upon his death MANTRASCO and its two subsidiaries, MANTRASCO
(Guam), Inc. and the Port Motors, Inc., would continue under the management of the respondents, a
trust agreement was executed by and among Reese, MANTRASCO ,the law firm of Ross, Selph,
Carrascoso and Jand ,and the respondents .
On October 19, 1954 Reese died. In 1955, after MANTRASCO made a partial payment of Reese's
shares, the certificate for the 24,700 shares in Reese's name was cancelled and a new certificate was
issued in the name of MANTRASCO, which was endorsed to the law firm of Ross, Selph, Carrascoso and
Janda, as trustees for and in behalf of MANTRASCO.
In 1963 the entire purchase price of Reese's interest in MANTRASCO was finally paid in full by
the latter, In 1964 the trust agreement was terminated and the trustees delivered to MANTRASCO all
the shares which they were holding in trust.

ISSUE:

Whether or not the issuance of the notices of assessment for deficiency income taxes to the
respondents for the year 1958 was proper.

RULING:

YES.

“A stock dividend always involves a transfer of surplus (or profit) to capital stock. A stock
dividend is a conversion of surplus or undivided profits into capital stock, which is distributed to
stockholders in lieu of a cash dividend.' Congress itself has defined the term 'dividend' in No. 115(a) of
the Act as meaning any distribution made by a corporation to its shareholders, whether in money or in
other property, out of its earnings or profits.
The declaration by the respondents and Reese's trustees of MANTRASCO's alleged treasury stock
dividends in favor of the former, brings the ultimate purpose which the parties to the trust instrument
aimed to realize: to make the respondents the sole owners of Reese's interest in MANTRASCO by
utilizing the periodic earnings of that company and its subsidiaries to directly subsidize their purchase
of the said interests, and by making it appear outwardly, through the formal declaration of non-existent
stock dividends in the treasury, that they have not received any income from those firms when, in fact,
by that declaration they secured to themselves the means to turn around as full owners of Reese's
shares. In other words, the respondents, using the trust instrument as a convenient technical device,
bestowed unto themselves the full worth and value of Reese's corporate holdings with the use of the very
earnings of the companies. Such package device, obviously not designed to carry out the usual stock
dividend purpose of corporate expansion reinvestment, e.g. the acquisition of additional facilities and
other capital budget items, but exclusively for expanding the capital base of the respondents in
MANTRASCO, cannot be allowed to deflect the respondents' responsibilities toward our income tax laws.
The conclusion is thus ineluctable that whenever the companies involved herein parted with a portion of
their earnings "to buy" the corporate holdings of Reese, they were in ultimate effect and result making a
distribution of such earnings to the respondents.
All these amounts are consequently subject to income tax as being, in truth and in fact, a flow of
cash benefits to the respondents.

MADRIGAL & COMPANY, INC.


vs.
HON. RONALDO B. ZAMORA, PRESIDENTIAL ASSISTANT FOR LEGAL AFFAIRS, THE HON.
SECRETARY OF LABOR, and MADRIGAL CENTRAL OFFICE EMPLOYEES UNION

205 | P a g e
Law 321_Corporation LAW_ Case Digest

G.R. No. L-48237. June 30, 1987

MADRIGAL & COMPANY, INC.


vs.
HON. MINISTER OF LABOR and MADRIGAL CENTRAL OFFICE EMPLOYEES UNION
No. L-49023. June 30, 1987

FACTS:

The petitioner was engaged, among several other corporate objectives, in the management of Rizal
Cement Co., Inc.Admittedly, the petitioner and Rizal Cement Co., Inc. are sister companies.Both are
owned by the same or practically the same stockholders.On December 28, 1973, the respondent, the
Madrigal Central Office Employees Union, sought for the renewal of its collective bargaining agreement
with the petitioner, which was due to expire on February 28, 1974.Specifically, it proposed a wage
increase of P200.00 a month, an allowance of P100.00 a month, and other economic benefits.The
petitioner, however, requested for a deferment in the negotiations.
After the petitioner's failure to sit down with the respondent union, the latter, commenced a
complaint for unfair labor practice.Pending the resolution of the case, the petitioner, in a letter informed
the Secretary of Labor that Rizal Cement Co., Inc., "from which it derives income" had "ceased operating
temporarily."In addition, because of the desire of the stockholders to phase out the operations of the
Madrigal & Co., Inc. due to lack of business incentives and prospects, and in order to prevent further
losses,it had to reduce its capital stock on two occasions, the Madrigal & Co., Inc. is without substantial
income to speak of, necessitating a reorganization, by way of retrenchment, of its employees and
operations.

ISSUE:

Whether or not the profits earned by the Corporation were in the nature of dividends declared on
its shareholdings in other companies in the earning of which the employees had no participation
whatsoever.

RULING:

NO.

The Court agreed with the National Labor Relations Commission that "the dividends received by
the company are corporate earnings arising from corporate investment."Indeed, as found by the
Commission, the petitioner had entered such earnings in its financial statements as profits, which it
would not have done if they were not in fact profits.
Moreover, it is incorrect to say that such profits — in the form of dividends — are beyond the
reach of the petitioner's creditors since the petitioner had received them as compensation for its
management services in favor of the companies it managed as a shareholder thereof. As such
shareholder, the dividends paid to it were its own money, which may then be available for wage
increments. It is not a case of a corporation distributing dividends in favor of its stockholders, in which
case, such dividends would be the absolute property of the stockholders and hence, out of reach by
creditors of the corporation. Here, the petitioner was acting as stockholder itself, and in that case, the
right to a share in such dividends, by way of salary increases, may not be denied its employees.

REPUBLIC PLANTERS BANK


vs.
HON. ENRIQUE A. AGANA, SR., as Presiding Judge, Court of First Instance of Rizal, Branch
XXVIII, Pasay City, ROBES-FRANCISCO REALTY & DEVELOPMENT CORPORATION and ADALIA F.
ROBES
G.R. No. 51765. March 3, 1997

FACTS:

On September 18, 1961, private respondent Corporation secured a loan from petitioner in the
amount of P120,000.00. Instead of giving the legal tender totaling to the full amount of the loan, which
is P120,000.00, petitioner lent such amount partially in the form of money and partially in the form of
stock certificates numbered 3204 and 3205, each for 400 shares with a par value of P10.00 per share,
or for P4,000.00 each, for a total of P8,000.00. Said stock certificates were in the name of private

206 | P a g e
Law 321_Corporation LAW_ Case Digest

respondent Adalia F. Robes and Carlos F. Robes, who subsequently, however, endorsed his shares in
favor of Adalia F. Robes.
On January 31, 1979, private respondents proceeded against petitioner and filed a Complaint
anchored on private respondents' alleged rights to collect dividends under the preferred shares in
question and to have petitioner redeem the same under the terms and conditions of the stock
certificates.
The trial court rendered the herein assailed decision in favor of private respondents ordering
petitioner to pay private respondents the face value of the stock certificates as redemption price, plus
1% quarterly interest thereon until full payment.

ISSUES:

Whether or not the corporation can declare dividends.

RULING:

YES.

Under the old Corporation Law in force at the time the contract between the petitioner and the
private respondents was entered into, it was provided that "no corporation shall make or declare any
dividend except from the surplus profits arising from its business, or distribute its capital stock or
property other than actual profits among its members or stockholders until after the payment of its
debts and the termination of its existence by limitation or lawful dissolution."Similarly, the present
Corporation Codeprovides that the board of directors of a stock corporation may declare dividends only
out of unrestricted retained earnings.
Thus, the declaration of dividends is dependent upon the availability of surplus profit or
unrestricted retained earnings, as the case may be. Dividends are thus payable only when there are
profits earned by the corporation and as a general rule, even if there are existing profits, the board of
directors has the discretion to determine whether or not dividends are to be declared.
Redeemable shares, on the other hand, are shares usually preferred, which by their terms are
redeemable at a fixed date, or at the option of either issuing corporation, or the stockholder, or both at a
certain redemption price.A redemption by the corporation of its stock is, in a sense, a repurchase of it
for cancellation.The present Code allows redemption of shares even if there are no unrestricted retained
earnings on the books of the corporation.
However, while redeemable shares may be redeemed regardless of the existence of unrestricted
retained earnings, this is subject to the condition that the corporation has, after such redemption,
assets in its books to cover debts and liabilities inclusive of capital stock. Redemption, therefore, may
not be made where the corporation is insolvent or if such redemption will cause insolvency or inability of
the corporation to meet its debts as they mature.

NORA A. BITONG
vs.
COURT OF APPEALS (FIFTH DIVISION), EUGENIA D. APOSTOL, JOSE A. APOSTOL, MR. & MS.
PUBLISHING CO., LETTY J. MAGSANOC, AND ADORACION G. NUYDA
G.R. No. 123553. July 13, 1998
NORA A. BITONG
vs.
COURT OF APPEALS (FIFTH DIVISION) and EDGARDO B. ESPIRITU
(CA-G.R. No. 33873) July 13, 1998

FACTS:

Bitong alleged that she was the treasurer and member of the BoD of Mr. & Mrs. Corporation. She
filed a complaint with the SEC to hold respondent spouses Apostol liable for fraud, misrepresentation,
disloyalty, evident bad faith, conflict of interest and mismanagement in directing the affairs of the
corporation to the prejudice of the stockholders. She alleges that certain transactions entered into by
the corporation were not supported by any stockholder’s resolution. The complaint sought to enjoin
Apostol from further acting as president-director of the corporation and from disbursing any money or
funds.
Apostol contends that Bitong was merely a holder-in-trust of the JAKA shares of the corporation,
hence, not entitled to the relief she prays for. SEC Hearing Panel issued a writ enjoining Apostol. After

207 | P a g e
Law 321_Corporation LAW_ Case Digest

hearing the evidence, SEC Hearing Panel dissolved the writ and dismissed the complaint filed by Bitong.
Bitong appealed to the SEC en banc which reversed SEC Hearing Panel decision. Apostol filed petition
for review with the CA. CA reversed SEC en banc ruling holding that Bitong was not the owner of any
share of stock in the corporation and therefore, not a real party in interest to prosecute the complaint.

ISSUE:

Whether or not petitioner validly declared dividends.

RULING:

YES.

The records show that the original stock and transfer book and the stock certificate book of Mr. &
Ms. were in the possession of petitioner before their custody was transferred to the Corporate Secretary,
Atty. Augusto San Pedro. On 25 May 1988, Assistant Corporate Secretary Renato Jose Unson wrote
Mr.& Ms. about the lost stock and transfer book which was also noted by the corporation's external
auditors, Punongbayan and Araullo, in their audit. Atty. Unson even informed respondent Eugenia D.
Apostol as President of Mr. & Ms. that steps would be undertaken to prepare and register a new Stock
and Transfer Book with the SEC. Incidentally, perhaps strangely, upon verification with the SEC, it was
discovered that the general file of the corporation with the SEC was missing. Hence, it was even possible
that the original Stock and Transfer Book might not have been registered at all.
This simply shows that as of 1988 there still existed certain issues affecting the ownership of the
JAKA shares, thus raising doubts whether the alleged transactions recorded in the Stock and Transfer
Book were proper, regular and authorized. JAKA retained its ownership of Mr.& Ms. shares as clearly
shown by its receipt of the dividends issued in December 1986. This only means, very obviously, that
Mr.& Ms. shares in question still belonged to JAKA and not to petitioner. For, dividends are distributed
to stockholders pursuant to their right to share in corporate profits. When a dividend is declared, it
belongs to the person who is the substantial and beneficial owner of the stock at the time regardless of
when the distribution profit was earned.

COMMISSIONER OF INTERNAL REVENUE,


vs.
THE COURT OF APPEALS, COURT OF TAX APPEALS AND A. SORIANO CORPORATION
G.R. NO. 108576; JANUARY 20, 1999

FACTS:

Sometime in the 1930s, Don Andres Soriano, a citizen and resident of the United States, formed
the corporation "A. Soriano Y Cia", predecessor of ANSCOR, with a P1,000,000.00 capitalization divided
into 10,000 common shares at a par value of P100/share. In 1937, Don Andres subscribed to 4,963
shares of the 5,000 shares originally issued. On September 12, 1945, ANSCOR's authorized capital
stock was increased to P2,500,000.00 divided into 25,000 common shares with the same par value of
the additional 15,000 shares, only 10,000 was issued which were all subscribed by Don Andres. A
month later, Don Andres transferred 1,250 shares each to his two sons, Jose and Andres, Jr., as their
initial investments in ANSCOR. Both sons are foreigners. By 1947, ANSCOR declared stock dividends.
As of that date, the records revealed that he has total shareholdings of 185,154 shares, 50,495 of
which are original issues and the balance of 134.659 shares as stock dividend declarations.
Correspondingly, one-half of that shareholdings or 92,577 shares were transferred to his wife, Doña
Carmen Soriano, as her conjugal share. The other half formed part of his estate.
A day after Don Andres died, ANSCOR increased its capital stock to P20M and in 1966 further
increased it to P30M. In the same year (December 1966), stock dividends worth 46,290 and 46,287
shares were respectively received by the Don Andres estate and Doña Carmen from ANSCOR. Hence,
increasing their accumulated shareholdings to 138,867 and 138,864 common shares each.

ISSUE:

Whether or not ANSCOR's redemption of stocks from its stockholder as well as the exchange of
common with preferred shares can be considered as "essentially equivalent to the distribution of taxable
dividend" making the proceeds thereof taxable.

208 | P a g e
Law 321_Corporation LAW_ Case Digest

RULING:

YES.

Stock dividends, strictly speaking, represent capital and do not constitute income to its recipient.
So that the mere issuance thereof is not yet subject to income tax as they are nothing but enrichment
through increase in value of capital investment." As capital, the stock dividends postpone the realization
of profits because the "fund represented by the new stock has been transferred from surplus to capital
and no longer available for actual distribution." In a loose sense, stock dividends issued by the
corporation, are considered unrealized gain, and cannot be subjected to income tax until that gain has
been realized. Before the realization, stock dividends are nothing but a representation of an interest in
the corporate properties. As capital, it is not yet subject to income tax.
However, if a corporation cancels or redeems stock issued as a dividend at such time and in such
manner as to make the distribution and cancellation or redemption, in whole or in part, essentially
equivalent to the distribution of a taxable dividend, the amount so distributed in redemption or
cancellation of the stock shall be considered as taxable income to the extent it represents a distribution
of earnings or profits accumulated after March first, nineteen hundred and thirteen.
It is not the stock dividends but the proceeds of its redemption that may be deemed as taxable
dividends.

 To Enter into a Management Contract

WOLRGANG AURBACH, JOHN GRIFFIN, DAVID P. WHITTINGHAM and CHARLES CHAMSAY


vs.
SANITARY WARES MANUFACTURING CORPORATOIN, ERNESTO V. LAGDAMEO, ERNESTO R.
LAGDAMEO, JR., ENRIQUE R. LAGDAMEO, GEORGE F. LEE, RAUL A. BONCAN, BALDWIN YOUNG
and AVELINO V. CRUZ
G.R. No. 75875. December 15, 1989

SANITARY WARES MANUFACTURING CORPORATION, et.al


vs.
THE COURT OF APPEALS, WOLFGANG AURBACH, JOHN GRIFFIN, DAVID P. WHITTINGHAM,
CHARLES CHAMSAY and LUCIANO SALAZAR
G.R. No. 75951 December 15, 1989

FACTS:

In 1961, Saniwares, a domestic corporation was incorporated for the primary purpose of
manufacturing and marketing sanitary wares. One of the incorporators, Mr. Baldwin Young went abroad
to look for foreign partners, European or American who could help in its expansion plans.
On August 15, 1962, ASI, a foreign corporation domiciled in Delaware, United States entered into
an Agreement with Saniwares and some Filipino investors whereby ASI and the Filipino investors agreed
to participate in the ownership of an enterprise which would engage primarily in the business of
manufacturing in the Philippines and selling here and abroad vitreous china and sanitary wares. The
parties agreed that the business operations in the Philippines shall be carried on by an incorporated
enterprise in the name of "Sanitary Wares Manufacturing Corporation."
Later, the 30% capital stock of ASI was increased to 40%. The corporation was also registered
with the Board of Investments for availment of incentives with the condition that at least 60% of the
capital stock of the corporation shall be owned by Philippine nationals.

ISSUES:

Whether or not the ASI group may vote their additional equity during elections of Saniwares'
board of directors.

RULING:

YES.

In regard to the question as to whether or not the ASI group may vote their additional equity
during elections of Saniwares' board of directors. As in other joint venture companies, the extent of ASI's

209 | P a g e
Law 321_Corporation LAW_ Case Digest

participation in the management of the corporation is spelled out in the Agreement. Section 5(a) hereof
says that three of the nine directors shall be designated by ASI and the remaining six by the other
stockholders, i.e., the Filipino stockholders. This allocation of board seats is obviously in consonance
with the minority position of ASI.
Having entered into a well-defined contractual relationship, it is imperative that the parties
should honor and adhere to their respective rights and obligations thereunder. Appellants seem to
contend that any allocation of board seats, even in joint venture corporations, are null and void to the
extent that such may interfere with the stockholder's rights to cumulative voting as provided in Section
24 of the Corporation Code.
On the one hand, the clearly established minority position of ASI and the contractual allocation of
board seats cannot be disregarded. On the other hand, the rights of the stockholders to cumulative
voting should also be protected.

PHILIPPINE NATIONAL BANK


vs.
PRODUCERS' WAREHOUSE ASSOCIATION
G.R. No. L-16510. January 9, 1922

FACTS:

The plaintiff is a corporation organized under the banking laws of the Philippine while defendant
is a domestic corporation doing a general warehouse business and the Philippine Fiber and Produce
Company, to which hereafter refer as the Produce Company, is another domestic corporation.
In May, 1916, the defendant, as party of the first part, entered into a written contract with the
Produce Company, as party of the second part, in and by which "the above-named party of the second
part is hereby named, constituted, and appointed as the general manager of the business of the party of
the first part, in all of the branches thereof, with the duties, powers, authority and compensation
hereinafter provided." It shall exercise a general and complete supervision over and management of the
business of the party of the first part," and "shall direct, manage, promote and advance the said
business, subject only to the control and instructions of the board of directors of the party of the first
part."
It is also alleged that in January, 1919, with the consent of the plaintiff, the Produce Company
removed from the warehouse of the defendant 1,112.15 piculs of copra described in receipt No. 1255, of
the declared value of P18,350.

ISSUE:

Whether or not the corporation has the power to enter into management contract.

RULING:

YES.

Under the written contract between them, the Produce Company was the general manager of the
defendant's warehouse business, and that it had authority to issue quedans in its name, and as its
corporate act and deed. That the quedans in question are duly authenticated, and were duly issued by
the defendant to, and in the name of, the Produce Company, and when issued were duly endorsed, and
delivered to the plaintiff for value. For aught that appears in the record, the bank was acting in good
faith, and the quedans were duly issued, endorsed and delivered to it as collateral in the ordinary course
of business. Although there may have been fraud, there is no allegation or proof that the bank was a
party to it, or had any knowledge of it, and this court has no right to assume that the bank was a party
to a fraud. Giving to the quedans their legal force and effect, it must follow that at the time the demand
was made; the bank was the owner and entitled to the possession of the copra therein described. The
receipts call for 15,699.34 piculs of copra, but plaintiff admits that, with its consent, 1,112.15 piculs of
copra, of the declared value of P18,350, were delivered to the Produce Company from and out of receipt
No. 1255. This would leave 14,587.19 piculs of copra evidenced by the quedans.

210 | P a g e
Law 321_Corporation LAW_ Case Digest

NIELSON & COMPANY, INC.


vs.
LEPANTO CONSOLIDATED MINING COMPANY
G.R. NO. L-21601 DECEMBER 28, 1968

FACTS:

Lepanto entered into a contract with Nielson wherein the latter was to manage and operate the
mining properties of the former claiming to be a contract of agency. However, Nielson claims that the
agreement is not one of agency.The phrase "Both parties to this agreement fully recognize that the terms
of this agreement are made possible only because of the faith and confidence of the officials of each
company have in the other" in paragraph XI of the management contract does not qualify the relation
between Lepanto and Nielson as that of principal and agent based on trust and confidence, such that
the contractual relation may be terminated by the principal at any time that the principal loses trust
and confidence in the agent. Rather, that phrase simply implies the circumstance that brought about
the execution of the management contract. In the annual report for 1936, submitted by Mr. C. A. Dewit,
President of Lepanto, to its stockholders, under date of March 15, 1937, it states that instead of giving a
monthly compensation to Nielson such was modified by giving the amount of P2,500 plus 10% of cash
value of the dividends declared and paid by Lepanto.
Thus, the contention of Lepanto that it had terminated the management contract in 1945,
following the liberation of the mines from Japanese control, because the relation between it and Nielson
was one of agency and as such it could terminate the agency at will, is, therefore, untenable.

ISSUE:

Whether or not the nature of the management contracta contract of agency.

RULING:

NO.

By the contract of agency a person binds himself to render some service or to do something in
representation or on behalf of another, with the consent or authority of the latter.
Under the contract, Nielson had agreed, for a period of five years, with the right to renew for a like
period, to explore, develop and operate the mining claims of Lepanto, and to mine, or mine and mill,
such pay ore as may be found therein and to market the metallic products recovered therefrom which
may prove to be marketable, as well as to render for Lepanto other services specified in the contract.
Moreover, the contract thus entered into pursuant to the offer made by Nielson and accepted by
Lepanto was a "detailed operating contract". It was not a contract of agency. Nowhere in the record is it
shown that Lepanto considered Nielson as its agent and that Lepanto terminated the management
contract because it had lost its trust and confidence in Nielson.
In the construction of an instrument where there are several provisions or particulars, such a
construction is, if possible, to be adopted as will give effect to all, and if some stipulation of any contract
should admit of several meanings, it shall be understood as bearing that import which is most adequate
to render it effectual.
Thus, by express stipulation of the parties, the management contract in question is not revocable
at the will of Lepanto. The Court ruled that this management contract is not a contract of agency as
defined in Article 1709 of the old Civil Code, but a contract of lease of services as defined in Article 1544
of the same Code.

J. M. TUASON & CO., INC., represented by it Managing PARTNER, GREGORIA ARANETA, INC.
vs.
QUIRINO BOLAÑOS

211 | P a g e
Law 321_Corporation LAW_ Case Digest

G.R. No. L-4935. May 28, 1954

FACTS:

The complaint described the land as a portion of a lot registered in the name of the plaintiff and
as containing an area of 13 hectares more or less. But the complaint was amended by reducing the area
of 6 hectares after the defendant had indicated the plaintiff’s surveyors the portion of the land claimed
and occupied by him. The defendant set up the defense of prescription.
After trial, the lower court rendered judgment for plaintiff, declaring defendant to be without any
right to the land in question and ordering him to restore possession thereof to plaintiff and to pay the
latter a monthly rent.

ISSUE:

Whether or not the trial court erred in not dismissing the case on the ground that the case was
not brought by the real party in interest.

RULING:

NO.

There is nothing to the contention that the persent action is not brought by the real party in
interest, that is, by J. Tuason & Co., Inc. The Rules of Court requires that action must be brought in the
name of the real party in interest.
The practice is for an attorney-at-law to bring the action that is to file the complaint, in the name
of the plaintiff. That practice appears to have been followed in this case, since the case is signed by the
law firm of Araneta and Araneta, “counsel for plaintiff” and commences with the statement “comes
plaintiff, through its undersigned counsel”. It is true that the complaint also states that the plaintiff is
“represented herein by its Managing Partner Gregorio Araneta, Inc. “ another corporation, but there is
nothing against one corporation being represented by another person, natural or juridical, in suit in
court.
The contention that Gregorio Araneta, Inc. cannot act as managing partner for plaintiff on the
theory that it is illegal for two corporations to enter into partnership is without merit, for the true rule is
that “through a corporation has no power to enter into a partnership, it nevertheless enter into a joint
venture with the another where the nature of that venture is in line with the business authorized by its
charter.”

 Ultra Vires Acts

HEIRS OF ANTONIO PAEL and ANDREA ALCANTARA and CRISANTO PAEL


vs.
COURT OF APPEALS, JORGE H. CHIN and RENATO B. MALLARI
G.R. No. 133547. December 7, 2001
MARIA DESTURA
vs.

212 | P a g e
Law 321_Corporation LAW_ Case Digest

COURT OF APPEALS, JORGE H. CHIN and RENATO B. MALLARI


G.R. No. 133843. December 7, 2001

FACTS:

PFINA acquired the properties from the Heirs of Pael by virtue of a deed of assignment dated
January 25, 1983. It filed a motion to intervene before the Court of Appeals; however, before it filed its
motion for intervention, or for a long period of fifteen (15) years, PFINA and the Heirs of Pael were totally
silent about the alleged deed of assignment. No steps were taken by either of them to register the deed
or secure transfer certificate of title evidencing the change of ownership during this long period of time.
At the time PFINA acquired the disputed properties in 1983, its corporate name was PFINA
Mining and Exploration, Inc., a mining company which had no valid grounds to engage in the highly
speculative business of urban real estate development.

ISSUE:

Whether or not the title could produce legal effect.

RULING:

YES.

Notwithstanding its belated filing, the motion for intervention of U.P. is granted, albeit the
adjudication thereof shall be limited to a determination of the alleged overlapping or encroachment
between U.P.’s title, on the one hand, and respondents’ TCT Nos. 52928 and 52929, on the other hand.
The Court highlighted the citation in the comment of Intervenor U.P., specifically citing the
decision in Roberto A. Pael et al. v. Court of Appeals, et al., supra, wherein the title of the Paels was
declared to be of dubious origin and a fabrication. Hence, since respondents derive their titles from a
defective title, their titles should also be null and void.
Considering the conflicting claims by U.P. and respondents, the ascertainment of boundaries of
the lands they respectively claim becomes imperative. The instant cases have altogether taken more
than eight (8) years. The boundaries of the properties covered by the disputed titles of respondents and
the boundaries of the lands covered by the title of U.P. are not discussed therein. Thus, in order to avoid
the institution of new cases and thus obviate further litigation, the case should be remanded to the
Court of Appeals for reception of evidence relevant to determining the boundaries of the conflicting
claims between U.P. and respondents Chin and Mallari over the property in dispute.

PILIPINAS LOAN COMPANY, INC.


vs.
HON. SECURITES AND EXCHANGE COMMISSION AND FILIPINAS PAWNSHOP, INC.
G.R. No. 104720. April 4, 2001

FACTS:

Private respondent Filipinas Pawnshop, Inc. (private respondent) is a duly organized corporation
registered with the Securities and Exchange Commission (SEC) on February 9, 1959. The articles of
incorporation of private respondent states that its primary purpose is to extend loans at legal interest on
the security of either personal properties or on the security of real properties, and to finance installment
sales of motor vehicles, home appliances and other chattels.
On September 11, 1990, private respondent filed a complaint against petitioner with the
Prosecution and Enforcement Department (PED) of the SEC and alleged that petitioner, contrary to the
restriction set by the Commission, has been operating and doing business as a pawnbroker, pawnshop
or "sanglaan" in the same neighborhood where private respondent has had its own pawnshop for 30

213 | P a g e
Law 321_Corporation LAW_ Case Digest

years in violation of its primary purpose and without the imprimatur of the Central Bank to engage in
the pawnshop business thereby causing unjust and unfair competition with private respondent.
On October 18, 1990, petitioner filed its Comment/Answer questioning the power of the SEC to
take cognizance of the complaint involving (1) a supposed violation of the Pawnshop Regulations Act
which is more properly within the jurisdiction of the Central Bank; and (2) the determination of whether
a corporate name is confusingly similar to another which is within the jurisdiction of the regular courts.
Petitioner denied that it is engaged in the pawnshop business, alleging that it is a lending investor duly
registered with the Central Bank.

ISSUE:

Whether or not Pilipinas Loan was acting beyond its authority.

RULING:

YES.

A corporation, under the Corporation Code, has only such powers as are expressly granted to it
by law and by its articles of incorporation,those which may be incidental to such conferred powers,
those reasonably necessary to accomplish its purposes and those which may be incident to its existence.
In the case at bar, the limit of the powers of petitioner as a corporation is very clear, it is categorically
prohibited from "engaging in pawnbroking as defined under PD 114". Hence, in determining what
constitutes pawnbrokerage, the relevant law to consider is PD 114. This reference to PD 114 is also in
line with Article 2123 of the Civil.
Clearly, the recital in the complaint of Filipinas Pawnshop that Pilipinas Loan is engaged in the
pawnshop business when it is not authorized to do so by its articles of incorporation amounts to fraud,
detrimental not only to the corporation but also to the stockholders and the public. The billboards of
Pilipinas loan uses the word “SANGLAAN” which cannot but give the impression to the public that its
establishment is more of a pawnshop than a lending institution servicing different kinds of loans. The
use of such word by petitioner was more calculated to attract customers who will acquire loans on the
security of personal properties alone.

ERNESTINA CRISOLOGO-JOSE
vs.
COURT OF APPEALS and RICARDO S. SANTOS, JR. in his own behalf and as Vice-President for
Sales of Mover Enterprises, Inc
G.R. No. 80599. September 15, 1989

FACTS:

In 1980, plaintiff Ricardo S. Santos, Jr. was the vice-president of Mover Enterprises, Inc. in-
charge of marketing and sales; and the president of the said corporation was Atty. Oscar Z. Benares. On
April 30, 1980, Atty. Benares, in accommodation of his clients, the spouses Jaime and Clarita Ong,
issued Check No. 093553 drawn against Traders Royal Bank, dated June 14, 1980, in the amount of
P45,000.00 payable to defendant Ernestina Crisologo-Jose. Since the check was under the account of
Mover Enterprises, Inc., the same was to be signed by its president, Atty. Oscar Z. Benares, and the
treasurer of the said corporation. However, since at that time, the treasurer of Mover Enterprises was
not available, Atty. Benares prevailed upon the plaintiff, Ricardo S. Santos, Jr., to sign the aforesaid
check as an alternate story. Plaintiff Ricardo S. Santos, Jr. did sign the check.
It appears that the check was issued to defendant Ernestina Crisologo-Jose in consideration of
the waiver or quitclaim by said defendant over a certain property which the Government Service
Insurance System (GSIS) agreed to sell to the clients of Atty. Oscar Benares, the spouses Jaime and
Clarita Ong. However, since the compromise agreement was not approved within the expected period of
time, the aforesaid check for P45,000.00 was replaced by Atty. Benares with another Traders Royal
Bank check dated August 10, 1980, in the same amount.

ISSUE:

214 | P a g e
Law 321_Corporation LAW_ Case Digest

Whether or not the accommodation party is Mover Enterprises, Inc. and hence should be made
liable.

RULING:

NO.

The provision of the Negotiable Instruments Law which holds an accommodation party liable on
the instrument to a holder for value, although such holder at the time of taking the instrument knew
him to be only an accommodation party, does not include nor apply to corporations which are
accommodation parties. This is because the issue or indorsement of negotiable paper by a corporation
without consideration and for the accommodation of another is ultra vires. Hence, one who has taken
the instrument with knowledge of the accommodation nature thereof cannot recover against a
corporation where it is only an accommodation party. If the form of the instrument, or the nature of the
transaction, is such as to charge the indorsee with knowledge that the issue or indorsement of the
instrument by the corporation is for the accommodation of another, he cannot recover against the
corporation thereon.
By way of exception, an officer or agent of a corporation shall have the power to execute or
indorse a negotiable paper in the name of the corporation for the accommodation of a third person only
if specifically authorized to do so. Corollarily, corporate officers, such as the president and vice-
president, have no power to execute for mere accommodation a negotiable instrument of the corporation
for their individual debts or transactions arising from or in relation to matters in which the corporation
has no legitimate concern. Since such accommodation paper cannot thus be enforced against the
corporation, especially since it is not involved in any aspect of the corporate business or operations, the
inescapable conclusion in law and in logic is that the signatories thereof shall be personally liable
therefor, as well as the consequences arising from their acts in connection therewith.

IRINEO G. CARLOS
vs.
MINDORO SUGAR CO., ET AL.
G.R. No. L-36207. October 26, 1932

FACTS:

The Mindoro Sugar Company is a corporation constituted in accordance with the laws of the
country and registered on July 30, 1917. According to its articles of incorporation, one of its principal
purposes was to acquire and exercise the franchise granted by Act No. 2720 to George H. Fairchild, to
substitute the organized corporation, the Mindoro Company, and to acquire all the rights and
obligations of the latter and of Horace Havemeyer and Charles J. Welch in the so-called San Jose Estate
in the Province of Mindoro.
The Philippine Trust Company is another domestic corporation, registered on October 21, 1917.
In its articles of incorporation, some of its purposes are expressed thus: "To acquire by purchase,
subscription, or otherwise, and to invest in, hold, sell, or otherwise dispose of stocks, bonds, mortgages,
and other securities, or any interest in either, or any obligations or evidences of indebtedness, of any
other corporation or corporations, domestic or foreign.
In pursuance of this resolution, the Mindoro Sugar Company executed in favor of the Philippine
Trust Company the deed of trust transferring all of its property to it in consideration of the bonds it had
issued to the value of P3,000,000.
The Philippine Trust Company sold thirteen bonds, Nos. 1219 to 1231, to Ramon Diaz for
P27,300, at a net profit of P100 per bond. The four bonds Nos. 1219, 1220, 1221, and 1222, here in
litigation, are included in the thirteen sold to Diaz. The Philippine Trust Company paid the appellant,
upon presentation of the coupons, the stipulated interest from the date of their maturity until the 1st of
July, 1928, when it stopped payments; and thenceforth it alleged that it did not deem itself bound to
pay such interest or to redeem the obligation because the guarantee given for the bonds was illegal and
void.

ISSUE:

Whether or not Philippine Trust Company bound itself legally and acted within its corporate
powers in guaranteeing the four bonds in question.

215 | P a g e
Law 321_Corporation LAW_ Case Digest

RULING:

YES.

The Philippine Trust Company has full powers to acquire personal property such as the bonds in
question. Being authorized to acquire the bonds, it was given implied power to guarantee them in order
to place them upon the market under better, more advantageous conditions, and thereby secure the
profit derived from their sale. It is not, however, ultra vires for a corporation to enter into contracts of
guaranty or suretyship where it does so in the legitimate furtherance of its purposes and business. And
it is well settled that where a corporation acquires commercial paper or bonds in the legitimate
transaction of its business it may sell them, and in furtherance of such a sale it may, in order to make
them the more readily marketable, indorse or guarantee their payment.
Guaranties of payment of bonds taken by a loan and trust company in the ordinary course of its
business, made in connection with their sale, are not ultra vires, and are binding.

MARIA CLARA PIROVANO ET AL.


vs.
THE DE LA RAMA STEAMSHIP CO.
G.R. No. L-5377. December 29, 1954

FACTS:

Plaintiffs herein are the minor children of the late Enrico Pirovano represented by their mother
and judicial guardian Estefania R. Pirovano. They seek to enforce certain resolutions adopted by the
Board of Directors and stockholders of the defendant company giving to said minor children of the
proceeds of the insurance policies taken on the life of their deceased father Enrico Pirovano with the
company as beneficiary. Defendant's main defense is: that said resolutions and the contract executed
pursuant thereto are ultra vires, and, if valid, the obligation to pay the amount given is not yet due and
demandable.
Plaintiff-appellant Pirovano is the owner of 3424 shares of stocks in defendant-appellee
Corporation which declared a dividend of P100 per share. Appellant wants to recover from appellee the
sum of P221, 975 after deducting the sum of P120, 424 which she had withdrawn or received from
appellee for advances she received after the death of her father, the late Esteban de la Rama.
Appellant’s theory is that the cash advances to her for her personal use and that of her children
were assumed by Esteban de la Rama. She claims that the advances made to her by appellees were
debited from the account of Hijos de I. de la Rama, another corporation practically owned by Esteban de
la Rama. She further claims that the appellee can only deduct from the amount of dividend she is
entitled to, the amount of cash advances which was not assumed by her father. The withdrawals by the
appellant were made during the period 1940 to 1949 during which the appellee made a deed of trust
with Hijos. The deed of trust was made to circumvent the prohibition of declaring dividends during the
period.

ISSUE:

Whether or not the donation made by the corporation of the proceeds of the insurance is a valid
act.

RULING:

YES.

Even assuming that the donation was ultra vires, still it cannot be invalidated or declared legally
ineffective for that reason alone, it appearing that the donation represents not only the act of the Board
but also that of the stockholders themselves since they expressly ratified the resolution. By this
ratification, the infirmity of the corporate act, if any, has been obliterated thereby making the act

216 | P a g e
Law 321_Corporation LAW_ Case Digest

perfectly valid and enforceable, especially so if the donation is not merely executory but consummated.
The defense of ultra vires cannot be set up against completed or consummated transactions.
An ultra vires act may either be an act performed merely outside the scope of the powers granted
to the corporation by its AOI or one which is contrary to law or violative of any principle which would
void any contract. A distinction has to be made with respect to corporate acts which are illegal and those
merely ultra vires. The former are contrary to law, morals, public order or policy, while the latter are not
void ab initio, but merely go beyond the scope of the powers in the AOI, and which renders the act
merely voidable and thus can be ratified by the stockholders.
The defendant corporation, therefore, is now prevented or estopped from contesting the validity of
the donation. This is especially so in this case when the very directors who conceived the idea of
granting said donation are practically the stockholders themselves, with few nominal exceptions.

REPUBLIC OF THE PHILIPPINES


vs.
ACOJE MINING COMPANY, INC.
G.R. No. L-18062. February 28, 1963

FACTS:

On May 17, 1948, the Acoje Mining Company, Inc. wrote the Director of Posts requesting the
opening of a post, telegraph and money order offices at its mining camp at Sta. Cruz, Zambales, to
service its employees and their families that were living in said camp. Acting on the request, the Director
of Posts wrote in reply stating that if aside from free quarters the company would provide for all
essential equipment and assign a responsible employee to perform the duties of a postmaster without
compensation from his office until such time as funds therefor may be available he would agree to put
up the offices requested. The company in turn replied signifying its willingness to comply with all the
requirements.
On April 11, 1949, the Director of Posts again wrote a letter to the company stating among other
things that "In cases where a post office will be opened under circumstances similar to the present, it is
the policy of this office to have the company assume direct responsibility for whatever pecuniary loss
may be suffered by the Bureau of Posts by reason of any act of dishonesty, carelessness or negligence on
the part of the employee of the company who is assigned to take charge of the post office," thereby
suggesting that a resolution be adopted by the board of directors of the company expressing conformity
to the above condition relative to the responsibility to be assumed buy it in the event a post office
branch is opened as requested.
On September 2, 1949, the company informed the Director of Posts of the passage by its board of
directors of a resolution The letter further states that the company feels that that resolution fulfills the
last condition imposed by the Director of Posts and that, therefore, it would request that an inspector be
sent to the camp for the purpose of acquainting the postmaster with the details of the operation of the
branch office.

ISSUE:

Whether or not the act of the Board in issuing the said resolution of conformity was ultra vires.

RULING:

NO.

The corporate act was a necessary corollary to promote the interest and welfare of the
corporation. This is further bolstered by the fact that the opening of the post was upon the request of
the company for the convenience and benefit of its employees, and not an idea of the Director of Posts.
Thus, having benefited from the agreement, the corporation is estopped from raising the defense that
the said corporate act by its board in conforming to the condition imposed by the Director of Posts is
ultra vires.
Neither can the corporation interpose the defense that its liability is only that of a guarantor. A
mere reading of the resolution of the Board of Directors dated August 31, 1949, upon which the plaintiff
based its claim, would show that the responsibility of the defendant company is not just that of a
guarantor. The phraseology and the terms employed are so clear and sweeping and that the defendant
assumed 'full responsibility for all cash received by the Postmaster.' Here the responsibility of the
defendant is not just that of a guarantor. It is clearly that of a principal."

217 | P a g e
Law 321_Corporation LAW_ Case Digest

REPUBLIC OF THE PHILIPPINES


vs.
SECURITY CREDIT AND ACCEPTANCE CORPORATION, ROSENDO T. RESUELLO, PABLO
TANJUTCO, ARTURO SORIANO, RUBEN BELTRAN, BIENVENIDO V. ZAPA, PILAR G. RESUELLO,
RICARDO D. BALATBAT, JOSE SEBASTIAN and VITO TANJUTCO JR.
G.R. No. L-20583. January 23, 1967

FACTS:

The Articles of Incorporation of defendant corporation were registered with the Securities and
Exchange Commission on March 27, 1961. Thereafter, the Board of Directors of the corporation adopted
a set of by-laws which were filed with said Commission on April 5, 1961
On September 19, 1961, the Superintendent of Banks of the Central Bank of the Philippines
asked its legal counsel an opinion on whether or not said corporation is a banking institution, within the
purview of Republic Act No. 337; that, acting upon this request, on October 11, 1961, said legal counsel
rendered an opinion resolving the query in the affirmative
On March 9, 1961, the corporation had applied with the Securities and Exchange Commission for
the registration and licensing of its securities under the Securities Act. However, SCAC’s registration of
its Articles of Incorporation was denied on the ground that it has not complied with the requirements
under the General Banking Act (RA No. 337). Later, a Search Warrant was issued against SCAC where
documents and records relative to its business operation were seized.
Even when SCAC was duly advised of the findings, SCAC and its BOD and Officers still continued
operations prompting the Solicitor General to file a quo warranto proceedings for the dissolution of
SCAC.

ISSUE:

Whether or not SCAC was illegally engaged in the business of banking.

RULING:

YES.

In dissolving SCAC, the Court held that the corporation was indeed engaged in the business of
banking without first securing the administrative authority required by RA No. 337.
Although, admittedly, SCAC has not secured the requisite authority to engage in banking,
defendants deny that its transactions partake of the nature of banking operations. Note however that, in
consequence of their propaganda campaign, a total of 59,463 savings account deposits have been made
by the public with SCAC and its 74 branches, with an aggregate deposit of P1,689,136.74, which has
been lent out to such persons as SCAC deemed suitable. It is clear that these transactions partake of
the nature of banking, as the term is used in Section 2 of RA No. 337. Indeed, a bank has been defined
as: A moneyed institute founded to facilitate the borrowing, lending, and safe-keeping of money and to
deal in notes, bills of exchange, and credits; an investment company which loans out the money of its
customers, collects the interests, and charges a commission to both lender and borrower is a bank; any
person engaged in the business carried on by banks of deposit, of discount, or of circulation is doing a
banking business, although but one of these functions is exercised.
The illegal transactions thus undertaken by SCAC to warrant its dissolution is apparent from the
fact that the foregoing misuser of the corporate funds and franchise affects the essence of its business,
that it is willful and has been repeated 59,643 times, and that its continuance inflicts injury upon the
public, owing to the number of persons affected thereby.

BY-LAWS
 Function

DILY DANY NACPIL


vs.

218 | P a g e
Law 321_Corporation LAW_ Case Digest

INTERNATIONAL BROADCASTING CORPORATION


G.R. No. 144767. March 21, 2002

FACTS:

Petitioner was Assistant General Manager for Finance/Administration and Comptroller of private
respondent Intercontinental Broadcasting Corporation (IBC) from 1996 until April 1997. According to
petitioner, when Emiliano Templo was appointed to replace IBC President Tomas Gomez III sometime in
March 1997, the former told the Board of Directors that as soon as he assumes the IBC presidency, he
would terminate the services of petitioner. Apparently, Templo blamed petitioner, along with a certain
Mr. Basilio and Mr. Gomez, for the prior mismanagement of IBC. Upon his assumption of the IBC
presidency, Templo allegedly harassed, insulted, humiliated and pressured petitioner into resigning
until the latter was forced to retire. However, Templo refused to pay him his retirement benefits,
allegedly because he had not yet secured the clearances from the Presidential Commission on Good
Government and the Commission on Audit.
IBC filed a motion to dismiss contending that petitioner was a corporate officer who was duly
elected by the Board of Directors of IBC; hence, the case qualifies as an intra-corporate dispute falling
within the jurisdiction of the SEC.
On the other hand, petitioner argues that he is not a corporate officer of IBC but an employee
thereof since he had not been elected nor appointed as Comptroller and Assistant Manager by the IBC's
Board of Directors but by an IBC General Manager. This is also because the IBC's By-Laws do not even
include the position of comptroller in its roster of corporate officers.He therefore contends that his
dismissal is a controversy falling within the jurisdiction of the labor courts.

ISSUE:

Whether or not petitioner is a corporate officer although the position of comptroller is not
expressly mentioned in the by-laws.

RULING:

NO.

The fact that the position of Comptroller is not expressly mentioned among the officers of the IBC
in the By-Laws is of no moment, because the IBC's Board of Directors is empowered under Section 25 of
the Corporation Code and under the corporation's By-Laws to appoint such other officers as it may
deem necessary.
The by-laws may and usually do provide for such other officers," and that where a corporate office
is not specifically indicated in the roster of corporate offices in the by-laws of a corporation, the board of
directors may also be empowered under the by-lawsto create additional officers as may be necessary.
Furthermore, as petitioner's appointment as comptroller required the approval and formal action of the
IBC's Board of Directors to become valid, it is clear therefore holds that petitioner is a corporate officer
whose dismissal may be the subject of a controversy cognizable by the SEC under Section 5(c) of P.D.
902-A which includes controversies involving both election and appointmentof corporate directors,
trustees, officers, and managers. Had petitioner been an ordinary employee, such board action would
not have been required.

PMI COLLEGES
vs.
THE NATIONAL LABOR RELATIONS COMMISSION and ALEJANDRO GA LVA N
G.R. No. 121466. August 15, 1997

FACTS:

On July 7, 1991, petitioner, an educational institution offering courses on basic seaman's


training and other marine-related courses, hired private respondent as contractual instructor with an
agreement that the latter shall be paid at an hourly rate of P30.00 to P50.00, depending on the
description of load subjects and on the schedule for teaching the same. Pursuant to this engagement,
private respondent then organized classes in marine engineering.

219 | P a g e
Law 321_Corporation LAW_ Case Digest

Initially, private respondent and other instructors were compensated for services rendered during
the first three periods of the abovementioned contract. However, for reasons unknown to private
respondent, he stopped receiving payment for the succeeding rendition of services. This claim of non-
payment was embodied in a letter dated March 3, 1992, written by petitioner's Acting Director, Casimiro
A. Aguinaldo, addressed to its President, Atty. Santiago Pastor, calling attention to and appealing for the
early approval and release of the salaries of its instructors including that of private respondent.
Private respondent's claims, were resisted by petitioner. Later in the proceedings, PMI Colleges
manifested that Mr. Tomas Cloma Jr., a member of the board of trustees write a letter to the Chairman
of the Board, clarifying the case of Galvan and stating therein, inter alia, that under PMI’s by-laws only
the Chairman is authorized to sign any contract and that Galvan, in any event, failed to submit
documents on the alleged shipyard and plant visits in Cavite Naval Base.

ISSUE:

Whether or not the contract of employment of Galvan valid even if the signatory therein was not
the Chairman of the Board.

RULING:

YES.

The contract of employment is valid. The contract remained valid even if the signatory thereon
was not the chairman of the board which allegedly violated petitioner’s by-laws. Since by-laws operate
merely as internal rules among the stockholders, they cannot affect or prejudice third persons who deal
with the corporation, unless they have knowledge of the same. No proof appears on record that private
respondent ever knew anything about the provisions of the said by-laws. In fact, petitioner itself merely
asserts the same without even bothering to attach a copy or excerpt thereof to show that there is such
provision. That this allegation has never been denied to private respondent nor necessarily signify
admission of its existence because technicalities of law and procedure and the rules obtaining in the
courts of law do not strictly apply to proceeding of this nature.

LOYOLA GRAND VILLAS HOMEOWNERS (SOUTH) ASSOCIATION, INC.


vs.
CA, HOME INSURANCE AND GUARANTY CORPORATION, EMDEN ENCARNACION and HORATIO
AYCARDO
G.R. No. 117188 August 7, 1997

FACTS:

Loyola Grand Villas Homeowners Association (LGVHA) was organized on February 8, 1983 as the
association of homeowners and residents of the Loyola Grand Villas. It was registered with the Home
Financing Corporation, the predecessor of herein respondent Home Insurance and Guaranty
Corporation (HIGC), as the sole homeowners' organization in the said subdivision. It was organized by
the developer of the subdivision and its first president was Victorio V. Soliven, himself the owner of the
developer. For unknown reasons, however, LGVHAI did not file its corporate by-laws.
Sometime in 1988, the officers of the LGVHAI tried to register its by-laws but failed to do so. Then the
officers that there were two other organizations within the subdivision the Loyola Grand Villas
homeowners North Association Incorporated (North Association) and the Loyola Grand Villas
homeowners South Association Incorporated (South Association). According to private respondents, a
non-resident and Soliven himself, respectively headed these associations. They also discovered that
these associations had five (5) registered homeowners each who were also the incorporators, directors

220 | P a g e
Law 321_Corporation LAW_ Case Digest

and officers thereof. None of the members of the LGVHAI was listed as member of the North Association
while three (3) members of LGVHAI were listed as members of the South Association. When Soliven
inquired about the status of LGVHAI, Atty. Joaquin A. Bautista, the head of the legal department of the
HIGC, informed him that LGVHAI had been automatically dissolved because it did not submit its by-
laws within the period required by the Corporation Code and there was non-user of corporate charter
because HIGC had not received any report on the association's activities. Apparently, this information
resulted in the registration of the North and South Association.

ISSUE:

Whether or not failure of LGVHAI to file its by-laws within one month from the date of its
incorporation result in its automatic dissolution.

RULING:

NO.

The Supreme Court ruled that the non-filing of the by-laws within the period of 1 month from the
issuance by SEC of the Certificate of Incorporation will not result to the automatic dissolution of the
corporation because the word “MUST” in Sec 46 of the Corporation Code is merely directory not
mandatory in meaning. In fact the second paragraph allows the filing of by-laws even prior to
incorporation.
This provision of the Code rules out mandatory compliance with the requirement of filing the by-
laws "within one (1) month after receipt of official notice of the issuance of its certificate of incorporation
by the Securities and Exchange Commission." It necessarily follows that failure to file the by-laws within
that period does not imply the "demise" of the corporation. By-laws may be necessary for the
"government" of the corporation but these are subordinate to the articles of incorporation as well as to
the Corporation Code and related statutes.

CITIBANK, N.A.
vs.
HON. SEGUNDINO G. CHUA, SANTIAGO M. KAPUNAN and LUIS L. VICTOR, ASSOCIATE JUSTICES
OF THE HON. COURT OF APPEALS, THIRD DIVISION, MANILA, HON. LEONARDO B. CANARES,
Judge of Regional, Trial Court of Cebu, Branch 10, and SPOUSES CRESENCIO AND ZENAIDA
VELEZ
G.R. No. 102300. March 17, 1993

FACTS:

Citibank is a foreign commercial banking corporation duly licensed to do business in the


Philippines. Private respondents, spouses Cresencio and Zenaida Velez, who were good clients alleged
that the petitioner bank extended to them credit lines sufficiently secured with real estate and chattel
mortgages on equipment. They claim that a restructuring agreement has been entered into between
them and the bank. However, the bank failed to comply thereto thus spouses Velez sued for specific
performance and damages.
On March 30, 1990, the date of the pre-trial conference, counsel for petitioner bank appeared,
presenting a special power of attorney executed by Citibank officer Florencia Tarriela in favor of
petitioner bank's counsel, the J.P. Garcia & Associates, to represent and bind petitioner bank at the pre-
trial conference of the case at bar. Inspite of this special power of attorney, counsel for spouses Velez
orally moved to declare petitioner bank as in default on the ground that the special power of attorney
was not executed by the Board of Directors of Citibank. Thus petitioner bank executed another special
power of attorney made by William W. Ferguson, Vice President and highest ranking officer of Citibank,
Philippines, constituting and appointing the J.P. Garcia & Associates to represent and bind the BANK.
Unsatisfied, private respondents moved again for declaration of default. Though the bank again
executed anotherspecial power of attorney through William W. Ferguson in favor of Citibank employees,
the court issued an order declaring petitioner bank as in default. The CA dismissed the petition filed by

221 | P a g e
Law 321_Corporation LAW_ Case Digest

the bank. The CA relied on Section 46 of the Corporation Code to support its conclusion that the by-
laws in question are without effect because they were not approved by the SEC.

ISSUE:

Whether or not petitioner bank's by-laws, which constitute the basis for Ferguson's special power
of attorney in favor of petitioner bank's legal counsel are effective, considering that petitioner bank has
been previously granted a license to do business in the Philippines.

RULING:

YES.

Section 46 (which was relied upon by the CA) starts with the phrase "Every corporation formed
under this Code", which can only refer to corporations incorporated in the Philippines. Hence, Section
46, in so far as it refers to the effectivity of corporate by-laws, applies only to domestic corporations and
not to foreign corporations. On the other hand, Section 125 of the same Code requires that a foreign
corporation applying for a license to transact business in the Philippines must submit, among other
documents, to the SEC, a copy of its articles of incorporation and by-laws, certified in accordance with
law. Unless these documents are submitted, the application cannot be acted upon by the SEC.
Since the SEC will grant a license only when the foreign corporation has complied with all the
requirements of law, it follows that when it decides to issue such license, it is satisfied that the
applicant's by-laws, among the other documents, meet the legal requirements. This, in effect, is an
approval of the foreign corporations’ by-laws. It may not have been made in express terms; still it is
clearly an approval. Therefore, petitioner bank's by-laws, though originating from a foreign jurisdiction,
are valid and effective in the Philippines.

 When to Adopt and File

LOYOLA GRAND VILLAS HOMEOWNERS (SOUTH) ASSOCIATION, INC.


vs.
CA, HOME INSURANCE AND GUARANTY CORPORATION, EMDEN ENCARNACION and HORATIO
AYCARDO
G.R. No. 117188 August 7, 1997

FACTS:

Loyola Grand Villas Homeowners Association (LGVHA) was organized on February 8, 1983 as the
association of homeowners and residents of the Loyola Grand Villas. It was registered with the Home
Financing Corporation, the predecessor of herein respondent Home Insurance and Guaranty
Corporation (HIGC), as the sole homeowners' organization in the said subdivision. It was organized by
the developer of the subdivision and its first president was Victorio V. Soliven, himself the owner of the
developer. For unknown reasons, however, LGVHAI did not file its corporate by-laws.
Sometime in 1988, the officers of the LGVHAI tried to register its by-laws but failed to do so. Then
the officers that there were two other organizations within the subdivision the Loyola Grand Villas
homeowners North Association Incorporated (North Association) and the Loyola Grand Villas
homeowners South Association Incorporated (South Association). According to private respondents, a
non-resident and Soliven himself, respectively headed these associations. They also discovered that
these associations had five (5) registered homeowners each who were also the incorporators, directors
and officers thereof. None of the members of the LGVHAI was listed as member of the North Association
while three (3) members of LGVHAI were listed as members of the South Association. When Soliven
inquired about the status of LGVHAI, Atty. Joaquin A. Bautista, the head of the legal department of the
HIGC, informed him that LGVHAI had been automatically dissolved because it did not submit its by-
laws within the period required by the Corporation Code and there was non-user of corporate charter
because HIGC had not received any report on the association's activities. Apparently, this information
resulted in the registration of the North and South Association.

ISSUE:

Whether or not failure of LGVHAI to file its by-laws within one month from the date of its
incorporation result in its automatic dissolution.

RULING:

222 | P a g e
Law 321_Corporation LAW_ Case Digest

NO.

The Supreme Court ruled that the non-filing of the by-laws within the period of 1 month from the
issuance by SEC of the Certificate of Incorporation will not result to the automatic dissolution of the
corporation because the word “MUST” in Sec 46 of the Corporation Code is merely directory not
mandatory in meaning. In fact the second paragraph allows the filing of by-laws even prior to
incorporation.
This provision of the Code rules out mandatory compliance with the requirement of filing the by-
laws "within one (1) month after receipt of official notice of the issuance of its certificate of incorporation
by the Securities and Exchange Commission." It necessarily follows that failure to file the by-laws within
that period does not imply the "demise" of the corporation. By-laws may be necessary for the
"government" of the corporation but these are subordinate to the articles of incorporation as well as to
the Corporation Code and related statutes.

 Authority to Elect Additional By-Laws Officers

HENRY FLEISCHER
vs.
BOTICA NOLASCO CO., INC.
G.R. No. L-23241. March 14, 1925

FACTS:

On November 15, 1923, the plaintiff filed an amended complaint against the Botica Nolasco, Inc.,
alleging that he became the owner of five shares of stock of said corporation, by purchase from their
original owner, one Manuel Gonzalez; that the said shares were fully paid; and that the defendant
refused to register said shares in his name in the books of the corporation in spite of repeated demands
to that effect made by him upon said corporation, which refusal caused him damages amounting to
P500. The defendant filed a demurrer on the ground that the amended complaint did not state facts
sufficient to constitute a cause of action, and that said amended complaint was ambiguous,
unintelligible, uncertain, which demurrer was overruled by the court.
The defendant answered the amended complaint denying generally and specifically each and
every one of the material allegations thereof, and, as a special defense, alleged that the defendant,
pursuant to article 12 of its by-laws, had preferential right to buy from the plaintiff said shares at the
par value of P100 a share, plus P90 as dividends corresponding to the year 1922, and that said offer
was refused by the plaintiff. The defendant prayed for a judgment absolving it from all liability under the
complaint and directing the plaintiff to deliver to the defendant the five shares of stock in question, and
to pay damages.

ISSUE:

Whether or not article 12 of the by-laws of the corporation is in conflict with the provisions of the
Corporation Law (Act No. 1459).

RULING:

YES.

The holder of shares, as owner of personal property, is at liberty, under said section, to dispose of
them in favor of whomsoever he pleases, without any other limitation in this respect, than the general
provisions of law.
Therefore, a stock corporation in adopting a by-law governing transfer of shares of stock should
take into consideration the specific provisions of section 35 of Act No. 1459, and said by-law should be
made to harmonize with said provisions. It should not be inconsistent therewith.
The by-law now in question was adopted under the power conferred upon the corporation by
section 13, paragraph 7, above quoted; but in adopting said by-law the corporation has transcended the
limits fixed by law in the same section, and has not taken into consideration the provisions of section 35
of Act No. 1459.

223 | P a g e
Law 321_Corporation LAW_ Case Digest

As a general rule, the by-laws of a corporation are valid if they are reasonable and calculated to
carry into effect the objects of the corporation, and are not contradictory to the general policy of the laws
of the land
The only restraint imposed by the Corporation Law upon transfer of shares is found in section 35
of Act No. 1459, quoted above, as follows: "No transfer, however, shall be valid, except as between the
parties, until the transfer is entered and noted upon the books of the corporation so as to show the
names of the parties to the transaction, the date of the transfer, the number of the certificate, and the
number of shares transferred." This restriction is necessary in order that the officers of the corporation
may know who are the stockholders, which is essential in conducting elections of officers, in calling
meeting of stockholders, and for other purposes. but any restriction of the nature of that imposed in the
by-law now in question, is ultra vires, violative of the property rights of shareholders, and in restraint of
trade.
JOHN GOKONGWEI, JR., petitioner
vs.
SECURITIES AND EXCHANGE COMMISSION, ANDRES M. SORIANO et.al. respondents.
G.R. No. L-45911 April 11, 1979.

FACTS:

Petitioner alleged that on September 18, 1976, individual respondents amended the by-laws of
the corporation, basing their authority to do so on a resolution of the stockholders adopted on March
13, 1961. It was contended that according to section 22 of the Corporation Law and Article VIII of the
by-laws of the corporation, the power to amend, modify, repeal or adopt new by-laws may be delegated
to the Board of Directors only by the affirmative vote of stockholders representing not less than 2/3 of
the subscribed and paid up capital stock of the corporation, which 2/3 should have been computed on
the basis of the capitalization at the time of the amendment. Since the amendment was based on the
1961 authorization, petitioner contended that the Board acted without authority and in usurpation of
the power of the stockholders. Petitioner averred that the membership of the Board of Directors had
changed since the authority was given in 1961, there being six (6) new directors.
It was claimed that prior to the questioned March 13, 1961 amendment, petitioner had all the
qualifications to be a director of respondent corporation, being a substantial stockholder thereof; that as
a stockholder, petitioner had acquired rights inherent in stock ownership, such as the rights to vote and
to be voted upon in the election of directors; and that in amending the by-laws, respondents purposely
provided for petitioner's disqualification and deprived him of his vested right as afore-mentioned, hence
the amended by-laws are null and void.

ISSUE:

Whether or not the disqualification of Gokongwei Jr. to run for directorship of the corporation
valid, as such was only provided in the amended by-laws of the corporation.

RULING:

YES.

It is recognized by all authorities that 'every corporation has the inherent power to adopt by-laws
'for its internal government, and to regulate the conduct and prescribe the rights and duties of its
members towards itself and among themselves in reference to the management of its affairs.'" At
common law, the rule was "that the power to make and adopt by-laws was inherent in every corporation
as one of its necessary and inseparable legal incidents.
Any person "who buys stock in a corporation does so with the knowledge that its affairs are
dominated by a majority of the stockholders and that he impliedly contracts that the will of the majority
shall govern in all matters within the limits of the act of incorporation and lawfully enacted by-laws and
not forbidden by law."
Under section 22 of the same law, the owners of the majority of the subscribed capital stock may
amend or repeal any by-law or adopt new by-laws. It cannot be said, therefore, that petitioner has a
vested right to be elected director, in the face of the fact that the law at the time such right as
stockholder was acquired contained the prescription that the corporate charter and the by-law shall be
subject to amendment, alteration and modification.
It is a settled that corporations have the power to make by-laws declaring a person employed in
the service of a rival company to be ineligible for the corporation's Board of Directors. ".An amendment
which renders ineligible, or if elected, subjects to removal, a director if he be also a director in a
corporation whose business is in competition with or is antagonistic to the other corporation is valid."

224 | P a g e
Law 321_Corporation LAW_ Case Digest

THE GOVERNMENT OF THE PHILIPPINE ISLANDS (on relation of the Attorney-General)


vs.
EL HOGAR FILIPINO
G.R. No. L-26649 July 13, 1927

FACTS:

This case has 17 causes of action proceeded by the Government of the Philippines through Quo
Warranto alleging that El Hogar Filipino, a corporation organized as a mutual building and loan
association under the provisions of the Corporation Law, has violated or went beyond its stated primary
purposes for mutual building and loan associations. Under Corporation Law Section 171 to 190,
inclusive, of this Act are devoted to the subject of building and loan associations, defining their objects
making various provisions governing their organization and administration, and providing for the
supervision to be exercised over them.. The respondent, El Hogar Filipino, was apparently the first
corporation organized in the Philippine Islands under the provisions cited, and the association has been
favored with extraordinary success. The articles of incorporation bear the date of December 28, 1910, at
which time capital stock in the association had been subscribed to the amount of P150,000 of which the
sum of P10,620 had been paid in. Under the law as it then stood, the capital of the Association was not
permitted to exceed P3,000,000, but by Act No. 2092, passed December 23, 1911, the statute was so
amended as to permit the capitalization of building and loan associations to the amount of ten millions.
Soon thereafter the association took advantage of this enactment by amending its articles so as to
provide that the capital should be in an amount not exceeding the then lawful limit. From the time of its
first organization the number of shareholders has constantly increased, with the result that on
December 31, 1925, the association had 5,826 shareholders holding 125,750 shares, with a total paid-
up value of P8,703,602.25. During the period of its existence prior to the date last above-mentioned the
association paid to withdrawing stockholders the amount of P7,618,257,.72; and in the same period it
distributed in the form of dividends among its stockholders the sum of P7,621,565.81.
As one of the causes of action, the respondent is charged with having a provision in its by-laws
stating that “The board of directors of the association, by the vote of an absolute majority of its
members, is empowered to cancel shares and to return to the owner thereof the balance resulting from
the liquidation thereof whenever, by reason of their conduct, or for any other motive, the continuation as
members of the owners of such shares is not desirable”.

ISSUE:

Whether or not the provision of the by-laws valid.

RULING:

YES.

The by-law is of course a patent nullity, since it is in direct conflict with the latter part of section
187 of the Corporation Law, which expressly declares that the board of directors shall not have the
power to force the surrender and withdrawal of unmatured stock except in case of liquidation of the
corporation or of forfeiture of the stock for delinquency. It is agreed that this provision of the by-laws
has never been enforced, and in fact no attempt has ever been made by the board of directors to make
use of the power therein conferred. It appears, however, that no annual meeting of the shareholders
called since that date has been attended by a sufficient number of shareholders to constitute a quorum,
with the result that the provision referred to has no been eliminated from the by-laws, and it still stands
among the by-laws of the association, notwithstanding its patent conflict with the law.

 Amendment and/or Rejection of By Laws

ENRIQUE SALAFRANCA
vs.
PHILAMLIFE (PVHA VILLAGE, HOMEOWNERS ASSOCIATION, INC.,et al., respondents.
G.R. No. 121791 December 23, 1998

FACTS:

225 | P a g e
Law 321_Corporation LAW_ Case Digest

Salafranca was hired as Administrative Officer by PVHA on May 1, 1981 and was extended
successive appointments. Sometime in 1987, PVHA decided to amend its by-laws. Included therein was
a provision regarding officers, specifically, the position of administrative officer under which said officer
shall hold office at the pleasure of the Board of Directors. In a letter dated December 7, 1992, PVHA
and Dazo informed Salafranca that they had decided to discontinue his services. Claiming that his
services had been unlawfully and unceremoniously dispensed with, Salafranca filed a complaint for
illegal dismissal with money claims and for damages.
The LA held that respondents’ contention that complainant’s term of employment was co-
terminous with the term of Office of the Board of Directors, is wanting in merit. The 1987 Amendment
would not be applicable to the case of complainant who had become a regular employee long time before
the Amendment took place. Moreover, the Amendment should be applied prospectively and not
retroactively. On appeal, the NLRC reversed the decision of the LA.

ISSUE:

Whether or not Salafranca was legally dismissed by private respondents pursuant to the 1987
amendment in the By-laws.

RULING:

NO.

Salafranca had already attained the status of a regular employee, as evidenced by his eleven
years of service with PVHA. Accordingly, petitioner enjoys the right to security of tenure and his services
may be terminated only for causes provided by law. While PVHA has the right to terminate the services
of Salafranca, this is subject to both substantive and procedural grounds. PVHA failed to substantiate
petitioner’s dismissal, rendering the latter’s termination illegal.
In an effort to validate the dismissal of Salafranca, respondents posit the theory that the latter’s
position is co-terminous with that of the Board of Directors, as provided for in its amended by-laws.
Admittedly, the right to amend the by-laws lies solely in the discretion of the employer, this being in the
exercise of management prerogative or business judgment. However this right, extensive as it may be,
cannot impair the obligation of existing contracts or rights.
PVHA’s insistence that it can legally dismiss Salafranca on the ground that his tenure has expired
is untenable. Salaranca, being a regular employee, is entitled to security of tenure; hence, his services
may only be terminated for causes provided by law. A contrary interpretation would not find
justification in the laws or the Constitution. If the Court was to rule otherwise, it would enable an
employer to remove any employee from his employment by the simple expediency of amending its by-
laws and providing that his/her position shall cease to exist upon the occurrence of a specified event.
If PVHA wanted to make the Salafranca’s position co-terminous with that of the Board of
Directors, then the amendment must be effective after Salafranca’s stay with PVHA, not during his term.
Obviously, the measure taken by the private respondent in amending its by-laws is nothing but a
devious, but crude, attempt to circumvent Salafranca’s right to security of tenure as a regular employee
guaranteed under the Labor Code.

MEETINGS OF STOCKHOLDERS AND THE BOARD OF DIRECTORS


 Notice Required

ROSITA PEÑA
vs.
COURT OF APPEALS
G.R. No. 91478 February 7, 1991

FACTS:

Pampanga Bus Co. (PAMBUSCO), original owners of the lots in question, mortgaged the same to
the Development Bank of the Philippines (DBP) in consideration of P935,000.00. This mortgage was
foreclosed. In the foreclosure, the said properties were awarded to Peña as highest bidder. Thereafter,
the board of directors of PAMBUSCO, through (3) out of its (5) directors, resolved to assign its right of
redemption over the aforesaid lots and authorized one of its members, Atty. Joaquin Briones "to execute
and sign a Deed of Assignment for and in behalf of PAMBUSCO in favor of any interested party.
Consequently, Briones executed a Deed of Assignment of PAMBUSCO's redemption right over the
subject lots in favor of Enriquez. Thereafter, Enriquez executed a deed of absolute sale of the subject

226 | P a g e
Law 321_Corporation LAW_ Case Digest

properties in favor of plaintiffs-appellants, the spouses Rising T. Yap and Catalina Lugue, for the sum of
P140,000.00.
Plaintiffs-appellants, the spouses Rising T. Yap and Catalina Lugue, are the registered owners of
the lots in question. In the complaint filed, appellants sought to recover possession over the subject
lands from defendants Rosita Peña and Washington Distillery on the ground that being registered
owners, they have to enforce their right to possession against defendants who have been allegedly in
unlawful possession thereof since October 1974 "when the previous owners assigned (their) right to
collect rentals in favor of plaintiffs. After trial, a decision was rendered by the court in favor of the
defendants.

ISSUE:

Whether or not the board resolution of PAMBUSCO is valid.

RULING:

NO.

Under Section 25 of the Corporation Code of the Philippines, the articles of incorporation or by-
laws of the corporation may fix a greater number than the majority of the number of board members to
constitute the quorum necessary for the valid transaction of business. Any number less than the
number provided cannot constitute a quorum and any act therein would not bind the corporation; all
that the attending directors could do is to adjourn.
Records show that PAMBUSCO ceased to operate as of November 15, 1949. Being a dormant
corporation for several years, it was highly irregular, if not anomalous, for a group of three (3)
individuals representing themselves to be the directors of PAMBUSCO to pass a resolution disposing of
the only remaining asset of the corporation in favor of a former corporate officer. As a matter of fact, the
three (3) alleged directors who attended the said meeting were not listed as directors of respondent
PAMBUSCO. Furthermore, PAMBUSCO was insolvent and its only remaining asset was its right of
redemption over the subject properties. Since the disposition of said redemption right of respondent
PAMBUSCO by virtue of the questioned resolution was not approved by the required number of
stockholders under the law, the said resolution, as well as the subsequent assignment to respondent
Enriquez should be struck down as null and void.

 Quorum Required

THE BOARD OF DIRECTORS And ELECTION COMMITTEE OF THE SMB WORKERS SAVINGS AND
LOAN ASSOCIATION, INC., ET AL.
vs.
HON BIENVENIDO A. TAN, ETC., ET AL.
G.R. No. L-12282 1959 March 31

FACTS:

On January 17, 1957 John de Castillo et al., commenced a suit in the Court of First Instance of
Manila to declare null and void the election of the members of the board of directors of the SMB Workers
Savings and Loan Association, Inc. and of the members of the Election Committee for the year 1957 held
on January 11 and 12 and to compel the board of directors of the association to call for and hold
another election in accordance with its constitution and by-laws and the Corporation Law. Such was
granted by the court, however, another suit was filed alleging that the subsequent meeting for the
elections would not be in accordance with the constitution and by-laws regarding notice to the
stockholders.

ISSUE:

Whether or not proper notice was given as regards the new meeting for the elections of the board
of directors.

RULING:

227 | P a g e
Law 321_Corporation LAW_ Case Digest

NO.

There was no proper notice. Notice of a special meeting of members should be given at least five
days before the date of the meeting. It appears that the notice was posted on 26 March and the election
was set for 28 March. Therefore, the five days previous notice required would not be complied with.

 Who Presides

ABELARDO JAVELLANA, TOMAS JONCO, et al., in their capacities as Councilors of the Municipal
Municipality of Buenavista, Province of Iloilo
vs.
SUSANO TAYO, as Mayor of the Municipal Municipality of Buenavista, Iloilo
G.R. No. L-18919 December 29, 1962

FACTS:

Petitioners were members of the municipal council. On several sessions, the mayor, herein
defendant, was absent prompting the council to decide emong themselves as to who to appoint as
presiding officers. The mayor refused to act on the resulting minutes also refused to sign the payrolls of
the council covering the per diems of the petitioners, alleging that the proceedings were illegal due to his
absence.
Despite the Provincial Fiscal and the Provincial Board upholding the controverted sessions of the
Municipal Council, the Mayor refused and still refuses to recognize the validity of the acts of the
Municipal Council and the legality of its regular session held in his absence.
The trial court ruled that attendance of the Mayor is not essential to the validity of the session as
long as there is quorum constituted in accordance with law. To declare that the proceedings of the
petitioners were null and void, is to encourage recalcitrant public officials who would frustrate valid
sessions for political end or consideration.

ISSUE:

Whether or not the sessions held by petitioners were valid and legal, having constituted a
quorum, and despite the absence of the defendant.

RULING:

228 | P a g e
Law 321_Corporation LAW_ Case Digest

YES.

The term "quorum" has been defined as "that number of members of the body which, when legally
assembled in their proper places, will enable the body to transact its proper business, or, in other
words, that number that makes a lawful body and gives it power to pass a law or ordinance or do any
other valid corporate act.
The Revised Administrative Code states that for the majority of the members of the council to
constitute a quorum to do business, the council "shall be presided by the Mayor and no one else.
The procedure, as provided in the Administrative Code, provides that in case of temporary
incapacity of the mayor, the council member having the highest number of votes can sit as presiding
officer. This rule on incapacity was declared as valid by the court in the case. Thus, the quorum
requirement was satisfied despite the continuous absence of the mayor on those scheduled sessions.
Thus, the questioned sessions and the resulting resolutions were declared valid.

 Who Could Attend and Vote

JULIO E. T. SALES and GEORGE V. AGONIAS and SMEC


vs.
SEC, SIHI and ATCO, represented by its President, ANSELMO TRINIDAD; VIMC, represented by its
President, et al.
G.R. No. 54330 January 13, 1989

FACTS:

SMEC sold 200M common shares of its capital stock in the amount of P2.6M to SIHI under a
Sales Agreement providing that the sale shall be only up to 5m shares per buyer. SIHI requested for the
transfer of the 200M shares to ATCO to which SMEC complied. During the time that ATCO held the
shares, it voted them in the SHs' meetings of SMEC. ATCO in turn sold 198,500,000 of the shares to
respondent VIMC. Upon request, SMEC BOD issued a resolution directing its President to sign the
certificate of stock that would effect the transfer. Before the 1979 annual SH meeting of SMEC,
petitioners sought to nullify the sales of the shares to VIMC with the SEC and to enjoin VIMC from
voting the said shares. VIMC was temporarily restrained and the meeting was held without the
participation of VIMC’s shares and BODs were elected only from the group of petitioners. In VIMC’s
answer, it questioned the said election. SEC denied the petition as well as motion to dismiss and lifted
the Restraining it issued earlier and allowed the shares of VIMC to be counted in determining the
quorum of the 1980 annual SHs meeting, which was already near, and the same shares were allowed to
vote and be voted for. Before the SC, petitioners contended that the SEC gravely abused its discretion in
not enjoining the participation of VIMC in the 1980 election considering that the sale of the shares to
VIMC was null and void as it was done in violation of the Sales Agreement on the limit of shares to be
sold to each buyer and that VIMC’s ownership of the shares is contrary to Sec. 13 (5-A) of the old
corporation law.

ISSUE:

Whether or not SEC acted with grave abuse of discretion in not permanently enjoining VIMC in
voting.

RULING:

229 | P a g e
Law 321_Corporation LAW_ Case Digest

NO.

SC found no grave abuse of discretion on the part of the SEC in not restraining VIMC. It adopted
the SEC resolution stating that the sale of the shares of stock had long been perfected and is presumed
valid until declared otherwise. As against this presumption, petitioners' prayer for injunction cannot
prevail as the issue of the validity of the sale is still to be resolved by the SEC.
Considering that the shares constitute the majority, it is more equitable that the same be allowed
to vote rather than be enjoined. As it has been ruled the removal of a majority SH from the management
of the corporation and/or the dissolution of a corporation in a suit filed by a minority SH is a drastic
measure. It should be resorted to only when the necessity is clear. With more reason, the Court will not
deprive a SH of his right to vote his shares in the annual SHs' meeting, except upon a clear showing of
its lawful denial under the articles of incorporation or by-laws of the corporation, as it is a right inherent
in stock ownership.

DOMINGO PONCE AND BUHAY PONCE


vs.
DEMETRIO B. ENCARNACION AND POTENCIANO GAPOL
G.R. NO. L-5883 NOVEMBER 28, 1953

FACTS:

Daguhoy Enterprises, Inc., was duly registered as such on 24 June 1948. On 16 April 1951 at a
meeting duly called, the voluntary dissolution of the corporation and the appointment of Gapol as
receiver were agreed upon and to that end a petition for voluntary dissolution was drafted which was
sent to, and signed by, the petitioner Domingo Ponce. Instead of filing the petition for voluntary
dissolution of the corporation as agreed upon, Gapol, who is the largest stockholder, changed his mind
and filed a complaint in the CFI of Manila to compel the petitioners to render an accounting of the funds
and assets of the corporation, to reimburse it, jointly and severally, a total sum of P18,690, plus
interest, which have been converted by the petitioner Domingo Ponce to his own use and benefit.
On 18 May 1951 Gapol filed a motion praying that the petitioners be removed as members of the
board of directors which was denied by the court. On 3 January 1952 Gapol filed a petition praying for
an order directing him to call a meeting of the stockholders of the corporation and to preside at such
meeting in accordance with section 26 of the Corporation Law. Two-days later, without notice to the
petitioners and to the other members of the board of directors and in violation of the Rules of Court
which require that the adverse parties be notified of the hearing of the motion three days in advance, the
respondent court issued the order as prayed for.

ISSUE:

Whether or not under and pursuant to section 26 of the Corporation Law, the respondent court
may issue the order complained of.

RULING:

NO.

Article 9 of the by-laws of the Daguhoy Enterprises, Inc., provides: The Board of Directors shall
compose of five (5) members who shall be elected by the stockholders in a general meeting called for that
purpose which shall be held every even year during the month of January. Article 22 of the by-laws
provides: The Chairman shall have the right to fix the date, the time and the place where the general
meeting shall be held, either special or general.
Section 26 of the Corporation Code provides: - Whenever, from any cause, there is no person
authorized to call a meeting, or when the officer authorized to do so refuses, fails, or neglects to call a
meeting, any judge of a Court of First Instance, on the showing of good cause therefor, may issue an
order to any stockholder or member of a corporation, directing him to call a meeting of the corporation
by giving the proper notice required by this Act or the by-laws; and if there be no person legally
authorized to preside at such meeting, the judge of the Court of First Instance may direct the person

230 | P a g e
Law 321_Corporation LAW_ Case Digest

calling the meeting to preside at the same until a majority of the members or stockholders representing
a majority of the stock present and permitted by law to be voted have chosen one of their number to act
as presiding officer for the purposes of the meeting.
Petitioners were not deprived of their right without due process of law. They had no right to
continue as directors of the corporation unless reelected by the stockholders in a meeting called for that
purpose every even year.

SALVADOR P. LOPEZ
vs.
ERICTA
G.R. No. L-32991 June 29, 1972

FACTS:

The first such appointment was extended on April 27, 1970, "effective May 1, 1970 until April 30,
1971, unless sooner terminated and subject to the appproval of the Board of Regents and to pertinent
University regulations." Pursuant thereto Dr. Blanco assumed office as ad interim Dean on May 1, 1970.
The Board of Regents met on May 26, 1970, and President Lopez submitted to it the ad interim
appointment of Dr. Blanco for reconsideration. The minutes of that meeting disclose that "the Board
voted to defer action on the matter in view of the objections cited by Regent Kalaw based on the petition
against the appointment, addressed to the Board, from a majority of the faculty and from a number of
alumni Dr. Blanco's appointment had lapsed.
On May 26, 1970, President Lopez extended another ad interim appointment to her, effective from
May 26, 1970 to April 30, 1971, with the same conditions as the first.However, such ad interim
appointment had not been confirmed by the Board of Regents. Due to the following votes: 5-yes, 3-no
and 4-abstain.
On August 18, 1970 Dr. Blanco wrote the President of the University, protesting the appointment
of Oseas A. del Rosario as Officer-in-Charge of the College of Education. Neither communication having
elicited any official reply, Dr. Blanco went to the Court of First Instance of Quezon City on a petition for
certiorari and prohibition with preliminary injunction.

ISSUE:

Whether or not respondent Dr. Consuelo S. Blanco was duly elected Dean of the College of
Education, University of the Philippines, in the meeting of the Board of Regents on July 9, 1970.

RULING:

NO.

The votes of abstention, viewed in their setting, can in no way be construed as votes for
confirmation of the appointment. There can be no doubt whatsoever as to the decision and
recommendation of the three members of the Personnel Committee: it was for rejection of the
appointment. No inference can be drawn from this that the members of the Personnel Committee, by
their abstention, intended to acquiesce in the action taken by those who voted affirmatively. Neither, for
that matter, can such inference be drawn from the abstention that he was abstaining because he was
not then ready to make a decision.
Dr. Blanco was clearly not the choice of a majority of the members of the Board of Regents, as
unequivocally demonstrated by the transcript of the proceedings. This fact cannot be ignored simply
because the Chairman, in submitting the question to the actual vote, did not frame it as accurately as
the preceding discussion called for, such that two of the Regents present (Silva and Kalaw) had to make
some kind of clarification.

231 | P a g e
Law 321_Corporation LAW_ Case Digest

VOTING
 Who May Exercise

Wilson P. Gamboa
vs.
Finance Secretary Margarito Teves, et al.,
G.R. No. 176579, June 28, 2011

FACTS:

This is a petition to nullify the sale of shares of stock of Philippine Telecommunications


Investment Corporation (PTIC) by the government of the Republic of the Philippines, acting through the
Inter-Agency Privatization Council (IPC), to Metro Pacific Assets Holdings, Inc. (MPAH), an affiliate of
First Pacific Company Limited (First Pacific), a Hong Kong-based investment management and holding
company and a shareholder of the Philippine Long Distance Telephone Company (PLDT). The petitioner
questioned the sale on the ground that it also involved an indirect sale of 12 million shares (or about 6.3
percent of the outstanding common shares) of PLDT owned by PTIC to First Pacific. With the this sale,
First Pacific’s common shareholdings in PLDT increased from 30.7 percent to 37 percent, thereby
increasing the total common shareholdings of foreigners in PLDT to about 81.47%. This, according to
the petitioner, violates Section 11, Article XII of the 1987 Philippine Constitution which limits foreign
ownership of the capital of a public utility to not more than 40%, thus: Section 11. No franchise,
certificate, or any other form of authorization for the operation of a public utility shall be granted except
to citizens of the Philippines or to corporations or associations organized under the laws of the
Philippines, at least sixty per centum of whose capital is owned by such citizens; nor shall such
franchise, certificate, or authorization be exclusive in character or for a longer period than fifty years.
Neither shall any such franchise or right be granted except under the condition that it shall be subject
to amendment, alteration, or repeal by the Congress when the common good so requires. The State shall
encourage equity participation in public utilities by the general public. The participation of foreign
investors in the governing body of any public utility enterprise shall be limited to their proportionate
share in its capital, and all the executive and managing officers of such corporation or association must
be citizens of the Philippines. (Emphasis supplied)

ISSUE:

Whether or not the term “capital” in Section 11, Article XII of the Constitution refer to the total
common shares only, or to the total outstanding capital stock (combined total of common and non-
voting preferred shares) of PLDT, a public utility.

HELD:

YES.

Considering that common shares have voting rights which translate to control, as opposed to
preferred shares which usually have no voting rights, the term “capital” in Section 11, Article XII of the
Constitution refers only to common shares. However, if the preferred shares also have the right to vote
in the election of directors, then the term “capital” shall include such preferred shares because the right
to participate in the control or management of the corporation is exercised through the right to vote in
the election of directors. In short, the term “capital” in Section 11, Article XII of the Constitution refers
only to shares of stock that can vote in the election of directors.
To construe broadly the term “capital” as the total outstanding capital stock, including both
common and non-voting preferred shares, grossly contravenes the intent and letter of the Constitution
that the “State shall develop a self-reliant and independent national economy effectively controlled by
Filipinos.” A broad definition unjustifiably disregards who owns the all-important voting stock, which
necessarily equates to control of the public utility.
Holders of PLDT preferred shares are explicitly denied of the right to vote in the election of
directors. PLDT’s Articles of Incorporation expressly state that “the holders of Serial Preferred Stock
shall not be entitled to vote at any meeting of the stockholders for the election of directors or for any
other purpose or otherwise participate in any action taken by the corporation or its stockholders, or to
receive notice of any meeting of stockholders.” On the other hand, holders of common shares are

232 | P a g e
Law 321_Corporation LAW_ Case Digest

granted the exclusive right to vote in the election of directors. PLDT’s Articles of Incorporation state that
“each holder of Common Capital Stock shall have one vote in respect of each share of such stock held by
him on all matters voted upon by the stockholders, and the holders of Common Capital Stock shall have
the exclusive right to vote for the election of directors and for all other purposes.”
It must be stressed, and respondents do not dispute, that foreigners hold a majority of the
common shares of PLDT. In fact, based on PLDT’s 2010 General Information Sheet (GIS), which is a
document required to be submitted annually to the Securities and Exchange Commission, foreigners
hold 120,046,690 common shares of PLDT whereas Filipinos hold only 66,750,622 common shares. In
other words, foreigners hold 64.27% of the total number of PLDT’s common shares, while Filipinos hold
only 35.73%. Since holding a majority of the common shares equates to control, it is clear that
foreigners exercise control over PLDT. Such amount of control unmistakably exceeds the allowable 40
percent limit on foreign ownership of public utilities expressly mandated in Section 11, Article XII of the
Constitution.
As shown in PLDT’s 2010 GIS, as submitted to the SEC, the par value of PLDT common shares
is P5.00 per share, whereas the par value of preferred shares is P10.00 per share. In other words,
preferred shares have twice the par value of common shares but cannot elect directors and have only
1/70 of the dividends of common shares. Moreover, 99.44% of the preferred shares are owned by
Filipinos while foreigners own only a minuscule 0.56% of the preferred shares. Worse, preferred shares
constitute 77.85% of the authorized capital stock of PLDT while common shares constitute only
22.15%. This undeniably shows that beneficial interest in PLDT is not with the non-voting preferred
shares but with the common shares, blatantly violating the constitutional requirement of 60 percent
Filipino control and Filipino beneficial ownership in a public utility.

PHILIPPINE COCONUT PRODUCERS FEDERATION, INC. (COCOFED), et al.


vs.
REPUBLIC OF THE PHILIPPINES
G.R. Nos. 177857-58 February 11, 2010

FACTS:

The Court, in its earlier resolution adverted to, approved, upon motion of petitioner Philippine
Coconut Producers Federation, Inc. (COCOFED), the conversion of the sequestered 753,848,312 Class
"A" and "B" common shares of San Miguel Corporation (SMC), registered in the name of Coconut
Industry Investment Fund (CIIF) Holding Companies (hereunder referred to as SMC Common Shares),
into 753,848,312 SMC Series 1 Preferred Shares. The oppositors herein made the following arguments:

233 | P a g e
Law 321_Corporation LAW_ Case Digest

(1) economic disadvantage and harm that government might suffer by such proposed conversion; (2)
they question the wisdom of PCGG in converting those sequestered shares; (3) that the conversion is
invalid in view of the Commission on Audit Circular No. 89- 296 which provides that disposal of
government property must be undertaken via public Auction; (4) that the conversion thereof needs the
acquiescence of the 14 CIIF companies; (5) As to the Motion to Intervene by UCPB, that it should be the
sole depositary of the proceeds of the dividends.

ISSUE:

Whether or not the arguments of the Oppositors herein have merits.

RULING:

NO.

Anent the 1st contention, it is not tenable because in fact this conversion is a business strategy to
preserve and conserve the value of the Government’s interest in the CIIF SMC shares. As to the 2 nd
argument, it is also untenable because it is not within the Courts to determine wisdom of other agencies
of the government. As to the 3rd argument, likewise untenable because FIRST, there is really no disposal
of SMC shares and SECOND, there is no yet government assets to talk about because the ownership
thereto is still to be determined, hence, those shares are akin to properties subject of attachment. As to
the 4th contention, PCGG need not obtain the acquiescence of the owners of those sequestered shares
with respect to any of its acts intended to preserve such assets. Otherwise, it would be impossible for it
to perform its function as provided by law. And as to the 5 th argument, it is also of no merit because the
Court has the discretion where to deposit those net dividends, whether it be on Development Bank of
the Philippines/ Land Bank of the Philippines or the UCPB.

REPUBLIC OF THE PHILIPPINES, represented by the PRESIDENTIAL COMMISSION ON GOOD


GOVERNMENT (PCGG)
vs.
COCOFED, ET AL. and BALLARES, ET AL., EDUARDO M. COJUANGCO JR. and the
SANDIGANBAYAN (First Division)
G.R. No. 147062-64 December 14, 2001

FACTS:

On the explicit premise that 'vast resources of the government have been amassed by former
President Ferdinand E. Marcos, his immediate family, relatives, and close associates both here and
abroad,' the Presidential Commission on Good Government (PCGG) was created by Executive Order No.
1 to assist the President in the recovery of the ill-gotten wealth thus accumulated whether located in the
Philippines or abroad. Several executive orders were then issued describing the properties to be
recovered.Among the properties sequestered by the Commission were shares of stock in the United
Coconut Planters Bank (UCPB) registered in the names of the alleged "one million coconut farmers," the
so-called Coconut Industry Investment Fund companies (CIIF companies) and Private Respondent
Eduardo Cojuangco Jr.

234 | P a g e
Law 321_Corporation LAW_ Case Digest

Six years later, on February 13, 2001, the Board of Directors of UCPB received from the ACCRA
Law Office a letter written on behalf of the COCOFED and the alleged nameless one million coconut
farmers, demanding the holding of a stockholders' meeting for the purpose of, among others, electing
the board of directors. In response, the board approved a Resolution calling for a stockholders' meeting
on March 6, 2001 at three o'clock in the afternoon. However, the same was meted by a Class Action
Omnibus Motion seeking to enjoin PCGG from voting the UCPB shares of stock registered in the
respective names of the more than one million coconut farmers; and to enjoin the PCGG from voting the
SMC shares registered in the names of the 14 CIF holding companies including those registered in the
name of the PCGG.

ISSUE:

Whether or not PCGG may vote the sequestered UCPB shares while the main case for their
reversion to the State is pending in the Sandiganbayan.

RULING:

YES.

The SC holds that the government should be allowed to continue voting those shares inasmuch
as they were purchased with coconut levy funds since those are prima facie public in character or, at
the very least, are "clearly affected with public interest."
The general rule is that the registered owner of the shares of a corporation exercises the right and
the privilege of voting. This principle applies even to shares that are sequestered by the government,
over which the PCGG as a mere conservator cannot, as a general rule, exercise acts of dominion. On the
other hand, it is authorized to vote these sequestered shares registered in the names of private persons
and acquired with allegedly ill-gotten wealth, if it is able to satisfy the two-tiered test. Unfortunately,
this test is not applicable under the circumstances of this case.
Hence, the Court granted PCGG the right to vote the sequestered shares because they appeared to be
assets belonging to the government itself.

RAMON C. LEE and ANTONIO DM. LACDAO


vs.
THE HON. COURT OF APPEALS, SACOBA MANUFACTURING CORP., PABLO GONZALES, JR. and
THOMAS GONZALES
G.R. No. 93695 February 4, 1992

FACTS:

In 1985, a complaint for sum of money was filed by the International Corporate Bank, Inc.
against the private respondents who, in turn, filed a third party complaint against Alfa Integrated Textile
Mills (ALFA) and the petitionersRamon C. Lee and Antonio Dm. Lacdao who were officers of ALFA.
Meanwhile, in 1988, the trial court issued an order requiring the issuance of an alias summons upon
ALFA through the DBP as a consequence of the petitioners' letter informing the court that the summons
for ALFA was erroneously served upon them considering that the management of ALFA had been
transferred to the Development Bank of the Philippines (DBP).
In a manifestation, the DBP claimed that it was not authorized to receive summons on behalf of
ALFA since the DBP had not taken over the company which has a separate and distinct corporate
personality and existence.

ISSUE:

Whether or not despite the execution of the Voting Trust Agreement, the summons be served
upon the petitioners who were officers and directors of ALFA (the trustor).

RULING:

235 | P a g e
Law 321_Corporation LAW_ Case Digest

NO.

There is no dispute as to the most immediate effect of a voting trust agreement on the status of a
stockholder who is a party to its execution from legal titleholder or owner of the shares subject of the
voting trust agreement, he becomes the equitable or beneficial owner.
Note that in order to be eligible as a director, what is material is the legal title to, not beneficial
ownership of, the stock as appearing on the books of the corporation
Considering that the voting trust agreement between ALFA and the DBP transferred legal ownership of
the stocks covered by the agreement to the DBP as trustee, the latter became the stockholder of record
with respect to the said shares of stocks. In the absence of a showing that the DBP had caused to be
transferred in their names one share of stock for the purpose of qualifying as directors of ALFA, the
petitioners can no longer be deemed to have retained their status as officers of ALFA which was the case
before the execution of the subject voting trust agreement. There appears to be no dispute from the
records that DBP has taken over full control and management of the firm.

Republic of the Philippines (Presidential Commission on Good Government),


vs.
Sandiganbayan
GR 107789 30 April 2003

FACTS:

On 7 August 1991, the PCGG conducted an Eastern Telecommunications, Philippines, Inc. (ETPI)
stockholders meeting during which a PCGG controlled board of directors was elected. A special
stockholders meeting was later convened by the registered ETPI stockholders wherein another set of
board of directors was elected, as a result of which two sets of such board and officers were elected.
Victor Africa, a stockholder of ETPI, alleging that the PCGG had since been "illegally 'exercising' the
rights of stockholders of ETPI," especially in the election of the members of the board of directors, filed a
motion before the Sandiganbayan, prayed that said court order the "calling and holding of the ETPI
annual stockholders meeting for 1992 under the court's control and supervision and prescribed
guidelines." The PCGG did not object to Africa's motion provided that "(1) An Order be issued upholding
the right of PCGG to vote all the Class "A" shares of ETPI; (2) In the alternative, in the remote event that
PCGG's right to vote the sequestered shares be not upheld, an Order be issued (a) disregarding the
Stock and Transfer Book and Booklet of Stock Certificates of ETPI in determining who can vote the
shares in an Annual Stockholders Meeting of ETPI, (b) allowing PCGG to vote 23.9% of the total
subscription in ETPI, and (c) directing the amendment of the Articles of Incorporation and By-laws of
ETPI providing for the minimum safeguards for the conservation of assets prior to the calling of a
stockholders meeting. The Sandiganbayan resolved Africa's motion, ordering the conduct of an annual
stockholders meeting of ETPI, for 1992. Assailing the foregoing resolution, the PCGG filed before the
Supreme Court a petition for Certiorari, Mandamus and Prohibition.

ISSUE:

Whether or notthe PCGG can vote the sequestered ETPI Class "A" shares in the stockholders
meeting for the election of the board of directors.

RULING:

236 | P a g e
Law 321_Corporation LAW_ Case Digest

YES.

The PCGG cannot vote sequestered shares to elect the ETPI Board of Directors or to amend the
Articles of Incorporation for the purpose of increasing the authorized capital stock unless there is a
prima facie evidence showing that said shares are ill-gotten and there is an imminent danger of
dissipation. (2)The ETPI Stock and Transfer Book should be the basis for determining which persons
have the right to vote in the stockholders meeting for the election of the ETPI Board of Directors. (3) The
PCGG is entitled to vote the shares ceded to it by Roberto S. Benedicto and his controlled corporations
under the Compromise Agreement, provided that the shares are first registered in the name of the
PCGG. The PCGG may not register the transfer of the Malacañang and the Nieto shares in the ETPI
Stock and Transfer Book; however, it may vote the same as conservator provided that the PCGG satisfies
the two-tiered test devised by the Court in Cojuangco v. Calpo. (4) The safeguards laid down in the case
of Cojuangco v. Roxas shall be incorporated in the ETPI Articles of Incorporation substantially
contemporaneous to, but not before, the election of the ETPI Board of Directors. (5) Members of the
Sandiganbayan shall not participate in the stockholders meeting for the election of the ETPI Board of
Directors.

 Voting Trust Agreement

ROSAURA P. CORDON
vs.
JESUS BALICANTA
A.C. No. 2797 October 4, 2002

FACTS:

Sometime in the early part of 1981, respondent enticed complainant and her daughter to organize
a corporation that would develop the said real properties into a high-scale commercial complex with a
beautiful penthouse for complainant. Relying on these apparently sincere proposals, complainant and
her daughter assigned 19 parcels of land to Rosaura Enterprises, Incorporated, and a newly-formed and
duly registered corporation in which they assumed majority ownership. The subject parcels of land were
then registered in the name of the corporation.
Thereafter, respondent single-handedly ran the affairs of the corporation in his capacity as
Chairman of the Board, President, General Manager and Treasurer. The respondent also made
complainant sign a document which turned out to be a voting trust agreement. Respondent likewise
succeeded in making complainant sign a special power of attorney to sell and mortgage some of the
parcels of land she inherited from her deceased husband. She later discovered that respondent
transferred the titles of the properties to a certain Tion Suy Ong who became the new registered owner
thereof. Respondent never accounted for the proceeds of said transfers. Other spurious transactions not
approved by the Board were entered into by the defendant through spurious board resolutions.

ISSUE:

Whether or not there was really a voting trust agreement made by the complainant in favor of the
defendant.

RULING:

NO.

The claim is baseless. The voting trust referred to by respondent, even if it were assumed to be
valid, covered only 266 shares of complainants yet she owned a total of 1,039 shares after she and her
daughter ceded in favor of the corporation 19 parcels of land. Being a former lawyer to complainant,
respondent should have ensured that her interest was safeguarded. Yet, complainant was apparently
and deliberately left it on the pretext that, she had executed a voting trust agreement in favor of
respondent. It is suspicious that complainant was made to sign a voting trust agreement on 21 August
1981 and immediately thereafter, the resolutions authorizing respondent to obtain a loan and to

237 | P a g e
Law 321_Corporation LAW_ Case Digest

mortgage the 9 parcels of land were passed and approved. It is further worth noting that complainant’s
voting trust where she allegedly entrusted 266 shares to respondent on August 21, 1981 had only a
validity of 5 years. Thus, she should have had her entire holdings of 1,283 shares back in her name in
August 1986.“Respondent’s purported minutes of stockholders’ meeting do not reflect this. “There was
no explanation whatsoever from respondent on how complainant and her daughter lost their 97%
control holding in the corporation. Respondent cannot take refuge in the contested voting trust
agreement supposedly executed by complainant and her daughter for the reason that it authorized
respondent to represent complainant for such matters.
Moreover the factual findings of the investigating commission, affirmed by the IBP Board,
disclosed that complainant and her daughter own 1,711 out of 1,750 shares of the outstanding capital
stock of the corporation, based on the Articles of Incorporation and deeds of transfer of the properties.
But respondent’s evidence showed that complainant had only 266 shares of stock in the corporation
while her daughter had none, notwithstanding the fact that there was nothing to indicate that
complainant and her daughter ever conveyed their shares to others.
NATIONAL INVESTMENT AND DEVELOPMENT CORPORATION, EUSEBIO VILLATUYA MARIO Y.
CONSING and ROBERTO S. BENEDICTO
vs.
HON. BENJAMIN AQUINO, et al.
G.R. No. L-34192 June 30, 1988

FACTS:

Batjak, is a Filipino-American corporation which has indebtedness to Philippine National Bank


(PNB) amounted to P11,915,000.00, As security for the payment of its obligations and advances against
shipments, Batjak mortgaged its three (3) coco-processing oil mills to Manila Bank, Republic Bank ,
and PCIB, respectively. In need for additional operating capital to place the three (3) coco-processing
mills at their optimum capacity and maximum efficiency and to settle, pay or otherwise liquidate
pending financial obligations with the different private banks, Batjak applied to PNB for additional
financial assistance. A Financial Agreement was submitted by PNB to Batjak for acceptance which was
duly accepted by Batjak. Upon receiving payment, RB, PCIB, and MBTC released in favor of PNB the
first and any mortgages they held on the properties of Batjak. Batjak executed a first mortgage in favor
of PNB on all its properties A Voting Trust Agreement was executed in favor of NIDC by the stockholders
representing 60% of the outstanding paid-up and subscribed shares of Batjak. This agreement was for a
period of five (5) years and, upon its expiration, was to be subject to negotiation between the parties.
Forced by the insolvency of Batjak, PNB instituted extrajudicial foreclosure proceedings against the oil
mills of Batjak. The properties were sold to PNB as the highest bidder. Three years thereafter, Batjak
wrote a letter to NIDC inquiring if the latter was still interested in negotiating the renewal of the Voting
Trust Agreement. Batjak wrote another letter to NIDC informing the latter that Batjak would now safely
assume that NIDC was no longer interested in the renewal of said Voting Trust Agreement.

ISSUE:

Whether or not the NIDC and PNB acquired ownership over the assets of Batjak despite a voting
trust agreement between Batjak’s stockholders and NIDC.

RULING:

YES.

What was assigned to NIDC was the power to vote the shares of stock of the stockholders of
Batjak, representing 60% of Batjak's outstanding shares, and who are the signatories to the agreement.
The power entrusted to NIDC also included the authority to execute any agreement or document that
may be necessary to express the consent or assent to any matter, by the stockholders. Nowhere in the
said provisions or in any other part of the Voting Trust Agreement is mention made of any transfer or
assignment to NIDC of Batjak's assets, operations, and management. NIDC was constituted as trustee
only of the voting rights of 60% of the paid-up and outstanding shares of stock in Batjak. Under the
provision on termination what was to be returned by NIDC as trustee to Batjak's stockholders, upon the
termination of the agreement, are the certificates of shares of stock belonging to Batjak's stockholders,
not the properties or assets of Batjak itself which were never delivered, in the first place to NIDC, under
the terms of said Voting Trust Agreement. A voting trust transfers only voting or other rights pertaining
to the shares subject of the agreement or control over the stock hence the acquisition by PNB-NIDC of
the properties in question was not made or effected under the capacity of a trustee but as a foreclosing
creditor for the purpose of recovering on a just and valid obligation of Batjak.

238 | P a g e
Law 321_Corporation LAW_ Case Digest

LEON J. LAMBERT
vs.
T. J. FOX
G.R. No. L-7991 January 29, 1914

FACTS:

Early in 1911 the firm known as John R. Edgar & Co., engaged in the retail book and stationery
business, found itself in such condition financially that its creditors, including the plaintiff and the
defendant, together with many others, agreed to take over the business, incorporate it and accept stock
therein in payment of their respective credits. This was done, the plaintiff and the defendant becoming
the two largest stockholders in the new corporation called John R. Edgar & Co., Incorporated. A few
days after the incorporation was completed plaintiff and defendant entered into an agreement whereby
the shockholders mutually and reciprocally agree not to sell, transfer, or otherwise dispose of any part of
their present holdings of stock in said John R. Edgar & Co. Inc., till after one year from the date hereof
and that Either party violating this agreement shall pay to the other the sum of one thousand (P1,000)
pesos as liquidated damages, unless previous consent in writing to such sale, transfer, or other
disposition be obtained.
Notwithstanding this contract the defendant Fox on October 19, 1911, sold his stock in the said
corporation to E. C. McCullough of the firm of E. C. McCullough & Co. of Manila, a strong competitor of
the said John R. Edgar & Co., Inc. This sale was made by the defendant against the protest of the
plaintiff and with the warning that he would be held liable under the contract hereinabove set forth and
in accordance with its terms. In fact, the defendant Foz offered to sell his shares of stock to the plaintiff
for the same sum that McCullough was paying them less P1,000, the penalty specified in the contract.

ISSUE:

Whether or not the suspension of the power to sell the stock is valid and legal.

RULING:

YES.

The suspension of the power to sell has a beneficial purpose, results in the protection of the
corporation as well as of the individual parties to the contract, and is reasonable as to the length of time
of the suspension. We do not here undertake to discuss the limitations to the power to suspend the right
of alienation of stock, limiting ourselves to the statement that the suspension in this particular case is
legal and valid.

CAPITAL STRUCTURE STOCKS AND STOCKHOLDERS


 As Legal/Stated Capital: Trust Fund Doctrine
239 | P a g e
Law 321_Corporation LAW_ Case Digest

PHILIPPINE LONG DISTANCE TELEPHONE COMPANY


vs.
NATIONAL TELECOMMUNICATIONS COMMISSION, JOSEPH A.SANTIAGO, in his capacity as NTC
Commissioner, and EDGARDO CABARRIOS
G.R. No. 152685 4 December 2007

FACTS:

Case pertains to Section 40 (e) the Public Service Act (PSA), as amended on March 15, 1984,
pursuant to Batas Pambansa Blg This. 325, which authorized the NTC to collect from public
telecommunications companies Supervision and Regulation Fees (SRF) of PhP 0.50 for every PhP 100 or
a fraction of the capital and stock subscribed or paid for of a stock corporation, partnership or single
proprietorship of the capital invested, or of the property and equipment, whichever is higher. Under
Section 40 (e) of the PSA, the NTC sent SRF assessments to petitioner Philippine Long Distance
Telephone Company (PLDT) starting sometime in 1988. The SRF assessments were based on the market
value of the outstanding capital stock, including stock dividends, of PLDT. PLDT protested the
assessments contending that the SRF ought to be based on the par value of its outstanding capital
stock. Its protest was denied by the NTC and likewise, its motion for reconsideration. PLDT appealed
before the CA. The CA modified the disposition of the NTC by holding that the SRF should be assessed
at par value of the outstanding capital stock of PLDT, excluding stock dividends.

ISSUE:

Whether or not the value transferred from the unrestricted retained earnings of PLDT to the
capital stock account pursuant to the issuance of stock dividends is the proper basis for the assessment
of the SRF.

RULING:

NO.

In the case of stock dividends, it is the amount that the corporation transfers from its surplus
profit account to its capital account. It is the same amount that can be loosely termed as the "trust
fund" of the corporation. The "Trust Fund" doctrine considers this subscribed capital as a trust fund for
the payment of the debts of the corporation, to which the creditors may look for satisfaction. Until the
liquidation of the corporation, no part of the subscribed capital may be returned or released to the
stockholder (except in the redemption of redeemable shares) without violating this principle. Thus,
dividends must never impair the subscribed capital; subscription commitments cannot be condoned or
remitted; nor can the corporation buy its own shares using the subscribed capital as the considerations
therefore.
When stock dividends are distributed, the amount declared ceases to belong to the corporation
but is distributed among the shareholders. Consequently, the unrestricted retained earnings of the
corporation are diminished by the amount of the declared dividend while the stockholders equity is
increased. Furthermore, the actual payment is the cash value from the unrestricted retained earnings
that each shareholder foregoes for additional stocks/shares which he would otherwise receive as
required by the Corporation Code to be given to the stockholders subject to the availability and
conditioned on a certain level of retained earnings.
In essence, therefore, the stockholders by receiving stock dividends are forced to exchange the
monetary value of their dividend for capital stock, and the monetary value they forego is considered the
actual payment for the original issuance of the stocks given as dividends. Therefore, stock dividends
acquired by shareholders for the monetary value they forego are under the coverage of the SRF and the
basis for the latter is such monetary value as declared by the board of directors.
NATIONAL TELECOMMUNICATIONS COMMISSION
vs.
HONORABLE COURT OF APPEALS and PHILIPPINE LONG DISTANCE TELEPHONE COMPANY
G.R. No. 127937 July 28, 1999

FACTS:

Sometime in 1988, the NTC served on the PLDT the following assessment notices and demands
for payment: 1. the amount of P7,495,161.00 as supervision and regulation fee under Section 40 (e) of
the PSA for the said year, 1988, computed at P0.50 per P100.00 of the Protestant's (PLDT) outstanding

240 | P a g e
Law 321_Corporation LAW_ Case Digest

capital stock as at December 31, 1987 which then consisted of Serial Preferred Stock amounting to
P1,277,934,390.00 and Common Stock of P221,097,785 (Million) or a total of P1,499,032,175.00; 2. the
amount of P9.0 Million as permit fee under Section 40 (f) of the PSA for the approval of the protestant's
increase of its authorized capital stock from P2.7 Billion to P4.5 Billion; and the amounts of
P12,261,600.00 and P33,472,030.00 as permit fees under Section 40(g) of the PSA in connection with
the Commission's decisions in NTC Cases Nos. 86-13 and 87-008 respectively, approving the
Protestant's equity participation in the Fiber Optic Interpacific Cable systems and X-5 Service
Improvement and Expansion Program.

ISSUE:

Whether or not the Court of Appeals erred in holding that the computation of supervision and
regulation fees under section 40 (f) of the public service act should be based on the par value of the
subscribed capital stock.

RULING:

NO.

The basis for computation of the fee to be charged by NTC on PLDT, is the capital stock
subscribed or paid and not, alternatively, the property and equipment. The term "capital" and other
terms used to describe the capital structure of a corporation are of universal acceptance, and their
usages have long been established in jurisprudence. Briefly, capital refers to the value of the property or
assets of a corporation. The capital subscribed is the total amount of the capital that persons
(subscribers or shareholders) have agreed to take and pay for, which need not necessarily be, and can
be more than, the par value of the shares. In fine, it is the amount that the corporation receives,
inclusive of the premiums if any, in consideration of the original issuance of the shares. In the case of
stock dividends, it is the amount that the corporation transfers from its surplus profit account to its
capital account. It is the same amount that can loosely be termed as the "trust fund" of the corporation.
The "Trust Fund" doctrine considers this subscribed capital as a trust fund for the payment of the debts
of the corporation, to which the creditors may look for satisfaction. Until the liquidation of the
corporation, no part of the subscribed capital may be returned or released to the stockholder (except in
the redemption of redeemable shares) without violating this principle. Thus, dividends must never
impair the subscribed capital; subscription commitments cannot be condoned or remitted; nor can the
corporation buy its own shares using the subscribed capital as the consideration therefor.

 Voting Control Test v. Beneficial Control Test

Wilson P. Gamboa
vs.
Finance Secretary Margarito Teves, et al.,
G.R. No. 176579, June 28, 2011

FACTS:

This is a petition to nullify the sale of shares of stock of Philippine Telecommunications


Investment Corporation (PTIC) by the government of the Republic of the Philippines, acting through the
Inter-Agency Privatization Council (IPC), to Metro Pacific Assets Holdings, Inc. (MPAH), an affiliate of
First Pacific Company Limited (First Pacific), a Hong Kong-based investment management and holding
company and a shareholder of the Philippine Long Distance Telephone Company (PLDT). The petitioner
questioned the sale on the ground that it also involved an indirect sale of 12 million shares (or about 6.3
percent of the outstanding common shares) of PLDT owned by PTIC to First Pacific. With the this sale,
First Pacific’s common shareholdings in PLDT increased from 30.7 percent to 37 percent, thereby
increasing the total common shareholdings of foreigners in PLDT to about 81.47%. This, according to

241 | P a g e
Law 321_Corporation LAW_ Case Digest

the petitioner, violates Section 11, Article XII of the 1987 Philippine Constitution which limits foreign
ownership of the capital of a public utility to not more than 40%.

ISSUE:

Whether or not the term “capital” in Section 11, Article XII of the Constitution refer to the total
common shares only, or to the total outstanding capital stock (combined total of common and non-
voting preferred shares) of PLDT, a public utility.

RULING:

NO.

The Court partly granted the petition and held that the term “capital” in Section 11, Article XII of
the Constitution refers only to shares of stock entitled to vote in the election of directors of a public
utility, i.e., to the total common shares in PLDT.
It must be stressed, and respondents do not dispute, that foreigners hold a majority of the
common shares of PLDT. In fact, based on PLDT’s 2010 General Information Sheet (GIS), which is a
document required to be submitted annually to the Securities and Exchange Commission, foreigners
hold 120,046,690 common shares of PLDT whereas Filipinos hold only 66,750,622 common shares. In
other words, foreigners hold 64.27% of the total number of PLDT’s common shares, while Filipinos hold
only 35.73%. Since holding a majority of the common shares equates to control, it is clear that
foreigners exercise control over PLDT. Such amount of control unmistakably exceeds the allowable 40
percent limit on foreign ownership of public utilities expressly mandated in Section 11, Article XII of the
Constitution.
As shown in PLDT’s 2010 GIS, as submitted to the SEC, the par value of PLDT common shares
is P5.00 per share, whereas the par value of preferred shares is P10.00 per share. In other words,
preferred shares have twice the par value of common shares but cannot elect directors and have only
1/70 of the dividends of common shares. Moreover, 99.44% of the preferred shares are owned by
Filipinos while foreigners own only a minuscule 0.56% of the preferred shares. Worse, preferred shares
constitute 77.85% of the authorized capital stock of PLDT while common shares constitute only
22.15%. This undeniably shows that beneficial interest in PLDT is not with the non-voting preferred
shares but with the common shares, blatantly violating the constitutional requirement of 60 percent
Filipino control and Filipino beneficial ownership in a public utility.

HEIRS OF WILSON P. GAMBOA


vs.
FINANCE SECRETARYMARGARITO B. TEVES, FINANCE UNDERSECRETARYJOHN P. SEVILLA,
AND COMMISSIONER RICARDO ABCEDE OF THE PRESIDENTIAL COMMISSION ON GOOD
GOVERNMENT(PCGG), et.al.
G.R. No. 176579 October 9, 2012

FACTS:

The Office of the Solicitor General (OSG) initially filed a motion for reconsideration on behalfofthe
SEC, assailing the 28 June 2011 Decision. However, it subsequently filed a Consolidated Comment on
behalf of the State, declaring expressly that it agrees with the Court's definition of the term "capital" in
Section 11, Article XII of the Constitution. During the Oral Arguments on 26 June 2012, the OSG
reiterated its position consistent with the Court's 28 June 2011 Decision.

ISSUE:

Whether or not the term "capital" in Section 11, Article XII of the Constitution has long been
settled and defined to refer to the total outstanding shares of stock, whether voting or non-voting.

RULING:

NO.

Since a specific class of shares may have rights and privileges or restrictions different from the
rest of the shares in a corporation, the 60-40 ownership requirement in favor of Filipino citizens in

242 | P a g e
Law 321_Corporation LAW_ Case Digest

Section 11, Article XII of the Constitution must apply not only to shares with voting rights but also to
shares without voting rights. Preferred shares, denied the right to vote in the election of directors, are
anyway still entitled to vote on the eight specific corporate matters mentioned above. Thus, if a
corporation, engaged in a partially nationalized industry, issues a mixture of common and preferred
non-voting shares, at least 60 percent of the common shares and at least 60 percent of the preferred
non-voting shares must be owned by Filipinos. Of course, if a corporation issues only a single class of
shares, at least 60 percent of such shares must necessarily be owned by Filipinos. In short, the 60-40
ownership requirement in favor of Filipino citizens must apply separately to each class of shares,
whether common, preferred non-voting, preferred voting or any other class of shares. This uniform
application of the 60-40 ownership requirement in favor of Filipino citizens clearly breathes life to the
constitutional command that the ownership and operation of public utilities shall be reserved
exclusively to corporations at least 60 percent of whose capital is Filipino-owned. Applying uniformly the
60-40 ownership requirement in favor of Filipino citizens to each class of shares, regardless of
differences in voting rights, privileges and restrictions, guarantees effective Filipino control of public
utilities, as mandated by the Constitution.

G.R. NOS. 174457-59


EXPRESS INVESTMENTS III PRIVATE LTD. AND EXPORT DEVELOPMENT CANADA
vs.
DAYAN TELECOMMUNICATIONS, INC., THE BANK OF NEW YORK (AS TRUSTEE FOR THE
HOLDERS OF THE US$200,000,000 13.5% SENIOR NOTES OF DAYAN TELECOMMUNICATIONS,
INC.) AND ATTY. REMIGIO A. NOVAL (AS THE COURT-APPOINTED REHABILITATION RECEIVER
OF BAYANTEL)
x---------------x

G.R. Nos. 175418-20


IN THE MATTER OF:
THE CORPORATE REHABILITATION OF DAYAN TELECOMMUNICATIONS, INC. PURSUANT TO
THE INTERIM RULES OF PROCEDURE ON CORPORATE REHABILITATION (A.M. NO. 00-8-10-SC)
THE BANK OF NEW YORK AS TRUSTEE FOR THE HOLDERS OF THE US$200,000,000 13.5%
SENIOR NOTES OF DAYAN TELECOMMUNICATIONS, INC.
DUE 2006 ACTING ON THE INSTRUCTIONS OF THE INFORMAL STEERING COMMITTEE: AVENUE
ASIA INVESTMENTS, L.P., AVENUE ASIA INTERNATIONAL, LTD., AVENUE ASIA SPECIAL
SITUATIONS FUND II, L.P. AND AVENUE ASIA CAPITAL PARTNERS, L.P.
vs.
DAYAN TELECOMMUNICATIONS, INC.

x---------------x
IN THE MATTER OF:
THE CORPORATE REHABILITATION OF BAY AN TELECOMMUNICATIONS, INC. PURSUANT TO
THE INTERIM RULES OF PROCEDURE ON CORPORATE REHABILITATION (A.M. NO. 00-8-10-SC)
AVENUE ASIA INVESTMENTS, L.P., AVENUE ASIA INTERNATIONAL, LTD., AVENUE ASIA SPECIAL
SITUATIONS FUND II, L.P., AVENUE ASIA CAPITAL PARTNERS, L.P. AND AVENUE ASIA SPECIAL
SITUATIONS FUND III, L.P.
vs.
DAYAN TELECOMMUNICATIONS, INC.

x---------------x
G.R. No. 177270 December 5, 2012

243 | P a g e
Law 321_Corporation LAW_ Case Digest

THE BANK OF NEW YORK AS TRUSTEE FOR THE HOLDERS OF THE US$200,000,000 13.5%
SENIOR NOTES OF BAY AN TELECOMMUNICATIONS, INC.
vs.
BAY AN TELECOMMUNICATIONS, INC.

FACTS:

Respondent Bayantel is a duly organized domestic corporation engaged in the business of


providing telecommunication services. It is 98.6% owned by Bayan Telecommunications Holdings
Corporation (BTHC), which in turn is 85.4% owned by the Lopez Group of Companies and Benpres
Holdings Corporation.
On various dates between the years 1995 and 2001, Bayantel entered into several credit
agreements. In July 1999, Bayantel issued US$200 million worth of 13.5% Senior Notes pursuant to an
Indenture dated July 22, 1999 that it entered into with The Bank of New York as trustee for the holders
of said notes.

ISSUES:

Whether or not the claims of secured and unsecured creditors should be treated pari
passu during rehabilitation.
Whether or not the debt-equity complies with the citizenship requirement under the Constitution.
RULING:

YES.

As between the creditors, the key phrase is "equality is equity." When a corporation threatened by
bankruptcy is taken over by a receiver, all the creditors should stand on equal footing. Not anyone of
them should be given any preference by paying one or some of them ahead of the others. This is
precisely the reason for the suspension of all pending claims against the corporation under receivership.
Instead of creditors vexing the courts with suits against the distressed firm, they are directed to file their
claims with the receiver who is a duly appointed officer of the SEC.
Since then, the principle of equality in equity has been cited as the basis for placing secured and
unsecured creditors in equal footing or in pari passu with each other during rehabilitation. In legal
parlance, pari passu is used especially of creditors who, in marshaling assets, are entitled to receive out
of the same fund without any precedence over each other.

YES.

Applying this, two steps must be followed in order to determine whether the conversion of debt to
equity in excess of 40% of the outstanding capital stock violates the constitutional limit on foreign
ownership of a public utility: First, identify into which class of shares the debt shall be converted,
whether common shares, preferred shares that have the right to vote in the election of directors or non-
voting preferred shares; Second, determine the number of shares with voting right held by foreign
entities prior to conversion. If upon conversion, the total number of shares held by foreign entities
exceeds 40% of the capital stock with voting rights, the constitutional limit on foreign ownership is
violated. Otherwise, the conversion shall be respected.
In its Rehabilitation Plan, among the material financial commitments made by respondent
Bayantel is that its shareholders shall "relinquish the agreed-upon amount of common stock[s] as
payment to Unsecured Creditors as per the Term Sheet." Evidently, the parties intend to convert the
unsustainable portion of respondent's debt into common stocks, which have voting rights. If we indulge
petitioners on their proposal, the Omnibus Creditors which are foreign corporations, shall have control
over 77.7% of Bayantel, a public utility company. This is precisely the scenario proscribed by the
Filipinization provision of the Constitution. Therefore, the Court of Appeals acted correctly in sustaining
the 40% debt-to-equity ceiling on conversion.

244 | P a g e
Law 321_Corporation LAW_ Case Digest

REDMONT CONSOLIDATED MINING CORPORATION


vs.
MCARTHUR MINING, INC. ET.AL.
SEC En Banc Case No. 09-09-177, March 25. 2010

Significantly, the SEC en banc, which is the collegial body statutorily empowered to issue rules
and opinions on behalf of the SEC, has adopted the 60-40 ownership requirement in favor of Filipino
citizens mandated by the Constitution for certain economic activities.
The avowed purpose of the Constitution is to place in the hands of Filipinos the exploitation of
our natural resources. Necessarily, therefore, the Rule interpreting the constitutional provision should
not diminish that right through the legal fiction of corporate ownership and control. But the
constitutional provision, as interpreted and practiced via the 1967 SEC Rules, has favored foreigners
contrary to the command of the Constitution. Hence, the Grandfather Rule must be applied to accurately
determine the actual participation, both direct and indirect, of foreigners in a corporation engaged in a
nationalized activity or business.

245 | P a g e
Law 321_Corporation LAW_ Case Digest

DEMOSTHENES P. AGAN, et al., petitioners,,


vs.
PHILIPPINE INTERNATIONAL AIR TERMINALS CO., INC., MANILA INTERNATIONAL AIRPORT
AUTHORITY, et al., Respondents.
G.R. No. 155001 January 21, 2004

FACTS:

Sometime in 1993, six business leaders, explored the possibility of investing in the new NAIA
airport terminal, so they formed Asians Emerging Dragon Corp. They submitted proposals to the
government for the development of NAIA Intl. Passenger Terminal III (NAIA IPT III). The NEDA approved
the NAIA IPT III project. Bidders were invited, and among the proposal Peoples Air Cargo (Paircargo) was
chosen. AEDC protested alleging that preference was given to Paircargo, but still the project was
awarded to Paircargo. Because of that, it incorporated into, Phil. Intl. Airport Terminals Co. (PIATCO).
The DOTC and PIATCO entered into a concession agreement in 1997 to franchise and operate the said
terminal for 21years. In Nov. 1998 it was amended in the matters of pertaining to the definition of the
obligations given to the concessionaire, development of facilities and proceeds, fees and charges, and the
termination of contract. Since MIAA is charged with the maintenance and operations of NAIA terminals I
and II, it has a contract with several service providers. The workers filed the petition for prohibition
claiming that they would lose their job, and the service providers joined them, filed a motion for
intervention. Likewise several employees of the MIAA filed a petition assailing the legality of
arrangements. A group of congressmen filed similar petitions. Pres. Arroyo declared in her speech that
she will not honor PIATCO contracts which the Exec. Branch's legal office concluded null and void.

ISSUE:
Whether or Not the 1997 concession agreement is void, together with its amendments for being
contrary to the constitution.

RULING:

YES.

The 1997 concession agreement is void for being contrary to public policy. The amendments have
the effect of changing it into and entirely different agreement from the contract bidded upon. The
amendments present new terms and conditions which provide financial benefit to PIATCO which may
have the altered the technical and financial parameters of other bidders had they know that such terms
were available. The 1997 concession agreement, the amendments and supplements thereto are set aside
for being null and void.
The petitioners have local standi. They are prejudiced by the concession agreement as their
livelihood is to be taken away from them.

246 | P a g e
Law 321_Corporation LAW_ Case Digest

 Voting v. Non-Voting

CECILIA CASTILLO, et al., and MEDICAL CENTER PARAÑAQUE, INC.


vs.
ANGELES BALINGHASAY, et al.
G.R. No. 150976 October 18, 2004

FACTS:

Petitioners are stockholders of MCPI holding Class “B” shares while the respondents are also
stockholders owning Class “A” shares. In a 1992 amendment of the Articles of Incorporation of MCPI,
the Articles of incorporation of the MCPI provides that, “except when otherwise provided by law, only
holders of Class “A” shares are entitled to vote and to have the right to be elected as directors and
corporate officers.
During the 2001meeting, petitioners raised an objection to the fact that only Class “A” shares are
allowed to vote and to be elected. They contended that the Class “B” share holders right to vote is
violated in violation of law.

ISSUE:

Whether or not holders of Class “B” shares of MCPI may be deprives of the right to vote and be
voted for as directors.

RULING:

NO.

The 1992 amendment contains a proviso “except as otherwise provided for by law” the law being
referred to by the proviso is that which is in force at the time of the amendment, in this case, was the
Corporation Code.
Under Sec. 6 of the Corporation Code, it provides that no share may be deprived of voting rights
except those classified and issued as “preffered” or “redeemable” shares unless otherwise provided in
this code.there is nothing in the articles of incorporation or an iota of evidence on record that shows that
Class “B” shares were categorized as either preffered or redeemable shares.

247 | P a g e
Law 321_Corporation LAW_ Case Digest

JULIO E. T. SALES and GEORGE V. AGONIAS and SMEC, petitioners,


vs.
SEC, SIHI and ATCO, represented by its President, ANSELMO TRINIDAD; VIMC, represented by its
President, et al., respondents;
G.R. No. 54330 January 13, 1989

FACTS:

SMEC sold 200M common shares of its capital stock in the amount of P2.6M to SIHI under a
Sales Agreement providing that the sale shall be only up to 5m shares per buyer. SIHI requested for the
transfer of the 200M shares to ATCO to which SMEC complied. During the time that ATCO held the
shares, it voted them in the SHs' meetings of SMEC. ATCO in turn sold 198,500,000 of the shares to
respondent VIMC. Upon request, SMEC BOD issued a resolution directing its President to sign the
certificate of stock that would effect the transfer. Before the 1979 annual SH meeting of SMEC,
petitioners sought to nullify the sales of the shares to VIMC with the SEC and to enjoin VIMC from
voting the said shares. VIMC was temporarily restrained and the meeting was held without the
participation of VIMC’s shares and BODs were elected only from the group of petitioners. In VIMC’s
answer, it questioned the said election. SEC denied the petition as well as motion to dismiss and lifted
the Restraining it issued earlier and allowed the shares of VIMC to be counted in determining the
quorum of the 1980 annual SHs meeting, which was already near, and the same shares were allowed to
vote and be voted for. Before the SC, petitioners contended that the SEC gravely abused its discretion in
not enjoining the participation of VIMC in the 1980 election considering that the sale of the shares to
VIMC was null and void as it was done in violation of the Sales Agreement on the limit of shares to be
sold to each buyer and that VIMC’s ownership of the shares is contrary to Sec. 13 (5-A) of the old
corporation law.

ISSUE:

Whether or not SEC acted with grave abuse of discretion in not permanently enjoining VIMC in
voting.

RULING:

YES.

SC found no grave abuse of discretion on the part of the SEC in not restraining VIMC. It adopted
the SEC resolution stating that the sale of the shares of stock had long been perfected and is presumed
valid until declared otherwise. As against this presumption, petitioners' prayer for injunction cannot
prevail as the issue of the validity of the sale is still to be resolved by the SEC.
Considering that the shares constitute the majority, it is more equitable that the same be allowed
to vote rather than be enjoined. As it has been ruled the removal of a majority SH from the management
of the corporation and/or the dissolution of a corporation in a suit filed by a minority SH is a drastic
measure. It should be resorted to only when the necessity is clear. With more reason, the Court will not
deprive a SH of his right to vote his shares in the annual SHs' meeting, except upon a clear showing of
its lawful denial under the articles of incorporation or by-laws of the corporation, as it is a right inherent
in stock ownership.

 Redeemable Preferred

REPUBLIC PLANTERS BANK


vs.
HON. ENRIQUE A. AGANA, SR., as Presiding Judge, Court of First Instance of Rizal, Branch
XXVIII, Pasay City, ROBES-FRANCISCO REALTY & DEVELOPMENT CORPORATION and ADALIA F.
ROBES

248 | P a g e
Law 321_Corporation LAW_ Case Digest

G.R. No. 51765. March 3, 1997

FACTS:

On September 18, 1961, private respondent Corporation secured a loan from petitioner in the
amount of P120,000.00. Instead of giving the legal tender totaling to the full amount of the loan, which
is P120,000.00, petitioner lent such amount partially in the form of money and partially in the form of
stock certificates numbered 3204 and 3205, each for 400 shares with a par value of P10.00 per share,
or for P4,000.00 each, for a total of P8,000.00. Said stock certificates were in the name of private
respondent Adalia F. Robes and Carlos F. Robes, who subsequently, however, endorsed his shares in
favor of Adalia F. Robes.
On January 31, 1979, private respondents proceeded against petitioner and filed a Complaint
anchored on private respondents' alleged rights to collect dividends under the preferred shares in
question and to have petitioner redeem the same under the terms and conditions of the stock
certificates.
The trial court rendered the herein assailed decision in favor of private respondents ordering
petitioner to pay private respondents the face value of the stock certificates as redemption price, plus
1% quarterly interest thereon until full payment.

ISSUES:

Whether or not the respondent court was correct in ordering petitioner to pay private respondents
the face value of the stock certificates as redemption price.

RULING:

NO.

A preferred share of stock is one which entitles the holder thereof to certain preferences over the
holders of common stock. The preferences are designed to induce persons to subscribe for shares of a
corporation.
Preferred shares take a multiplicity of forms. The most common forms may be classified into two:
(1) preferred shares as to assets; and (2) preferred shares as to dividends. The former is a share which
gives the holder thereof preference in the distribution of the assets of the corporation in case of
liquidation;the latter is a share the holder of which is entitled to receive dividends on said share to the
extent agreed upon before any dividends at all are paid to the holders of common stock.There is no
guaranty, however, that the share will receive any dividends.
The redemption of said shares cannot be allowed. As pointed out by the petitioner, the Central
Bank made a finding that said petitioner has been suffering from chronic reserve deficiency, and that
such finding resulted in a directive, issued on January 31, 1973 by then Gov. G.S. Licaros of the Central
Bank, to the President and Acting Chairman of the Board of the petitioner bank prohibiting the latter
from redeeming any preferred share, on the ground that said redemption would reduce the assets of the
Bank to the prejudice of its depositors and creditors.Redemption of preferred shares was prohibited for a
just and valid reason. The directive issued by the Central Bank Governor was obviously meant to
preserve the status quo, and to prevent the financial ruin of a banking institution that would have
resulted in adverse repercussions, not only to its depositors and creditors, but also to the banking
industry as a whole.

 Treasury

COMMISSIONER OF INTERNAL REVENUE


vs.
MANNING, MCDONALD, SIMMONS
No. L-28398 6 August 1975

FACTS:

In 1952 the MANTRASCO had an authorized capital stock of P2,500,000 divided into 25,000
common shares; 24,700 of these were owned by Julius S. Reese, and the rest, at 100 shares each, by
the three respondents.

249 | P a g e
Law 321_Corporation LAW_ Case Digest

On October 19, 1954 Reese died. In 1955, after MANTRASCO made a partial payment of Reese's
shares, the certificate for the 24,700 shares in Reese's name was cancelled and a new certificate was
issued in the name of MANTRASCO, which was endorsed to the law firm of Ross, Selph, Carrascoso and
Janda, as trustees for and in behalf of MANTRASCO. In 1958, at a special meeting of MANTRASCO
stockholders, the following resolution was passed:"RESOLVED, that the 24,700 shares in the Treasury be
reverted back to the capital account of the company as a stock dividend to be distributed to shareholders
of record."
In 1963 the entire purchase price of Reese's interest in MANTRASCO was finally paid in full by
the latter, In 1964 the trust agreement was terminated and the trustees delivered to MANTRASCO all
the shares which they were holding in trust. On the basis of their examination, the BIR examiners
concluded that the distribution of Reese's shares as stock dividends was in effect a distribution of the
"asset or property of the corporation as may be gleaned from the payment of cash for the redemption of
said stock and distributing the same as stock dividend." On April 14, 1965 the CIR issued notices of
assessment for deficiency income taxes to the respondents for the year 1958.

ISSUE:

Whether or not the issuance of the notices of assessment for deficiency income taxes to the
respondents for the year 1958 was proper.

RULING:

YES.

The declaration by the respondents and Reese's trustees of MANTRASCO's alleged treasury stock
dividends in favor of the former, brings, however, into clear focus the ultimate purpose which the parties
to the trust instrument aimed to realize: to make the respondents the sole owners of Reese's interest in
MANTRASCO by utilizing the periodic earnings of that company and its subsidiaries to directly subsidize
their purchase of the said interests, and by making it appear outwardly, through the formal declaration
of non-existent stock dividends in the treasury, that they have not received any income from those firms
when, in fact, by that declaration they secured to themselves the means to turn around as full owners of
Reese's shares. In other words, the respondents, using the trust instrument as a convenient technical
device, bestowed unto themselves the full worth and value of Reese's corporate holdings with the use of
the very earnings of the companies. Such package device, obviously not designed to carry out the usual
stock dividend purpose of corporate expansion reinvestment, e.g. the acquisition of additional facilities
and other capital budget items, but exclusively for expanding the capital base of the respondents in
MANTRASCO, cannot be allowed to deflect the respondents' responsibilities toward our income tax laws.
The conclusion is thus ineluctable that whenever the companies involved herein parted with a portion of
their earnings "to buy" the corporate holdings of Reese, they were in ultimate effect and result making a
distribution of such earnings to the respondents. All these amounts are consequently subject to income
tax as being, in truth and in fact, a flow of cash benefits to the respondents.

SAN MIGUEL CORPORATION


vs.
SANDIGANBAYAN (First Division), EDUARDO M. COJUANGCO, JR., et al.,
G.R. No. 118661 SEPTEMBER 14, 2000

FACTS:

CIIF sold 33,133,266 shares of the outstanding capital stock of SMC to Andres Soriano III of the
SMC Group payable in 4 installments. April 1, 1986, Soriano paid the initial 500M to the UCPB as
administrator of CIIF. The sale was transacted through the stock exchange and the shares were
registered in the name of AHSI. On April 7, 1986, PCGG sequestered the shares subject of the sale,
thus SMC suspended payment of the balance. UCPB, filed a complaint for rescission and damages with
the RTC Makati. SMC assailed the RTC jurisdiction on the ground that primary jurisdiction was vested
with the PCGG since the SMC shares were sequestered shares. SC dismissed the complaint for
rescission without prejudice to the ventilation of the parties’ claims before the Sandiganbayan (SBN).
The Republic, thru the OSG opposed the Agreement contending that the involved coco-levy funds,
whether in the form of earnings or dividends therefrom, or in the form of the value of liquidated
corporate assets represented by all sequestered share (like the value of assets sold/mortgaged to finance
the 500M 1st installment), or in the form of cash, or, as in the case of subject Settlement, in the form of
proceeds of sale or of payments of certain alleged obligations are public funds and are beyond or outside
the commerce and not within the private disposition of private individuals.

250 | P a g e
Law 321_Corporation LAW_ Case Digest

ISSUE:

Whether or not there is a valid delivery of certificates of stock of smc shares and the dividends
thereon to the PCGG.

RULING:

YES.

No grave abuse of discretion on the part of SBN when it ordered petitioners to deliver the treasury
shares to the PCGG and pay their corresponding dividends for the following reasons: Under the
Corporation Code, Treasury shares are shares of stocks which have been issued and fully paid for, but
subsequently reacquired by, the issuing corporation by purchase, redemption, donation or through
some lawful means. These 26.45M shares or any portion thereof can, therefore, become treasury
shares, i.e., property of SMC, only if the sale between the UCPB Group and the SMC Group is allowed;
otherwise these shares cannot even begin to be deemed to have been re-acquired by the issuing
corporation, the SMC.

 Trust Fund Doctrine

NATIONAL TELECOMMUNICATIONS COMMISSION


vs.
COURT OF APPEALS AND PHILIPPINE LONG DISTANCE TELEPHONE COMPANY
G.R. NO. 127937 1999 JULY 28

FACTS:

Sometime in 1988, the National Telecommunications Commission (NTC) served on the Philippine
Long Distance Telephone Company (PLDT) assessment notices and demands for payment. PLDT
challenged the aforesaid assessments. On September 29, 1993, the NTC rendered a Decision, denying
the protest of PLDT. On October 22, 1993, PLDT interposed a Motion for Reconsideration, which was
denied by NTC in an Order issued on May 3, 1994. On May 12, 1994, PLDT appealed the aforesaid
Decision to the Court of Appeals, which came out with its questioned Decision of October 30, 1996,
modifying the disposition of NTC. On November 20, 1996, NTC moved for partial reconsideration of the
Decision, with respect to the basis of the assessment under Section 40(e), i.e., par value of the
subscribed capital stock. It also sought a partial reconsideration of the fee of fifty (P0.50) centavos for
the issuance or increasing of the capital stock under Section 40 (f). With the denial of its motions for
reconsideration by the Resolution of the Court of Appeals dated January 27, 1997, petitioner found its
way to the Court via the Petition for Certiorari.

ISSUE:

Whether or not the Court of Appeals erred in holding that the computation of Supervision and
regulation Fees under Section 40(f) of the Public Service Act should be based on the par value of the
subscribed capital stock.

251 | P a g e
Law 321_Corporation LAW_ Case Digest

RULING:

NO.

Succinct and clear is the ruling of this Court in the case of Philippine Long Distance Telephone
Company vs. Public Service Commission, 66 SCRA 341, that the basis for computation of the fee to be
charged by NTC on PLDT, is “the capital stock subscribed or paid and not, alternatively, the property
and equipment.” The fee in question is based on the capital stock subscribed or paid, nothing less
nothing more.
The “Trust Fund” doctrine considers this subscribed capital as a trust fund for the payment of the debts
of the corporation, to which the creditors may look for satisfaction. Until the liquidation of the
corporation, no part of the subscribed capital may be returned or released to the stockholder (except in
the redemption of redeemable shares) without violating this principle. Thus, dividends must never
impair the subscribed capital; subscription commitments cannot be condoned or remitted; nor can the
corporation buy its own shares using the subscribed capital as the consideration therefore.
In the same way that the Court in PLDT vs. PSC has rejected the “value of the property and
equipment” as being the proper basis for the fee imposed by Section 40(e) of the Public Service Act, as
amended by Republic Act No. 3792, so also must the Court disallow the idea of computing the fee on
“the par value of [PLDT’s] capital stock subscribed or paid excluding stock dividends, premiums, or
capital in excess of par.” Neither is the assessment made by the National Telecommunications
Commission on the basis of the market value of the subscribed or paid-in capital stock acceptable since
it is itself a deviation from the explicit language of the law.

ONG YONG, et al., petitioner


vs.
TIU, et al., respondent
G.R. No. 144476 8 April 2003

FACTS:

In 1994, the construction of the Masagana Citimall in Pasay City was threatened with stoppage
and incompletion when its owner, the First Landlink Asia Development Corporation (FLADC), which was
owned by David S. Tiu, Cely Y. Tiu, Moly Yu Gow, Belen See Yu, D. Terence Y. Tiu, John Yu and
Lourdes C. Tiu (the Tius), encountered dire financial difficulties. It was heavily indebted to the Philippine
National Bank (PNB) for P190 million. To stave off foreclosure of the mortgage on the two lots where the
mall was being built, the Tius invited Ong Yong, Juanita Tan Ong, Wilson T. Ong, Anna L. Ong, William
T. Ong and Julia Ong Alonzo (the Ongs), to invest in FLADC. Under the Pre-Subscription Agreement
they entered into, the Ongs and the Tius agreed to maintain equal shareholdings in FLADC: the Ongs
were to subscribe to 1,000,000 shares at a par value of P100.00 each while the Tius were to subscribe
to an additional 549,800 shares at P100.00 each in addition to their already existing subscription of
450,200 shares. Furthermore, they agreed that the Tius were entitled to nominate the Vice-President
and the Treasurer plus 5 directors while the Ongs were entitled to nominate the President, the Secretary
and 6 directors (including the chairman) to the board of directors of FLADC. Moreover, the Ongs were
given the right to manage and operate the mall. Accordingly, the Ongs paid P100 million in cash for
their subscription to 1,000,000 shares of stock while the Tius committed to contribute to FLADC a four-
storey building and two parcels of land respectively valued at P20 million (for 200,000 shares), P30
million (for 300,000 shares) and P49.8 million (for 49,800 shares) to cover their additional 549,800
stock subscription therein. The Ongs paid in another P70 million 3 to FLADC and P20 million to the
Tius over and above their P100 million investment, the total sum of which (P190 million) was used to
settle the P190 million mortgage indebtedness of FLADC to PNB. The business harmony between the
Ongs and the Tius in FLADC, however, was shortlived because the Tius, on 23 February 1996,
rescinded the Pre-Subscription Agreement. The Tius accused the Ongs of (1) refusing to credit to them
the FLADC shares covering their real property contributions; (2) preventing David S. Tiu and Cely Y. Tiu
from assuming the positions of and performing their duties as Vice-President and Treasurer,
respectively, and (3) refusing to give them the office spaces agreed upon. The controversy finally came to
a head when the case was commenced by the Tius on 27 February 1996 at the Securities and Exchange
Commission (SEC), seeking confirmation of their rescission of the Pre-Subscription Agreement.

252 | P a g e
Law 321_Corporation LAW_ Case Digest

ISSUE:

Whether or not the rescission of Pre-Subscription Agreement would result in unauthorized


liquidation.

RULING:

YES.

The rescission of the Pre-Subscription Agreement will effectively result in the unauthorized
distribution of the capital assets and property of the corporation, thereby violating the Trust Fund
Doctrine and the Corporation Code, since rescission of a subscription agreement is not one of the
instances when distribution of capital assets and property of the corporation is allowed. Rescission will,
in the final analysis, result in the premature liquidation of the corporation without the benefit of prior
dissolution in accordance with Sections 117, 118, 119 and 120 of the Corporation Code.

 What is a “Subscription”

ONG YONG, et al., petitioner


vs.
TIU, et al., respondent
G.R. No. 144476 8 April 2003

FACTS:

In 1994, the construction of the Masagana Citimall in Pasay City was threatened with stoppage
and incompletion when its owner, the First Landlink Asia Development Corporation (FLADC), which was
owned by David S. Tiu, Cely Y. Tiu, Moly Yu Gow, Belen See Yu, D. Terence Y. Tiu, John Yu and
Lourdes C. Tiu (the Tius), encountered dire financial difficulties. It was heavily indebted to the Philippine
National Bank (PNB) for P190 million. To stave off foreclosure of the mortgage on the two lots where the
mall was being built, the Tius invited Ong Yong, Juanita Tan Ong, Wilson T. Ong, Anna L. Ong, William
T. Ong and Julia Ong Alonzo (the Ongs), to invest in FLADC. Under the Pre-Subscription Agreement
they entered into, the Ongs and the Tius agreed to maintain equal shareholdings in FLADC: the Ongs
were to subscribe to 1,000,000 shares at a par value of P100.00 each while the Tius were to subscribe
to an additional 549,800 shares at P100.00 each in addition to their already existing subscription of
450,200 shares. Furthermore, they agreed that the Tius were entitled to nominate the Vice-President
and the Treasurer plus 5 directors while the Ongs were entitled to nominate the President, the Secretary
and 6 directors (including the chairman) to the board of directors of FLADC. Moreover, the Ongs were
given the right to manage and operate the mall. Accordingly, the Ongs paid P100 million in cash for
their subscription to 1,000,000 shares of stock while the Tius committed to contribute to FLADC a four-
storey building and two parcels of land respectively valued at P20 million (for 200,000 shares), P30
million (for 300,000 shares) and P49.8 million (for 49,800 shares) to cover their additional 549,800
stock subscription therein. The Ongs paid in another P70 million 3 to FLADC and P20 million to the
Tius over and above their P100 million investment, the total sum of which (P190 million) was used to
settle the P190 million mortgage indebtedness of FLADC to PNB.

ISSUE:

Whether or not the pre-Subscription Agreement executed by the Ongs is actually a subscription
contract.

RULING:

YES.

FLADC was originally incorporated with an authorized capital stock of 500,000 shares with the
Tius owning 450,200 shares representing the paid-up capital. When the Tius invited the Ongs to invest
in FLADC as stockholders, an increase of the authorized capital stock became necessary to give each
group equal (50-50) shareholdings as agreed upon in the Pre-Subscription Agreement. The authorized
capital stock was thus increased from 500,000 shares to 2,000,000 shares with a par value of P100

253 | P a g e
Law 321_Corporation LAW_ Case Digest

each, with the Ongs subscribing to 1,000,000 shares and the Tius to 549,800 more shares in addition to
their 450,200 shares to complete 1,000,000 shares. Thus, the subject matter of the contract was the
1,000,000 unissued shares of FLADC stock allocated to the Ongs. Since these were unissued shares,
the parties' Pre-Subscription Agreement was in fact a subscription contract as defined under Section 60,
Title VII of the Corporation Code. A subscription contract necessarily involves the corporation as one of
the contracting parties since the subject matter of the transaction is property owned by the corporation
— its shares of stock. Thus, the subscription contract (denominated by the parties as a Pre-Subscription
Agreement) whereby the Ongs invested P100 million for 1,000,000 shares of stock was, from the
viewpoint of the law, one between the Ongs and FLADC, not between the Ongs and the Tius.

254 | P a g e
Law 321_Corporation LAW_ Case Digest

BAYLA, et al., petitioner


vs.
SILANG TRAFFIC CO., INC., respondent
G.R. Nos. L-48195 and 48196 May 1, 1942

FACTS:

Petitioners in G.R. No. 48195 instituted this action in the Court of First Instance of Cavite against
the respondent Silang Traffic Co., Inc. (cross-petitioner in G.R. No. 48196), to recover certain sums of
money which they had paid severally to the corporation on account of shares of stock they individually
agreed to take and pay for under certain specified terms and conditions. The agreements signed by the
other petitioners were of the same date (March 30, 1935) and in identical terms as the foregoing except
as to the number of shares and the corresponding purchase price. The petitioners agreed to purchase a
total of 46 shares and, up to April 30, 1937, had paid the corresponding amount on account
thereof.Petitioners' action for the recovery of the sums above mentioned is based on a resolution by the
board of directors of the respondent corporation on August 1, 1937.
The respondent corporation set up the following defenses: (1) That the above-quoted resolution is
not applicable to the petitioners Sofronio T. Bayla, Josefa Naval, and Paz Toledo because on the date
thereof "their subscribed shares of stock had already automatically reverted to the defendant, and the
installments paid by them had already been forfeited"; and (2) that said resolution of August 1, 1937,
was revoked and cancelled by a subsequent resolution of the board of directors of the defendant
corporation dated August 22, 1937.

ISSUE:

Whether or not the agreement was a contract of subscription to the capital stock of the
respondent corporation.

RULING:

NO.

Whether a particular contract is a subscription or a sale of stock is a matter of construction and


depends upon its terms and the intention of the parties. In the Unson case just cited, this Court held
that a subscription to stock in an existing corporation is, as between the subscriber and the
corporation, simply a contract of purchase and sale.
It seems clear from the terms of the contracts in question that they are contracts of sale and not
of subscription. The lower courts erred in overlooking the distinction between subscription and
purchase "A subscription, properly speaking, is the mutual agreement of the subscribers to take and
pay for the stock of a corporation, while a purchase is an independent agreement between the individual
and the corporation to buy shares of stock from it at stipulated price." In some particulars the rules
governing subscriptions and sales of shares are different. For instance, the provisions of our Corporation
Law regarding calls for unpaid subscription and assessment of stock do not apply to a purchase of
stock. Likewise the rule that corporation has no legal capacity to release an original subscriber to its
capital stock from the obligation to pay for his shares, is inapplicable to a contract of purchase of
shares.

255 | P a g e
Law 321_Corporation LAW_ Case Digest

SALMON, DEXTER & Co., plaintiff


vs.
TIMOTEO UNSON, defendant
G.R. No. L-23608 March 17, 1925

FACTS:

The plaintiff seeks to recover of the defendant the sum of P1,000 with legal interest on a
subscription for capital stock contract. The defense is that the defendant is released from his obligation
on the subscription agreement by virtue of the increase of the capital stock of the plaintiff from
P250,000, the amount mentioned in the agreement, to P500,000, the amount agreed upon the
stockholders prior to the defendant's signing the agreement. On this issue, judgment in the lower court
was with the plaintiff.
The plaintiff is Salmon, Dexter and Company, a domestic corporation. It was organized under the
name of C.S. Salmon and Company on May 28, 1918, with a capital stock of P250,000. Thereafter,
pursuant to a resolution of the board of directors of the corporation of June 24, 1920, a meeting of the
stockholders was had on July 14, 1920, at which the capital stock of C.S. Salmon and Company was
increased to P500,000. The certificate of increase of capital stock from P250,000 to P500,000, and
articles of incorporation, as amended, of Salmon, Dexter and Company were filed with the Mercantile
Registry of the Bureau of Commerce and Industry on September 16, 1920.
On July 28, 1920, Timoteo Unson, the defendant, to follow the allegation in the third paragraph
of the complaint, "became a subscriber of C.S. Salmon and Company, by signing an agreement in
writing and delivering the same to C.S. Salmon and Company, the name of which company was later
changed to Salmon, Dexter and Company."

ISSUE:

Whether or not the contract entered into by the parties is a Subscription contract.

RULING:

YES.

After incorporation, one may become a shareholder by subscription, or by purchasing stock


directly from the corporation, or from individual owners thereof. A distinction is drawn by the
authorities between a subscription to the capital stock of the corporation after its organization and a
sale of shares by it. Whether a particular contract is a subscription or a sale of stock is a matter of
construction, and depends upon its terms and the intention of the parties. It has been held that a
subscription to stock in an existing corporation is, as between the subscriber and the corporation,
simply a contract of purchase and sale.
Admitting that the terminology of the agreement is not conclusive, and admitting that it is a
contract between a subscriber and the corporation, and thus simply a contract of purchase and sale,
then under the last hypothesis we have to determine if the contract is avoided by misrepresentation.
In our opinion, a contract different from that which was entered into cannot be made for the
parties and imposed upon Unson. Unson has the right to stand upon the contract he has made. In our
opinion also, there was such a non-disclosure of a material fact as was equivalent to false
representation. This representation was of a character that the party to whom it was made had a right to
rely upon it.

SUNSET VIEW CONDOMINIUM CORPORATION, petitioner


vs.
CAMPOS, respondent
G.R. No. L-52361 April 27, 1981

FACTS:

The private respondent, Aguilar-Bernares Realty, a sole proprietorship with business name
registered with the Bureau of Commerce, owned and operated by the spouses Emmanuel G. Aguilar and

256 | P a g e
Law 321_Corporation LAW_ Case Digest

Zenaida B. Aguilar, is the assignee of a unit, "Solana", in the Sunset View Condominium Project with La
Perla Commercial, Incorporated, as assignor. 3 The La Perla Commercial, Incorporated bought the
"Solana" unit on installment from the Tower Builders, Inc. 4 The petitioner, Sunset View Condominium
Corporation, filed for the collection of assessments levied on the unit against Aguilar-Bernares Realty,
private respondent herein, a complaint dated June 22, 1979 docketed as Civil Case No. 7303-P of the
Court of First Instance of Pasay City, Branch XXX. The private respondent filed a Motion to Dismiss the
complaint on the grounds (1) that the complaint does not state a cause of action: (2) that the court has
no jurisdiction over the subject or nature other action; and (3) that there is another action pending
between the same parties for the same cause. The petitioner filed its opposition thereto. The motion to
dismiss was granted on December 11, 1979 by the respondent Judge who opined that the private
respondent is, pursuant to Section 2 of Republic Act No. 4726, a "holder of a separate interest" and
consequently, a shareholder of the plaintiff condominium corporation; and that "the case should be
properly filed with the Securities & Exchange Commission which has exclusive original jurisdiction on
controversies arising between shareholders of the corporation." the motion for reconsideration thereof
having been denied, the petitioner, alleging grave abuse of discretion on the part of respondent Judge,
filed the instant petition for certiorari praying that the said orders be set aside.

ISSUE:

Whether or not a purchaser of a condominium unit in the condominium project managed by the
petitioner, who has not yet fully paid the purchase price thereof, automatically a stockholder of the
petitioner Condominium Corporation.

RULING:

NO.

The share of stock appurtenant to the unit win be transferred accordingly to the purchaser of the
unit only upon full payment of the purchase price at which time he will also become the owner of the
unit. Consequently, even under the contract, it is only the owner of a unit who is a shareholder of the
Condominium Corporation. Inasmuch as owners is conveyed only upon full payment of the purchase
price, it necessarily follows that a purchaser of a unit who has not paid the full purchase price thereof is
not The owner of the unit and consequently is not a shareholder of the Condominium Corporation.
That only the owner of a unit is a stockholder of the Condominium Corporation is inferred from
Section 10 of the Condominium Act. Pursuant to such statutory provision, ownership of a unit is a
condition sine qua non to being a shareholder in the condominium corporation. It follows that a
purchaser of a unit who is not yet the owner thereof for not having fully paid the full purchase price, is
not a shareholder By necessary implication, the "separate interest" in a condominium, which entitles the
holder to become automatically a share holder in the condominium corporation, as provided in Section 2
of the Condominium Act, can be no other than ownership of a unit. This is so because nobody can be a
shareholder unless he is the owner of a unit and when he ceases to be the owner, he also ceases
automatically to be a shareholder.
The private respondents, therefore, who have not fully paid the purchase price of their units and are
consequently not owners of their units are not members or shareholders of the petitioner condominium
corporation.

257 | P a g e
Law 321_Corporation LAW_ Case Digest

VELASCO, petitioner
vs.
POIZAT, respondent
G.R. No. L-11528 March 15, 1918

FACTS:

From the amended complaint filed in this cause upon February 5, 1915, it appears that the
plaintiff, as assignee in insolvency of "The Philippine Chemical Product Company" (Ltd.) is seeking to
recover of the defendant, Jean M. Poizat, the sum of P1,500, upon a subscription made by him to the
corporate stock of said company. It appears that the corporation in question was originally organized by
several residents of the city of Manila, where the company had its principal place of business, with a
capital of P50,000, divided into 500 shares. The defendant subscribed for 20 shares of the stock of the
company, an paid in upon his subscription the sum of P500, the par value of 5 shares . The action was
brought to recover the amount subscribed upon the remaining shares.
It appears that the defendant was a stock holder in the company from the inception of the
enterprise, and for sometime acted as its treasurer and manager. While serving in this capacity he called
in and collected all subscriptions to the capital stock of the company, except the aforesaid 15 shares
subscribed by himself and another 15 shares owned by Jose R. Infante.
Upon July 13, 1914, a meeting of the board of directors of the company was held at which a
majority of the stock was presented. Upon this occasion two resolutions, important to be here noted,
were adopted. The first was a proposal that the directors, or shareholders, of the company should make
good by new subscriptions, in proportion to their respective holdings, 15 shares which had been
surrendered by Infante.

ISSUE:

Whether or not Poizat is liable for his unpaid subscription.

RULING:

YES.

A stock subscription is a contract between the corporation on one side, and the subscriber on the
other, and courts will enforce it for or against either. It is a rule, accepted by the Supreme Court of the
United States that a subscription for shares of stock does not require an express promise to pay the
amount subscribed, as the law implies a promise to pay on the part of the subscriber. Section 36 of the
Corporation Law clearly recognizes that a stock subscription is subsisting liability from the time the
subscription is made, since it requires the subscriber to pay interest quarterly from that date unless he
is relieved from such liability by the by-laws of the corporation. The subscriber is as much bound to pay
the amount of the share subscribed by him as he would be to pay any other debt, and the right of the
company to demand payment is no less incontestable.
The provisions of the Corporation Law (Act No. 1459) give recognition of two remedies for the
enforcement of stock subscriptions. The first and most special remedy given by the statute consists in
permitting the corporation to put up the unpaid stock for sale and dispose of it for the account of the
delinquent subscriber. In this case the provisions of section 38 to 48, inclusive, of the Corporation Law
are applicable and must be followed.
It is generally accepted doctrine that the statutory right to sell the subscriber's stock is merely a
remedy in addition to that which proceeds by action in court; and it has been held that the ordinary
legal remedy by action exists even though no express mention thereof is made in the statute.

 Acquisition and Ownership of Shares in a Corporation; Extent of Proprietary


Right/Doctrine of Limited Liability

Conjuangco
vs
Republic
12 April 2011 GR NO. 166859

FACTS:

258 | P a g e
Law 321_Corporation LAW_ Case Digest

Last April 12, 2011, the Supreme Court en banc rendered its ruling on one of the most crucial
case against the ill-gotten wealth of the Marcoses and their associates dating back from the Martial Law
era involving the shares in San Miguel Corporation (SMC) allegedly bought with coconut levy funds
exacted from the poor marginal coconut farmers all over the country. This block of shares was
purportedly owned by businessman Eduardo “Danding” Cojuangco and is one of many cases filed way
back in 1987 by the Presidential Commission on Good Government, as part of its mandate to recover
illgotten wealth.
In a nutshell, four justices of the Supreme Court ruled that the Government of the Philippines
(dubbed as the Republic of the Philippines or Republic, for short, in this case) failed to submit further
evidence to prove that the loans from United Coconut Planters Bank (UCPB) and the Coconut Industry
Investment Fund (CIIF) secured by Eduardo “Danding” Cojuangco to purchase the shares of SMC, were
public in character. A dissenting opinion by Justice Conchita CarpioMorales held the view that it was
Dganding who has failed in his burden of showing that such funds were not taken from public funds,
while another dissenting opinion, by Justice Brion, takes the view of the majority handing a loss to the
Government but urging instead a prosecution of the government lawyers handling the case of Danding,
for their mishandling of the case, citing several instances when they could have presented stronger
evidence and have taken other steps to bolster their case, but didn't.

ISSUE :

Whether or not Cojuangco breach his “fiduciary duties” as an officer and member of the Board of
Directors of the UCPB? Did his acquisition and holding of the contested SMC shares come under a
constructive trust in favor of the Republic.

RULING:

NO.

The thrust of the Republic that the funds were borrowed or lent might even preclude any
consequent trust implication. In a contract of loan, one of the parties (creditor) delivers money or other
consumable thing to another (debtor) on the condition that the same amount of the same kind and
quality shall be paid.
To say that a relationship is fiduciary when existing laws do not provide for such requires
evidence that confidence is reposed by one party in another who exercises dominion and influence.
Absent any special facts and circumstances proving a higher degree of responsibility, any dealings
between a lender and borrower are not fiduciary in nature. This explains why, for example, a trust
receipt transaction is not classified as a simple loan and is characterized as fiduciary, because the Trust
Receipts Law punishes the dishonesty and abuse of confidence in the handling of money or goods to the
prejudice of another regardless of whether the latter is the owner.
Based on the foregoing, a debtor can appropriate the thing loaned without any responsibility or
duty to his creditor to return the very thing that was loaned or to report how the proceeds were used.
Nor can he be compelled to return the proceeds and fruits of the loan, for there is nothing under our
laws that compel a debtor in a contract of loan to do so. As owner, the debtor can dispose of the thing
borrowed and his act will not be considered misappropriation of the thing.

MANUEL C. ESPIRITU, JR., AUDIE LLONA, FREIDA F. ESPIRITU, CARLO F. ESPIRITU, RAFAEL F.
ESPIRITU, ROLANDO M. MIRABUNA, HERMILYN A. MIRABUNA, KIM ROLAND A. MIRABUNA,
KAYE ANN A. MIRABUNA, KEN RYAN A. MIRABUNA, JUANITO P. DE CASTRO, GERONIMA A.
ALMONITE and MANUEL C. DEE, who are the officers and directors of BICOL GAS REFILLING
PLANT CORPORATION
vs.
PETRON CORPORATION and CARMEN J. DOLOIRAS, doing business under the name "KRISTINA
PATRICIA ENTERPRISES,
G.R. No. 170891 November 24, 2009

FACTS:

Petron sold and distributed LPG in cylinder tanks that carried its trademark
"Gasul."1 Respondent Carmen J. Doloiras owned and operated Kristina Patricia Enterprises, the
exclusive distributor of Gasul LPGs in the whole of Sorsogon. Jose Nelson Doloiras served as KPE’s
manager. Bicol Gas Refilling Plant Corporation was also in the business of selling and distributing LPGs
in Sorsogon but theirs carried the trademark "Bicol Savers Gas." Petitioner Audie Llona managed Bicol
Gas.

259 | P a g e
Law 321_Corporation LAW_ Case Digest

On August 4, 2001 KPE’s Jose saw a particular Bicol Gas truck on the Maharlika Highway. While
the truck carried mostly Bicol Savers LPG tanks, it had on it one unsealed 50-kg Gasul tank and one
50-kg Shellane tank. He offered to make a swap for these but Llona declined, saying the Bicol Gas
owners wanted to send those tanks to Batangas. Later Bicol Gas told Jose that it had no more Gasul
tanks left in its possession.

ISSUE:

Whether or not all the Petitioners are liable.

RULING:

NO.

The "owners" of a corporate organization are its stockholders and they are to be distinguished
from its directors and officers. The petitioners here, with the exception of Audie Llona, are being charged
in their capacities as stockholders of Bicol Gas. But the Court of Appeals forgets that in a corporation,
the management of its business is generally vested in its board of directors, not its
stockholders. Stockholders are basically investors in a corporation. They do not have a hand in running
the day-to-day business operations of the corporation unless they are at the same time directors or
officers of the corporation. Before a stockholder may be held criminally liable for acts committed by the
corporation, therefore, it must be shown that he had knowledge of the criminal act committed in the
name of the corporation and that he took part in the same or gave his consent to its commission,
whether by action or inaction.
The finding of the Court of Appeals that the employees "could not have committed the crimes
without the consent, [abetment], permission, or participation of the owners of Bicol Gas" is a sweeping
speculation especially since, as demonstrated above, what was involved was just one Petron Gasul tank
found in a truck filled with Bicol Gas tanks. Although the KPE manager heard petitioner Llona say that
he was going to consult the owners of Bicol Gas regarding the offer to swap additional captured
cylinders, no indication was given as to which Bicol Gas stockholders Llona consulted. It would be
unfair to charge all the stockholders involved, some of whom were proved to be minors. No evidence was
presented establishing the names of the stockholders who were charged with running the operations of
Bicol Gas. The complaint even failed to allege who among the stockholders sat in the board of directors
of the company or served as its officers.

CRISOSTOMO, petitioner
vs.
S.E.C, respondent
G.R. Nos. 89095 & 89555 November 6, 1989

FACTS:

Sixto Crisostomo, Felipe Crisostomo (deceased), Veronica Palanca, Juanito Crisostomo, Carlos
Crisostomo, Ricardo Alfonso, Regino Crisostomo and Ernesto Crisostomo (known as the Crisostomo
group) were the original stockholders of the United Doctors Medical Center (UDMC) which was organized
in 1968 with an authorized capital stock of P1,000,000 (later increased to P15,000,000 in 1972). They
owned approximately 40% of UDMC's outstanding capital stock, while the 60% majority belonged to the
members of the United Medical Staff Association (UMSA), numbering approximately 150 doctors and
medical personnel of UDMC.
In 1988, UDMC defaulted in paying its loan obligation of approximately P55 million to the DBP.
In the last quarter of 1987, UDMC's assets (principally its hospital) and those of the Crisostomos which
had been given as collateral to the DBP, faced foreclosure by the Asset Privatization' rust (APT), which
had taken over UDMC's loan obligation to the DBP.
To stave off the threatened foreclosure, UDMC, through its principal officers, Ricardo Alfonso and
Juanito Crisostomo, persuaded the Yamadas and Enatsu (Shoji Yamada and Tomotada Enatsu are
Japanese doctors) to invest fresh capital in UDMC. The wife of Tomotada Enatsu, Edita Enatsu, is a
Filipina. They invested approximately P57 million in UDMC.
The investment was effected by means of: (1) a Stock Purchase Agreement; and (2) an Amended
Memorandum of Agreement whereby the group subscribed to 82.09% of the outstanding shares of

260 | P a g e
Law 321_Corporation LAW_ Case Digest

UDMC. Upon the completion of the governmental approval process, shares of stock, duly signed by
UDMC's authorized officers, were issued to the Yamadas and Enatsus.
As it had been agreed in the Amended Memorandum of Agreement between UDMC and the
Japanese group that upon the latter's acquisition of the controlling interest in UDMC, the corporation
would be reorganized, a special stockholders' meeting and board of directors' meeting were scheduled to
be held on August 20, 1988.

ISSUE:

Whether or not the investment of the Japanese group in UDMC is unconstitutional.

RULING:

NO.

While 82% of UDMC's capital stock is indeed subscribed by the Japanese group, only 30%
(equivalent to 171,721 shares or P17,172.00) is owned by the Japanese citizens, namely, the Yamada
spouses and Tomotada Enatsu. 52% is owned by Edita Enatsu, who is a Filipino. Accordingly, in its
application for approval/registration of the foreign equity investments of these investors, UDMC
declared that 70% of its capital stock is owned by Filipino citizens, including Edita Enatsu. That
application was approved by the Central Bank on August 3, 1988.
The investments in UDMC of Doctors Yamada and Enatsu do not violate the Constitutional
prohibition against foreigners practising a profession in the Philippines (Section 14, Article XII, 1987
Constitution) for they do not practice their profession (medicine) in the Philippines, neither have they
applied for a license to do so. They only own shares of stock in a corporation that operates a hospital. No
law limits the sale of hospital shares of stock to doctors only. The ownership of such shares does not
amount to engaging (illegally,) in the practice of medicine, or, nursing. If it were otherwise, the
petitioner's stockholding in UDMC would also be illegal.

GARCIA, petitioner
vs.
LIM CHU SING, respondent
G.R. No. L-39427 February 24, 1934

FACTS:

On June 20, 1930, the defendant-appellant Lim Chu Sing executed and delivered to the
Mercantile Bank of China promissory note for the sum of P19,605.17 with interest thereon at 6 per cent
per annum, payable monthly as follows: P1,000 on July 1, 1930; P500 on August 1, 1930; and P500 on
the first of every month thereafter until the amount of the promissory note together with the interest
thereon is fully paid (Exhibit A). One of the conditions stipulated in said promissory note is that in case
of defendant's default in the payment of any of the monthly installments, as they become due, the entire
amount or the unpaid balance thereof together with interest thereon at 6 per cent per annum, shall
become due and payable on demand. The defendant had been, making several partial payments
thereon, leaving an unpaid balance of P9,105.17. However, he defaulted in the payment of several
installments by reason of which the unpaid balance of P9,105.17 on the promissory note has ipso
facto become due and demandable.

ISSUE:

Whether or not it is proper to compensate the defendant-appellant's indebtedness of P9,105.17,


which is claimed in the complaint, with the sum of P10,000 representing the value of his shares of stock
with the plaintiff entity, the Mercantile Bank of China.

RULING:

NO.

According to the weight of authority, a share of stock or the certificate thereof is not indebtedness
to the owner or evidence of indebtedness and, therefore, it is not a credit. Stockholders, as such, are not
creditors of the corporation. It is the prevailing doctrine of the American courts, repeatedly asserted in
the broadest terms, that the capital stock of a corporation is a trust fund to be used more particularly
for the security of creditors of the corporation, who presumably deal with it on the credit of its capital

261 | P a g e
Law 321_Corporation LAW_ Case Digest

stock. Therefore, the defendant-appellant Lim Chu Sing not being a creditor of the Mercantile Bank of
China, although the latter is a creditor of the former, there is no sufficient ground to justify
compensation.

262 | P a g e
Law 321_Corporation LAW_ Case Digest

MAGSAYSAY-LABRADOR, petitioner
vs.
COURT OF APPEALS, respondent
G.R. No. 58168 December 19, 1989

FACTS:

On February 9, 1979, Adelaida Rodriguez-Magsaysay, widow and special administratix of the


estate of the late Senator Genaro Magsaysay, brought before the then Court of First Instance of
Olongapo an action against Artemio Panganiban, Subic Land Corporation (SUBIC), Filipinas
Manufacturer's Bank (FILMANBANK) and the Register of Deeds of Zambales. In her complaint, she
alleged that in 1958, she and her husband acquired, thru conjugal funds, a parcel of land with
improvements, known as "Pequena Island", covered by TCT No. 3258; that after the death of her
husband, she discovered [a] an annotation at the back of TCT No. 3258 that "the land was acquired by
her husband from his separate capital;" [b] the registration of a Deed of Assignment dated June 25,
1976 purportedly executed by the late Senator in favor of SUBIC, as a result of which TCT No. 3258 was
cancelled and TCT No. 22431 issued in the name of SUBIC; and [c] the registration of Deed of Mortgage
dated April 28, 1977 in the amount of P 2,700,000.00 executed by SUBIC in favor of FILMANBANK; that
the foregoing acts were void and done in an attempt to defraud the conjugal partnership considering
that the land is conjugal, her marital consent to the annotation on TCT No. 3258 was not obtained, the
change made by the Register of Deeds of the titleholders was effected without the approval of the
Commissioner of Land Registration and that the late Senator did not execute the purported Deed of
Assignment or his consent thereto, if obtained, was secured by mistake, violence and intimidation. She
further alleged that the assignment in favor of SUBIC was without consideration and consequently null
and void. She prayed that the Deed of Assignment and the Deed of Mortgage be annulled and that the
Register of Deeds be ordered to cancel TCT No. 22431 and to issue a new title in her favor.

ISSUE:

Whether or not petitioner’s ownership in the outstanding capital stock of SUBIC entitles them to
a significant vote in the corporate affairs.

RULING:

NO.

The words "an interest in the subject" mean a direct interest in the cause of action as pleaded,
and which would put the intervenor in a legal position to litigate a fact alleged in the complaint, without
the establishment of which plaintiff could not recover.
Here, the interest, if it exists at all, of petitioners-movants is indirect, contingent, remote,
conjectural, consequential and collateral. At the very least, their interest is purely inchoate, or in sheer
expectancy of a right in the management of the corporation and to share in the profits thereof and in the
properties and assets thereof on dissolution, after payment of the corporate debts and obligations.
While a share of stock represents a proportionate or aliquot interest in the property of the
corporation, it does not vest the owner thereof with any legal right or title to any of the property, his
interest in the corporate property being equitable or beneficial in nature. Shareholders are in no legal
sense the owners of corporate property, which is owned by the corporation as a distinct legal person.

NICOLAS, petitioner
vs.
COURT OF APPEALS, respondent
G.R. No. 122857 March 27, 1998

FACTS:

On February 19, 1987, petitioner Roy Nicolas and private respondent Blesito Buan entered into a
Portfolio Management Agreement, wherein the former was to manage the stock transactions of the latter

263 | P a g e
Law 321_Corporation LAW_ Case Digest

for a period of three months with an automatic renewal clause. However, upon the initiative of the
private respondent the agreement was terminated on August 19, 1987, and thereafter he requested for
an accounting of all transactions made by the petitioner.
Three weeks after the termination of the agreement, petitioner demanded from the private
respondent the amount of P68,263.67 representing his alleged management fees covering the periods of
June 30, July 31 and August 19, 1987 as provided for in the Portfolio Management Agreement. But the
demands went unheeded, much to the chagrin of the petitioner.
Rebuffed, petitioner filed a complaint for collection of sum of money against the private
respondent before the trial court. In his answer, private respondent contended that petitioner
mismanaged his transactions resulting in losses, thus, he was not entitled to any management fees.

ISSUE:

Whether or not petitioner is entitled to management fees.

RULING:

NO.

To begin with, petitioner has the burden to prove that the transaction realized gains or profits to
entitle him to said management fees, as provided in the Agreement. Accordingly, petitioner submitted
the profit and loss statements for the period of June 30, July 31 and August 19, 1987, showing a total
profit of P341,318.34, of which 20% would represent his management fees amounting to P68,263.70.
Unfortunately, the profit and loss statements presented by the petitioner are nothing but bare
assertions, devoid of any concrete basis or specifics as to the method of arriving at the amounts
indicated in the documents. In fact, it did not even state when the stocks were purchased, the type of
stocks (whether Class “A” or “B” or common or preferred) bought, when the stocks were sold, the
acquisition and selling price of each stock, when the profits, if any, were delivered to the private
respondent, the cost of safekeeping or custody of the stocks, as well as the taxes paid for each
transaction. With respect to the alleged losses, it has been held that where a profit or loss statement
shows a loss, the statement must show income and items of expense to explain the method of
determining such loss. However, in the instant petition, petitioner hardly elucidated the reasons and the
factors behind the losses incurred in the course of the transactions.
In short, no evidentiary value can be attributed to the profit and loss statements submitted by the
petitioner. These documents can hardly be considered a credible or true reflection of the
transactions. It is an incomplete record yielding easily to the inclusion or deletion of certain
matters. The contents are subject to suspicion since they are not reflective of all pertinent and relevant
data. Thus, even assuming the admissibility of these alleged profit and loss statements, they are devoid
of any evidentiary weight, for the amounts are conclusions without premises, its bases left to
speculation, conjectures, assertions and guesswork.

264 | P a g e
Law 321_Corporation LAW_ Case Digest

RAMOS, petitioner
vs.
COURT OF APPEALS, respondent
G.R. No. L-41295 December 4, 1989

FACTS:

On August 14 and 26, 1969, CMS Stock Brokerage, Inc. (or CMS) sold to Lopez, Locsin, Ledesma
& Co., Inc. (or LLL) on the floor of the Makati Stock Exchange (or MSE) 2,650 shares of Benguet
Consolidated Corporation for P297,650 on a delayed delivery basis of 10 to 20 days, evidenced by
Exchange Contracts Nos. B-11807 and B-11814 both dated August 14, 1969 and B-13084 dated August
26, 1969. LLL bought the shares for the account of its clients, the third-party defendants, Rene
Ledesma, Jose Maria Lopez, Cesar A. Lopez, Jr. and Alfredo Ramos. CMS failed to deliver the shares of
stocks within the agreed period, but LLL did not demand delivery.
On January 6, 1970, CMS informed LLL that it would deliver the shares the next day. LLL wrote
CMS that it would not accept the shares because its principals had cancelled their orders. In its reply,
CMS insisted that LLL take delivery of the Benguet shares.
In CMS's Clearing House Report of January 9, 1970, the disposition of the shares in favor of LLL
appeared, but the latter refused to acknowledge receipt of the covering disposal letter. CMS then
deposited the letter in the Office of the Exchange Executive, Secretary with the notation "Refused
acceptance pending decision of the Exchange".
When the controversy was submitted to the Board of Governors of the Exchange for
determination, the Board issued Resolution No. 523 on August 10, 1970 advising the parties to litigate
the matter in court.
Accordingly, CMS filed in the Court of First Instance of Rizal a complaint to compel LLL to accept
the Benguet shares, to pay the price of P297,650, as well as P25,000 as attorney's fees and costs. LLL's
motion to dismiss the complaint was denied.

ISSUE:

Whether or not appellate court erred in rendering a summary judgment, in failing to find that
CMS has no right to damages against the petitioner there being no privity of contract between them, and
in refusing to exonerate the petitioner from personal liability for the orders he placed with LLL for the
account of the Alakor Corporation.

RULING:

NO.

In the case at bar, the stock purchases of LLL were on a 10-20 day delayed delivery basis.
Accordingly, after that period lapsed, the Buying Member (LLL) should have advised the Selling Member
CMS in writing.
As observed by the trial court, Section 1, Article V of the Exchange Rules does not vest on the
buyer, respondent LLL, a right to rescind its contract with CMS upon the latter's default. The Exchange
Rules obligate the buyer to make a demand, and if the selling member fails to deliver the ordered shares
despite the demand, the buyer is further obligated to deliver a copy of his demand letter to the
Chairman of the Floor Trading and Arbitration Committee so that the latter may purchase the shares for
the selling member's account. Said rules were held binding on members of the Exchange. Inasmuch as
petitioner placed his order for Benguet shares through a member of the Exchange (LLL), he is indirectly
bound by the rules of the Exchange.
The defendants' lack of knowledge regarding the truth of the allegation in the complaint, that the
failure of CMS to deliver the Benguet shares on time was due to oversight, did not constitute an obstacle
to the rendition of a summary judgment by the trial court, for although an averment of lack of
knowledge has the effect of a denial, it does not raise a genuine issue.
SAW, petitioner
vs.
COURT OF APPEALS, respondent
G.R. No. 90580 April 8, 1991

FACTS:

A collection suit with preliminary attachment was filed by Equitable Banking Corporation against
Freeman, Inc. and Saw Chiao Lian, its President and General Manager. The petitioners moved to

265 | P a g e
Law 321_Corporation LAW_ Case Digest

intervene, alleging that (1) the loan transactions between Saw Chiao Lian and Equitable Banking Corp.
were not approved by the stockholders representing at least 2/3 of corporate capital; (2) Saw Chiao Lian
had no authority to contract such loans; and (3) there was collusion between the officials of Freeman,
Inc. and Equitable Banking Corp. in securing the loans. The motion to intervene was denied, and the
petitioners appealed to the Court of Appeals.
Meanwhile, Equitable and Saw Chiao Lian entered into a compromise agreement which they
submitted to and was approved by the lower court. But because it was not complied with, Equitable
secured a writ of execution, and two lots owned by Freeman, Inc. were levied upon and sold at public
auction to Freeman Management and Development Corp.
The Court of Appeals sustained the denial of the petitioners' motion for intervention, holding that
"the compromise agreement between Freeman, Inc., through its President, and Equitable Banking Corp.
will not necessarily prejudice petitioners whose rights to corporate assets are at most inchoate, prior to
the dissolution of Freeman, Inc. And intervention under Sec. 2, Rule 12 of the Revised Rules of Court is
proper only when one's right is actual, material, direct and immediate and not simply contingent or
expectant."
It also ruled against the petitioners' argument that because they had already filed a notice of
appeal, the trial judge had lost jurisdiction over the case and could no longer issue the writ of execution.

ISSUE:

Whether or not the Honorable Court of Appeals erred in holding that the petitioners cannot
intervene in Civil Case No. 88-44404 because their rights as stockholders of Freeman are merely
inchoate and not actual, material, direct and immediate prior to the dissolution of the corporation.

RULING:

NO.

The petitioners base their right to intervene for the protection of their interests as stockholders on
Everett v. Asia Banking Corp. where it was held: The well-known rule that shareholders cannot
ordinarily sue in equity to redress wrongs done to the corporation, but that the action must be brought
by the Board of Directors, has its exceptions.
Equitable demurs, contending that the collection suit against Freeman, Inc, and Saw Chiao Lian
is essentially in personam and, as an action against defendants in their personal capacities, will not
prejudice the petitioners as stockholders of the corporation. The Everett case is not applicable because it
involved an action filed by the minority stockholders where the board of directors refused to bring an
action in behalf of the corporation. In the case at bar, it was Freeman, Inc. that was being sued by the
creditor bank.
On the second assignment of error, Equitable maintains that the petitioners' appeal could only
apply to the denial of their motion for intervention and not to the main case because their personality as
party litigants had not been recognized by the trial court.
After examining the issues and arguments of the parties, the Court finds that the respondent
court committed no reversible error in sustaining the denial by the trial court of the petitioners' motion
for intervention.
 Consideration for Stocks

APODACA, petitioner
vs.
NATIONAL LABOR RELATIONS COMMISSION, respondent
G.R. No. 80039 April 18, 1989

FACTS:

Petitioner was employed in respondent corporation. On August 28, 1985, respondent Jose M.
Mirasol persuaded petitioner to subscribe to 1,500 shares of respondent corporation at P100.00 per
share or a total of P150,000.00. He made an initial payment of P37,500.00. On September 1, 1975,
petitioner was appointed President and General Manager of the respondent corporation. However, on
January 2, 1986, he resigned.
On December 19, 1986, petitioner instituted with the NLRC a complaint against private
respondents for the payment of his unpaid wages, his cost of living allowance, the balance of his
gasoline and representation expenses and his bonus compensation for 1986. Petitioner and private
respondents submitted their position papers to the labor arbiter. Private respondents admitted that
there is due to petitioner the amount of P17,060.07 but this was applied to the unpaid balance of his

266 | P a g e
Law 321_Corporation LAW_ Case Digest

subscription in the amount of P95,439.93. Petitioner questioned the set-off alleging that there was no
call or notice for the payment of the unpaid subscription and that, accordingly, the alleged obligation is
not enforceable.
In a decision dated April 28, 1987, the labor arbiter sustained the claim of petitioner for
P17,060.07 on the ground that the employer has no right to withhold payment of wages already earned
under Article 103 of the Labor Code. Upon the appeal of the private respondents to public respondent
NLRC, the decision of the labor arbiter was reversed in a decision dated September 18, 1987. The NLRC
held that a stockholder who fails to pay his unpaid subscription on call becomes a debtor of the
corporation and that the set-off of said obligation against the wages and others due to petitioner is not
contrary to law, morals and public policy.

ISSUE:

Whether or not an obligation arising from non-payment of stock subscriptions to a corporation


can be offset against a money claim of an employee against the employer.

RULING:

NO.

The unpaid subscriptions are not due and payable until a call is made by the corporation for
payment. Private respondents have not presented a resolution of the board of directors of respondent
corporation calling for the payment of the unpaid subscriptions. It does not even appear that a notice of
such call has been sent to petitioner by the respondent corporation.
What the records show is that the respondent corporation deducted the amount due to petitioner
from the amount receivable from him for the unpaid subscriptions. No doubt such set-off was without
lawful basis, if not premature. As there was no notice or call for the payment of unpaid subscriptions,
the same is not yet due and payable.
Lastly, assuming further that there was a call for payment of the unpaid subscription, the NLRC
cannot validly set it off against the wages and other benefits due petitioner. Article 113 of the Labor
Code allows such a deduction from the wages of the employees by the employer, only in three instances,
to wit: No employer, in his own behalf or in behalf of any person, shall make any deduction from the
wages of his employees, except in certain cases.

FUA CUN, petitioner


vs.
SUMMERS, respondent
G.R. No. L-19441 March 27, 1923

FACTS:

It appears from the evidence that on August 26, 1920, one Chua Soco subscribed for five
hundred shares of stock of the defendant Banking Corporation at a par value of P100 per share, paying
the sum of P25,000, one-half of the subscription price, in cash, for which a receipt was issued.
On May 18, 1921, Chua Soco executed a promissory note in favor of the plaintiff Fua Cun for the
sum of P25,000 payable in ninety days and drawing interest at the rate of 1 per cent per month,
securing the note with a chattel mortgage on the shares of stock subscribed for by Chua Soco, who also
endorsed the receipt above mentioned and delivered it to the mortgagee. The plaintiff thereupon took the
receipt to the manager of the defendant Bank and informed him of the transaction with Chua Soco, but
was told to await action upon the matter by the Board of Directors.
In the meantime Chua Soco appears to have become indebted to the China Banking Corporation
in the sum of P37,731.68 for dishonored acceptances of commercial paper and in an action brought
against him to recover this amount, Chua Soco's interest in the five hundred shares subscribed for was
attached and the receipt seized by the sheriff. The attachment was levied after the defendant bank had
received notice of the facts that the receipt had been endorsed over to the plaintiff. Fua Cun thereupon
brought the present action maintaining that by virtue of the payment of the one-half of the subscription
price of five hundred shares Chua Soco in effect became the owner of two hundred and fifty shares and
praying that his, the plaintiff's, lien on said shares, by virtue of the chattel mortgage, be declared to hold
priority over the claim of the defendant Banking Corporation; that the defendants be ordered to deliver
the receipt in question to him; and that he be awarded the sum of P5,000 in damages for wrongful
attachment.

267 | P a g e
Law 321_Corporation LAW_ Case Digest

ISSUE:

Whether or not the trial court erred in declaring that Chua Soco, through the payment of the
P25,000, acquired the right to two hundred and fifty shares fully paid up, upon which shares the
plaintiff holds a lien superior to that of the defendant Banking Corporation and ordering that the receipt
be returned to said plaintiff.

RULING:

YES.

The claim of the defendant Banking Corporation upon which it brought the action in which the
writ of attachment was issued, was for the non-payment of drafts accepted by Chua Soco and had no
direct connection with the shares of stock in question. At common law a corporation has no lien upon
the shares of stockholders for any indebtedness to the corporation and our attention has not been called
to any statute creating such lien here. On the contrary, section 120 of the Corporation Act provides that
"no bank organized under this Act shall make any loan or discount on the security of the shares of its
own capital stock, nor be the purchaser or holder of any such shares, unless such security or purchase
shall be necessary to prevent loss upon a debt previously contracted in good faith, and stock so
purchased or acquired shall, within six months from the time of its purchase, be sold or disposed of at
public or private sale, or, in default thereof, a receiver may be appointed to close up the business of the
bank in accordance with law."
The reasons for this doctrine are obvious; if banking corporations were given a lien on their own
stock for the indebtedness of the stockholders, the prohibition against granting loans or discounts upon
the security of the stock would become largely ineffective.

NATIONAL EXCHANGE CO., INC., petitioner


vs.
I.B. DEXTER, respondent
G.R. No. L-27872 February 25, 1928

FACTS:

This action was instituted in the Court of First Instance of Manila by the National Exchange Co.,
Inc., as assignee (through the Philippine National Bank) of C. S. Salmon & Co., for the purpose of
recovering from I. B. Dexter a balance of P15,000, the par value of one hundred fifty shares of the
capital stock of C. S. Salmon & co., with interest and costs. Upon hearing the cause the trial judge gave
judgment for the plaintiff to recover the amount claimed, with lawful interest from January 1, 1920, and
with costs. From this judgment the defendant appealed.
It appears that on August 10, 1919, the defendant, I. B. Dexter, signed a written subscription to
the corporate stock of C. S. Salmon & Co. in the following form: I hereby subscribe for three hundred
(300) shares of the capital stock of C. S. Salmon and Company, payable from the first dividends declared
on any and all shares of said company owned by me at the time dividends are declared, until the full
amount of this subscription has been paid.
Upon this subscription the sum of P15,000 was paid in January, 1920, from a dividend declared
at about that time by the company, supplemented by money supplied personally by the subscriber.
Beyond this nothing has been paid on the shares and no further dividend has been declared by the
corporation. There is therefore a balance of P15,000 still paid upon the subscription.

ISSUE:

Whether or not the stipulation contained in the subscription to the effect that the subscription is
payable from the first dividends declared on the shares has the effect of relieving the subscriber from
personal liability in an action to recover the value of the shares.

RULING:

NO.

In discussing this problem we accept as sound law the proposition propounded by the appellant's
attorneys and taken from Fletcher's Cyclopedia as follows: In the absence of restrictions in its character,
a corporation, under its general power to contract, has the power to accept subscriptions upon any special
terms not prohibited by positive law or contrary to public policy, provided they are not such as to require

268 | P a g e
Law 321_Corporation LAW_ Case Digest

the performance of acts which are beyond the powers conferred upon the corporation by its character, and
provided they do not constitute a fraud upon other subscribers or stockholders, or upon persons who are
or may become creditors of the corporation.
Pursuant to such, we find that the Philippine Commission inserted in the Corporation Law,
enacted March 1, 1906, the following provision: "no corporation shall issue stock or bonds except in
exchange for actual cash paid to the corporation or for property actually received by it at a fair valuation
equal to the par value of the stock or bonds so issued."
The prohibition against the issuance of shares by corporations except for actual cash to the par
value of the stock to its full equivalent in property is thus enshrined in both the organic and statutory
law of the Philippine; Islands; and it would seem that our lawmakers could scarely have chosen
language more directly suited to secure absolute equality stockholders with respect to their liability
upon stock subscriptions. Now, if it is unlawful to issue stock otherwise than as stated it is self-evident
that a stipulation such as that now under consideration, in a stock subcription, is illegal, for this
stipulation obligates the subcriber to pay nothing for the shares except as dividends may accrue upon
the stock. In the contingency that dividends are not paid, there is no liability at all. This is
discrimination in favor of the particular subcriber, and hence the stipulation is unlawful.
NIELSON & CO., INC., plaintiff
vs.
LEPANTO CONSOLIDATED MINING CO., defendant
G.R. No. L-21601 December 17, 1966

FACTS:

An operating agreement was executed before World War II (on 30 January 1937) between Nielson
& Co. Inc. and the Lepanto Consolidated Mining Co. whereby the former operated and managed the
mining properties owned by the latter for a management fee of P2,500.00 a month and a 10%
participation in the net profits resulting from the operation of the mining properties, for a period of 5
years. In 1940, a dispute arose regarding the computation of the 10% share of Nielson in the profits. The
Board of Directors of Lepanto, realizing that the mechanics of the contract was unfair to Nielson,
authorized its President to enter into an agreement with Nielson modifying the pertinent provision of the
contract effective 1 January 1940 in such a way that Nielson shall receive (1) 10% of the dividends
declared and paid, when and as paid, during the period of the contract and at the end of each year, (2)
10% of any depletion reserve that may be set up, and (3) 10% of any amount expended during the year
out of surplus earnings for capital account. In the latter part of 1941, the parties agreed to renew the
contract for another period of 5 years, but in the meantime, the Pacific War broke out in December
1941. In January 1942 operation of the mining properties was disrupted on account of the war. In
February 1942, the mill, power plant, supplies on hand, equipment, concentrates on hand and mines,
were destroyed upon orders of the United States Army, to prevent their utilization by the invading
Japanese Army. The Japanese forces thereafter occupied the mining properties, operated the mines
during the continuance of the war, and who were ousted from the mining properties only in August
1945. Shortly after the mines were liberated from the Japanese invaders in 1945, a disagreement arose
between NIELSON and LEPANTO over the status of the operating contract which as renewed expired in
1947.

ISSUE:

Whether or not the management contract is a contract of agency.

RULING:

NO.

In the performance of this principal undertaking Nielson was not in any way executing juridical
acts for Lepanto, destined to create, modify or extinguish business relations between Lepanto and third
persons. In other words, in performing its principal undertaking Nielson was not acting as an agent of
Lepanto, in the sense that the term agent is interpreted under the law of agency, but as one who was
performing material acts for an employer, for a compensation. It is true that the management contract
provides that Nielson would also act as purchasing agent of supplies and enter into contracts regarding
the sale of mineral, but the contract also provides that Nielson could not make any purchase, or sell the
minerals, without the prior approval of Lepanto. It is clear, therefore, that even in these cases Nielson
could not execute juridical acts which would bind Lepanto without first securing the approval of
Lepanto. Nielson, then, was to act only as an intermediary, not as an agent. Further, from the
statements in the annual report for 1936, and from the provision of paragraph XI of the Management

269 | P a g e
Law 321_Corporation LAW_ Case Digest

contract, that the employment by Lepanto of Nielson to operate and manage its mines was principally in
consideration of the know-how and technical services that Nielson offered Lepanto. The contract thus
entered into pursuant to the offer made by Nielson and accepted by Lepanto was a "detailed operating
contract". It was not a contract of agency. Nowhere in the record is it shown that Lepanto considered
Nielson as its agent and that Lepanto terminated the management contract because it had lost its trust
and confidence in Nielson.

TRILLANA, petitioner
vs.
QUEZON COLLEGE, INC., respondent
G.R. No. L-5003 June 27, 1953

FACTS:

Damasa Crisostomo sent a letter to the Board of Trustees of the Quezon College subscribing to
200 shares of its capital stock at par value of Php100 each. Damasa Crisostomo died on October 26,
1948. As no payment appears to have been made on the subscription mentioned in her letter, the
Quezon College, Inc. presented a claim before the Court of First Instance of Bulacan in her testate
proceeding, for the collection of the sum of P20,000, representing the value of the subscription to the
capital stock of the Quezon College, Inc. This claim was opposed by the administrator of the estate, and
the Court of First Instance of Bulacan, after hearing issued an order dismissing the claim of the Quezon
College, Inc. on the ground that the subscription in question was neither registered in nor authorized by
the Securities and Exchange Commission. From this order the Quezon College, Inc. has appealed.

ISSUE:

Whether or not the subscription applied for by Damasa Crisostomo is an enforceable contract.

RULING:

NO.

It appears that the application sent by Damasa Crisostomo to the Quezon College, Inc. was
written on a general form indicating that an applicant will enclose an amount as initial payment and will
pay the balance in accordance with law and the regulations of the College. On the other hand, in the
letter actually sent by Damasa Crisostomo, the latter (who requested that her subscription for 200
shares be entered) not only did not enclose any initial payment but stated that "babayaran kong lahat
pagkatapos na ako ay makapagpahuli ng isda." There is nothing in the record to show that the Quezon
College, Inc. accepted the term of payment suggested by Damasa Crisostomo, or that if there was any
acceptance the same came to her knowledge during her lifetime. As the application of Damasa
Crisostomo is obviously at variance with the terms evidenced in the form letter issued by the Quezon
College, Inc., there was absolute necessity on the part of the College to express its agreement to
Damasa's offer in order to bind the latter. Conversely, said acceptance was essential, because it would
be unfair to immediately obligate the Quezon College, Inc. under Damasa's promise to pay the price of
the subscription after she had caused fish to be caught. In other words, the relation between Damasa
Crisostomo and the Quezon College, Inc. had only thus reached the preliminary stage whereby the latter
offered its stock for subscription on the terms stated in the form letter, and Damasa applied for
subscription fixing her own plan of payment, — a relation, in the absence as in the present case of
acceptance by the Quezon College, Inc. of the counter offer of Damasa Crisostomo, that had not ripened
into an enforceable contract.
Indeed, the need for express acceptance on the part of the Quezon College, Inc. becomes the more
imperative, in view of the proposal of Damasa Crisostomo to pay the value of the subscription after she
has harvested fish, a condition obviously dependent upon her sole will and, therefore, facultative in
nature, rendering the obligation void, under article 1115 of the old Civil Code which provides as follows:
"If the fulfillment of the condition should depend upon the exclusive will of the debtor, the conditional
obligation shall be void. If it should depend upon chance, or upon the will of a third person, the
obligation shall produce all its effects in accordance with the provisions of this code." It cannot be
argued that the condition solely is void, because it would have served to create the obligation to pay,
wherein only the potestative condition was held void because it referred merely to the fulfillment of an
already existing indebtedness.

 Unpaid Subscriptions: Call: When necessary


270 | P a g e
Law 321_Corporation LAW_ Case Digest

GARCIA, plaintiff
vs.
SUAREZ, defendant
G.R. No. L-45493 April 21, 1939

FACTS:

On October 4, 1924, the appellant subscribed to sixteen shares of the capital stock of the
Compañia Hispano-Filipina, Inc., a corporation which is duly formed and organized. Of the sixteen
subscribed shares, at the par value of P100 each, the appellant only paid P400, the value of four shares.
On June 5, 1931, the plaintiff-appellee was appointed by the court receiver of the Compañia Hispano-
Filipina, Inc., to collect all the credits of said corporation, pay its debts and dispose of the remainder of
its assets and of its properties. On June 18, 1931, the plaintiff-appellee in vain made demand upon the
defendant-appellant to pay the balance of his subscription. On July 10, 1933, the plaintiff, as receiver,
brought an action in the Court of First Instance of Manila to recover from the defendant-appellant and
other shareholders the balance of their subscriptions, but the complaint was dismissed for lack of
prosecution. On October 10, 1935, a similar complaint was filed against the appellant, and after trial,
judgment was rendered therein ordering the said defendant to pay to the plaintiff, as receiver of
Compañia Hispano-Filipina, Inc., the sum of P1,200, with legal interest thereon from October 4, 1924,
and the costs. The defendant appealed and in this instance contends that the trial court erred in holding
that the action of the plaintiff-appellee has not prescribed, and that the appellant has not been released
from his obligation to pay the balance of his subscription.

ISSUE:

Whether or not the obligation contracted by the appellant to pay the value of his subscription was
demandable from the date of subscription in the absence of any stipulation to the contrary.

RULING:

NO.

Section 37 of the Corporation Law provides when the obligation to pay interest arises and when
payment should be made, but it is absolutely silent as to when the subscription to a stock should be
paid. Of course, the obligation to pay arises from the date of the subscription, but the coming into being
of an obligation should not be confused with the time when it becomes demandable. In a loan for
example, the obligation to pay arises from the time the loan is taken; but the maturity of that obligation,
the date when the debtor can be compelled to pay, is not the date itself of the loan, because this would
be absurd. The date when payment can be demanded is necessarily distinct from and subsequent to
that the obligation is contracted.
By the same token, the subscription to the capital stock of the corporation, unless otherwise
stipulation, is not payable at the moment of the subscription but on a subsequent date which may be
fixed by the corporation. Hence, section 38 of the Corporation Law, amended by Act No. 3518, provides
that: The board of directors or trustees of any stock corporation formed, organized, or existing under this
Act may at any time declare due and payable to the corporation unpaid subscriptions to the capital stock.
The board of directors of the Compañia Hispano-Filipino, Inc., not having declared due and
payable the stock subscribed by the appellant, the prescriptive period of the action for the collection
thereof only commenced to run from June 18, 1931 when the plaintiff, in his capacity as receiver and in
the exercise of the power conferred upon him by the said section 38 of the Corporation Law, demanded
of the appellant to pay the balance of his subscription. The present action having been filed on October
10, 1935, the defense of prescription is entirely without basis.

271 | P a g e
Law 321_Corporation LAW_ Case Digest

PHILIPPINE NATIONAL BANK, plaintiff


vs.
BITULOK SAWMILL INC., defendant
G.R. Nos. L-24177-85 June 29, 1968

FACTS:

The Philippine Lumber Distributing Agency, Inc., according to the lower court, "was organized
sometime in the early part of 1947 upon the initiative and insistence of the late President Manuel Roxas
of the Republic of the Philippines who for the purpose, had called several conferences between him and
the subscribers and organizers of the Philippine Lumber Distributing Agency, Inc." The purpose was
praiseworthy, to insure a steady supply of lumber, which could be sold at reasonable prices to enable
the war sufferers to rehabilitate their devastated homes. At the beginning, the lumber producers were
reluctant to organize the cooperative agency as they believed that it would not be easy to eliminate from
the retail trade the alien middlemen who had been in this business from time immemorial, but because
the late President Roxas made it clear that such a cooperative agency would not be successful without a
substantial working capital which the lumber producers could not entirely shoulder, and as an
inducement he promised and agreed to finance the agency by making the Government invest P9.00 by
way of counterpart for every peso that the members would invest therein."
Accordingly, "the late President Roxas instructed the Hon. Emilio Abello, then Executive Secretary
and Chairman of the Board of Directors of the Philippine National Bank, for the latter to grant said
agency an overdraft in the original sum of P250,000.00 which was later increased to P350,000.00,
which was approved by said Board of Directors of the Philippine National Bank on July 28, 1947,
payable on or before April 30, 1958, with interest at the rate of 6% per annum, and secured by the
chattel mortgages on the stock of lumber of said agency." The Philippine Government did not invest the
P9.00 for every peso coming from defendant lumber producers. The loan extended to the Philippine
Lumber Distributing Agency by the Philippine National Bank was not paid.

ISSUE:

Whether or not the non-compliance with a plain statutory command, considering the
persuasiveness of the plea that defendants-appellees would "not have subscribed to the capital stock" of
the Philippine Lumber Distributing Agency "were it not for the assurance of the then President of the
Republic that the Government would back it up by investing P9.00 for every peso" subscribed, a
condition which was not fulfilled, such commitment not having been complied with, be justified.

RULING:

NO.

It would be unwarranted to ascribe to the late President Roxas the view that the payment of the
stock subscriptions, as thus required by law, could be condoned in the event that the counterpart fund
to be invested by the Government would not be available. Even if such were the case, however, and such
a promise were in fact made, to further the laudable purpose to which the proposed corporation would
be devoted and the possibility that the lumber producers would lose money in the process, still the plain
and specific wording of the applicable legal provision as interpreted by this Court must be controlling. It
is a well-settled principle that with all the vast powers lodged in the Executive, he is still devoid of the
prerogative of suspending the operation of any statute or any of its terms.

VELASCO, plaintiff
vs.
POIZAT, defendant
G.R. No. L-11528 March 15, 1918

FACTS:

From the amended complaint filed in this cause upon February 5, 1915, it appears that the
plaintiff, as assignee in insolvency of "The Philippine Chemical Product Company" (Ltd.) is seeking to
recover of the defendant, Jean M. Poizat, the sum of P1,500, upon a subscription made by him to the

272 | P a g e
Law 321_Corporation LAW_ Case Digest

corporate stock of said company. It appears that the corporation in question was originally organized by
several residents of the city of Manila, where the company had its principal place of business, with a
capital of P50,000, divided into 500 shares. The defendant subscribed for 20 shares of the stock of the
company, an paid in upon his subscription the sum of P500, the par value of 5 shares . The action was
brought to recover the amount subscribed upon the remaining shares.
It appears that the defendant was a stock holder in the company from the inception of the
enterprise, and for sometime acted as its treasurer and manager. While serving in this capacity he called
in and collected all subscriptions to the capital stock of the company, except the aforesaid 15 shares
subscribed by himself and another 15 shares owned by Jose R. Infante.
Upon July 13, 1914, a meeting of the board of directors of the company was held at which a
majority of the stock was presented. Upon this occasion two resolutions, important to be here noted,
were adopted. The first was a proposal that the directors, or shareholders, of the company should make
good by new subscriptions, in proportion to their respective holdings, 15 shares which had been
surrendered by Infante.

ISSUE:

Whether or not Poizat is liable for his unpaid subscription.

RULING:

YES.

A stock subscription is a contract between the corporation on one side, and the subscriber on the
other, and courts will enforce it for or against either. It is a rule, accepted by the Supreme Court of the
United States that a subscription for shares of stock does not require an express promise to pay the
amount subscribed, as the law implies a promise to pay on the part of the subscriber. Section 36 of the
Corporation Law clearly recognizes that a stock subscription is subsisting liability from the time the
subscription is made, since it requires the subscriber to pay interest quarterly from that date unless he
is relieved from such liability by the by-laws of the corporation. The subscriber is as much bound to pay
the amount of the share subscribed by him as he would be to pay any other debt, and the right of the
company to demand payment is no less incontestable.
The provisions of the Corporation Law (Act No. 1459) given recognition of two remedies for the
enforcement of stock subscriptions. The first and most special remedy given by the statute consists in
permitting the corporation to put up the unpaid stock for sale and dispose of it for the account of the
delinquent subscriber. In this case the provisions of section 38 to 48, inclusive, of the Corporation Law
are applicable and must be followed. The other remedy is by action in court.
It is generally accepted doctrine that the statutory right to sell the subscriber's stock is merely a
remedy in addition to that which proceeds by action in court; and it has been held that the ordinary
legal remedy by action exists even though no express mention thereof is made in the statute.

 Court Action

LUMANLAN, plaintiff
vs.
CURA, et al., defendants
G.R. No. L-39861 March 21, 1934

FACTS:

The appellant is a corporation duly organized under the laws of the Philippine Islands with its
central office in the City of Manila. The plaintiff-appellee Bonifacio Lumanlan, on July 31, 1922,
subscribed for 300 shares of stock of said corporation at a par value of P50 or a total of P15,000. Julio
Valenzuela, Pedro Santos and Francisco Escoto, creditors of this corporation, filed suit against it in the
Court of First Instance of Manila, case No. 37007, praying that a receiver be appointed, as it appeared
that the corporation at that time had no assets except credits against those who had subscribed for
shares of stock. The court named Tayag as receiver for the purpose of collecting, said subscriptions. As
Bonifacio Lumanlan had only paid P1,500 of the P15,000, par value of the stock for which he
subscribed, the receiver on August 30, 1930, filed a suit against him in the Court of First Instance of
Manila, civil case No. 37492, for the collection of P15,109, P13,500 of which was the amount he owed
for unpaid stock and P1,609 for loans and advances by the corporation to Lumanlan. In that case

273 | P a g e
Law 321_Corporation LAW_ Case Digest

Lumanlan was sentenced to pay the corporation the above-mentioned sum of P15,109 with legal interest
thereon from August 30, 1930, and costs. Lumanlan appealed from this decision.

ISSUE:

Whether or not Bonifacio Lumanlan is entitled to a credit against the judgment in case No. 37492
for P11,840 and an additional sum of P2,000, which is 25 per cent on the principal debt, as he had to
file this suit to collect, or receive credit for the sum which he had paid Valenzuela for and in place of the
corporation, or a total of P13,840.

RULING:

YES.

It appears from the record that during the trial of the case now under consideration, the Bank of
the Philippine Islands appeared in this case as assignee in the "Involuntary Insolvency of Dizon & Co.,
Inc. That bank was appointed assignee in case No. 43065 of the Court of First Instance of the City of
Manila on November 28, 1932. It is therefore evident that there are still other creditors of Dizon & Co.,
Inc. This being the case that corporation has a right to collect all unpaid stock subscriptions and any
other amounts which may be due it.
It is established doctrine that subscriptions to the capital of a corporation constitute a fund to
which the creditors have a right to look for satisfaction of their claims and that the assignee in
insolvency can maintain an action upon any unpaid stock subscription in order to realize assets for the
payment of its debts. The Corporation Law clearly recognizes that a stock subscription is a subsisting
liability from the time the subscription is made, since it requires the subscriber to pay interest quarterly
from that date unless he is relieved from such liability by the by-laws of the corporation. The subscriber
is as much bound to pay the amount of the share subscribed by him as he would be to pay any other
debt, and the right of the company to demand payment is no less incontestable.

274 | P a g e
Law 321_Corporation LAW_ Case Digest

EDWARD KELLER & Co., Ltd., petitioner


vs.
COB GROUP MARKETING, respondent
G.R. No. L-68097 January 16, 1986

FACTS:

Edward A. Keller & Co., Ltd. appointed COB Group Marketing, Inc. as exclusive distributor of its
household products, Brite and Nuvan in Panay and Negros, as shown in the sales agreement dated
March 14, 1970 . Under that agreement Keller sold on credit its products to COB Group Marketing.
As security for COB Group Marketing's credit purchases up to the amount of P35,000, one
Asuncion Manahan mortgaged her land to Keller. Manahan assumed solidarily with COB Group
Marketing the faithful performance of all the terms and conditions of the sales agreement.
In July, 1970 the parties executed a second sales agreement whereby COB Group Marketing's
territory was extended to Northern and Southern Luzon. As security for the credit purchases up to
P25,000 of COB Group Marketing for that area, Tomas C. Lorenzo, Jr. and his father Tomas, Sr. (now
deceased) executed a mortgage on their land in Nueva Ecija. Like Manahan, the Lorenzos were solidarily
liable with COB Group Marketing for its obligations under the sales agreement.
The credit purchases of COB Group Marketing, which started on October 15, 1969, limited up to
January 22, 1971. On May 8, the board of directors of COB Group Marketing were apprised by Jose E.
Bax the firm's president and general manager, that the firm owed Keller about P179,000. Bax was
authorized to negotiate with Keller for the settlement of his firm's liability. On the same day, May 8, Bax
and R. Oefeli of Keller signed the conditions for the settlement of COB Group Marketing's liability.
Twelve days later, or on May 20, COB Group Marketing, through Bax executed two second chattel
mortgages over its 12 trucks (already mortgaged to Northern Motors, Inc.) as security for its obligation to
Keller amounting to P179,185.16 as of April 30, 1971.

ISSUE:

Whether or not the lower courts erred in nullifying the admissions of liability made in 1971 by
Bax as president and general manager of COB Group Marketing and in giving credence to the alleged
overpayment computed by Bax.

RULING:

YES.

The lower courts not only allowed Bax to nullify his admissions as to the liability of COB Group
Marketing but they also erroneously rendered judgment in its favor in the amount of its supposed
overpayment in the sum of P100,596.72, in spite of the fact that COB Group Marketing was declared in
default and did not file any counterclaim for the supposed overpayment. The lower courts harped on
Keller's alleged failure to thresh out with representatives of COB Group Marketing their "diverse
statements of credits and payments". This contention has no factual basis. That means that there was a
conference on the COB Group Marketing's liability. Bax in that discussion did not present his
reconciliation statements to show overpayment.
Bax admitted that Keller sent his company monthly statements of accounts but he could not
produce any formal protest against the supposed inaccuracy of the said statements. He lamely explained
that he would have to dig up his company's records for the formal protest. He did not make any written
demand for reconciliation of accounts.
As to the liability of the stockholders, it is settled that a stockholder is personally liable for the
financial obligations of a corporation to the extent of his unpaid subscription.

 Effect of Delinquency

VALLEY GOLF & COUNTRY CLUB, Inc., petitioner


vs.
VDA. DE CARAM, respondent
G.R. No. 158805 April 16, 2009

FACTS:

275 | P a g e
Law 321_Corporation LAW_ Case Digest

Valley Golf & Country Club (Valley Golf) is a duly constituted non-stock, non-profit corporation
which operates a golf course. The members and their guests are entitled to play golf on the said course
and otherwise avail of the facilities and privileges provided by Valley Golf. The shareholders are likewise
assessed monthly membership dues.
In 1961, the late Congressman Fermin Z. Caram, Jr. (Caram), the husband of the present
respondent, subscribed to purchased and paid for in full one share (Golf Share) in the capital stock of
Valley Golf. He was issued Stock Certificate No. 389 dated 26 January 1961 for the Golf Share. The
Stock Certificate likewise indicates a par value of P9,000.00.
Valley Golf would subsequently allege that beginning 25 January 1980, Caram stopped paying
his monthly dues, which were continually assessed until 31 June 1987. Valley Golf claims to have sent
five (5) letters to Caram concerning his delinquent account within the period from 27 January 1986
until 3 May 1987, all forwarded to P.O. Box No. 1566, Makati Commercial Center Post Office, the
mailing address which Caram allegedly furnished Valley Golf.
The Golf Share was sold at public auction on 11 June 1987 for P25,000.00 after the Board of
Directors had authorized the sale in a meeting on 11 April 1987, and the Notice of Auction Sale was
published in the 6 June 1987 edition of the Philippine Daily Inquirer.

ISSUE:

Whether or not a non-stock corporation seize and dispose of the membership share of a fully-paid
member on account of its unpaid debts to the corporation when it is authorized to do so under the
corporate by-laws but not by the Articles of Incorporation.

RULING:

NO.

It may be conceded that the actions of Valley Golf were, technically speaking, in accord with the
provisions of its by-laws on termination of membership, vaguely defined as these are. Yet especially
since the termination of membership in Valley Golf is inextricably linked to the deprivation of property
rights over the Golf Share, the emergence of such adverse consequences make legal and equitable
standards come to fore.
The commentaries of Lopez advert to an SEC Opinion dated 29 September 1987 which we can
cite with approval. Lopez cites: In order that the action of a corporation in expelling a member for cause
may be valid, it is essential, in the absence of a waiver, that there shall be a hearing or trial of the
charge against him, with reasonable notice to him and a fair opportunity to be heard in his defense. If
the method of trial is not regulated by the by-laws of the association, it should at least permit
substantial justice. The hearing must be conducted fairly and openly and the body of persons before
whom it is heard or who are to decide the case must be unprejudiced.
It is unmistakably wise public policy to require that the termination of membership in a non-
stock corporation be done in accordance with substantial justice. No matter how one may precisely
define such term, it is evident in this case that the termination of Caram’s membership betrayed the
dictates of substantial justice.

276 | P a g e
Law 321_Corporation LAW_ Case Digest

CALATAGAN GOLF CLUB, INC., petitioner


vs.
CLEMENTE Jr., respondent
G.R. No. 165443 April 16, 2009

FACTS:

Clemente applied to purchase one share of stock of Calatagan, indicating in his application for
membership his mailing address at "Phimco Industries, Inc. – P.O. Box 240, MCC," complete residential
address, office and residence telephone numbers, as well as the company (Phimco) with which he was
connected, Calatagan issued to him Certificate of Stock No. A-01295 on 2 May 1990 after
paying P120,000.00 for the share.
Calatagan charges monthly dues on its members to meet expenses for general operations, as well
as costs for upkeep and improvement of the grounds and facilities. The provision on monthly dues is
incorporated in Calatagan’s Articles of Incorporation and By-Laws. It is also reproduced at the back of
each certificate of stock.
Calatagan declared Clemente delinquent for having failed to pay his monthly dues for more than
sixty (60) days, specifically P5,600.00 as of 31 October 1992. Calatagan also included Clemente’s name
in the list of delinquent members posted on the club’s bulletin board. On 1 December 1992, Calatagan’s
board of directors adopted a resolution authorizing the foreclosure of shares of delinquent members,
including Clemente’s; and the public auction of these shares.

ISSUE:

Whether or not the action of Clemente had prescribed pursuant to Section 69 of the Corporation
Code, and that the requisite notices under both the law and the by-laws had been rendered to Clemente.

RULING:

YES.

There are fundamental differences that defy equivalence or even analogy between the sale of
delinquent stock under Section 68 and the sale that occurred in this case. At the root of the sale of
delinquent stock is the non-payment of the subscription price for the share of stock itself. The
stockholder or subscriber has yet to fully pay for the value of the share or shares subscribed. In this
case, Clemente had already fully paid for the share in Calatagan and no longer had any outstanding
obligation to deprive him of full title to his share. Perhaps the analogy could have been made if Clemente
had not yet fully paid for his share and the non-stock corporation, pursuant to an article or by-law
provision designed to address that situation, decided to sell such share as a consequence. But that is
not the case here, and there is no purpose for us to apply Section 69 to the case at bar.
It is plain that Calatagan had endeavored to install a clear and comprehensive procedure to
govern the payment of monthly dues, the declaration of a member as delinquent, and the constitution of
a lien on the shares and its eventual public sale to answer for the member’s debts. Under Section 91 of
the Corporation Code, membership in a non-stock corporation "shall be terminated in the manner and
for the causes provided in the articles of incorporation or the by-laws." The By-law provisions are
elaborate in explaining the manner and the causes for the termination of membership in Calatagan,
through the execution on the lien of the share. The Court is satisfied that the By-Laws, as written,
affords due protection to the member by assuring that the member should be notified by the Secretary
of the looming execution sale that would terminate membership in the club. In addition, the By-Laws
guarantees that after the execution sale, the proceeds of the sale would be returned to the former
member after deducting the outstanding obligations. If followed to the letter, the termination of
membership under this procedure outlined in the By-Laws would accord with substantial justice.

 Issuance of Certificates of Stock

FUA CUN, petitioner


vs.
SUMMERS, respondent
G.R. No. L-19441 March 27, 1923

FACTS:

277 | P a g e
Law 321_Corporation LAW_ Case Digest

It appears from the evidence that on August 26, 1920, one Chua Soco subscribed for five
hundred shares of stock of the defendant Banking Corporation.
On May 18, 1921, Chua Soco executed a promissory note in favor of the plaintiff Fua Cun for the
sum of P25,000 payable in ninety days and drawing interest at the rate of 1 per cent per month,
securing the note with a chattel mortgage on the shares of stock subscribed for by Chua Soco, who also
endorsed the receipt above mentioned and delivered it to the mortgagee. The plaintiff thereupon took the
receipt to the manager of the defendant Bank and informed him of the transaction with Chua Soco, but
was told to await action upon the matter by the Board of Directors.
In the meantime Chua Soco appears to have become indebted to the China Banking Corporation
in the sum of P37,731.68 for dishonored acceptances of commercial paper and in an action brought
against him to recover this amount, Chua Soco's interest in the five hundred shares subscribed for was
attached and the receipt seized by the sheriff. The attachment was levied after the defendant bank had
received notice of the facts that the receipt had been endorsed over to the plaintiff. Fua Cun thereupon
brought the present action maintaining that by virtue of the payment of the one-half of the subscription
price of five hundred shares Chua Soco in effect became the owner of two hundred and fifty shares and
praying that his, the plaintiff's, lien on said shares, by virtue of the chattel mortgage, be declared to hold
priority over the claim of the defendant Banking Corporation; that the defendants be ordered to deliver
the receipt in question to him; and that he be awarded the sum of P5,000 in damages for wrongful
attachment.

ISSUE:

Whether or not the interest held by Chua Soco was merely an equity which could not be made the
subject of a chattel mortgage.

RULING:

NO.

Though the courts have uniformly held that chattel mortgages on shares of stock and other
choses in action are valid as between the parties, there is still much to be said in favor of the
defendants' contention that the chattel mortgage here in question would not prevail over liens of third
parties without notice; an equity in shares of stock is of such an intangible character that it is
somewhat difficult to see how it can be treated as a chattel and mortgaged in such a manner that the
recording of the mortgage will furnish constructive notice to third parties.
In regard to a chattel mortgage of shares of stock: These certificates of stock are in the pockets of
the owner, and go with him where he may happen to locate, as choses in action, or evidence of his right,
without any means on the part of those with whom he proposes to deal on the faith of such a security of
ascertaining whether or not this stock is in pledge or mortgaged to others. The chief office of the company
may be at one place to-day and at another tomorrow. The owner may have no fixed or permanent abode,
and with his notes in one pocket and his certificates of stock in the other.
But a determination of this question is not essential in the present case. There can be no doubt
that an equity in shares of stock may be assigned and that the assignment is valid as between the
parties and as to persons to whom notice is brought home. Such an assignment exists here, though it
was made for the purpose of securing a debt.

BALTAZAR, plaintiff
vs.
LINGAYEN GULF ELECTRIC POWER CO., INC., respondent
G.R. No. L-16236 June 30, 1965

FACTS:

The Lingayen Gulf Electric Power Co., Inc., hereinafter referred to as Corporation, was doing
business in the Philippines, with principal offices at Lingayen, Pangasinan, and with an authorized
capital stock of P300.000.00 divided into 3,000 shares of voting stock at P100.00 par value, per share.
Plaintiffs Baltazar and Rose were among the incorporators, having subscribed to 600 and 400 shares of
the capital stock, or a total par value of P60,000.00 and P40.000.00, respectively. It is alleged that it has
always been the practice and procedure of the Corporation to issue certificates of stock to its individual
subscribers for unpaid shares of stock. Of the 600 shares of capital stock subscribed by Baltazar, he
had fully paid 535 shares of stock, and the Corporation issued to him several fully paid up and non-
assessable certificates of stock, corresponding to the 535 shares. After having made transfers to third

278 | P a g e
Law 321_Corporation LAW_ Case Digest

persons and acquired new ones, Baltazar had to his credit, on the filing of the complaint 341 shares
fully paid and non-assessable.
The respondents Ungson, Estrada, Fernandez and Yuson were small stockholders of the
Corporation, all holding a total number of fully paid-up shares of stock, of not more than 100 shares,
with a par value of P10,000.00 and the defendant Acena, was likewise an incorporator and stockholder,
holding 600 shares of stock, for which certificate of stock were issued to him and as such, was the
largest individual stockholder thereof. Defendants Ungson, Estrada, Fernandez and Yuzon, constituted
the majority of the holdover seven-member Board of Directors of the Corporation, in 1955, two (2) of said
defendants having been elected as members of the Board in the annual stockholders' meeting held in
May 1954, largely on the vote of their co-defendant Acena, while the other two (2) were elected mainly on
the vote of the plaintiffs and their group of stockholders. Let the first group be called theUngson
group and the second, the Baltazar group.

ISSUE:

Whether or not a stockholder, in a stock corporation, subscribes to a certain number of shares of


stock, and he pays only partially, for which he is issued certificates of stock, is he entitled to vote the
latter, notwithstanding the fact that he has not paid the balance of his subscription, which has been
called for payment or declared delinquent.

RULING:

YES.

The cases at bar do not come under the aegis of the principle enunciated in the Fua Cun v.
Summers case, because it was the practice and procedure, since the inception of the corporation, to
issue certificates of stock to its individual subscribers for unpaid shares of stock and gave voting power
to shares of stock fully paid. And even though no agreement existed, the ruling in said case does not
now reflect the correct view on the matter, for better than an agreement or practice, there is the law,
which renders the said case of Fua Cun-Summers, obsolescent.
In the cases at bar, the defendant-corporation had chosen to apply payments by its stockholders
to definite shares of the capital stock of the corporation and had fully paid capital stock shares
certificates for said payments; its call for payment of unpaid subscription and its declaration of
delinquency for non-payment of said call affecting only the remaining number of shares of its capital
stock for which no fully paid capital stock shares certificates have been issued, "and only these have
been legally shorn of their voting rights by said declaration of delinquency" (amended decision).

279 | P a g e
Law 321_Corporation LAW_ Case Digest

TAN, petitioner
vs.
SECURITIES AND EXCHANGE COMMISSION, respondent
G.R. No. 95696 March 3, 1992

FACTS:

Respondent corporation was registered on October 1, 1979. As incorporator, petitioner had four
hundred (400) shares of the capital stock standing in his name at the par value of P100.00 per share,
evidenced by Certificate of Stock No. 2. He was elected as President and subsequently reelected, holding
the position as such until 1982 but remained in the Board of Directors until April 19, 1983 as director.
On January 31, 1981, while petitioner was still the president of the respondent corporation, two
other incorporators, namely, Antonia Y. Young and Teresita Y. Ong, assigned to the corporation their
shares, represented by certificate of stock No. 4 and 5 after which, they were paid the corresponding
40% corporate stock-in-trade.
Petitioner's certificate of stock No. 2 was cancelled by the corporate secretary and respondent
Patricia Aguilar by virtue of Resolution No. 1981 (b), which was passed and approved while petitioner
was still a member of the Board of Directors of the respondent corporation.
Due to the withdrawal of the aforesaid incorporators and in order to complete the membership of
the five (5) directors of the board, petitioner sold fifty (50) shares out of his 400 shares of capital stock to
his brother Angel S. Tan. Another incorporator, Alfredo B. Uy, also sold fifty (50) of his 400 shares of
capital stock to Teodora S. Tan and both new stockholders attended the special meeting, Angel Tan was
elected director and on March 27, 1981, the minutes of said meeting was filed with the SEC. These facts
stand unchallenged.

ISSUE:

Whether or not the cancellation and transfer of petitioner's shares and Certificate of Stock No. 2
as well as the issuance and cancellation of Certificate of Stock No. 8 was patently and palpably
unlawful, null and void, invalid and fraudulent.

RULING:

YES.

Under the instant case, the fact of the matter is, the new holder, Angel S. Tan has already
exercised his rights and prerogatives as stockholder and was even elected as member of the board of
directors in the respondent corporation with the full knowledge and acquiescence of petitioner. Due to
the transfer of fifty (50) shares, Angel S. Tan was clothed with rights and responsibilities in the board of
the respondent corporation when he was elected as officer thereof.
Besides, in Philippine jurisprudence, a certificate of stock is not a negotiable instrument.
"Although it is sometime regarded as quasi-negotiable, in the sense that it may be transferred by
endorsement, coupled with delivery, it is well-settled that it is non-negotiable, because the holder
thereof takes it without prejudice to such rights or defenses as the registered owner/s or transferor's
creditor may have under the law, except insofar as such rights or defenses are subject to the limitations
imposed by the principles governing estoppel."
To follow the argument put up by petitioner which was upheld by the Cebu SEC Extension Office
Hearing Officer, Felix Chan, that the cancellation of Stock Certificate Nos. 2 and 8 was null and void for
lack of delivery of the cancelled "mother" Certificate No. 2 whose endorsement was deliberately withheld
by petitioner, is to prescribe certain restrictions on the transfer of stock in violation of the corporation
law itself as the only law governing transfer of stocks. While Section 47(s) grants stock corporations the
authority to determine in the by-laws "the manner of issuing certificates" of shares of stock, however,
the power to regulate is not the power to prohibit, or to impose unreasonable restrictions of the right of
stockholders to transfer their shares.
EMBASSY FARMS, INC., petitioner
vs.
COURT OF APPEALS, respondent
G.R. No. 80682 August 13, 1990

FACTS:

It appears on record that sometime on August 2, 1984, Alexander G. Asuncion (AGA for short)
and Eduardo B. Evangelists (EBE for short) entered into a Memorandum of Agreement (Annex "A" of the

280 | P a g e
Law 321_Corporation LAW_ Case Digest

petition). Under said agreement EBE obligated himself to transfer to AGA 19 parcels of agricultural land
registered in his name with an aggregate area of 104,447 square meters located in Loma de Gato,
Marilao, Bulacan, together with the stocks, equipment and facilities of a piggery farm owned by
Embassy Farms, Inc., a registered corporation wherein ninety (90) per cent of its shares of stock is
owned by EBE. EBE also obligated himself to cede, transfer and convey "in a manner absolute and
irrevocable any and all of his shares of stocks" in Embassy Farins Inc. to AGA or his nominees "until the
total of said shares of stock so transferred shall constitute 90% of the paid-in-equity of said corporation"
within a reasonable time from signing of the document. Likewise, EBE obligated to turnover to AGA the
effective control and management of the piggery upon the signing of the agreement.
On the other hand, AGA obligated himself, upon signing of the agreement to pay to EBE the total
sum of close to P8,630,000.00. Within reasonable time from signing of the agreement AGA obligated
himself to organize and register a new corporation with an authorized capital stock of P10,000,000.00
which upon registration will take over all the rights and liabilities of AGA.

ISSUE:

Whether or not there has been an effective transfer of shares of stock from AGA to other persons.

RULING:

NO.

There being no delivery of the indorsed shares of stock AGA cannot therefore effectively transfer
to other person or his nominees the undelivered shares of stock. For an effective transfer of shares of
stock the mode and manner of transfer as prescribed by law must be followed (Navea v. Peers Marketing
Corp., 74 SCRA 65). As provided under Section 3 of Batas Pambansa Bilang 68, otherwise known as the
Corporation Code of the Philippines, shares of stock may be transferred by delivery to the transferree of
the certificate properly indorsed. Title may be vested in the transferree by the delivery of the duly
indorsed certificate of stock (18 C.J.S. 928, cited in Rivera v. Florendo, 144 SCRA 643). However, no
transfer shall be valid, except as between the parties until the transfer is properly recorded in the books
of the corporation.
In the case at bar the indorsed certificate of stock was not actually delivered to AGA so that EBE
is still the controlling stockholder of Embassy Farms despite the execution of the memorandum of
agreement and the turn over of control and management of the Embassy Farms to AGA on August 2,
1984.
When AGA filed on April 10, 1986 an action for the rescission of contracts with damages the
Pasig Court merely restored and established the status quo prior to the execution of the memorandum
of agreement by the issuance of a restraining order on July 10, 1987 and the writ of preliminary
injunction on July 30, 1987. It would be unjust and unfair to allow AGA and his nominees to control
and manage the Embassy Farms despite the fact that AGA who is the source of their supposed shares of
stock in the corporation is not asking for the delivery of the indorsed certificate of stock but for the
rescission of the memorandum of agreement. Rescission would result in mutual restitution (Magdalena
Estate v. Myrick, 71 Phil. 344) so it is but proper to allow EBE to manage the farm.
 Right to Transfer of Shares/Validity of Restrictions on Right

Makati Sports Club Inc


vs.
Cecile Cheng
G.R. No. 178523 June 16, 2010

FACTS:

October 20, 1994: Makati Sports Club Inc (MSCI) BOD adopted a resolution authorizing the sale
of 19 unissued shares at a floor price of P400,000 and P450,000 per share for Class A and B,
respectively. Cheng was a Treasurer and Director of Makati Sports Club in 1995
On July 7, 1995, Hodreal expressed his interest to buy a share, for this purpose he sent the letter
requesting to be wait listed. On November 1995, McFoods acquiried shares of Makati Sports Club
at P1,800,000 through Urban Bank. Thereafter, Cheng advised sale by McFoods to Hodreal of the share
evidenced by a certificate
new certificate was issued. Investigation showed that Cheng profited from the transaction because of her
knowledge MSCI sought judgment that would order respondents to pay the sum of P1,000,000.00,
representing the amount allegedly defrauded, together with interest and damages.

281 | P a g e
Law 321_Corporation LAW_ Case Digest

ISSUE:

Whether or not MSCI was defrauded by Cheng's collaboration with Mc Foods.

RULING:

NO.

No evidence on record that the Membership Committee acted on Hodreal's letter. SEC. 29. (a) The
Membership Committee shall process applications for membership; ascertain that the requirements for
stock ownership, including citizenship, are complied with; submit to the Board its recommended on
applicants for inclusion in the Waiting List; take charge of auction sales of shares of stock; and exercise
such other powers and perform such other functions as may be authorized by the Board. Membership
Committee failed to question the alleged irregularities attending Mc Foods’ purchase price
of P1,800,000.00 is P1,400,000.00 more than the floor price – it is not detrimental.
Upon payment and the execution of the Deed of Absolute Sale, it had the right to demand the
delivery of the stock certificate in its name. The right of a transferee to have stocks transferred to its
name is an inherent right flowing from its ownership of the stocks certificate of stock paper
representative or tangible evidence of the stock itself and of the various interests therein not a stock in
the corporation but is merely evidence of the holder’s interest and status in the corporation, his
ownership of the share represented thereby MSCI failed to repurchase Mc Foods’ Class "A" share within
the 30 day pre-emptive period and no proof that Cheng personally profited.

FLEISCHER, plaintiff
vs.
BOTICA NOLASCO CO., INC., defendant
G.R. No. L-23241 March 14, 1925

FACTS:

Manuel Gonzalez was the original owner of the five shares of stock in question, Nos. 16, 17, 18,
19 and 20 of the Botica Nolasco, Inc.; that on March 11, 1923, he assigned and delivered said five
shares to the plaintiff, Henry Fleischer, by accomplishing the form of endorsement provided on the back
thereof, together with other credits, in consideration of a large sum of money owed by Gonzalez to
Fleischer; that on March 13, 1923, Dr. Eduardo Miciano, who was the secretary-treasurer of said
corporation, offered to buy from Henry Fleischer, on behalf of the corporation, said shares of stock, at
their par value of P100 a share, for P500; that by virtue of article 12 of the by-laws of Botica Nolasco,
Inc., said corporation had the preferential right to buy from Manuel Gonzalez said shares; that the
plaintiff refused to sell them to the defendant; that the plaintiff requested Doctor Miciano to register said
shares in his name; that Doctor Miciano refused to do so, saying that it would be in contravention of the
by-laws of the corporation.
It also appears from the record that on the 13th day of March, 1923, two days after the
assignment of the shares to the plaintiff, Manuel Gonzales made a written statement to the Botica
Nolasco, Inc., requesting that the five shares of stock sold by him to Henry Fleischer be noted
transferred to Fleischer's name. He also acknowledged in said written statement the preferential right of
the corporation to buy said five shares.

ISSUE:

282 | P a g e
Law 321_Corporation LAW_ Case Digest

Whether or not article 12 of the by-laws of the Botica Nolasco, Inc., is in conflict with the
provisions of the Corporation Law (Act No. 1459).

RULING:

NO.

It does not suggest that any discrimination may be created by the corporation in favor or against
a certain purchaser. The holder of shares, as owner of personal property, is at liberty, under said
section, to dispose of them in favor of whomsoever he pleases, without any other limitation in this
respect, than the general provisions of law. Therefore, a stock corporation in adopting a by-law
governing transfer of shares of stock should take into consideration the specific provisions of section 35
of Act No. 1459, and said by-law should be made to harmonize with said provisions. It should not be
inconsistent therewith.
The only restraint imposed by the Corporation Law upon transfer of shares is found in section 35
of Act No. 1459, quoted above, as follows: "No transfer, however, shall be valid, except as between the
parties, until the transfer is entered and noted upon the books of the corporation so as to show the
names of the parties to the transaction, the date of the transfer, the number of the certificate, and the
number of shares transferred." This restriction is necessary in order that the officers of the corporation
may know who are the stockholders, which is essential in conducting elections of officers, in calling
meeting of stockholders, and for other purposes. but any restriction of the nature of that imposed in the
by-law now in question, is ultra vires, violative of the property rights of shareholders, and in restraint of
trade.
And moreover, the by-laws now in question cannot have any effect on the appellee. He had no
knowledge of such by-law when the shares were assigned to him. He obtained them in good faith and for
a valuable consideration. He was not a privy to the contract created by said by-law between the
shareholder Manuel Gonzalez and the Botica Nolasco, Inc. Said by-law cannot operate to defeat his
rights as a purchaser.

CYRUS PADGETT
vs.
BABCOCK & TEMPLETON, INC., and W. R. BABCOCK
G.R. No. L-38684, December 21, 1933

FACTS:

The appellee was an employee of the Appellant Corporation and rendered services as such from
January 1, 1923, to April 15, 1929. During that period he bought 35 shares thereof at P100 a share at
the suggestion of the president of said corporation. He was also the recipient of 9 shares by way of
bonus during Christmas seasons. In this way the said appellee became the owner of 44 shares for which
the 12 certificates, Exhibits F to F-11, were issued in his favor. The word "nontransferable" appears on
each and every one of these certificates. Before severing his connections with the said corporation, the
appellee proposed to the president that the said corporation buy his 44 shares at par value plus the
interest thereon, or that he be authorized to sell them to other persons. The corporation bought similar
shares belonging to other employees, at par value. Sometime later, the said president offered to buy the
appellee's shares first at P85 each and then at P80. The appellee did not agree thereto.

ISSUE:

Whether or not the defendant obliged to buy his shares of stock at par value.

RULING:

NO.

A restriction imposed upon a certificate of shares, similar to the ones under consideration, is null
and void on the ground that it constitutes and unreasonable limitation of the right of ownership and is
in restraint of trade.
Shares of corporate stock being regarded as property, the owner of such shares may, as a general
rule, dispose of them as he sees fit, unless the corporation has been dissolved, or unless the right to do
so is properly restricted, or the owner's privilege of disposing of his shares has been hampered by his
own action.

283 | P a g e
Law 321_Corporation LAW_ Case Digest

Any restriction on a stockholder's right to dispose of his shares must be construed strictly; and
any attempt to restrain a transfer of shares is regarded as being in restraint of trade, in the absence of a
valid lien upon its shares, and except to the extent that valid restrictive regulations and agreements
exist and are applicable. Subject only to such restrictions, a stockholder cannot be controlled in or
restrained from exercising his right to transfer by the corporation or its officers or by other stockholders,
even though the sale is to a competitor of the company, or to an insolvent person, or even though a
controlling interest is sold to one purchaser.

284 | P a g e
Law 321_Corporation LAW_ Case Digest

RURAL BANK OF SALINAS, INC., MANUEL SALUD, LUZVIMINDA TRIAS and FRANCISCO TRIAS
vs.
COURT OF APPEALS, SECURITIES AND EXCHANGE COMMISSION, MELANIA A. GUERRERO, LUZ
ANDICO, WILHEMINA G. ROSALES, FRANCISCO M. GUERRERO, JR., and FRANCISCO GUERRERO
, SR.
G.R. No. 96674, June 26, 1992

FACTS:

Clemente G. Guerrero, President of the Rural Bank of Salinas, Inc., executed a Special Power of
Attorney in favor of his wife, private respondent Melania Guerrero, giving and granting the latter full
power and authority to sell or otherwise dispose of and/or mortgage 473 shares of stock of the Bank
registered in his name (represented by the Bank's stock certificates nos. 26, 49 and 65), to execute the
proper documents therefor, and to receive and sign receipts for the dispositions. On February 27, 1980,
and pursuant to said Special Power of Attorney, private respondent Melania Guerrero, as Attorney-in-
Fact, executed a Deed of Assignment for 472 shares out of the 473 shares, in favor of private
respondents Luz Andico (457 shares), Wilhelmina Rosales (10 shares) and Francisco Guerrero, Jr. (5
shares).Almost four months later, or two (2) days before the death of Clemente Guerrero on June 24,
1980, private respondent Melania Guerrero, pursuant to the same Special Power of Attorney, executed
a Deed of Assignmentfor the remaining one (1) share of stock in favor of private respondent Francisco
Guerrero, Sr.
Subsequently, private respondent Melania Guerrero presented to petitioner Rural Bank of Salinas
the two (2) Deeds of Assignment for registration with a request for the transfer in the Bank's stock and
transfer book of the 473 shares of stock so assigned, the cancellation of stock certificates in the name of
Clemente G. Guerrero, and the issuance of new stock certificates covering the transferred shares of
stocks in the name of the new owners thereof. However, petitioner Bank denied the request of
respondent Melania Guerrero.

ISSUE:

Whether or not a Mandamus lie against the Rural Bank of Salinas to register in its stock and
transfer book the transfer of 473 shares of stock to private respondents.

RULING:

YES.

Section 5 (b) of P.D. No. 902-A grants to the SEC the original and exclusive jurisdiction to hear
and decide cases involving intracorporate controversies. An intra-corporate controversy has been defined
as one which arises between a stockholder and the corporation. There is neither distinction,
qualification, nor any exception whatsoever. The case at bar involves shares of stock, their registration,
cancellation and issuances thereof by petitioner Rural Bank of Salinas. It is therefore within the power
of respondent SEC to adjudicate.
A corporation, either by its board, its by-laws, or the act of its officers, cannot create restrictions
in stock transfers, because: Restrictions in the traffic of stock must have their source in legislative
enactment, as the corporation itself cannot create such impediment. By-laws are intended merely for the
protection of the corporation, and prescribe regulation, not restriction; they are always subject to the
charter of the corporation. The corporation, in the absence of such power, cannot ordinarily inquire into
or pass upon the legality of the transactions by which its stock passes from one person to another, nor
can it question the consideration upon which a sale is based.
Whenever a corporation refuses to transfer and register stock in cases like the
present, mandamus will lie to compel the officers of the corporation to transfer said stock in the books of
the corporation.

285 | P a g e
Law 321_Corporation LAW_ Case Digest

MARSH THOMSON
vs.
COURT OF APPEALS and THE AMERICAN CHAMPER OF COMMERCE OF THE PHILIPPINES, INC.
G.R. No. 116631, October 28, 1998

FACTS:

A. Lewis Burridge, retired as AmCham's President while petitioner was still working with private
respondent, his superior,. Before Burridge decided to return to his home country, he wanted to transfer
his proprietary share in the Manila Polo Club (MPC) to petitioner. However, through the intercession of
Burridge, private respondent paid for the share but had it listed in petitioner's name. This was made
clear in an employment advice dated January 13, 1986, wherein petitioner was informed by private
respondent.
Burridge transferred said proprietary share to petitioner, as confirmed in a letter of notification to
the Manila Polo Club. Upon his admission as a new member of the MPC, petitioner paid the transfer fee
of P40,000.00 from his own funds; but private respondent subsequently reimbursed this amount.
MPC issued Proprietary Membership Certificate Number 3398 in favor of petitioner. But
petitioner, however, failed to execute a document recognizing private respondent's beneficial ownership
over said share.
When petitioner's contract of employment was up for renewal in 1989, he notified private
respondent that he would no longer be available as Executive Vice President after September 30, 1989.
Still, the private respondent asked the petitioner to stay on for another six (6) months.

ISSUE:

Whether or not private respondent the beneficial owner of the disputed share.

RULING:

YES.

In the present case, as the Executive Vice-President of AMCHAM, petitioner occupied a fiduciary
position in the business of AMCHAM. It released the funds to acquire a share in the Club for the use of
petitioner but obliged him to "execute such document as necessary to acknowledge beneficial ownership
thereof by the Chamber". A trust relationship is, therefore, manifestly indicated.
The beneficiary of a trust has beneficial interest in the trust property, while a creditor has merely
a personal claim against the debtor. In trust, there is a fiduciary relation between a trustee and a
beneficiary, but there is no such relation between a debtor and creditor. While a debt implies merely an
obligation to pay a certain sum of money, a trust refers to a duty to deal with a specific property for the
benefit of another. If a creditor-debtor relationship exists, but not a fiduciary relationship between the
parties, there is no express trust. However, it is understood that when the purported trustee of funds is
entitled to use them as his or her own (and commingle them with his or her own money), a debtor-
creditor relationship exists, not a trust.
Moreover, petitioner failed to present evidence to support his allegation of being merely a debtor
when the private respondent paid the purchase price of the MPC share. Applicable here is the rule that a
trust arises in favor of one who pays the purchase money of property in the name of another, because of
the presumption that he who pays for a thing intends a beneficial interest therein for himself.

ENRIQUE T. YUCHENGCO, INC., A. T. YUCHENGCO, INC., ANNABELLE Y. PUEY and MONA LISA Y.
ABAYA
vs.
CONRADO M. VELAYO
G.R. No. L-50439, July 20, 1982

FACTS:

286 | P a g e
Law 321_Corporation LAW_ Case Digest

Conrado M. Velayo offered to sell to the plaintiffs-appellees 2,265 shares of common stock of the
RIC Tours Philippines, Inc. ("Ric Tours Phil., for short) a Philippine Corporation then duly licensed as a
tourist operator, constituting 70% of the subscribed and outstanding capital stock of the said
corporation. Appellees paid the entire purchase price of P367,500.00 to appellant Velayo, and the latter,
on his part, delivered to the former all the 2,265 shares of stock of Ric Tours Phil.
Appellant claims that the shares of stock of Ric Tours Phil. were sold to another group without
previous clearance from the Department of Tourism because he really was not aware of the rule
requiring prior approval by the Department of Tourism for the validity of transfers of shares of local tour
operators.
On September 3, 1974, appellees wrote a letter to appellant demanding rescission of the contract,
the restitution of a sum of money.

ISSUE:

Whether or not the "Stock Purchase Agreement" entered into by the appellees and appellant
Velayo annulled, or in the alternative, declared void ab initio.

RULING:

NO.

The provision governing the Agreement sought to be annuled is Sec 4, Part IV of the Rules and
Regulations Governing the Business of Tour Operators and Tour Guides, which recites as follows: Sec.
4. No transfer of rights to a license of a tour operator or ownership of shares or interests in the agency
shall be valid unless made with the prior approval of the Department.
The above-quoted rule is clear and mandatory. It requires the prior approval of the Department of
Tourism for the validity of any transfer of rights to a license of a tour operator or ownership of shares or
interests in any tour agency. In the case at bar, it was admitted by both parties, that the Stock Purchase
Agreement was made without the prior approval of the Department of Tourism. Pursuant to paragraph
7, article 1409 of the New Civil Code, such agreement would be inexistent and null and void from the
beginning. For it is well-settled that any contract entered into must be in accordance with, and not
repugnant to, an applicable statute whose terms are deemed embodied therein and without the need for
the parties of expressly making reference to it. Inasmuch as the agreement between the parties is null
and void from the beginning, it produces no legal effect. No valid transfer of ownership of Ric Tours Phil.,
to the appellees, therefore, took place upon delivery to them by the appellant of the shares of stock of
said corporation as to make them suffer the consequence of the subsequent revocation by the
Department of Tourism of the license of Ric Tours Phil., as they would indeed suffer much loss after
parting with their money for which they would receive nothing. The doctrine of res suo domino
perit advanced by the defendant cannot, therefore, be applied.

287 | P a g e
Law 321_Corporation LAW_ Case Digest

LIM TAY
vs.
COURT OF APPEALS, GO FAY AND CO. INC., SY GUIOK, and THE ESTATE OF ALFONSO LIM
G.R. No. 126891, August 5, 1998

FACTS:

On January 8, 1980, Respondent-Appellee Sy Guiok secured a loan from the petitioner in the
amount of P40,000 payable within six (6) months. To secure the payment of the aforesaid loan and
interest thereon, Respondent Guiok executed a Contract of Pledge in favor of the [p]etitioner whereby he
pledged his three hundred (300) shares of stock in the Go Fay & Company Inc., Respondent
Corporation, for brevity's sake. Respondent Guiok obliged himself to pay interest on said loan at the rate
of 10% per annum from the date of said contract of pledge. On the same date, Alfonso Sy Lim secured a
loan from the [p]etitioner in the amount of P40,000 payable in six (6) months. To secure the payment of
his loan, Sy Lim executed a "Contract of Pledge" covering his three hundred (300) shares of stock in
Respondent Corporation. Under said contract, Sy Lim obliged himself to pay interest on his loan at the
rate of 10% per annum from the date of the execution of said contract.
However, Respondent Guiok and Sy Lim failed to pay their respective loans and the accrued
interests thereon to the [p]etitioner. In October, 1990, the petitioner filed a "Petition for Mandamus"
against Respondent Corporation, with the SEC entitled "Lim Tay versus Go Fay & Company. Inc., SEC
Case No. 03894".

ISSUE:

Whether or not there is there dacion en pago.

RULING:

NO.

At the outset, it must be underscored that petitioner did not acquire ownership of the shares by
virtue of the contracts of pledge. Article 2112 of the Civil Code states: The creditor to whom the credit
has not been satisfied in due time, may proceed before a Notary Public to the sale of the thing pledged.
This sale shall be made at a public auction and with notification to the debtor and the owner of the
thing pledged in a proper case, stating the amount for which the public sale is to be held. If at the first
auction the thing is not sold, a second one with the same formalities shall be held; and if at the second
auction there is no sale either, the creditor may appropriate the thing pledged. In this case he shall be
obliged to give an acquaintance for his entire claim.
There is no showing that petitioner made any attempt to foreclose or sell the shares through
public or private auction, as stipulated in the contracts of pledge and as required by Article 2112 of the
Civil Code. Therefore, ownership of the shares could not have passed to him. The pledgor remains the
owner during the pendency of the pledge and prior to foreclosure and sale, as explicitly provided by
Article 2103 of the same Code: Unless the thing pledged is expropriated, the debtor continues to be the
owner thereof.
Neither did petitioner acquire the shares by virtue of a novation of the contract of pledge.
Novation is defined as "the extinguishment of an obligation by a subsequent one which terminates it,
either by changing its object or principal conditions, by substituting a new debtor in place of the old
one, or by subrogating a third person to the rights of the creditor." Novation of a contract must not be
presumed. "In the absence of an express agreement, novation takes place only when the old and the new
obligations are incompatible on every point.

 Transfer of Shares of Stock and Registration

JOSELITO MUSNI PUNO (as heir of the late Carlos Puno)


vs.
PUNO ENTERPRISES, INC., represented by JESUSA PUNO
G.R. No. 177066 September 11, 2009

FACTS:

288 | P a g e
Law 321_Corporation LAW_ Case Digest

Carlos L. Puno, who died on June 25, 1963, was an incorporator of respondent Puno Enterprises,
Inc. On March 14, 2003, petitioner Joselito Musni Puno, claiming to be an heir of Carlos L. Puno,
initiated a complaint for specific performance against respondent. Petitioner averred that he is the son of
the deceased with the latter’s common-law wife, Amelia Puno. As surviving heir, he claimed entitlement
to the rights and privileges of his late father as stockholder of respondent. The complaint thus prayed
that respondent allow petitioner to inspect its corporate book, render an accounting of all the
transactions it entered into from 1962, and give petitioner all the profits, earnings, dividends, or income
pertaining to the shares of Carlos L. Puno.
Respondent filed a motion to dismiss on the ground that petitioner did not have the legal
personality to sue because his birth certificate names him as "Joselito Musni Muno." Apropos, there was
yet a need for a judicial declaration that "Joselito Musni Puno" and "Joselito Musni Muno" were one and
the same.

ISSUE:

Whether or not there was an automatic transfer of shares of stovk.

RULING:

NO.

Upon the death of a shareholder, the heirs do not automatically become stockholders of the
corporation and acquire the rights and privileges of the deceased as shareholder of the corporation. The
stocks must be distributed first to the heirs in estate proceedings, and the transfer of the stocks must
be recorded in the books of the corporation. Section 63 of the Corporation Code provides that no
transfer shall be valid, except as between the parties, until the transfer is recorded in the books of the
corporation.16 During such interim period, the heirs stand as the equitable owners of the stocks, the
executor or administrator duly appointed by the court being vested with the legal title to the
stock.17 Until a settlement and division of the estate is effected, the stocks of the decedent are held by
the administrator or executor. Consequently, during such time, it is the administrator or executor who
is entitled to exercise the rights of the deceased as stockholder.
Thus, even if petitioner presents sufficient evidence in this case to establish that he is the son of
Carlos L. Puno, he would still not be allowed to inspect respondent’s books and be entitled to receive
dividends from respondent, absent any showing in its transfer book that some of the shares owned by
Carlos L. Puno were transferred to him. This would only be possible if petitioner has been recognized as
an heir and has participated in the settlement of the estate of the deceased.
Corollary to this is the doctrine that a determination of whether a person, claiming proprietary
rights over the estate of a deceased person, is an heir of the deceased must be ventilated in a special
proceeding instituted precisely for the purpose of settling the estate of the latter. The status of an
illegitimate child who claims to be an heir to a decedent’s estate cannot be adjudicated in an ordinary
civil action, as in a case for the recovery of property. The doctrine applies to the instant case, which is
one for specific performance — to direct Respondent Corporation to allow petitioner to exercise rights
that pertain only to the deceased and his representatives.

289 | P a g e
Law 321_Corporation LAW_ Case Digest

IMELDA O. COJUANGCO, PRIME HOLDINGS, INC., AND THE ESTATE OF RAMON U. COJUANGCO
vs.
SANDIGANBAYAN, REPUBLIC OF THE PHILIPPINES, AND THE SHERIFF OF SANDIGANBAYAN
G.R. No. 183278 April 24, 2009

FACTS:

The Republic of the Philippines (Republic) filed before the Sandiganbayan a "Complaint for
Reconveyance, Reversion, Accounting, Restitution and Damages," docketed as Civil Case 0002, praying
for the recovery of alleged ill-gotten wealth from the late President Marcos and former First Lady Imelda
Marcos and their cronies, including some 2.4 million shares of stock in the Philippine Long Distance
Telephone Company (PLDT).
The complaint, which was later amended to implead herein petitioners Ramon and Imelda
Cojuangco (the Cojuangcos), alleged that the Marcoses’ ill-gotten wealth included shares in the PLDT
covered by shares of stock in the Philippine Telecommunications Investment Corporation (PTIC),
registered in the name of Prime Holdings, Inc. (Prime Holdings).

ISSUE:

Whether or not the Republic entitled to the dividends, interests, and earnings thereof.

RULING:

YES.

It would be absurd to award the shares to the Republic as their owner and not include the
dividends and interests accruing thereto. An owner who cannot exercise the "juses" or attributes of
ownership -- the right to possess, to use and enjoy, to abuse or consume, to accessories, to dispose or
alienate, to recover or vindicate, and to the fruits - is a crippled owner.
This Court, in directing the reconveyance to the Republic of the 111,415 shares of PLDT stock
owned by PTIC in the name of Prime Holdings, declared the Republic as the owner of said shares and,
necessarily, the dividends and interests accruing thereto.
Ownership is a relation in law by virtue of which a thing pertaining to one person is completely
subjected to his will in everything not prohibited by law or the concurrence with the rights of another.
Its traditional elements or attributes include jus utendi or the right to receive from the thing what it
produces.

REPUBLIC OF THE PHILIPPINES represented by the PRESIDENTIAL COMMISSION ON GOOD


GOVERNMENT (PCGG)
vs.
SANDIGANBAYAN (SECOND DIVISION) and ROBERTO S. BENEDICTO
G.R. No. 129406 March 6, 2006

FACTS:

The case is one of several suits involving ill-gotten or unexplained wealth that petitioner Republic,
through the PCGG, filed with the Sandiganbayan against private respondent Roberto S. Benedicto and
others pursuant to Executive Order (EO) No. 14, series of 1986. Pursuant to its mandate under EO No.
1, series of 1986, the PCGG issued writs placing under sequestration all business enterprises, entities
and other properties, real and personal, owned or registered in the name of private respondent
Benedicto, or of corporations in which he appeared to have controlling or majority interest. Among the
properties thus sequestered and taken over by PCGG fiscal agents were the 227 shares in NOGCCI
owned by private respondent Benedicto and registered in his name or under the names of corporations

290 | P a g e
Law 321_Corporation LAW_ Case Digest

he owned or controlled.Following the sequestration process, PCGG representatives sat as members of


the Board of Directors of NOGCCI, which passed, sometime in October 1986, a resolution effecting a
corporate policy change. The change consisted of assessing a monthly membership due of P150.00 for
each NOGCCI share. Prior to this resolution, an investor purchasing more than one NOGCCI share was
exempt from paying monthly membership due for the second and subsequent shares that he/she
owned.
As sequestrator of the 227 shares of stock in question, PCGG did not pay the corresponding
monthly membership due thereon totalingP2,959,471.00. On account thereof, the 227 sequestered
shares were declared delinquent to be disposed of in an auction sale.

ISSUE:

Whether or not the Sandiganbayan, Second Division, gravely abuse its discretion in holding that
the PCGG is at fault for not paying the membership dues on the 227 sequestered NOGCCI shares of
stock, a failing which eventually led to the foreclosure sale thereof.

RULING:

NO.

The PCGG’s posture that to the owner of the sequestered shares rests the burden of paying the
membership dues is untenable. For one, it lost sight of the reality that such dues are basically
obligations attached to the shares, which, in the final analysis, shall be made liable, thru delinquency
sale in case of default in payment of the dues. For another, the PCGG as sequestrator-receiver of such
shares is, as stressed earlier, duty bound to preserve the value of such shares. Needless to state,
adopting timely measures to obviate the loss of those shares forms part of such duty and due diligence.
Given the circumstances leading to the auction sale of the subject NOGCCI shares, PCGG’s
lament about public respondent Sandiganbayan having erred or, worse still, having gravely abused its
discretion in its determination as to who is at fault for the loss of the shares in question can hardly be
given cogency.

291 | P a g e
Law 321_Corporation LAW_ Case Digest

THE RURAL BANK OF LIPA CITY, INC., THE OFFICERS AND DIRECTORS, BERNARDO BAUTISTA,
JAIME CUSTODIO, OCTAVIO KATIGBAK, FRANCISCO CUSTODIO, and JUANITA BAUTISTA OF
THE RURAL BANK OF LIPA CITY, INC.,
vs.
HONORABLE COURT OF APPEALS, HONORABLE COMMISSION EN BANC, SECURITIES AND
EXCHANGE COMMISSION, HONORABLE ENRIQUE L. FLORES, JR., in his capacity as Hearing
Officer, REYNALDO VILLANUEVA, SR, AVELINA M. VILLANUEVA, CATALINO VILLANUEVA,
ANDRES GONZALES, AURORA LACERNA, CELSO LAYGO, EDGARDO REYES, ALEJANDRA
TONOGAN
G.R. No. 124535 September 28, 2001

FACTS:

Private respondent Reynaldo Villanueva, Sr., a stockholder of the Rural Bank of Lipa City,
executed a Deed of Assignment, wherein he assigned his shares, as well as those of eight (8) other
shareholders under his control with a total of 10,467 shares, in favor of the stockholders of the Bank
represented by its directors Bernardo Bautista, Jaime Custodio and Octavio Katigbak. Sometime
thereafter, Reynaldo Villanueva, Sr. and his wife, Avelina, executed an Agreement wherein they
acknowledged their indebtedness to the Bank in the amount of Four Million Pesos (P4,000,000.00), and
stipulated that said debt will be paid out of the proceeds of the sale of their real property described in
the Agreement. At a meeting of the Board of Directors of the Bank on November 15, 1993, the Villanueva
spouses assured the Board that their debt would be paid on or before December 31 of that same year;
otherwise, the Bank would be entitled to liquidate their shareholdings, including those under their
control.
When the Villanueva spouses failed to settle their obligation to the Bank on the due date, the
Board sent them a letter3 demanding: (1) the surrender of all the stock certificates issued to them; and
(2) the delivery of sufficient collateral to secure the balance of their debt amounting to P3,346,898.54.

ISSUE:

Whether or not the transfer of title to such shares is ineffective until and unless the duly indorsed
certificate of stock is delivered to them notwithstanding the execution of the deed of assignment in favor
of the petitioners.

RULING:

NO.

The rule is that the delivery of the stock certificate duly endorsed by the owner is the operative
act of transfer of shares from the lawful owner to the transferee. Thus, title may be vested in the
transferee only by delivery of the duly indorsed certificate of stock.
We have uniformly held that for a valid transfer of stocks, there must be strict compliance with
the mode of transfer prescribed by law. The requirements are: (a) There must be delivery of the stock
certificate: (b) The certificate must be endorsed by the owner or his attorney-in-fact or other persons
legally authorized to make the transfer; and (c) To be valid against third parties, the transfer must be
recorded in the books of the corporation. As it is, compliance with any of these requisites has not been
clearly and sufficiently shown.
It may be argued that despite non-compliance with the requisite endorsement and delivery, the
assignment was valid between the parties, meaning the private respondents as assignors and the
petitioners as assignees. While the assignment may be valid and binding on the petitioners and private
respondents, it does not necessarily make the transfer effective. Consequently, the petitioners, as mere
assignees, cannot enjoy the status of a stockholder, cannot vote nor be voted for, and will not be entitled
to dividends, insofar as the assigned shares are concerned. Parenthetically, the private respondents
cannot, as yet, be deprived of their rights as stockholders, until and unless the issue of ownership and
transfer of the shares in question is resolved with finality.
BATANGAS LAGUNA TAYABAS BUS COMPANY, INC., DOLORES A. POTENCIANO, MAX JOSEPH A.
POTENCIANO, MERCEDELIN A. POTENCIANO, and DELFIN C. YORRO
vs.
BENJAMIN M. BITANGA, RENATO L. LEVERIZA, LAUREANO A. SIY, JAMES A. OLAYVAR,
EDUARDO A. AZUCENA, MONINA GRACE S. LIM, and GEMMA M. SANTOS
G.R. No. 137934 August 10, 2001

FACTS:

292 | P a g e
Law 321_Corporation LAW_ Case Digest

Dolores A. Potenciano, Max Joseph A. Potenciano, Mercedelin A. Potenciano, Delfin C. Yorro, and
Maya Industries, Inc., entered into a Sale and Purchase Agreement, whereby they sold to BMB Property
Holdings, Inc., represented by its President, Benjamin Bitanga, their 21,071,114 shares of stock in
BLTB. The said shares represented 47.98% of the total outstanding capital stock of BLTB. The
contracting parties stipulated that the downpayment was conditioned upon receipt by the buyer of
certain documents upon signing of the Agreement, namely, the Secretary's Certificate stating that the
Board of Directors of Maya Industries, Inc. authorized the sale of its shares in BLTB and the execution
of the Agreement, and designating Dolores A. Potenciano as its Attorney-in-Fact; the Special Power of
Attorney executed by each of the sellers in favor of Dolores A. Potenciano for purposes of the Agreement;
the undated written resignation letters of the Directors of BLTB, except Henry John A. Potenciano,
Michael A. Potericiano and Candido A. Potenciano); a revocable proxy to vote the subject shares made by
the sellers in favor of the buyer; a Declaration of Trust made by the sellers in favor of the buyer
acknowledging that the subject shares shall be held in trust by the sellers for the buyer pending their
transfer to the latter's name; and the duly executed capital gains tax return forms covering the sale,
indicating no taxable gain on the same.

ISSUE:

Whether or not the Bitanga group vote or be voted upon.

RULING:

NO.

We are in full accord with the SEC En Banc on this matter. Indeed, until registration is
accomplished, the transfer, though valid between the parties, cannot be effective as against the
corporation. Thus, the unrecorded transferee, the Bitanga group in this case, cannot vote nor be voted
for. The purpose of registration, therefore, is two-fold: to enable the transferee to exercise all the rights
of a stockholder, including the right to vote and to be voted for, and to inform the corporation of any
change in share ownership so that it can ascertain the persons entitled to the rights and subject to the
liabilities of a stockholder. Until challenged in a proper proceeding, a stockholder of record has a right to
participate in any meeting; his vote can be properly counted to determine whether a stockholders'
resolution was approved, despite the claim of the alleged transferee. On the other hand, a person who
has purchased stock, and who desires to be recognized as a stockholder for the purpose of voting, must
secure such a standing by having the transfer recorded on the corporate books. Until the transfer is
registered, the transferee is not a stockholder but an outsider.
The Court finds no error either in jurisdiction or judgment on the part of the SEC En Banc, since
its conclusions of law were anchored on established principles and jurisprudence. The petition is
denied.

SPOUSES JOSE ABEJO AND AURORA ABEJO, TELEC. TRONIC SYSTEMS, INC.
vs.
HON. RAFAEL DE LA CRUZ, JUDGE OF THE REGIONAL TRIAL COURT (NATIONAL CAPITAL
JUDICIAL REGION, BRANCH CLX-PASIG), SPOUSES AGAPITO BRAGA AND VIRGINIA BRAGA,
VIRGILIO BRAGA AND NORBERTO BRAGA
G.R. No. L-63558 May 19, 1987

FACTS:

This case involves a question of who, between the Regional Trial Court and the Securities and
Exchange Commission (SEC), has original and exclusive jurisdiction over the dispute between the
principal stockholders of the corporation Pocket Bell Philippines, Inc. (Pocket Bell), a "tone and voice
paging corporation," namely, the spouses Jose Abejo and Aurora Abejo (hereinafter referred to as the
Abejos) and the purchaser, Telectronic Systems, Inc. (hereinafter referred to as Telectronics) of their
133,000 minority shareholdings (for P5 million) and of 63,000 shares registered in the name of Virginia
Braga and covered by five stock certificates endorsed in blank by her (for P1,674,450.00), and the
spouses Agapito Braga and Virginia Braga (hereinafter referred to as the Bragas), erstwhile majority

293 | P a g e
Law 321_Corporation LAW_ Case Digest

stockholders. With the said purchases, Telectronics would become the majority stockholder, holding
56% of the outstanding stock and voting power of the corporation Pocket Bell.
With the said purchases in 1982, Telectronics requested the corporate secretary of the
corporation, Norberto Braga, to register and transfer to its name, and those of its nominees the total
196,000 Pocket Bell shares in the corporation's transfer book, cancel the surrendered certificates of
stock and issue the corresponding new certificates of stock in its name and those of its nominees.
Norberto Braga, the corporate secretary and son of the Bragas, refused to register the aforesaid transfer
of shares in t e corporate oo s, asserting that the Bragas claim preemptive rights over the 133,000 Abejo
shares and that Virginia Braga never transferred her 63,000 shares to Telectronics but had lost the five
stock certificates representing those shares. This triggered off the series of intertwined actions between
the protagonists, all centered on the question of jurisdiction over the dispute, which were to culminate
in the filing of the two cases at bar.

ISSUE:

Whether or not the trial court have jurisdiction over aforesaid case.

RULING:

YES.

The very complaint of the Bragas for annulment of the sales and transfers as filed by them in the
regular court questions the validity of the transfer and endorsement of the certificates of stock, claiming
alleged pre-emptive rights in the case of the Abejos' shares and alleged loss of thio certificates and lack
of consent and consideration in the case of Virginia Braga's shares. Such dispute c learly involve's
controversies "between and among stockholders, " as to the Abej os' right to sell and dispose of their
shares to Telectronics, the validity of the latter's acquisition of Virginia Braga's shares, who between the
Bragas and the Abejos' transferee should be recognized as the controlling shareholders of the
corporation, with the right to elect the corporate officers and the management and control of its
operations. Such a dispute and case clearly fag within the original and exclusive jurisdiction of the SEC
to decide, under Section 5 of P.D. 902-A, above-quoted. The restraining order issued by the Regional
Trial Court restraining Telectronics agents and representatives from enforcing their resolution
constituting themselves as the new set of officers of Pocket Bell and from assuming control of the
corporation and discharging their functions patently encroached upon the SEC's exclusive jurisdiction
over such specialized corporate controversies calling for its special competence.

294 | P a g e
Law 321_Corporation LAW_ Case Digest

BATONG BUHAY GOLD MINES, INC.


vs.
THE COURT OF APPEALS and INC. MINING CORPORATION
G.R. No. L-45048 January 7, 1987

FACTS:

The defendant Batong Buhay Gold Mines, Inc. issued Stock Certificate No. 16807 covering 62,495
shares with a par value of P0.01 per share to Francisco Aguac who was then legally married to Paula G.
Aguac, but the said spouses had lived separately for more than fourteen (14) years prior to the said date.
Later, Francisco Aguac sold his shares covered by Stock Certificate No. 16807 for the sum of
P9,374.70 in favor of the plaintiff, the said transaction being evidenced by a deed of sale (Exhibit D). The
said sale was made by Francisco Aguac without the knowledge or consent of his wife Paula G. Aguac.
On the same date of the sale, Paula G. Aguac wrote a letter to the president of defendant Batong
Buhay Gold Mines, Inc. asking that the transfer of the shares sold by her husband be withheld,
inasmuch as the same constituted conjugal property and her share of proceeds of the sale was not given
to her (Exhibit 1).

ISSUE:

Whether or not the Court of Appeals may award damages by way of unrealized profits despite the
absence of supporting evidence, or merely on the basis of pure assumption, speculation or conjecture;
or can the respondent recover damages by way of unrealized profits when it has not shown that it was
damaged in any manner by the act of petitioner.

RULING:

NO.

The stipulation of facts of the parties does not at all show that private respondent intended to
sell, or would sell or would have sold the stocks in question on specified dates. While it is true that
shares of stock may go up or down in value (as in fact the concerned shares here really rose from fifteen
(15) centavos to twenty three or twenty four (23/24) centavos per share and then fell to about two (2)
centavos per share, still whatever profits could have been made are purely SPECULATIVE, for it was
difficult to predict with any decree of certainty the rise and fall in the value of the shares. Thus this
Court has ruled that speculative damages cannot be recovered.
It is easy to say now that had private respondent gained legal title to the shares, it could have
sold the same and reaped a profit of P5,624.95 but it could not do so because of petitioner's refusal to
transfer the stocks in the former's name at the time demand was made, but then it is also true that
human nature, being what it is, private respondent's officials could also have refused to sell and instead
wait for expected further increases in value.

CHEMPHIL EXPORT AND IMPORT CORPORATION


vs.
THE HON. COURT OF APPEALS (Former Twelfth Division), PHILIPPINE INVESTMENTS SYSTEMS
ORGANIZATION (PISO), BANK OF THE PHILIPPINE ISLANDS (BPI), PHILIPPINE COMMERCIAL
INDUSTRIAL BANK (PCIB), RIZALCOMMERCIAL BANKING CORPORATION (RCBC) and LAND BANK
OF THE PHILIPPINES (LBP)
G.R. No. 97217 April 10, 1992

295 | P a g e
Law 321_Corporation LAW_ Case Digest

FACTS

Dynetics, Inc. and Antonio M. Garcia filed a complaint for declaratory relief and/or injunction
against the PISO, BPI, LBP, PCIB and RCBC or the consortium with the Regional Trial Court of Makati,
Branch 45 (Civil Case No. 8527), seeking judicial declaration, construction and interpretation of the
validity of the surety agreement that Dynetics and Garcia had entered into with the consortium and to
perpetually enjoin the latter from claiming, collecting and enforcing any purported obligations which
Dynetics and Garcia might have undertaken in said agreement.
The consortium filed their respective answers with counterclaims alleging that the surety
agreement in question was valid and binding and that Dynetics and Garcia were liable under the terms
of the said agreement. It likewise applied for the issuance of a writ of preliminary attachment against
Dynetics and Garcia.

ISSUE:

Whether or not the attachment of shares of stock, in order to bind third persons, must be
recorded in the stock and transfer book of the corporation.

RULING:

NO.

Section 7(d), Rule 57 of the Rules of Court was complied with by the consortium (through the
Sheriff of the trial court) when the notice of garnishment over the Chemphil shares of Garcia was served
on the president of Chemphil on July 19, 1985. Indeed, to bind third persons, no law requires that an
attachment of shares of stock be recorded in the stock and transfer book of a corporation. The
statement attributed by the Regional Trial Court to the Supreme Court in Samahang Magsasaka, Inc.
vs. Gonzalo Chua Guan, G.R. No. L-7252, February 25, 1955 (unreported), to the effect that "as between
two attaching creditors, the one whose claim was registered first on the books of the corporation enjoys
priority," is an obiter dictum that does not modify the procedure laid down in Section 7(d), Rule 57 of
the Rules of Court.
Therefore, ruled the Court of Appeals, the attachment made over the Chemphil shares in the
name of Garcia on July 19, 1985 was made in accordance with law and the lien created thereby
remained valid and subsisting at the time Garcia sold those shares to FCI (predecessor-in-interest of
appellee CEIC) in 1988.

GONZALO CHUA GUAN


vs.
SAMAHANG MAGSASAKA, INC., and SIMPLICIO OCAMPO, ADRIANO G. SOTTO, and EMILIO
VERGARA, as president, secretary and treasurer respectively of the same
G.R. No. L-42091 November 2, 1935

FACTS:

Samahang Magsasaka, Inc., is a corporation duly organized under the laws of the Philippine
Islands with principal office in Cabanatuan, Nueva Ecija, and that the individual defendants are the
president, secretary and treasurer respectively of the same; that on June 18, 1931, Gonzalo H. Co Toco
was the owner of 5,894 shares of the capital stock of the said corporation represented by nine

296 | P a g e
Law 321_Corporation LAW_ Case Digest

certificates having a par value of P5 per share; that on said date Gonzalo H. Co Toco, a resident of
Manila, mortgaged said 5,894 shares to Chua Chiu to guarantee the payment of a debt of P20,000 due
on or before June 19, 1932. The said certificates of stock were delivered with the mortgage to the
mortgagee, Chua Chiu. The said mortgage was duly registered in the office of the register of deeds of
Manila on June 23, 1931, and in the office of the said corporation on September 30, 1931. On
November 28, 1931, Chua Chiu assigned all his right and interest in the said mortgage to the plaintiff
and the assignment was registered in the office of the register of deeds in the City of Manila on
December 28, 1931, and in the office of the said corporation on January 4, 1932.
The debtor, Gonzalo H. Co Toco, having defaulted in the payment of said debt at maturity, the
plaintiff foreclosed said mortgage and delivered the certificates of stock and copies of the mortgage and
assignment to the sheriff of the City of Manila in order to sell the said shares at public auction. The
sheriff auctioned said 5,894 shares of stock on December 22, 1932, and the plaintiff having been the
highest bidder for the sum of P14,390, the sheriff executed in his favor a certificate of sale of said
shares. The plaintiff tendered the certificates of stock standing in the name of Gonzalo H. Co Toco to the
proper officers of the corporation for cancellation and demanded that they issue new certificates in the
name of the plaintiff. The said officers (the individual defendants) refused and still refuse to issue said
new shares in the name of the plaintiff.

ISSUE:

Whether or not the mortgage registered considered the certificate of registration in the
corporation.

RULING:

YES.

By analogy with the foregoing and considering the ownership of shares in a corporation as
property distinct from the certificates which are merely the evidence of such ownership, it seems to us a
reasonable construction of section 4 of Act No. 1508 to hold that the property in the shares may be
deemed to be situated in the province in which the corporation has its principal office or place of
business. If this province is also the province of the owner's domicile, a single registration sufficient. If
not, the chattel mortgage should be registered both at the owner's domicile and in the province where
the corporation has its principal office or place of business. In this sense the property mortgaged is not
the certificate but the participation and share of the owner in the assets of the corporation.

COLLECTOR OF INTERNAL REVENUE


vs.
ANGLO CALIFORNIA NATIONAL BANK (CROCKER-ANGLO NATIONAL BANK), as Treasurer for
CALAMBA SUGAR ESTATE, INC
G .R. No. L-12476 January 29, 1960

FACTS:

Calamba Sugar Estate, Inc., herein represented by its trustee, the Anglo California National
Bank, is a foreign corporation organized and existing under the laws of the State of California, U.S.A.,
duly licensed (on May 8, 1946) to do business in the Philippines. It has consistently filed its income tax
returns here through its resident attorney-in-fact. On May 14, 1956, the petitioners Collector of Internal
Revenue the corporation of an assessment for alleged deficiency income taxes for the years 1953, 1954
and 1955 in the respective amounts of P138,855.00, P131,759.00 and P393,459.00, supposedly based
upon capital again derived from the respondent's sale to the Pasumil Planters, Inc., of P250,000 shares
of the capital stock of the Pampanga Sugar Mills (a domestic corporation) and of a promissory note,
dated January 1, 1950, executed by the Pampanga Sugar Mills in the sum of $500,000.00. In an appeal
by the respondent from the ruling of the Collector, the Court of Tax Appeals reversed said ruling and
absolved the respondent form liability.

ISSUE:

297 | P a g e
Law 321_Corporation LAW_ Case Digest

Whether or not respondent is liable for tax regarding the transfer of share of stock.

RULING:

YES.

It is hardly disputable that although shares of stock of a corporation represent equities may
consist of real as well as personal properties therein, they are considered under applicable law and
jurisprudence as intangible personal properties.
Section 24 of the National Internal Revenue Codes levies income taxes on foreign
corporations only on income derived from sources within the Philippines; and with respect to capital gains
on the sale of personal properties, section 37 (e) of the same Tax Code deems the place of sale as also
that place or source of the capital gain:
In this case, it is admitted that the negotiation, perfection and consummation of the contract of
sale were all done in California, U.S.A. It follows that title to the shares of stock passed from the vendor
to the vendee at said place, from which time the incidents of ownership vested on the buyer. The petition
is denied.

298 | P a g e
Law 321_Corporation LAW_ Case Digest

APOLINARIO G. DE LOS SANTOS and ISABELO ASTRAQUILLO


vs.
J. HOWARD MCGRATH ATTORNEY GENERAL OF THE UNITED STATES, SUCCESSOR TO THE
PHILIPPINE ALIEN PROPERTY ADMINISTRATION OF THE UNITED STATES, REPUBLIC OF THE
PHILIPPINES
G.R. No. L-4818 February 28, 1955

FACTS:

This action involves the title to 1,600,000 shares of stock of the Lepanto Consolidated Mining Co.,
Inc., a corporation duly organized and existing under the laws of the Philippines, hereinafter referred to,
for the sake of brevity, as the Lepanto. Originally, one-half of said shares of stock were claimed by
plaintiff, Apolinario de los Santos, and the other half, by his co-plaintiff Isabelo Astraquillo. During the
pendency of this case, the latter has allegedly conveyed and assigned his interest in and to said half
claimed by him to the former. The shares of stock in question are covered by several stock certificates
issued in favor of Vicente Madrigal, who is registered in the books of the Lepanto as owner of said stocks
and whose indorsement in blank appears on the back of said certificates, all of which, except certificates
No. 2279 — marked Exhibit 2 — covering 55,000 shares, are in plaintiffs' possession. So was said
Exhibit 2, up to sometime in 1945 or 1946 when said possession was lost under the conditions set forth
in subsequent pages.

ISSUE:

Whether or not the plaintiffs had the owners of the shares of stock in question.

RULING:

NO.

In the case at bar, neither madrigal nor the Mitsuis had alienated shares of stock in question. It
is not even claimed that either had, through negligence, given — occasion for an improper or irregular
disposition of the corresponding stock certificates. Plaintiffs merely argue without any evidence
whatsoever thereon — that Kitajimamight have, or must have, assigned the certificates on or
before December 1942, although, as above stated, this is, not only, improbable, under the conditions,
then obtaining, but, also., impossible, considering that, in April 1943, Kitajima delivered the
instruments to Miwa, who kept them in its possession until 1945. At any rate, such assignment by Miwa
— granting for the sake of argument the accuracy of the surmise of plaintiffs herein — was
unauthorized by the mitsuis, who, in the light of the precedents cited above, are not chargeable with
negligence. In other words, assuming that Kitajima had been guilty of embezzlement, by negotiating the
stock certificates in question for his personal benefit, as claimed by the plaintiffs, the title of his
assignees and successors in interest would still be subject to the rights of the registered owner, namely,
Madrigal, and consequently, of the party for whose benefit and account the latter held the corresponding
shares of stock, that is to say, the Mitsuis.
In conclusion, when the Property Custodian issued the Vesting Order complained of, the shares
of stock in question belonged to the Mitsuis, admittedly an enemy corporation, so that Vesting Order is
in conformity with law and should be upheld. Wherefore, the decision appealed from is hereby reversed,
and the complaint, accordingly, dismissed, with costs against the plaintiffs-appellees. It is so ordered.

GLORIA M. DE ERQUIAGA, administratrix of the estate of the late SANTIAGO DE ERQUIAGA &
HON. FELICIANO S. GONZALES
vs.
HON. COURT OF APPEALS, AFRICA VALDEZ VDA. DE REYNOSO, JOSES V. REYNOSO, JR.,
EERNESTO , SYLVIA REYNOSO, LOURDES REYNOSO, CECILE REYNOSO, EDNA REYNOSO,
ERLINDA REYNOSO & EMILY REYNOSO
G.R. No. 47206 September 27, 1989

299 | P a g e
Law 321_Corporation LAW_ Case Digest

FACTS:

Santiago de Erquiaga was the owner of 100% or 3,100 paid-up shares of stock of the Erquiaga
Development Corporation which owns the Hacienda San Jose in Irosin, Sorsogon (p. 212, Rollo). He
entered into an Agreement with Jose L. Reynoso to sell to the latter his 3,100 shares (or 100%) of
Erquiaga Development Corporation for P900,000 payable in installments on definite dates fixed in the
contract but not later than November 30, 1968.
Because Reynoso failed to pay the second and third installments on time, the total price of the
sale was later increased to P971,371.70 payable on or before December 17, 1969. The difference of
P71,371.70 represented brokers' commission and interest (CFI Decision, pp. 75, 81, 90, 99,Rollo). As of
December 17, 1968, Reynoso was able to pay the total sum of P410,000 to Erquiaga who thereupon
transferred all his shares (3,100 paid-up shares) in Erquiaga Development Corporation to Reynoso, as
well as the possession of the Hacienda San Jose, the only asset of the corporation However, as provided
in paragraph 3, subparagraph (c) of the contract to sell, Reynoso pledged 1,500 shares in favor of
Erquiaga as security for the balance of his obligation. Reynoso failed to pay the balance of P561,321.70
on or before December 17, 1969, as provided in the promissory notes he delivered to Erquiaga.

ISSUE:

Whether or not the Corporation Code Applicable.

RULING:

YES.

We find no reversible error in the Court of Appeals' decision directing the clerk of court of the trial
court to execute a deed of conveyance to Erquiaga of the 1,600 shares of stock of the Erquiaga
Development Corporation still in Reynoso's name and/or possession, in accordance with the procedure
in Section 10, Rule 39 of the Rules of Court. Neither did it err in annulling the trial court's order: (1)
allowing Erquiaga to vote the 3,100 shares of Erquiaga Development Corporation without having
effected the transfer of those shares in his name in the corporate books; and (2) authorizing Erquiaga to
call a special meeting of the stockholders of the Erquiaga Development Corporation and to vote the
3,100 shares, without the pre-requisite registration of the shares in his name. It is a fundamental rule
in Corporation Law (Section 35) that a stockholder acquires voting rights only when the shares of stock
to be voted are registered in his name in the corporate books.
The order of respondent Court directing Erquiaga to return the sum of P410,000 (or net P348,000
after deducting P62,000 due from Reynoso under the decision) as the price paid by Reynoso for the
shares of stock, with legal rate of interest, and the return by Reynoso of Erquiaga's 3,100 shares with
the fruits(construed to mean not only dividends but also fruits of the corporation's Hacienda San Jose)
is in full accord with Art. 1385 of the Civil Code.
The Hacienda San Jose and 1,500 shares of stock have already been returned to Erquiaga.
Therefore, upon the conveyance to him of the remaining 1,600 shares, Erquiaga (or his heirs) should
return to Reynoso the price of P410,000 which the latter paid for those shares. Pursuant to the
rescission decreed in the final judgment, there should be simultaneous mutual restitution of the
principal object of the contract to sell (3,100 shares) and of the consideration paid (P410,000).
NEMESIO GARCIA
vs.
NICOLAS JOMOUAD, Ex-officio Provincial Sheriff of Cebu and SPOUSES JOSE ATINON & SALLY
ATINON
G.R. No. 133969 January 26, 2000

FACTS:

Petitioner filed an action for injunction with prayer for preliminary injunction against respondents
spouses Jose and Sally Atinon and Nicolas Jomouad, ex-officio sheriff of Cebu.
Said action stemmed from an earlier case for collection of sum of money, docketed as Civil Case
No. CEB-10433, before the RTC, Branch 10 of Cebu, filed by the spouses Atinon against Jaime Dico. In
that case (collection of sum of money), the trial court rendered judgment ordering Dico to pay the
spouses Atinon the sum of P900,000.00 plus interests.
After said judgment became final and executory, respondent sheriff proceeded with its execution.
In the course thereof, the Proprietary Ownership Certificate (POC) No. 0668 in the Cebu Country Club,
which was in the name of Dico, was levied on and scheduled for public auction. Claiming ownership

300 | P a g e
Law 321_Corporation LAW_ Case Digest

over the subject certificate, petitioner filed the aforesaid action for injunction with prayer for preliminary
injunction to enjoin respondents from proceeding with the auction.

ISSUE:

Whether or not the appellate court erroneously rely on Section 63 of the Corporation Code in
upholding the levy on the subject certificate to satisfy the judgment debt of Dico in Civil Case No. CEB-
14033.

RULING:

NO.

The transfer of the subject certificate made by Dico to petitioner was not valid as to the spouses
Atinon, the judgment creditors, as the same still stood in the name of Dico, the judgment debtor, at the
time of the levy on execution. In addition, as correctly ruled by the CA, the entry in the minutes of the
meeting of the Club's board of directors noting the resignation of Dico as proprietary member thereof
does not constitute compliance with Section 63 of the Corporation Code. Said provision of law strictly
requires the recording of the transfer in the books of the corporation, and not elsewhere, to be valid as
against third parties. Accordingly, the CA committed no reversible error in rendering the assailed
decision.

301 | P a g e
Law 321_Corporation LAW_ Case Digest

BENITO H. LOPEZ
vs.
THE COURT OF APPEALS and THE PHILIPPINE AMERICAN GENERAL INSURANCE CO., INC.
G.R. No. L-33157 June 29, 1982

FACTS:

Petitioner Benito H. Lopez obtained a loan in the amount of P20,000.00 from the Prudential Bank
and Trust Company. Also, he executed a promissory note for the same amount, in favor of the said
Bank, binding himself to repay the said sum one (1) year after the said date, with interest at the rate of
10% per annum. In addition to said promissory note, he executed Surety Bond No. 14164 in which he,
as principal, and Philippine American General Insurance Co., Inc. (PHILAMGEN) as surety, bound
themselves jointly and severally in favor of Prudential Bank for the payment of the sum of P20,000.00.
On the same occasion, Lopez also executed in favor of Philamgen an indemnity agreement whereby he
agreed "to indemnify the Company and keep it indemnified and hold the same harmless from and
against any and all damages, losses, costs, stamps, taxes, penalties, charges and expenses of whatever
kind and nature which the Company shall or may at any time sustain or incur in consequence of having
become surety upon the bond." 1 At the same time, Lopez executed a deed of assignment of 4,000 shares
of the Baguio Military Institution entitled "Stock Assignment Separate from Certificate". With the
execution of this deed of assignment, Lopez endorsed the stock certificate and delivered it to Philamgen.

ISSUE:

Whether or not where, as in this case, a party "sells, assigns and transfers" and delivers shares of
stock to another, duly endorsed in blank, in consideration of a contingent obligation of the former to the
latter, and, the obligations having arisen, the latter causes the shares of stock to be transferred in its
name, hass the juridical nature of the transaction-a dation in payment or a pledge.

RULING:

YES.

Considering the explicit terms of the deed denominated "Stock Assignment Separate from
Certificate", hereinbefore copied verbatim, Lopez sold, assigned and transferred unto Philamgen the
stocks involved "for and in consideration of the obligations undertaken" by Philamgen "under the terms
and conditions of the surety bond executed by it in favor of the Prudential Bank" and "for value
received". On its face, it is neither pledge nor dation in payment. The document speaks of an outright
sale as there is a complete and unconditional divestiture of the incorporeal property consisting of stocks
from Lopez to Philamgen. The transfer appears to have been an absolute conveyance of the stocks to
Philamgen whether or not Lopez defaults in the payment of P20,000.00 to Prudential Bank. While it is a
conveyance in consideration of a contingent obligation, it is not itself a conditional conveyance.
It is true that if Lopez should "well and truly perform and fulfill all the undertakings, covenants,
terms, conditions, and agreements stipulated" in his promissory note to Prudential Bank, the obligation
of Philamgen under the surety bond would become null and void. Corollarily, the stock assignment,
which is predicated on the obligation of Philamgen under the surety bond, would necessarily become
null and void likewise, for want of cause or consideration under Article 1352 of the New Civil Code. But
this is not the case here because aside from the obligations undertaken by Philamgen under the surety
bond, the stock assignment had other considerations referred to therein as "value received". Hence,
based on the manifest terms thereof, it is an absolute transfer.

302 | P a g e
Law 321_Corporation LAW_ Case Digest

ENRIQUE MONSERRAT
vs.
CARLOS G. CERON, ET AL. ERMA, INC., and, THE SHERIFF OF MANILA
G.R. No. 37078 September 27, 1933

FACTS:

Enrique Monserrat, was the president and manager of the Manila Yellow Taxicab Co., Inc., and
the owner of P1,200 common shares of stock thereof. On March 25, 1930, in consideration of the
interest shown and the financial aid extended him in the organization of the corporation by Carlos G.
Ceron, one of the defendants herein, Enrique Monserrat assigned to the former the usufruct of half of
the aforesaid common shares of stock, the corresponding certificate of stock No. 7, having been issued
in the name of said Carlos G, Ceron to that effect on March 24, 1930. (Exhibit 1.) Said assignment or
transfer only gave the transferee the right to enjoy, during his lifetime, the profits which might be
derived from the shares assigned him, prohibiting him from selling, mortgaging, encumbering, alienating
or otherwise exercising any act implying absolute ownership of all or any of the shares in question, the
transferor having reserved for himself and his heirs the right to vote derived from said shares of stock
and to recover the ownership thereof at the termination of the usufruct (Exhibit A). Stock certificate No.
7 was recorded in the name of Carlos G. Ceron and the aforesaid deed of transfer Exhibit A, was noted
by himself as secretary, on page 22 of the Stock and Transfer Book of the Manila Yellow Taxicab Co.,
Inc.

ISSUE:

Whether or not it is necessary to enter upon the books of the corporation a mortgage constituted
on common shares of stock in order that such mortgage may be valid and may have force and effect as
against third persons.

RULING:

YES.

A "transfer" is the act by which owner of a thing delivers it to another with the intent of passing
the rights which he has in it to the latter, and a chattel mortgage is not within the meaning of such
term.
Therefore, the chattel mortgage is not the transfer referred to in section 35 of Act No. 1459
commonly known as the Corporation law, which transfer should be entered and noted upon the books of
a corporation in order to be valid, and which, as has already been said, means the absolute and
unconditional conveyance of the title and ownership of a share of stock.
If, in accordance with said section 35 of the Corporation Law, only the transfer or absolute
conveyance of the ownership of the title to a share need be entered and noted upon the books of the
corporation in order that such transfer may ba valid, therefore, inasmuch as a chattel mortgage of the
aforesaid title is not a complete and absolute alienation of the dominion and ownership thereof, its entry
and notation upon the books of the corporation is not necessary requisite to its validity.

EUGENIO J. PUYAT, ERWIN L. CHIONGBIAN, EDGARDO P. REYES, ANTONIO G. PUYAT, JAIME R.


BLANCO, RAFAEL R. RECTO and REYNALDO L. LARDIZABAL
vs.
HON. SIXTO T. J. DE GUZMAN, JR., as Associate Commissioner of the Securities & Exchange
Commission, EUSTAQUIO T. C. ACERO, R. G. VILDZIUS, ENRIQUE M. BELO, MANUEL G. ABELLO,
SERVILLANO DOLINA, JUANITO MERCADO and ESTANISLAO A. FERNANDEZ

303 | P a g e
Law 321_Corporation LAW_ Case Digest

G.R. No. L-51122 March 25, 1982

FACTS:

On May 14, 2979, an election for the eleven Directors of the International Pipe Industries
Corporation (IPI) a private corporation, was held. b) May 25, 1979. The Acero Group instituted at the
Securities and Exchange Commission (SEC) quo warrantoproceedings, docketed as Case No. 1747 (the
SEC Case), questioning the election of May 14, 1979. The Acero Group claimed that the stockholders'
votes were not properly counted. The Puyat Group claims that at conferences of the parties with
respondent SEC Commissioner de Guzman, Justice Estanislao A. Fernandez, then a member of the
Interim Batasang Pambansa, orally entered his appearance as counsel for respondent Acero to which
the Puyat Group objected on Constitutional grounds. Section 11, Article VIII, of the 1973 Constitution,
then in force, provided that no Assemblyman could "appear as counsel before ... any administrative
body", and SEC was an administrative body. Incidentally, the same prohibition was maintained by the
April 7, 1981 plebiscite. The cited Constitutional prohibition being clear, Assemblyman Fernandez did
not continue his appearance for respondent Acero.

ISSUE:

Whether or not in intervening in the SEC Case, Assemblyman Fernandez is, in effect, appearing
as counsel, albeit indirectly, before an administrative body in contravention of the Constitutional
provision.

RULING:

YES.

Ordinarily, by virtue of the Motion for Intervention, Assemblyman Fernandez cannot be said to be
appearing as counsel. Ostensibly, he is not appearing on behalf of another, although he is joining the
cause of the private respondents. His appearance could theoretically be for the protection of his
ownership of ten (10) shares of IPI in respect of the matter in litigation and not for the protection of the
petitioners nor respondents who have their respective capable and respected counsel.
However, certain salient circumstances militate against the intervention of Assemblyman
Fernandez in the SEC Case. He had acquired a mere P200.00 worth of stock in IPI, representing ten
shares out of 262,843 outstanding shares. Realizing, perhaps, the validity of the objection, he decided,
instead, to "intervene" on the ground of legal interest in the matter under litigation. And it may be noted
that in the case filed before the Rizal Court of First Instance (L-51928), he appeared as counsel for
defendant Excelsior, co-defendant of respondent Acero therein.
Under those facts and circumstances, we are constrained to find that there has been an indirect
"appearance as counsel before ... an administrative body" and, in our opinion, that is a circumvention of
the Constitutional prohibition. The "intervention" was an afterthought to enable him to appear actively
in the proceedings in some other capacity. To believe the avowed purpose, that is, to enable him
eventually to vote and to be elected as Director in the event of an unfavorable outcome of the SEC Case
would be pure naivete. He would still appear as counsel indirectly.
In brief, we hold that the intervention of Assemblyman Fernandez in SEC. No. 1747 falls within
the ambit of the prohibition contained in Section 11, Article VIII of the Constitution.

304 | P a g e
Law 321_Corporation LAW_ Case Digest

ENRIQUE RAZON
vs.
INTERMEDIATE APPELLATE COURT and VICENTE B. CHUIDIAN, in his capacity as Administrator
of the Estate of the Deceased JUAN T. CHUIDIAN
G .R. No. 74306 March 16, 1992

FACTS:

In his complaint filed on June 29, 1971, and amended on November 16, 1971, Vicente B.
Chuidian prayed that defendants Enrique B. Razon, E. Razon, Inc., Geronimo Velasco, Francisco de
Borja, Jose Francisco, Alfredo B. de Leon, Jr., Gabriel Llamas and Luis M. de Razon be ordered to
deliver certificates of stocks representing the shareholdings of the deceased Juan T. Chuidian in the E.
Razon, Inc. with a prayer for an order to restrain the defendants from disposing of the said shares of
stock, for a writ of preliminary attachment v. properties of defendants having possession of shares of
stock and for receivership of the properties of defendant corporation.
In their answer filed on June 18, 1973, defendants alleged that all the shares of stock in the
name of stockholders of record of the corporation were fully paid for by defendant, Razon; that said
shares are subject to the agreement between defendants and incorporators; that the shares of stock
were actually owned and remained in the possession of Razon. Appellees also alleged . . . that neither
the late Juan T. Chuidian nor the appellant had paid any amount whatsoever for the 1,500 shares of
stock in question

ISSUE:

Whether or not petitioner have right over the ownership of the 1,500 shares of stock in E. Razon,
Inc.

RULING:

NO.

In the instant case, there is no dispute that the questioned 1,500 shares of stock of E. Razon, Inc.
are in the name of the late Juan Chuidian in the books of the corporation. Moreover, the records show
that during his lifetime Chuidian was ellected member of the Board of Directors of the corporation which
clearly shows that he was a stockholder of the corporation. From the point of view of the corporation,
therefore, Chuidian was the owner of the 1,500 shares of stock. In such a case, the petitioner who
claims ownership over the questioned shares of stock must show that the same were transferred to him
by proving that all the requirements for the effective transfer of shares of stock in accordance with the
corporation's by laws, if any, were followed or in accordance with the provisions of law.
The petitioner failed in both instances. The petitioner did not present any by-laws which could
show that the 1,500 shares of stock were effectively transferred to him. In the absence of the
corporation's by-laws or rules governing effective transfer of shares of stock, the provisions of the
Corporation Law are made applicable to the instant case.
The law is clear that in order for a transfer of stock certificate to be effective, the certificate must
be properly indorsed and that title to such certificate of stock is vested in the transferee by the delivery
of the duly indorsed certificate of stock. Since the certificate of stock covering the questioned 1,500
shares of stock registered in the name of the late Juan Chuidian was never indorsed to the petitioner,
the inevitable conclusion is that the questioned shares of stock belong to Chuidian. The petitioner's
asseveration that he did not require an indorsement of the certificate of stock in view of his intimate
friendship with the late Juan Chuidian can not overcome the failure to follow the procedure required by
law or the proper conduct of business even among friends. To reiterate, indorsement of the certificate of
stock is a mandatory.

AQUILINO RIVERA, ISAMU AKASAKO, FUJIYAMA HOTEL & RESTAURANT, INC.


vs.
THE HON. ALFREDO C. FLORENDO, as Judge of the Court of First Instance of Manila (Branch
XXXVI), LOURDES JUREIDINI and MILAGROS TSUCHIYA
G.R. No. L-57586. October 8, 1986

FACT:

305 | P a g e
Law 321_Corporation LAW_ Case Digest

Petitioner corporation was organized and register under Philippine laws with a capital stock of
P1,000,000.00 divided into 10,000 shares of P100.00 par value each by the herein petitioner Rivera and
four (4) other incorporators. Sometime thereafter petitioner Rivera increased his subscription from the
original 1,250 to a total of 4899 shares.
Subsequently, Isamu Akasako, a Japanese national and co-petitioner who is allegedly the real
owner of the shares of stock in the name of petitioner Aquilino Rivera, sold 2550 shares of the same to
private respondent Milagros Tsuchiya for a consideration of P440,000.00 with the assurance that
Milagros Tsuchiya will be made the President and Lourdes Jureidini a director after the purchase.
Aquilino Rivera who was in Japan also assured private respondents by overseas call that he will sign the
stock certificates because Isamu Akasako is the real owner. However, after the sale was consummated
and the consideration was paid with a receipt of payment therefor shown, Aquilino Rivera refused to
make the indorsement unless he is also paid.

ISSUE:

Whether or not the respondent court of first instance have no jurisdiction over the petition for
mandamus and receivership "as well as in placing the corporate assets under provisional receivership in
the guise of a writ of preliminary mandatory injunction.

RULING:

YES.

It has already been settled that an intracorporate controversy would call for the jurisdiction of the
Securities and Exchange Commission. On the other hand, an intra-corporate controversy has been
defined as "one which arises between a stockholder and the corporate. There is no distinction,
qualification, nor any exemption whatsoever."
This Court has also ruled that cases of private respondents who are not shareholders of the
corporation, cannot be a "controversy arising out of intracorporate or partnership relations between and
among stockholders, members or associates; between any or all of them and the corporation,
partnership or association, of which they are stockholders, members or associates, respectively."

306 | P a g e
Law 321_Corporation LAW_ Case Digest

JOSEFA SANTAMARIA, assisted by her husband, FRANCISCO SANTAMARIA, Jr. vs.


THE HONGKONG AND SHANGHAI BANKING CORPORATION and R. W. TAPLIN.
G.R. No. L-2808 August 31, 1951

FACTS:

Mrs. Josefa T. Santamaria bought 10,000 shares of the Batangas Minerals, Inc., through the
offices of Woo, Uy-Tioco & Naftaly, a stock brokerage firm and pay therefore the sum of P8,041.20 as
shown by receipt Exh. B. The buyer received Stock Certificate No. 517 issued in the name of Woo, Uy-
Tioco & Naftaly and indorsed in bank by this firm.
On March 9, 1937, Mrs. Santamaria placed an order for the purchase of 10,000 shares of the
Crown Mines, Inc. with R.J. Campos & Co., a brokerage firm, and delivered Certificate No. 517 to the
latter as security therefor with the understanding that said certificate would be returned to her upon
payment of the 10,000 Crown Mines, Inc. shares. Exh. D. is the receipt of the certificate in question
signed by one Mr. Cosculluela, Manager of the R.J. Campos & Co., Inc. According to certificate Exh. E,
R. J. Campos & Co., Inc. bought for Mrs. Josefa Santamaria 10,000 shares of the Crown Mines, Inc. at .
225 a share, or the total amount of P2,250. Two days later, on March 11, Mrs. Santamaria went to R.J.
Campos & Co., Inc. to pay for her order of 10,000 Crown Mines shares and to get back Certificate No.
517. Cosculluela then informed her that R.J. Campos & Co., Inc. was no longer allowed to transact
business due to a prohibition order from Securities and Exchange Commission. She was also inform
that her Stock certificate was in the possession of the Hongkong and Shanghai Banking Corporation.

ISSUE:

Whether or not the obligation of the defendant Bank to have inquired into the ownership of the
certificate when it received it from R.J. Campos & Co., Inc. and not conclude that the Bank was
negligent for not having done so, contrary to the claim of the plaintiff that defendant Bank acted
negligently, if not in bad faith, in accepting delivery of said certificate from RJ. Campos & Co., Inc.

RULING:

YES.

Certificate No. 517 came into the possession of the defendant Bank because R.J. Campos & Co.,
Inc. had opened an overdraft account with said Bank and to this effect it had executed on April 16,
1946, a letter of hypothecation by the terms of which R.J. Campos & Co., Inc. pledged to the said Bank
"all Stocks, Shares and Securities which I/we may hereafter come into their possession on my/our
account and whether originally deposited for safe custody only or for any other purpose whatever or
which may hereafter be deposited by me/us in lieu of or in addition to the Stocks, Shares, and
Securities now deposited or for any other purpose whatsoever."
It should be noted that the certificate of stock in question was issued in the name of the
brokerage firm-Woo, Uy-Tioco & Naftaly and that it was duly indorsed in blank by said firm, and that
said indorsement was guaranteed by R.J. Campos & Co., Inc., which in turn indorsed it in blank. This
certificate is what it is known as street certificate. Upon its face, the holder was entitled to demand its
transfer into his name from the issuing corporation. The Bank was not obligated to look beyond the
certificate to ascertain the ownership of the stock at the time it received the same from R.J. Campos &
Co., Inc., for it was given to the Bank pursuant to their letter of hypothecation. Even if said certificate
had been in the name of the plaintiff but indorsed in blank, the Bank would still have been justified in
believing that R.J. Campos & Co., Inc. had title thereto for the reason that it is a well-known practice
that a certificate of stock, indorsed in blank, is deemed quasi negotiable, and as such the transferee
thereof is justified in believing that it belongs to the holder and transferor.
MANUEL A. TORRES, JR., (Deceased), GRACIANO J. TOBIAS, RODOLFO L. JOCSON, JR., MELVIN
S. JURISPRUDENCIA, AUGUSTUS CESAR AZURA and EDGARDO D. PABALAN
vs.
COURT OF APPEALS, SECURITIES AND EXCHANGE COMMISSION, TORMIL REALTY &
DEVELOPMENT CORPORATION, ANTONIO P. TORRES, JR., MA. CRISTINA T. CARLOS, MA. LUISA
T. MORALES and DANTE D. MORALES.
G.R. No. 120138 September 5, 1997

FACTS:

The late Manuel A. Torres, Jr. was the major stockholder of Tormil Realty & Development
Corporation while private respondents who are the children of Judge Torres' deceased brother Antonio

307 | P a g e
Law 321_Corporation LAW_ Case Digest

A. Torres, constituted the minority stockholders. In particular, their respective shareholdings and
positions in the corporation.
In 1984, Judge Torres, in order to make substantial savings in taxes, adopted an "estate
planning" scheme under which he assigned to Tormil Realty & Development Corporation (Tormil for
brevity) various real properties he owned and his shares of stock in other corporations in exchange for
225,972 Tormil Realty shares. Hence, on various dates in July and August of 1984, ten (10) deeds of
assignment were executed by the late Judge Torres.Consequently, the aforelisted properties were duly
recorded in the inventory of assets of Tormil Realty and the revenues generated by the said properties
were correspondingly entered in the corporation's books of account and financial records.
Due to the insufficient number of shares of stock issued to Judge Torres and the alleged refusal
of private respondents to approve the needed increase in the corporation's authorized capital stock (to
cover the shortage of 972 shares due to Judge Torres under the "estate planning" scheme), on 11
September 1986, Judge Torres revoked the two (2) deeds of assignment covering the properties in
Makati and Pasay City.

ISSUE:

Whether or not the deed of assignment executed can be revoked.

RULING:

NO.

The shortage of 972 shares would not be valid ground for respondent Torres to unilaterally revoke
the deeds of assignment he had executed on July 13, 1984 and July 24, 1984 wherein he voluntarily
assigned to TORMIL real properties covered by TCT No. 374079 (Makati) and TCT No. 41527, 41528 and
41529 (Pasay) respectively. A comparison of the number of shares that respondent Torres received from
TORMIL by virtue of the "deeds of assignment" and the stock certificates issued by the latter to the
former readily shows that TORMIL had substantially performed what was expected of it. In fact, the first
two issuances were in satisfaction to the properties being revoked by respondent Torres. Hence, the
shortage of 972 shares would never be a valid ground for the revocation of the deeds covering Pasay and
Quezon City properties.
Moreover, we agree with the contention of the Solicitor General that the shortage of shares should
not have affected the assignment of the Makati and Pasay City properties which were executed in 13 and
24 July 1984 and the consideration for which have been duly paid or fulfilled but should have been
applied logically to the last assignment of property — Judge Torres' Ayala Fund shares — which was
executed on 29 August 1984.

LEE E. WON alias RAMON LEE


vs.
WACK WACK GOLF and COUNTRY CLUB, INC.
G.R. No. L-10122 August 30, 1958

Facts:

The defendant (a non-stock corporation) issued to Iwao Teruyama Membership Certificate No. 201
which was assigned to M. T. Reyes on April 22, 1944. Subsequently in the same year 1944, M. T. Reyes
transferred and assigned said certificate to the plaintiff. On April 26, 1955, the plaintiff filed an action in
the Court of First Instance of Manila against the defendant, alleging that shortly after the rehabilitation
of the defendant after the war, the plaintiff asked the defendant to register in its books the assignment
in favor of the plaintiff and to issue to the latter a new certificate, but that the defendant had refused
and still refuses to do so unlawfully; and praying that the plaintiff be declared the owner of one share of
stock of the defendant and that the latter be ordered to issue a correspondent new certificate. On June
6, 1955, the defendant filed a motion to dismiss, alleging that from 1944, when the plaintiff's right of
action had accrued, to April 26, 1955, when the complaint was filed, eleven years have elapsed, and that
therefore the complaint was filed beyond the 5-year period fixed in Article 1149 of the Civil Code. On
July 30, 1955, the Court of First Instance of Manila issued an order dismissing the complaint. As
plaintiff's motion for reconsideration filed on August 27, 1955 and second motion for reconsideration
filed on September 13, 1955, were both denied, the plaintiff has taken the present appeal.

Issue:

Whether or not the plaintiff is entitled to the registration of the transferred share of stock.

308 | P a g e
Law 321_Corporation LAW_ Case Digest

RULING:

NO.

The certificate in question contains a condition to the effect that no assignment thereof "shall be
effective with respect to the club until such assignment is registered in the books of the club, as
provided in the By-Laws." The decisive question that arises is whether the plaintiff was bound, under
said condition and By-Laws of the defendant or any statutory rule for that matter, to present and
register the certificate assigned to him in 1944 within any definite or fixed period. The defendant has not
made herein any pretense to that effect; but it contends that from the moment the certificate was
assigned to the plaintiff, the latter's right to have the assignment registered commenced to exist. This
contention is correct, but it would not follow that said right should be exercised immediately or within a
definite period. The existence of a right is one thing, and the duration of said right is another.
On the other hand, it is stated in the appealed order of dismissal that the plaintiff sought to
register the assignment on April 13, 1955; whereas in plaintiff's brief it is alleged that it was only in
February, 1955, when the defendant refused to recognize the plaintiff. If, as already observed, there is
no fixed period for registering an assignment, how can the complaint be considered as already barred by
the Statute of Limitations when it was filed on April 26, 1955, or barely a few days (according to the
lower court) and two months (according to the plaintiff), after the demand for registration and its denial
by the defendant. Plaintiff's right was violated only sometime in 1955, and it could not accordingly have
asserted any cause of action against the defendant before that.

 Lost or Destroy Certificates

PHILEX MINING CORPORATION


vs.
HON. DOMINGO CORONEL REYES, Presiding Judge, Court of First Instance of Albay, 10th
Judicial District, Branch IV, and RICHARD HUENEFELD.
G.R. No. L-57707 November 19, 1982

FACTS:

Richard Huenefeld, is a stockholder of petitioner Philex Mining Corporation. He originally owned


800,000 shares of stock. On February 6, 1980, First Asian wrote Huenefeld informing him that the
stock certificate had been delivered to him at his address at Michelle Apartment, 2030 A. Mabini Street,
Manila; and that if the certificate could not be located that Huenefeld execute an Affidavit of Loss, with
the notice of loss to be published once a week for three (3) consecutive weeks in a newspaper of general
circulation in accordance with the procedure prescribed BY Republic Act No. 201.
Huenefeld, through counsel, replied that RA 201 is not applicable because the stock certificate
was not lost in the possession of the stockholder; that assuming it was, the expenses of publication and
premiums for the bond should be at Philex's expense; and demanded the issuance of a replacement
stock certificate. Huenefeld also submitted an Affidavit of Loss but did not comply with the other
requirements on publication.

ISSUE:

Whether or not the Court of First Instance hasve jurisdiction over the present controversy, which
Philex contends is an intra-corporate one.

RULING:

NO.

Evident from the foregoing is that an intra-corporate controversy is one which arises between a
stockholder and the corporation. There is no distinction, qualification, nor any exemption whatsoever.
The provision is broad and covers all kinds of controversies between stockholders and corporations. The
issue of whether or not a corporation is bound to replace a stockholder's lost certificate of stock is a

309 | P a g e
Law 321_Corporation LAW_ Case Digest

matter purely between a stockholder and the corporation. It is a typical intra-corporate dispute. The
question of damages raised is merely incidental to that main issue.
Section 5 of Presidential Decree No. 902-A provides: In addition to the regulatory and adjudicative
functions of the Securities and Exchange Commission over corporations, partnerships and other forms
of associations registered with it as expressly granted under existing laws and decrees; it shall have
original and exclusive jurisdiction to hear and decide cases involving: Controversies arising out of intra-
corporate or partnership relations, between and among stockholders, members, or associates; between
any or all of them and the corporation, partnership or association of which they are stockholders,
members, or associates, respectively and between such corporation, partnership or association and the
state insofar as it concerns their individual franchise or right to exist as such entity.

CORPORATE BOOKS AND RECORDS


 Stock and Transfer Book

NORA A. BITONG
vs.
COURT OF APPEALS (FIFTH DIVISION), EUGENIA D. APOSTOL, JOSE A. APOSTOL, MR. & MS.
PUBLISHING CO., LETTY J. MAGSANOC
G.R. No. 123553 July 13, 1998

FACTS:

Petitioner Alleged before the SEC that she had been the Treasurer and a Member of the Board of
Directors of Mr. & Ms. from the time it was incorporated on 29 October 1976 to 11 April 1989, and was
the registered owner of 1,000 shares of stock out of the 4,088 total outstanding shares, petitioner
complained of irregularities committed from 1983 to 1987 by Eugenia D. Apostol, President and
Chairperson of the Board of Directors. Petitioner claimed that except for the sale of the name Philippine
Inquirer to Philippine Daily Inquirer (PDI hereafter) all other transactions and agreements entered into
by Mr. & Ms. with PDI were not supported by any bond and/or stockholders' resolution. And, upon
instructions of Eugenia D. Apostol, Mr. & Ms. made several cash advances to PDI on various occasions
amounting to P3.276 million. On some of these borrowings PDI paid no interest whatsoever. Despite the
fact that the advances made by Mr. & Ms. to PDI were booked as advances to an affiliate, there existed
no board or stockholders' resolution, contract nor any other document which could legally authorize the
creation of and support to an affiliate.
Petitioner further alleged that respondents Eugenia and Jose Apostol were stockholders, directors
and officers in both Mr. & Ms. and PDI. In fact on 2 May 1986 respondents Eugenia D. Apostol, Leticia
J. Magsanoc and Adoracion G. Nuyda subscribed to PDI shares of stock at P50,000.00 each or a total of
P150,000.00.

ISSUE:

Whether or not the petitioner is the holder of the proper certificates of share of stock.

RULING:

NO.

The certificate of stock itself once issued is a continuing affirmation or representation that the
stock described therein is valid and genuine and is at least prima facie evidence that it was legally
issued in the absence of evidence to the contrary. However, this presumption may be
rebutted. 13 Similarly, books and records of a corporation which include even the stock and transfer
book are generally admissible in evidence in favor of or against the corporation and its members to prove
the corporate acts, its financial status and other matters including one's status as a stockholder. They
are ordinarily the best evidence of corporate acts and proceedings.

310 | P a g e
Law 321_Corporation LAW_ Case Digest

However, the books and records of a corporation are not conclusive even against the corporation
but are prima facie evidence only. Parol evidence may be admitted to supply omissions in the records,
explain ambiguities, or show what transpired where no records were kept, or in some cases where such
records were contradicted. The effect of entries in the books of the corporation which purport to be
regular records of the proceedings of its board of directors or stockholders can be destroyed by
testimony of a more conclusive character than mere suspicion that there was an irregularity in the
manner in which the books were kept.
Thus, while petitioner asserts in her petition that Certificate of Stock No. 008 dated 25 July 1983
was issued in her name, private respondents argue that this certificate was signed by respondent
Eugenia D. Apostol as President only in 1989 and was fraudulently antedated by petitioner who had
possession of the Certificate Book and the Stock and Transfer Book.
 Inspection of Corporate Books and Records

SY TIONG SHIOU, JUANITA TAN SY, JOLIE ROSS TAN, ROMER TAN, CHARLIE TAN, and JESSIE
JAMES TAN
vs.
SY CHIM and FELICIDAD CHAN SY
G.R. No. 174168 March 30, 2009

FACTS:

Four criminal complaints were filed by Spouses Sy against Sy Tiong Shiou, Juanita Tan Sy, Jolie
Ross Tan, Romer Tan, Charlie Tan and Jessie James Tan. Two of the complaints, I.S. Nos. 03E-15285
and 03E-15286, were for alleged violation of Section 74 in relation to Section 144 of the Corporation
Code. In these complaints, the Spouses Sy averred that they are stockholders and directors of Sy Siy Ho
& Sons, Inc. (the corporation) who asked Sy Tiong Shiou, et al., officers of the corporation, to allow them
to inspect the books and records of the business on three occasions to no avail. Sy Tiong Shiou, et al.
denied the request, citing civil and intra-corporate cases pending in court.
In the two other complaints, I.S. No. 03E-15287 and 03E-15288, 6 Sy Tiong Shiou was charged
with falsification under Article 172, in relation to Article 171 of the Revised Penal Code (RPC), and
perjury under Article 183 of the RPC. According to the Spouses Sy, Sy Tiong Shiou executed under oath
the 2003 General Information Sheet (GIS) wherein he falsely stated that the shareholdings of the
Spouses Sy had decreased despite the fact that they had not executed any conveyance of their shares.

ISSUE:

Whether or not Section 74 of the Corporation Code is violated.

RULING:

YES.

In a criminal complaint for violation of Section 74 of the Corporation Code, the defense of
improper use or motive is in the nature of a justifying circumstance that would exonerate those who
raise and are able to prove the same. Accordingly, where the corporation denies inspection on the
ground of improper motive or purpose, the burden of proof is taken from the shareholder and placed on
the corporation. However, where no such improper motive or purpose is alleged, and even though so
alleged, it is not proved by the corporation, and then there is no valid reason to deny the requested
inspection.
In the instant case, however, the Court finds that the denial of inspection was predicated on the
pending civil case against the Spouses Sy.
Even in their Joint Counter-Affidavit dated 23 September 2003, Sy Tiong Shiou, et al. did not
make any allegation that "the person demanding to examine and copy excerpts from the corporation’s
records and minutes has improperly used any information secured through any prior examination of the
records or minutes of such corporation or of any other corporation, or was not acting in good faith or for
a legitimate purpose in making his demand." Instead, they merely reiterated the pendency of the civil
case. There being no allegation of improper motive, and it being undisputed that Sy Tiong Shiou, et al.
denied Sy Chim and Felicidad Chan Sy’s request for inspection, the Court rules and so holds that the
DOJ erred in dismissing the criminal charge for violation of Section 74 in relation to Section 144 of the
Corporation Code.

311 | P a g e
Law 321_Corporation LAW_ Case Digest

VICTOR AFRICA
vs.
PRESIDENTIAL COMMISSION ON GOOD GOVERNMENT, JOSE LAURETA, MELQUIADES
GUTIERREZ, EDUARDO M. VILLANUEVA, EDUARDO DE LOS ANGELES and ROMAN MABANTA, JR.
G.R. No. 83831 January 9, 1992

FACTS:

Victor Africa, who claims to be an employee of ETPI holding the positions of vice-president,
general counsel (on official leave without pay), corporate secretary and special assistant to the chairman
(and president), filed directly with this Court on June 30, 1988 a petition for injunction docketed as G.R.
No. 83831, seeking to enjoin the PCGG and its nominees/designees to the board of directors and the
newly-installed officers of ETPI from implementing their alleged illegal, invalid and immoral act of
ousting him from his offices and positions at the ETPI pending the determination of whether they have
validly, legally and morally assumed their supposed positions and offices as "directors" and/or "officers"
of ETPI.
He contends that the reasons advanced by the PCGG-sponsored board of directors for ousting
him from his offices (redundancy, need to conserve company funds and loss of confidence) are flimsy,
whimsical and arbitrary, evidencing not only the PCGG-sponsored board's discriminatory and
oppressive attitude towards him but, more importantly, its clear intent to harass him into refraining
from questioning before several tribunals all the invalid, illegal and immoral acts of said PCGG-
sponsored board which have caused and are still causing ETPI damages because they constitute
dissipation of assets.

ISSUE:

Whether or not the acts and orders of the PCGG which led to the nomination and election of the
new members of the board of directors and officers of the ETPI correct.

RULING:

YES.

In upholding therein the right of a stockholder of a sequestered company to inspect and/or


examine the records of a corporation pursuant to Section 74 of the Corporation Code, the Court found
nothing in Executive Orders Nos. 1, 2 and 14, as well as in BASECO, to indicate an implied amendment
of the Corporation Code, much less an implied modification of a stockholder's right of inspection as
guaranteed by Section 74 thereof. The only express limitation on the right of inspection, according to the
Court, is that (1) the right of inspection should be exercised at reasonable hours on business days; (2)
the person demanding the right to examine and copy excerpts from the corporate records and minutes
has not improperly used any information secured through any previous examination of the records of
such corporation; and (3) the demand is made in good faith or for a legitimate purpose.

REPUBLIC OF THE PHILIPPINES


vs.
SANDIGANBAYAN (THIRD DIVISION) and MACARIO ASISTIO, JR.
G.R. No. 90529 August 16, 1991

312 | P a g e
Law 321_Corporation LAW_ Case Digest

FACTS:

In a Joint Letter-Complaint to the Ombudsman dated January 8,1989, Messrs. Arnel Blancaflor
and Rodolfo Santos, residents of Kalookan City, charged respondent Macario Asistio, Jr., who is the
incumbent Mayor of Kalookan City, with having violated the Anti-Graft and Corrupt Practices Act (R.A.
3019), specifically Section 8 thereof.
In said Joint Sworn Letter-Complaint, they alleged that during his incumbency as Kalookan City
Mayor in 1981, 1982 and 1983, respondent Asistio acquired wealth in the amounts of P2,142,637.50,
P11,463,734.40 and P3,658,351.00, respectively, or a total of P17,264,722.90, which he deposited in his
personal account, CA-4670-00136-3, in the Republic Planters Bank, Sangandaan Branch, Kalookan
City. In support of their allegations, they attached the original copies of the bank deposits and receipts
which indicated the various sums deposited within the three-year period and which had been machine
validated from January 5, 1981 thru December 20, 1983. However, in his Sworn Statements of Assets
and Liabilities for the period ending December 31,1982 and December 31, 1984, said respondent had a
total income of only P234,128.68 and P255,324.02, respectively; and as against its total assets (real and
personal properties) of P2,859,327.94 as of December 31, 1982, he had loans payable in the amount of
P2,425,575.60, and against total assets of P5,143,260.98 as of December 31, 1984, he had loans
payable in the amount of P2,660,094.74. The Preliminary Investigation was conducted by Special
Prosecution Officer Margarito P. Gervacio, Jr., before whom authenticated xerox copies of the original
ledger cards of CA-4670- 00136-3 in the name of respondent Macario Asistio, Jr. were produced and
presented by the officer-in-charge of the Republic Planters Bank, Sangandaan Branch, Kalookan City.

ISSUE:

Whether or not the Sandiganbayan should be the proper court to try the subject of the case.

RULING:

NO.

A perusal of the law originally creating the Office of the Ombudsman then (to be known as the
Tanodbayan), and the amendatory laws issued subsequent thereto will show that, at its inception, the
Office of the Ombudsman was already vested with the power to investigate and prosecute civil and
criminal cases before the Sandiganbayan and even the regular courts.
Nonetheless, while we do not discount the authority of the Ombudsman, we believe and so hold
that the exercise of his correlative powers to both investigate and initiate the proper action for the
recovery of ill-gotten and/or unexplained wealth is restricted only to cases for the recovery of ill-gotten
and/or unexplained wealth which were amassed after February 25, 1986. 18 Prior to said date, the
Ombudsman is without authority to initiate such forfeiture proceedings. We, however, uphold his
authority to investigate cases for the forfeiture or recovery of such ill-gotten and/or unexplained wealth
amassed even before the aforementioned date, pursuant to his general investigatory power under
Section 15(l) of Republic Act No. 6770.
In the case at bar, the alleged unexplained wealth of respondent Macario Asistio, Jr. was
supposed to have been acquired from 1981 to 1983. Verily, the Ombudsman, like the Special
Prosecutor, is without authority to initiate and file the petition for forfeiture against respondent Asistio.

313 | P a g e
Law 321_Corporation LAW_ Case Digest

JOHN GOKONGWEI, JR.


vs.
SECURITIES AND EXCHANGE COMMISSION, SAN MIGUEL CORPORATION, ANDRES M. SORIANO,
JOSE M. SORIANO, ENRIQUE ZOBEL, ANTONIO ROXAS, EMETERIO BUNAO, WALTHRODE B.
CONDE, MIGUEL ORTIGAS, EMIGDIO TANJUATCO and EDUARDO VISAYA
G.R. No. L-52129 April 21, 1980

FACTS:

Petitioner seeks to nullify and set aside the resolution en banc dated May 7, 1979 of respondent
Securities and Exchange Commission in SEC Case No. 1375, sustaining the findings of the San Miguel
Corporation's Board of Directors that petitioner is engaged in a business competitive with or
antagonistic to that of the San Miguel Corporation and, therefore, ineligible for election as director,
pursuant to Section 3, Article III of the amended by-laws. Petitioner alleges that the matter of
petitioner's disqualification should not have been heard in view of the pendency of petitioner's motion for
reconsideration with this Court; that when respondent Commission sustained the disqualification of
petitioner, it failed to consider that private respondents are precluded from disqualifying petitioner
because of the rule of pari delicto.

ISSUE:

Whether or not respondent SEC gravely abuse its discretion in denying petitioner's request for an
examination of the records of San Miguel International Inc., a fully owned subsidiary of San Miguel
Corporation.

RULING:

YES.

The stockholder's right of inspection of the corporation's books and records is based upon their
ownership of the assets and property of the corporation. It is, therefore, an incident of ownership of the
corporate property, whether this ownership or interest be termed an equitable ownership, a beneficial
ownership, or a ownership. This right is predicated upon the necessity of self-protection. It is generally
held by majority of the courts that where the right is granted by statute to the stockholder, it is given to
him as such and must be exercised by him with respect to his interest as a stockholder and for some
purpose germane thereto or in the interest of the corporation. In other words, the inspection has to be
germane to the petitioner's interest as a stockholder, and has to be proper and lawful in character and
not inimical to the interest of the corporation.
"The right to examine the books of the corporation must be exercised in good faith, for specific
and honest purpose, and not to gratify curiosity, or for specific and honest purpose, and not to gratify
curiosity, or for speculative or vexatious purposes. The weight of judicial opinion appears to be, that on
application for mandamus to enforce the right, it is proper for the court to inquire into and consider the
stockholder's good faith and his purpose and motives in seeking inspection. Thus, it was held that "the
right given by statute is not absolute and may be refused when the information is not sought in good
faith or is used to the detriment of the corporation." But the "impropriety of purpose such as will defeat
enforcement must be set up the corporation defensively if the Court is to take cognizance of it as a
qualification. In other words, the specific provisions take from the stockholder the burden of showing
propriety of purpose and place upon the corporation the burden of showing impropriety of purpose or
motive. It appears to be the general rule that stockholders are entitled to full information as to the
management of the corporation and the manner of expenditure of its funds, and to inspection to obtain
such information, especially where it appears that the company is being mismanaged or that it is being
managed for the personal benefit of officers or directors or certain of the stockholders to the exclusion of
others."
RAMON A. GONZALES
vs.
THE PHILIPPINE NATIONAL BANK
G.R. No. L-33320 May 30, 1983

FACTS:

Petitioner Ramon A. Gonzales instituted in the erstwhile Court of First Instance of Manila a
special civil action for mandamus against the herein respondent praying that the latter be ordered to
allow him to look into the books and records of the respondent bank in order to satisfy himself as to the

314 | P a g e
Law 321_Corporation LAW_ Case Digest

truth of the published reports that the respondent has guaranteed the obligation of Southern Negros
Development Corporation in the purchase of a US$ 23 million sugar-mill to be financed by Japanese
suppliers and financiers; that the respondent is financing the construction of the P 21 million Cebu-
Mactan Bridge to be constructed by V.C. Ponce, Inc., and the construction of Passi Sugar Mill at Iloilo by
the Honiron Philippines, Inc., as well as to inquire into the validity of Id transactions. The petitioner has
alleged hat his written request for such examination was denied by the respondent. The trial court
having dismissed the petition for mandamus, the instant appeal to review the said dismissal was filed.

ISSUE:

Whether or not the denial for the request of petitioner for inspection valid.

RULING:

YES.

Although the petitioner has claimed that he has justifiable motives in seeking the inspection of
the books of the respondent bank, he has not set forth the reasons and the purposes for which he
desires such inspection, except to satisfy himself as to the truth of published reports regarding certain
transactions entered into by the respondent bank and to inquire into their validity. The circumstances
under which he acquired one share of stock in the respondent bank purposely to exercise the right of
inspection do not argue in favor of his good faith and proper motivation. Admittedly he sought to be a
stockholder in order to pry into transactions entered into by the respondent bank even before he became
a stockholder. His obvious purpose was to arm himself with materials which he can use against the
respondent bank for acts done by the latter when the petitioner was a total stranger to the same. He
could have been impelled by a laudable sense of civic consciousness, but it could not be said that his
purpose is germane to his interest as a stockholder.
We also find merit in the contention of the respondent bank that the inspection sought to be
exercised by the petitioner would be violative of the provisions of its charter. The Superintendent of
Banks and the Auditor General, or other officers designated by law to inspect or investigate the
condition of the National Bank, shall not reveal to any person other than the President of the
Philippines, the Secretary of Finance, and the Board of Directors the details of the inspection or
investigation, nor shall they give any information relative to the funds in its custody, its current
accounts or deposits belonging to private individuals, corporations, or any other entity, except by order
of a Court of competent jurisdiction. Any director, officer, employee, or agent of the Bank, who violates
or permits the violation of any of the provisions of this Act, or any person aiding or abetting the
violations of any of the provisions of this Act, shall be punished by a fine not to exceed ten thousand
pesos or by imprisonment of not more than five years, or both such fine and imprisonment.
The Philippine National Bank is not an ordinary corporation. Having a charter of its own, it is not
governed, as a rule, by the Corporation Code of the Philippines. The provision of Section 74 of Batas
Pambansa Blg. 68 of the new Corporation Code with respect to the right of a stockholder to demand an
inspection or examination of the books of the corporation may not be reconciled with the abovequoted
provisions of the charter of the respondent bank.
ANTONIO PARDO
vs.
THE HERCULES LUMBER CO., INC., and IGNACIO FERRER
G.R. No. L-22442 August 1, 1924

FACTS:

The petitioner, Antonio Pardo, a stockholder in the Hercules Lumber Company, Inc., one of the
respondents herein, seeks by this original proceeding in the Supreme Court to obtain a writ
of mandamus to compel the respondents to permit the plaintiff and his duly authorized agent and
representative to examine the records and business transactions of said company. To this petition the
respondents interposed an answer, in which, after admitting certain allegations of the petition, the
respondents set forth the facts upon which they mainly rely as a defense to the petition. To this answer
the petitioner in turn interposed a demurrer, and the cause is now before us for determination of the
issue thus presented.

ISSUE:

Whether or not the respondent have the right to deny inspection request by petitioner.

315 | P a g e
Law 321_Corporation LAW_ Case Digest

RULING:

YES.

The general right given by the statute may not be lawfully abridged to the extent attempted in this
resolution. It may be admitted that the officials in charge of a corporation may deny inspection when
sought at unusual hours or under other improper conditions; but neither the executive officers nor the
board of directors have the power to deprive a stockholder of the right altogether. A by-law unduly
restricting the right of inspection is undoubtedly invalid. Authorities to this effect are too numerous and
direct to require extended comment. Under a statute similar to our own it has been held that the
statutory right of inspection is not affected by the adoption by the board of directors of a resolution
providing for the closing of transfer books thirty days before an election.
It will be noted that our statute declares that the right of inspection can be exercised "at
reasonable hours." This means at reasonable hours on business days throughout the year, and not
merely during some arbitrary period of a few days chosen by the directors.
In addition to relying upon the by-law, to which reference is above made, the answer of the
respondents calls in question the motive which is supposed to prompt the petitioner to make inspection;
and in this connection it is alleged that the information which the petitioner seeks is desired for ulterior
purposes in connection with a competitive firm with which the petitioner is alleged to be connected. It is
also insisted that one of the purposes of the petitioner is to obtain evidence preparatory to the
institution of an action which he means to bring against the corporation by reason of a contract of
employment which once existed between the corporation and himself. These suggestions are entirely
apart from the issue, as, generally speaking, the motive of the shareholder exercising the right is
immaterial.

W. G. PHILPOTTS
vs.
PHILIPPINE MANUFACTURING COMPANY and F. N. BERRY
GR. No. L-15568 November 8, 1919

FACTS:

W. G. Philpotts, a stockholder in the Philippine Manufacturing Company, one of the respondents


herein, seeks by this proceeding to obtain a writ of mandamus to compel the respondents to permit the
plaintiff, in person or by some authorized agent or attorney, to inspect and examine the records of the
business transacted by said company since January 1, 1918. The petition is filed originally in this court
under the authority of section 515 of the Code of Civil Procedure, which gives to this tribunal concurrent
jurisdiction with the Court of First Instance in cases, among others, where any corporation or person
unlawfully excludes the plaintiff from the use and enjoyment of some right to which he is entitled. The
respondents interposed a demurrer, and the controversy is now before us for the determination of the
questions thus presented.

ISSUE:

Whether or not the right to inspect records and transactions of the corporation is permitted.

RULING:

YES.

Now it is our opinion, and we accordingly hold, that the right of inspection given to a stockholder
in the provision above quoted can be exercised either by himself or by any proper representative or
attorney in fact, and either with or without the attendance of the stockholder. This is in conformity with

316 | P a g e
Law 321_Corporation LAW_ Case Digest

the general rule that what a man may do in person he may do through another; and we find nothing in
the statute that would justify us in qualifying the right in the manner suggested by the respondents.
This conclusion is supported by the undoubted weight of authority in the United States, where it
is generally held that the provisions of law conceding the right of inspection to stockholders of
corporations are to be liberally construed and that said right may be exercised through any other
properly authorized person. As was said in Foster vs. White (86 Ala., 467), "The right may be regarded
as personal, in the sense that only a stockholder may enjoy it; but the inspection and examination may
be made by another.
In order that the rule above stated may not be taken in too sweeping a sense, we deem it
advisable to say that there are some things which a corporation may undoubtedly keep secret,
notwithstanding the right of inspection given by law to the stockholder; as for instance, where a
corporation, engaged in the business of manufacture, has acquired a formula or process, not generally
known, which has proved of utility to it in the manufacture of its products. It is not our intention to
declare that the authorities of the corporation, and more particularly the Board of Directors, might not
adopt measures for the protection of such process form publicity. There is, however, nothing in the
petition which would indicate that the petitioner in this case is seeking to discover anything which the
corporation is entitled to keep secret; and if anything of the sort is involved in the case it may be
brought out at a more advanced stage of the proceedings.

317 | P a g e
Law 321_Corporation LAW_ Case Digest

REPUBLIC OF THE PHILIPPINES


vs.
SANDIGANBAYAN (THIRD DIVISION) and MACARIO ASISTIO, JR.
G.R. No. 90529 August 16, 1991

FACTS:

In a Joint Letter-Complaint to the Ombudsman dated January 8,1989 Messrs. Arnel Blancaflor
and Rodolfo Santos, charged respondent Macario Asistio, Jr., who is the incumbent Mayor of Kalookan
City, with having violated the Anti-Graft and Corrupt Practices Act (R.A. 3019), specifically Section 8
thereof. In said Joint Sworn Letter-Complaint, they alleged that during his incumbency as Kalookan
City Mayor in 1981, 1982 and 1983, respondent Asistio acquired wealth in the amounts of
P2,142,637.50, P11,463,734.40 and P3,658,351.00, respectively, or a total of P17,264,722.90, which he
deposited in his personal account, CA-4670-00136-3, in the Republic Planters Bank, Sangandaan
Branch, Kalookan City. In support of their allegations, they attached the original copies of the bank
deposits and receipts which indicated the various sums deposited within the three-year period and
which had been machine validated from January 5, 1981 thru December 20, 1983.
However, in his Sworn Statements of Assets and Liabilities for the period ending December
31,1982 and December 31, 1984 said respondent had a total income of only P234,128.68 and
P255,324.02, respectively; and as against its total assets (real and personal properties) of P2,859,327.94
as of December 31, 1982, he had loans payable in the amount of P2,425,575.60, and against total
assets of P5,143,260.98 as of December 31, 1984, he had loans payable in the amount of
P2,660,094.74.

ISSUE:

Whether or not the sequestration automatically deprive a stockholder of his right of inspection.

RULING:

NO.

The right of a stockholder to inspect and/or examine the records of a corporation is explicitly
provided in Section 74 of the Corporation Code. The records of all business transactions of the
corporation and the minutes of any meeting shall be open to the inspection of any director, trustee,
stockholder or member of the corporation at reasonable hours on business days and he may demand, in
writing, for a copy of excerpts from said records or minutes, at his expense.
One thing is certain, and should be stated at the outset: the PCGG cannot exercise acts of
dominion over property sequestered, frozen or provisionally taken over. As already earlier stressed with
no little insistence, the act of sequestration; freezing or provisional takeover of property does not import
or bring about a divestment of title over said property; does not make the PCGG the owner thereof. In
relation to the property sequestered, frozen or provisionally taken over, the PCGG is a conservator, not
an owner.
The PCGG does not become, ipso facto, the owner of the shares just because the same have been
sequestered; nor does it become the stockholder of record by virtue of such sequestration.

MERGER AND CONSOLIDATION


 Effects of Merger or Consolidation

BANK OF THE PHILIPPINE ISLANDS


vs.
BPI EMPLOYEES UNION-DAVAO CHAPTER-FEDERATION OF UNIONS IN BPI UNIBANK
G.R. No. 164301 August 10, 2010

318 | P a g e
Law 321_Corporation LAW_ Case Digest

FACTS:

The Bangko Sentral ng Pilipinas approved the Articles of Merger executed on January 20, 2000
by and between BPI, herein petitioner, and FEBTC. This Article and Plan of Merger was approved by the
Securities and Exchange Commission on April 7, 2000. Pursuant to the Article and Plan of Merger, all
the assets and liabilities of FEBTC were transferred to and absorbed by BPI as the surviving corporation.
FEBTC employees, including those in its different branches across the country, were hired by petitioner
as its own employees, with their status and tenure recognized and salaries and benefits maintained.
Respondent BPI Employees Union-Davao Chapter - Federation of Unions in BPI Unibank is the exclusive
bargaining agent of BPI’s rank and file employees in Davao City. The former FEBTC rank-and-file
employees in Davao City did not belong to any labor union at the time of the merger. Prior to the
effectivity of the merger, or on March 31, 2000, respondent Union invited said FEBTC employees to a
meeting regarding the Union Shop Clause of the existing CBA between petitioner BPI and respondent
Union.
After the meeting called by the Union, some of the former FEBTC employees joined the Union,
while others refused. Later, however, some of those who initially joined retracted their membership.

ISSUE:

Whether or not the FEBTC employees were automatically absorbed by petitioner upon the merger
between FEBTC and BPI covered by the Union Shop Clause found in the existing CBA between
petitioner and respondent Union.

RULING:

NO.

In legal parlance, however, human beings are never embraced in the term "assets and liabilities."
Moreover, BPI’s absorption of former FEBTC employees was neither by operation of law nor by legal
consequence of contract. There was no government regulation or law that compelled the merger of the
two banks or the absorption of the employees of the dissolved corporation by the surviving corporation.
Had there been such law or regulation, the absorption of employees of the non-surviving entities of the
merger would have been mandatory on the surviving corporation. In the present case, the merger was
voluntarily entered into by both banks presumably for some mutually acceptable consideration. In fact,
the Corporation Code does not also mandate the absorption of the employees of the non-surviving
corporation by the surviving corporation in the case of a merger.
Significantly, too, the Articles of Merger and Plan of Merger dated April 7, 2000 did not contain
any specific stipulation with respect to the employment contracts of existing personnel of the non-
surviving entity which is FEBTC. Unlike the Voluntary Arbitrator, this Court cannot uphold the
reasoning that the general stipulation regarding transfer of FEBTC assets and liabilities to BPI as set
forth in the Articles of Merger necessarily includes the transfer of all FEBTC employees into the employ
of BPI and neither BPI nor the FEBTC employees allegedly could do anything about it. Even if it is so, it
does not follow that the absorbed employees should not be subject to the terms and conditions of
employment obtaining in the surviving corporation.

PHILIPPINE NATIONAL BANK & NATIONAL SUGAR DEVELOPMENT CORPORATION


vs.
ANDRADA ELECTRIC & ENGINEERING COMPANY
G.R. No. 142936 April 17, 2002

FACTS:

The plaintiff alleged that it is a partnership duly organized, existing, and operating under the laws
of the Philippines, with office and principal place of business at Nos. 794-812 Del Monte Avenue,
Quezon City, while the Philippine National Bank (herein referred to as PNB), is a semi-government
corporation duly organized, existing and operating under the laws of the Philippines, with office and
principal place of business at Escolta Street, Sta. Cruz, Manila; whereas, the other defendant, the
National Sugar Development Corporation, is also a semi-government corporation and the sugar arm of
the PNB, with office and principal place of business at the 2nd Floor, Sampaguita Building.
The plaintiff and the defendant PASUMIL entered into a contract.That out of the total obligation of
P777,263.80, the defendant PASUMIL had paid only P250,000.00, leaving an unpaid balance, as of

319 | P a g e
Law 321_Corporation LAW_ Case Digest

June 27, 1973, amounting to P527,263.80, as shown in the Certification of the chief accountant of the
PNB, a machine copy of which is appended as Annex ‘C’ of the complaint; that out of said unpaid
balance of P527,263.80, the defendant PASUMIL made a partial payment to the plaintiff of P14,000.00,
in broken amounts, covering the period from January 5, 1974 up to May 23, 1974, leaving an unpaid
balance of P513,263.80; that the defendant PASUMIL and the defendant PNB, and now the defendant
NASUDECO, failed and refused to pay the plaintiff their just, valid and demandable obligation; that the
President of the NASUDECO is also the Vice-President of the PNB, and this official holds office at the
10th Floor of the PNB, Escolta, Manila, and plaintiff besought this official to pay the outstanding
obligation of the defendant PASUMIL, inasmuch as the defendant PNB and NASUDECO now owned and
possessed the assets of the defendant PASUMIL, and these defendants all benefited from the works, and
the electrical, as well as the engineering and repairs, performed by the plaintiff; that because of the
failure and refusal of the defendants to pay their just, valid, and demandable obligations, plaintiff
suffered actual damages in the total amount of P513,263.80; and that in order to recover these sums,
the plaintiff was compelled to engage the professional services of counsel, to whom the plaintiff agreed to
pay a sum equivalent to 25% of the amount of the obligation due by way of attorney’s fees.

ISSUE:

Whether or not PNB liable for the unpaid debts of PASUMIL to respondent.

RULING:

NO.

A consolidation is the union of two or more existing entities to form a new entity called the
consolidated corporation. A merger, on the other hand, is a union whereby one or more existing
corporations are absorbed by another corporation that survives and continues the combined business.
The merger, however, does not become effective upon the mere agreement of the constituent
corporations. Since a merger or consolidation involves fundamental changes in the corporation, as well
as in the rights of stockholders and creditors, there must be an express provision of law authorizing
them. For a valid merger or consolidation, the approval by the Securities and Exchange Commission
(SEC) of the articles of merger or consolidation is required. These articles must likewise be duly
approved by a majority of the respective stockholders of the constituent corporations.
In the case at bar, we hold that there is no merger or consolidation with respect to PASUMIL and
PNB. The procedure prescribed under Title IX of the Corporation Code was not followed.
CHESTER BABST
vs.
COURT OF APPEALS, BANK OF THE PHILIPPINE ISLANDS, ELIZALDE STEEL CONSOLIDATED,
INC., and PACIFIC MULTI-COMMERCIAL CORPORATION
G.R. No. 99398 & 104625 January 26, 2001

FACTS:

The complaint was commenced principally to enforce payment of a promissory note and three
domestic letters of credit which Elizalde Steel Consolidated, Inc. (ELISCON) executed and opened with
the Commercial Bank and Trust Company (CBTC). On June 8, 1973, ELISCON obtained from CBTC a
loan in the amount of P 8,015,900.84, with interest at the rate of 14% per annum, evidenced by a
promissory note.2 ELISCON defaulted in its payments, leaving an outstanding indebtedness in the
amount of P2,795,240.67 as of October 31, 1982.
The letters of credit, on the other hand, were opened for ELISCON by CBTC using the credit
facilities of Pacific Multi-Commercial Corporation (MULTI) with the said bank, pursuant to the
Resolution of the Board of Directors of MULTI adopted on August 31, 1977. Subsequently, on September
26, 1978, Antonio Roxas Chua and Chester G. Babst executed a Continuing Suretyship, 5 whereby they
bound themselves jointly and severally liable to pay any existing indebtedness of MULTI to CBTC to the
extent of P8,000,000.00 each.

ISSUE:

Whether or not BPI consent to the assumption by DBP of the obligations of ELISCON.

RULING:

YES.

320 | P a g e
Law 321_Corporation LAW_ Case Digest

Indeed, there exist clear indications that BPI was aware of the assumption by DBP of the
obligations of ELISCON. In fact, BPI admits that the Development Bank of the Philippines (DBP), for a
time, had .proposed a formula for the settlement of Eliscon's past obligations to its creditors, including
the plaintiff [BPI], but the formula was expressly rejected by the plaintiff as not acceptable (long before
the filing of the complaint at bar).
Indeed, the authority granted by BPI to its account officer to attend the creditors' meeting was an
authority to represent the bank, such that when he failed to object to the substitution of debtors, he did
so on behalf of and for the bank. Even granting arguendo that the said account officer was not so
empowered, BPI could have subsequently registered its objection to the substitution, especially after it
had already learned that DBP had taken over the assets and assumed the liabilities of ELISCON. Its
failure to do so can only mean an acquiescence in the assumption by DBP of ELISCON's obligations. As
repeatedly pointed out by ELISCON and MULTI, BPI's objection was to the proposed payment formula,
not to the substitution itself.
BPI gives no cogent reason in withholding its consent to the substitution, other than its desire to
preserve its causes of action and legal recourse against the sureties of ELISCON. It must be
remembered, however, that while a surety is solidarily liable with the principal debtor, his obligation to
pay only arises upon the principal debtor's failure or refusal to pay.
In the case at bar, there was no indication that the principal debtor will default in payment. In
fact, DBP, which had stepped into the shoes of ELISCON, was capable of payment. Its authorized capital
stock was increased by the government. More importantly, the National Development Company took
over the business of ELISCON and undertook to pay ELISCON's creditors, and earmarked for that
purpose the amount of P4,015,534.54 for payment to BPI. The original obligation having been
extinguished, the contracts of suretyship executed separately by Babst and MULTI, being accessory
obligations, are likewise extinguished.

ASSOCIATED BANK
vs.
HON. COURT OF APPEALS, PROVINCE OF TARLAC and PHILIPPINE NATIONAL BANK
G.R. No. 107382/G.R. No. 107612 January 31, 1996

FACTS:

The Province of Tarlac maintains a current account with the Philippine National Bank (PNB)
Tarlac Branch where the provincial funds are deposited. Checks issued by the Province are signed by
the Provincial Treasurer and countersigned by the Provincial Auditor or the Secretary of the
Sangguniang Bayan. A portion of the funds of the province is allocated to the Concepcion Emergency
Hospital. The allotment checks for said government hospital are drawn to the order of "Concepcion
Emergency Hospital, Concepcion, Tarlac" or "The Chief, Concepcion Emergency Hospital, Concepcion,
Tarlac." The checks are released by the Office of the Provincial Treasurer and received for the hospital by
its administrative officer and cashier.
In January 1981, the books of account of the Provincial Treasurer were post-audited by the
Provincial Auditor. It was then discovered that the hospital did not receive several allotment checks
drawn by the Province. On February 19, 1981, the Provincial Treasurer requested the manager of the
PNB to return all of its cleared checks which were issued from 1977 to 1980 in order to verify the
regularity of their encashment. After the checks were examined, the Provincial Treasurer learned that 30
checks amounting to P203,300.00 were encashed by one Fausto Pangilinan, with the Associated Bank
acting as collecting bank.
It turned out that Fausto Pangilinan, who was the administrative officer and cashier of payee
hospital until his retirement on February 28, 1978, collected the questioned checks from the office of the
Provincial Treasurer. He claimed to be assisting or helping the hospital follow up the release of the
checks and had official receipts. Pangilinan sought to encash the first check with Associated Bank.

ISSUE:

Whether or not there is merger in this case.

RULING:

YES.

Ordinarily, in the merger of two or more existing corporations, one of the combining corporations
survives and continues the combined business, while the rest are dissolved and all their rights,

321 | P a g e
Law 321_Corporation LAW_ Case Digest

properties and liabilities are acquired by the surviving corporation. Although there is dissolution of the
absorbed corporations, there is no winding up of their affairs or liquidation of their assets, because the
surviving corporation automatically acquires all their rights, privileges and powers, as well as their
liabilities.
The merger, however, does not become effective upon the mere agreement of the constituent
corporations. The procedure to be followed is prescribed under the Corporation Code. Section 79 of said
Code requires the approval by the Securities and Exchange Commission (SEC) of the articles of merger
which, in turn, must have been duly approved by a majority of the respective stockholders of the
constituent corporations. The effectivity date of the merger is crucial for determining when the merged
or absorbed corporation ceases to exist; and when its rights, privileges, properties as well as liabilities
pass on to the surviving corporation.
Consistent with the aforementioned Section 79, the September 16, 1975 Agreement of Merger,
which Associated Banking Corporation (ABC) and Citizens Bank and Trust Company (CBTC) entered
into, provided that its effectivity "shall, for all intents and purposes, be the date when the necessary
papers to carry out this merger shall have been approved by the Securities and Exchange Commission."

ALGER ELECTRIC, INC


vs.
COURT OF APPEALS and NORTHERN CEMENT CORPORATION
G.R. No. L-34298 February 28, 1985

FACTS:

Petitioner Alger Electric, Inc., was granted a legislative franchise for a period of fifty (50) years
from June 22, 1963 with the right, privilege, and authority to construct, maintain and operate an
electric light, heat, and power system for the generation and/or distribution of electric light, heat,
and/or power for sale within the municipalities of Sto. Tomas, Damortis and Rosario, province of La
Union, and in the municipality of Sison, province of Pangasinan.
Respondent Northern Cement Corporation (Northern) and the National Power Corporation (NPC)
executed a contract for NPC to directly supply electric power to Northern's cement plant located in
Labayog, Sison, Pangasinan. As a result, the petitioner filed a petition for prohibition with preliminary
injunction against Northern and NPC in the Court of First Instance of Manila. The petition alleged that
the contract was patently illegal.
The appellate court sustained the position of respondent Northern and set aside the questioned
October 24, 1969 order of the trial court. It also ordered the trial court to act on the respondent's motion
to dismiss the case. The appellate court ruled that the Court of First Instance of Manila did not have
jurisdiction over the original complaint considering that the act sought to be enjoined was to be
performed in Sison, Pangasinan which is outside of the court's territorial jurisdiction. It, therefore, held
that the original "petition" could no longer be amended otherwise it would be in violation of the legal
prohibition of a complaint not amendable in order to confer jurisdiction on the court in which it is filed,
if the cause of action originally set forth was not within The court's jurisdiction. This decision is now
challenged in this petition.

ISSUE:

Whether or not Northern commit illegal acts when it entered into a contract with NPC.

RULING:

NO.

We have interpreted monopolistic claims of corporations, which want to protect themselves


through the exclusion of competitors and antagonistic parties, as necessarily yielding to the higher
claims of public interest. This interpretation is even more called for when the exclusiveness is claimed
on the basis of a public franchise.
Section 2 of Republic Act No. 3826 was obviously enacted to prevent the NPC from distributing or
selling electric power where petitioner Alger is already selling or is able to sell its own self-generated
electricity. In this case, Northern is a bulk purchaser of power. It had never purchase's Alger's electricity
before the suit was filed. It is not the usual consumer — residential or commercial — for whom retail
sales are Ideal. Exclusivity is given by law with the understanding that the company enjoying it is self-
sufficient and capable of supplying the needed service or product at moderate or reasonable prices. It
would be against public interest where the firm granted a monopoly is merely ail unnecessary conduit of

322 | P a g e
Law 321_Corporation LAW_ Case Digest

electric power, jacking up prices as a superfluous middleman or an inefficient producer which cannot
supply cheap electricity to power intensive industries. It is in the public interest when industries
dependent on the heavy use of electricity are given reliable and direct power at the lowest costs thus
enabling the sale of nationally marketed products at prices within the reach of the masses. Applying the
above principles to the specific facts of this case, Northern cannot be said to have committed an act void
ab initio when it concluded the questioned contract with NPC. Accordingly, the respondent corporation
is not liable for damages to the petitioner.
COMMISSIONER OF INTERNAL REVENUE
vs.
NORTON and HARRISON COMPANY
G.R. No. L-17618 August 31, 1964

FACTS:

Norton and Harrison is a corporation organized in 1911, (1) to buy and sell at wholesale and
retail, all kinds of goods, wares, and merchandise; (2) to act as agents of manufacturers in the United
States and foreign countries; and (3) to carry on and conduct a general wholesale and retail mercantile
establishment in the Philippines. Jackbilt is, likewise, a corporation organized on February 16, 1948
primarily for the purpose of making, producing and manufacturing concrete blocks. Under date of July
27, 1948. Norton and Jackbilt entered into an agreement whereby Norton was made the sole and
exclusive distributor of concrete blocks manufactured by Jackbilt. Pursuant to this agreement,
whenever an order for concrete blocks was received by the Norton & Harrison Co. from a customer, the
order was transmitted to Jackbilt which delivered the merchandise direct to the customer. Payment for
the goods is, however, made to Norton, which in turn pays Jackbilt the amount charged the customer
less a certain amount, as its compensation or profit. To exemplify the sales procedures adopted by the
Norton and Jackbilt, the following may be cited. In the case of the sale of 420 pieces of concrete blocks
to the American Builders on April 1, 1952, the purchaser paid to Norton the sum of P189.00 the
purchase price. Out of this amount Norton paid Jackbilt P168.00, the difference obviously being its
compensation. As per records of Jackbilt, the transaction was considered a sale to Norton. It was under
this procedure that the sale of concrete blocks manufactured by Jackbilt was conducted until May 1,
1953, when the agency agreement was terminated and a management agreement between the parties
was entered into.

ISSUE:

Whether or not the acquisition of all the stocks of the Jackbilt by the Norton & Harrison Co.,
merged the two corporations into a single corporation.

RULING:

YES.

It has been settled that the ownership of all the stocks of a corporation by another corporation
does not necessarily breed an identity of corporate interest between the two companies and be
considered as a sufficient ground for disregarding the distinct personalities. However, in the case at bar,
we find sufficient grounds to support the theory that the separate identities of the two companies should
be disregarded. There was no limit to the advances given to Jackbilt. The income tax return of Norton for
1954 shows that as President and Treasurer of Norton and Jackbilt, he received from Norton
P56,929.95, but received from Jackbilt the measly amount of P150.00, a circumstance which points out
that remuneration of purported officials of Jackbilt are deemed included in the salaries they received
from Norton. The same is true in the case of Eduardo Garcia, an employee of Norton but a member of
the Board of Jackbilt. His Income tax return for 1956 reveals that he received from Norton in salaries
and bonuses P4,220.00, but received from Jackbilt, by way of entertainment, representation, travelling
and transportation allowances P3,000.00. However, in the withholding statement, it was shown that the
total of P4,200.00 and P3,000.00 was received by Garcia from Norton, thus portraying the oneness of
the two companies. The Income Tax Returns of Albert Golden and Dioscoro Ramos both employees of
Norton but board members of Jackbilt, also disclose the game method of payment of compensation and
allowances. The offices of Norton and Jackbilt are located in the same compound. Payments were
effected by Norton of accounts for Jackbilt and vice versa. Payments were also made to Norton of
accounts due or payable to Jackbilt and vice versa.

COMMISSIONER OF INTERNAL REVENUE


vs.

323 | P a g e
Law 321_Corporation LAW_ Case Digest

VICENTE A. RUFINO, ET AL.


G.R. NOS. L-33665-68 FEBRUARY 27, 1987

FACTS:

The private respondents were the majority stockholders of the Eastern Theatrical Co., Inc., a
corporation organized for a period of twenty-five years terminating in 1959. It was organized to engage in
the business of operating theaters, opera houses, places of amusement and other related business
enterprises, more particularly the Lyric and Capitol Theaters in Manila. The President of this corporation
during the year in question was Ernesto D. Rufino.
The private respondents are also the majority and controlling stockholders of another
corporation, the Eastern Theatrical Co Inc., which was organized in 1958, for a term of 50 years. This
corporation is engaged in the same kind of business as the Old Corporation. The General-Manager of
this corporation (hereinafter referred to as the New Corporation) at the time was Vicente A. Rufino.
In a special meeting of stockholders of the Old Corporation it provided for the continuation of its
business after the end of its corporate life, and upon the recommendation of its board of directors, a
resolution was passed authorizing the Old Corporation to merge with the New Corporation by
transferring its business, assets, goodwill, and liabilities to the latter, which in exchange would issue
and distribute to the shareholders of the Old Corporation one share for each share held by them in the
said Corporation.
It was expressly declared that the merger of the Old Corporation with the New Corporation was
necessary to continue the exhibition of moving pictures at the Lyric and Capitol Theaters even after the
expiration of the corporate existence of the former.

ISSUE:

Whether or not the merger of the two corporations is valid.

RULING:

YES.

The term "merger" or "consolidation," shall be understood to mean: (1) The ordinary merger or
consolidation, or (2) the acquisition by one corporation of all or substantially all the properties of
another corporation solely for stock, provided, that for a transaction to be regarded as a merger or
consolidation, it must be undertaken for a bona fide business purpose and not solely for the purpose of
escaping the burden of taxation, provided further, that in determining whether a bona fide business
purpose exists, each and every step of the transaction shall be considered and the whole transaction or
series of transactions shall be treated as a single unit.
No taxable gain was derived by the private respondents from the questioned transaction. Contrary
to the claim of the petitioner, there was a valid merger although the actual transfer of the properties
subject of the Deed of Assignment was not made on the date of the merger. In the nature of things, this
was not possible. Obviously, it was necessary for the Old Corporation to surrender its net assets first to
the New Corporation before the latter could issue its own stock to the shareholders of the Old
Corporation because the New Corporation had to increase its capitalization for this purpose.
There is no impediment to the exchange of property for stock between the two corporations being
considered to have been effected on the date of the merger. That, in fact, was the intention, and the
reason why the Deed of Assignment was made retroactive to January 1, 1959. Such retroaction provided
in effect that all transactions set forth in the merger agreement shall be deemed to be taking place
simultaneously on January 1, 1959, when the Deed of Assignment became operative.

SOLID MANILA CORPORATION


vs.
BIO HONG TRADING., INC. AND COURT OF APPEALS
G.R. NO. 90596 APRIL 8, 1991

FACTS:

The petitioner is the owner of a parcel of land. The same lies in the vicinity of another parcel,
registered in the name of the private respondent corporation. The private respondent's title came from a
prior owner, and in their deed of sale, the parties thereto reserved as an easement of way. As a
consequence, an annotation was entered in the private respondent's title.

324 | P a g e
Law 321_Corporation LAW_ Case Digest

The petitioner claims that ever since, it had (as well as other residents of neighboring estates)
made use of the above private alley and maintained and contributed to its upkeep, until sometime in
1983, when, and over its protests, the private respondent constructed steel gates that precluded
unhampered use. On December 6, 1984, the petitioner commenced suit for injunction against the
private respondent, to have the gates removed and to allow full access to the easement.

ISSUE:

Whether or not the court a quo errs in holding that an easement had been extinguished by
merger.

RULING:

NO.

There is no question that an easement, as described in the deed of sale executed between the
private respondent and the seller, had been constituted on the private respondent's property, and has
been annotated. Specifically, the same charged the private respondent as follows, that the alley shall
remain open at all times, and no obstructions whatsoever shall be placed thereon, that the owner of the
lot on which the alley has been constructed shall allow the public to use the same, and allow the City to
lay pipes for sewer and drainage purposes, and shall not ask for any indemnity for the use thereof. Its
act, therefore, of erecting steel gates across the alley was in defiance of these conditions and a violation
of the deed of sale, and, of course, the servitude of way.
No genuine merger took place as a consequence of the sale in favor of the private respondent
corporation. According to the Civil Code, a merger exists when ownership of the dominant and servient
estates is consolidated in the same person. Merger then, as can be seen, requires full ownership of both
estates.
However, the servitude in question is a personal servitude, that is to say, one constituted not in
favor of a particular tenement (a real servitude) but rather, for the benefit of the general public. In a
personal servitude, there is therefore no "owner of a dominant tenement" to speak of, and the easement
pertains to persons without a dominant estate, in this case, the public at large.
Merger presupposes the existence of a prior servient-dominant owner relationship, and the
termination of that relation leaves the easement of no use. Unless the owner conveys the property in
favor of the public, if that is possible, hence, no genuine merger can take place that would terminate a
personal easement. In the case at bar, the defense of merger is, clearly, not a valid defense, indeed, a
sham one, because merger is not possible, and secondly, the sale unequivocally preserved the existing
easement. In other words, the answer does not, in reality, tender any genuine issue on a material fact
and cannot militate against the petitioner's clear cause of action.

NON-STOCK CORPORATIONS
 Purposes

CHINESE YOUNG MEN’S CHRISTIAN ASSOCIATION OF THE PHILIPPINE ISLANDS, ET AL.


vs.
VICTOR CHING AND COURT OF APPEALS
G.R. NO. L-36929 JUNE 18, 1976

FACTS:

Respondent Victor Ching filed an action for mandamus with preliminary injunction against the
herein petitioners. He anchored his action upon the claim that the Membership Campaign of the
Chinese YMCA for 1966 held, only 175 applications for membership were submitted, canvassed and
accepted on the last day of the membership campaign. Not more than 240 membership applications, as
reported, issue of the Chinese Commercial News, were filed.
It is to be noted that respondent Victor Ching is a member of the Board of Directors of the
Chinese YMCA, while herein petitioners, William Golangco and Juanito K. Tan, are its president and
recording secretary, respectively. In the campaign for membership for the year 1966, a rivalry had

325 | P a g e
Law 321_Corporation LAW_ Case Digest

developed between two groups in the association, one headed by respondent Ching and the other by
petitioner Golangco.
On the last day of the membership campaign, respondent Ching and herein petitioner Golangco
were in the office of the Chinese YMCA. Respondent Ching, after it was agreed upon that there was going
to be no extension of the membership campaign. After trial, a decision was rendered annulling the 1966
annual membership campaign of the respondent. On appeal, the appealed decision was affirmed.

ISSUE:

Whether or not the membership campaign is valid.

RULING:

YES.

175 membership applications were undisputedly filed within the deadline (including the 75
withdrawn by respondent) and yet the 100 remaining unquestioned memberships were nullified by the
questioned decision without the individuals concerned ever having been impleaded or heard (except the
individual petitioners president and secretary).
The appealed decision thus contravened the established principle that the courts cannot strip a
member of a non-stock non-profit corporation of his membership therein without cause. Otherwise, that
would be an unwarranted and undue interference with the well-established right of a corporation to
determine its membership. In order that membership may be acquired in a non-stock corporation and
valid by-laws must be complied with, except in so far as they may be and are waived. But provisions in
the by-laws as to formal steps to be taken to acquire membership may be waived by the corporation, or
it may be estopped to assert that they have not been taken.
Finally, the appealed decision did not give due importance to the undisputed fact therein stated
that "at the board meeting of the association held on December 7, 1965, a list of 174 applications for
membership, old and new, was submitted to the board and approved by the latter, over the objection of
the petitioner who was present at said meeting." Such action of the petitioner association's board of
directors approving the 174 membership applications of old and new members constituting its active
membership as duly processed and screened by the authorized committee just be deemed a waiver on
its part of any technicality or requirement of form.

326 | P a g e
Law 321_Corporation LAW_ Case Digest

THE COLLECTOR OF INTERNAL REVENUE


vs.
THE CLUB FILIPINO, INC. DE CEBU
G.R. No. L-12719 May 31, 1962

FACTS:

The "Club Filipino, Inc. de Cebu," is a civic corporation organized under the laws of the
Philippines with an original authorized capital stock of P22,000.00, which was subsequently increased
to P200,000.00. Neither in the articles or by-laws is there a provision relative to dividends and their
distribution, although it is covenanted that upon its dissolution, the Club's remaining assets, after
paying debts, shall be donated to a charitable Philippine Institution in Cebu.
The Club owns and operates a club house, a bowling alley, a golf course (on a lot leased from the
government), and a bar-restaurant where it sells wines and liquors, soft drinks, meals and short orders
to its members and their guests. The bar-restaurant was a necessary incident to the operation of the
club and its golf-course. The club is operated mainly with funds derived from membership fees and
dues. Whatever profits it had, were used to defray its overhead expenses and to improve its golf-course.
In 1951. as a result of a capital surplus, arising from the re-valuation of its real properties, the value or
price of which increased, the Club declared stock dividends; but no actual cash dividends were
distributed to the stockholders. In 1952, a BIR agent discovered that the Club has never paid percentage
tax on the gross receipts of its bar and restaurant, although it secured B-4, B-9(a) and B-7 licenses. In a
letter dated December 22, 1852, the Collector of Internal Revenue assessed against and demanded from
the Club certain amount of tax. The Club wrote the Collector, requesting for the cancellation of the
assessment. The request having been denied, the Club filed the instant petition for review.

ISSUE:

Whether or not the respondent Club liable for the payment of the sum of 12,068.84, as fixed and
percentage taxes and surcharges prescribed in sections 182, 183 and 191 of the Tax Code, under which
the assessment was made, in connection with the operation of its bar and restaurant.

RULING:

NO.

Having found as a fact that the Club was organized to develop and cultivate sports of all class and
denomination, for the healthful recreation and entertainment of its stockholders and members; that
upon its dissolution, its remaining assets, after paying debts, shall be donated to a charitable Philippine
Institution in Cebu; that it is operated mainly with funds derived from membership fees and dues; that
the Club's bar and restaurant catered only to its members and their guests; that there was in fact no
cash dividend distribution to its stockholders and that whatever was derived on retail from its bar and
restaurant was used to defray its overall overhead expenses and to improve its golf-course (cost-plus-
expenses-basis), it stands to reason that the Club is not engaged in the business of an operator of bar
and restaurant.
The facts that the capital stock of the respondent Club is divided into shares, does not detract
from the finding of the trial court that it is not engaged in the business of operator of bar and
restaurant. What is determinative of whether or not the Club is engaged in such business is its object or
purpose, as stated in its articles and by-laws. It is a familiar rule that the actual purpose is not
controlled by the corporate form or by the commercial aspect of the business prosecuted, but may be
shown by extrinsic evidence, including the by-laws and the method of operation.

 Voting

ANTONIO LITONJUA and ARNOLD LITONJUA


vs.
THE HON. COURT OF APPEALS, ET. AL.
G.R. No. 120294 February 10, 1998

FACTS:

327 | P a g e
Law 321_Corporation LAW_ Case Digest

Wack Wack Golf and Country Club is a non-profit corporation which offers sports, recreational
and social activities to its members. Petitioner Antonio Litonjua is an Associate Member of said
corporation and his son, co-petitioner Arnold Litonjua, is a Junior Member thereof. The individual
respondents are the members of the Board of Directors and Membership Committee of Wack Wack. On
10 January 1985, pursuant to its by-laws, respondent club posted the monthly list of delinquent
members on its premises. Included therein was petitioner Antonio Litonjua.
After Antonio Litonjua discovered that his name was on the January 1985 delinquent list, he
proceeded to the Cashier's Office of the club and was informed therein that the reason behind his
delinquency was his failure to pay his November 1984 dues (which should have been paid before the
end of December 1984 as provided in the corporate by-laws). Antonio Litonjua alleged that he was not
able to pay his monthly bill on time because he has not received his statement of account for November
1984. As proof, he presented a sealed envelope which he allegedly presumed to be the November 1984
bill (but was actually the December 1984 statement of account) and explained that he received it only on
12 January 1985.
A check with the accounting office, however, revealed that the November 1984 statement of
account had already been delivered to Antonio Litonjua's office and was received by his employee
allegedly named "Aquino." Petitioner asserted that, he did not receive said account and had no employee
by the name of "Aquino." Based on the foregoing, Antonio Litonjua was able to convince the auxiliary
clerks in the Cashier's Office to delete his name from the list of delinquent members. Consequently,
Antonio Litonjua continued to avail of the club facilities. Later, Antonio Litonjua was advised of another
outstanding balance in the amount of P9,414.00. Again, he issued a check in payment thereof. As a
result, his name was deleted from the February 1985 list of delinquent members.

ISSUE:

Whether or not the statement of account for November 1984 was duly delivered to and received
by Antonio Litonjua's office on 12 December 1984.

RULING:

NO.

According to Mr. Limbo's testimony on record, the Court failed to find therein any statement that
he delivered the November 1984 account to Antonio Litonjua himself. Mr. Limbo was consistent in his
testimony to the effect that on 12 December 1984 he delivered the November 1984 statement of account
at the office of Antonio Litonjua and it was received by an employee of the latter who signed the Special
Delivery Receipt. On cross-examination, Mr. Limbo did not waver from his testimony that Antonio
Litonjua's November 1984 bill was duly received by the latter's employee.Against the testimony of Mr.
Victor Limbo, coupled with documentary evidence in the form of the signed Special Delivery Receipt,
petitioners presented no proof other than the bare denial of Antonio Litonjua that he never received his
statement of account for November 1984 and that he has no "Aquino" in his employ. Petitioners could
have readily offered in evidence a record or list of Antonio Litonjua's employees to prove that he has no
employee by the name of "Aquino" but, strangely, beyond his mere say-so no such evidence was
adduced.

328 | P a g e
Law 321_Corporation LAW_ Case Digest

THE PHILIPPINE PUBLIC SCHOOL TEACHERS ASSOCIATION (PPSTA) COMMISSION ON


ELECTIONS
vs.
Honorable SERGIO A. F. APOSTOL
G.R. No. L-36966 February 28, 1974

FACTS:

On July 20, 1972, private respondent Eufemia M. San Luis as a member of the Philippine Public
School Teachers Association (PPSTA), a fraternal non-stock association of public school teachers
throughout the country, filed with respondent court of first instance at Quezon City a complaint with
preliminary injunction for the annulment of the 1972 annual elections of the PPSTA board of directors
held on June 26-28, 1972 at Teachers Camp in Baguio City for having been held outside its principal
office at Quezon City against herein petitioners as defendants.

ISSUE:

Whether or not the elections of the Board of Directors are null and void.

RULING:

NO.

The Court finds it unnecessary to rule upon the parties' above conflicting contentions, since it
finds to be decisive petitioners' contention that respondent has no personality and standing as a single
individual member out of thousands of members of the PPSTA to bring the action below for annulment
of the PPSTA 1972 annual convention and elections, as she was not even a chapter delegate to the said
convention and she was duly represented thereat in accordance with the PPSTA's by-laws by her duly
authorized chapter delegates who have raised no question as to the proceedings.Article IX, section 5 of
the by-laws expressly provides that "only official delegates to the representative assembly are entitled to
take part in the discussions and to vote."
Respondent's action below was in essence one of quo warranto which is governed by Rule 66 of
the Rules of Court Section 6 thereof provides that in order that an individual may directly bring the
action, he or she must claim to entitled to the public office or position allegedly unlawfully held or
usurped.Otherwise, the action must be brought by the Solicitor General or fiscal with leave of the court
upon the complaint of the realtor under section 4 of the Rule.
The general rule is that actions for quo warranto should be brought by the Solicitor General or a
fiscal in cases of usurpation of an office established by law or by the Constitution under color of an
executive appointment, or the abuse of a public franchise under color of a legislative grant, for these are
public wrongs and not private injuries. Since, under our system all power emanates from the people,
who constitute the sovereignty, the right to inquire into the authority by which a person assumes to
exercise the functions of a public office or franchise is regarded as inherent in the people on the right
their sovereignty. Hence, the action should be brought by the Solicitor General or the fiscal who
represents the sovereign power.
Respondent manifestly lays no claim herself to the office of PPSTA director nor has the present
action been filed with leave of court by the Solicitor General or fiscal upon her relation as a party having
an interest injuriously affected, as required by the cited Rule. Her action must therefore fail on this
score and the judgment erroneously rendered by respondent court shall be set aside.

CLOSE CORPORATIONS
 Requirements for Formation

MANUEL R. DULAY ENTERPRISES, INC


vs.
THE HONORABLE COURT OF APPEALS
G.R. No. 91889 August 27, 1993

329 | P a g e
Law 321_Corporation LAW_ Case Digest

FACTS:

Manuel R. Dulay Enterprises, Inc, a domestic corporation with the following as members of its
Board of Directors: Manuel R. Dulay with 19,960 shares and designated as president, treasurer and
general manager, Atty. Virgilio E. Dulay with 10 shares and designated as vice-president; Linda E.
Dulay with 10 shares; Celia Dulay-Mendoza with 10 shares; and Atty. Plaridel C. Jose with 10 shares
and designated as secretary, owned a property covered by TCT No. 17880 and known as Dulay
Apartment consisting of sixteen (16) apartment units on a six hundred eighty-nine (689) square meters
lot, more or less, located at Seventh Street (now Buendia Extension) and F.B. Harrison Street, Pasay
City.
Petitioner corporation through its president, Manuel Dulay, obtained various loans for the
construction of its hotel project, Dulay Continental Hotel (now Frederick Hotel). It even had to borrow
money from petitioner Virgilio Dulay to be able to continue the hotel project. As a result of said loan,
petitioner Virgilio Dulay occupied one of the unit apartments of the subject property since property since
1973 while at the same time managing the Dulay Apartment at his shareholdings in the corporation was
subsequently increased by his father.
Manuel Dulay by virtue of Board Resolution petitioner corporation sold the subject property to
private respondents spouses Maria Theresa and Castrense Veloso in the amount of P300,000.00 as
evidenced by the Deed of Absolute Sale.Subsequently, private respondent Maria Veloso, without the
knowledge of Manuel Dulay, mortgaged the subject property to private respondent Manuel A. Torres for
a loan of P250,000.00 which was duly annotated.Upon the failure of private respondent Maria Veloso to
pay private respondent Torres, the subject property was sold on April 5, 1978 to private respondent
Torres as the highest bidder in an extrajudicial foreclosure sale as evidenced by the Certificate of
Sheriff's Sale issued on April 20, 1978.

ISSUE:

Whether or not the doctrine of piercing the veil of corporate entity is applicable.

RULING:

NO.

Petitioner Corporation is classified as a close corporation and consequently a board resolution


authorizing the sale or mortgage of the subject property is not necessary to bind the corporation for the
action of its president. At any rate, corporate action taken at a board meeting without proper call or
notice in a close corporation is deemed ratified by the absent director unless the latter promptly files his
written objection with the secretary of the corporation after having knowledge of the meeting which, in
his case, petitioner Virgilio Dulay failed to do.
It is relevant to note that although a corporation is an entity which has a personality distinct and
separate from its individual stockholders or members,the veil of corporate fiction may be pierced when it
is used to defeat public convenience justify wrong, protect fraud or defend crime. The privilege of being
treated as an entity distinct and separate from its stockholder or members is therefore confined to its
legitimate uses and is subject to certain limitations to prevent the commission of fraud or other illegal or
unfair act. When the corporation is used merely as an alter ego or business conduit of a person, the law
will regard the corporation as the act of that person.
SAN JUAN STRUCTURAL AND STEEL FABRICATORS, INC.,
vs.
COURT OF APPEALS, et.al.
G.R. No. 129459 September 29, 1998

FACTS:

San Juan Structural and Steel Fabricators, Inc.'s amended complaint alleged that on 14 February
1989, plaintiff-appellant entered into an agreement with defendant-appellee Motorich Sales Corporation
for the transfer to it of a parcel of land. On March 1, 1989. Mr. Andres T. Co, president of plaintiff-
appellant corporation, wrote a letter to defendant-appellee Motorich Sales Corporation requesting for a
computation of the balance to be paid: that said letter was coursed through defendant-appellee's broker.
Linda Aduca, who wrote the computation of the balance: that on March 2, 1989, plaintiff-appellant was
ready with the amount corresponding to the balance, covered by Metrobank Cashier's Check No.
004223, payable to defendant-appellee Motorich Sales Corporation; that plaintiff-appellant and
defendant-appellee Motorich Sales Corporation were supposed to meet in the office of plaintiff-appellant
but defendant-appellee's treasurer, Nenita Lee Gruenberg, did not appear; that defendant-appellee

330 | P a g e
Law 321_Corporation LAW_ Case Digest

Motorich Sales Corporation despite repeated demands and in utter disregard of its commitments had
refused to execute the Transfer of Rights/Deed of Assignment which is necessary to transfer the
certificate of title.

ISSUE:

Whether or not the doctrine of piercing the veil of corporate fiction be applied to Motorich.

RULING:

NO.

First, petitioner itself concedes having raised the issue belatedly, not having done so during the
trial, but only when it filed its sur-rejoinder before the Court of Appeals. Thus, this Court cannot
entertain said issue at this late stage of the proceedings. It is well-settled the points of law, theories and
arguments not brought to the attention of the trial court need not be, and ordinarily will not be,
considered by a reviewing court, as they cannot be raised for the first time on appeal. Allowing
petitioner to change horses in midstream, as it were, is to run roughshod over the basic principles of fair
play, justice and due process.
Second, even if the above mentioned argument were to be addressed at this time, the Court still
finds no reason to uphold it. True, one of the advantages of a corporate form of business organization is
the limitation of an investor's liability to the amount of the investment. This feature flows from the legal
theory that a corporate entity is separate and distinct from its stockholders. However, the statutorily
granted privilege of a corporate veil may be used only for legitimate purposes. On equitable
considerations, the veil can be disregarded when it is utilized as a shield to commit fraud, illegality or
inequity; defeat public convenience; confuse legitimate issues; or serve as a mere alter ego or business
conduit of a person or an instrumentality, agency or adjunct of another corporation.

331 | P a g e
Law 321_Corporation LAW_ Case Digest

SERGIO F. NAGUIAT
vs.
NATIONAL LABOR RELATIONS COMMISSION
G.R. No. 116123 March 13, 1997

FACTS:

CFTI held a concessionaire's contract with the Army Air Force Exchange Services ("AAFES") for
the operation of taxi services within Clark Air Base. Sergio F. Naguiat was CFTI's president, while
Antolin T. Naguiat was its vice-president. Like Sergio F. Naguiat Enterprises, Incorporated a trading
firm, it was a family-owned corporation.
Individual respondents were previously employed by CFTI as taxicab drivers. During their
employment, they were required to pay a daily "boundary fee" in the amount of US$26.50 for those
working from 1:00 a.m. to 12:00 noon, and US$27.00 for those working from 12:00 noon to 12:00
midnight. All incidental expenses for the maintenance of the vehicles they were driving were accounted
against them, including gasoline expenses. The drivers worked at least three to four times a week,
depending on the availability of taxicabs. They earned not less than US$15.00 daily. Due to the phase-
out of the US military bases in the Philippines, from which Clark Air Base was not spared, the AAFES
was dissolved, and the services of individual respondents were officially terminated on November 26,
1991.

ISSUE:

Whether or not Sergio F. Naguiat Enterprises, Inc. is a separate and distinct juridical entity which
cannot be held jointly and severally liable for the obligations of CFTI.

RULING:

YES.

From the evidence proffered by both parties, there is no substantial basis to hold that Naguiat
Enterprises is an indirect employer of individual respondents much less a labor only contractor. On the
contrary, petitioners submitted documents such as the drivers' applications for employment with CFTI,
and social security remittances and payroll of Naguiat Enterprises showing that none of the individual
respondents were its employees. Moreover, in the contract between CFTI and AAFES, the former, as
concessionaire, agreed to purchase from AAFES for a certain amount within a specified period a fleet of
vehicles to be "kept on the road" by CFTI, pursuant to their concessionaire's contract. This indicates
that CFTI became the owner of the taxicabs which became the principal investment and asset of the
company.
Private respondents failed to substantiate their claim that Naguiat Enterprises managed,
supervised and controlled their employment. It appears that they were confused on the personalities of
Sergio F. Naguiat as an individual who was the president of CFTI, and Sergio F. Naguiat Enterprises,
Inc., as a separate corporate entity with a separate business. They presumed that Sergio F. Naguiat,
who was at the same time a stockholder and director of Sergio F. Naguiat Enterprises, Inc., was
managing and controlling the taxi business on behalf of the latter. A closer scrutiny and analysis of the
records, however, evince the truth of the matter: that Sergio F. Naguiat, in supervising the taxi drivers
and determining their employment terms, was rather carrying out his responsibilities as president of
CFTI. Hence, Naguiat Enterprises as a separate corporation does not appear to be involved at all in the
taxi business. From the foregoing, the ineludible conclusion is that CFTI was the actual and direct
employer of individual respondents, and that Naguiat Enterprises was neither their indirect employer
nor labor-only contractor. It was not involved at all in the taxi business.

SPECIAL CORPORATIONS
 Art. IV, Sec. 28 (3) and Art. 29 (2), 1987 Constitution

REPUBLIC OF THE PHILIPPINES


vs.
THE HONORABLE INTERMEDIATE APPELLATE COURT
G.R. No. L-68303 January 15, 1988

332 | P a g e
Law 321_Corporation LAW_ Case Digest

FACTS:

The properties in dispute number three undivided lots and Lot No. 2410-B, Psd-864 (Lot 2461
Cad 99)] altogether consisting of a total of 1,024 hectares of ricelands. They are all located in Tiptipon,
Panamao, Sulu. The title thereto stood allegedly in the name of Sultan Jamalul Kiram, who died in
1936. The private respondent, a niece of the late Sultan, now claims that the original certificate of title
thereto was destroyed as a consequence of a fire that gutted the office of the Register of Deeds of Sulu
sometime in February, 1974. She likewise alleges that the owner's copy thereof was lost on account of
the same misfortune. On October 18,1979, she went to the then Court of First Instance of Sulu, Branch
I, at Jolo, now Regional Trial Court, the Honorable Jainal D. Rasul, District Judge, presiding, for
reconstitution.
The Solicitor General presented in the trial court no opposition to the application, and based on
the evidence of the private respondent, the assailed order was issued on June 4, 1980. The Solicitor
General appealed to the then Intermediate Appellate Court, now Court of Appeals, which however
affirmed in toto, on May 24, 1984, the order of the trial court.

ISSUE:

Whether or not the Petition be granted.

RULING:

YES.

It is not disputed, to begin with, that the notices (of hearing) were not posted on the main
entrances of the provincial and municipal halls of the locality in which the lands are located. Under
Section 13, of Republic Act No. 26: The court shall cause a notice of the petition, filed under the
preceding section, to be published, at the expense of the petitioner, twice issues of the Official Gazette,
and to be posted on the main of the municipality or city in which the land is situated, at the provincial
building and of the municipal building at least thirty days prior to the date of hearing. The court shall
likewise cause a copy of the notice to be sent, by registered mail or otherwise, at the expense of the
petitioner, to every person named therein whose address is known, at least thirty days prior to the date
of hearing. Said notice shall state, among other things, the number of the lost or destroyed certificate of
title, if known, the name of the registered owner, the names of the occupants or persons in possession of
the property, the owners of the adjoining properties and all other interested parties, the location, area
and boundaries of the property, and the date on which all persons having any interest therein must
appear and file their claim or objections to the petition. The petitioner shall, at the hearing, submit proof
of the publication, posting and service of the notice as directed by the court.
We have held that such a mode of publication is a jurisdictional requirement. The failure on the
part of the applicant to comply with it confers no jurisdiction upon the court. Neither is there any
showing that the adjacent owners or other interested parties were actually notified of the pending
application. This too taints the petition with a jurisdictional defect. It is not enough that there is
publication in the Official Gazette. Publication of the notice in the Official. The Republic cannot be
faulted for nursing doubts about the private respondent's assertions. In the first place, the private
respondent claims that two deeds have been lost.

THE DIRECTOR OF LANDS


vs.
THE HONORABLE COURT OF APPEALS and IGLESIA NI CRISTO
G.R. No. L-56613 March 14, 1988

FACTS:

On November 28, 1973, private respondent Iglesia ni Cristo filed an application with the then
Court of First Instance of Cavite for registration in its name of a parcel of land with an area of 379
square meters located at Poblacion, Municipality of Amadeo, Cavite. In said application, private
respondent alleged inter alia that it was the owner in fee simple of the land afore-described, having
acquired title thereto by virtue of a Deed of Absolute Sale executed in 1947 by Aquelina de la Cruz in its
favor and that applicant and its predecessors-in-interest had been in actual, continuous, public,
peaceful and adverse possession and occupation of said land in the concept of owner for more than
thirty [30] years. Private respondent prayed that should the Land Registration Act not be applicable, the
provisions of Chapter VIII of Commonwealth Act No. 141, as amended by Republic Act No. 6236 be

333 | P a g e
Law 321_Corporation LAW_ Case Digest

applied as applicant and its predecessors-in-interest had been in possession of the land for more than
thirty [30] years and had introduced improvements thereon, including the fencing thereof on all sides.
The Republic of the Philippines, represented by the Director of Lands, opposed the application on
the following grounds: 1] the applicant and its predecessors-in-interest did not possess sufficient title to
acquire ownership in fee simple of the parcel of land applied for; 2] neither the applicant nor its
predecessors-in-interest have been in open, continuous, exclusive and notorious possession and
occupation of the land in question; and, 3] the subject parcel of land is a portion of the public domain
belonging to the Republic of the Philippines not subject to private appropriation.

ISSUE:

Whether or not the Petition be granted.

RULING:

NO.

Petitioner's heavy reliance on the case of Director of lands v. Reyes, 68 SCRA 177, is misplaced.
The original tracing cloth plan was deemed essential in that case as the lands involved were vast tracts
of uncultivated, mountainous and thickly forested lands which were necessarily difficult to Identify,
unlike the land subject matter of the instant registration case which is more readily Identifiable by
reason of its location, its comparatively smaller size of 379 square meters as well as the chapel
constructed thereon by private respondent in 1968. Moreover, the documentary evidence presented
therein consisting in the blue-prints of two [2] survey plans were not approved by the Director of Lands
unlike Exhibit "O" which bore the approval of the Land Registration Commission at the time it was
empowered by law to approve original survey plans and which was re- verified and approved by the
Bureau of Lands when the authority to approve original survey plans was withdrawn from the Land
Registration Commission by P.D. No. 239.

 Corporation Aggregate

IGLESIA EVANGELICA METODISTA EN LAS ISLAS FILIPINAS (IEMELIF) (Corporation Sole), INC.,
et al
vs.
BISHOP NATHANAEL LAZARO, et al
G.R. No. 184088 July 6, 2010

FACTS:

Bishop Nicolas Zamora established the petitioner Iglesia Evangelica Metodista En Las Islas
Filipinas, Inc. (IEMELIF) as a corporation sole with Bishop Zamora acting as its "General
Superintendent." Thirty-nine years later in 1948, the IEMELIF enacted and registered a by-laws that
established a Supreme Consistory of Elders (the Consistory), made up of church ministers, who were to
serve for four years. The by-laws empowered the Consistory to elect a General Superintendent, a
General Secretary, a General Evangelist, and a Treasurer General who would manage the affairs of the
organization. For all intents and purposes, the Consistory served as the IEMELIF’s board of directors.
Apparently, although the IEMELIF remained a corporation sole on paper (with all corporate
powers theoretically lodged in the hands of one member, the General Superintendent), it had always
acted like a corporation aggregate. The Consistory exercised IEMELIF’s decision-making powers without
ever being challenged. Subsequently, during its 1973 General Conference, the general membership
voted to put things right by changing IEMELIF’s organizational structure from a corporation sole to a
corporation aggregate. On May 7, 1973 the Securities and Exchange Commission (SEC) approved the
vote. For some reasons, however, the corporate papers of the IEMELIF remained unaltered as a

334 | P a g e
Law 321_Corporation LAW_ Case Digest

corporation sole. Only in 2001, about 28 years later, did the issue reemerge. In answer to a query from
the IEMELIF, the SEC replied on April 3, 2001 that, although the SEC Commissioner did not in 1948
object to the conversion of the IEMELIF into a corporation aggregate, that conversion was not properly
carried out and documented.

ISSUE:

Whether or not a corporation sole be converted into a corporation aggregate by mere amendment
of its articles of incorporation.

RULING:

YES.

The Corporation Code provides no specific mechanism for amending the articles of incorporation
of a corporation sole. But, as the RTC correctly held, Section 109 of the Corporation Code allows the
application to religious corporations of the general provisions governing non-stock corporations.
Although a non-stock corporation has a personality that is distinct from those of its members
who established it, its articles of incorporation cannot be amended solely through the action of its board
of trustees. The amendment needs the concurrence of at least two-thirds of its membership. If such
approval mechanism is made to operate in a corporation sole, its one member in whom all the powers of
the corporation technically belongs, needs to get the concurrence of two-thirds of its membership. The
one member, here the General Superintendent, is but a trustee, according to Section 110 of the
Corporation Code, of its membership. There is no point to dissolving the corporation sole of one member
to enable the corporation aggregate to emerge from it. Whether it is a non-stock corporation or a
corporation sole, the corporate being remains distinct from its members, whatever be their number. The
increase in the number of its corporate membership does not change the complexion of its corporate
responsibility to third parties. The one member, with the concurrence of two-thirds of the membership of
the organization for whom he acts as trustee, can self-will the amendment.
G.R. No. 172447 September 18, 2009
IGLESIA EVANGELICA METODISTA EN LAS ISLAS FILIPINAS (IEMELIF), INC., Petitioner,
vs.
NATANAEL B. JUANE, Respondent.
x - - - - - - - - - - - - - - - - - - - - - - -x
G.R. No. 179404
NATANAEL B. JUANE, Petitioner,
vs.
IGLESIA EVANGELICA METODISTA EN LAS ISLAS FILIPINAS (IEMELIF), INC., Respondent.
DECISION

FACTS:

IEMELIF is a religious corporation existing and duly organized under Philippine laws. [Juane] is a
former minister or pastor of IEMELIF. He was elected as one of the members of the Highest Consistory of
Elders (or Board of Trustees) of IEMELIF in the February 2000 IEMELIF General Conference. During the
concluding Anniversary Service of said General Conference, IEMELIF Bishop Nathanael P. Lazaro, the
General Superintendent of the whole IEMELIF Church and the General Administrator of the IEMELIF
Cathedral in Tondo, Manila, during the reading of the "IEMELIF Workers’ Assignment", announced the
appointment and assignment of Juane as Resident Pastor of the Cathedral Congregation in Tondo,
Manila. By virtue and as a consequence of such appointment, Defendant Rev. Juane was authorized to
stay at and occupy the Resident Pastor’s residence inside the Cathedral complex. By the same reason,
he also took charge of the Cathedral facilities and other property of the church in said premises.

ISSUE:

Whether or not the transformation of IEMELF from corporation sole to an aggregate one is valid.

RULING:

YES.

335 | P a g e
Law 321_Corporation LAW_ Case Digest

Juane maintains that the "IEMELIF" that filed the Complaint before the MeTC had no personality
to eject him from the subject property. The Church has remained a corporation sole, since its
transformation to a corporation aggregate was legally defective. Juane, thus, claims that he is now the
corporation sole, who is entitled to the physical possession of the subject property as owner thereof. In
fact, on the basis of these same arguments.
Even if the transformation of IEMELIF from a corporation sole to a corporation aggregate was
legally defective, its head or governing body, i.e., Bishop Lazaro, whose acts were approved by the
Highest Consistory of Elders, still did not change. A corporation sole is one formed by the chief
archbishop, bishop, priest, minister, rabbi or other presiding elder of a religious denomination, sect, or
church, for the purpose of administering or managing, as trustee, the affairs, properties and
temporalities of such religious denomination, sect or church. As opposed to a corporation aggregate, a
corporation sole consists of a single member, while a corporation aggregate consists of two or more
persons. If the transformation did not materialize, the corporation sole would still be Bishop Lazaro, who
himself performed the questioned acts of removing Juane as Resident Pastor of the Tondo Congregation.
If the transformation did materialize, the corporation aggregate would be composed of the Highest
Consistory of Elders, which nevertheless approved the very same acts. As either Bishop Lazaro or the
Highest Consistory of Elders had the authority to appoint Juane as Resident Pastor of the IEMELIF
Tondo Congregation, it also had the power to remove him as such or transfer him to another
congregation.
DISSOLUTION OF CORPORATIONS
 Where Creditors are Not Affected

TEODORO B. VESAGAS, AND WILFRED D. ASIS


vs.
THE HONORABLE COURT OF APPEALS
G.R. NO. 142924 DECEMBER 5, 2001

FACTS:

The respondent spouses Delfino and Helenda Raniel are members in good standing of the Luz
Villaga Tennis Clud, Inc. (club). They alleged that petitioner Teodoro B. Vesagas, who claims to be the
club's duly elected president, in conspiracy with petitioner Wilfred D. Asis, who, in turn, claims to be its
duly elected vice-president and legal counsel, summarily stripped them of their lawful membership,
without due process of law. Thereafter, respondent spouses filed a Complaint with the Securities and
Exchange Commission (SEC) on March 26, 1997 against the petitioners. It was docketed as SEC Case
No. 03-97-5598.In this case, respondents asked the Commission to declare as illegal their expulsion
from the club as it was allegedly done in utter disregard of the provisions of its by-laws as well as the
requirements of due process. They likewise sought the annulment of the amendments to the by-laws
made on December 8, 1996, changing the annual meeting of the club from the last Sunday of January
to November and increasing the number of trustees from nine to fifteen. Finally, they prayed for the
issuance of a Temporary Restraining Order and Writ of Preliminary Injunction. The application for TRO
was denied by SEC Hearing Officer Soller in an Order dated April 29, 1997.

ISSUE:

Whether or not SEC has jurisdiction.

RULING:

YES.

Petitioners' attempt to impress upon this court that the club has never been a corporation is
devoid of merit. It must fail in the face of the Commission's explicit finding that the club was duly
registered and a certificate of incorporation was issued in its favor. It ought to be remembered that the
question of whether the club was indeed registered and issued a certification or not is one which
necessitates a factual inquiry. On this score, the finding of the Commission, as the administrative
agency tasked with among others the function of registering and administering corporations, is given
great weight and accorded high respect. We therefore have no reason to disturb this factual finding
relating to the club's registration and incorporation.
Moreover, by their own admission contained in the various pleadings which they have filed in the
different stages of this case, petitioners themselves have considered the club as a corporation. This
admission, under the rules of evidence, binds them and may be taken or used against them. Since the

336 | P a g e
Law 321_Corporation LAW_ Case Digest

admission was made in the course of the proceedings in the same case, it does not require proof, and
actually may be contradicted only by showing that it was made through palpable mistake or that no
such admission was made.

 Where Creditors are Affected

AVON DALE GARMENTS, INC.


vs.
NATIONAL LABOR RELATIONS COMMISSION, ET.AL.
G.R. No. 117932 July 20, 1995

FACTS:

Private respondents were employees of petitioner Avon Dale Garments, Inc. and its predecessor-
in-interest, Avon Dale Shirt Factory. Following a dispute brought about by the rotation of workers, a
compromise agreement was entered into between petitioner and private respondents wherein the latter
were terminated from service and given their corresponding separation pay.
However, upon refusal of the petitioner to include in the computation of private respondents'
separation pay the period during which the latter were employed by Avon Dale Shirt Factory, private
respondents filed a complaint with the labor arbiter claiming a deficiency in their separation pay
(docketed as NLRC-NCR-00-02-00810-93). According to private respondents, their previous employment
with petitioner's predecessor-in-interest, Avon Dale Shirt Factory, should be credited in computing their
separation pay considering that Avon Dale Shirt factory was not dissolved and they were not in turn
hired as new employees by Avon Dale Garments, Inc.

ISSUE:

Whether or not the petitioner be held liable for private respondents' separation pay from Avon
Dale Shirt Factory.

RULING:

YES.

Petitioner failed to establish that Avon Dale Garments, Inc., is a separate and distinct entity from
Avon Dale Shirt Factory, absent any showing that there was indeed an actual closure and cessation of
the operations of the latter. The mere filing of the Articles of Dissolution with the Securities and
Exchange Commission, without more, is not enough to support the conclusion that actual dissolution of
an entity in fact took place. On the contrary, the prevailing circumstances in this case indicated that
Petitioner Company is not distinct from its predecessor Avon Dale Shirt Factory, but in fact merely
continued the operations of the latter under the same owners, the same business venture, at same
address, and even continued to hire the same employees.
Thus, conformably with established jurisprudence, the two entities cannot be deemed as separate
and distinct where there is a showing that one is merely the continuation of the other. In fact, even a
change in the corporate name does not make a new corporation, whether effected by a special act or
under a general law; it has no effect on the identity of the corporation, or on its property, rights, or
liabilities.Respondent NLRC therefore, did not commit any grave abuse of discretion in holding that
petitioner should likewise include private respondents' employment with Avon Dale Shirt Factory in
computing private respondents' separation pay as petitioner failed to substantiate its claim that it is a
distinct entity.

337 | P a g e
Law 321_Corporation LAW_ Case Digest

DAGUHOY ENTERPRISES, INC.


vs.
RITA L. PONCE
G.R. No. L-6515 October 18, 1954

FACTS:

The Daguhoy Enterprises, Inc., a local corporation, with principal office in the City of Manila filed
in the Court of First Instance of the City Civil Case No. 15923 against Rita L. Ponce and her husband
Domingo Ponce, for the collection of a loan of P6,190 with interest at 12 per cent per annum from June
24, 1950, plus P2,500 as attorney's fees and P34 as expenses of litigation. Defendant filed an answer
admitting practically all the allegations of the complaint, set up affirmative defenses, and a counterclaim
asking for the cancellation of the mortgage which secured the payment of the loan of P6,190. They also
filed a petition for the inclusion of Potenciano Gapol as a third party litigant, at the same time filing a
third party complaint against him asking for damages in the amount of P25,000. The plaintiff
corporation answered the counterclaim and opposed the petition for the inclusion of a third party
litigant. Thereafter, plaintiff corporation filed a motion for judgment on the pleadings which petition was
opposed by the defendants. Then, on October 9, 1952, the trial court rendered judgment against
defendants.

ISSUE:

Whether or not the said deposit relieve the present defendants from the payment of interests from
the time of deposit, on the theory that the deposit amounted to a payment of the loan.

RULING:

NO.

It should be remembered that Civil Case No. 13753 though in the same Court of First Instance of
Manila, is a separate and different action, for accounting not only for the amount of the loan but for
other sums. The plaintiff in that case was Gapol in behalf of the Daguhoy Enterprises, Inc. and the
defendants are Domingo Ponce and his son Buhay M. Ponce. The parties in the present case are
different. Furthermore, when the plaintiff in said case 13753 petitioned the trial court for permission to
withdraw the deposit, presumably to pay the loan involved in the present action, his petition was denied
by the court because of the opposition of the defendants therein, one of whom is Domingo Ponce, co-
defendant of Rita Ponce in the present case. The result was that the present plaintiff corporation could
not take possession and dispose of said amount. In other words, the loan is not yet paid.

 Involuntary
338 | P a g e
Law 321_Corporation LAW_ Case Digest

PHILIPPINE NATIONAL BANK


vs.
CIF OF RIZAL
G.R. NO. 63201 MAY 27, 1992

FACTS:

On March 1, 1954, private respondents entered into a contract of lease with Philippine Blooming
Mills, Co., Inc., (PBM for brevity) whereby the letter shall lease the aforementioned parcels of land as
factory site. PBM was duly organized and incorporated on January 19, 1952 with a corporate term of
twenty-five (25) years. This leasehold right of PBM covering the parcels of land was duly annotated at
the back of the above stated certificates of title as Entry No. 9367/T-No. 32843. The contract of lease
provides that the term of the lease is for twenty years beginning from the date of the contract and "is
extendable for another term of twenty years at the option of the LESSEE should its term of existence be
extended in accordance with law."
On October 11, 1963, PBM executed in favor of Philippine National Bank (PNB for brevity),
petitioner herein, a deed of assignment, conveying and transferring all its rights and interests under the
contract of lease which it executed with private respondents. The assignment was for and in
consideration of the loans granted by PNB to PBM. The deed of assignment was registered and
annotated at the back of the private respondents' certificates of title as Entry No. 85215/T-No. 32843.

ISSUE:

Whether or not the cancellation of the entries on respondent's certificates of title valid and proper.

RULING:

YES.

The contract of lease expressly provides that the term of the lease shall be twenty years from the
execution of the contract but can be extended for another period of twenty years at the option of the
lessee should the corporate term be extended in accordance with law. Clearly, the option of the lessee to
extend the lease for another period of twenty years can be exercised only if the lessee as corporation
renews or extends its corporate term of existence in accordance with the Corporation Code which is the
applicable law. Thus, in the instant case, the initial term of the contract of lease which commenced on
March 1, 1954 ended on March 1, 1974. PBM as lessee continued to occupy the leased premises beyond
that date with the acquiescence and consent of the respondents as lessor. Records show however, that
PBM as a corporation had a corporate life of only twenty-five (25) years which ended an January 19,
1977. It should be noted however that PBM allowed its corporate term to expire without complying with
the requirements provided by law for the extension of its corporate term of existence.
There is no need for the institution of a proceeding for quo warranto to determine the time or date
of the dissolution of a corporation because the period of corporate existence is provided in the articles of
incorporation. When such period expires and without any extension having been made pursuant to law,
the corporation is dissolved automatically insofar as the continuation of its business is concerned.
Considering the foregoing in relation to the contract of lease between the parties herein, when
PBM's corporate life ended on January 19, 1977 and its 3-year period for winding up and liquidation
expired on January 19, 1980, the option of extending the lease was likewise terminated on January 19,
1977 because PBM failed to renew or extend its corporate life in accordance with law. From then on, the
respondents can exercise their right to terminate the lease pursuant to the stipulations in the contract.

 Liquidation: Methods

METROPOLITAN BANK and TRUST COMPANY


vs.
CENTRO DEVELOPMENT CORPORATION, CHONGKING KEHYENG, MANUEL CO KEHYENG and
Quirino Kehyeng
G.R. No. 180974 June 13, 2012

FACTS:

339 | P a g e
Law 321_Corporation LAW_ Case Digest

On March 20, 1990, in a special meeting of the board of directors of respondent Centro
Development Corporation, its president Go Eng Uy was authorized to mortgage its properties and assets
to secure the medium-term loan of P 84 million of Lucky Two Corporation and Lucky Two Repacking.
The properties and assets consisted of a parcel of land with a building and improvements located at
Salcedo St., Legaspi Village, Makati City, and covered by Transfer Certificate of Title Nos. 139880 and
139881. This authorization was subsequently approved on the same day by the stockholders. Maria
Jacinta V. Go, the corporate secretary, issued a Secretary’s Certificate.
San Carlos failed to pay these outstanding obligations despite demand. Thus, petitioner, as
trustee of the MTI, enforced the conditions thereof and initiated foreclosure proceedings, denominated
as Foreclosure No. S-04-11, on the mortgaged properties.

ISSUE:

Whether or not the petitioner, as creditor or as trustee, had a cause of action to move for the
extrajudicial foreclosure of the subject properties mortgaged under the MTI.

RULING:

NO.

It is the intent of the COMPANY that the BORROWERS will obtain additional loans or credit
accommodations from certain other banking or financial institutions in accordance with arrangements
made by the BORROWERS with the CREDITORS.
All obligations covered by this indenture shall be evidenced by a mortgage participation certificate
in the form of schedule ii hereof, the issuance of which by the trustee to the participating creditor/s
shall be in accordance with section 7 of this indenture, provided the aggregate loan values of the
collateral, based on the latest appraisal thereof, are not exceeded.
Moreover, it is worthy to note that respondents do not assail the previous MTI executed with BPI.
They do not question the validity of the mortgage constituted over all or substantially all of respondent
Centro’s assets pursuant to the 21 March 1994 MTI in the amount of P 84 million. Nor do they question
the additional loans increasing the value of the mortgage to P 144 million; or the use of Centro’s
properties as collateral for the loans of San Carlos, Lucky Two Corporation, and Lucky Two Repacking.

METROPOLITAN BANK & TRUST COMPANY, INC.


vs.
THE BOARD OF TRUSTEES OF RIVERSIDE MILLS CORPORATION PROVIDENT AND RETIREMENT
FUND
G.R. No. 176959 September 8, 2010

FACTS:

RMC established a Provident and Retirement Plan for its regular employees. Under the Plan, RMC
and its employees shall each contribute 2% of the employee’s current basic monthly salary, with RMC’s
contribution to increase by 1% every five (5) years up to a maximum of 5%. The contributions shall form
part of the provident fund (the Fund) which shall be held, invested and distributed by the Commercial
Bank and Trust Company. On October 15, 1979, the Board of Trustees of RMCPRF (the Board) entered
into an Investment Management Agreement with Philbank (petitioner Metropolitan Bank and Trust
Company). Pursuant to the Agreement, petitioner shall act as an agent of the Board and shall hold,
manage, invest and reinvest the Fund in Trust Account No. 1797 in its behalf. The Agreement shall be
in force for one (1) year and shall be deemed automatically renewed unless sooner terminated either by
petitioner bank or by the Board.

340 | P a g e
Law 321_Corporation LAW_ Case Digest

In 1984, RMC ceased business operations. Nonetheless, petitioner continued to render


investment services to respondent Board. In a letter dated September 27, 1995, petitioner informed
respondent Board that Philbank’s Board of Directors had decided to apply the remaining trust assets
held by it in the name of RMCPRF against part of the outstanding obligations of RMC. Subsequently,
respondent RMC Unpaid Employees Association, Inc. (Association), representing the terminated
employees of RMC, learned of Trust Account No. 1797. Through counsel, they demanded payment of
their share in a letter dated February 4, 1997. When such demand went unheeded, the Association,
along with the individual members of RMCPRF, filed a complaint for accounting against the Board and
its officers.

ISSUE:

Whether or not the functions of the Board of Trustees ceased upon with RMC’s closure.

RULING:

NO.

Under Section 122 of the Corporation Code, a dissolved corporation shall nevertheless continue
as a body corporate for three (3) years for the purpose of prosecuting and defending suits by or against it
and enabling it to settle and close its affairs, to dispose and convey its property and to distribute its
assets, but not for the purpose of continuing the business for which it was established. Within those
three (3) years, the corporation may appoint a trustee or receiver who shall carry out the said purposes
beyond the three (3)-year winding-up period. Thus, a trustee of a dissolved corporation may commence a
suit which can proceed to final judgment even beyond the three (3)-year period of liquidation.
In the same manner, during and beyond the three (3)-year winding-up period of RMC, the Board
of Trustees of RMCPRF may do no more than settle and close the affairs of the Fund. The Board retains
its authority to act on behalf of its members, albeit, in a limited capacity. It may commence suits on
behalf of its members but not continue managing the Fund for purposes of maximizing profits. Here, the
Board’s act of issuing the Resolution authorizing petitioner to release the Fund to its beneficiaries is still
part of the liquidation process, which is, satisfaction of the liabilities of the Plan, and does not amount
to doing business. Hence, it was properly within the Board’s power to promulgate.

YAM
vs.
COURT OF APPEALS
GR No. 104726 11 February 1999

FACTS:

Parties entered into several loan agreements, the petitioners, Yam and Lent, being the borrowers
while the private respondent, Manphil Investment Corporaton, the lender. In said contract, petitioners
were given a loan of P500,000.00 by private respondent. The contract provided for the payment of 12%
annual interest, 2% monthly penalty, 1 1/2% monthly service charge, and 10% attorney's fees.
Denominated the first Industrial Guarantee and Loan Fund (IGLF), the loan was secured by a chattel
mortgage on the printing machinery in petitioners' establishment.
By April 2, 1985, petitioners had paid their first loan of P500,000.00. On November 4, 1985,
private respondent was placed under receivership by the Central Bank and Ricardo Lirio and Cristina
Destajo were appointed as receiver and in-house examiner, respectively.
A check was sent to respondent as partial payment of the second loan which was marked as full
payment in the vouchers. Demands were made for the balance of the same, however, it was unheeded
prompting respondent to file a case against the petitioner for collection of the balance.
The trial court ruled in favor of respondents which the Court of Appeals affirmed.

ISSUE:

Whether or not petitioner is liable to the penalties and service charges of the loan.

RULING:

YES.

341 | P a g e
Law 321_Corporation LAW_ Case Digest

The alleged condonation of the penalties and service charges by Sobrepeñas, president of
respondent, must be in writing to be binding between and among the parties. Since it was not reduced
in writing, the same is not effective. Further, the alleged condonation happened after the respondent
corporation was placed under receivership. As held in Villanueva v. Court of Appeals “the appointment
of a receiver operates to suspend the authority of a corporation and of its directors and officers over its
property and effects, such authority being reposed in the receiver.” Thus, Sobrepeñas had no authority
to condone the debt. Petition denied.

ALHAMBRA CIGAR & CIGARETTE MANUFACTURING COMPANY, INC.


vs.
SECURITIES OF EXCHANGE COMMISSION
G.R. NO. L-23606 JULY 29, 1968

FACTS:

Petitioner Alhambra Cigar and Cigarette Manufacturing Company, Inc. was duly incorporated
under Philippine laws on January 15, 1912. By its corporate articles it was to exist for fifty (50) years
from incorporation. Its term of existence expired on January 15, 1962. On that date, it ceased
transacting business, entered into a state of liquidation. Thereafter, a new corporation. Alhambra
Industries, Inc. was formed to carry on the business of Alhambra.
On June 20, 1963 within Alhambra's three-year statutory period for liquidation - Republic Act
3531 was enacted into law. It amended Section 18 of the Corporation Law; it empowered domestic
private corporations to extend their corporate life beyond the period fixed by the articles of incorporation
for a term not to exceed fifty years in any one instance. Previous to Republic Act 3531, the maximum
non-extendible term of such corporations was fifty years.
On July 15, 1963, at a special meeting, Alhambra's board of directors resolved to amend
paragraph "Fourth" of its articles of incorporation to extend its corporate life for an additional fifty years,
or a total of 100 years from its incorporation. FOURTH. That the term for which said corporation is to
exist is fifty (50) years from and after the date of incorporation, and for an additional period of fifty (50)
years thereafter.On October 28, 1963, Alhambra's articles of incorporation as so amended certified
correct by its president and secretary and a majority of its board of directors, were filed with respondent
Securities and Exchange Commission (SEC).

ISSUE:

Whether or not the corporation can still extend its corporate term within the three-year statutory
period for liquidation.

RULING:

NO.

A corporation cannot extend its life by amendment of its articles of incorporation effected during
the three-year period for liquidation when its original term of existence had already expired. Since the

342 | P a g e
Law 321_Corporation LAW_ Case Digest

privilege of extension is purely statutory, all of the statutory conditions precedent must be complied with
in order that the extension may be effectuated. And, generally these conditions must be complied with,
and the steps necessary to effect the extension must be taken, during the life of the corporation, and
before the expiration of the term of existence as original fixed by its charter or the general law, since, as
a rule, the corporation is ipso facto dissolved as soon as that time expires. So where the extension is by
amendment of the articles of incorporation, the amendment must be adopted before that time.
And, similarly, the filing and recording of a certificate of extension after that time cannot relate
back to the date of the passage of a resolution by the stockholders in favor of the extension so as to save
the life of the corporation. The contrary is true, however, and the doctrine of relation will apply, where
the delay is due to the neglect of the officer with whom the certificate is required to be filed, or to a
wrongful refusal on his part to receive it. And statutes in some states specifically provide that a renewal
may be had within a specified time before or after the time fixed for the termination of the corporate
existence.

CHUNG KA BIO
vs.
INTERMEDIATE APPELLATE COURT
G.R. NO. 71837 JULY 26, 1988

FACTS:

The Philippine Blooming Mills Company, Inc. was incorporated on January 19, 1952, for a term
of 25 years which expired on January 19,1977. On May 14, 1977, the members of its board of directors
executed a deed of assignment of all of the accounts receivables, properties, obligations and liabilities of
the old PBM in favor of Chung Siong Pek in his capacity as treasurer of the new PBM, then in the
process of reincorporation.On June 14, 1977, the new PMB was issued a certificate of incorporation by
the Securities and Exchange Commission.
On May 5, 1981, Chung Ka Bio and the other petitioners herein, all stockholders of the old PBM,
filed with the SEC a petition for liquidation (but not for dissolution) of both the old PBM and the new
PBM. The allegation was that the former had become legally non-existent for failure to extend its
corporate life and that the latter had likewise been ipso facto dissolved for non-use of the charter and
continuous failure to operate within 2 years from incorporation.

ISSUE:

Whether or not the board of directors of an already dissolved corporation have the inherent
power, without the express consent of the stockholders, to convey all its assets to a new corporation.

RULING:

YES.

While we agree that the board of directors is not normally permitted to undertake any activity
outside of the usual liquidation of the business of the dissolved corporation, there is nothing to prevent
the stockholders from conveying their respective shareholdings toward the creation of a new corporation
to continue the business of the old. Winding up is the sole activity of a dissolved corporation that does
not intend to incorporate anew. If it does, however, it is not unlawful for the old board of directors to
negotiate and transfer the assets of the dissolved corporation to the new corporation intended to be
created as long as the stockholders have given their consent. This was not prohibited by the Corporation
Act. In fact, it was expressly allowed by Section 28-1/2.
The petitioners and the private respondents are not strangers but relatives and close business
associates. The PBM office is in the heart of Metro Manila. The new corporation, like the old, employs as
many as 2,000 persons, the same personnel who worked for the old PBM. Additionally, one of the
petitioners, Chung Siong Pek was one of the directors who executed the deed of assignment in favor of
the old PBM and it was he also who received the deeded assets on behalf and as treasurer of the new
PBM. Surely, these circumstances must operate to bar the petitioners now from questioning the deed of
assignment after this long period of inaction in the protection of the rights they are now belatedly
asserting. Laches has operated against them.

343 | P a g e
Law 321_Corporation LAW_ Case Digest

REPUBLIC OF THE PHILIPPINES


vs.
MARSMAN DEVELOPMENT COMPANY
G.R. NO. L-18956 APRIL 27, 1972

FACTS:

Sometime before October 15, 1953 an investigation was conducted on the business operation and
activities of the corporation leading to the discovery that certain taxes were due (from) it on logs
produced from its concession. The Bureau of Internal Revenue made three assessments, totalling
P59,133.78, and demanded payment thereof. Defendants however failed to pay the taxes hence the
filing of charges in court. The defendants contend that the present action is already barred under
section 77 of the Corporation Law, Act No. 1459, as amended, which allows the corporate existence of a
corporation to continue only for three years after its dissolution, for the purpose of presenting or
defending suits by or against it, and to settle and close its affairs.
They point out that inasmuch as the Marsman Development Co. was extra-judicially dissolved on
April 23, 1954, a fact admitted in the amended complaint, the filing of both the original complaint on
September 8, 1958 and the amended complaint on August 26, 1956 was beyond the aforesaid three-
year period.

ISSUE:

Whether or not the right of the government to collect the sums has already prescribed.

RULING:

NO.

The stress given by appellants to the extinction of the corporate and juridical personality as such
of appellant corporation by virtue of its extra-judicial dissolution which admittedly took place on April
23, 1954 is misdirected.
Further, at any time during said three years said corporation is authorized and empowered to
convey all of its property to trustees for the benefit of members, stock-holders, creditors, and others
interested. From and after any such conveyance by the corporation of its property in trust for the benefit
of its members, stockholders, creditors, and others in interest, all interest which the corporation had in
the property terminates, the legal interest vests in the trustee, and the beneficial interest in the
members, stockholders, creditors, or other persons in interest.
Thus, in whatever way the matter may be viewed, the Government became the creditor of the
corporation before the completion of its dissolution by the liquidation of its assets. Appellant F.H.
Burgess, whom it chose as liquidator, became in law the trustee of all its assets for the benefit of all
persons enumerated in Section 78, including its creditors, among whom is the Government, for the
taxes herein involved. To assume otherwise would render the extra-judicial dissolution illegal and void,
since, according to Section 62 of the Corporation Law, such kind of dissolution is permitted only when it
"does not affect the rights of any creditor having a claim against the corporation."
It is immaterial that the present action was filed after the expiration of three years after April 23,
1954, for at the very least, and assuming that judicial enforcement of taxes may not be initiated after
said three years despite the fact that the actual liquidation has not been terminated and the one in
charge thereof is still holding the assets of the corporation, obviously for the benefit of all the creditors
thereof, the assessment aforementioned, made within the three years, definitely established the
Government as a creditor of the corporation for whom the liquidator is supposed to hold assets of the
corporation. And since the suit at bar is only for the collection of taxes finally assessed against the

344 | P a g e
Law 321_Corporation LAW_ Case Digest

corporation within the three years invoked by appellants, their fourth assignment of error cannot be
sustained.

TAN TIONG BIO


vs.
COMMISSION OF INTERNAL REVENUE
G.R. NO. L-15778 APRIL 23, 1962

FACTS:

On October 19, 1946, the Central Syndicate, a corporation organized under the laws of the
Philippines, thru its General Manager, David Sycip, sent a letter to the Collector of Internal Revenue
advising the latter that it purchased from Dee Hong Lue the entire stock of surplus properties which the
said Dee Hong Lue had bought from the Foreign Liquidation Commission and that as it assumed Dee
Hong Lue's obligation to pay the 3-1/2% sales tax on said surplus goods, it was remitting the sum of
P43,750.00 in his behalf as deposit to answer for the payment of said sales tax with the understanding
that it would later be adjusted after the determination of the exact consideration of the sale.
On January 31, 1948, the syndicate again wrote the Collector requesting the refund of P1,103.28
representing alleged excess payment of sales tax due to the adjustment and reduction of the purchase
price in the amount of P31,522.18. The Collector decided after a thorough investigation of the facts that
the Central Syndicate was the importer and original seller of the surplus goods in question and,
therefore, the one liable to pay the sales tax. Accordingly, on January 4, 1952, the Collector assessed
against the syndicate the amount of P33,797.88 and P300.00 as deficiency sales tax, inclusive of the
25% surcharge and compromise penalty, respectively, and on the same date, in a separate letter, he
denied the request of the syndicate for the refund of the sum of P1,103.28.

ISSUE:

Whether the sales tax of a dissolved corporation can be enforced against its successors-in-interest
who are the present petitioners.

RULING:

YES.

The creditor of a dissolved corporation may follow its assets once they passed into the hands of
the stockholders. And it has been stated, with reference to the effect of dissolution upon taxes due from
a corporation, "that the hands of the government cannot, of course, collect taxes from a defunct
corporation, it loses thereby none of its rights to assess taxes which had been due from the corporation,
and to collect them from persons, who by reason of transactions with the corporation, hold property
against which the tax can be enforced and that the legal death of the corporation no more prevents such
action than would the physical death of an individual prevent the government from assessing taxes
against him and collecting them from his administrator, who holds the property which the decedent had
formerly possessed". Bearing in mind that our corporation law is of American origin, the foregoing
authorities have persuasive effect in considering similar cases in this jurisdiction. This must have been
taken into account when in G.R. No. L-8800 this Court said that petitioners could be held personally
liable for the taxes in question as successors-in-interest of the defunct corporation.
Considering that the Central Syndicate realized from the sale of the surplus goods a net profit of
P229,073.83, and that the sale of said goods was the only transaction undertaken by said syndicate,
there being no evidence to the contrary, the conclusion is that said net profit remained intact and was
distributed among the stockholders when the corporation liquidated and distributed its assets on
August 15, 1948, immediately after the sale of the said surplus goods. Petitioners are therefore the
beneficiaries of the defunct corporation and as such should be held liable to pay the taxes in question.
However, there being no express provision requiring the stockholders of the corporation to be solidarily
liable for its debts which liability must be express and cannot be presumed.
 Duration

REYNOLDS PHILIPPINE CORPORATION


vs.
COURT OF APPEALS
G.R. NO. L-36187 JANUARY 17, 1989

FACTS:

345 | P a g e
Law 321_Corporation LAW_ Case Digest

In its complaint of June 2, 1966, the petitioner sought to recover from the private respondent
Serg's Products, Inc. the sum of P32,565.62 representing the unpaid price of aluminum foils and cores
sold and delivered by it to the latter. The private respondent denied liability for payment of the account
on the ground that the aluminum foils and cores were ordered or purchased by Serg's Chocolate
Products, a partnership of Antonio Goquiolay and Luis Sequia Mendoza, not Serg's Products, Inc., a
corporation managed and controlled by Antonio Goquiolay and his wife Conchita Goquiolay, as majority
stockholders and principal officers.

ISSUE:

Whether or not the real debtor of the petitioner were the Private Respondents.

RULING:

YES.

Although the commercial documents were indeed in the name of "Serg's Chocolate Products," the
following facts proved that the true purchaser of the aluminum foils and cores from the petitioner, was
"Serg's Products, Inc." not the partnership denominated "Serg's Chocolate Products."
The attempt to make the two factories appear as two separate businesses, when in reality they
are but one, is but a devise to defeat the ends of the law and should not be permitted to prevail.
Although the coffee factory is a corporation and, by legal fiction, an entity existing separate and apart
from persons composing it, T and his family, it is settled that this fiction of law, which had been
introduced as a matter of convenience and to subserve the ends of justice cannot be invoked to further
an end subversive of that purpose.

MAMBULAO LUMBER COMPANY


vs.
PHILIPPINE NATIONAL BANK
GR No. L-22973 30 January 1968

FACTS:

The plaintiff applied for an industrial loan with interest with the PNB. To secure the payment of
the loan, the plaintiff mortgaged to defendant PNB a parcel of land, together with the buildings and
improvements existing thereon as well as various sawmill equipment, rolling unit and other fixed assets
of the plaintiff. However, the plaintiff failed to pay the amortizations on the amounts released to and
received by it.

346 | P a g e
Law 321_Corporation LAW_ Case Digest

Repeated demands were made upon the plaintiff to pay its obligation but it failed or otherwise
refused to do so. Upon inspection and verification made by employees of the PNB, it was found that the
plaintiff had already stopped operation about the end of 1957 or early part of 1958. Thus, PNB
requested for the foreclosure of the real estate mortgage as well as the chattel mortgage.

ISSUE:

Whether or not petitioner foreclosure of the mortgage is tenable.

RULING:

NO.

It is clear that there was no further necessity to foreclose the mortgage of herein appellant's
chattels since the obligation has already been paid for. On this ground alone, it may be declared that the
sale of appellant's chattels, illegal and void. The Court took into consideration the fact that the PNB
must have been led to believe that the stipulated 10% of the unpaid loan for attorney's fees in the real
estate mortgage was legally maintainable, and in accordance with such belief, herein appellee bank
insisted that the proceeds of the sale of appellant's real property was deficient to liquidate the latter's
total indebtedness. Be that as it may, however, still the subsequent sale of herein appellant's chattels
illegal and objectionable on other grounds. The parties have agreed that in case of foreclosure, the sale
should be made elsewhere not necessarily where the properties are located. This stipulation is allowed
under the law which provides for the general rule. However, the sale was made in the place where the
properties are situated. A clear violation of the agreement of the parties. Thus, the foreclosure is not
tenable.

 Powers of Corporation at Liquidation

VITALIANO N. AGUIRRES II and FIDEL N. AGUIRRE


vs.
FQB+7, INC., NATHANIEL D. BOCOBO, PRISCILA BOCOBO and ANTONIO DE VILLA
G.R. No. 170770 January 9, 2013

FACTS:

On October 5, 2004, Vitaliano filed, in his individual capacity and on behalf of FQB+7, Inc., a
Complaint for intra-corporate dispute, injunction, inspection of corporate books and records, and
damages, against respondents Nathaniel D. Bocobo, Priscila D. Bocobo and Antonio De Villa. The
Complaint alleged that FQB+7 was established in 1985 with the following directors and subscribers, as
reflected in its Articles of Incorporation.
The substantive changes found in the GIS, respecting the composition of directors and
subscribers of FQB+7, prompted Vitaliano to write to the "real" Board of Directors (the directors reflected
in the Articles of Incorporation), represented by Fidel N. Aguirre. In this letter dated April 29, 2004,

347 | P a g e
Law 321_Corporation LAW_ Case Digest

Vitaliano questioned the validity and truthfulness of the alleged stockholders meeting held on
September 3, 2002. He asked the "real" Board to rectify what he perceived as erroneous entries in the
GIS, and to allow him to inspect the corporate books and records. The "real" Board allegedly ignored
Vitaliano’s request.

ISSUE:

Whether or not dissolved corporation may continue as a body corporate for the limited purpose of
liquidating the corporate assets and distributing them to its creditors, stockholders, and others in
interest.

RULING:

YES.

A corporation’s board of directors is not rendered functus officio by its dissolution. Since Section
122 allows a corporation to continue its existence for a limited purpose, necessarily there must be a
board that will continue acting for and on behalf of the dissolved corporation for that purpose. In fact,
Section 122 authorizes the dissolved corporation’s board of directors to conduct its liquidation within
three years from its dissolution. Jurisprudence has even recognized the board’s authority to act as
trustee for persons in interest beyond the said three-year period. Thus, the determination of which
group is the bona fide or rightful board of the dissolved corporation will still provide practical relief to
the parties involved.
The same is true with regard to Vitaliano’s shareholdings in the dissolved corporation. A party’s
stockholdings in a corporation, whether existing or dissolved, is a property right which he may vindicate
against another party who has deprived him thereof. The corporation’s dissolution does not extinguish
such property right.
Further, Intra-corporate disputes remain even when the corporation is dissolved.

CATMON SALES INTERNATIONAL CORP.


vs.
ATTY. MANUEL D. YNGSON, JR.
G.R. No. 179761 January 15, 2010

FACTS:

On February 8, 1999, petitioner Catmon Sales International Corporation filed a Petition for
Declaration in a State of Suspension of Payments with the SEC. On May 10, 2000, the SEC declared
petitioner technically insolvent considering that there was no settlement reached with its creditors and
that its inability to pay its creditors had lasted for a period longer than one year from the filing of the
petition. In the same Order, the SEC appointed respondent Manuel D. Yngson, Jr. of Receivers and
Liquidators, Inc. as petitioner’s liquidator.
On May 31, 2001, the SEC terminated the services of respondent. Respondent, in turn, submitted
his Accomplishment Report summarizing all the activities he had undertaken and billed the SEC the
total sum of P623,214.35, representing his liquidator’s fee and reimbursement of out-of-pocket
expenses. On December 18, 2001, the SEC ordered that an audit be conducted to determine the proper
amount to be paid to respondent. The Corporation Finance Department noted a slight difference in the
liquidator’s computation. On September 23, 2004, respondent manifested to the SEC that he was willing
to reduce his liquidator’s fee provided that his request for administrative expenses be settled in full. On
June 23, 2005, the SEC, through its General Counsel, ordered the members of the Board of Directors of
petitioner to pay respondent his claim for reimbursement of the expenses incurred in the performance of
his duties as liquidator, together with his liquidator’s fee, for a total amount of P398,284.40

348 | P a g e
Law 321_Corporation LAW_ Case Digest

ISSUE:

Whether or not SEC has the power to fix the amount of the liquidator’s fee.

RULING:

YES.

However, to countenance petitioner’s posturing would be to unduly delimit the broad powers
granted to the SEC under Presidential Decree No. 902-A, specifically the all-encompassing provision in
Section 3 that the SEC has "absolute jurisdiction, supervision and control" over all corporations who are
the grantees of primary franchises and/or license or permit issued by the government to operate in the
Philippines. There is no gainsaying, therefore, that the SEC is authorized to determine the fees of
receivers and liquidators not only when there is "failure of agreement" between the parties but also in
the absence thereof. A contrary ruling would give license to corporations under liquidation or
receivership to refuse to participate in negotiations for the fixing of the compensation of their liquidators
or receivers so as to evade their obligation to pay the same.
Petitioner may not have been given the chance to meet face to face with respondent for the
purpose of determining the latter’s fee. But this fact alone should not invalidate the amount fixed by the
SEC. What matters is the reasonableness of the fee in light of the services rendered by the liquidator. It
is the policy of the SEC to provide uniform/fair and reasonable compensation or fees for the comparable
services rendered by the duly designated members of the Management Committee (MANCOM),
rehabilitation receivers and liquidators in corporations or partnerships placed under
MANCOM/receivership or liquidation, pursuant to Section 6(d) of Presidential Decree No. 902-A, the
SEC Rules on Corporate Recovery, the Corporation Code of the Philippines, the Securities Regulation
Code, and other related laws enforced by the SEC.
Clearly, the fee fixed by the SEC was not without basis. Besides, as correctly held by the CA,
"respondent actually rendered services in accordance with his oath of office as liquidator for which he is
entitled to be compensated by petitioner."
RENE KNECHT AND KNECHT INC.
vs.
UNITED CIGARETTE INC.
GR NO. 139370 JULY 4, 2002

FACTS:

Rose Packing Company, Inc. (Rose Packing), a domestic corporation, owns parcels of land one of
which is covered by TCT No. 73620 which was mortgaged with the Philippine Commercial and Industrial
Bank (PCIB). Said parcels of land were later sold to United Cigarette Corporation (UCC), through its
President Rene Knecht, where Rose Packing made a warranty that the lots are free from all liens and
encumbrances, except the real estate mortgage constituted over the area covered by TCT No. 73620.
Before the deed of sale could be executed, the parties found that Rose Packing’s actual obligation
with the PCIB far exceeded the P250,000.00 which UCC assumed to pay under their agreement. So the
PCIB demanded additional collateral from UCC as a condition precedent for the approval of the sale of
the mortgaged property. However, UCC did not comply.
Meanwhile, Rose Packing again offered to sell the same lots to other prospective buyers without
the knowledge of UCC and without returning to the latter the earnest money it earlier paid.

ISSUE:

Whether or not the execution of the judgment would still lie against a dissolved corporation.

RULING:

YES.

In Reburiano vs. Court of Appeals, a case with similar facts, this Court Held:
“the trustee (of a dissolved corporation) may commence a suit which can proceed to final judgment even
beyond the three-year period (of liquidation), no reason can be conceived why a suit already commenced
by the corporation itself during its existence, not by a mere trustee who, by fiction, merely continues the
legal personality of the dissolved corporation, should not be accorded similar treatment – to proceed to
final judgment and execution thereof.”

349 | P a g e
Law 321_Corporation LAW_ Case Digest

The dissolution of UCC itself, or the expiration of its three-year liquidation period, should not be a
bar to the enforcement of its rights as a corporation. One of these rights, to be sure, includes the
UCC’s right to seek from the court the execution of a valid and final judgment in Civil Case No. 9165 –
through its trustee/liquidator Encarnacion Gonzales Wong – for the benefit of its stockholders, creditors
and any other person who may have legal claims against it. To hold otherwise would be to allow
petitioners to unjustly enrich themselves at the expense of UCC. This, in effect, renders nugatory all the
efforts and expenses of UCC in its quest to secure justice, not to mention the undue delay in disposing
of this case prejudicial to the administration of justice.

CELIA B. CHUA
vs.
NATIONAL LABOR RELATIONS COMMISSION
G.R. NOS. 89971-75 OCTOBER 17, 1990

FACTS:

In December, 1985, Stanford Microsystems, Inc. (Stanford) filed a petition for suspension of
payments and appointment of rehabilitation receiver with the Securities and Exchange Commission
(SEC). At that time, Stanford had seven (7) secured creditor banks and more or less seven thousand one
hundred twenty-four (7,124) employees. On February 5, 1986, the SEC declared Stanford to be in a
state of suspension of payments. It issued an order appointing Sycip Gorres & Velayo & Co. (SGV) as the
rehabilitation receiver. In view of these developments, the former employees of Stanford filed with the
Department of Labor and Employment (DOLE) cases for money claims.In January, 1987, the SEC
disapproved the Rehabilitation Plan submitted by SGV and dismissed Stanford's Petition for Suspension
of Payments and Appointment of a Rehabilitation Receiver. Subsequently, the SEC ordered Stanford's
liquidation.
At the time Stanford filed a petition for suspension of payments and appointment of rehabilitation
receiver with SEC, Stanford had seven (7) secured creditor banks and approximately 7,124 employees.
On March 13, 1987, the seven secured creditor banks of Stanford and 6,341 former employees executed
a Memorandum of Agreement to speed up the orderly liquidation of Stanford. All the creditor banks and
the said employees were represented by their respective counsel in the negotiations which were
supervised by Regional Director Luna C. Piezas of the DOLE, National Capital Region.

ISSUE:

Whether or not the SEC has original and exclusive jurisdiction over the liquidation of Stanford
including the procedures for settling the money claims of former workers and employees.

RULING:

YES.

Jurisdiction over liquidation proceedings of insolvent corporations is vested in the Securities and
Exchange Commission (SEC) pursuant to Presidential Decree No. 902-A, as amended. On the other
hand, jurisdiction over money claims of employees against their employers is vested in the Labor Arbiter
whose decision may be appealed to the National Labor Relations Commission (NLRC) pursuant to Article
217 of the Labor Code.
An insolvency proceeding is similar to the settlement of a decedent's estate in that it is a
proceeding in rem and is binding against the whole world. Therefore, all persons which have interest in
the subject matter involved, whether or not they are given notice are equally bound. Thus, "a liquidation
of similar import or other equivalent general liquidation must also necessarily be a proceeding in rem so

350 | P a g e
Law 321_Corporation LAW_ Case Digest

that all other interested personswhether known to the parties or not may be bound by such
proceedings."
The rule is that a declaration of bankruptcy or a judicial liquidation must be present before
preferences over various money claims may be enforced. Since a liquidation proceeding is a proceeding
in rem, all claims of creditors whether preferred or non-preferred, the Identification of the preferred ones
and the totality of the employer's asset should be brought into the picture. There can then be an
authoritative, fair and binding adjudication. The money claims of workers pose a special problem of
jurisdiction when liquidation proceedings are on-going because of the highly preferred nature given by
law to said claims. Further, the rule is that such money claims were correctly submitted in the course of
the liquidation proceedings at the SEC.
LUIS C. CLEMENTE
vs.
COURT OF APPEALS
G.R. NO. 82407 MARCH 27, 1995

FACTS:

The parties in this case wrestled concerning the ownership of a piece of land.The defendants
(herein private respondents), claimed ownership of the property by virtue of acquisitive prescription.
Plaintiffs on the other hand claimed ownership based on the following allegations:
The "Sociedad Popular Calambeña" organization conceived by the parties as a "Sociedad
Anonima," was organized on or about the advent of the early American occupation of the Philippines. Its
principal business was cockfighting or the operation and management of a cockpit.On June 8, 1911, or
during its existence, the "Sociedad" acquired by installments the parcel of land(subject of the case)
above described from the Friar Lands Estate of Calamba, Laguna at the total cost of P2,676.00.
Plaintiffs evidence also shows that Mariano Elepaño and Pablo Clemente, now both deceased,
were original stockholders of the aforesaid "sociedad." Pablo Clemente subscribed and paid FOUR
HUNDRED EIGHTEEN (418) shares of stocksworth TWO THOUSAND (P2,000.00) PESOS. Pablo
Clemente's shares of stocks were however later distributed and apportioned to his heirs, in accordance
with a Project of Partition to Luis Clemente, shares worth P510; to Ricardo Clemente, shares worth
P510; to Leonor Clemente de Elepaño, shares also worth P510, and to Placida Clemente de Belarmino
shares worth P510. On September 24, 1932, in accordance with the aforesaid project of .partition, the
"sociedad" issued stock certificates to the aforesaid heirs of Pablo Clemente.

ISSUE:

Whether or not petitioners can be held liable, given their submissions, to have succeeded in
establishing for themselves a firm title to the property in question.

RULING:

YES.

The Court find petitioners' evidence to be direly wanting. Except in showing that they are the
successors-in-interest of Elepaño and Clemente, petitioners have been unable to come up with any
evidence to substantiate their claim of ownership of the corporate asset. Absent a corporate liquidation,
it is the corporation, not the stockholders, which can assert, if at all, any title to the corporate assets.
The court, even then, expressed some reservations on the corporation's being able to still validly pursue
such a claim. It said: even assuming that their parents were the only stockholders of Sociedad, and
assuming further that Sociedad has ceased to exist, these do not ipso facto vest ownership over the
property in the hands of plaintiffs-appellants. Again, assuming that sociedad is a duly-organized entity,
under the laws of the Philippines, its corporate existence is separate and distinct from its stockholders
and from other corporations to which it may be connected (If it was not organized and registered under
Philippine laws as a private corporation, it is a de facto corporation, as found by the court below, with
the right to exercise corporate powers, and thus it is imperative that any of the modes of transferring
ownership from said entity must be shown.
If, indeed, the sociedad has long become defunct, it should behoove petitioners, or anyone else
who may have any interest in the corporation, to take appropriate measures before a proper forum for a
peremptory settlement of its affairs. We might invite attention to the various modes provided by the
Corporation Code for dissolving, liquidating or winding up, and terminating the life of the corporation.

351 | P a g e
Law 321_Corporation LAW_ Case Digest

CARLOS GELANO
vs.
COURT OF APPEALS
G.R. NO. L-39050 FEBRUARY 24, 1981

FACTS:

Private respondent Insular Sawmill, Inc. is a corporation organized on September 17, 1945 with a
corporate life of fifty (50) years, or up to September 17, 1995, with the primary purpose of carrying on a
general lumber and sawmill business. To carry on this business, private respondent leased the
paraphernal property of petitioner-wife Guillermina M. Gelano for P1,200.00 a month. It was while
private respondent was leasing the aforesaid property that its officers and directors had come to know
petitioner-husband Carlos Gelano who received from the corporation cash advances on account of
rentals to be paid by the corporation on the land
Out of the cash advances in the total sum of P25,950.00, petitioner Carlos Gelano was able to
pay only P5,950.00 thereby leaving an unpaid balance of P20,000.00 which he refused to pay despite
repeated demands by private respondent. Petitioner Guillermina M. Gelano refused to pay on the ground
that said amount was for the personal account of her husband asked for by, and given to him, without
her knowledge and consent and did not benefit the family.

ISSUE:

Whether or not a dissolved corporation, could still continue prosecuting and defending suits after
its dissolution and beyond the period of three years.

RULING:

YES.

For this reason, Section 78 of the same law authorizes the corporation, "at any time during said
three years to convey all of its property to trustees for the benefit of members, Stockholders, creditors
and other interested," evidently for the purpose, among others, of enabling said trustees to prosecute
and defend suits by or against the corporation begun before the expiration of said period.
In the case at bar, when Insular Sawmill, Inc. was dissolved on December 31, 1960, under Section 77 of
the Corporation Law, it still has the right until December 31, 1963 to prosecute in its name the present
case. After the expiration of said period, the corporation ceased to exist for all purposes and it can no
longer sue or be sued.
However, a corporation that has a pending action and which cannot be terminated within the
three-year period after its dissolution is authorized under Section 78 to convey all its property to
trustees to enable it to prosecute and defend suits by or against the corporation beyond the Three-year
period although private respondent (did not appoint any trustee, yet the counsel who prosecuted and
defended the interest of the corporation in the instant case and who in fact appeared in behalf of the
corporation may be considered a trustee of the corporation at least with respect to the matter in
litigation only. Said counsel had been handling the case when the same was pending before the trial
court until it was appealed before the Court of Appeals and finally to this Court.
It was therefore held by the Supreme Court that there was a substantial compliance with Section
78 of the Corporation Law and as such, private respondent Insular Sawmill, Inc. could still continue
prosecuting the present case even beyond the period of three (3) years from the time of its dissolution.
The trustee may commence a suit which can proceed to final judgment even beyond the three-
year period. No reason can be conceived why a suit already commenced By the corporation itself during
its existence, not by a mere trustee who, by fiction, merely continues the legal personality of the
dissolved corporation should not be accorded similar treatment allowed to proceed to final judgment and
execution thereof.

JAMES REBURIANO
vs.
COURT OF APPEALS
G.R. NO. 102965 JANUARY 21, 1999

FACTS:

A certain judgment was rendered in favor of the private respondent which became final and
executor. It appears that prior to the promulgation of the decision of the trial court, private respondent

352 | P a g e
Law 321_Corporation LAW_ Case Digest

amended its articles of incorporation to shorten its term of existence to July 8, 1983. The amended
articles of incorporation was approved by the Securities and Exchange Commission on March 2, 1984.
The trial court was not notified of this fact. On February 13, 1991, petitioners moved to quash the writ
of execution alleging that the private respondent was no longer in existence and had no more juridical
personality and so, as such, it no longer had the capacity to sue and be sued; That after the [private
respondent], as a corporation, lost its existence and juridical personality, Atty. Romualdo M. Jubay had
no more client in this case and so his appearance in this case was no longer possible and tenable; That
in view of the foregoing premises, therefore, the decision rendered by this Honorable Court and by the
Honorable Court of Appeals are patent nullity, for lack of jurisdiction and lack of capacity to sue and be
sued on the part of the [private respondent.]; That the above-stated change in the situation of parties,
whereby the [private respondent] ceased to exist since 8 July 1983, renders the execution of the decision
inequitable or impossible.

ISSUE:

Whether or not a dissolved and non-existing corporation could be represented by a lawyer as


counsel.

RULING:

YES.

It is to be noted that the time during which the corporation, through its own officers, may
conduct the liquidation of its assets and sue and be sued as a corporation is limited to three years from
the time the period of dissolution commences: but there is no time limit within which the trustees must
complete a liquidation placed in their hands. It is provided only that the conveyance to the trustees
must be made within the three-year period. It may be found impossible to complete the work of
liquidation within the three-year period or to reduce disputed claims to judgment.
The authorities are to the effect that suits by or against a corporation abate when it ceased to be
an entity capable of suing or being sued; but trustees to whom the corporate assets have been conveyed
pursuant to the authority may sue and be sued as such in all matters connected with the liquidation.
There is, therefore, no reason why the suit filed by private respondent should not be allowed to
proceed to execution. It is conceded by petitioners that the judgment against them and in favor of
private respondent in C.A. G.R. No. 16070 had become final and executory. The only reason for their
refusal to execute the same is that there is no existing corporation to which they are indebted.
Such argument is untenable. The law specifically allows a trustee to manage the affairs of the
corporation in liquidation. Consequently, any supervening fact, such as the dissolution of the
corporation, repeal of a law, or any other fact of similar nature would not serve as an effective bar to the
enforcement of such right.

REPUBLIC PLANTERS BANK


vs.
COURT OF APPEALS,
G.R. No. 93073 December 21, 1992

FACTS:

Shozo Yamaguchi and Fermin Canlas were President/Chief Operating Officer and Treasurer
respectively, of Worldwide Garment Manufacturing, Inc. By virtue of Board Resolution No.1 dated
August 1, 1979, defendant Shozo Yamaguchi and private respondent Fermin Canlas were authorized to
apply for credit facilities with the petitioner Republic Planters Bank in the forms of export advances and
letters of credit/trust receipts accommodations.
Petitioner bank issued nine promissory notes. In some promissory notes, the name Worldwide
Garment Manufacturing, Inc. was apparently rubber stamped above the signatures of defendant and
private respondent.
On December 20, 1982, Worldwide Garment Manufacturing, Inc. noted to change its corporate
name to Pinch Manufacturing Corporation. Subsequently, petitioner bank filed a complaint for the

353 | P a g e
Law 321_Corporation LAW_ Case Digest

recovery of sums of money covered among others, by the nine promissory notes with interest thereon,
plus attorney's fees and penalty charges.
The complainant was originally brought against Worldwide Garment Manufacturing, Inc. inter
alia, but it was later amended to drop Worldwide Manufacturing, Inc. as defendant and substitute Pinch
Manufacturing Corporation it its place. Defendants Pinch Manufacturing Corporation and Shozo
Yamaguchi did not file an Amended Answer and failed to appear at the scheduled pre-trial conference
despite due notice.
Only private respondent Fermin Canlas filed an Amended Answer wherein he denied having
issued the promissory notes in question since according to him, he was not an officer of Pinch
Manufacturing Corporation, but instead of Worldwide Garment Manufacturing, Inc., and that when he
issued said promissory notes in behalf of Worldwide Garment Manufacturing, Inc., the same were in
blank, the typewritten entries not appearing therein prior to the time he affixed his signature.

ISSUE:

Whether or not the amendment in a corporation's Articles of Incorporation effecting a change of


corporate name extinguished the personality of the original corporation.

RULING:

NO.

The corporation, upon such change in its name, is in no sense a new corporation, nor the
successor of the original corporation. It is the same corporation with a different name, and its character
is in no respect changed. A change in the corporate name does not make a new corporation, and
whether effected by special act or under a general law, has no affect on the identity of the corporation, or
on its property, rights, or liabilities.
The corporation continues, as before, responsible in its new name for all debts or other liabilities
which it had previously contracted or incurred. As a general rule, officers or directors under the old
corporate name bear no personal liability for acts done or contracts entered into by officers of the
corporation, if duly authorized. Inasmuch as such officers acted in their capacity as agent of the old
corporation and the change of name meant only the continuation of the old juridical entity, the
corporation bearing the same name is still bound by the acts of its agents if authorized by the Board.

FOREIGN CORPORATIONS
 Definition and Rights

AVON
vs.
COURT OF APPEALS et. al.
G.R. No. 97642 August 29, 1997

FACTS:

Yupangco Cotton Mills engaged to secure with Worldwide Security and Insurance Co. Inc., a
foreign corporation not doing business in the Philippines with no office, place of business or agents in
the Philippines, several of its properties for the periods July 6, 1979 to July 6, 1980 and from October 1,
1980 to October 1, 1981, under separate insurance policies for the same amount. Both contracts were
covered by reinsurance treaties between Worldwide Surety and Insurance and several foreign
reinsurance companies, including the petitioners.
On December 16, 1979 and May 2, 1981, within the respective effectivity periods of the policies,
the properties therein insured were razed by fire, thereby giving rise to the obligation of the insurer to
indemnify the Yupangco Cotton Mills. Partial payments were made by Worldwide Surety and Insurance
and some of the reinsurance companies.
On May 2, 1983, Worldwide Surety and Insurance, in a Deed of Assignment, acknowledged a
remaining balance of P19,444,447.75 still due Yupangco Cotton Mills, and assigned to the latter all
reinsurance proceeds still collectible from all the foreign reinsurance companies. Thus, in its interest as
assignee and original insured, Yupangco Cotton Mills instituted this collection suit against the
petitioners.

ISSUE:

354 | P a g e
Law 321_Corporation LAW_ Case Digest

Whether or not a foreign corporation has rights under Philippine law.

RULING:

YES.

A foreign corporation, is one which owes its existence to the laws of another state, and generally,
has no legal existence within the state in which it is foreign. It was also held that corporations have no
legal status beyond the bounds of the sovereignty by which they are created.
Nevertheless, it is widely accepted that foreign corporations are, by reason of state comity,
allowed to transact business in other states and to sue in the courts of such fora. In the Philippines
foreign corporations are allowed such privileges, subject to certain restrictions, arising from the state's
sovereign right of regulation. Before a foreign corporation can transact business in the country, it must
first obtain a license to transact business here and secure the proper authorizations under existing law.
If a foreign corporation engages in business activities without the necessary requirements, it
opens itself to court actions against it, but it shall not be allowed to maintain or intervene in an action,
suit or proceeding for its own account in any court or tribunal or agency in the Philippines.
The purpose of the law in requiring that foreign corporations doing business in the country be
licensed to do so, is to subject them to the jurisdiction of the Philippine courts, otherwise, a foreign
corporation illegally doing business here because of its refusal or neglect to obtain the required license
and authority to do business may successfully though unfairly plead such neglect or illegal act so as to
avoid service and thereby impugn the jurisdiction of the local courts.
The same danger does not exist among foreign corporations that are indubitably not doing
business in the Philippines. Indeed, if a foreign corporation does not do business here, there would be
no reason for it to be subject to the State's regulation.

PALTING
vs.
SAN JOSE PETROLEUM INC.
G.R. No.L-14441. December 17, 1966

FACTS:

SAN JOSE OIL, is a domestic mining corporation, 90% of the outstanding capital stock of which
is owned by respondent SAN JOSE PETROLEUM, a foreign (Panamanian) corporation, the majority
interest of which is owned by OIL INVESTMENTS, INC., another foreign (Panamanian) company.
This latter corporation in turn is wholly (100%) owned by PANTEPEC OIL COMPANY, C. A., and
PANCOASTAL PETROLEUM COMPANY, C. A., both organized and existing under the laws of Venezuela.
As of September 30, 1956, there were 9,979 stockholders of PANCOASTAL PETROLEUM found in 49
American states and U.S. territories, holding 3,476,988 shares of stock; whereas, as of November 30,
1956, PANTEPEC OIL COMPANY was said to have 3,077,916 shares held by 12,373 stockholders
scattered in 49 American states.
In the two lists of stockholders, there is no indication of the citizenship of these stockholders, or
of the total number of authorized stocks of each corporation for the purpose of determining the
corresponding percentage of these listed stockholders in relation to the respective capital stock of said
corporation.

ISSUE:

Whether or not the "tie-up" between the two corporations is violative of the Constitution, the
Laurel-Langley Agreement, the Petroleum Act of 1949, and the Corporation Law.

RULING:

YES.

The privilege to utilize, exploit, and develop the natural resources of this country was granted, by
Article III of the Constitution, to Filipino citizens or to corporations or associations 60% of the capital of
which is owned by such citizens. With the Parity Amendment to the Constitution, the same right was
extended to citizens of the United States and business enterprises owned or controlled, directly or
indirectly, by citizens of the United States.

355 | P a g e
Law 321_Corporation LAW_ Case Digest

There could be no serious doubt as to the meaning of the word "citizens" used in the
aforementioned provisions of the Constitution. The right was granted to 2 types of persons: natural
persons (Filipino or American citizens) and juridical persons (corporations 60% of which capital is
owned by Filipinos and business enterprises owned or controlled directly or indirectly, by citizens of the
United States). In American law, "citizen" has been defined as "one who, under the constitution and laws
of the United States, has a right to vote for representatives in congress and other public officers, and
who is qualified to fill offices in the gift of the people."
SAN JOSE PETROLEUM an American business is not entitled to parity rights in the Philippines.
In the circumstances, we have to hold that the respondent SAN JOSE PETROLEUM, as presently
constituted, is not a business enterprise that is authorized to exercise the parity privileges under the
Parity Ordinance, the Laurel-Langley Agreement and the Petroleum Law. Its tie-up with SAN JOSE OIL
is, consequently, illegal.

 Documentary

GMBH & CO.


vs.
Hon. ISNANI et al
G.R. No. 109272 August 10, 1994

FACTS:

Petitioner is a multinational company organized and existing under the laws of the Federal
Republic of Germany. On July 6, 1983, petitioner filed an application with the SEC for the
establishment of a RAH in the Philippines. The application was approved by the BOI on Sep. 6, 1983.
Consequently, on Sep. 20, 1983, the SEC issued a Certificate of Registration and License to petitioner.
Private respondent Romana Lanchinebre was a sales representative of petitioner. On March 1992,
she secured a loan of P25,000.00 from petitioner. Subsequently she made additional cash advances in
the sum of P10,000.00. Of the total amount, P12,170.37 remained unpaid. Despite demand, private
respondent Romana failed to settle her obligation.
On September 2, 1992, petitioner filed a Complaint for collection of sum of money against private
respondents spouses Romana and Teofilo Lanchinebre w/ the RTC. Instead of filing their Answer,
private respondents moved to dismiss the Complaint. This was opposed by petitioner.

ISSUE:

Whether or not the petitioner has capacity to sue and be sued in the Philippines despite the fact
that petitioner is duly licensed by the SEC to set up and operate a RAH in the country and that it has
continuously operated as such for the last 9 years.

RULING:

YES.

It is clear that petitioner is a foreign corporation doing business in the Philippines. Hence,
Petitioner is covered by the Omnibus Investment Code of 1987. Petitioner does not engage in commercial
dealings or activities in the country because it is precluded from doing so by P.D. No. 218, under which
it was established.
Nonetheless, it has been continuously, since 1983, acting as supervision, communications and
coordination center for its home office's affiliates in Singapore, and in the process has named its local
agent and has employed Philippine nationals like private respondent Romana Lanchinebre (an
employee).
From this uninterrupted performance by petitioner of acts pursuant to its primary purposes and
functions as a regional/area headquarters for its home office, it is clear that petitioner is doing business
in the country. Moreover, private respondents are estopped from assailing the personality of petitioner.

356 | P a g e
Law 321_Corporation LAW_ Case Digest

The rule is that a party is estopped to challenge the personality of a corporation after having
acknowledged the same by entering into a contract with it.
And the doctrine of estoppel to deny corporate existence applies to foreign as well as to domestic
corporations. One who has dealt with a corporation of foreign origin as a corporate entity is estopped to
deny its corporate existence and capacity. The principle will be applied to prevent a person contracting
with a foreign corporation from later taking advantage of its noncompliance with the statutes chiefly in
cases where such person has received the benefits of the contract.

 Appointment of Resident Agent

NEW YORK MARINE MANAGERS, INC.


vs.
COURT OF APPEALS
G.R. No. 111837 October 24, 1995

FACTS:

Petitioner is a foreign corporation organized under the laws of the United States while defendant
is a local domestic corporation organized under Philippine law. On 25 July 1990 American Natural Soda
Ash Corporation (ANSAC) loaded in Portland , U.S.A., a shipment of soda ash on board the vessel "MS
Abu Hanna" for delivery to Manila . The supplier/shipper insured the shipment with petitioner. Upon
arrival in Manila the shipment was unloaded and transferred to the vessel "MV Biyayang Ginto" owned
by private respondent. Since the shipment allegedly sustained wettage, hardening and contamination, it
was rejected as total loss by the consignees. When the supplier sought to recover the value of the cargo
loss from petitioner the latter paid the claim in the amount of US$58,323.96.
On 20 November 1991 petitioner as subrogee filed with the RTC Manila a complaint for damages
against private respondent. Thereafter, private respondent filed a motion to dismiss the complaint one of
its grounds cited being plaintiff having no legal capacity to sue. Such was denied by the trial court upon
opposition of the petitioner.

ISSUE:

Whether or not petitioner can seek for relief from our courts.

RULING:

NO.

A foreign corporation not engaged in business in the Philippines may exercise the right to file an
action in Philippine courts for an isolated transaction. When the allegations in the complaint have a
bearing on the plaintiff's capacity to sue and merely state that the plaintiff is a foreign corporation
existing under the laws of the United States, such averment conjures two alternative possibilities: either
the corporation is engaged in business in the Philippines, or it is not so engaged.
In the first, the corporation must have been duly licensed in order to maintain the suit; in the
second, and the transaction sued upon is singular and isolated, no such license is required. In either
case, compliance with the requirement of license, or the fact that the suing corporation is exempt
therefrom, as the case may be, cannot be inferred from the mere fact that the party suing is a foreign
corporation.
The qualifying circumstance being an essential part of the plaintiff's capacity to sue must be
affirmatively pleaded. Hence, the ultimate fact that a foreign corporation is not doing business in the
Philippines must first be disclosed for it to be allowed to sue in Philippine courts under the isolated
transaction rule. Failing in this requirement, the complaint filed by petitioner with the trial court, it
must be said, fails to show its legal capacity to sue.
In the case at bar, petitioner's complaint is fatally defective for failing to allege its duly authorized
representative or resident agent in this jurisdiction. The pleadings filed by counsel for petitioner do not
suffice. True, a lawyer is generally presumed to be properly authorized to represent any cause in which
he appears, and no written power of attorney is required to authorize him to appear in court for his

357 | P a g e
Law 321_Corporation LAW_ Case Digest

client, but such is disputable. Where said authority has been challenged or attacked by the adverse
party the lawyer is required to show proof of such authority or representation in order to bind his client.
The requirement of the production of authority is essential because the client will be bound by his
acquiescence resulting from his knowledge that he was being represented by said attorney.
 Amendment of License

AETNA CASUALTY & SURETY Co.


vs.
PACIFIC STAR LINE et al
G.R. No.L-26809 December 29, 1977

FACTS:

Smith Bell & Co. (Philippines), Inc. and Aetna Surety Casualty & Surety Co. Inc., as subrogee,
instituted a case to recover the amount of US$2,300.00 representing the value of the stolen and
damaged cargo plus litigation expenses and exemplary damages against Pacific Star Line, The Bradman
Co. Inc., Manila Port Service and/or Manila Railroad Company, Inc. alleging that Pacific Star Line, as a
common carrier, was operating the vessel SS Ampal on a commercial run between United States and
Philippine Ports including Manila; that the defendant, The Bradman Co. Inc., was the ship agent in the
Philippines for the SS Ampal and/or Pacific Star Line; that the Manila Railroad Co. Inc. and Manila Port
Service were the arrastre operators in the port of Manila and were authorized to delivery cargoes
discharged into their custody on presentation of release papers from the Bureau of Customs and the
steamship carrier and/or its agents; that plaintiff sustained losses due to the negligence of Pacific Star
Line prior to delivery of the cargo to Manila or, in the alternative, due to the negligence of Manila Port
Service after delivery of the cargo to it by the SS Ampal.
The defendants alleged that the plaintiff, Aetna Casualty & Surety Company, is a foreign
corporation not duly licensed to do business in the Philippines and, therefore, without capacity to sue
and be sued.

ISSUE:

Whether or not Aetna Casualty & Surety Company, a foreign corporation not doing business in
the Philippines, can claim damages against defendant.

RULING:

YES.

It is settled that if a foreign corporation is not engaged in business in the Philippines, it may not
be denied the right to file an action in Philippine courts for isolated transactions.
The object of Sections 68 and 69 of the Corporation Law was not to prevent the foreign
corporation from performing single acts, but to prevent it from acquiring a domicile for the purpose of
business without taking the steps necessary to render it amenable to suit in the local courts. It was
never the purpose of the Legislature to exclude a foreign corporation which happens to obtain an
isolated order for business from the Philippines, from securing redress in the Philippine courts.
Aetna Casualty & Surety Company is not transacting business of insurance in the Philippines for
which it must have a license. The contract of insurance was entered into in New York, U.S.A., and
payment was made to the consignee in its New York branch.
Since, Aetna Casualty & Surety Company is not engaged in the business of insurance in the
Philippines but is merely collecting a claim assigned to it by the consignee, it is not barred from filing
the instant case although it has not secured a license to transact insurance business in the Philippines.
The case is remanded to the trial court for further proceedings to determine the liability of the
defendants-appellees.

HATHIBHAI BULAKHIDAS
vs.
HONORABLE PEDRO L. NAVARRO
L-49695 April 7, 1986

358 | P a g e
Law 321_Corporation LAW_ Case Digest

FACTS:

Petitioner, a foreign partnership, filed a complaint against a domestic corporation, DSC, before
the CFI of Rizal for the recovery of damages allegedly caused by the failure of the said shipping
corporation to deliver the goods shipped to it by petitioner to their proper destination.
Paragraph 1 of said complaint alleged that plaintiff is "a foreign partnership firm not doing
business in the Philippines" and that it is "suing under an isolated transaction." Defendant filed a
motion to dismiss the complaint on the ground that plaintiff has no capacity to sue and that the
complaint does not state a valid cause of action against defendant.
Acting on said motion to dismiss, the CFI dismissed the complaint on the ground that plaintiff
being "a foreign corporation or partnership not doing business in the Philippines it cannot exercise the
right to maintain suits before our Courts."

ISSUE:

Whether a foreign corporation “not engaged in business in the Philippines” can institute an action
before Philippine courts.

RULING:

YES.

It is settled that if a foreign corporation is not engaged in business in the Philippines, it may not
be denied the right to file an action in Philippine courts for isolated transactions. The object of Sections
68 and 69 of the Corporation law was not to prevent the foreign corporation from performing single acts,
but to prevent it from acquiring a domicile for the purpose of business without taking the steps
necessary to render it amenable to suit in the local courts. It was never the purpose of the Legislature to
exclude a foreign corporation which happens to obtain an isolated order for business from the
Philippines, from securing redress in the Philippine courts.
No general rule or governing principle can be laid down as to what constitutes 'doing' or
'engaging' in or 'transacting' business. Indeed, each case must be judged in the light of its peculiar
environmental circumstances. The true test, however, seems to be whether the foreign corporation is
continuing the body or substance of the business or enterprise for which it was organized or whether it
has substantially retired from it and turned it over to another. The term implies a continuity of
commercial dealings and arrangements, and contemplates, to that extent, the performance of acts or
works or the exercise of some of the functions normally incident to, and in progressive prosecution of,
the purpose and object of its organization.
While plaintiff is a foreign corporation without license to transact business in the Philippines, it
does not follow that it has no capacity to bring the present action. Such license is not necessary because
it is not engaged in business in the Philippines. A foreign corporation not engaged in business in the
Philippines is not barred from seeking redress from the courts of the Philippines.

SCHMID & OBERLY, INC.


vs.
RJL MARTINEZ FISHING CORPORATION
G.R. No. 75198 October 18, 1988

FACTS:

RJL MARTINEZ is engaged in the business of deep-sea fishing. The transaction, which gave rise
to the present controversy, involves twelve (12) "Nagata"-brand generators. Schmid gave RJL its
Quotation dated August 19, 1975 stipulating that payment would be made by confirming an irrevocable
letter of credit in favor of NAGATA CO . Agreeing with the terms of the Quotation, RJL opened a letter of

359 | P a g e
Law 321_Corporation LAW_ Case Digest

credit. SCHMID transmitted to NAGATA CO. an order for the twelve (12) generators to be shipped
directly to RJL MARTINEZ. NAGATA CO. thereafter sent RJL MARTINEZ the bill of lading and its own
invoice and, in accordance with the order, shipped the generators directly to RJL MARTINEZ.
All generators subject of the two transactions burned out after continuous use. SCHMID, upon
information from RJL, brought the matter to the attention of NAGATA CO. In July 1976, NAGATA CO.
sent two technical representatives who made an ocular inspection and conducted tests on some of the
burned out generators. The tests revealed that the generators were overrated.
As indicated both in the quotation and in the invoice, the capacity of a generator was supposed
to be 5 KVA, however, it turned out that the actual capacity was only 4 KVA. The Japanese technicians
advised RJL MARTINEZ to ship three (3) generators to Japan, which the company did.

ISSUE:

Whether or not the transaction between the parties was an indent, not sale.

RULING:

YES.

There is no statutory definition of "indent" in this jurisdiction. However, the Rules and
Regulations to Implement Presidential Decree No. 1789 states that foreign firm which does business
through the middlemen acting in their own names, such as indentors, commercial brokers or
commission merchants, shall not be deemed doing business in the Philippines. But such indentors,
commercial brokers or commission merchants shall be the ones deemed to be doing business in the
Philippines.
Therefore, an indentor is a middleman in the same class as commercial brokers and commission
merchants. It would appear that there are three parties to an indent transaction, namely, the buyer, the
indentor, and the supplier who is usually a non-resident manufacturer residing in the country where
the goods are to be bought. An indentor may therefore be best described as one who, for compensation,
acts as a middleman in bringing about a purchase and sale of goods between a foreign supplier and a
local purchaser.
In the case at bar, the admissions of the parties and the facts appearing on record more than
suffice to warrant the conclusion that SCHMID was not a vendor, but was merely an indentor, in the
questioned transaction. The afore-quoted penal provision in the Corporation Law finds no application to
SCHMID and its officers and employees relative to the transactions in the instant case.
An indentor, acting in his own name, is not, however, covered by the above-quoted provision. In
fact, the provision of the Rules and Regulations implementing the Omnibus Investments Code quoted
above, which was copied from the Rules implementing Republic Act No. 5455, recognizes the distinct
role of an indentor, such that when a foreign corporation does business through such indentor, the
foreign corporation is not deemed doing business in the Philippines. Not being the vendor, SCHMID
cannot be held liable for the implied warranty for hidden defects under the Civil Code.

 “Doing business” With or Without License: Suits By or Against Foreign


Corporation

STEELCASE, INC.
vs.
DESIGN INTERNATIONAL SELECTIONS, INC.
G.R. No. 171995 April 18, 2012

FACTS:

Petitioner Steelcase, Inc. is a foreign corporation existing under the laws of Michigan, United
States of America (U.S.A.), and engaged in the manufacture of office furniture with dealers worldwide.
Respondent Design International Selections, Inc. ("DISI") is a corporation existing under Philippine Laws
and engaged in the furniture business, including the distribution of furniture.
Sometime in 1986 or 1987, Steelcase and DISI orally entered into a dealership agreement
whereby Steelcase granted DISI the right to market, sell, distribute, install, and service its products to
end-user customers within the Philippines. The business relationship continued smoothly until it was
terminated sometime in January 1999 after the agreement was breached with neither party admitting
any fault. Steelcase filed a complaint for sum of money against DISI alleging, among others, that DISI
had an unpaid account of US$600,000.00.

360 | P a g e
Law 321_Corporation LAW_ Case Digest

ISSUE:

Whether or not Steelcase is not doing business in the Philippines without license.

RULING:

YES.

Based on this list, the Supreme Court said that the appointment of a distributor in the
Philippines is not sufficient to constitute "doing business" unless it is under the full control of the
foreign corporation. If the distributor is an independent entity which buys and distributes products,
other than those of the foreign corporation, for its own name and its own account, the latter cannot be
considered to be doing business in the Philippines.
Applying these rules, the Supreme Court said that DISI was founded in 1979 and is
independently owned and managed. In addition to Steelcase products, DISI also distributed products of
other companies including carpet tiles, relocatable walls and theater settings. The dealership agreement
between Steelcase and DISI had been described by the owner himself as a buy and sell arrangement.
This clearly belies DISI’s assertion that it was a mere conduit through which Steelcase conducted its
business in the country. From the preceding facts, the only reasonable conclusion that can be reached
is that DISI was an independent contractor, distributing various products of Steelcase and of other
companies, acting in its own name and for its own account. As a result, Steelcase cannot be considered
to be doing business in the Philippines by its act of appointing a distributor as it falls under one of the
exceptions under R.A. No. 7042.
A foreign corporation doing business in the Philippines without a license may maintain suit in the
Philippines against a domestic corporation or person who is party to a contract as the domestic
corporation or person is deemed estopped from challenging the personality of the foreign corporation.

Philippine Deposit Insurance Corporation


vs.
Citibank
GR NO.170290 April 11, 2012

FACTS:

Petitioner Philippine Deposit Insurance Corporation (PDIC) is a government instrumentality


created by virtue of Republic Act (R.A.) No. 3591, as amended by R.A. No.9302.Respondent Citibank,
N.A. (Citibank) is a banking corporation while respondent Bank of America, S.T. & N.A. (BA) is a
national banking association, both of which are dulyorganized and existing under the laws of the United
States of America and duly licensed todo business in the Philippines, with offices in Makati City.In
1977, PDIC conducted an examination of the books of account of Citibank. It discovered that Citibank,
in the course of its banking business, from September 30, 1974 toJune 30, 1977, received from its head
office and other foreign branches a totalof P11,923,163,908.00 in dollars, covered by Certificates of
Dollar Time Deposit that wereinterest-bearing with corresponding maturity dates. These funds, which
were lodged inthe books of Citibank under the account “Their Account -Head Office/Branches-Foreign
Currency,” were not reported to PDIC as deposit liabilities that were subject to assessment for
insurance. As such, in a letter dated March 16, 1978, PDIC assessed Citibank fordeficiency in the sum
of P1,595,081.96.Similarly, sometime in 1979, PDIC examined the books of accounts of BA
whichrevealed that from September 30, 1976 to June 30, 1978, BA received from its head officeand its
other foreign branches a total of P629,311,869.10 in dollars, covered by Certificatesof Dollar Time
Deposit that were interest-bearing with corresponding maturity dates and lodged in their books under
the account “Due to Head Office/Branches.”

ISSUE:

Whether or not a branch of a bank has a separate legal Personality.

361 | P a g e
Law 321_Corporation LAW_ Case Digest

RULING:

NO.

A branch has no separate legal personality. This Court is of the opinion that the key to the
resolution of this controversy is the relationship of the Philippine branches of Citibank and BA to their
respective head offices and their other foreign branches.
The Court begins by examining the manner by which a foreign corporation canestablish its
presence in the Philippines. It may choose to incorporate its own subsidiary asa domestic corporation, in
which case such subsidiary would have its own separate andindependent legal personality to conduct
business in the country. In the alternative, it maycreate a branch in the Philippines, which would not be
a legally independent unit, andsimply obtain a license to do business in the Philippines.
In the case of Citibank and BA, it is apparent that they both did not incorporate a separate
domestic corporation to represent its business interests in the Philippines. Their Philippine branches
are, as the name implies, merely branches, without a separate legal personality from their parent
company, Citibank and BA. Thus, being one and the same entity, the funds placed by the respondents
in their respective branches inthe Philippines should not be treated as deposits made by third parties
subject to deposit insurance under the PDIC Charter.
Deposit insurance is superfluous and entirely unnecessary when, as in this case, the institution
holding the funds and the one which made theplacements are one and the same legal entity.

CARGILL, INC.
vs.
INTRA STRATA ASSURANCE CORPORATION
G.R. No. 168266 March 15, 2010

FACTS:

Cargill (foreign) is a corporation organized and existing under the laws of theState of Delaware.
Cargill executed a contract with Northern Mindanao Corporation (NMC)(domestic), whereby NMC agreed
to sell to petitioner 20,000 to 24,000 metrictons of molasses to be delivered from Jan 1 to 30 1990 for
$44 per metric ton. The contract provided that CARGILL was to open a Letter of Credit with theBPI.
NMC was permitted to draw up 500,000 representing the minimum priceof the contract. The contract
was amended 3 times (in relation to the amount and the price).But the third amendment required NMC
to put up a performance bond whichwas intended to guarantee NMC’s performance to deliver the
molasses duringthe prescribed shipment periods.
In compliance, INTRA STRATA issued a performance bond to guaranteeNMC’s delivery. NMC was
only able to deliver 219551 metric tons out of the agreed 10,500.Thus CARGILL sent demand letters to
INTRA claiming payment under theperformance and surety bonds. When INTRA failed to pay, CARGILL
filed acomplaint.
CARGILL NMC and INTRA entered into a compromise agreement approvedby the court, such
provided that NMC would pay CARGILL 3 million uponsigning and would deliver to CARGILL 6,991
metric tons of molasses. ButNMC still failed to comply.

ISSUE:

Whether or not petitioner is doing or transacting business in the Philippines in contemplation of


the law and established jurisprudence.

RULING:

NO.

The determination of whether a foreign corporation is doing business in the Philippines must be
based on the facts of each case. In the case at bar, the transactions entered into by the respondent with
the petitioners are not a series of commercial dealings which signify an intent on the part of the
respondent to do business in the Philippines but constitute an isolated one which does not fall under

362 | P a g e
Law 321_Corporation LAW_ Case Digest

the category of "doing business." The records show that the only reason why the respondent entered into
the second and third transactions with the petitioners was because it wanted to recover the loss it
sustained from the failure of the petitioners to deliver the crude coconut oil under the first transaction
and in order to give the latter a chance to make good on their obligation.
In the present case, petitioner is a foreign company merely importing molasses from a Philipine
exporter. A foreign company that merely imports goods from a Philippine exporter, without opening an
office or appointing an agent in the Philippines, is not doing business in the Philippines.

SEHWANI, INC.
vs.
IN-N-OUT BURGER
GR No. 171053 15 October 2007

FACTS:

Respondent IN-N-OUT Burger, Inc., a foreign corporation organized under the laws of California,
U.S.A., and not doing business in the Philippines, filed before the Bureau of Legal Affairs of the IPO
(BLA-IPO), an administrative complaint against petitioners Sehwani, Inc. and Benita’s Frites, Inc. for
violation of intellectual property rights, attorney’s fees and damages with prayer for the issuance of a
restraining order or writ of preliminary injunction in registering, as it did, the mark claimed by
respondent.
The BLA-IPO held that petitioner is not guilty of unfair competition although respondent has the
capacity to sue and ordered petitioner to cease and desist from using the mark which the higher courts
affirmed.

ISSUE:

Whether or not respondent has legal capacity to sue.

RULING:

YES.

Respondent has the legal capacity to sue for the protection of its trademarks, albeit it is not doing
business in the Philippines. Section 160 in relation to Section 3 of R.A. No. 8293, provides:
SECTION 160. Right of Foreign Corporation to Sue in Trademark or Service Mark Enforcement
Action. — Any foreign national or juridical person who meets the requirements of Section 3 of this Act
and does not engage in business in the Philippines may bring a civil or administrative action hereunder
for opposition, cancellation, infringement, unfair competition, or false designation of origin and false
description, whether or not it is licensed to do business in the Philippines under existing laws.
Section 3 thereof provides:
SECTION 3. International Conventions and Reciprocity. — Any person who is a national or who
is domiciled or has a real and effective industrial establishment in a country which is a party to any
convention, treaty or agreement relating to intellectual property rights or the repression of unfair
competition, to which the Philippines is also a party, or extends reciprocal rights to nationals of the
Philippines by law, shall be entitled to benefits to the extent necessary to give effect to any provision of
such convention, treaty or reciprocal law, in addition to the rights to which any owner of an intellectual
property right is otherwise entitled by this Act.
The question of whether or not respondent’s trademarks are considered “well-known” is factual in
nature, involving as it does the appreciation of evidence adduced before the BLA-IPO. The settled rule is
that the factual findings of quasi-judicial agencies, like the IPO, which have acquired expertise because
their jurisdiction is confined to specific matters, are generally accorded not only respect, but, at times,
even finality if such findings are supported by substantial evidence.

363 | P a g e
Law 321_Corporation LAW_ Case Digest

This being the case, the issue on the legal capacity to sue of the respondent is answered in the
affirmative.

LORENZO SHIPPING CORP.


vs. CHUBB and SONS, Inc. et al
G.R. No. 147724 June 8, 2004

FACTS:

Lorenzo Shipping Corporation is a domestic corporation engaged in coastwise shipping. Gearbulk


ltd. is a foreign corporation licensed as a common carrier under the laws of Norway and doing business
in the Philippines through its agent Philippine Transmarine Carrier Inc. Philippine Transmarine Carrier
(PTC) is the agent of Gearbulk in the Philippines. Sumitomo Corporation (SC), a foreign corporation
organized under the laws of the USA is the consignee. It insured the shipment with Chubb and Sons,
Inc., a foreign corporation organized and licensed to engage in insurance business under the laws of the
United States of America.
On November 21, 1987, Mayer Steel Pipe Corporation of Binondo, Manila, loaded 581 bundles of
black steel on board one of the vessels of LS for shipment to Davao City from Manila. Upon reaching the
Sasa Wharf in Davao on Dec.2, PTC discovered that the steels were submerged in seawater. An
inspector made a finding that the steel pipes were no longer in good condition for rust were already
forming.
Gearbulk noted the damage and shipped the pipes on its vessel to the USA . Due to its heavily
rusted condition, the consignee Sumitomo rejected the damaged steel pipes and declared them unfit for
the purpose they were intended. It then filed a marine insurance claim with respondent Chubb and
Sons, Inc.

ISSUE:

Whether or not Chubb and Sons, as a mere assignee of a foreign corporation which has no
authority to sue in the Philippines, has capacity to sue before the Philippine courts.

RULING:

YES.

Subrogration contemplates full substitution such that it places the party subrogated in the shoes
of the creditor, and he may use all means which the creditor could employ to enforce payment. The
rights to which the subrogee succeeds are the same as, but not greater than, those of the person for
whom he is substituted. The law on corporations is clear in depriving foreign corporations which are
doing business in the Philippines without a license from bringing or maintaining actions before, or
intervening in Philippine courts but a foreign corporation needs no license to sue before Philippine
courts on an isolated transaction.
For "doing business" is not really the number or the quantity of the transactions, but more
importantly, the intention of an entity to continue the body of its business in the country. The phrase
"isolated transaction" has a definite and fixed meaning, i.e. a transaction or series of transactions set
apart from the common business of a foreign enterprise in the sense that there is no intention to engage
in a progressive pursuit of the purpose and object of the business organization. Whether a foreign
corporation is "doing business" does not necessarily depend upon the frequency of its transactions, but
more upon the nature and character of the transactions. Furthermore, respondent insurer Chubb and
Sons, by virtue of the right of subrogation provided for in the policy of insurance, is the real party in
interest in the action for damages before the court a quo against the carrier Lorenzo Shipping to recover
for the loss sustained by its insured.
It then, thus possesses the right to enforce the claim and the significant interest in the litigation.
In the case at bar, it is clear that respondent insurer was suing on its own behalf in order to enforce its
right of subrogation.

364 | P a g e
Law 321_Corporation LAW_ Case Digest

MR HOLDINGS, Ltd.
vs.
BAJAR
G.R. No. 138104 April 11, 2002

FACTS:

ADB extended a loan to Marcopper under a Principal Loan Agreement and Complementary Loan
Agreement. A Support and Standby Credit Agreement was also executed between ADB and Placer Dome
(owner of 40% of Marcopper), whereby the latter agreed to provide with a cash flow support for the
payment of its obligations to ADB. Marcopper also executed a Deed of Real Estate and Chattel Mortgage
in favor of ADB covering all its assets in Marinduque. Marcopper defaulted in its payment. Thus, MR
Holding, LTD (placer Dome’s subsidiary corporation) assumed Marcopper’s obligation to ADB.
Marcopper likewise executed a Deed of assignment in favor of petitioner.
It appeared that SolidBank Corporation obtained a partial judgment against Marcopper in a case
filed with the RTC. A writ of execution was issued and then an auction sale was scheduled. This event
prompted petitioner to serve an "Affidavit of Third-Party Claim" upon respondent sheriffs, asserting
ownership over all the assets of Marcopper by virtue of the Deed of Assignment. The RTC of Manila
denied the affidavit.
Petitioner filed with the RTC of Boac, Marinduque a complaint for reivindication of properties with
prayer for preliminary injunction and temporary restraining order against respondents. The application
for writ of preliminary injunction was denied.

ISSUE:

Whether or not petitioner is doing business in the Philippines.

RULING:

NO.

There are other statutes defining the term "doing business" in the same tenor as those above-
quoted, and as may be observed, one common denominator among them all is the concept of
"continuity."
The expression "doing business" should not be given such a strict and literal construction as to
make it apply to any corporate dealing whatever. At this early stage and with petitioner’s acts or
transactions limited to the assignment contracts, it cannot be said that it had performed acts intended
to continue the business for which it was organized. It may not be amiss to point out that the purpose
or business for which petitioner was organized is not discernible in the records. No effort was exerted by
the Court of Appeals to establish the nexus between petitioner’s business and the acts supposed to
constitute "doing business." Thus, whether the assignment contracts were incidental to petitioner’s
business or were continuation thereof is beyond determination.
Significantly, a view subscribed upon by many authorities is that the mere ownership by a foreign
corporation of a property in a certain state, unaccompanied by its active use in furtherance of the
business for which it was formed, is insufficient in itself to constitute doing business.
In the final analysis, we are convinced that petitioner was engaged only in isolated acts or
transactions. Single or isolated acts, contracts, or transactions of foreign corporations are not regarded
as a doing or carrying on of business. Typical examples of these are the making of a single contract,
sale, sale with the taking of a note and mortgage in the state to secure payment therefor, purchase, or
note, or the mere commission of a tort. In these instances, there is no purpose to do any other business
within the country.

THE COMMISSIONER OF CUSTOMS


vs.
K.M.K. GANI, INDRAPAL & CO
G.R. No. 73722 February 26, 1990

FACTS:

365 | P a g e
Law 321_Corporation LAW_ Case Digest

Two containers loaded with 103 cartons of merchandise covered by eleven (11) airway bills of
several supposedly Singapore-based consignees arrived at the Manila International Airport on board
Philippine Air Lines (PAL) from Hongkong. The cargoes were consigned to these different entities: K.M.K
Gani (hereafter referred to as K.M.K.) and Indrapal and Company (hereafter referred to as INDRAPAL),
the private respondents in the petition before us; and Sin Hong Lee Trading Co., Ltd., AAR TEE
Enterprises, and C. Ratilal, all purportedly based in Singapore.
The Bureau's agency on such matters, the Suspected Cargo and Anti-Narcotics (SCAN),
dispatched an agent to verify the information. The SCAN agent saw an empty PAL van parked directly
alongside the plane's belly from which cargoes were being unloaded. When the SCAN agent asked the
van's driver why he was at the site, the driver drove away in his vehicle. The SCAN agent then
sequestered the unloaded cargoes.
These goods were transferred to the International Cargo Terminal under Warrant of Seizure and
Detention and thereafter subjected to Seizure and Forfeiture proceedings for "technical smuggling.

ISSUE:

Whether or not the private respondents failed to establish their personality to sue in a
representative capacity, hence making their action dismissible.

RULING:

YES.

The law is clear: "No foreign corporation transacting business in the Philippines without a license,
or its successors or assigns, shall be permitted to maintain or intervene in any action, suit or proceeding
in any court or administrative agency of the Philippines; but such corporation may be sued or proceeded
against before Philippine courts or administrative tribunals on any valid cause of action recognized
under Philippine laws."
However, the Court in a long line of cases has held that a foreign corporation not engaged in
business in the Philippines may not be denied the right to file an action in the Philippine courts for an
isolated transaction. The fact that a foreign corporation is not doing business in the Philippines must be
disclosed if it desires to sue in Philippine courts under the "isolated transaction rule." Without this
disclosure, the court may choose to deny it the right to sue.
In the case at bar, the private respondents K.M.K. and INDRAPAL aver that they are "suing upon
a singular and isolated transaction." But they failed to prove their legal existence or juridical personality
as foreign corporations.
Under the "isolated transaction rule," only foreign corporations and not just any business
organization or entity can avail themselves of the privilege of suing before Philippine courts even without
a license. The first paragraph of their petition before the Court, containing the allegation of their
identities, does not even aver their corporate character. On the contrary, K.M.K. alleges that it is a
"single proprietorship" while INDRAPAL hides under the vague identification as a "firm," although both
describe themselves with the phrase "doing business in accordance with the laws of Singapore."
Absent such proof that the private respondents are corporations (foreign or not), the respondent
Court of Tax Appeals should have barred their invocation of the right to sue within Philippine
jurisdiction under the "isolated transaction rule" since they do not qualify for the availment of such
right.

COMMUNICATION MATERIALS and DESIGN, INC.


vs.
COURT OF APPEALS
G.R. No. 102223 August 22, 1996

FACTS:

Petitioners COMMUNICATION MATERIALS AND DESIGN, INC., (CMDI, for brevity) and ASPAC
MULTI-TRADE INC., (ASPAC, for brevity) are both domestic corporations, while petitioner Francisco S.
Aguirre is their President and majority stockholder. Private Respondents ITEC, INC. and/or ITEC,
INTERNATIONAL, INC. (ITEC, for brevity) are corporations duly organized and existing under the laws of
the State of Alabama, United States of America. There is no dispute that ITEC is a foreign corporation
not licensed to do business in the Philippines.
ITEC entered into a contract with petitioner ASPAC referred to as "Representative Agreement".
ITEC engaged ASPAC as its "exclusive representative" in the Philippines for the sale of ITEC's products,

366 | P a g e
Law 321_Corporation LAW_ Case Digest

in consideration of which, ASPAC was paid a stipulated commission. ASPAC was able to incorporate and
use the name "ITEC" in its own name. Thus , ASPAC Multi-Trade, Inc. became legally and publicly
known as ASPAC-ITEC (Philippines).
ASPAC sold electronic products, exported by ITEC, to their sole customer, the Philippine Long
Distance Telephone Company, (PLDT, for brevity). ITEC charges the petitioners and another Philippine
Corporation, DIGITAL BASE COMMUNICATIONS, INC. (DIGITAL, for brevity), the President of which is
likewise petitioner Aguirre, of using knowledge and information of ITEC's products specifications to
develop their own line of equipment and product support, which are similar, if not identical to ITEC's
own, and offering them to ITEC's former customer.

ISSUE:

Whether or not private respondent ITEC is an unlicensed corporation doing business in the
Philippines, and if it is, whether or not this fact bars it from invoking the injunctive authority of our
courts.

RULING:

YES.

Generally, a "foreign corporation" has no legal existence within the state in which it is foreign.
This proceeds from the principle that juridical existence of a corporation is confined within the territory
of the state under whose laws it was incorporated and organized, and it has no legal status beyond such
territory. Such foreign corporation may be excluded by any other state from doing business within its
limits, or conditions may be imposed on the exercise of such privileges.
The purpose of the law in requiring that foreign corporations doing business in the Philippines be
licensed to do so and that they appoint an agent for service of process is to subject the foreign
corporation doing business in the Philippines to the jurisdiction of its courts. The object is not to
prevent the foreign corporation from performing single acts, but to prevent it from acquiring a domicile
for the purpose of business without taking steps necessary to render it amenable to suit in the local
courts.
The implication of the law is that it was never the purpose of the legislature to exclude a foreign
corporation which happens to obtain an isolated order for business from the Philippines, and thus, in
effect, to permit persons to avoid their contracts made with such foreign corporations.
With the abovestated precedents in mind, we are persuaded to conclude that private respondent
had been "engaged in" or "doing business" in the Philippines for some time now. This is the inevitable
result after a scrutiny of the different contracts and agreements entered into by ITEC with its various
business contacts in the country, particularly ASPAC and Telephone Equipment Sales and Services, Inc.

COLUMBIA PICTURES, INC.


vs.
COURT OF APPEALS
G.R. No. 110318 August 28, 1996

FACTS:

Complainants thru counsel lodged a formal complaint with the NBI for violation of PD No. 49 and
sought its assistance in their anti-film piracy drive. Agents of the NBI and private researchers made
discreet surveillance on various video establishments in Metro Manila including Sunshine Home Video
Inc., owned and operated by Danilo A. Pelindario.
NBI Senior Agent Lauro C. Reyes applied for a search warrant with the court a quo against
Sunshine seeking the seizure, among others, of pirated video tapes of copyrighted films all of which were
enumerated in a list attached to the application; and, television sets, video cassettes and/or laser disc
recordings equipment and other machines and paraphernalia used or intended to be used in the
unlawful exhibition, showing, reproduction, sale, lease or disposition of videograms tapes in the
premises above described.
The search warrant was served to Sunshine and/or their representatives. In the course of the
search of the premises indicated in the search warrant, the NBI Agents found and seized various video
tapes of duly copyrighted motion pictures/films owned or exclusively distributed by private
complainants, and machines, equipment, television sets, paraphernalia, materials, accessories all of
which were included in the receipt for properties accomplished by the raiding team. Copy of the receipt
was furnished and/or tendered to Mr. Danilo A. Pelindario, registered owner-proprietor of Sunshine
Home Video.

367 | P a g e
Law 321_Corporation LAW_ Case Digest

ISSUE:

Whether or not petitioners have capacity to sue in court.

RULING:

NO.

The obtainment of a license prescribed by Section 125 of the Corporation Code is not a condition
precedent to the maintenance of any kind of action in Philippine courts by a foreign corporation.
As thus interpreted, any foreign corporation not doing business in the Philippines may maintain
an action in our courts upon any cause of action, provided that the subject matter and the defendant
are within the jurisdiction of the court. It is not the absence of the prescribed license but “doing
business” in the Philippines without such license which debars the foreign corporation from access to
our courts.
In other words, although a foreign corporation is without license to transact business in the
Philippines, it does not follow that it has no capacity to bring an action. Such license is not necessary if
it is not engaged in business in the Philippines. Based on Article 133 of the Corporation Code and
gauged by such statutory standards, petitioners are not barred from maintaining the present action.
There is no showing that, under our statutory or case law, petitioners are doing, transacting, engaging
in or carrying on business in the Philippines as would require obtention of a license before they can seek
redress from our courts. No evidence has been offered to show that petitioners have performed any of
the enumerated acts or any other specific act indicative of an intention to conduct or transact business
in the Philippines.
Accordingly, the certification issued by the Securities and Exchange Commission stating that its
records do not show the registration of petitioner film companies either as corporations or partnerships
or that they have been licensed to transact business in the Philippines, while undeniably true, is of no
consequence to petitioners’ right to bring action in the Philippines.

ERIKS PTE. LTD.


vs.
COURT OF APPEALS
G.R. No. 118843 February 6, 1997

FACTS:

Petitioner is a non-resident foreign corporation engaged in the manufacture and sale of elements
used in sealing pumps, valves and pipes for industrial purposes, valves and control equipment used for
industrial fluid control and PVC pipes and fittings for industrial uses.
On various dates, private respondent Delfin Enriquez, Jr., doing business under the name and
style of Delrene EB Controls Center and/or EB Karmine Commercial, ordered and received from
petitioner various elements used in sealing pumps, valves, pipes and control equipment, PVC pipes and
fittings.
The transfers of goods were perfected in Singapore, for private respondent's account, F.O.B.
Singapore, with a 90-day credit term. Subsequently, demands were made by petitioner upon private
respondent to settle his account, but the latter failed/refused to do so.

ISSUE:

Whether Petitioner Corporation may maintain an action in Philippine courts considering that it
has no license to do business in the country.

RULING:

YES.

The resolution of this issue depends on whether petitioner's business with private respondent
may be treated as isolated transactions. Granting that there is no distributorship agreement between
herein parties, yet by the mere fact that plaintiff, each time that the defendant posts an order delivers
the items as evidenced by the several invoices and receipts of various dates only indicates that plaintiff
has the intention and desire to repeat the said transaction in the future in pursuit of its ordinary
business. Furthermore, "and if the corporation is doing that for which it was created, the amount or

368 | P a g e
Law 321_Corporation LAW_ Case Digest

volume of the business done is immaterial and a single act of that character may constitute doing
business.
More than the sheer number of transactions entered into, a clear and unmistakable intention on
the part of petitioner to continue the body of its business in the Philippines is more than apparent. As
alleged in its complaint, it is engaged in the manufacture and sale of elements used in sealing pumps,
valves, and pipes for industrial purposes, valves and control equipment used for industrial fluid control
and PVC pipes and fittings for industrial use. Thus, the sale by petitioner of the items covered by the
receipts, which are part and parcel of its main product line, was actually carried out in the progressive
prosecution of commercial gain and the pursuit of the purpose and object of its business, pure and
simple.
Further, its grant and extension of 90-day credit terms to private respondent for every purchase
made, unarguably shows an intention to continue transacting with private respondent, since in the
usual course of commercial transactions, credit is extended only to customers in good standing or to
those on whom there is an intention to maintain long-term relationship.
This being so, the existence of a distributorship agreement between the parties, as alleged but not
proven by private respondent, would, if duly established by competent evidence, be merely
corroborative, and failure to sufficiently prove said allegation will not significantly affect the finding of
the courts below.
Thus, the court holds that the series of transactions in question could not have been isolated or
casual transactions. What is determinative of "doing business" is not really the number or the quantity
of the transactions, but more importantly, the intention of an entity to continue the body of its business
in the country. The number and quantity are merely evidence of such intention.
FAR EAST INTERNATIONAL
vs.
NANKAI KOGYO
L- 13525 NOVEMBER 3, 1962

FACTS:

Far East entered into a contract of sale of steel scrap with Nankai , a foreign corporation
incorporated under Japanese laws. Nankai opened a letter of credit with the China Banking Corporation
issued by Nippon Kangyo. Four days before the expiration of the Far East license, three boats of Nankai
arrived. Upon the expiration of the export license, only 1, 058.6 metric tons of steel scrap was loaded on
the SS Mina. The license was never renewed.
On April 27, 1957, Nankai confirmed and acknowleged delivery of the 1,058.6 metric tons of steel
scrap, but asked for damages amounting to $148,135.00 consisting of dead freight charges, damages,
bank charges, phone and cable expenses.
On May 4, 1957, Far East wrote the Everett Steamship Corporation, requesting the issuance of a
complete set of the Bill of Lading for the shipment, in order that payment thereof be effected against the
Letter of Credit. Under date of May 7, 1957, the Everett informed Far East that they were not in a
position to comply because the Bill of Lading was issued and signed in Tokyo by the Master of the boat,
upon request of the Charterer, defendant herein.
As repeated requests, both against the shipping agent and the buyers (Nankai), for the issuance
of the of Bill Lading were ignored, Far East filed on May 16, 1957, the present complaint for Specific
Performance, damages, a writ of preliminiry mandatory injunction directed against Nankai and the
shipping company, to issue and deliver to the plaintiff, a complete set of negotiable of Lading for the
1,058.6 metric tons of scrap and a writ of preliminary injunction against the China Banking Corporation
and the Nankai to maintain the Letter Credit.
Far East filed for the issuance of a complete set of Bill of Lading in order that payment be effected
against the Letter of Credit. The respondent refused.

ISSUE:

Whether or not the court had jurisdiction over the subject matter and the person of the
defendant.

RULING:

YES.

Mr. Ishida who personally signed the contract for the purpose of selling scrap in question in
behalf on Nankai Kogyo is the Trade Manager of the said corporation. Mr. Tominaga was the Chief of the

369 | P a g e
Law 321_Corporation LAW_ Case Digest

Petroleum Section of the same company and Mr. Yoshida was the man- in- charge of the Import Section
of the company’s Tokyo branch. All of these officers are served with summons.
The testimony of Atty. Pablo Ocampo that appellant was doing business in the Philippines was
corroborated by no less than Nabuo Yoshida, one of the appellant’s officers, that he was sent to the
Philippines by his company to look into the operation of mines, thereby revealing the defendant’s desire
to continue engaging business in the Philippines, after receiving the shipment of the scrap iron under
consideration, making the Philippines a base thereof.
That a single act may bring the corporation within the purview of the statute where it is an act of
the ordinary business of the corporation. The single act or transaction is not merely incidental or casual,
but is of such character as distinctly to indicate a purpose on the part of the foreign corporation to do
other business in the state, and to make the state a basis of operations for the conduct of a part of the
corporation’s ordinary business.

FACILITIES MANAGEMENT CORPORATION


vs.
DE LA OSA
G.R. No. L-38649 March 26, 1979

FACTS:

Leonardo dela Osa sought his reinstatement with full backwages, as well as the recovery of his
overtime compensation, swing shift and graveyard shift differentials. Petitioner alleged that he was
employed by respondents as follows: (1) painter (2) houseboy with an hourly rate of $1.26 from
December, 1964 to November, 1965, inclusive; (3) houseboy with an hourly rate of $1.33 from
December, 1965 to August, 1966, inclusive; and (4) cashier with an hourly rate of $1.40 from August,
1966 to March 27, 1967, inclusive.
He further averred that from December, 1965 to August, 1966, inclusive, he rendered overtime
services daily, and that this entire period was divided into swing and graveyard shifts to which he was
assigned, but he was not paid both overtime and night shift premiums despite his repeated demands
from respondents.
Respondents filed on August 7, 1967 their letter- answer without substantially denying the
material allegations of the basic petition but interposed the following special defenses, namely: That
respondents Facilities Management Corporation and J. S. Dreyer are domiciled in Wake Island which is
beyond the territorial jurisdiction of the Philippine Government; that respondent J. V. Catuira, though
an employee of respondent corporation presently stationed in Manila, is without power and authority of
legal representation; and that the employment contract between petitioner and respondent corporation
carries -the approval of the Department of Labor of the Philippines.

ISSUE:

Whether or not the mere act by a non-resident foreign corporation of recruiting Filipino workers
for its own use abroad is in law doing business in the Philippines.

RULING:

YES.

"Under the rules and regulations promulgated by the Board of Investments which took effect Feb.
3, 1969, implementing Rep. Act No. 5455, which took effect Sept. 30, 1968, the phrase 'doing business'
has been exemplified with illustrations, among them being as follows: "the performance within the
Philippines of any act or combination of acts enumerated in section 1(1) of the Act shall constitute 'doing
business' therein. In particular, 'doing business' includes: "(1)Soliciting orders, purchases (sales) or
service contracts. Concrete and specific solicitations by a foreign firm, not acting independently of the
foreign firm, amounting to negotiation or fixing of the terms and conditions of sales or service contracts,
regardless of whether the contracts are actually reduced to writing, shall constitute doing business even
if the enterprise has no office or fixed place of business in the Philippines. "(2)Appointing a
representative or distributor who is domiciled in the Philippines, unless said representative or
distributor has an independent status, i.e., it transacts business in its name and for its own account,
and not in the name or for the account of the principal. "(4)Opening offices, whether called 'liaison'
offices, agencies or branches, unless proved otherwise. "(10)Any other act or acts that imply a continuity
of commercial dealings or arrangements, and contemplate to that extent the performance of acts or

370 | P a g e
Law 321_Corporation LAW_ Case Digest

works, or the exercise of some of the functions normally incident to, or in the progressive prosecution of,
commercial gain or of the purpose and objective of the business organization."

H.B. ZACHRY COMPANY INTERNATIONAL


vs.
COURT OF APPEALS
G.R. No. 106989 May 10, 1994

FACTS:

VBC entered into a written Subcontract Agreement with Zachry, a foreign corporation. The latter
had been engaged by the United States Navy to design and construct 264 Family Housing Units at the
US Naval Base at Subic, Zambales. Under the agreement, specifically under Section 3 on Payment, VBC
was to perform all the construction work on the housing project and would be paid "for the performance
of the work the sum (U.S.$6,468,000.00)
When VBC had almost completed the project, Zachry complained of the quality of work, making it
a reason for its decision to take over the management of the project, which paragraph c, Section 7 of the
Subcontract Agreement authorized.
However, prior to such take-over, the parties executed an agreement, in accordance with which,
VBC submitted to Zachry a detailed computation of the cost to complete the subcontract on the housing
project. According to VBC's computation, there remains a balance of $1,103,000.00 due in its favor as of
18 January 1990. Zachry, however, not only refused to acknowledge the indebtedness but continually
failed to submit to VBC a statement of accumulated costs, as a result of which VBC was prevented from
cheking the accuracy of the said costs. On 2 March 1990, VBC wrote Zachry a letter demanding
compliance with its obligations. Zachry still failed to do so. VBC made representations to pursue its
claim, including a formal claim with the Officer-in-Charge of Construction, NAVFAC Contracts,
Southwest Pacific, which also failed.

ISSUE:

Whether or not a writ of preliminary attachment may issue ex parte against a defendant before
acquisition of jurisdiction of the latter's person by service of summons or his voluntary submission to
the Court's authority.

RULING:

NO.

The Court rules that the question must be answered in the affirmative and that consequently, the
petition for review will have to be granted. It is incorrect to theorize that after an action or proceeding
has been commenced and jurisdiction over the person of the plaintiff has been vested in the court, but
before the acquisition of jurisdiction over the person of the defendant (either by service of summons or
his voluntary submission to the court's authority), nothing can be validly done by the plaintiff or the
court.
It is wrong to assume that the validity of acts done during this period should be dependent on, or
held in suspension until, the actual obtention of jurisdiction over the defendant's person. The obtention
by the court of jurisdiction over the person of the defendant is one thing; quite another is the acquisition
of jurisdiction over the person of the plaintiff or over the subject-matter or nature of the action, or the
res or object thereof.
For the guidance of all concerned, the Court reiterates and reaffirms the proposition that writs of
attachment may properly issue ex parte provided that the Court is satisfied that the relevant requisites
therefore have been fulfilled by the applicant, although it may, in its discretion, require prior hearing on
the application with notice to the defendant; but that levy on property pursuant to the writ thus issued
may not be validly effected unless preceded, or contemporaneously accompanied, by service on the
defendant of summons, a copy of the complaint (and of the appointment of guardian ad litem, if any),
the application for attachment (if not incorporated in but submitted separately from the complaint), the
order of attachment, and the plaintiff's attachment bond.
HUTCHISON PORTS PHILIPPINES LIMITE
vs.
SBMA

371 | P a g e
Law 321_Corporation LAW_ Case Digest

G.R. No. 131367 August 31, 2000

FACTS:

The Subic Bay Metropolitan Authority (or SBMA) advertised in leading national daily newspapers
and in one international publication, an invitation offering to the private sector the opportunity to
develop and operate a modern marine container terminal within the Subic Bay Freeport Zone. Out of
seven bidders who responded to the published invitation, three were declared by the SBMA as qualified
bidders after passing the pre-qualification evaluation conducted by the SBMA’s Technical Evaluation
Committee (or SBMA-TEC). Among these is the petitioner.
Thereafter, the services of three (3) international consultants recommended by the World Bank for
their expertise were hired by SBMA to evaluate the business plans submitted by each of the bidders,
and to ensure that there would be a transparent and comprehensive review of the submitted bids. The
SBMA also hired the firm of Davis, Langdon and Seah Philippines, Inc. to assist in the evaluation of the
bids and in the negotiation process after the winning bidder is chosen. All the consultants, after such
review and evaluation unanimously concluded that HPPL’s Business Plan was “far superior to that of
the two other bidders.”
However, even before the sealed envelopes containing the bidders’ proposed royalty fees could be
opened at the appointed time and place, RPSI formally protested that ICTSI is legally barred from
operating a second port in the Philippines based on Executive Order No. 212 and Department of
Transportation and Communication (DOTC) Order 95-863.

ISSUE:

Whether or not petitioner HPPL has the legal capacity to even seek redress from the Court.

RULING:

YES.

Admittedly, petitioner HPPL is a foreign corporation, organized and existing under the laws of the
British Virgin Islands. While the actual bidder was a consortium composed of petitioner, and two other
corporations, namely, Guoco Holdings (Phils.) Inc. and Unicol Management Services, Inc., it is only
petitioner HPPL that has brought the controversy before the Court, arguing that it is suing only on an
isolated transaction to evade the legal requirement that foreign corporations must be licensed to do
business in the Philippines to be able to file and prosecute an action before Philippines courts.
There is no general rule or governing principle laid down as to what constitutes “doing” or
“engaging in” or “transacting” business in the Philippines. Each case must be judged in the light of its
peculiar circumstances. Thus, it has often been held that a single act or transaction may be considered
as “doing business” when a corporation performs acts for which it was created or exercises some of the
functions for which it was organized. The amount or volume of the business is of no moment, for even a
singular act cannot be merely incidental or casual if it indicates the foreign corporation’s intention to do
business.
Participating in the bidding process constitutes “doing business” because it shows the foreign
corporation’s intention to engage in business here. The bidding for the concession contract is but an
exercise of the corporation’s reason for creation or existence. Thus, it has been held that “a foreign
company invited to bid for IBRD and ADB international projects in the Philippines will be considered as
doing business in the Philippines for which a license is required.”

LA CHEMISE LACOSTE, S. A.
vs.
FERNANDEZ
G.R. No. L-65659 May 21, 1984

FACTS:

The petitioner is a foreign corporation, organized and existing under the laws of France and not
doing business in the Philippines. It is the actual owner of the abovementioned trademarks used on
clothings and other goods specifically sporting apparels sold in many parts of the world and which have
been marketed in the Philippines since 1964. The main basis of the private respondent's case is its
claim of alleged prior registration.

372 | P a g e
Law 321_Corporation LAW_ Case Digest

In 1975, Hemandas & Co., a duly licensed domestic firm applied for and was issued Reg. No. SR-
2225 (SR stands for Supplemental Register) for the trademark "CHEMISE LACOSTE & CROCODILE
DEVICE" by the Philippine Patent Office for use on T-shirts, sportswear and other garment products of
the company. Two years later, it applied for the registration of the same trademark under the Principal
Register. The Patent Office eventually issued an order which granted the application."Thereafter,
Hemandas & Co. assigned to respondent Gobindram Hemandas all rights, title, and interest in the
trademark "CHEMISE LACOSTE & DEVICE".
The petitioner filed its application for registration of the trademark "Crocodile Device" and
"Lacoste". The former was approved for publication while the latter was opposed by Games and
Garments. The petitioner filed with the National Bureau of Investigation (NBI) a letter-complaint alleging
therein the acts of unfair competition being committed by Hemandas and requesting their assistance in
his apprehension and prosecution.

ISSUE:

Whether or not petitioner has the capacity to sue.

RULING:

YES.

The petitioner is a foreign corporation not doing business in the Philippines. The marketing of its
products in the Philippines is done through an exclusive distributor, Rustan Commercial Corporation.
The latter is an independent entity which buys and then markets not only products of the petitioner but
also many other products bearing equally well-known and established trademarks and tradenames. In
other words, Rustan is not a mere agent or conduit of the petitioner.
The court finds and concludes that the petitioner is not doing business in the Philippines. Rustan
is actually a middleman acting and transacting business in its own name and or its own account and
not in the name or for the account of the petitioner. More important is the nature of the case which led
to this petition. What preceded this petition for certiorari was a letter-complaint filed before the NBI
charging Hemandas with a criminal offense, i.e., violation of Article 189 of the Revised Penal Code. If
prosecution follows after the completion of the preliminary investigation being conducted by the Special
Prosecutor the information shall be in the name of the People of the Philippines and no longer the
petitioner which is only an aggrieved party since a criminal offense is essentially an act against the
State.
It is the latter which is principally the injured party although there is a private right violated.
Petitioner's capacity to sue would become, therefore, of not much significance in the main case. We
cannot allow a possible violator of our criminal statutes to escape prosecution upon a far-fetched
contention that the aggrieved party or victim of a crime has no standing to sue. In upholding the right of
the petitioner to maintain the present suit before our courts for unfair competition or infringement of
trademarks of a foreign corporation, we are moreover recognizing our duties and the rights of foreign
states under the Paris Convention for the Protection of Industrial Property to which the Philippines and
France are parties.
MARUBENI NEDERLAND B.V.
vs.
TENSUAN
G.R. NO. 61950 SEPTEMBER 28, 1990

FACTS:

On October 23, 1976, in Tokyo, Japan, petitioner Marubeni Nederland B.V. and D.B. Teodoro
Development Corporation entered into a contract whereby petitioner agreed to supply all the necessary
equipment, machinery, materials, technical know-how and the general design of the construction of
DBT's lime plant at the Guimaras Islands in Iloilo for a total contract price of US$5,400,000.00 on a
deferred payment basis.
Simultaneously with the supply contract, the parties entered into two financing contracts, namely
a construction loan agreement in the amount of US$1,600,000.00 and a cash loan agreement for
US$1,500,000.00. The obligation of DBT to pay the loan amortizations on their due dates under the
three (3) contracts were absolutely and unconditionally guaranteed by the National Investment and
Development Corporation (NIDC).
Pursuant to the terms of the financing contracts, the loan amortizations of DBT fell due on
January 7, 1980, July 7, 1980 and January 7, 1981. But before the first installment became due, DBT
wrote a letter to the NIDC interposing certain claims against the petitioner and at the same time

373 | P a g e
Law 321_Corporation LAW_ Case Digest

requesting NIDC for a revision of the repayment schedule and of the amounts due under the contracts
on account of petitioner's delay in the performance of its contractual commitments. In due time, the
problems regarding the lime plant were ironed out and the parties signed a "Settlement Agreement" on
July 2, 1981.

ISSUE:

Whether or not petitioner Marubeni Nederland B.V. can be considered as "doing business" in the
Philippines and therefore subject to the jurisdiction of our courts.

RULING:

YES.

The Court reiterated that there is no general rule or principle that can be laid down to determine
what constitutes doing or engaging in business. Each case must be judged in the light of its peculiar
factual milieu and upon the language of the statute applicable.
It ruled that petitioner can be sued in the regular courts because it is doing business in the
Philippines. The applicable law is Republic Act No. 5455 as implemented by the following rules and
regulations of the Board of Investments which took effect on February 3, 1969. In said Act, it
enumerates acts that shall constitute “doing business” which includes.
It cannot be denied that petitioner had solicited the lime plant business from DBT through the
Marubeni Manila branch. Records show that the "turn-key proposal for the 300 T/D Lime Plant" was
initiated by the Manila office through its Mr. T. Hojo. In a follow-up letter dated August 3, 1976, Hojo
committed the firm to a price reduction of $200,000.00 and submitted the proposed contract forms.
As reflected in the letterhead used, it was Marubeni Corporation, Tokyo, Japan which assumed
an active role in the initial stages of the negotiation. Petitioner Marubeni Nederland B.V. had no visible
participation until the actual signing of the October 28, 1976 agreement in Tokyo and even there, in the
space reserved for petitioner, it was the signature. of "S. Adachi as General Manager of Marubeni
Corporation, Tokyo on behalf of Marubeni Nederland B.V." which appeared.

PHILIPPINE COLUMBIA ENTERPRISES CO.


vs.
LANTIN
G.R. No.L-29072 June 7, 1971

FACTS:

Private respondent Katoh & Co.,Ltd, alleged in its civil complaint that it is a corporation duly
organized under the laws of Japan, with head office in Tokyo, Japan. The complaint alleged ten (10)
causes of action against the defendants Philippines Columbia Enterprises Co., with principal place of
business in Manila, and the general partners, thereof, Rufino Dy Chin and Fermin Sy, who reside in
Manila.
These ten(10) causes of action are for the collection of payment of ten(10) different shipments of
angle bars, mild steel bars, and cold rolled steel sheets allegedly ordered in May, July, October and
November, 1966 by the defendants from the plaintiff which plaintiff had duly shipped and defendants
duly received but which defendant refused to pay.
The complaint does not allege that plaintiff has secured a license to transact business in the
Philippines but its alleges that it” has not been and is not engaged in business in the Philippines and
that the transactions averred in this complaint were exports made and consummated in Tokyo, Japan in
pursuance of international trade.”

ISSUE:

Whether or not plaintiff’s allegations in its complaint, particularly in its ten (10) causes of action,
constitute by themselves an admission that it is transacting business in the Philippines.

RULING:

NO.

374 | P a g e
Law 321_Corporation LAW_ Case Digest

An examination of complaint will show that the same expressly avers that the transactions upon
which respondent plaintiff is suing were” consummated in Tokyo” and hence, not in the Philippines.
Petitioners- defendant’s assertion that the contracts were made in the Philippines squarely contradicts
the averments in the complaint. And the basic and well-known rule is that whether a cause of action is
pleaded or not must be ascertained solely upon the face of the complaint.
Since the petitioner’s averment that the plaintiff’s transactions were made in the Philippines,
being contradictory of the complaint, can not be set up in motion to dismiss for lack of cause of action,
but must be pleaded in an answer, any reception of evidence on the point would merely duplicate the
trial on the merits, and should be deferred.
Therefore, the court below committed no abuse of discretion amounting to excess of jurisdiction
in resolving to defer action on motion to dismiss. The last objection of the petitioners to the deferment
order is that if they file a counterclaim in their answer against respondent foreign corporation, they
would be recognizing the legal capacity of said corporation which they are precisely questioning. This
fear is without legal basis, for actions by foreign corporations are governed by rules different from those
in actions against them.
A counterclaim partakes of the nature of a complaint and/or cause of action against the plaintiff,
so that if the petitioner’s-defendants should file a counterclaim, the private respondent-plaintiff Katoh &
Co.,Ltd., would not be maintaining a suit and, consequently, Section 69 of the Corporation Law would
not apply.

PHILIP MORRIS, INC.


vs.
FORTUNE TOBACCO
GR NO. 158589 JUNE 27,2006

FACTS:

Petitioner Philip Morris, Inc., a corporation organized under the laws of the State of Virginia,
United States of America, is, per Certificate of Registration No. 18723 issued on April 26, 1973 by the
Philippine Patents Office (PPO), the registered owner of the trademark “MARK VII” for cigarettes.
Similarly, petitioner Benson & Hedges (Canada), Inc., a subsidiary of Philip Morris, Inc., is the
registered owner of the trademark “MARK TEN” for cigarettes as evidenced by PPO Certificate of
Registration No. 11147. And as can be seen in Trademark Certificate of Registration No. 19053, another
subsidiary of Philip Morris, Inc., the Swiss company Fabriques de Tabac Reunies, S.A., is the assignee of
the trademark “LARK,” which was originally registered in 1964 by Ligget and Myers Tobacco Company.
On the other hand, respondent Fortune Tobacco Corporation, a company organized in the
Philippines, manufactures and sells cigarettes using the trademark “MARK.”
Petitioners then filed a Complaint for Infringement of Trademark and Damages against
respondent Fortune Tobacco Corporation. In the Complaint with prayer for the issuance of a preliminary
injunction, petitioners alleged that they are foreign corporations not doing business in the Philippines
and are suing on an isolated transaction.
They averred that the countries in which they are domiciled grant to corporate or juristic persons
of the Philippines the privilege to bring action for infringement, without need of a license to do business
in those countries. Petitioners likewise manifested being registered owners of the trademark “MARK VII”
and “MARK TEN” for cigarettes as evidenced by the corresponding certificates of registration and an
applicant for the registration of the trademark “LARK MILDS”.

ISSUE:

Whether or not Petitioner has the legal capacity to sue the respondent for alleged infringement.

RULING:

YES.

375 | P a g e
Law 321_Corporation LAW_ Case Digest

A “trademark” is any distinctive word, name, symbol, emblem, sign, or device, or any combination
thereof adopted and used by a manufacturer or merchant on his goods to identify and distinguish them
from those manufactured, sold, or dealt in by others. Inarguably, a trademark deserves protection.
A foreign corporation may have the capacity to sue for infringement but the question of whether
they have an exclusive right over their symbol as to justify issuance of the controversial writ will depend
on actual use of their trademarks in the Philippines in line with Sections 2 and 2-A of RA116.
It is thus incongruous for petitioners to claim that when a foreign corporation not licensed to do
business in the Philippines files a complaint for infringement, the entity need not be actually using its
trademark in commerce in the Philippines. Such a foreign corporation may have the personality to file a
suit for infringement but it may not necessarily be entitled to protection due to absence of actual use of
the emblem in the local market.
Hence, it may be stated right off that the registration of a trademark unaccompanied by actual
use thereof in the country accords the registrant only the standing to sue for infringement in Philippine
courts. Entitlement to protection of such trademark in the country is entirely a different matter.

PUMA SPORTS
vs.
INTERMEDIATE APPELLATE COURT
G.R. NO. 75067 FEBRUARY 26, 1988

FACTS:

On July 25, 1985, petitioner PUMA SPORTSSCHUFABRIKEN, a German corporation


manufacturing “PUMA PRODUCTS”, filed a complaint for infringement of patent or trademark with a
prayer for the issuance of a writ preliminary injunction against the private respondent Mil-Oro
Manufacturing Corp. before the RTC of Makati.
On July 31, 1985, the trial court issued a temporary restraining order, restraining the private
respondent and the Director of Patents from using the trademark "PUMA' or any reproduction,
counterfeit copy or colorable imitation thereof, and to withdraw from the market all products bearing the
same trademark.

ISSUE:

Whether or not the Court of Appeals erred in holding that petitioner had no legal capacity to sue.

RULING:

YES.

Petitioner maintains that it has substantially complied with the requirements of Section 21-A of
Republic Act (RA) No. 166, as amended. According to the petitioner, its complaint specifically alleged
that it is not doing business in the Philippines and is suing under the said Republic Act. Said Section
21-A requires that “the country of which the said corporation or juristic person is a citizen, or in which
it is domiciled, by treaty, convention or law, grants a similar privilege to corporate or juristic persons of
the Philippines” but does not mandatorily require that such reciprocity between the Federal Republic of
Germany and the Philippines be pleaded.
Such reciprocity arrangement is embodied in and supplied by the Union Convention for the
Protection of industrial Property (Paris Convention) to which both the Philippines and Federal Republic
of Germany are signatories and that since the Paris Convention is a treaty which, pursuant to our
constitution, forms part of the law of the land, our courts are bound to take judicial notice of such
treaty, and, consequently, this fact need not be averred in the complaint.
The Convention of the Union of Paris for the Protection of Industrial Property to which the
Philippines became a party on September 27, 1965 provides in its Article 8 thereof that” a trade name
(corporation name) shall be protected in all the countries of the Union without the obligation of filing or
registration, whether or not it forms part of the trademark.” Thus, petitioner had the legal capacity to file
the action.

376 | P a g e
Law 321_Corporation LAW_ Case Digest

SBMA
vs.
UNIVERSAL INTERNATIONAL GROUP OF TAIWAN
GR. NO. 131680 SEPTEMBER 14, 2000

FACTS:

In 1995, a ‘Lease and Development Agreement’ was executed by respondent UIG and petitioner
SBMA under which respondent UIG shall lease from petitioner SBMA the Binictican Golf Course and
appurtenant facilities thereto to be transformed into a world class 18-hole golf course, golf club/resort,
commercial tourism and residential center. The contract in pertinent part contains pre-termination
clauses, which provides in its Section 22 thereof the acts which constitute what is “default”.
In 1997, Petitioner SBMA sent a letter to private respondent UIG calling its attention to its alleged
several contractual violations in view of private respondent UIG’s failure to deliver its various
contractual obligations, primarily its failure to complete the rehabilitation of the Golf Course in time for
the APEC Leader’s Summit, and to pay accumulated lease rentals and utilities, and to post the required
performance bond.
Respondent UIG, in its letter, interposed as an excuse the alleged default of its main contractor
FF Cruz, resulting in their filing of suit against the latter, and committed itself to comply with its
obligations within a few days. Private respondent UIG, however, failed to comply with its undertakings.
Petitioner SBMA sent a letter to private respondent UIG declaring the latter in default of its
contractual obligations to SBMA under Section 22.1 of the Lease and Development Agreement and
required it to show cause why petitioner SBMA should not pre-terminate the agreement. Private
respondents paid the rental arrearages but the other obligations remained unsatisfied.

ISSUE:

Whether or not respondents had the capacity to sue and possess material interest to institute an
action against petitioners.

RULING:

YES.

As a general rule, unlicensed foreign non-resident corporations cannot file suits in the Philippines
as provided in Section 133 of the Corporation Code. A corporation has legal status only within the state
or territory in which it was organized. For this reason, a corporation organized in another country has
no personality to file suits in the Philippines. In order to subject a foreign corporation doing business in
the country to the jurisdiction of our courts, it must acquire a license from the SEC and appoint an
agent for service of process. Without such license, it cannot institute a suit in the Philippines.
However, that the licensing requirement was “never intended to favor domestic corporations who
enter into solitary transactions with unwary foreign firms and then repudiate their obligations simply
because the latter are not licensed to do business in this country.” After contracting with a foreign
corporation, a domestic firm is estopped from denying the former’s capacity to sue.
In this case, SBMA is estopped from questioning the capacity to sue of UIG. In entering into the
LDA with UIG, SBMA effectively recognized its personality and capacity to institute the suit before the
trial court.
The petition is partially granted. the writ of preliminary injunction is lifted and the case is to the
rtc for trial on the merits.

377 | P a g e
Law 321_Corporation LAW_ Case Digest

TIME, INC.
vs.
REYES
G.R. No.L-28882 May 31, 1971

FACTS:

This is a petition by Time, Inc. for certiorari and prohibition, with preliminary injunctions, to
annul certain orders of the respondent CFI of Rizal, issued in its Civil Case No. 10403, entitled “ Antonio
J. Villegas and Juan Ponce Enrile vs. Time, Inc.,” and to prohibit the said Rizal court from further
proceeding with the said civil case contending that it is the Manila CFI which has the jurisdiction.
The petition alleges that the petitioner time, Inc., is an American Corporation with principal
offices at Rockefeller Center, New York City, N.Y., and is the publisher of “Time”, a weekly magazine; the
petition, however, does not alleged the petitioner’s legal capacity to sue in the courts of the Philippines.
In said civil case, therein plaintiffs Antonio J. Villegas and Juan Ponce Enrile seek to recover from
the therein petitioner damages upon an alleged of libel arising from a publication of time(Asia Edition)
magazine, in its issue of 18 August 1967, of an essay, entitled “Corruption in Asia”.

ISSUE:

Whether or not the petition will prosper.

RULING:

YES.

The dismissal of the present petition is asked on the ground that the petitioner foreign
corporation failed to allege its capacity to sue in the courts of the Philippines.
The Court failed to see how these doctrines can be a propos in the case at bar, since the
petitioner is not maintaining any suit” but is merely defending one against itself; it did not file any
complaint but only a corollary defensive petition to prohibit the lower court from further proceeding with
a suit that it had no jurisdiction to entertain.
Petitioner’s failure to aver its legal capacity to institute the present petition is not fatal, for a
foreign corporation may by writ of prohibition, seek relief against the wrongful assumption of
jurisdiction. And a foreign corporation seeking a writ of prohibition against further maintenance a suit,
on the ground of want jurisdiction, is not bound by the ruling of the court in which the suit was
brought, on the motion to quash service of summons, that it has jurisdiction”.
The writs applied for are granted: the respondent Court of First Instance of Rizal is declared
without jurisdiction to take cognizance of its Civil Case No. 10403; and its orders issued in connection
therewith are hereby annulled and set aside. Respondent court is further commanded to desist from
further proceedings in Civil case No. 10403 aforesaid.

UNIVERSAL RUBBER PRODUCTS, INC.


vs.
COURT OF APPEALS
G.R. No.L-30266 June 29, 1984

FACTS:

378 | P a g e
Law 321_Corporation LAW_ Case Digest

Respondent corporations, Converse Rubber Corp. and Edwardson Manufacturing Co., sued
petitioner Universal Rubber products, Inc. before the CFI of Rizal for unfair competition with damages
and attorney’s fees.
On March 4, 1968, petitioner filed a motion in the court below praying that the subpoena duces
tecum dated February 13, 1968 be quashed which was denied. A motion for reconsideration was likewise
denied.
Consequently, on August 6, 1968, petitioner Universal Rubber Products, Inc. filed its present
petition for certiorari with preliminary injunction, alleging that in so denying its motion to quash the
subpoena duces tecum and its subsequent motion for reconsideration, respondent Judge acted with
grave abuse of discretion amounting to an excess of jurisdiction.
Petitioner Universal Rubber Products, Inc. contends that private respondent Converse Rubber
Corp. is a foreign corporation not licensed to do business in the Philippines and that respondent
Edwardson is merely its licensee; that respondent Converse has no goodwill to speak of and that it has
no registration right over its own name.

ISSUE:

Whether or not respondent Converse Rubber Corp. has capacity to sue.

RULING:

YES.

The Court held that “the disability of a foreign corporation from suing in the Philippines is limited
to suits to enforce any legal or contracts rights arising from, or growing out, of any business which it
has transacted in the Philippine Islands.” On the other hand, where the purpose of the suit is “to protect
its reputation, its corporate name, its goodwill, whenever that reputation, its corporate name or goodwill
have, through the natural development of its trade, established themselves”, an unlicensed foreign
corporation may sue in the Philippines.
Hence, it is clear that Section 29 of the Corporation Law does not disqualify plaintiff- appellee
Converse Rubber, which does not have a branch office in any part of the Philippines and is not” doing
business” in the Philippines, from filing and prosecuting this action of unfair competition.
Therefore Converse Rubber Corp. can file and prosecute the action of unfair competition.

VAN ZUIDEN BROS., LTD.


vs.
GTVL INDUSTRIES, INC.
G.R. NO. 147905 MAY 28, 2007

FACTS:

Petitioner Zuiden, is a corporation, incorporated under the laws of Hong Kong. Zuiden is not
engaged in business in the Philippines, but is suing before the Philippine Courts, for the reasons
hereinafter stated. It is engaged in the importation and exportation of several products, including lace
products. On several occasions, GTVL purchased lace products from petitioner.
The procedure for these purchases, as per the instructions of GTVL, was that Zuiden delivers the
products purchased by GTVL, to a certain Hong Kong corporation, known as Kenzar Ltd. and the

379 | P a g e
Law 321_Corporation LAW_ Case Digest

products are then considered as sold, upon receipt by Kenzar of the goods purchased by GTVL. Kenzar
had the obligation to deliver the products to the Philippines and/or to follow whatever instructions GTVL
had on the matter.
Insofar as Zuiden is concerned, upon delivery of the goods to KENZAR in Hong Kong, the
transaction is concluded; and GTVL became obligated to pay the agreed purchase price. However,
commencing October 31, 1994 up to the present, GTVL has failed and refused to pay the agreed
purchase price for several deliveries ordered by it and delivered by Zuiden.

ISSUE:

Whether or not petitioner, an unlicensed foreign corporation, has legal capacity to sue before
Philippine courts.

RULING:

YES.

An unlicensed foreign corporation not doing business in the Philippines can sue before Philippine
courts. In the present case, the series of transactions between petitioner and respondent cannot be
classified as "doing business" in the Philippines under Section 3(d) of RA 7042.
An essential condition to be considered as "doing business" in the Philippines is the actual
performance of specific commercial acts within the territory of the Philippines for the plain reason that
the Philippines has no jurisdiction over commercial acts performed in foreign territories.
Here, there is no showing that petitioner performed within the Philippine territory the specific
acts of doing business mentioned in Section 3(d) of RA 7042. Petitioner did not also open an office here
in the Philippines, appoint a representative or distributor, or manage, supervise or control a local
business. While petitioner and respondent entered into a series of transactions implying a continuity of
commercial dealings, the perfection and consummation of these transactions were done outside the
Philippines.
Further, the series of transactions between petitioner and respondent transpired and were
consummated in Hong Kong. There was no single activity which petitioner performed here in the
Philippines pursuant to its purpose and object as a business organization. Moreover, petitioner’s desire
to do business within the Philippines is not discernible from the allegations of the complaint or from its
attachments. Therefore, there is no basis for ruling that petitioner is doing business in the Philippines.
Considering that petitioner is not doing business in the Philippines, it does not need a license in
order to initiate and maintain a collection suit against respondent for the unpaid balance of
respondent’s purchases.

SECURITIES AND EXCHANGE COMMISSION LAW


(P.D. No. 902-A, as Amended by R.A. No. 8799 or Securities Regulation Code)

 POWERS AND FUNCTIONS OF THE SEC

SECURITIES OF EXCHANGE COMMISSION


vs.
PERFORMANCE FOREIGN EXCHANGE CORPORATION (PFEC)
G.R. NO. 154131 JULY 20, 2006

FACTS:

Respondent Performance Foreign Exchange Corporation(PFEC) is a domestic corporation duly


registered under Securities and Exchange Commission (SEC) with its primary purpose to operate as a
broker/agent between market participants in transactions involving, but not limited to, foreign
exchange, deposits, interest rate instruments, fixed income securities, bonds/bills, repurchased
agreements of fixed income securities, certificate of deposits, bankers acceptances, bills of exchange,
over-the-counter option of the aforementioned instruments, Lesser Developed Country’s (L.D.C.) debt,
energy and stock indexes and all related, similar or derivative products, other than acting as a broker
for the trading of securities pursuant to the Revised Securities Act of the Philippines. Its secondary
purpose is to engage in money changer or exchanging foreign currencies into domestic currency,
Philippine currency or other foreign currencies into another currency.

380 | P a g e
Law 321_Corporation LAW_ Case Digest

Respondent’s officers complied and explained before the CED the nature of their business. In
2001, Emilio B. Aquino, Director of CED, issued a Cease and Desist Order in CED Case No. 99-2297,
stating that his department conducted an inquiry on respondent’s business operations for possible
violation of Republic Act (R.A.) No. 8799,otherwise known as The Securities Regulation Code; that the
outcome of the inquiry shows that respondent is engaged in the trading of foreign currency futures
contracts in behalf of its clients without the necessary license; that such transaction can be deemed as
a direct violation of Section 11 of R.A. No. 87994 and the related provisions of its Implementing Rules
and Regulations; and that it is imperative to enjoin respondent from further operating as such to protect
the interest of the public.

ISSUE:

Whether or not petitioner SEC has acted with grave abuse of discretion in issuing the Cease and
Desist Order and its subsequent Order making it permanent.

RULING:

NO.

Under Section 64 of R.A. No. 8799, there are two essential requirements that must be complied
with by the SEC before it may issue a cease and desist order: First, it must conduct proper investigation
or verification; and Second, there must be a finding that the act or practice, unless restrained, will
operate as a fraud on investors or is otherwise likely to cause grave or irreparable injury or prejudice to
the investing public.
In the present case, the first requirement is not present. Petitioner did not conduct proper
investigation or verification before it issued the challenged orders. The clarificatory conference
undertaken by petitioner regarding respondent’s business operations cannot be considered a proper
investigation or verification process to justify the issuance of the Cease and Desist Order. It was merely
an initial stage of such process, considering that after it issued the said order following the clarificatory
conference, petitioner still sought verification from the BSP on the nature of respondent’s business
activity.
Thus, the cease and desist order stays against the corporation until the latter shall be able to
submit the appropriate endorsement from the Bangko Sentral ng Pilipinas that it can engage in financial
derivative transactions.
ARRANZA ET.AL
vs.
B. F. HOMES, INC
G.R. NO. 131683 JUNE 19, 2000

FACTS:

Respondent BF Homes, Inc. (BFHI), is a domestic corporation engaged in developing subdivisions


and selling residential lots. One of the subdivisions that respondent developed was the BF Homes
Parañaque Subdivision, which now sprawls across not only a portion of the City of Parañaque but also
those of the adjoining cities of Las Piñas and Muntinlupa.
When the Central Bank ordered the closure of Banco Filipino, which had substantial investments
in respondent BFHI, respondent filed with the SEC a petition for rehabilitation and a declaration that it
was in a state of suspension of payments. On 18 March 1985, the SEC placed respondent under a
management committee. Upon that committee’s dissolution on 2 February 1988, the SEC appointed
Atty. Florencio B. Orendain as a Receiver, and approved a Revised Rehabilitation Plan.
As a Receiver, Orendain instituted a central security system and unified the sixty~five
homeowners’ associations into an umbrella homeowners’ association called United BF Homeowners’
Associations, Inc. (UBFHAI), which was thereafter incorporated with the Home Insurance and Guaranty
Corporation (HIGC). In 1989, respondent, through Orendain, turned over to UBFHAI control and
administration of security in the subdivision, the Clubhouse and the open spaces along Concha Cruz
Drive.
Through the Philippine Waterworks and Construction Corporation (PWCC), respondent’s
managing company for waterworks in the various BF Homes subdivisions, respondent entered into an
agreement with UBFHAI for the annual collection of community assessment fund and for the purchase
of eight new pumps to replace the over~capacitated pumps in the old wells.

ISSUE:

381 | P a g e
Law 321_Corporation LAW_ Case Digest

Whether or not the Securities and Exchange Commission has jurisdiction to resolve the present
controversy.

RULING:

YES.

The fact that respondent is under receivership does not divest the HLURB of that jurisdiction. A
receiver is a person appointed by the court, or in this instance, by a quasi~judicial administrative
agency, in behalf of all the parties for the purpose of preserving and conserving the property and
preventing its possible destruction or dissipation, if it were left in the possession of any of the parties.
The appointment of a receiver does not dissolve a corporation, nor does it interfere with the
exercise of its corporate rights. In this case where there appears to be no restraints imposed upon
respondent as it undergoes rehabilitation receivership, respondent continues to exist as a corporation
and hence, continues or should continue to perform its contractual and statutory responsibilities to
petitioners as homeowners.
No violation of the SEC order suspending payments to creditors would result as far as petitioners’
complaint before the HLURB is concerned. Since what petitioners seek to enforce are respondent’s
obligations as a subdivision developer. Such claims are basically not pecuniary in nature although it
could incidentally involve monetary considerations. All that petitioners’ claims entail is the exercise of
proper subdivision management on the part of the SEC~appointed Board of Receivers towards the end
that homeowners shall enjoy the ideal community living that respondent portrayed they would have
when they bought real estate from it. It can not also be considered as having claims against respondent.

QUASHA
vs.
SECURITIES OF EXCHANGE COMMISSION
G.R. NO. L-47536 MAY 31, 1978

FACTS:

Petitioner had filed on October 10, 1977 and October 17, 1977 his complaints and continuing
opposition with respondent commission against the filing of respondent Manila Polo Club, Inc.'s
Amended Articles of Incorporation and Amended By-Laws which would convert said club into a
proprietary club, assailing the amendments as illegal, inequitable and immoral, alleging inter alia that
the amendments have the effect of enabling the members to appropriate the club's property and to use it
as their contribution to the new club; the real estate assets of Manila Polo Club consist of 25 hectares,
more or less, of prime real estate in 'he middle of Forbes lark Makati, Metro Manila. which are
conservatively valued at present market valuation of P200 Million and its buildings, improvements,
facilities and other equipments at about P20 Million.
The more or less 2,000 actual members who will become proprietary owners of the Club's assets
under the proposed conversion will inequitably enrich themselves if this Honorable Commission will
allow the comparatively paltry of P12,500.00 to be paid for each proprietary membership;" "the value
which the Club now commands results from the accrued contribution of past (and present) members'
money, time, effort and foresight; and the conversion plan does not in any way compensate the
predecessors of the present membership (and also those of the present membership who do not opt for
conversion) who substantially contributed to making the Club what it is today" and further claiming that
the amendments had not been duly adopted by the required two-thirds vote.
Petitioner prayed for the disapproval and cancellation of respondent club's amended articles and
by-laws and denial of its application to register its proprietary shares, and prayed for a restraining order
meanwhile enjoining it from selling and/or accepting any payments for the questioned proprietary
shares.

ISSUE:

Whether or not the hearing officer of the SEC is empowered to issue the questioned order denying
the injunctive relief.

RULING:

YES.

382 | P a g e
Law 321_Corporation LAW_ Case Digest

The Court ruled that in view of the extremely limited time, with the commission's hearing officer
having issued his questioned order denying injunctive relief only on December 22, 1977 at the height of
the Christmas holiday with just a few days before The scheduled deadline of December 28, 1977,
petitioner properly filed the present petition directly with this Court without going through the
prescribed procedure of filing an appeal with respondent Commission en banc within the 30-day
reglementary period since such recourse was obviously not a plain, speedy and adequate remedy.
The questions raised by petitioner in his pending complaints with respondent commission
warrant a full-blowing trial' on the merits" after which the main issues may be duly adjudicated as
contended by him, and since respondents likewise concur in this stand, the case will be remanded to
respondent commission for such trial and determination on the merits.
Finally, petitioner has not satisfactorily established his right to the restraining order prayed for.
Considering that petitioner submitted the incident on the basis of his memorandum without
presentation of evidence, the Court holds that respondent commission did not act with grave abuse of
discretion in denying the restraining order prayed for. The case is ordered remanded to respondent
commission for a full trial and hearing and determination on the merits.
TRADER’S ROYAL BANK
vs.
COURT OF APPEALS
G.R. No. L-78412 September 26, 1989

FACTS:

On March 30,1982, the Philippine Blooming Mills, Inc. (PBM) and Alfredo Ching jointly submitted
to the Securities and Exchange Commission a petition for suspension of payments (SEC No. 2250)
where Alfredo Ching was joined as co-petitioner because under the law, he was allegedly entitled, as
surety, to avail of the defenses of PBM and he was expected to raise most of the stockholders' equity of
Pl00 million being required under the plan for the rehabilitation of PBM. Traders Royal Bank was
included among PBM's creditors named in Schedule A accompanying PBM's petition for suspension of
payments.
On May 13, 1983, the petitioner bank filed a case against PBM and Alfredo Ching, to collect
P22,227,794.05 exclusive of interests, penalties and other bank charges representing PBM's
outstanding obligation to the bank. Alfredo Ching, a stockholder of PBM, was impleaded as co-defendant
for having signed as a surety for PBM's obligations to the extent of ten million pesos (Pl0,000,000) under
a Deed of Suretyship dated July 21, 1977.
In its en banc decision, the SEC declared that it had assumed jurisdiction over petitioner Alfredo
Ching pursuant to Section 6, Rule 3 of the new Rules of Procedure of the SEC providing that "parties in
interest without whom no final determination can be had of an action shall be joined either as
complainant, petitioner or respondent" to prevent multiplicity of suits.
On July 9, 1982, the SEC issued an Order placing PBM's business, including its assets and
liabilities, under rehabilitation receivership, and ordered that "all actions for claims listed in Schedule A
of the petition pending before any court or tribunal are hereby suspended in whatever stage the same
may be, until further orders from the Commission". As directed by the SEC, said order was published
once a week for three consecutive weeks in the Bulletin Today, Philippine Daily Express and Times
Journal at the expense of PBM and Alfredo Ching.

ISSUE:

Whether or not the court a quo could acquire jurisdiction over Ching in his personal and
individual capacity as a surety of PBM in the collection suit filed by the bank, despite the fact that
PBM's obligation to the bank had been placed under receivership by the SEC.

RULING:

YES.

Although Ching was impleaded in SEC Case No. 2250, as a co-petitioner of PBM, the SEC could
not assume jurisdiction over his person and properties. The Securities and Exchange Commission was
empowered, as rehabilitation receiver, to take custody and control of the assets and properties of PBM
only, for the SEC has jurisdiction over corporations only not over private individuals, except
stockholders in an intra-corporate dispute (Sec. 5, P.D. 902-A and Sec. 2 of P.D. 1758). Being a nominal
party in SEC Case No. 2250, Ching's properties were not included in the rehabilitation receivership that
the SEC constituted to take custody of PBM's assets.

383 | P a g e
Law 321_Corporation LAW_ Case Digest

Therefore, the petitioner bank was not barred from filing a suit against Ching, as a surety for
PBM. An anomalous situation would arise if individual sureties for debtor corporations may escape
liability by simply co- filing with the corporation a petition for suspension of payments in the SEC whose
jurisdiction is limited only to corporations and their corporate assets.

VICMAR DEVELOPMENT
vs.
COURT OF APPEALS
G.R. No. 81547 May 21, 1990

FACTS:

Sometime in August, 1982, a conflict arose between petitioner Vicente Angliongto and private
respondent Rufino T. Nasser on the matter of exclusive control and management of Petitioner
Corporation. On July 7, 1983, petitioner Corporation by petitioner Angliongto, filed a verified petition in
the public respondent SEC against private respondent Nasser, alleging, that private respondent Nasser
was a Director, Executive Vice-President and General Manager of petitioner Corporation from 1974 to
August 26, 1982 but during the annual meeting of stockholders of petitioner corporation held on August
26, 1982, private respondent Nasser was not re-elected as member of the Board of Directors or to his
previous management positions.
In view of the result of the annual stockholders' meeting, private respondent Nasser was then
advised by the incoming president, herein petitioner Angliong to that the latter would actively manage
the corporate affairs of Petitioner Corporation.
In view thereof, private respondent Nasser was asked to turn over all corporate books and records
in his possession to the duly elected officers, among others, which demand remained (un)heeded by
private respondent Nasser as the latter continued to hold office as Executive Vice-President and General
Manager of petitioner Corporation, performing acts and entering into transactions inimical to the
interests of the petitioner Corporation and its stockholders.
Said petition also prayed for the issuance of a restraining order and thereafter, a permanent
injunction to enjoin private respondent Nasser from representing himself as an officer of petitioner
Corporation, among other things, and for him, to surrender all corporate books and records to the duly
elected officers of said Corporation.

ISSUE:

Whether or not the Securities and Exchange Commission has abused its discretion in recalling its
Order to enforce a writ of preliminary injunction.

RULING:

NO.

The facts reveal that the writ of preliminary injunction issued on September 19, 1983 enjoined
private respondent Nasser from acting as, and/or representing himself to be, the Executive Vice-
President and/or General Manager and/or officer in any capacity of petitioner Corporation. Upon
presentment of the Agreement dated November 10, 1983 showing a transfer of ownership, control and
management of Vicmar Corporation by Vicente Angliongto unto Nasser, the SEC correctly recalled the
order of March 5, 1986 directing the implementation of the aforesaid writ, pending hearing on the
motion dated March 17, 1986.
To allow execution of the writ of preliminary injunction in favor of the petitioners despite having
transferred their rights of ownership, control and management over said corporation to respondent
Nasser would be baseless, the contract having shown prima facie that the latter is entitled to remain as
Vice-President and General Manager of petitioner Corporation.
Thus, no grave abuse of discretion can be attributed to the SEC in recalling the order to enforce a
writ of preliminary injunction in this wise. After all, the issuance or recall of preliminary writ of
injunction is an interlocutory matter that remains at all times within the control of the court (Alvaro v.
Zapata, 118 SCRA 728 [1982]). The grant or denial of an injunction rests upon the sound discretion of
the lower tribunal, in the exercise of which the Supreme Court will not interfere except in a clear case of
abuse.

384 | P a g e
Law 321_Corporation LAW_ Case Digest

 ORIGINAL AND EXCLUSIVE JURISDICTION OF THE REGIONAL TRIAL COURTS

Orendain
vs.
BF Homes, Inc.
G.R. No. 146313 October 31, 2006

FACTS:

BF Homes, Inc. availed financial assistance from various sources to buy properties and convert
them into residential subdivisions. Despite its solvent status, it filed a Petition for Rehabilitation and for
Declaration in a State of Suspension of Payments. SEC ordered the appointment of a rehabilitation
receiver, FBO Management Networks, Inc., with Orendain as Chairman to prevent paralyzation of BF
Homes’ business operations. Deed of Absolute Sale was executed by and between BF Homes—
represented by Orendain—as absolute and registered owner, and the Local Superior of the Franciscan
Sisters of the Immaculate Phils., Inc. (LSFSIPI) over a parcel of land.
BF Homes filed a Complaint with the RTC against LSFSIPI and Orendain for reconveyance of the
property alleging that the LSFSIPI transacted with Orendain in his individual capacity and therefore,
neither FBO Management, Inc. nor Orendain had title to the property transferred. Moreover, it averred
that the selling price was grossly inadequate or insufficient amounting to fraud and conspiracy with the
LSFSIPI.

ISSUE:

Whether or not a simple reconveyance suit is within the jurisdiction of the RTC.

RULING:

YES.

It is the RTC which has jurisdiction. Clearly, the controversy involves matters purely civil in
character and is beyond the ambit of the limited jurisdiction of the SEC. The better policy in
determining which body has jurisdiction over a case would be to consider not only [1] the status or
relationship of the parties but also [2] the nature of the question that is the subject of their controversy.”
More so, the first element requires that the controversy must arise out of intra-corporate or
partnership relations between any or all of the parties and the corporation, partnership or association of
which they are stockholders, members or associates; between any or all of them and the corporation,
partnership or association of which they are stockholders, members or associates, respectively; and
between such corporation, partnership or association and the State insofar as it concerns their
individual franchises.
The second element requires that the dispute among the parties be intrinsically connected with
the regulation of the corporation. If the nature of the controversy involves matters that are purely civil in
character, necessarily, the case does not involve an intra-corporate controversy.
The determination of whether a contract is simulated or not is an issue that could be resolved by
applying pertinent provisions of the Civil Code. Section 5 of PD No. 902-A does not apply in the instant
case.
The LSFSIPI is neither an officer nor a stockholder of BF Homes, and this case does not involve
intra-corporate proceedings. In addition, the seller Orendain, is being sued in his individual capacity for
the unauthorized sale of the property in controversy. In addition, jurisdiction over the case for
reconveyance is clearly vested in the RTC as provided in paragraph (2), Section 19, B.P. Blg. 129.

ALFREDO P. PASCUAL and LORETA S. PASCUAL


vs.
COURT OF APPEALS (former Seventh Division), ERNESTO P. PASCUAL and HON. ADORACION
ANGELES, in her capacity as Presiding Judge, RTC, Kaloocan City, Branch 121
G.R. No. 138542 August 25, 2000

FACTS:

385 | P a g e
Law 321_Corporation LAW_ Case Digest

Ernesto P. Pascual filed a complaint in the Regional Trial Court for "accounting, reconveyance of
real property based on implied trust resulting from fraud, declaration of nullity of TCT, recovery of sums
of money, and damages" against his brother, petitioner Alfredo, and the latter’s wife Loreta Pascual.
Petitioners filed a motion to dismiss on the ground that the complaint raises an intra-corporate
controversy between the parties over which original and exclusive jurisdiction is vested in the Securities
and Exchange Commission (SEC).

ISSUE:

Whether or not complaint against the Petitioners involves an intra-corporate dispute cognizable
by the SEC and, therefore, the Regional Trial Court should have dismissed the complaint.

RULING:

NO.

The regular courts, not the SEC, have jurisdiction over this case.1âwphi1 Petitioners and private
respondent never had any corporate relations in Phillens. It appears that private respondent was never a
stockholder in Phillens, of which the parties’ predecessor-in-interest, Luciano Pascual, Sr., was a
stockholder and whose properties are being litigated. Private respondent’s allegation is that, upon the
death of their father, he became co-owner in the estate left by him, and part of this estate includes the
corporate interests in Phillens. He also alleges that petitioners repudiated the trust relationship created
between them and appropriated to themselves even the property that should have belonged to
respondent. It is thus clear that there is no corporate relationship involved here. That petitioner Alfredo
Pascual was a corporate officer holding in trust for his brother their father’s corporate interests did not
create an intra-corporate relationship between them.
Nor is the controversy corporate in nature. As we have stated before, the grant of jurisdiction
must be viewed in the light of the nature and function of the SEC under the law. In the case at bar, the
corporation whose properties are being contested no longer exists, it having been completely dissolved in
1993; consequently, the supervisory authority of the SEC over the corporation has likewise come to an
end.

 DEVICES OF SCHEMES AMOUNTING TO FRAUD OF MISREPRESENTATION

HERNANI N. FABIA
vs.
COURT OF APPEALS
G.R. No. 132684 20 August 2000

FACTS:

Fabia was the President of private respondent MTCP, a domestic corporation engaged in providing
maritime courses and seminars to prospective overseas contract workers and seamen. He was likewise a
Director and stockholder thereof. MTCP filed an affidavit-complaint for estafa against Hernani N. Fabia
alleging that on various occasions Fabia drew cash advances from MTCP, covered by cash vouchers
which he failed to liquidate despite repeated demands. Fabia’s defense is that such were in the nature of

386 | P a g e
Law 321_Corporation LAW_ Case Digest

simple loans that had already been liquidated and paid. The Office of the City Prosecutor of Manila
dismissed the complaint for lack of jurisdiction for the reason that the controversy pertained to the
relationship between a corporation and a former officer it was the Securities and Exchange Commission
(SEC) which had original and exclusive jurisdiction over the case.

ISSUE:

Whether or not SEC have jurisdiction over the case?

RULING:

YES.

The jurisdiction of the SEC to "intra-corporate disputes" defined as any act or omission of the
Board of Directors/Trustees of corporations, or of partnerships, or of other associations, or of their
stockholders, officers, or partners, including any fraudulent devices, schemes or representations, in
violation of any law or rules and regulations administered and enforced by the Commission. Petitioner
was the President as well as a Director and stockholder in private respondent MTCP, who was charged
with the misappropriation or diversion of corporate funds after having failed to liquidate the amount he
had received as cash advances from the company. The charge against petitioner is for estafa, an offense
punishable under The Revised Penal Code (RPC), and prosecution for the offense is presently before the
regular courts. However, jurisdiction is determined not from the law upon which the cause of action is
based, nor the type of proceedings initiated, but rather, it is gleaned from the allegations stated in the
complaint. It is evident from the complaint that the acts charged are in the nature of an intra-corporate
dispute as they involve fraud committed by virtue of the office assumed by petitioner as President,
Director, and stockholder in MTCP, and committed against the MTCP corporation. This sufficiently
removes the action from the jurisdiction of the regular courts, and transposes it into an intra-corporate
controversy within the jurisdiction of the SEC. The fact that a complaint for estafa, a felony punishable
under the RPC, has been filed against petitioner does not negate and nullify the intra-corporate nature
of the cause of action, nor does it transform the controversy from intra-corporate to a criminal one.
Accordingly, as the matter involves an intra-corporate dispute within the jurisdiction of the SEC, the
issue of whether prior non-accounting precludes a finding of probable cause for the charge of estafa no
longer finds relevance.
However, in conformity with RA 8799, The Securities Regulation Code, amending PD 902-A,
which has effectively transferred the jurisdiction of the Securities and Exchange Commission over all
cases enumerated under Sec. 5 of PD 902-A to the courts of general jurisdiction or the appropriate
Regional Trial Courts.

387 | P a g e
Law 321_Corporation LAW_ Case Digest

A & A CONTINENTAL COMMODITIES PHILIPPINES, INC.


vs.
SEC and ROLANDO G. AGUILA
G.R. No. L-55343 16 August 1993

FACTS:

Petitioner is a domestic corporation engaged in the commodities brokerage business. On August


3, 1979, petitioner and private respondent entered into a contract for the purchase or sale of
commodities. On January 21, 1980, private respondent bought, through petitioner, 7 contracts of
copper. The margin requirement: for the 7 contracts was P18,750 per contract or a total amount of
P131,250.00, which amount was earmarked from private respondent's cash deposit with petitioner of
P306,326.46. On January 23, 1980, petitioner, allegedly "without valid and justifiable cause,
maliciously, arbitrarily, wantonly, fraudulently, and recklessly" ordered private respondent to increase
his margin requirements per contract and gave private respondent up to 5 P.M. of the same day w/in
which to deposit with petitioner the additional amount of. Private respondent requested additional time
within which to raise the amount, but petitioner informed him that it would immediately sell his 7
copper contracts should he fail to deposit the additional amount by 5 P.M. that same day. Private
respondent then requested that should petitioner proceed with the sale, the same be not effected
immediately upon the opening of trading if prices were low but at a later time. However, petitioner did
not accede to the request and sold 5 contracts immediately upon the opening of trading on January 24,
1980 and the other two at a later time.

ISSUE:

Whether or not the SEC has jurisdiction over the case.

RULING:

YES.

Poring over the complaint filed by private respondent, the court find that the complaint is praying
for two reliefs based on the same set of facts. One is for the revocation of the certificate of registration of
petitioner; the other is for a sum of money.
On the action to revoke the certificate of registration of petitioner, there is no doubt that the SEC
has jurisdiction over the same. Section 6(L) of Presidential Decree No. 902-A clearly provides that the
SEC shall possess the power to suspend or revoke, after proper notice and hearing, the franchise or
certificate registration of corporations, partnerships or associations.
On the other aspect of the SEC's jurisdiction over the action for a sum of money, we likewise rule
that the Commission has the legal competence to decide said issue. It is axiomatic that jurisdiction over
the subject matter of a case is conferred by law and is determined by the allegations of the complaint,
irrespective of whether or not the plaintiff is entitled to recover upon all or some of the claims asserted
therein.
Considering that Petitioners' Complaints sufficiently allege acts amounting to fraud and
misrepresentation committed by Respondent Corporation, the SEC must be held to retain its original
and exclusive jurisdiction over these five (5) cases notwithstanding the revocation by the Central Bank of
Respondent Corporation's license or permit to operate as a financing company and despite the fact that
the suits involve collections of sums of money paid to said corporation, the recovery of which would
ordinarily fall within the jurisdiction of regular Courts. The fraud committed is detrimental to the
interest of the public and, therefore, encompasses a category of relationship within the SEC jurisdiction.

388 | P a g e
Law 321_Corporation LAW_ Case Digest

MANUEL M. ALLEJE
vs.
COURT OF APPEALS, SPORTS HEALTH AND PHYSICAL EDUCATION (SHAPE) CENTRE, INC.,
and/or ARMIE E. ELMA, Presiding Judge, RTC -Pasig, Br. 153
G.R. No. 107152 25 January 1995

FACTS:

SHAPE is a duly registered non-stock, non-profit corporation the primary purpose of which is to
foster and promote health, conduct physical education and fitness exercises as well as pleasure and
recreation activities by establishing and maintaining facilities, sports centers, and the like. Petitioner
Manuel M. Alegre was the Executive Vice President (EVP) of SHAPE until his termination of 6 June
1991.
On 16 October 1991 SHAPE filed a complaint for injunction and damages with application for
preliminary injunction and/or temporary restraining order with preliminary attachment against
petitioner Alleje before theRTC of Pasig.
On 14 December 1991, SHAPE filed a supplemental complaint alleging that after the filing of the
original complaint, it discovered that Alleje coined a new name for his health and fitness centers and
advertised the same as "SHAPE CAMP," thus tending to misrepresent to the public that SHAPE had
entered into some kind of arrangement with Alleje when it fact it had not.
Instead of filing an answer, Alleje filed a motion to dismiss contending that the Regional Trial
Court has no jurisdiction over the present action which partakes of an intracorporate controversy solely
cognizable by the SEC.

ISSUE:

Whether or not the Regional Trial Court of Pasig, and not the SEC, has jurisdiction over an action
instituted by respondent SPORTS HEALTH AND PHYSICAL EDUCATION (SHAPE) CENTER, INC.,
against one of its officers, petitioner herein, for recovery of corporate funds and assets allegedly
misappropriated by him.

RULING:

NO.

The SEC has jurisdiction. It is axiomatic that jurisdiction over the subject matter of a case is
conferred by law and is determined by the allegations of the complaint irrespective of whether plaintiff is
entitled to some or all of the claims asserted therein. The averments in the original complaint as well as
the supplemental complaint of SHAPE sufficiently show the alleged acts committed by Alleje, an officer,
against SHAPE which amount to fraud and misrepresentation and thus detrimental to the interest of the
public. "Fraud" is defined as a generic term embracing all multifarious means which human ingenuity
can devise, and which are resorted to by one individual to secure an advantage over another by false
suggestions or by suppression of truth and includes all surprise, trick, cunning, dissembling and any
unfair way by which another is cheated.
Clearly, the complaint alleges that as an officer of SHAPE Alleje employed devises or schemes
tantamount to fraud and misrepresentation in order to divert corporate funds and assets for his
personal use. This has transposed an otherwise ordinary action for recovery of certain properties and
sum of money with damages into an intracorporate controversy which calls for the adjudicative powers
of the SEC pursuant to Sec. 5, par. (a), of PD 902-A. In other words, the complaint filed SHAPE before
the Pasig trial court imputes unmistakable acts of fraud to Alleje as an officer of SHAPE which have
supposedly resulted in its heavy financial losses. The fraud committed is detrimental to the interest not
only of the corporation itself but also of its members who have unselfishly agreed among themselves
that no part of the net income of the corporation shall inure to any of them. This (fraud) encompasses a
category of relationship within the SEC jurisdiction, despite the fact that the complaint ultimately
involves collection of money, the recovery of which would ordinarily fall within the legal competence of
the regular courts.
MARIA LUISA FLOR C. BAÑEZ, SPOUSES PETRONILO ESTEVES AND LUISA ESTEVES AND
CONSUELO M. CAULBOY
vs.
DIMENSIONAL CONSTRUCTION TRADE AND DEVELOPMENT CORPORATION AND THE RTC.
G.R. No. L-62648 22 November 1985

FACTS:

389 | P a g e
Law 321_Corporation LAW_ Case Digest

On October 13, 1980 the petitioners herein filed with the RTC a complaint praying therein that
the Dimensional Construction Trade and Development Corporation (Dimensional), be ordered to pay
them the sums of money which were already due to them under the various promissory notes issued by
said corporation.On August 20, 1981, or 9 months thereafter, petitioners moved to have Dimensional
declared in default for failure to file a responsive pleading which the court granted. Before a judgment in
the case can be rendered, Dimensional filed on December 16, 1981, a motion to dismiss contending that
the SEC and not the CFI had jurisdiction over the case because there was fraud committed against the
stockholders of the corporation. Petitioners filed their opposition to said omnibus motion averring that
Dimensional has no right to be heard in any manner after losing its standing in court due to the default
order issued against it. Petitioners argued that their complaint is simply a suit for the collection of sums
of money on account of promissory notes which have already matured and therefore their case is
originally and exclusively cognizable by the CFI. The RTC dismissed the case.

ISSUE:

Whether or not the RTC correct in ruling that jurisdiction falls with the SEC and not the RTC.

RULING:

NO.

The recitals of the complaint in the case disclose that plaintiff's cause of action is merely for the
collection of the various sums of money that have already become payable to petitioners due to the
promissory notes executed by defendant corporation which have already matured. There is no allegation
nor any mention whatsoever in plaintiff's complaint that a device or scheme was resorted to by private
respondent corporation amounting to fraud and misrepresentation. It is, therefore, difficult to consider
that petitioners' case falls within the jurisdiction of the Securities and Exchange Commission pursuant
to PD 902-A. Paradoxically, despite the absence of imputation of fraud and misrepresentation being
alleged by plaintiff, it is the defendant corporation itself which insinuates the existence of fraud and
misrepresentation on its part. Evidently, the defendant's challenge to the jurisdiction of the court below
is principally intended to negate the effects of the order of default earlier issued against it as well as the
evidence already adduced by petitioners in the court below. The tactical step resorted by the private
respondent in the trial court appears to be its deliberate attempt to unduly delay the satisfaction of the
reliefs claimed for by petitioners and to avoid the effects of its failure to file any answer to the complaint
and controvert the evidence already adduced against it.
In the promissory notes issued by private Respondent Corporation, it is clearly indicated therein
that the sums of money received by private respondent were in the nature of investments of the
petitioners, agreed upon by the parties to be returned by the corporation upon the maturity of said
promissory notes. As the money received by private respondent do not constitute payment of
subscription of shares, the petitioners herein did not become members of respondent Dimensional Trade
and Development Corporation.

390 | P a g e
Law 321_Corporation LAW_ Case Digest

RAUL SESBREÑO
vs.
HON. COURT OF APPEALS, DELTA MOTORS CORPORATION AND PILIPINAS BANK
G.R. No. 89252 May 24, 1993

FACTS:

Hermilo Rodis, Sr. et.al. was charged with estafa before the Regional Trial Court of Cebu.
Respondents moved to quash the information on the ground that the Securities and Exchange
Commission (SEC), not the regular courts, had jurisdiction over the offense charged and that the facts
stated herein did not constitute an offense The trial court denied the motion and private respondent
elevated the case to the then Intermediate Appellate Court . On August 16, 1983, the appellate court
dismissed the petition. Hence, trial ensued in the criminal case. However, after the prosecution had
rested its case, private respondent filed a motion to dismiss on demurrer to evidence based on the core
proposition that there was no criminal offense of estafa from the non-payment of a money market
placement.

ISSUE:

Whether or not private respondent may be held liable for estafa.

RULING:

YES.

In money market placement, the investor is a lender who loans his money to a borrower through
a middleman or dealer. Petitioner here loaned his money to a borrower through Philfinance. When the
latter failed to deliver back petitioner's placement with the corresponding interest earned at the maturity
date, the liability incurred by Philfinance was a civil one. As such, petitioner could have instituted
against Philfinance before the ordinary courts a simple action for recovery of the amount he had
invested and he could have prayed therein for damages. It appears, however, that petitioner did not even
implead Philfinance in the complaint for damages arising from the no return of investment with respect
to the same money market placement involved herein, which he eventually filed against Delta Motors
Corporation and Pilipinas Bank before the Regional Trial Court of Cebu City .What is involved here in a
money market transaction. As defined by Lawrence Smith, 'the money market is a market dealing in
standardized short-term credit instruments (involving large amounts) where lenders and borrowers do
not deal directly with each other but through a middle man or dealer in the open market. It involves
'commercial papers' which are instruments 'evidencing indebtedness of any person or entity . . . which
are issued, endorsed, sold or transferred or in any manner conveyed to another person or entity, with or
without recourse.' The fundamental function of the money market device in its operation is to match
and bring together in a most impersonal manner both the 'fund users and the 'fund suppliers.' The
money market is an 'impersonal market', free from personal considerations. The market mechanism is
intended 'to provide quick mobility of money and securities. The Court of Appeals, therefore, correctly
ruled that a money market transaction partakes of the nature of a loan and therefore nonpayment
thereof would not give rise to criminal liability for estafa through misappropriation or conversion.

391 | P a g e
Law 321_Corporation LAW_ Case Digest

CONTROVERSIES ARISING OUT OF INTRA-CORPORATE OF PARTNERSHIP


RELATIONS

VITALIANO N. AGUIRRE II and FIDEL N. AGUIRRE


vs.
FQB+7, INC., NATHANIEL D. BOCOBO, PRISCILA BOCOBO and ANTONIO DE VILLA
G.R. No. 170770 January 9, 2013

FACTS:

On October 5, 2004, Vitaliano filed, in his individual capacity and on behalf of FQB+7, Inc., a
Complaint for intra-corporate dispute, injunction, inspection of corporate books and records, and
damages, against respondents Nathaniel D. Bocobo, Priscila D. Bocobo and Antonio De Villa. The
Complaint alleged that FQB+7 was established in 1985 with the following directors and subscribers, as
reflected in its Articles of Incorporation.
The substantive changes found in the GIS, respecting the composition of directors and
subscribers of FQB+7, prompted Vitaliano to write to the "real" Board of Directors (the directors reflected
in the Articles of Incorporation), represented by Fidel N. Aguirre. In this letter dated April 29, 2004,
Vitaliano questioned the validity and truthfulness of the alleged stockholders meeting held on
September 3, 2002. He asked the "real" Board to rectify what he perceived as erroneous entries in the
GIS, and to allow him to inspect the corporate books and records.

ISSUE:

Whether the RTC has jurisdiction over an intra-corporate dispute involving a dissolved
corporation.

RULING:

YES.

Intra-corporate disputes remain even when the corporation is dissolved. Jurisdiction over the
subject matter is conferred by law. R.A. No. 8799 conferred jurisdiction over intra-corporate
controversies on courts of general jurisdiction or RTCs, to be designated by the Supreme Court. Thus,
as long as the nature of the controversy is intra-corporate, the designated RTCs have the authority to
exercise jurisdiction over such cases.
Thus, to be considered as an intra-corporate dispute, the case: (a) must arise out of intra-
corporate or partnership relations, and (b) the nature of the question subject of the controversy must be
such that it is intrinsically connected with the regulation of the corporation or the enforcement of the
parties’ rights and obligations under the Corporation Code and the internal regulatory rules of the
corporation.
Examining the case before us in relation to these two criteria, the Court finds and so holds that
the case is essentially an intra-corporate dispute. It obviously arose from the intra-corporate relations
between the parties, and the questions involved pertain to their rights and obligations under the
Corporation Code and matters relating to the regulation of the corporation. We further hold that the
nature of the case as an intra-corporate dispute was not affected by the subsequent dissolution of the
corporation.

PHILIP L. GO, PACIFICO Q. LIM and ANDREW Q. LIM


vs.
DISTINCTION PROPERTIES DEVELOPMENT AND CONSTRUCTION, INC.
G.R. No. 194024 April 25, 2012

FACTS:

392 | P a g e
Law 321_Corporation LAW_ Case Digest

Petitioners are registeredindividual owners of condominium units in Phoenix Heights


Condominium developedby the respondent.In August 2008, petitioners, as condominium unit-owners,
filed a complaint beforethe HLURB against DPDCI for unsound business practices and violation of
theMDDR, alleging that DPDCI committed misrepresentation in their circulated flyersand brochures as
to the facilities or amenities that would be available in thecondominium and failed to perform its
obligation to comply with the MDDR.
In defense, DPDCI alleged that the brochure attached to the complaint was “a merepreparatory
draft”. HLURB rendered its decision in favor of petitioners. DPDCI filedwith the CA its Petition for
Certiorari and Prohibition on the ground that HLURB actedwithout or beyond its jurisdiction.The CA
ruled that the HLURB had no jurisdiction over the complaint filed bypetitioners as the controversy did
not fall within the scope of the administrative agency’s authority.

ISSUES:

Whether or not the HLURB has jurisdiction over the complaint filed by the petitioners.

RULING:

NO.

Jurisdiction over the subject matter of a case is conferred by law and determinedby the
allegations in the complaint which comprise a concise statement of theultimate facts constituting the
plaintiff's cause of action. The nature of an action, aswell as which court or body has jurisdiction over it,
is determined based on theallegations contained in the complaint of the plaintiff, irrespective of whether
or notthe plaintiff is entitled to recover upon all or some of the claims asserted therein. The
averments in the complaint and the character of the relief sought are the ones tobe consulted. Once
vested by the allegations in the complaint, jurisdiction alsoremains vested irrespective of whether or not
the plaintiff is entitled to recover uponall or some of the claims asserted therein. Thus, it was ruled that
the jurisdiction ofthe HLURB to hear and decide cases is determined by the nature of the cause ofaction,
the subject matter or property involved and the parties.In this case, the complaint filed by petitioners
alleged causes of action thatapparently are not cognizable by the HLURB considering the nature of the
action andthe reliefs sought.

STRATEGIC ALLIANCE DEVELOPMENT CORPORATION


vs.
STAR INFRASTRUCTURE DEVELOPMENT CORPORATION ET AL.
G.R. No. 187872 November 17, 2010

FACTS:

Respondents Aderito Z. Yujuico and Bonifacio C. Sumbilla, in their respective capacities as then
President and Treasurer of STRADEC, executed a Promissory Note for and in consideration of a loan in
the sum of P10,000,000.00 ostensibly extended in favor of said corporation by respondent Robert L.
Wong, one of the incorporators of SIDC. As security for the payment of the principal as well as the
stipulated interests thereon, a pledge constituted over STRADEC’s entire shareholdings in SIDC was
executed by respondent Yujuico on 1 April 2005. In view of STRADEC’s repeated default on its

393 | P a g e
Law 321_Corporation LAW_ Case Digest

obligations,11 however, the shares thus pledged were sold by way of the 26 April 2005 notarial sale
conducted in Makati City by respondent Raymond M. Caraos. Having tendered the sole bid
of P11,800,000.00, respondent Wong was issued the corresponding certificates of stocks by respondent
Bede S. Tabalingcos, SIDC’s Corporate Secretary for the years 2004 and 2005, after the transfer was
recorded in the corporation’s stock and transfer book.
On 17 July 2006, Cezar T. Quiambao, in his capacity as President and Chairman of the Board of
Directors of STRADEC, commenced the instant suit with the filing of the petition which was docketed as
Civil Case No. 7956 before Branch 2 of the Regional Trial Court of Batangas City, sitting as a Special
Commercial Court (SCC).

ISSUE:

Whether or not the RTC has jurisdiction over the case.

RULING:

YES.

In addition to being conferred by law, it bears emphasizing that the jurisdiction of a court or
tribunal over the case is determined by the allegations in the complaint and the character of the relief
sought, irrespective of whether or not the plaintiff is entitled to recover all or some of the claims asserted
therein. Moreover, pursuant to Section 5.2 of Republic Act No. 8799, otherwise known as the Securities
Regulation Code, the jurisdiction of the SEC over all cases enumerated under Section 5 of Presidential
Decree No. 902-A has been transferred to RTCs designated by this Court as SCCs pursuant to A.M. No.
00-11-03-SC promulgated on 21 November 2000.
It should be noted that the SCCs are still considered courts of general jurisdiction. Section 5.2 of
R.A. No. 8799 directs merely the Supreme Court's designation of RTC branches that shall exercise
jurisdiction over intra-corporate disputes. Nothing in the language of the law suggests the diminution of
jurisdiction of those RTCs to be designated as SCCs. The assignment of intra-corporate disputes to
SCCs is only for the purpose of streamlining the workload of the RTCs so that certain branches thereof
like the SCCs can focus only on a particular subject matter.
The RTC exercising jurisdiction over an intra-corporate dispute can be likened to an RTC
exercising its probate jurisdiction or sitting as a special agrarian court. The designation of the SCCs as
such has not in any way limited their jurisdiction to hear and decide cases of all nature, whether civil,
criminal or special proceedings.

GD EXPRESS WORLDWIDE N.V. and AMIHAN MANAGEMENT SERVICES, INC.


vs.
HON. COURT OF APPEALS (FOURTH DIVISION), HON. SECURITIES AND EXCHANGE COMMISSION
(en banc), HON. ROSITA R. GUERRERO, in her capacity as Hearing Officer, and FILCHART
AIRWAYS, INC., Respondents
G.R. No. 136978 May 8, 2009

FACTS:

Petitioner GD Express Worldwide N.V. (GD Express) is a corporation duly organized and existing
under the laws of the Netherlands. On 27 September 1990, its predecessor-in-interest, TNT Limited
(TNT) entered into a joint venture agreement with Philippine Aerospace Development Corporation (PADC)
for the establishment of a domestic corporation as their corporate vehicle to operate as an international
air freight carrier. The joint venture agreements stipulated that PADC would own 80% of the shares of
stock of the corporate vehicle while TNT would own the remaining 20%.
The agreements essentially laid down the relationship between TNT and PADC and the management,
control and existence of the corporation. Also, pursuant to the joint venture agreements, PADC and TNT
registered with the SEC a corporation to be known as Air Philippines Corporation (APC).

394 | P a g e
Law 321_Corporation LAW_ Case Digest

Subsequently, on 11 December 1992, APC amended its articles of incorporation to change its
corporate name to Pacific East Asia Cargo Airlines, Inc. (PEAC). On 02 April 1993, TNT transferred all its
shares in PEAC to petitioner GD Express. PEAC immediately commenced operations. Herein petitioner
Amihan Management Services, Inc. (Amihan), a domestic corporation, was contracted to undertake the
daily operations in PEAC pursuant to the joint venture agreement.
Sometime in 1994, the Office of the President mandated the Committee on Privatization to require
the Asset Privatization Trust (APT) to dispose of PADC’s 80% share in PEAC. Thus, petitioner GD
Express and PADC executed the Terms of Reference that would govern the disposition of PADC’s equity
comprising 12,800 subscribed shares of stock in PEAC.

ISSUE:

Whether or not an intra-corporate dispute is exclusively cognizable by the SEC.

RULING:

NO.

The designation of certain RTC branches to handle specific cases is nothing new. For instance,
pursuant to the provisions of the R.A. No. 6657 or the Comprehensive Agrarian Reform Law, the
Supreme Court has assigned certain RTC branches to hear and decide cases under Sections 56 and 57
of R.A. No. 6657.
The RTC exercising jurisdiction over an intra-corporate dispute can be likened to an RTC
exercising its probate jurisdiction or sitting as a special agrarian court. The designation of the SCCs as
such has not in any way limited their jurisdiction to hear and decide cases of all nature, whether civil,
criminal or special proceedings.
Incidentally, not all the prayers and reliefs sought by respondent Filchart in SEC Case No. 08-
97-5746 can be characterized as intra-corporate in nature. For instance, respondent Filchart’s petition
does not allege that the cause of action for the nullification of the management contract between PEAC
and petitioner Amihan is being instituted as a derivative suit. It is an ordinary action for the nullification
of a contract, which is cognizable by courts of general jurisdiction.

IGLESIA EVANGELICA METODISTA EN LAS ISLAS FILIPINAS (IEMELIF), INC.


vs.
NATANAEL B. JUANE
G.R. No. 172447. September 18, 2009

FACTS:

IEMELIF is a religious corporation existing and duly organized under Philippine laws. IEMELIF is
the absolute and registered owner of a parcel of land with Transfer Certificate of Title No. 62080
particularly described as a parcel of land with Lot No. 77-B-2. Likewise it is the absolute and registered
owner of a parcel of land with TCT No. 14366 and situated on the SE line of Calle Sande Nos. 1462-
1466, District of Tondo, Manila. On these lots the Cathedral of the Iglesia Evangelica Metodista en las
Islas Filipinas is located together with other improvements including the Pastor’s residence and the
church’s school.
Juane is a former minister or pastor of IEMELIF. He was elected as one of the members of the
Highest Consistory of Elders (or Board of Trustees) of IEMELIF in the February 2000 IEMELIF General
Conference. During the concluding Anniversary Service of said General Conference, IEMELIF Bishop
Nathanael P. Lazaro, the General Superintendent of the whole IEMELIF Church and the General
Administrator of the IEMELIF Cathedral in Tondo, Manila, during the reading of the "IEMELIF Workers’
Assignment", announced the appointment and assignment of Juane as Resident Pastor of the Cathedral
Congregation in Tondo, Manila. By virtue and as a consequence of such appointment, Defendant Rev.
Juane was authorized to stay at and occupy the Resident Pastor’s residence inside the Cathedral
complex. By the same reason, he also took charge of the Cathedral facilities and other property of the
church in said premises. One year thereafter, during the traditional concluding IEMELIF Anniversary

395 | P a g e
Law 321_Corporation LAW_ Case Digest

Service of the February 2001 General Conference, Juane was re-assigned and re-appointed by Bishop
Lazaro to the same position.

ISSUE:

Whether or not the transformation of IEMELIF from corporation sole to corporation aggregate
would change its head or governing body.

RULING:

NO.

As opposed to a corporation aggregate, a corporation sole consists of a single member, while a


corporation aggregate consists of two or more persons. If the transformation did not materialize, the
corporation sole would still be Bishop Lazaro, who himself performed the questioned acts of removing
Juane as Resident Pastor of the Tondo Congregation. If the transformation did materialize, the
corporation aggregate would be composed of the Highest Consistory of Elders, which nevertheless
approved the very same acts. As either Bishop Lazaro or the Highest Consistory of Elders had the
authority to appoint Juane as Resident Pastor of the IEMELIF Tondo Congregation, it also had the
power to remove him as such or transfer him to another congregation.

396 | P a g e
Law 321_Corporation LAW_ Case Digest

THE INTESTATE ESTATE OF ALEXANDER T. TY


vs.
COURT OF APPEALS
G.R. No. 114672 19 April 2001

FACTS:

Sylvia S. Ty was married to Alexander T. Ty. Son of private respondent Alejandro B. Ty. When
Alexander died, petitioner was appointed administratrix of her late husband’s intestate estate.
Thereafter, petitioner filed a motion for leave to sell or mortgage estate property in order to generate
funds for the payment of deficiency estate taxes. Private respondent Alejandro Ty then filed two
complaints for the the recovery of the pieces of property that were placed in the name of deceased
Alexander by private respondent, the same property being sought to be sold out, mortgaged, or disposed
of by petitioner. recovery of the above-mentioned property, praying for the declaration for nullity of the
deed of absolute sale of the shares of stock executed by private respondent in favor of the deceased
Alexander, and for Private respondent claimed in both cases that even if said property were placed in the
name of deceased Alexander, they were acquired through private respondent’s money, without any
cause or consideration from deceased Alexander.

ISSUE:

Whether or not the case is under the jurisdiction of the RTC.

RULING:

YES.

The proper forum for such a dispute is a regular trial court since in the cases at bar, the
relationship of private respondent when he sold his shares of stock to his son was one of vendor and
vendee, nothing else. The Court agrees with the ruling of the Court of appeals that no special corporate
skill is necessary in resolving the issue of the validity of the transfer of shares from one stockholder to
another of the same corporation. Both actions, although involving different property, sought to declare
the nullity of the transfers of said property to the decedent on the ground that they were not supported
by any cause or consideration, and thus, are considered void ab initio for being absolutely simulated or
fictitious. The determination whether a contract is simulated or not is an issue that could be resolved
by applying pertinent provisions of the Civil Code particularly those relative to obligations and contracts.
Disputes concerning the application of the Civil Code are properly cognizable by courts of general
jurisdiction.

397 | P a g e
Law 321_Corporation LAW_ Case Digest

HERNANI FABIA
vs.
COURT OF APPEALS
G.R. No. 132684 20 August 2001

FACTS:

Hernani N. Fabia was the President of private respondent MTCP, a domestic corporation engaged
in providing maritime courses and seminars to prospective overseas contract workers and seamen. On 3
January 1996 MTCP through its new President Exequiel B. Tamayo filed an affidavit-complaint for
estafa against Hernani N. Fabia with the Office of the City Prosecutor of Manila alleging that on various
occasions from January to July 1994 Fabia drew cash advances from MTCP, covered by cash vouchers,
which he failed to liquidate despite repeated demands. Petitioner Fabia in his 20 March 1996 Reply-
Affidavit and Motion to Dismiss admitted having received the various amounts covered by the cash
vouchers but reasoned that they were in the nature of simple loans that had already been liquidated
and paid as shown by the receipts and vouchers which he had attached to his pleadings. On 8 April
1996 the Office of the City Prosecutor of Manila dismissed the complaint for lack of jurisdiction for the
reason that the controversy pertained to the relationship between a corporation and a former officer
thereof, hence, it was the SEC which had original and exclusive jurisdiction over the case.

ISSUE:

Whether or not the case is an intra-corporate dispute.

RULING:

YES.

PD 902-A confines the jurisdiction of the SEC to "intra-corporate disputes" defined as any act or
omission of the Board of Directors/Trustees of corporations, or of partnerships, or of other associations,
or of their stockholders, officers, or partners, including any fraudulent devices, schemes or
representations, in violation of any law or rules and regulations administered and enforced by the
Commission. This underscores the relationship of the party-litigants with each other, and indicates that
the nature of the cause of action should be limited to fraudulent devices, schemes or representations, in
violation of any law, rules and/or regulations administered and enforced by the Commission for the
cause of action to fall within the ambit of authority of the SEC, elements that are both present in the
instant case. Petitioner was the President as well as a Director and stockholder in private respondent
MTCP, who was charged with the misappropriation or diversion of corporate funds after having failed to
liquidate the amount he had received as cash advances from the company. The charge against petitioner
is for estafa, an offense punishable under The RPC, and prosecution for the offense is presently before
the regular courts. However, as correctly pointed out by private respondent MTCP, jurisdiction is
determined not from the law upon which the cause of action is based, nor the type of proceedings
initiated, but rather, it is gleaned from the allegations stated in the complaint. It is evident from the
complaint that the acts charged are in the nature of an intra-corporate dispute as they involve fraud
committed by virtue of the office assumed by petitioner as President, Director, and stockholder in MTCP,
and committed against the MTCP Corporation. This sufficiently removes the action from the jurisdiction
of the regular courts, and transposes it into an intra-corporate controversy within the jurisdiction of the
SEC. The fact that a complaint for estafa, a felony punishable under the RPC, has been filed against
petitioner does not negate and nullify the intra-corporate nature of the cause of action, nor does it
transform the controversy from intra-corporate to a criminal one.
Accordingly, as the matter involves an intra-corporate dispute within the jurisdiction of the SEC,
the issue of whether prior non-accounting precludes a finding of probable cause for the charge of estafa
no longer finds relevance.
TEODORO VESAGAS
vs.
COURT OF APPEALS
G.R. No. 142924 5 December 2001

FACTS:

Plaintiffs Deflin and Raniel spouses were members in good standing of Luz VillageTennis Club
alleging that club president Vesagas summarily stripped them of their membership without due process
of law. Thus, plaintiffs moved to declare as illegal their expulsion from the club in utter disregard of the

398 | P a g e
Law 321_Corporation LAW_ Case Digest

provisions of the club’s by laws. Respondents, on the other hand, moved to dismiss the complaint on the
ground that the SEC lacked the jurisdiction over the subject matter. They contend that since its
inception in the 1970’s, the club in practice has not been a corporation, and that it was only the
plaintiffs who surreptitiously caused the club’s registration with the SEC. Further, they argued that the
club has ceased to be a corporate body at any rates thus, no intracorpoarate relationship as between the
parties.

ISSUE:

Whether or not the SEC had jurisdiction over the case.

RULING:

YES.

The case falls within the SEC jurisdiction. Petitioners are estopped from denying the personality
of the corporation by their very own acts. The dispute was considered as intra-corporate in character
because the parties involved are officers and members of the club; the conflict arose from this relation
between the parties, and the subject of complaint involved expulsions from the club membership,
validity of amendments of by laws.

399 | P a g e
Law 321_Corporation LAW_ Case Digest

SPOUSES JOSE ABEJO AND AURORA ABEJO


vs.
HON. RAFAEL DE LA CRUZ
G.R. No. L-63558 19 May 1987

FACTS:

The Spouses Abejo sold their minority shareholdings in PBPI to TSI as well as the shares of
Virginia Braga in PBPI by virtue of the stock certificates covering the latter’s shares endorsed in blank
by Braga. TSI therefore asked that the transfer be recorded in the books of the corporation. The
corporation’s treasurer, Norberto Braga, son of the Spouses Braga, refused to enter the same contending
that it has pre-emptive rights over the Abejo shares and that the certificates covering the shares of his
parent were lost. Abejo and TSI prayed for Mandamus before the SEC for Braga to enter the name of
TSI in the transfer book. Norberto Braga sought to dismiss contending that the SEC has no jurisdiction
over the nature of the action since it does not involve an intra-corporate controversy between
stockholders, the principal petitioners, TSI, not being a stockholder of record of Pocket Bell. Later, SEC
ordered that the name of TSI be entered in the books. Meanwhile, pending SEC hearing, Braga filed a
complaint before the CFI prating for nullity and rescission of the sale of the Abejos to TSI invoking his
pre-emptive right and a complaint for nullity of the sale of Virginia’s shares. Abejos sought to dismiss
the case before the CFI invoking lack of jurisdiction.

ISSUE:

Whether or not the SEC has jurisdiction over the controversy.

RULING:

YES.

SC held that the SEC has original and exclusive jurisdiction over the dispute between the
principal stockholders of the corporation PBPI, namely, the Abejos and Telectronics, the purchasers of
the 56% majority stock on the one hand, and the Bragas, erstwhile majority stockholders, on the other,
and that the SEC, through its en banc Resolution of May 15, 1984 correctly ruled in dismissing the
Bragas' petition questioning its jurisdiction, that "the issue is not the ownership of shares but rather the
nonperformance by the Corporate Secretary of the ministerial duty of recording transfers of shares of
stock of the Corporation of which he is secretary."
Section 6 further grants the SEC "in order to effectively exercise such jurisdiction," the power,
inter alia, "to issue preliminary or permanent injunctions, whether prohibitory or mandatory, in all
cases in which it has jurisdiction, and in which cases the pertinent provisions of the Rules of Court
shall apply." The very complaint of the Bragas for annulment of the sales and transfers as filed by them
in the regular court questions the validity of the transfer and endorsement of the certificates of stock,
claiming alleged preemptive rights in the case of the Abejos' shares and alleged loss of the certificates
and lack of consent and consideration in the case of Virginia Braga's shares. Such dispute clearly
involves controversies "between and among stockholders," as to the Abejos' right to sell and dispose of
their shares to Telectronics, the validity of the latter's acquisition of Virginia Braga's shares, who
between the Bragas and the Abejos' transferee should be recognized as the controlling shareholders of
the corporation, with the right to elect the corporate officers and the management and control of its
operations. Such a dispute and case clearly fall within the original and exclusive jurisdiction of the SEC.

400 | P a g e
Law 321_Corporation LAW_ Case Digest

AGUINALDO
vs.
SECURITIES EXCHANGE COMMISSION
G.R. NO. 102965 January 21, 1999

FACTS:

23% of the outstanding capital stock of NADECOR is owned by a U.S. Corporation, the Sawyer
Adecor International, Inc. Aytona, Aguinaldo, Calalang, Ricafort, and five others were elected as
directors of NADECOR by the stockholders at a meeting held and during the organization meeting held
on the same day, petitioners Aytona and Aguinaldo, and one R.H. Borsoto were elected Chairman of the
Board, President, and Corporate Secretary, respectively, of the NADECOR. Pursuant to Section 5, Article
1 of the Amended By-Laws, those present at the stockholders' meeting on August 17, 1981, formally
convened and elected private respondent Benjamin A. Aritao as acting chairman, and appointed Atty.
Eusebio V. Tan as acting secretary, for the continuation of the said stockholders' meeting. Immediately
after the stockholders' meeting, the newly-elected Board of Directors held an organization meeting at
which the following were elected as corporate officers, namely: Conrado T. Calalang, chairman and
president; Salvador O. Rivera, treasurer; and Benjamin V. Aritao, corporate secretary. Thereafter, private
respondents submitted to the respondent SEC the secretary's certificate attesting to the election of the
above-named directors and officers of the NADECOR. Private respondents filed a petition for mandamus
with prayer for preliminary injunction and/or restraining order against herein petitioners with the SEC,
praying that petitioners herein be directed to recognize the individual private respondents as the lawful
and duly elected directors and officers of the corporation, among others. SEC Director and Hearing
Officer Villanueva issued a restraining order against herein petitioners.

ISSUE:

Whether or not the SEC committed grave abuse of secretion in the issuance and continued
enforcement of the TRO, and the delay of its en banc division in resolving the petitions which also pray
for the lifting of the questioned TRO.

RULING:

YES.

Section 6 of PD 902-A grants the SEC in order to effectively exercise such jurisdiction, the power
to issue preliminary or permanent injunctions, whether prohibitory or mandatory, in all cases in which
it has jurisdiction, and in which cases the pertinent provisions of the Rules of Court shall apply. Since
the SEC is at least a co-equal body of the Regional Trial Court when it adjudicates controversies over
which it has jurisdiction, it follows that the temporary restraining order issued by SEC must have the
same life-span as that issued by the trial court. It is a well-settled rule that a temporary restraining
order issued by a trial court has a life of only 20 days. To the extent, therefore, that the enforcement of
the temporary restraining order issued by the respondent SEC exceeded 20 days, the SEC committed
grave abuse of discretion. However, although the questioned order no longer has any force and effect,
the respondent SEC still has the jurisdiction and obligation to proceed with the hearing of the case on
the merits and to issue the appropriate orders pursuant thereto subject to review by the CA and
eventually by the SC.

401 | P a g e
Law 321_Corporation LAW_ Case Digest

JOSE PENEYRA
vs.
HON. INTERMEDIATE APPELLATE COURT
G.R. No. L-68935 January 22, 1990

FACTS:

On May 7, 1976, the Board of Trustees of the Corregidor College Inc. awarded the management
and operation of its canteen at a monthly rental of P80.00 to petitioners herein who are stockholders of
the said College. Subsequently, upon instructions of Eulogio Dizon, Chairman of the Board of Trustees
of Corregidor College, Inc., the rental payments of petitioners were refused, and subsequently partial
demolition of the canteen was effected. Consequently, petitioners filed in the then Court of First Instance
an action against Eulogio R. Dizon for damages with preliminary mandatory injunction. Petitioners filed
their motion for reconsideration of the order denying the admission of their amended complaint. Two
days later, Eulogio Dizon died. Thereafter, his counsel moved to dismiss the complaint by reason thereof
which was granted. Petitioners filed a special civil action of certiorariand mandamus against
respondent judge before the IAC which later on dismissed the petition holding that the Securities and
Exchange Commission (SEC) has jurisdiction over the case, the same being an intracorporate dispute,
that the amendment to include Corregidor College, Inc. cannot be allowed and that the action for
damages against Eulogio Dizon was extinguished by his death.

ISSUE:

Whether or not the Securities and Exchange Commission has jurisdiction over the case.

RULING:

NO.

While it is true that petitioners herein are stockholders of Corregidor College, Inc., the. complaint
in Civil Case No. 774-G did not stem directly from such relationship, but rather from the award to
petitioners of the management and operation of its canteen at a monthly rental of P80.00. The
management of a canteen, even if awarded to a stockholder, is outside or merely incidental to the
central operations of an educational institution. Petitioners thus convincingly argue that "the
controversy is not one where petitioners are bringing the action as stockholders but rather as operators
of the canteen under an agreement with said Board. In short, the cause of action here is for damages
arising from a violation of a contract of management operation of the College canteen by defendant
Dizon. Certainly, the present controversy cannot qualify as an intra-controversy, its root being a
contractual breach separate and distinct from the corporate relationship between petitioners and
Corregidor College, Inc., which, it must be noted, was not even named as a defendant in the original
complaint. It was therefore patent error for the Court of Appeals to immediately rule that the present
case belongs to the SEC just because petitioners alleged that they are stockholders of Corregidor
College, Inc.
Under Section 3 of Presidential Decree 902-A, the jurisdiction of the SEC is limited to matters
intrinsically connected with the regulation of corporations, partnerships and associations and those
dealing with the internal affairs of such entities. P.D. 902-A does not confer in the SEC absolute
jurisdiction and control over all matters affecting corporations. To uphold the appellate court's ruling
would remove without legal imprimatur from the regular courts all controversies over matters involving
or affecting corporations.

402 | P a g e
Law 321_Corporation LAW_ Case Digest

MAINLAND CONSTRUCTION
vs.
MOVILLA
G. R. No. 118088 November 23, 1995

FACTS:

Mainland Construction Co., Inc. is a domestic corporation, engaged in the construction of roads
and bridges and the operation of a service shop for the maintenance of equipment. Respondents, on the
other hand, are the surviving heirs of complainant, Ernesto Movilla, who died during the pendency of an
action with the Labor Arbiter for unpaid salaries and the non-payment of other benefits. Ernesto was an
accountant of the corporation until he became an administrative manager. The Department of Labor
and Employment conducted a routine inspection on petitioner corporation and found that it committed
such violation of non-payment of salaries and other benefits. On the basis of this finding, petitioner
corporation was ordered by DOLE to pay to its thirteen employees which included Movilla, the total
amount of P309,435.89, representing their salaries, holiday pay, service incentive leave pay differentials,
unpaid wages and 13th month pay. All the employees listed in the DOLE's Order were paid by petitioner
corporation, except Ernesto Movilla. Ernesto Movilla filed a case against petitioner corporation for
unpaid wages, separation pay and attorney's fees, with the Department of Labor and Employment,
Regional Arbitration, Branch XI, Davao City. The Labor Arbiter rendered judgment dismissing the
complaint on the ground of lack of jurisdiction claiming that the case is within the jurisdiction of the
Securities and Exchange Commission being an intra corporate matter.

ISSUE:

Where or not the NLRC has jurisdiction over the controversy.

RULING:

NO.

In order that the SEC can take cognizance of a case, the controversy must pertain to any of the
following relationships: (a) between the corporation, partnership or association and the public; (b)
between the corporation, partnership or association and its stockholders, partners, members or officers;
(c) between the corporation, partnership or association and the State as far as its franchise, permit or
license to operate is concerned; and (d) among the stockholders, partners or associates themselves. The
fact that the parties involved in the controversy are all stockholders or that the parties involved are the
stockholders and the corporation, does not necessarily place the dispute within the ambit of the
jurisdiction of SEC. The better policy to be followed in determining jurisdiction over a case should be to
consider concurrent factors such as the status or relationship of the parties or the nature of the
question that is the subject of their controversy. In the absence of any one of these factors, the SEC will
not have jurisdiction. Furthermore, it does not necessarily follow that every conflict between the
corporation and its stockholders would involve such corporate matters as only the SEC can resolve in
the exercise of its adjudicatory or quasi-judicial powers.
In the case at bench, the claim for unpaid wages and separation pay filed by the complainant
against Petitioner Corporation involves a labor dispute. It does not involve an intra-corporate matter,
even when it is between a stockholder and a corporation. It relates to an employer-employee relationship
which is distinct from the corporate relationship of one with the other.
It is pertinent to note that petitioner corporation is not prohibited from hiring its corporate
officers to perform services under a circumstance which will make him an employee. Moreover, although
a director of a corporation is not, merely by virtue of his position, its employee, said director may act as
an employee or accept duties that make him also an employee.
SECURITIES AND EXCHANGE COMMISSION
vs.
COURT OF APPEALS
G.R. No. 93832 August 23, 1991

FACTS:

Petitioner Johnny K.H. Uy and private respondents Ban Hua Uy-Flores and Ban Ha Uy-Chua are
brother and sisters. They own several corporations, including LTBS Marketing Corporation and the Soon
Kee Commercial, Inc. All the three (3) above-named individuals, including other members of the Uy
family, were interlocking stockholders and officers of the two aforementioned corporations. Thus, private

403 | P a g e
Law 321_Corporation LAW_ Case Digest

respondents Ban Hua Uy-Flores and Ban Ha Uy-Chua were the managing directors of the said
corporations and were in custody of the corporate accounting and tax records as well as the funds of
UBS Marketing Corporation and Soon Kee Commercial, Inc. Private respondent Roland King is the
accountant of the said corporations and other allied Uy family enterprises. Due to serious disagreements
and conflicts, the members of the Uy family, through several conciliation meetings held before their
selected Board of Mediators, agreed to divide the family business so that the UBS Marketing Corporation
would go to petitioner Johnny K.H. Uy while the Soon Kee Commercial, Inc. would go to the rest of the
Uy family, including herein private respondents Ban Hua Uy-Flores and Ban Ha Uy-Chua. Petitioners
Johnny K.H. Uy and UBS Marketing Corporation filed with the Securities and Exchange Commission a
complaint against the private respondents for the recovery of UBS Marketing Corporation's corporate
books, books of account, and the accounting and turn over of the funds and properties belonging to
UBS Marketing Corporation. Instead of filing an answer, the private respondents moved to dismiss the
complaint 5 on the ground that the SEC had no jurisdiction over their person and over the nature of the
action because there was no intra-corporate relationship between the parties to the suit.

ISSUE:

Whether or not the SEC has jurisdiction over the dispute.

RULING:

YES.

Under Section 5 of PD No. 902-A, as amended by PD No. 1653, the SEC has original and
exclusive jurisdiction to hear and decide cases involving controversies arising out of intra-corporate or
partnership relations, between and among stockholders, members or associates; between any or all of
them and the corporation, partnership or association of which they are stockholders, members of
associates, respectively; and between such corporations, partnership or association and the state insofar
as it concerns their individual franchise or right to exist as such entity.
In the case at bar, at the time of the execution of the Deed of Assignment wherein the petitioner
Johnny K-H. Uy and his wife, Magdalena Uy, assigned all their stockholdings in Soon Kee Commercial,
Inc. to the private respondents Ban Hua Uy Flores and Ban Ha Uy-Chua and other members of the UY
family, and the Deed of Assignment wherein the private respondents Ban Hua Uy-Flores and Ban Ha
Uy-Chua, assigned all their stockholdings in UBS Marketing Corporation to the petitioner Johnny K.H.
Uy or to his wife, the petitioner Johnny KH. Uy and the private respondents Ban Hua Uy-Flores and Ban
Ha Uy-Chua were all interlocking stockholders and officers of the two (2) corporations owned by the Uy
family. Hence, the deeds of assignment were intra-corporate transactions which arose from intra-
corporate relations or between and among the stockholders of the two (2) family corporations. The
controversy subject of SEC Case No. 03328 is, therefore, an intra-corporate controversy which falls
within the original and exclusive jurisdiction of the SEC under Section 5(b) of PD No. 902-A, as
amended.

404 | P a g e
Law 321_Corporation LAW_ Case Digest

SUNSET VIEW CONDOMINIUM CORPORATION


vs.
JOSE C. CAMPOS, JR
G.R. No. L-52361 April 27, 1981

FACTS:

The private respondent, Aguilar-Bernares Realty, a sole proprietorship with business name
registered with the Bureau of Commerce, owned and operated by the spouses Emmanuel G. Aguilar and
Zenaida B. Aguilar, is the assignee of a unit, "Solana", in the Sunset View Condominium Project with La
Perla Commercial, Incorporated, as assignor. The La Perla Commercial, Incorporated bought the
"Solana" unit on installment from the Tower Builders, Inc. The petitioner, Sunset View Condominium
Corporation, filed for the collection of assessments levied on the unit against Aguilar-Bernares Realty,
private respondent. The petitioner filed its amended complaint for the collection of overdue accounts on
assessments and insurance premiums and the interest against the private respondent Lim Siu Leng to
whom was assigned a unit called "Alegria" of the Sunset View Condominium Project by Alfonso Uy who
had entered into a "Contract to Buy and Sell" with Tower Builders, Inc. over the said unit on installment
basis. The private respondent filed a motion to dismiss on the ground of lack of jurisdiction, alleging
that the amount sought to be collected is an assessment. The correctness and validity of which is
certain to involve a dispute between her and the petitioner corporation; that she has automatically
become, as a purchaser of the condominium unit, a stockholder of the petitioner pursuant to Section 2
of the Condominium Act, Republic Act No. 4726; that the dispute is intra-corporate and is consequently
under the exclusive jurisdiction of the Securities & Exchange Commission as provided in Section 5 of
P.D. No. 902-A.

ISSUES:

Whether or not the Securities & Exchange Commission has jurisdiction over cases for collection
of assessments assessed by the Condominium Corporation on condominium units the full purchase
price of which has not been paid.

RULING:

NO.

Section 5 of the Condominium Act expressly provides that the shareholding in the Condominium
Corporation will be conveyed only in a proper case. It provides that any transfer or conveyance of a unit
or an apartment, office or other space therein, shall include the transfer or conveyance of the undivided
interests in the common areas or, in a proper case, the membership or shareholding in the
condominium corporation. It is clear then that not every purchaser of a condominium unit is a
shareholder of the condominium corporation. The Condominium Act leaves to the Master Deed the
determination of when the shareholding will be transferred to the purchaser of a unit.
Pursuant to the above statutory provision, ownership of a unit is a condition sine qua nonto
being a shareholder in the condominium corporation. The private respondents, therefore, who have not
fully paid the purchase price of their units and are consequently not owners of their units are not
members or shareholders of the petitioner condominium corporation.
Inasmuch as the private respondents are not shareholders of the petitioner condominium
corporation, the instant case for collection cannot be a controversy arising out of intra corporate or
partnership relations between and among stockholders, members or associates; between any or all of
them and the corporation, partnership or association of which they are stockholders, members or
associates, respectively" which controversies are under the original and exclusive jurisdiction of the
Securities & Exchange Commission, pursuant to Section 5 (b) of P.D. No. 902- A. The subject matters of
the instant cases according to the allegations of the complaints are under the jurisdiction of the regular
courts.
WESTERN INSTITUTE OF TECHNOLOGY, INC.
vs.
SALAS
G.R. No. 113032 August 21, 1997

FACTS:

Private respondents Ricardo T. Salas, Salvador T. Salas, Soledad Salas-Tubilleja, Antonio S.


Salas, and Richard S. Salas, belonging to the same family, are the majority and controlling members of

405 | P a g e
Law 321_Corporation LAW_ Case Digest

the Board of Trustees of Western Institute of Technology, Inc., a stock corporation engaged in the
operation, among others, of an educational institution. According to petitioners, the minority
stockholders of WIT, a Special Board Meeting was held. In attendance were other members of the Board
including one of the petitioners Reginald Villasis. In said meeting, the Board of Trustees passed
Resolution No. 48, s. 1986, granting monthly compensation to the private respondents as corporate
officers retroactive June 1, 1985. A few years later, petitioners Homero Villasis, Prestod Villasis,
Reginald Villasis and Dimas Enriquez filed an affidavit-complaint against private respondents before the
Office of the City Prosecutor, as a result of which two (2) separate criminal informations, one for
falsification of a public document and the other for estafa, were filed before the Regional Trial Court. The
charge for falsification of public document was anchored on the private respondents' submission of
WIT's income statement for the fiscal year 1985-1986 with the Securities and Exchange Commission
reflecting therein the disbursement of corporate funds for the compensation of private respondents
based on Resolution No. 4, series of 1986, making it appear that the same was passed by the board on
March 30, 1986, when in truth, the same was actually passed on June 1, 1986, a date not covered by
the corporation's fiscal year 1985-1986. Thereafter, trial for the two criminal cases, was consolidated.
After a full-blown hearing, Judge Porfirio Parian handed down a verdict of acquittal on both counts
without imposing any civil liability against the accused therein. Petitioners filed a Motion for
Reconsideration of the civil aspect of the RTC Decision which was, however, denied in an Order.

ISSUE:

Whether or not the case is derivative suit correctly filed in the Regional Trial Court.

RULING:

NO.

Granting, for purposes of discussion, that this is a derivative suit as insisted by petitioners,
which it is not, the same is outrightly dismissible for having been wrongfully filed in the regular court
devoid of any jurisdiction to entertain the complaint. The ease should have been filed with the Securities
and Exchange Commission (SEC) which exercises original and exclusive jurisdiction over derivative
suits, they being intra-corporate disputes, per Section 5 (b) of P.D. No. 902-A: “In addition to the
regulatory and adjudicative functions of the Securities and Exchange Commission over corporations,
partnerships and other forms of associations registered with it as expressly granted under existing laws
and decrees, it shall have original and exclusive jurisdiction to hear and decide cases involving:
Controversies arising out of intra-corporate or partnership relations, between and among stockholders,
members, or associates; between any or all of them and the corporation, partnership or association of
which they are stockholders, members or associates, respectively; and between such corporation,
partnership or association and the State insofar as it concerns their individual franchise or right to exist
as such entity.

CONTROVERSIES IN THE ELECTION OR APPOINTMENT/DISMISSAL

RENATO REAL
vs.
SANGU PHILIPPINES, INC. and/ or KIICHI ABE
G.R. No. 168757. January 19, 2011

FACTS:

Petitioner Renato Real was the Manager of respondent corporation Sangu Philippines, Inc., a
corporation engaged in the business of providing manpower for general services, like janitors, janitresses
and other maintenance personnel, to various clients. In 2001, petitioner, together with 29 others who
were either janitors, janitresses, leadmen and maintenance men, all employed by respondent
corporation, filed their respective Complaints for illegal dismissal against the latter and respondent
Kiichi Abe, the corporation’s Vice-President and General Manager. These complaints were later on
consolidated.
With regard to petitioner, he was removed from his position as Manager through Board
Resolution 2001-033 adopted by respondent corporation’s Board of Directors. Petitioner complained that

406 | P a g e
Law 321_Corporation LAW_ Case Digest

he was neither notified of the Board Meeting during which said board resolution was passed nor
formally charged with any infraction. He just received from respondents a letter 4 dated March 26, 2001
stating that he has been terminated from service effective March 25, 2001 for the following reasons: (1)
continuous absences at his post at Ogino Philippines Inc. for several months which was detrimental to
the corporation’s operation; (2) loss of trust and confidence; and, (3) to cut down operational expenses to
reduce further losses being experienced by respondent corporation.

ISSUE:

Whether or not petitioner’s complaint for illegal dismissal constitutes an intra-corporate


controversy and thus, beyond the jurisdiction of the Labor Arbiter.

RULING:

NO.

With the elements of intra-corporate controversy being absent in this case, we thus hold that
petitioner’s complaint for illegal dismissal against respondents is not intra-corporate. Rather, it is a
termination dispute and, consequently, falls under the jurisdiction of the Labor Arbiter pursuant to
Section 217 of the Labor Code.
With the foregoing, it is clear that the CA erred in affirming the decision of the NLRC which
dismissed petitioner’s complaint for lack of jurisdiction. In cases such as this, the Court normally
remands the case to the NLRC and directs it to properly dispose of the case on the merits. "However,
when there is enough bases on which a proper evaluation of the merits of petitioner’s case may be had,
the Court may dispense with the time-consuming procedure of remand in order to prevent further
delays in the disposition of the case." ‘It is already an accepted rule of procedure for us to strive to settle
the entire controversy in a single proceeding, leaving no root or branch to bear the seeds of litigation. If,
based on the records, the pleadings, and other evidence, the dispute can be resolved by us, we will do so
to serve the ends of justice instead of remanding the case to the lower court for further proceedings."We
have gone over the records before us and we are convinced that we can now altogether resolve the issue
of the validity of petitioner’s dismissal and hence, we shall proceed to do so.

MARC II MARKETING, INC. and LUCILA V. JOSON


vs.
ALFREDO M. JOSON
G.R. No. 171993.December 12, 2011

FACTS:

Marc II Marketing, Inc. and Lucila Joson is assailing the decision of the CA for reversing and
settling aside the Resolution of the National Labor Relations Commission. Marc II Marketing, Inc. is a
corporation duly organized and existing under and by virtue of the laws of the Philippines. It is primarily
engaged in buying, marketing, selling and distributing in retail or wholesale for export or import
household appliances and products and other items. Petitioner Lucila is the President and majority
stockholder of the corporation. Before Marc II Marketing, Inc. was officially incorporated, Alfredo has
already been engaged by Lucila, in her capacity as President, to work as General Manager of the
corporation and it was formalized through the execution of a Management Contract dated in 1994 under
Marc Marketing, Inc., as Marc II Marketing, Inc. was yet to be incorporated. For occupying the said
position, respondent was among the corporation’s corporate officers by the express provision of Section
1, Article IV of its by-laws.
Alfredo was appointed as one of its officers with the designation or title of General Manager to
function as a managing director with other duties and responsibilities that the Board may provide and
authorized. However, in 1997, Marc II Marketing Inc. decided to stop and cease its operation as
evidenced by an Affidavit of Non-Operation due to poor sales collection aggravated by the inefficient
management of its affairs. Alfredo was informed of the cessation of its business operations and the
termination of his services as General Manager. He filed action for reinstatement and money claim
against petitioners.

407 | P a g e
Law 321_Corporation LAW_ Case Digest

ISSUE:

Whether or not Marc II Marketing Inc.’s Board of Directors could create a position for corporate
officers through an enabling clause found in its corporate by-laws.

RULING:

YES.

Accordingly, in determining whether the SEC (now the RTC) has jurisdiction over the controversy,
the status or relationship of the parties and the nature of the question that is the subject of their
controversy must be taken into consideration. With all the foregoing, this Court is fully convinced that,
indeed, respondent, though occupying the General Manager position, was not a corporate officer of
Petitioner Corporation rather he was merely its employee occupying a high-ranking position.
Accordingly, respondent’s dismissal as Petitioner Corporation’s General Manager did not amount
to an intra-corporate controversy. Jurisdiction therefor properly belongs with the Labor Arbiter and not
with the RTC.
Having established that respondent was not Petitioner Corporation’s corporate officer but merely
its employee, and that, consequently, jurisdiction belongs to the Labor Arbiter.

408 | P a g e
Law 321_Corporation LAW_ Case Digest

MATLING INDUSTRIAL AND COMMERCIAL CORPORATION


vs.
RICARDO R. COROS
G.R. No. 157802. October 13, 2010

FACTS:

After his dismissal by Matling as its Vice President for Finance and Administration, the
respondent filed on August 10, 2000 a complaint for illegal suspension and illegal dismissal against
Matling and some of its corporate officers (petitioners) in the NLRC, Sub-Regional Arbitration Branch
XII, Iligan City.
The petitioners moved to dismiss the complaint, raising the ground, among others, that the
complaint pertained to the jurisdiction of the Securities and Exchange Commission (SEC) due to the
controversy being intra-corporate inasmuch as the respondent was a member of Matling’s Board of
Directors aside from being its Vice-President for Finance and Administration prior to his termination.
The respondent opposed the petitioners’ motion to dismiss, insisting that his status as a member
of Matling’s Board of Directors was doubtful, considering that he had not been formally elected as such;
that he did not own a single share of stock in Matling, considering that he had been made to sign in
blank an undated indorsement of the certificate of stock he had been given in 1992; that Matling had
taken back and retained the certificate of stock in its custody; and that even assuming that he had been
a Director of Matling, he had been removed as the Vice President for Finance and Administration, not as
a Director, a fact that the notice of his termination dated April 10, 2000 showed.

ISSUE:

Whether or not the case is considered as an intra corporate controversy.

RULING:

NO.

It appears that private respondent was appointed Accounting Clerk by the Bank on July 14,
1963. From that position she rose to become supervisor. Then in 1982, she was appointed Assistant
Vice-President which she occupied until her illegal dismissal on July 19, 1991. The bank’s contention
that she merely holds an elective position and that in effect she is not a regular employee is belied by the
nature of her work and her length of service with the Bank. As earlier stated, she rose from the ranks
and has been employed with the Bank since 1963 until the termination of her employment in 1991. As
Assistant Vice President of the Foreign Department of the Bank, she is tasked, among others, to collect
checks drawn against overseas banks payable in foreign currency and to ensure the collection of foreign
bills or checks purchased, including the signing of transmittal letters covering the same. It has been
stated that "the primary standard of determining regular employment is the reasonable connection
between the particular activity performed by the employee in relation to the usual trade or business of
the employer. Additionally, "an employee is regular because of the nature of work and the length of
service, not because of the mode or even the reason for hiring them." As Assistant Vice-President of the
Foreign Department of the Bank she performs tasks integral to the operations of the bank and her
length of service with the bank totaling 28 years speaks volumes of her status as a regular employee of
the bank. In fine, as a regular employee, she is entitled to security of tenure; that is, her services may be
terminated only for a just or authorized cause. This being in truth a case of illegal dismissal, it is no
wonder then that the Bank endeavored to the very end to establish loss of trust and confidence and
serious misconduct on the part of private respondent but, as will be discussed later, to no avail.

GARCIA
vs.
EASTERN TELECOMMUNICATIONS PHILIPPINES, INC.
G.R. No. 173115 April 16, 2009

FACTS:

Atty. Virgilio R. Garcia was the Vice President and Head of Business Support Services and
Human Resource Departments of the Eastern Telecommunications Philippines, Inc. (ETPI) while Atty.
Salvador C. Hizon is the President/Chief Executive Officer. On 16 January 2000, Atty. Garcia was

409 | P a g e
Law 321_Corporation LAW_ Case Digest

placed under preventive suspension based on three complaints for sexual harassment and was
eventually dismissed though a letter by the Atty. Hizon. A complaint-affidavit for illegal dismissal with
prayer for full backwages and recovery of moral and exemplary damages was filed by Atty. Virgilio R.
Garcia against ETPI and Atty. Salvador C. Hizon. Atty. Garcia filed a Motions to Inhibit, praying that
Labor Arbiter Libo-on inhibit himself from further proceeding with the case, on the ground that he was a
fraternity brother of Atty. Hizon but said motions were denied. Upon appeal to the NLRC, the motion to
inhibit was granted and the case was re raffled to another Labor Arbiter who found the preventive
suspension and subsequent dismissal of Atty. Garcia illegal. An Alias writ of execution was issued for
the garnishment of the amount representing his monthly salaries for two months and thirteenth month
pay which was satisfied. Upon appeal, The Commission ruled that the dismissal of Atty. Garcia, being
ETPI’s Vice President, partook of the nature of an intra-corporate dispute cognizable by Regional Trial
Courts and not by Labor Arbiters.

ISSUE:

Whether or not the question of legality or illegality of the removal or termination of employment of
an officer of a corporation is an intra corporate controversy that falls under the original exclusive
jurisdiction of the Regional Trial Courts.

RULING:

YES.

A corporate officer’s dismissal or removal is always a corporate act and/or an intra-corporate


controversy, over which the Regional Trial Court has original and exclusive jurisdiction. Before a
dismissal or removal could properly fall within the jurisdiction of the SEC, it has to be first established
that the person removed or dismissed was a corporate officer. In the case the by-laws of ETPI.
Atty. Garcia tries to deny he is an officer of ETPI. Not being a corporate officer, he argues that the
Labor Arbiter has jurisdiction over the case. One of the corporate officers provided for in the by-laws of
ETPI is the Vice-President. It is therefore clear from the by-laws and from Atty. Garcia himself that he is
a corporate officer. One who is included in the by-laws of a corporation in its roster of corporate officers
is an officer of said corporation and not a mere employee. Being a corporate officer, his removal is
deemed to be an intra-corporate dispute cognizable by the SEC and not by the Labor Arbiter.
Atty. Garcia’s ouster as Vice-President, who is a corporate officer of ETPI, partakes of the nature
of an intra-corporate controversy, jurisdiction over which is vested in the SEC now the RTC. The Labor
Arbiter thus erred in assuming jurisdiction over the case filed by Atty. Garcia, because he had no
jurisdiction over the subject matter of the controversy.

410 | P a g e
Law 321_Corporation LAW_ Case Digest

ARMANDO T. DE ROSSI
vs.
NATIONAL LABOR RELATIONS COMMISSION
G.R. No. 108710 September 14, 1999

FACTS:

An Italian citizen, petitioner was the Executive Vice-President and General Manager of private
respondent, Matling Industrial and Commercial Corporation (MICC). He started work on July 1, 1985.
On August 10, 1988, MICC terminated his employment. Aggrieved, petitioner filed with the NLRC,
National Capital Region on September 21, 1989, a complaint for illegal dismissal with corresponding
damages. MICC based petitioner's dismissal on the ground that the petitioner failed to secure his
employment permit, grossly mismanaged the business affairs of the company, and misused corporate
funds. However, petitioner argued that it was the duty of the company to secure his work permit during
the term of his office, and that his termination was illegal for lack of just cause. On November 27 1991,
Labor Arbiter Asuncion rendered a decision in favor of petitioner where a writ of execution was issued to
collect the back wages of petitioner and giving MICC the option to reinstate petitioner physically or
constructively through payroll reinstatement. Upon appeal, the NLRC dismissed the case for lack of
jurisdiction.

ISSUE:

Whether or not the NLRC has jurisdiction over the dismissal case.

RULING:

NO.

The SEC, and not the NLRC, has original and exclusive jurisdiction over cases involving the
removal of corporate officers. Section 5, paragraph (c) of P.D. 902-A unequivocally provides that SEC
has jurisdiction over intra-corporate affairs regarding the election or appointment of officers of a
corporation.
An "office" is created by the charter of the corporation under which a corporation is organized,
and the officer is elected by the directors or stockholders. In the present case, private respondents aver
that the officers and their terms of office are prescribed by the corporation's by-laws.The by-laws being
in force, clearly petitioner is considered an officer of MICC, elected and/or designated by its board of
directors.
A corporate officer's removal from his office is a corporate act. If such removal occasions an intra-
corporate controversy, its nature is not altered by the reason or wisdom, or lack thereof, with which the
Board of Directors might have in taking such action. When petitioner, as Executive Vice-President
allegedly diverted company funds for his personal use resulting in heavy financial losses to the
company, this matter would amount to fraud. Such fraud would be detrimental to the interest not only
of the corporation but also of its members. This type of fraud encompasses controversies in a
relationship within the corporation covered by SEC jurisdiction. Perforce, the matter would come within
the area of corporate affairs and management, and such a corporate controversy would call for the
adjudicative expertise of the SEC, not the Labor Arbiter or the NLRC.

411 | P a g e
Law 321_Corporation LAW_ Case Digest

LESLIE W. ESPINO
vs.
NATIONAL LABOR RELATIONS COMMISSION
G.R. Nos. 109642-43 January 5, 1995

FACTS:

Petitioner Leslie W. Espino was the Executive Vice President-Chief Operating Officer of private
respondent Philippine Airlines (PAL) when his services were terminated sometime in December 1990 by
the Board of Directors of PAL as a result of the findings of the panels created by then President Corazon
C. Aquino to investigate the administrative charges filed against him and other senior officers for their
purported involvement in four, denominated "Goldair," "Robelle," "Kasbah/La Primavera," and "Middle
East" which allegedly prejudiced the interests of both PAL and the Philippine Government. Except for
the conflict of interest charges in the "Robelle" case, petitioner and several other senior officers of PAL
were uniformly charged in the three (3) other aforementioned cases of gross incompetence,
mismanagement, inefficiency, negligence, mismanagement, dereliction of duty, failure to observe and/or
implement administrative and executive policies, and related acts or omissions resulting in the
concealment or coverup and prevention of the seasonable discovery of anomalous transactions which, as
a consequence, caused prejudice to the best interest of PAL and the Government. As a result of his
termination, petitioner Espino filed a complaint for illegal dismissal against PAL with the National Labor
Relations Commission, praying, among others, for reinstatement with backwages, recovery of P50
Million as moral damages, P10 Million as exemplary damages and attorney's fees which was granted and
a writ of execution was issued.

ISSUE:

Whether or not the NLRC has jurisdiction over the case.

RULING:

NO.

In intra-corporate concerning the election or appointment of officers of a corporation, Section 5,


PD 902-A specifically provides that in addition to the regulatory and adjudicative functions of the
Securities and Exchange Commission over corporations, partnerships and other forms of associations
registered with it as expressly granted under existing laws and decrees, it shall have original and
exclusive jurisdiction to hear and decide cases involving controversies in the election or appointments of
directors, trustees, officers or managers of such corporations, partnerships or associations.
Indisputably, the position of Executive Vice President-Chief Operating Officer from which
petitioner Espino claims to have been illegally dismissed, is an elective office under Section 7, Article III
is an elective corporate office under Section 1, Article IV of the Amended by-Laws of PAL. He lost that
position when his appointment or election as Executive Vice President-Chief Operating Officer, together
with other senior officers who were similarly charged administratively, were deferred by the Board of
Directors in its organizational meeting on October 19, 1990. He was later considered by the Board as
resigned from the service, for reasons earlier stated, and the said position was later abolished.
A corporate officer's dismissal is always a corporate act and/or an intra-corporate controversy
and that nature is not altered by the reason or wisdom which the Board of Directors may have in taking
such action.Furthermore, it must be noted that the reason behind the non-election of petitioner to the
position of Executive Vice President-Chief Operating Officer arose from, or is closely connected with, his
involvement in the alleged irregularities in the aforementioned cases which, upon investigation and
recommendation, were resolved by the PAL Board of Directors against him and other senior officers.
Evidently, this intra-corporate ruling places the instant case under the specialized competence and
expertise of the SEC.
JOSEMARIA G. ESTRADA
vs.
NATIONAL LABOR RELATIONS COMMISSION
G.R. No. 106722 October 4, 1996

FACTS:

Petitioner Josemaria Estrada was the Senior Vice-President marketing Group of private
respondent Philippine Airlines Inc., responsible for the development of corporate marketing plans and
strategies of PAL and for providing direction on all passenger and cargo sales and services activities at

412 | P a g e
Law 321_Corporation LAW_ Case Digest

international and domestic airports. In June of 1990, petitioner was implicated by then Solicitor General
Francisco Chavez in the much-publicized P2 billion anomaly in PAL. Accordingly, he was
administratively charged and thereafter preventively suspended until he was finally dismissed.

ISSUE:

Whether or not the NLRC has jurisdiction over the case for illegal termination filed by petitioner.

RULING:

YES.

A corporate officer’s dismissal is always a corporate act and/or intra-corporate controversy and
that nature is not altered by the reason or wisdom which the Board of Directors may have in taking
such action. Not the least insignificant in the case at bench is that petitioners dismissal is intertwined
with still another intra-corporate affair, earlier so ascribed as the two-billion-peso PAL scam, that
inevitably places the case under the specialized competence of the SEC and well beyond the ambit of a
labor arbiters normal jurisdiction under the general provisions of Article 217 of the Labor Code.
The fact that petitioner sought payment of his back wages, other benefits, as well as moral and
exemplary damages and attorneys fees in his complaint for illegal dismissal will not operate to prevent
the SEC from exercising its jurisdiction under PD 902-A. While the affirmative reliefs and monetary
claims sought by petitioner in his complaint may, at first glance, mislead one into placing the case
under the jurisdiction of the Labor Arbiter, a closer examination reveals that they are actually part of
the perquisites of his elective position; hence, intimately linked with his relations with the corporation.
Anent the issue on estoppel, suffice it to state that there is nothing on record to show that PAL
was guilty of the same. In fact, the court noted that initially at the arbitration level, PAL already
questioned the jurisdiction of the labor Arbiter on the ground that petitioner’s recourse should have
been with the Office of the President. While the reason therein proffered by PAL may be incorrect, it did
not alter the fact that PAL indeed questioned the jurisdiction of the labor Arbiter. At any rate, our settled
rule is that jurisdiction over the subject matter is conferred by law, and may be questioned at anytime
even on appeal.

413 | P a g e
Law 321_Corporation LAW_ Case Digest

ISLAMIC DIRECTORATE OF THE PHILIPPINES


vs.
COURT OF APPEALS
G.R. No. 117897 May 14, 1997

FACTS:

Petitioner IDP-Tamano Group alleges that Islamic leaders of all Muslim major tribal groups in the
Philippines headed by Dean Cesar Adib Majul organized and incorporated the ISLAMIC DIRECTORATE
OF THE PHILIPPINES (IDP), the primary purpose of which is to establish an Islamic Center in Quezon
City for the construction of a Mosque, Madrasah, and other religious infrastructures so as to facilitate
the effective practice of Islamic faith in the area. The Libyan government donated money to the IDP to
purchase land at Culiat, Tandang Sora, Quezon City, to be used as a Center for the Islamic populace.
After the purchase of the land by the Libyan government in the name of IDP, Martial Law was declared
by the late President Ferdinand Marcos. Most of the members of the 1971 Board of Trustees flew to the
Middle East to escape political persecution. Two Muslim groups sprung, the Carpizo Group, headed by
Engineer Farouk Carpizo, and the Abbas Group, led by Mrs. Zorayda Tamano and Atty. Firdaussi
Abbas. Both groups claimed to be the legitimate IDP. Significantly, on October 3, 1986, the SEC, in a
suit between these two contending groups, came out with a Decision in SEC Case No. 2687 declaring
the election of both the Carpizo Group and the Abbas Group as IDP board members to be null and void.
Neither group, however, took the necessary steps prescribed by the SEC in its October 3, 1986
Decision, and, thus, no valid election of the members of the Board of Trustees of IDP was ever called.
Although the Carpizo Group attempted to submit a set of by-laws, the SEC found that, aside from
Engineer Farouk Carpizo and Atty. Musib Buat, those who prepared and adopted the by-laws were not
bona fide members of the IDP, thus rendering the adoption of the by-laws likewise null and void.
without having been properly elected as new members of the Board of Trustee of IDP.

ISSUE:

Whether or not the sale of two (2) parcels of land between the IDP-Carpizo Group and private
respondent INC null and void.

RULING:

YES.

There can be no question as to the authority of the SEC to pass upon the issue as to who among
the different contending groups is the legitimate Board of Trustees of the IDP since this is a matter
properly falling within the original and exclusive jurisdiction of the SEC by virtue of Sections 3 and 5(c)
of Presidential Decree No. 902-A. If the SEC can declare who is the legitimate IDP Board, then by parity
of reasoning, it can also declare who is not the legitimate IDP Board. This is precisely what the SEC did
in SEC Case No. 4012 when it adjudged the election of the Carpizo Group to the IDP Board of Trustees
to be null and void. By this ruling, the SEC in effect made the unequivocal finding that the IDP-Carpizo
Group is a bogus Board of Trustees. Consequently, the Carpizo Group is bereft of any authority
whatsoever to bind IDP in any kind of transaction including the sale or disposition of ID property.
The SEC already declared the election of the Carpizo Group as well as the Abbas Group)to the IDP
Board as null and void for being violative of the Articles of Incorporation. In this case, the IDP, owner of
the subject parcels of land, never gave its consent, thru a legitimate Board of Trustees, to the disputed
Deed of Absolute Sale executed in favor of INC. This is, therefore, a case not only of vitiated consent, but
one where consent on the part of one of the supposed contracting parties is totally wanting. Ineluctably,
the subject sale is void and produces no effect whatsoever.

414 | P a g e
Law 321_Corporation LAW_ Case Digest

BIENVENIDO ONGKINGCO
vs.
NATIONAL LABOR RELATIONS
G.R. No. 119877 March 31, 1997

FACTS:

Petitioner Galeria de Magallanes Condominium Association, Inc. is a non-stock, non-profit


corporation formed with a primary purpose to hold title to the common areas of the Galeria de
Magallanes Condominium Project and to manage and administer the same for the use and convenience
of the residents and/or owners. Petitioner Bienvenido Ongkingco was the president of Galeria at the
time private respondent filed his complaint. On 1 September 1990, Galeria's Board of Directors
appointed private respondent Federico B. Guilas as Administrator/Superintendent. He was given a
"monthly salary of P10,000.00 subject to review after five (5) months and subsequently thereafter as
Galeria's finances improved.
As Administrator, private respondent was tasked with the maintenance of the performance and
elegance of the common areas of the condominium and external appearance of the compound thereof for
the convenience and comfort of the residents as well as to keep up the quality image, and hence the
value of the investment for the owners thereof. However, through a resolution passed by the Board of
Directors of Galeria, private respondent was not re-appointed as Administrator. As a result, private
respondent instituted a complaint against petitioners for illegal dismissal and non-payment of salaries
with the NLRC.

ISSUE:

Whether or not private respondent was illegally dismissed.

RULING:

YES.

While it may be true that the termination of the complainant was effected allegedly by a
resolution of the Board of Directors of the respondent association, this did not make the dispute
intracorporate in nature. The complainant is neither a member of the association nor an officer thereof.
he is an employee of the respondent association occupying the position of administrator who is in
charge with the function of managing and administering the building or condominium owned by the
members. Indeed, there is a whale of difference between a member of the association who is a part
owner of the building and a mere employee performing managerial and administrative functions which
are necessary in the usual undertaking of the respondent Association. The complainant falls under the
second category.
It needs to be stressed that the fact that the complainant was removed by the Board of Directors
did not change the issue from an illegal dismissal case to an intracorporate one. For, what remains to be
resolved here is whether or not the complainant's removal from his position as Administrator was for a
just and valid cause and in compliance with due process. And, as the facts now stand, the issue is
within the scope of authority of the National Labor Relations Commission to resolve. As obtaining in this
case, no intracorporate controversy exists, hence, the jurisdiction of the NLRC should be sustained.

415 | P a g e
Law 321_Corporation LAW_ Case Digest

EFREN P. PAGUIO
vs.
NATIONAL LABOR RELATIONS COMMISSION
G.R. No. 147816 May 9, 2003

FACTS:

On 22 June 1992, respondent Metromedia Times Corporation entered, for the fifth time, into an
agreement with petitioner Efren P. Paguio, appointing the latter to be an account executive of the firm. 1
Again, petitioner was to solicit advertisements for "The Manila Times," a newspaper of general
circulation, published by respondent company. Petitioner, for his efforts, was to receive compensation
consisting of a 15% commission on direct advertisements less withholding tax and a 10% commission
on agency advertisements based on gross revenues less agency commission and the corresponding
withholding tax. The commissions, released every fifteen days of each month, were to be given to
petitioner only after the clients would have paid for the advertisements. Apart from commissions,
petitioner was also entitled to a monthly allowance of P2,000.00 as long as he met the P30,000.00-
monthly quota. When his services were terminated, petitioner filed for illegal dismissal.

ISSUE:

Whether or not petitioner’s contractual relationship with respondent company was one of a
regular employment and whether his dismissal was valid.

RULING:

YES.

A regular employee is one who is engaged to perform activities which are necessary and desirable
in the usual business or trade of the employer as against those which are undertaken for a specific
project or are seasonal. Even in these latter cases, where such person has rendered at least one year of
service, regardless of the nature of the activity performed or of whether it is continuous or intermittent,
the employment is considered regular as long as the activity exists, it not being indispensable that he be
first issued a regular appointment or be formally declared as such before acquiring a regular status.
That petitioner performed activities which were necessary and desirable to the business of the
employer, and that the same went on for more than a year, could hardly be denied. Petitioner was an
account executive in soliciting advertisements, clearly necessary and desirable, for the survival and
continued operation of the business of respondent corporation. Robina Gokongwei, its President, herself
admitted that the income generated from paid advertisements was the lifeblood of the newspaper's
existence. Implicitly, respondent corporation recognized petitioner's invaluable contribution to the
business when it renewed, not just once but five times, its contract with petitioner.
A lawful dismissal must meet both substantive and procedural requirements; in fine, the
dismissal must be for a just or authorized cause and must comply with the rudimentary due process of
notice and hearing. It is not shown that respondent company has fully bothered itself with either of
these requirements in terminating the services of petitioner. The notice of termination recites no valid or
just cause for the dismissal of petitioner nor does it appear that he has been given an opportunity to be
heard in his defense.

416 | P a g e
Law 321_Corporation LAW_ Case Digest

PEARSON & GEORGE, (S.E. ASIA), INC.


vs.
NLRC
G.R. No. 113928 February 1, 1996

FACTS:

The petitioner insists that the Labor Arbiter and the NLRC do not have jurisdiction over the
private respondent’s complaint for illegal dismissal arising out of his removal as Managing Director of
the petitioner due to his non-reelection and the abolition of the said position. It claims that the matter
is intra-corporate and thus falls within the exclusive jurisdiction of the Securities and Exchange
Commission pursuant to Section 5(c) of P.D. No. 902-A.
Private respondent Leopoldo Llorente was a member of the Board of Directors of the petitioner
and was elected as Vice-Chairman of the Board and as Managing Director for a term of one year and
until his successor should have been duly elected pursuant to the petitioner’s by-laws. On 29 January
1990, Llorente was preventively suspended, with pay, by reason of alleged anomalous transactions
entered by him, which were prejudicial to the interest of the petitioner. Llorente, protested his
suspension and requested an examination of the supporting documents to enable him to explain the
accusations leveled against him, but to no avail.
At the regular stockholders’ meeting on 5 March 1990, the stockholders of the petitioner elected a
new set of directors. Llorente was not reelected. On the same day, the new Board of Directors held a
meeting wherein it elected a new set of officers and abolished the position of Managing Director. The
petitioner’s counsel informed Llorente of his non-reelection, the abolition of the position of Managing
Director, and his termination for cause. Llorente filed with the Labor Arbiter a complaint for unfair labor
practice, illegal dismissal, and illegal suspension alleging therein that he was dismissed without due
process of law.

ISSUE:

Whether or not it is the NLRC which has jurisdiction over the complaint for illegal dismissal
which the private respondent had filed with the NLRC.

RULING:

NO.

The removal of Llorente as Managing Director is purely an intra-corporate dispute which falls
within the exclusive jurisdiction of the SEC and not of the NLRC. In reality, Llorente was not dismissed.
If he lost the position of Managing Director, it was primarily because he was not reelected as Director
during the regular stockholders’ meeting. The office of Managing Director presupposes that its occupant
is a Director; hence, one who is not a Director of the petitioner or who has ceased to be a Director
cannot be elected or appointed as a Managing Director. The holding of the position of Director is a
prerequisite for the election, appointment, or designation of Managing Director. If a Managing Director
should lose his position because he ceased to be a Director for any reason, such as non-reelection as in
the case of Liorente, such loss is not dismissal but failure to qualify or to maintain a prerequisite for
that position. Then too, the position of Managing Director was abolished.
Any question relating or incident to the election of the new Board of Directors, the non-reelection
of Liorente as a Director, his loss of the position of Managing Director, or the abolition of the said office
are intra-corporate matters. Disputes arising therefrom are intra-corporate disputes which, if
unresolved within the corporate structure of the petitioner, may be resolved in an appropriate action
only by the SEC pursuant to its authority under paragraphs (b) and (c), Section 5 of P.D. No. 902-A.

417 | P a g e
Law 321_Corporation LAW_ Case Digest

APODACA, petitioner
vs.
NATIONAL LABOR RELATIONS COMMISSION, respondent
G.R. No. 80039 April 18, 1989

FACTS:

Petitioner was employed in respondent corporation. On August 28, 1985, respondent Jose M.
Mirasol persuaded petitioner to subscribe to 1,500 shares of respondent corporation at P100.00 per
share or a total of P150,000.00. He made an initial payment of P37,500.00. On September 1, 1975,
petitioner was appointed President and General Manager of the respondent corporation. However, on
January 2, 1986, he resigned.
On December 19, 1986, petitioner instituted with the NLRC a complaint against private
respondents for the payment of his unpaid wages, his cost of living allowance, the balance of his
gasoline and representation expenses and his bonus compensation for 1986. Petitioner and private
respondents submitted their position papers to the labor arbiter. Private respondents admitted that
there is due to petitioner the amount of P17,060.07 but this was applied to the unpaid balance of his
subscription in the amount of P95,439.93. Petitioner questioned the set-off alleging that there was no
call or notice for the payment of the unpaid subscription and that, accordingly, the alleged obligation is
not enforceable.
In a decision dated April 28, 1987, the labor arbiter sustained the claim of petitioner for
P17,060.07 on the ground that the employer has no right to withhold payment of wages already earned
under Article 103 of the Labor Code. Upon the appeal of the private respondents to public respondent
NLRC, the decision of the labor arbiter was reversed in a decision dated September 18, 1987. The NLRC
held that a stockholder who fails to pay his unpaid subscription on call becomes a debtor of the
corporation and that the set-off of said obligation against the wages and others due to petitioner is not
contrary to law, morals and public policy.

ISSUE:

Whether or not an obligation arising from non-payment of stock subscriptions to a corporation


can be offset against a money claim of an employee against the employer.

RULING:

NO.

The unpaid subscriptions are not due and payable until a call is made by the corporation for
payment. Private respondents have not presented a resolution of the board of directors of Respondent
Corporation calling for the payment of the unpaid subscriptions. It does not even appear that a notice of
such call has been sent to petitioner by the respondent corporation.
What the records show is that the respondent corporation deducted the amount due to petitioner
from the amount receivable from him for the unpaid subscriptions. No doubt such set-off was without
lawful basis, if not premature. As there was no notice or call for the payment of unpaid subscriptions,
the same is not yet due and payable.
Lastly, assuming further that there was a call for payment of the unpaid subscription, the NLRC
cannot validly set it off against the wages and other benefits due petitioner. Article 113 of the Labor
Code allows such a deduction from the wages of the employees by the employer, only in three instances,
to wit: Wage Deduction. — No employer, in his own behalf or in behalf of any person, shall make any
deduction from the wages of his employees, except for certain instances.

PSBA
vs.
LEANO
GR L- 58468 February 24, 1984

FACTS:

Tan is one of the stockholders of PSBA. He was a director and Executive Vice- President enjoying
salaries and allowances. During a regular meeting, the Board of Directors declared all corporate

418 | P a g e
Law 321_Corporation LAW_ Case Digest

positions vacant except those of the president and chairman and at the same time elected new set of
officers. Tan was not re elected for which he filed for illegal dismissal before the NLRC. He also instituted
a one million peso damage suit before the Court of First Instance for the illegal and oppressive removal.
He lodged another complaint with the SEC questioning the validity of the elections and his ouster. The
SEC issued a subpoena duces tecum commanding the production of all corporate documents, records,
books. The Labor Arbiter also issued a subpoena duces tecum for the production of the same records
and documents. Petitioners moved for the dismissal of the complaint before the NLRC invoking the
principle against split jurisdiction.

ISSUE:

Whether or not the NLRC has jurisdiction over the case.

RULING:

NO.

PSBA is a domestic corporation duly organized and existing under our laws. General management
is vested in a Board of seven elected annually by stockholders entitled to vote, who serve until the
election and qualification of their successors. Any vacancy in the board of directors is filled up by a
majority vote of the subscribed capital stock entitled to vote at a meeting generally called for the
purpose, and the directors so chosen shall hold office for the unexpired term. Corporate officers are
provided for, among them, the Executive Vice-President, who is elected by the board from their own
number. The officers receive such salaries as the board may fix. The by-laws likewise provide that
should the office be rendered vacant by reason of death, resignation, disqualification or otherwise, the
board, by a majority vote may choose a successor who shall hold office for the unexpired term of the
predecessor.
The controversy is intra-corporate in nature. It revolves around the election of directors, officers
and managers of PSBA, the relation between and among its stockholders, and between them and the
corporation. PD 902-A vests in the SEC the original and exclusive jurisdiction to hear and decide cases
involving controversies arising out of intra-corporate relations between and among stockholders, and
between the stockholders and the corporation. It also has exclusive jurisdiction over controversies
involving the election and appointment of officers, directors, trustees or managers of such corporation.
The case is not a case of dismissal. The case is that of a corporate office having been declared
vacant and of Tan’s not having been re-elected thereafter. The matter of whom to elect is a prerogative
that belongs to the Board and involves the exercise of deliberate choice and the faculty of discriminative
selection. Generally speaking, the relationship of a person to the corporation, whether as an officer or as
agent or employee, is not determined by the services performed but by the incidents of the relationship
as they actually exist.

419 | P a g e
Law 321_Corporation LAW_ Case Digest

PURIFICACION G. TABANG
vs.
NATIONAL LABOR RELATIONS COMMISSION
G.R. No. 121143 January 21, 1997

FACTS:

Purificacion Tabang was a founding member, a member of the Board of Trustees, and the
corporate secretary of private respondent Pamana Golden Care Medical Center Foundation, Inc., a non-
stock corporation engaged in extending medical and surgical services. Medical Director and Hospital
Administrator of private respondent's Pamana Golden Care Medical Center in Calamba, Laguna.
Although the memorandum was silent as to the amount of remuneration for the position, petitioner
claims that she received a monthly retainer fee of five thousand pesos (P5,000.00) from private
respondent, but the payment thereof was allegedly stopped in November, 1991.
As medical director and hospital administrator, petitioner was tasked to run the affairs of the
aforesaid medical center and perform all acts of administration relative to its daily operations.Petitioner
was allegedly informed personally by Dr. Ernesto Naval that in a special meeting held on April 30, 1993,
the Board of Trustees passed a resolution relieving her of her position as Medical Director and Hospital
Administrator, and appointing the latter and Dr. Benjamin Donasco as acting Medical Director and
acting Hospital Administrator, respectively. Petitioner averred that she thereafter received a copy of said
board resolution. Petitioner then filed a complaint for illegal dismissal and non-payment of wages,
allowances and 13th month pay before the labor arbiter but the complaint was dismissed for lack of
jurisdiction.

ISSUE:

Whether or not the NLRC has jurisdiction over the case.

RULING:

NO.

It is the SEC which has jurisdiction over the case. The charges against private respondent
partake of the nature of an intra-corporate controversy. Similarly, the determination of the rights of
petitioner and the concomitant liability of private respondent arising from her ouster as a medical
director and/or hospital administrator, which are corporate offices, is an intra-corporate controversy
subject to the jurisdiction of the SEC. Contrary to the contention of petitioner, a medical director and a
hospital administrator are considered as corporate officers under the by-laws of respondent corporation.
The president, vice-president, secretary and treasurer are commonly regarded as the principal or
executive officers of a corporation and modern corporation statutes usually designate them as the
officers of the corporation. However, other offices are sometimes created by the charter or by-laws of a
corporation or the board of directors may be empowered under the by-laws of a corporation to create
additional offices as may be necessary.
In the case at bar, considering that herein petitioner, unlike an ordinary employee, was appointed
by Respondent Corporation’s Board of Trustees in its memorandum, she is deemed an officer of the
corporation. Section 5(c) of Presidential Decree No. 902-A, provides that the SEC exercises exclusive
jurisdiction over controversies in the election appointment of directors, trustees, officers or managers of
corporations, partnerships or associations, applies in the present dispute. Accordingly, jurisdiction over
the same is vested in the SEC, and not in the Labor Arbiter or the NLRC.
A corporate officer's dismissal is always a corporate act, or an intra-corporate controversy, and
the nature is not altered by the reason or wisdom with which the Board of Directors may have in taking
such action.

420 | P a g e
Law 321_Corporation LAW_ Case Digest

UNION MOTOR CORPORATION


vs.
NATIONAL LABOR RELATIONS COMMISSION
G.R. No. 159738 December 9, 2004

FACTS:

Alejandro Etis was hired by the petitioner as an automotive mechanic at the service department
where his latest monthly salary was P6,330.00.During his employment, he was awarded the Top
Technician for the month of May in 1995 and Technician of the Year (1995).He also became a member of
the Exclusive P40,000.00 Club and received the Model Employee Award in the same year. On September
22, 1997, the respondent made a phone call to Rosita dela Cruz, the company nurse, and informed her
that he had to take a sick leave as he had a painful and unbearable toothache. Finding that the
respondents ailment was due to a tooth inflammation, the doctor referred him to a dentist for further
management where he was scheduled to have a tooth extraction but the date was extended because
there was still inflammation. Upon instructions from the management, Mr. Dumagan, a company
security guard, visited the respondent in his house and confirmed that the latter was ill. The petitioner
issued an Inter Office Memorandum through the manager of its Human Resources Department,
terminating the services of the respondent for having incurred more than five (5) consecutive absences
without proper notification. The petitioner considered the consecutive absences of the respondent as
abandonment of office under Section 6.1.1, Article III of the Company Rules. On October 4, 1997, Dr.
Pamor successfully extracted the respondents tooth. As soon as he had recovered, the respondent
reported for work, but was denied entry into the company’s premises. He was also informed that his
employment had already been terminated.

ISSUE:

Whether or not private respondent was illegally dismissed.

RULING:

YES

The company rules do not require that the notice of an employee’s absence and the reasons
therefore be in writing and for such notice to be given to any specific office and/or employee of the
petitioner. Hence, the notice may be verbal; it is enough then that an officer or employee of the
petitioner, competent and responsible enough to receive such notice for and in behalf of the petitioner,
was informed of such absence and the corresponding reason. The evidence on record shows that the
respondent informed the petitioner of his illness through the company nurse. The security guard who
was dispatched by the petitioner to verify the information received by the company nurse, confirmed the
respondents illness. Respondent complied with the requisite of giving notice of his illness and the
reason for his absences to the petitioner.
From these disquisitions, it is clear that the absences of private respondent are justifiable. The
petitioner, likewise, failed to prove the factual basis for its dismissal of the respondent on the ground of
gross and habitual negligence under Article 282(b) of the Labor Code of the Philippines, or even under
Section 6.1.1, Rule III of the Company Rules. Dismissal is the ultimate penalty that can be meted to an
employee. Thus, it must be based on just cause and must be supported by clear and convincing
evidence. To effect a valid dismissal, the law requires not only that there be just and valid cause for
termination; it, likewise, enjoins the employer to afford the employee the opportunity to be heard and to
defend himself.

PETITIONS FOR DECLARATION IN THE STATE OF SUSPENSION OF PAYMENTS

G.R. NOS. 174457-59


EXPRESS INVESTMENTS III PRIVATE LTD. AND EXPORT DEVELOPMENT CANADA
vs.
DAYAN TELECOMMUNICATIONS, INC., THE BANK OF NEW YORK (AS TRUSTEE FOR THE
HOLDERS OF THE US$200,000,000 13.5% SENIOR NOTES OF DAYAN TELECOMMUNICATIONS,

421 | P a g e
Law 321_Corporation LAW_ Case Digest

INC.) AND ATTY. REMIGIO A. NOVAL (AS THE COURT-APPOINTED REHABILITATION RECEIVER
OF BAYANTEL)

FACTS:

Respondent Bayantel is a duly organized domestic corporation engaged in the business of


providing telecommunication services. It is 98.6% owned by Bayan Telecommunications Holdings
Corporation (BTHC), which in turn is 85.4% owned by the Lopez Group of Companies and Benpres
Holdings Corporation.
On various dates between the years 1995 and 2001, Bayantel entered into several credit
agreements. In July 1999, Bayantel issued US$200 million worth of 13.5% Senior Notes pursuant to an
Indenture dated July 22, 1999 that it entered into with The Bank of New York as trustee for the holders
of said notes.

ISSUES:

Whether or not the claims of secured and unsecured creditors should be treated pari
passu during rehabilitation.

RULING:

YES.

As between the creditors, the key phrase is "equality is equity." When a corporation threatened by
bankruptcy is taken over by a receiver, all the creditors should stand on equal footing. Not anyone of
them should be given any preference by paying one or some of them ahead of the others. This is
precisely the reason for the suspension of all pending claims against the corporation under receivership.
Instead of creditors vexing the courts with suits against the distressed firm, they are directed to file their
claims with the receiver who is a duly appointed officer of the SEC.
Since then, the principle of equality in equity has been cited as the basis for placing secured and
unsecured creditors in equal footing or in pari passu with each other during rehabilitation. In legal
parlance, pari passu is used especially of creditors who, in marshaling assets, are entitled to receive out
of the same fund without any precedence over each other.

ADVENT CAPITAL AND FINANCE CORPORATION


vs.
NICASIO I. ALCANTARA and EDITHA I. ALCANTARA
G.R. No. 183050 January 25, 2012

FACTS:
Petitioner Advent Capital and Finance Corporation (Advent Capital) filed a petition for
rehabilitation withthe Regional Trial Court (RTC) of Makati City and the RTC named Atty. Danilo L.
Concepcion as rehabilitation receiver. Upon audit of Advent Capital’s books, Atty. Concepcion found that
respondents Nicasio and Editha Alcantara (collectively, the Alcantaras) owed Advent Capital
representing trust feesthat it supposedly earned for managing their several trust accounts. Then, Atty.
Concepcion requested Belson Securities, Inc. (Belson) to deliver to him, as Advent Capital’s

422 | P a g e
Law 321_Corporation LAW_ Case Digest

rehabilitation receiver, the cash dividends that Belson held under the Alcantaras’ Trust Account. Belson
refused, however, citing the Alcantara’ objections as well as the absence of an appropriate order from
the rehabilitation Court.
Thus, Atty. Concepcion filed a motion before the rehabilitation court to direct Belson to release
the money to him and thereafter the rehabilitation court granted Atty. Concepcion’s motion and in
compliance to the order, Belson turned over the subject dividends to him.Thereafter, the Alcantaras filed
a special civil action of certiorari before the Court of Appeals (CA), seeking to annul the rehabilitation
court’s order and the CA granted the petition and directing Atty. Concepcion to account for the
dividends and deliver them to the Alcantaras.

ISSUE:

Whether or not the cash dividends held by Belson and claimed by both the petitioner and
therespondents, could be claimed by the Advent Capital upon the order of the rehabilitation court.

RULING:

NO.

The rehabilitation court has no jurisdiction to hear and adjudicate the conflicting claims of
theparties over the dividends that Belson held in trust for their owners. The rehabilitation court has
notbeen given the power to resolve ownership disputes between Advent Capital and third parties.
Neither Belson nor the Alcantaras are its debtors or creditors with interest in the rehabilitation.
Advent Capital must file a separate action for collection to recover the trust fees that it allegedly
earned and, with the trial court’s authorization if warranted, put the money in escrow for payment
towhoever it rightly belongs. Having failed to collect the trust fees at the end of each calendar quarter
asstated in the contract, all it had against the Alcantaras was a claim for payment which is a
proper subject for an ordinary action for collection. It cannot enforce its money claim by simply filing a
motionin the rehabilitation case for delivery of money belonging to the Alcantaras but in the possession
of athird party.
Rehabilitation proceedings are summary and non-adversarial in nature, and do not
contemplateadjudication of claims that must be threshed out in ordinary court proceedings. Adversarial
proceedingssimilar to that in ordinary courts are inconsistent with the commercial nature of a
rehabilitation case.The latter must be resolved quickly and expeditiously for the sake of the corporate
debtor, its creditorsand other interested parties. Thus, the "incorporate the concept of prohibited
pleadings,affidavit evidence in lieu of oral testimony, clarificatory hearings instead of the traditional
approach of receiving evidence, and the grant of authority to the court to decide the case, or any
incident, on thebasis of affidavits and documentary evidence."
Here, Advent Capital’s claim is disputed and requires a full trial on the merits. It must be
resolvedin a separate action where the Alcantaras’ claim and defenses may also be presented and heard.

SIOCHI FISHERY ENTERPRISES, INC., JUN-JUN FISHING CORPORATION, DEDE FISHING


CORPORATION, BLUE CREST AQUA-FARMS, INC., and ILOILO PROPERTY VENTURES, INC.,
vs.
BANK OF THE PHILIPPINE ISLANDS
G.R. No. 193872 October 19, 2011

FACTS:

Petitioners Siochi Fishery Enterprises, Inc., Jun-Jun Fishing Corporation, Dede Fishing
Corporation, Blue Crest Aqua-Farms, Inc. and Iloilo Property Ventures, Inc. (petitioners) are domestic
corporations of the Siochi family. Petitioners are engaged in various businesses and have interlocking
stockholders and directors. Their principal office is located at 31 Don B. Bautista Boulevard, Dampalit,
Malabon City.
In the course of their business, petitioners borrowed from respondent Bank of the Philippine
Islands (BPI) and from Ayala Life Assurance, Inc. As of 30 June 2004, petitioners’ total obligation
amounted to P85,362,262.05.
On 15 July 2004, petitioners filed with the RTC a petition for corporate rehabilitation. Petitioners
prayed that the RTC (1) issue a stay order; (2) declare petitioners in a state of suspension of payments;
(3) approve petitioners’ proposed rehabilitation plan; and (4) appoint a rehabilitation receiver.

ISSUE:

423 | P a g e
Law 321_Corporation LAW_ Case Digest

Whether or not the Court of Appeals erred in setting aside the RTC’s Order because it is within
the RTC’s discretion to disregard the procedural formalities, and the lower court has factual basis in its
finding that [petitioners] are capable of rehabilitation.

RULING:

NO.

The rehabilitation plan is an indispensable requirement in corporate rehabilitation


proceedings. Section 5 of the Rules enumerates the essential requisites of a rehabilitation plan: The
rehabilitation plan shall include (a) the desired business targets or goals and the duration and coverage
of the rehabilitation; (b) the terms and conditions of such rehabilitation which shall include the manner
of its implementation, giving due regard to the interests of secured creditors; (c) the material financial
commitments to support the rehabilitation plan; (d) the means for the execution of the rehabilitation
plan, which may include conversion of the debts or any portion thereof to equity, restructuring of the
debts, dacion en pago, or sale of assets or of the controlling interest; (e) a liquidation analysis that
estimates the proportion of the claims that the creditors and shareholders would receive if the debtor’s
properties were liquidated; and (f) such other relevant information to enable a reasonable investor to
make an informed decision on the feasibility of the rehabilitation plan.
With respect to the Appraisal Report, it bears to stress that the same was commissioned by
respondent corporations and petitioner was not afforded the opportunity to contest the same. Also, it is
extant from the records that some of the properties included therein do not belong to respondent
corporations but to their officers, namely, Ferdinand Siochi, Mario Siochi, Jr., Gerald Siochi and Jose
Patrick Siochi. Thus, these properties should not be considered as part of respondent corporations’
assets as their officers have a separate personality from the corporation itself. In turn, this renders
doubtful their declaration in their Rehabilitation Plan that they have "sufficient collaterals to back-up
their bank loans.

JOSE MARCEL PANLILIO, ERLINDA PANLILIO, NICOLE MORRIS and MARIO T. CRISTOBAL
vs.
REGIONAL TRIAL COURT, BRANCH 51, CITY OF MANILA, represented by HON. PRESIDING
JUDGE ANTONIO M. ROSALES; PEOPLE OF THE PHILIPPINES; and the SOCIAL SECURITY
SYSTEM
G.R. No. 173846. February 2, 2011

FACTS:

On October 15, 2004, Jose Marcel Panlilio, Erlinda Panlilio, Nicole Morris and Marlo Cristobal
(petitioners), as corporate officers of Silahis International Hotel, Inc. (SIHI), filed with the Regional Trial
Court (RTC) of Manila, Branch 24, a petition for Suspension of Payments and Rehabilitation 4 in SEC
Corp. Case No. 04-111180.
On October 18, 2004, the RTC of Manila, Branch 24, issued an Orderstaying all claims against SIHI
upon finding the petition sufficient in form and substance.
The Court shares the view of the private complainants and the SSS that the said stay order does
not include the prosecution of criminal offenses. Precisely, the law "criminalizes" the non-remittance of
SSS contributions by an employer to protect the employees from unscrupulous employers. Clearly, in
these cases, public interest requires that the said criminal acts be immediately investigated and
prosecuted for the protection of society.

ISSUE:

Whether or not the suspension of "all claims" as an incident to a corporate rehabilitation also
contemplate the suspension of criminal charges filed against the corporate officers of the distressed
corporation.

RULING:

424 | P a g e
Law 321_Corporation LAW_ Case Digest

NO.

The prosecution of the officers of the corporation has no bearing on the pending rehabilitation of
the corporation, especially since they are charged in their individual capacities. Such being the case, the
purpose of the law for the issuance of the stay order is not compromised, since the appointed
rehabilitation receiver can still fully discharge his functions as mandated by law. It bears to stress that
the rehabilitation receiver is not charged to defend the officers of the corporation. If there is anything
that the rehabilitation receiver might be remotely interested in is whether the court also rules that
petitioners are civilly liable. Such a scenario, however, is not a reason to suspend the criminal
proceedings, because as aptly discussed in Rosario, should the court prosecuting the officers of the
corporation find that an award or indemnification is warranted, such award would fall under the
category of claims, the execution of which would be subject to the stay order issued by the rehabilitation
court. The penal sanctions as a consequence of violation of the SSS law, in relation to the revised penal
code can therefore be implemented if petitioners are found guilty after trial.
However, any civil indemnity awarded as a result of their conviction would be subject to the stay
order issued by the rehabilitation court. Only to this extent can the order of suspension be considered
obligatory upon any court, tribunal, branch or body where there are pending actions for claims against
the distressed corporation.

425 | P a g e
Law 321_Corporation LAW_ Case Digest

RICARDO V. CASTILLO
vs.
UNIWIDE WAREHOUSE CLUB, INC
G.R. No. 169725 April 30, 2010

FACTS:

Petitioner filed for illegal dismissal against respondents Uniwide Warehouse Club, Inc. and its
president, Jimmy N. Gow. The complaint contained a prayer for the payment of worked Saturdays for
the year 2001; holiday pay; separation pay; actual, moral and exemplary damages; and attorney's fees.
However, almost two months from the filing of the Complaint, respondents submitted a Motion to
Suspend Proceedings on the ground that in June 1999, the Uniwide Group of Companies had petitioned
the Securities and Exchange Commission for suspension of payments and for approval of its proposed
rehabilitation plan. It appears that on June 29, 1999, the SEC had ruled favorably on the petition and
ordered that all claims, actions and proceedings against herein respondents pending before any court,
tribunal, board, office, body or commission be suspended, and that following the appointment of an
interim receiver, the suspension order had been extended to until February 7, 2000. On April 11, 2000,
the SEC declared the Uniwide Group of Companies to be in a state of suspension of payments and
approved its rehabilitation plan. Labor Arbiter Lilia S. Savari denied the Motion to Suspend Proceedings
in the present case. Respondents lodged an appeal with the NLRC which sustained the Labor Arbiter
and held that as early as February 7, 2000 the suspension order of the SEC should be considered lifted
already and that with the approval of the rehabilitation plan, the suspension of the proceedings in the
instant labor case would no longer be necessary.

ISSUE:

Whether or not the illegal dismissal case proceedings should be suspended.

RULING:

NO.

Corporate rehabilitation connotes the restoration of the debtor to a position of successful


operation and solvency, if it is shown that its continued operation is economically feasible and its
creditors can recover by way of the present value of payments projected in the rehabilitation plan, more
if the corporation continues as a going concern than if it is immediately liquidated. It contemplates a
continuance of corporate life and activities in an effort to restore and reinstate the corporation to its
former position of successful operation and solvency, the purpose being to enable the company to gain a
new lease on life and allow its creditors to be paid their claims out of its earnings.
An essential function of corporate rehabilitation is the mechanism of suspension of all actions
and claims against the distressed corporation, which operates upon the due appointment of a
management committee or rehabilitation receiver. Jurisprudence is settled that the suspension of
proceedings referred to in the law uniformly applies to "all actions for claims" filed against a corporation,
partnership or association under management or receivership, without distinction, except only those
expenses incurred in the ordinary course of business. The reason behind the imperative nature of a
suspension or stay order in relation to the creditors' claims cannot be downplayed, for indeed the
indiscriminate suspension of actions for claims intends to expedite the rehabilitation of the distressed
corporation by enabling the management committee or the rehabilitation receiver to effectively exercise
its/his powers free from any judicial or extrajudicial interference that might unduly hinder or prevent
the rescue of the debtor company. To allow such other actions to continue would only add to the burden
of the management committee or rehabilitation receiver, whose time, effort and resources would be
wasted in defending claims against the corporation, instead of being directed toward its restructuring
and rehabilitation.

426 | P a g e
Law 321_Corporation LAW_ Case Digest

PACIFIC WIDE REALTY AND DEVELOPMENT CORPORATION


vs.
PUERTO AZUL LAND, INC.
G.R. No. 178768. November 25, 2009

FACTS:

Puerto Azul Land, Inc. (PALI) is the owner and developer of the Puerto Azul Complex situated in
Ternate, Cavite. Its business involves the development of Puerto Azul into a satellite city with residential
areas, resort, tourism and retail commercial centers with recreational areas. In order to finance its
operations, it obtained loans from various banks, the principal amount of which amounted to Six
Hundred Forty Million Two Hundred Twenty-Five Thousand Three Hundred Twenty-Four Pesos
(P640,225,324.00). PALI and its accommodation mortgagors, i.e., Ternate Development Corporation
(TDC), Ternate Utilities, Inc. (TUI), and Mrs. Trinidad Diaz-Enriquez, secured the loans.
In the beginning, PALI’s business did very well. However, it started encountering problems when
the Philippine Stock Exchange rejected the listing of its shares in its initial public offering which sent a
bad signal to the real estate market. This resulted in potential investors and real estate buyers shying
away from the business venture. The situation was aggravated by the 1997 Asian financial crisis and
the decline of the real estate market. Consequently, PALI was unable to keep up with the payment of its
obligations, both current and those that were about to fall due. One of its creditors, the Export and
Industry Bank (EIB), later substituted by Pacific Wide Realty and Development Corporation (PWRDC),
filed foreclosure proceedings on PALI’s mortgaged properties. Thrust to a corner, PALI filed a petition for
suspension of payments and rehabilitation, accompanied by a proposed rehabilitation plan and three (3)
nominees for the appointment of a rehabilitation receiver.
On December 13, 2005, the RTC rendered a Decision approving PALI’s petition for suspension of
payments and rehabilitation.

ISSUE:

Whether or not the terms of the rehabilitation plan are unreasonable and in violation of the non-
impairment clause.

RULING:

NO.

An indispensable requirement in the rehabilitation of a distressed corporation is the rehabilitation plan,


and Section 5 of the Interim Rules of Procedure on Corporate Rehabilitation provides the requisites
thereof. The Court finds nothing onerous in the terms of PALI’s rehabilitation plan. The Interim Rules on
Corporate Rehabilitation provides for means of execution of the rehabilitation plan, which may include,
among others, the conversion of the debts or any portion thereof to equity, restructuring of the debts,
dacion en pago, or sale of assets or of the controlling interest.
The restructuring of the debts of PALI is part and parcel of its rehabilitation. Moreover, per
findings of fact of the RTC and as affirmed by the CA, the restructuring of the debts of PALI would not be
prejudicial to the interest of PWRDC as a secured creditor. There is nothing unreasonable or onerous
about the 50% reduction of the principal amount when, as found by the court a quo, a Special Purpose
Vehicle (SPV) acquired the credits of PALI from its creditors at deep discounts of as much as 85%.
Meaning, PALI’s creditors accepted only 15% of their credit’s value. Stated otherwise, if PALI’s creditors
are in a position to accept 15% of their credit’s value, with more reason that they should be able to
accept 50% thereof as full settlement by their debtor. They also find no merit in PWRDC’s contention
that there is a violation of the impairment clause. Section 10, Article III of the Constitution mandates
that no law impairing the obligations of contract shall be passed. This case does not involve a law or an
executive issuance declaring the modification of the contract among debtor PALI. or have opposed the
plan or whether or not their claims have been scheduled.
PHILIPPINE NATIONAL BANK and EQUITABLE PCI BANK
vs.
COURT OF APPEALS
G.R. No. 165571 January 20, 2009

FACTS:

Petitioners are members of the consortium of creditor banks acting as trustee for the consortium.
Private respondents filed with the SEC a verified petition for rehabilitation with prayer for suspension of

427 | P a g e
Law 321_Corporation LAW_ Case Digest

actions and proceedings pending rehabilitation stating that they possess sufficient properties to cover
their obligations but foresee inability to pay them within a period of one year. They cited the sudden
non-renewal and/or massive withdrawal by creditors of their loans to ASB Holdings, the glut in the real
estate market, severe drop in the sale of real properties, peso devaluation, and decreased investor
confidence in the economy which resulted in the non-completion of and failure to sell their projects and
default in the servicing of their credits as they fell due. The ASB Group had assets worth PhP
19,410,000,000 and liabilities worth PhP 12,700,000,000.
Faced with at least 712 creditors, 317 contractors/suppliers, and 492 condominium unit buyers,
and the prospect of having secured and non-secured creditors press for payments and threaten to
initiate foreclosure proceedings, the ASB Group pleaded for suspension of payments while working for
rehabilitation with the help of the SEC. Finding the petition sufficient in form and substance, the SEC
Hearing Panel issued an order suspending for 60 days all actions for claims against the ASB Group,
enjoining the latter from disposing its properties in any manner except in the ordinary course of
business and from paying outstanding liabilities, and appointing Atty. Monico V. Jacob as interim
receiver of the ASB Group. Atty. Jacob was later replaced by Atty. Fortunato Cruz as interim receiver.
The consortium of creditor banks, which included petitioners, filed their Opposition praying for the
dismissal of the petition.

ISSUE:

Whether or not the rehabilitation proceedings is merely a temporary suspension of payments of


obligations falling due by the distressed corporation and not cancellation or repudiation of those
contractual obligations.

RULING:

NO.

The mere fact that the ASB Group averred that it has sufficient assets to cover its obligations
does not make it "solvent" enough to prevent it from filing a petition for rehabilitation. A corporation may
have considerable assets but if it foresees the impossibility of meeting its obligations for more than one
year, it is considered as technically insolvent. The period referred to the corporation’s inability to pay its
obligations; when such inability extends beyond one year, the corporation is considered technically
insolvent. Said inability may be established from the start by way of a petition for rehabilitation, or it
may be proved during the proceedings for suspension of payments, if the latter was the first remedy
chosen by the ailing corporation. If the corporation opts for a direct petition for rehabilitation on the
ground of technical insolvency, it should show in its petition and later prove during the proceedings that
it will not be able to meet its obligations for longer than one year from the filing of the petition.
Appointment of an interim receiver becomes automatic. By that statutory provision, it is clear
that the approval of the Rehabilitation Plan and the appointment of a rehabilitation receiver merely
suspend the actions for claims against respondent corporations. Petitioner bank’s preferred status over
the unsecured creditors relative to the mortgage liens is retained, but the enforcement of such
preference is suspended.

PRYCE CORPORATION
vs.
COURT OF APPEALS
G.R. No. 172302 February 04, 2008

FACTS:

Pryce Corporation has its primary purpose to develop real estate in Mindanao. It engaged in the
development of memorial parks, operated a major hotel in Cagayan de Oro City, and produced industrial
gases. Asian financial crisis, however, badly affected petitioner’s operations, resulting in heavy losses. It
could not meet its obligations as they became due. It incurred losses of P943.09 million in 2001,
P479.05 million in 2002, and P125.86 million in 2003. Thus petitioner filed a petition for rehabilitation
where it prayed for the appointment of a Rehabilitation Receiver from among the nominees named
therein and the staying of the enforcement of all claims, monetary or otherwise against it. Petitioner also
prayed that after due hearing, its proposed Rehabilitation Plan be approved. Some of the proposed
rehabilitation plans were that the bank creditors will be paid through dacion en pago of assets already
mortgaged to them, in case of insufficiency, the deficiency shall be settled by way of dacion of memorial
park lots owned by the petitioner, and that all penalties shall be waived by the creditors. The creditors
opposed the petition.

428 | P a g e
Law 321_Corporation LAW_ Case Digest

ISSUE:

Whether or not the petition for rehabilitation of petitioner Pryce Corporation be granted.

RULING:

YES.

Section 6 of the Interim Rules of Procedure on Corporate Rehabilitation provides among others
that if the court finds the petition to be sufficient in form and substance, it shall, not later than five days
from the filing of the petition, issue an Order fixing the initial hearing on the petition not earlier than
forty five days but not later than sixty days from the filing thereof; directing all creditors and all
interested parties to file and serve on the debtor a verified comment on or opposition to the petition, with
supporting affidavits and documents, not later than ten days before the date of the initial hearing and
putting them on notice that their failure to do so will bar them from participating in the proceedings;
and directing the creditors and interested parties to secure from the court copies of the petition and its
annexes within such time as to enable themselves to file their comment on or opposition to the petition
and to prepare for the initial hearing of the petition.
In the case at bench, when the commercial court appointed a rehabilitation receiver, the very next
day after the filing of the Petition for Rehabilitation, it is highly doubtful and well-nigh impossible, that,
without any hearing yet held, the commercial court could have already gathered enough evidence before
it to determine whether there was any imminent danger of dissipation of assets or of paralization of
business operations to warrant the appointment of a rehabilitation receiver.In determining whether
petitioner’s financial situation is serious and whether there is a clear and imminent danger that it will
lose its corporate assets, the RTC, acting as commercial court, should conduct a hearing wherein both
parties can present their respective evidence. Hence, a remand of the records of this case to the RTC is
imperative.

429 | P a g e
Law 321_Corporation LAW_ Case Digest

UNIWIDE HOLDINGS INC.


vs.
JANDECS TRANSPORTATION
GR 168522 DECEMBER 19, 2007

FACTS:

Petitioner and respondent Jandecs Transportation Co., Inc. entered into a contract of Assignment
of Leasehold Rights under which the latter was to operate food and snack stalls at petitioner's Uniwide
Coastal Mall in Parañaque City. The contract was for a period of 18 years, commencing October 1, 1997
up to September 30, 2015, for a consideration of P2,460,630.15. The parties also agreed that
respondent's stalls would be located near the movie houses and would be the only stalls to sell food and
beverages in that area.Respondent paid the contract price in full. Petitioner, however, failed to turn over
the stall units as agreed upon. Respondent sought the rescission of the contract and the refund of its
payment. Petitioner refused both.Respondent filed a complaint for breach of contract, rescission of
contract, damages and issuance of a writ of preliminary attachment. Petitioner filed for the suspension
of proceedings praying that the action in Court be held in abeyance in view of the SEC's order of
suspension of payments and approval of its rehabilitation plan.

ISSUE:

Whether or not the proceedings be suspended.

RULING:

NO.

The reason for suspending actions for claims against the corporation is not really to enable the
management committee or the rehabilitation receiver to substitute the corporation in any pending action
against it before any court, tribunal, board or body but to enable the management committee or the
rehabilitation receiver to effectively exercise its powers free from any judicial or extra-judicial
interference that might unduly hinder or prevent the “rescue” of the debtor corporation. To allow such
other action to continue would only add to the burden of the management committee or rehabilitation
receiver, whose time, effort and resources would be wasted in defending claims against the corporation
instead of being directed toward its restructuring and rehabilitation.
The power to rescind obligations is implied in reciprocal ones, in case one of the obligors should
not comply with what is incumbent upon him. The injured party may choose between the fulfillment and
the rescission of the obligation, with the payment of damages in either case. He may also seek
rescission, even after he had chosen fulfillment, if the latter should become impossible.
Certainly, petitioner's failure to deliver the units on the commencement date of the lease on
October 1, 1997 gave respondent the right to rescind the contract after the latter had already paid the
contract price in full. Furthermore, respondent's right to rescind the contract cannot be prevented by
the fact that petitioner had the option to substitute the stalls. Even if petitioner had that option, it did
not, however, mean that it could insist on the continuance of the contract by forcing respondent to
accept the substitution. Neither did it mean that its previous default had been obliterated completely by
the exercise of that option.

430 | P a g e
Law 321_Corporation LAW_ Case Digest

BANK OF THE PHILIPPINE ISLANDS


vs.
SECURITIES OF EXCHANGE COMMISSION
G.R. No. 164641 December 20, 2007

FACTS:

The Bank of the Philippine Islands through its predecessor-in- interest, Far East Bank and Trust
Company extended credit accommodations to the ASB Group with an outstanding aggregate principal
amount of P86,800,000.00, secured by a real estate mortgage over two properties. The ASB Group filed
a petition for rehabilitation and suspension of payments before the SEC. The Rehabilitation Plan
provides, among others, a dacion en pagoby the ASB Group to BPI of one of the properties mortgaged to
the latter at the ASB Group as selling value of P84,000,000.00 against the total amount of the ASB
Group’s exposure to the bank. In turn, ASB Group would require the release of the other property
mortgaged to BPI, to be thereafter placed in the asset pool. BPI opposed the Rehabilitation Plan and
moved for the dismissal of the ASB Group’s petition for rehabilitation as it violates its rights as creditor.
However, the SEC hearing panel issued an order approving ASB Group’s proposed rehabilitation plan
and appointed Mr. Fortunato Cruz as rehabilitation receiver.

ISSUE:

Whether or not the Rehabilitation plan violates the rights of BPI as creditor.

RULING:

NO.

Rehabilitation proceedings in our jurisdiction, much like the bankruptcy laws of the United
States, have equitable and rehabilitative purposes. On the one hand, they attempt to provide for the
efficient and equitable distribution of an insolvent debtor’s remaining assets to its creditors; and on the
other, to provide debtors with a "fresh start" by relieving them of the weight of their outstanding debts
and permitting them to reorganize their affairs. It is to effect a feasible and viable rehabilitation by
preserving a foundering business as going concern, because the assets of a business are often more
valuable when so maintained than they would be when liquidated.
The Court reiterates that the SEC’s approval of the Rehabilitation Plan did not impair BPI’s right
to contract. Besides, the mere fact that the Rehabilitation Plan proposes a dacion en pago approach
does not render it defective on the ground of impairment of the right to contract. Dacion en pago is a
special mode of payment where the debtor offers another thing to the creditor who accepts it as
equivalent of payment of an outstanding debt. The undertaking really partakes in a sense of the nature
of sale, that is, the creditor is really buying the thing or property of the debtor, the payment for which is
to be charged against the debtor’s debt. As such, the essential elements of a contract of sale, namely;
consent, object certain, and cause or consideration must be present. Being a form of contract, the
dacion en pago agreement cannot be perfected without the consent of the parties involved.
Thus, if BPI does not find the dacion en pagomodality acceptable, the ASB Group can propose to
settle its debts at such amount as is equivalent to the selling price of the mortgaged properties. If BPI
still refuses this option, it can assert its rights in the liquidation and distribution of the ASB Group’s
assets. It will not lose its status as a secured creditor, retaining its preference over the other creditors.

431 | P a g e
Law 321_Corporation LAW_ Case Digest

PHILIPPINE AIRLINES
vs.
HEIRS of ZAMORA
G.R. No. 164267 November 23, 2007

FACTS:

Zamora was a cargo representative assigned at the International Cargo Operations - Import
Operations Division of petitioner Philippine Airlines, Inc. Zamora received a Memorandum informing
him of his temporary transfer to the Domestic Cargo Operations (DCO) Zamora refused to follow the
directive because: first, there was no valid and legal reason for his transfer; second, the transfer
violated the collective bargaining agreement between the management and the employees union that no
employee shall be transferred without just and proper cause; and third, the transfer did not comply with
the 15-day prior notice rule. The transfer was for the purpose of diffusing the tension between him and
his immediate superior. The management issued several directives informing Zamora of his transfer.
However, Zamora refused to receive these and continued reporting to the ICO-IOD. Consequently, he
was reported absent at the DCO. His salaries were subsequently withheld. He also ignored the
management’s directive requiring him to explain in writing his continued absence. He was informed of
his termination due to Insubordination/Neglect of Customer, Disrespect to Authority, and AWOL.

ISSUES:

Whether or not the proceedings should be suspended for the rehabilitation of the petitioner.

RULING:

YES.

Petitioner had been placed by the Securities and Exchange Commission under a Permanent
Rehabilitation Receiver. Such being the case, a suspension of all actions for claims against petitioner
pending before any court, tribunal or board was, ipso jure, in order. The suspension of all actions for
claims against a corporation embraces all phases of the suit, be it before the trial court or any tribunal
or before this Court. No other action may be taken, including the rendition of judgment during the state
of suspension. It must be stressed that what are automatically stayed or suspended are the proceedings
of a suit and not just the payment of claims during the execution stage after the case had become final
and executory. Once the process of rehabilitation, however, is completed, this Court will proceed to
complete the proceedings on the suspended actions.
Furthermore, the actions that are suspended cover all claims against the corporation whether for
damages founded on a breach of contract of carriage, labor cases, collection suits or any other claims of
a pecuniary nature. No exception in favor of labor claims is mentioned in the law.

432 | P a g e
Law 321_Corporation LAW_ Case Digest

ALEMAR'S SIBAL & SONS, INC.


vs.
JESUS M. ELBINIAS
G.R. No. 75414 June 4, 1990

FACTS:

Private respondent G.A. Yupangco and Co. Inc. filed an action for collection of a sum of money
with prayer for damages and preliminary attachment against Alemar's Bookstore, a business entity
owned and managed by petitioner Alemar's Sibal & Sons, Inc. Subsequently Ledesma, Saludo and
Associates, as intervenor-movant, filed an omnibus motion informing the respondent trial court that the
petitioner Alemar's has been placed under rehabilitation receivership by the Securities and Exchange
Commission and that movant has been appointed as its receiver. In its opposition, G.A. Yupangco
maintained that it received notice of the receivership only on January 10, 1985 or after one month after
the collection suit. It further averred that the motion to intervene by the receiver was not seasonably
made. G.A. Yupangco urged the issuance of a writ of execution to implement the default judgment which
had become final and executory, there being no motion for reconsideration or appeal. The corresponding
writ was issued on January. Petitioner Alemar's moved for the discharge of the writ on the ground that
its issuance was improper since the proceedings have been suspended pursuant to the court order.

ISSUE:

Whether or not respondent court can validly proceed with the execution of a final decision for the
payment of a sum of money despite the fact that the judgment debtor has been placed under
receivership.

RULING:

YES.

It is the general rule that once a decision becomes final and executory, its enforcement becomes
the ministerial duty of the court. Equally settled is that the rule admits of certain exceptions, one of
which is where it becomes imperative in the higher interest of justice to direct the deferment of
execution. In the instant case, the stay of execution is warranted by the fact that petitioner has been
placed under rehabilitation receivership. It must be stressed that the SEC had earlier ordered the
suspension of all actions for claims against Alemar's in order that all the assets of said petitioner could
be inventoried and kept intact for the purpose of ascertaining an equitable scheme of distribution among
its creditors.
During rehabilitation receivership, the assets are held in trust for the equal benefit of all creditors
to preclude one from obtaining an advantage or preference over another by the expediency of an
attachment, execution or otherwise. As between creditors, the key phrase is equality is equity. When a
corporation threatened by bankruptcy is taken over by a receiver, all the creditors should stand on an
equal footing. Not anyone of them should be given any preference by paying one or some of them ahead
of the others. This is precisely the reason for the suspension of all pending claims against the
corporation under receivership. Instead of creditors vexing the courts with suits against the distressed
firm, they are directed to file their claims with the receiver who is a duly appointed officer of the SEC.

433 | P a g e
Law 321_Corporation LAW_ Case Digest

BAROTAC SUGAR MILLS, INC.


vs.
COURT OF APPEALS
G.R. No. 123379 July 15, 1997

FACTS:

Private respondent Pittsburgh Trade Center Co., Inc., filed a complaint for a sum of money
against BAROTAC. Instead of filing an answer, BAROTAC filed a Motion to Suspend Proceedings on the
ground that a Petition for Suspension of Payments With Prayer for the Appointment of a Management or
Rehabilitation Committee had been filed with the Securities and Exchange Commission. This motion
met opposition from PITTSBURGH. The court issued an Order denying petitioner’s motion ruling that
upon the filing of the petition, the SEC has not yet placed it under receivership. At the time the
Complaint in the instant case was filed with the respondent court, there was no order yet from the SEC
for the appointment of a management or rehabilitation committee or that which will indicate that
petitioner had been placed under management or receivership.

ISSUE:

Whether or not the mere filing with the SEC of such petition suspends the proceedings in the
RTC.

RULING:

NO.

The appointment of a management committee or rehabilitation receiver may only take place after
the filing with the SEC of an appropriate petition for suspension of payments. This is clear from a
reading of sub-paragraph (d) of Section 5 and sub-paragraph (d) of Section 6 of P.D. No. 902-A, as
amended by P.D. Nos. 1653 and 1758.
The conclusion then is inevitable that pursuant to the underscored proviso in sub-paragraph (c)
of the aforementioned Section 6, taken together with sub-paragraph (d) of Section 5 and sub-paragraph
(d) of Section 6, a court is ipsojure suspended only upon the appointment of a management committee
or a rehabilitation receiver. Since there is no showing at all that a management committee or
rehabilitation receiver for BAROTAC has been appointed by the SEC, suspension of the proceedings
before the RTC of Quezon City is not warranted.

434 | P a g e
Law 321_Corporation LAW_ Case Digest

BF HOMES, INCORPORATED
vs.
COURT OF APPEALS
G.R.No. 77143 October 3, 1990
G.R. No. 76879 October 3, 1990

FACTS:

BF Homes, Inc. is a domestic corporation previously engaged in the business of developing and
selling residential lots and houses and other related realty matters. On July 19, 1984, BF contracted a
loan from Rosalinda R. Roa and Vicente Mendoza in the amount of P250,000.00 with interest at the rate
of 33% per annum payable after 32 days. The obligation was embodied in a promissory note and
secured by two post-dated checks issued by BF in favor of the lenders. On September 25, 1984, BF filed
a Petition for Rehabilitation and for a Declaration in a State of Suspension of Payments under Sec. 5(d)
of P.D. No. 902-A with a prayer that upon the filing of the petition and in the meantime, all claims
against it for any and all accounts or indebtedness be suspended, but allowing petitioner to continue
with its normal operations. It also asked for the approval of the proposed rehabilitation plan. Roa and
Mendoza filed a complaint against BF for the recovery of the loan of P250,000.00, with interest and
attorney's fees. The complaint also prayed for the issuance of a writ of preliminary attachment against
the properties of BF. The trial court issued the writ against properties of BF sufficient to satisfy the
principal claim in the amount of P257,333.33. The SEC, finding an urgent need to rehabilitate BF
issued an order creating a management committee and suspending all actions for claims against BF
pending before any court, tribunal or board.

ISSUE:

Whether or not the action for collection be suspended pending the outcome of the rehabilitation
proceedings.

RULING:

YES.

Under Sec. 6(d) of P.D. No. 902-A, the management committee or rehabilitation receiver is
empowered to take custody and control of all existing assets and properties of such corporations under
management; to evaluate the existing assets and liabilities, earnings and operations of such
corporations; to determine the best way to salvage and protect the interest of investors and creditors; to
study, review and evaluate the feasibility of continuing operations and restructure and rehabilitate such
entities if determined to be feasible by the SEC.
In light of these powers, the reason for suspending actions for claims against the corporation
should not be difficult to discover. It is not really to enable the management committee or the
rehabilitation receiver to substitute the defendant in any pending action against it before any court,
tribunal, board or body. Obviously, the real justification is to enable the management committee or
rehabilitation receiver to effectively exercise its/his powers free from any judicial or extra-judicial
interference that might unduly hinder or prevent the "rescue" of the debtor company. To allow such
other action to continue would only add to the burden of the management committee or rehabilitation
receiver, whose time, effort and resources would be wasted in defending claims against the corporation
instead of being directed toward its restructuring and rehabilitation.

435 | P a g e
Law 321_Corporation LAW_ Case Digest

BANK OF THE PHILIPPINE ISLANDS


vs.
COURT OF APPEALS
G.R. No. 97178 January 10, 1994

FACTS:

Petitioner Bank of Philippine filed with the Regional Trial Court a complaint against respondent
Ruby Industrial Corporation for foreclosure of real estate mortgage. After filing its answer with
counterclaim, respondent RUBY submitted to the trial court a motion for suspension of the proceedings
on the ground that the Securities and Exchange Commission issued an Order placing RUBY under a
rehabilitation plan.Petitioner BPI filed a motion for reopening of the proceedings, invoking our ruling
order of suspension of payments of Philfinance as well as for all actions or claims against Philfinance
could only be applied to claims of unsecured creditors. Such order can not extend to creditors holding a
mortgage, pledge or any lien on the property unless they give up the property, security or lien in favor of
all the creditors of Philfinance. The trial court denied the motion of BPI on the basis that the suspension
of payment applies to all creditors, whether secured or unsecured, in order to place them on equal
footing.
Petitioner then filed with the Court of Appeals a petition for certiorari and mandamus to set aside
the Orders of 22 August 1990 and 19 October 1990, alleging grave abuse of discretion on the part of the
trial judge in refusing to reopen the case. In the instant petition, it is alleged that the Court of Appeals
has decided a question of substance not in accord with the applicable decision of this Court and/or
sanctioned a departure by the trial court from the accepted and usual course of judicial proceedings as
to call for the exercise by this Court of its power of supervision.

ISSUE:

Whether or not petitioner, which is a secured creditor of respondent RUBY, may still judicially
enforce its claim against the latter which has already been placed by SEC under rehabilitation pursuant
to Sec. 5 and Sec. 6, pars. (c) and (d), P.D. 902-A.

RULING:

YES.

In the instant case, the action of petitioner for foreclosure of real estate mortgage had been filed
against respondent RUBY and was pending with the trial court when RUBY was placed by SEC under
rehabilitation through the creation of a management committee pursuant to Sec. 6, par. (d), P.D. 902-A.
In its order of 10 August 1984, SEC directed that all actions or claims against RUBY pending before any
court, tribunal, branch or body be deemed suspended. On the basis of this order, the jurisdiction of this
trial court over the case was also considered suspended. As a result, SEC acquired jurisdiction, which is
bolstered by the fact that it had already appointed a rehabilitation receiver for the distressed corporation
and had directed that all proceedings or claims against it be suspended.
While it is recognized that petitioner is a preferred creditor whose claim is secured by real estate
mortgage on the properties of respondent RUBY, its right to enforce its claim in court is suspended with
the placing by SEC of respondent under rehabilitation. This rule will enable the management committee
or rehabilitation receiver to effectively exercise his/its power free from any judicial or extrajudicial
interference that might unduly hinder the rescue of the distressed company.

436 | P a g e
Law 321_Corporation LAW_ Case Digest

CHING
vs.
LBP
G.R. No. 73123 September 2, 1991

FACTS:

On September 19, 1980, private respondents Filand Manufacturing and Estate Development Co.,
Inc. and Emilio Ching obtained from petitioner Land Bank of the Philippines a loan in the amount of Ten
Million Pesos (P10,000,000.00). Private respondents having failed to pay the loan on its due date,
petitioner instituted a complaint for recovery thereof. During the pendency of the collection suit, private
respondents filed a petition for declaration of insolvency. Cited as ground therefore was their inability to
pay the various debts and liabilities incurred by them, either jointly or solidarily or guaranteed by one
for the other, in the course of their businesses, such inability being due to business reserves brought
about by the fire on January 2, 1984 which gutted the old Holiday Plaza Building then owned and
operated by Filand Manufacturing, as well as the economic crisis which gripped the country following
the assassination of former Senator Benigno S. Aquino in 1983.

ISSUE:

Whether or not it is the SEC which has jurisdiction over proceedings for suspension of payments
and voluntary and involuntary insolvency.

RULING:

YES.

Section 5, par. (d) should be construed as vesting upon the SEC original and exclusive
jurisdiction only over petitions to be declared in a state of suspension of payments. This qualification
effectively circumscribes the jurisdiction of the SEC over insolvent corporations, partnerships and
associations, and consequently, over proceedings for the declaration of insolvency. It demonstrates
beyond doubt that jurisdiction over insolvency proceedings pertains neither in the first instance nor
exclusively to the SEC but only in continuation of or as an incident to the exercise of its jurisdiction over
petitions to be declared in a state of suspension of payments wherein the petitioning corporation,
partnership or association had previously been placed under a rehabilitation receiver or management
committee by the SEC itself.
Viewed differently, where the petition filed is one for declaration of a state of suspension of
payments due to a recognition of the inability to pay one's debts and liabilities, and where the
petitioning corporation either: (a) has sufficient property to cover all its debts but foresees the
impossibility of meeting them when they fall due. However, if the petitioning corporation has no
sufficient assets to cover its liabilities and is not under a rehabilitation receiver or a management
committee created under P.D. No. 902-A and does not seek merely to have the payments of its debts
suspended, but seeks a declaration of insolvency, as in this case.
As declared by the law itself, these are merely ancillary powers to enable the SEC to effectively
exercise its jurisdiction. These additional ancillary powers can be exercised only in connection with an
action pending before the SEC and therefore had to be viewed in relation to Section 5 which defines the
SEC's original and exclusive jurisdiction. Section 6 does not enlarge or add to the exclusive and original
jurisdiction of the SEC as particularly enumerated under Section 5 of said Presidential Decree, as
amended. Construing P.D. 902-A, as amended, in relation to Act 1956, the court ruled that insofar as
petitions for declaration of insolvency of private corporations are concerned, it is the regular court that
has exclusive and original jurisdiction thereon. The SEC may entertain such petitions only as an
incident of and in continuation of its already acquired jurisdiction over petitions to be declared in the
state of suspension of payments in the two cases provided in Section 5 (d) of P.D. 902-A, as amended.

437 | P a g e
Law 321_Corporation LAW_ Case Digest

PHILIPPINE COMMERCIAL INTERNATIONAL BANK


vs.
COURT OF APPEALS
G.R. No. L-76853 April 18, 1989

FACTS:

On March 3, 1981, Philippine Underwriters Finance Corporation executed a pledge agreement


involving certain shares of stocks and bonds in favor of Insular Bank of Asia and America now PCIB as a
security for its outstanding obligation. On June 18, 1981, the Securities and Exchange Commission
placed Philfinance under suspension of payments upon the directive of the President of the Philippines
to conserve the assets of the Corporation and obtain an equitable payment to all its creditors. On
August 7, 1981, SEC appointed a Receivership Committee to conserve the assets of Phil-finance and
determine the best way to protect the creditors, as well as make the necessary representations with any
court or other body for the consolidation of all claims against Philfinance which are pending before such
court or body in order to forestall the probability of inequitable disposition/satisfaction of said claims.
Upon the basis of the findings of the Receivership Committee and on its own, SEC ordered the
dissolution and liquidation of Philfinance. From this order, some concerned parties appealed to this
Court and respondent Court of Appeals.
Meanwhile, Philfinance failed to satisfy its outstanding obligation with PCIB which prompted the
latter to post a Notice of Auction Sale of the pledged shares of stocks and bonds on August 18, 1986 by
the other petitioner, Notary Public Melchor B. Francisco. On August 15, 1986, the Receiver filed a
petition for a writ of preliminary injunction with the Regional Trial Court to stop the aforementioned
auction sale which the trial court denied but which denial was reversed by the CA.

ISSUE:

Whether or not suspension of payments apply only to unsecured creditors.

RULING:

NO.

SEC's order for suspension of payments of Philfinance as well as for all actions of claims against
Philfinance could only be applied to claims of unsecured creditors. Such order can not extend to
creditors holding a mortgage, pledge or any lien on the property unless they give up the property,
security or lien in favor of all the creditors of Philfinance. The rights of a preferred creditor remain to be
respected and recognized in every existing situation. To hold otherwise would render the said rights
inutile and illusory. Besides, there was no substantial difference between the suspension of actions in
the instant case and that under the Insolvency Law. Consequently, the herein order of suspension,
could not have a different interpretation as regards secured credits than that already given by this
Court. The records show that PCIB neither surrendered the pledged shares of stock and bonds nor
participated in the proceedings before the SEC regarding the suspension of payments or actions of
claims against Philfinance or in the latter's subsequent dissolution and liquidation. The pledged
properties being still in PCIB's possession, the Receiver could not possess the same for equitable
distribution to the creditors of Philfinance.
The fact that the SEC order for the dissolution and liquidation of Philfinance has already been
upheld by the court had been taken judicial notice of. In view of this development, it appears that the
Rehabilitation Receiver has no more right to enjoin the auction sale since its prayer for injunctive relief
was based on the order for suspension of payments which was in turn based on the directive of the
President of the Philippines to conserve the assets of the corporation and obtain an equitable payment to
all its creditors.

438 | P a g e
Law 321_Corporation LAW_ Case Digest

RADIOLA-TOSHIBA PHILIPPINES, INC.


vs.
INTERMEDIATE APPELLATE COURT
G.R. No. 75222 July 18, 1991

FACTS:

On July 2, 1980, three creditors filed a petition for the involuntary insolvency of Carlos
Gatmaytan and Teresita Gatmaytan, the private respondents herein. On July 9, 1980, the respondent
court issued an order taking cognizance of the said petition and statingthat the Court forbids the
payment of any debts, and the delivery of any property owing and belonging to said respondents-debtors
from other persons, or, to any other persons for the use and benefit of the same respondents-debtors
and/or the transfer of any property by and for the said respondents-debtors to another, upon
petitioners' putting up a bond by way of certified and reputable sureties. On April 12, 1983, petitioners-
creditors filed second urgent motion for issuance of insolvency order and resolution of the case, alleging
among other things, they caused to be investigated the real properties in the names of Carlos
Gatmaytan and Teresita Gatmaytan and they were surprised to find out that some of the aforesaid
properties were already transferred to Radiola-Toshiba Phil. Inc. Judgment was rendered declaring the
insolvency of respondents-debtors Carlos Gatmaytan and Teresita Gatmaytan. Petitioner filed a
supplemental opposition to the same second urgent motion and motion to direct respondent sheriff to
issue a final certificate of sale for the properties in its favor. On September 21, 1982, the court ordered
the consolidation of ownership of petitioner over said properties but respondent sheriff of Angeles City
refused to issue a final certificate of sale in favor of petitioner.

ISSUE:

Whether or not the levy on attachment in favor of the petitioner is dissolved by the insolvency
proceedings against respondent spouses commenced four months after said attachment.

RULING:

NO.

Sec. 32 states that as soon as an assignee is elected or appointed and qualified, the clerk of the
court shall, by an instrument under his hand and seal of the court, assign and convey to the assignee
all the real and personal property, estate, and effects of the debtor with all his deeds, books, and papers
relating thereto, and such assignment shall relate back to the commencement of the proceedings in
insolvency, and shall relate back to the acts upon the adjudication was founded, and by operation of law
shall vest the title to all such property, estate, and effects in the assignee, although the same is then
attached on process, as the property of the debtor. Such assignment shall operate to vest in the assignee
all of the estate of the insolvent debtor not exempts by law from execution. The provision is very clear
that attachments dissolved are those levied within one month next preceding the commencement of the
insolvency proceedings and judgments vacated and set aside are judgments entered in any action,
including judgment entered by default or consent of the debtor, where the action was filed within thirty
days immediately prior to the commencement of the insolvency proceedings.
There is a cut off period of one month in attachment cases and thirty days in judgments entered
in actions commenced prior to the insolvency proceedings. Section 79, on the other hand, relied upon by
private respondents provides for the right of the plaintiff if the attachment is not dissolved before the
commencement of proceedings in insolvency, or is dissolved by an undertaking given by the defendant,
if the claim upon which the attachment suit was commenced is proved against the estate of the debtor.
Therefore, there is no conflict between the two provisions.

RIZAL COMMERCIAL BANKING CORPORATION


vs.
INTERMEDIATE APPELLATE COURT
G.R. No. 74851 December 9, 1999

FACTS:

On September 28, 1984, BF Homes filed a Petition for Rehabilitation and for Declaration of
Suspension of Payments with the SEC. One of the creditors listed in its inventory of creditors and
liabilities was RCBC. On October 26, 1984, RCBC requested the Provincial Sheriff of Rizal to extra-

439 | P a g e
Law 321_Corporation LAW_ Case Digest

judicially foreclose its real estate mortgage on some properties of BF Homes. A notice of extra-judicial
foreclosure sale was issued by the Sheriff on October 29, 1984, scheduled on November 29, 1984, copies
furnished both BF Homes as mortgagor and RCBC as mortgagee. On motion of BF Homes, the SEC a
temporary restraining order effective for 20 days, enjoining RCBC and the sheriff from proceeding with
the public auction sale. The sale was rescheduled to January 29, 1985. On January 25, 1985, the SEC
ordered the issuance of a writ of preliminary injunction upon petitioners filing of a bond. However,
petitioner did not file a bond until January 29, 1985, the very day of the auction sale, so no writ of
preliminary injunction was issued by the SEC. Presumably, unaware of the filing of the bond, the
sheriffs proceeded with the public auction sale on January 29, 1985, in which RCBC was the highest
bidder for the properties auctioned. On February 5, 1985, BF Homes filed in the SEC a consolidated
motion to annul the auction sale and to cite RCBC and the sheriff for contempt. RCBC opposed the
motion.
Because of the proceedings in the SEC, the sheriff withheld the delivery to RCBC of a certificate of
sale covering the auctioned properties. On February 13, 1985, the SEC belatedly issued a writ of
preliminary injunction stopping the auction sale which had been conducted by the sheriff two weeks
earlier.

ISSUE:

Whether or not preferred creditors of distressed corporations stand on the same footing with all
other creditors.

RULING:

YES.

The issue of whether or not preferred creditors of distressed corporations stand on equal footing
with all other creditors gains relevance and materiality only upon the appointment of a management
committee, rehabilitation receiver, board, or body. Insofar as petitioner RCBC is concerned, the
provisions of Presidential Decree No. 902-A are not yet applicable and it may still be allowed to assert its
preferred status because it foreclosed on the mortgage prior to the appointment of the management
committee on March 18, 1985. Paragraph (c), Section 6 of Presidential Decree 902-A, providesthat upon
appointment of a management committee, rehabilitation receiver, board or body, pursuant to this
Decree, all actions for claims against corporations, partnerships or associations under management or
receivership pending before any court, tribunal, board or body shall be suspended accordingly. It is thus
adequately clear that suspension of claims against a corporation under rehabilitation is counted or
figured up only upon the appointment of a management committee or a rehabilitation receiver.
In other words, once a management committee, rehabilitation receiver, board or body is
appointed pursuant to P.D. 902-A, all actions for claims against a distressed corporation pending before
any court, tribunal, board or body shall be suspended accordingly. This suspension shall not prejudice
or render ineffective the status of a secured creditor as compared to a totally unsecured creditor. P.D.
902-A does not state anything to this effect. What it merely provides is that all actions for claims against
the corporation, partnership or association shall be suspended. This should give the receiver a chance to
rehabilitate the corporation if there should still be a possibility for doing so.
RUBBERWORLD (PHILS.), INC.
vs.
NATIONAL LABOR RELATIONS COMMISSION
G.R. No. 126773 April 14, 1999

FACTS:

Petitioner is a domestic corporation which used to be in the business of manufacturing footwear,


bags and garments. It filed with the Securities and Exchange Commission a petition for suspension of
payments praying that it be declared in a state of suspension of payments and that the SEC accordingly
issue an order restraining its creditors from enforcing their claims against petitioner corporation. It
further prayed for the creation of a management committee as well as for the approval of the proposed
rehabilitation plan and memorandum of agreement between Petitioner Corporation and its creditors.
The SEC favorably ruled on the petition for suspension of payments. Private respondents, who claim to
be employees of Petitioner Corporation, filed against petitioners their respective complaints for illegal
dismissal, unfair labor practice, damages and payment of separation pay, retirement benefits, 13th
month pay and service incentive pay. Petitioners moved to suspend the proceedings in the above labor
cases on the strength of the SEC Order.

440 | P a g e
Law 321_Corporation LAW_ Case Digest

The Labor Arbiter denied the motion holding that the injunction contained in the SEC Order
applied only to the enforcement of established rights and did not include the suspension of proceedings
involving claims against petitioner which have yet to be ascertained. The Labor Arbiter further held that
the order of the SEC suspending all actions for claims against petitioners does not cover the claims of
private respondents in the labor cases because said claims and the concomitant liability of petitioners
still had to be determined, thus carrying no dissipation of the assets of petitioners.

ISSUE:

Whether or not the labor proceedings should be suspended.

RULING:

NO.

The applicable law is PD 902-A, as amended which provides that upon the appointment by, the
SEC of a management committee or a rehabilitation receiver, all actions for claims against the
corporation pending before any court, tribunal or board shall ipso jurebe suspended. The justification
for the automatic stay of all pending actions for claims is to enable the management committee or the
rehabilitation receiver to effectively exercise its/his powers free from any judicial or extra-judicial
interference that might unduly hinder or prevent the 'rescue' of the debtor company. To allow such other
actions to continue would only add to the burden of the management committee or rehabilitation
receiver, whose time, effort and resources would be wasted in defending claims against the corporation
instead of being directed toward its restructuring and rehabilitation.
Parenthetically, the rehabilitation of a financially distressed corporation benefits its employees,
creditors, stockholders and, in a larger sense, the general public. And in considering whether to
rehabilitate or not, the SEC gives preference to the interest of creditors, including employees. The reason
that shareholders can recover their investments only upon liquidation of' the corporation, and only if
there are assets remaining after all corporate creditors are paid. No exception in favor of labor claims is
mentioned in the law. Since the law makes no distinction or exemptions, neither should the Court.
Allowing labor cases to proceed clearly defeats the purpose of the automatic stay and severely
encumbers the management committee's time and resources. The said committee would need to defend
against these suits, to the detriment of its primary and urgent duty to work towards rehabilitating the
corporation and making it viable again. To rule otherwise would open the floodgates to other similarly
situated claimants and forestall if not defeat the rescue efforts.

441 | P a g e
Law 321_Corporation LAW_ Case Digest

UNION BANK OF THE PHILIPPINES


vs.
HONORABLE COURT OF APPEALS
G.R. No. 131729 May 19, 1998

FACTS:

Private respondents EYCO Group of Companies filed with the SEC a Petition for the Declaration
of Suspension of Payment, Formation and Appointment of Rehabilitation Receiver/Committee, Approval
of Rehabilitation Plan with Alternative Prayer for Liquidation and Dissolution of Corporations alleging,
among other things, that the present combined financial condition of the petitioners clearly indicates
that their assets are more than enough to pay off the credits but that due to factors beyond the control
and anticipation of the management the inability of the EYCO Group of Companies to meet the
obligations as they fall due on the schedule agreed with the creditors has now become a stark reality.
The SEC Hearing Panel then issued an order setting its hearing. At the same time, said panel also
directed the suspension of all actions, claims and proceedings against private respondents pending
before any court, tribunal, office, board and/or commission.Without notifying the members of the
consortium, petitioner, however, decided to break away from the group by suing private respondents in
the regular courts. Aggrieved, petitioner immediately took recourse to the Court of Appeals by filing
therewith a Petition for Certiorari with Prayer for the Issuance of a Temporary Restraining Order and/or
Writ of Preliminary Injunction.

ISSUE:

Whether or not the SEC can validly acquire jurisdiction over a petition for suspension of
payments filed pursuant to Section 5 (d) of P.D. No. 902- A, as amended, when such petition joins as co-
petitioners the petitioning corporate entities and individual stockholders thereof.

RULING:

NO.

The SEC's jurisdiction on matters of suspension of payments is confined only to those initiated by
corporations, partnerships or associations. Administrative agencies like the SEC are tribunals of limited
jurisdiction and, as such, can exercise only those powers which are specifically granted to them by their
enabling statutes. Consequently, where no authority is granted to hear petitions of individuals for
suspension of payments, such petitions are beyond the competence of the SEC.
In a case of misjoinder of parties which in this case is the co-filing of the petition for suspension
of payments by both the Yutingcos and the EYCO group, the remedy has never been to dismiss the
petition in its entirety but to dismiss it only as against the party upon whom the tribunal or body cannot
acquire jurisdiction. The result, therefore, is that the petition with respect to EYCO shall subsist and
may be validly acted upon by the SEC. The Yutingcos, on the other hand, shall be dropped from the
petition and be required to pursue their remedies in the regular courts of competent jurisdiction.
Aside from the fact that these allegations are evidentiary in nature and still remains to be proved,
we have likewise consistently ruled that what determines the nature of an action, as well as which court
or body has jurisdiction over it, are the allegations of the complaint, or a petition as in this case, and the
character of the relief sought. That the merits of the case after due proceedings are later found to veer
away from the claims asserted by EYCO in its petition, as when it is shown later that it is actually
insolvent and may not be entitled to suspension of payments, does not divest the SEC at all of its
jurisdiction already acquired at its inception through the allegations made in the petition.

SECURITIES REGULATION CODE


(Republic Act No. 8799)

ELEMENTS OF AN INVESTMENT CONTRACT

WHAT SECURITIES ARE REQUIRED TO BE REGISTERED


442 | P a g e
Law 321_Corporation LAW_ Case Digest

EXEMPT TRANSACTIONS

PUBLIC COMPANIES

TRADING IN SECURITIES: MARGIN REQUIREMENTS

FRAUDULENT TRANSACTIONS

SIGNIFICANT FACTS ON INSIDER’S DUTY TO DISCLOSE WHEN TRADING

DISCLOSURE REGULATIONS FOR PUBLICLY-LISTED SHARES

TENDER OFFERS

443 | P a g e
Law 321_Corporation LAW_ Case Digest

444 | P a g e

You might also like